From owner-obm-l@sucuri.mat.puc-rio.br Wed May 1 11:08:57 2002 Return-Path: Received: (from majordom@localhost) by sucuri.mat.puc-rio.br (8.9.3/8.9.3) id LAA01291 for obm-l-list; Wed, 1 May 2002 11:07:05 -0300 Received: from imo-r09.mx.aol.com (imo-r09.mx.aol.com [152.163.225.105]) by sucuri.mat.puc-rio.br (8.9.3/8.9.3) with ESMTP id LAA01288 for ; Wed, 1 May 2002 11:07:02 -0300 From: Lltmdrtm@aol.com Received: from Lltmdrtm@aol.com by imo-r09.mx.aol.com (mail_out_v32.5.) id z.86.19f95e64 (4186) for ; Wed, 1 May 2002 09:11:47 -0400 (EDT) Message-ID: <86.19f95e64.2a014393@aol.com> Date: Wed, 1 May 2002 09:11:47 EDT Subject: [obm-l] Me pegou!!! Ajuda To: obm-l@mat.puc-rio.br MIME-Version: 1.0 Content-Type: multipart/alternative; boundary="part1_86.19f95e64.2a014393_boundary" X-Mailer: AOL 7.0 for Windows BR sub 61 Sender: owner-obm-l@sucuri.mat.puc-rio.br Precedence: bulk Reply-To: obm-l@mat.puc-rio.br --part1_86.19f95e64.2a014393_boundary Content-Type: text/plain; charset="ISO-8859-1" Content-Transfer-Encoding: quoted-printable Jo=E3o, Maria e Andr=E9 compraram em parceria um bilhete de rifa por 300 rea= is,=20 que sorteado pagou um pr=EAmio de 300.000 reais. No rateio do pr=EAmio coube= a=20 Andr=E9 o dobro da quantia paga a Jo=E3o. O lucro l=EDquido de Maria corresp= ondeu a=20 39,96% do total do pr=EAmio. Calcule a quantia, em reais, paga por Jo=E3o, n= a=20 compra do bilhete. --part1_86.19f95e64.2a014393_boundary Content-Type: text/html; charset="ISO-8859-1" Content-Transfer-Encoding: quoted-printable
Jo=E3o, Maria e Andr=E9 compraram em parceria um bilhete de rifa por 300 rea= is, que sorteado pagou um pr=EAmio de 300.000 reais. No rateio do pr=EAmio c= oube a Andr=E9 o dobro da quantia paga a Jo=E3o. O lucro l=EDquido de Maria=20= correspondeu a 39,96% do total do pr=EAmio. Calcule a quantia, em reais, pag= a por Jo=E3o, na compra do bilhete.
--part1_86.19f95e64.2a014393_boundary-- ========================================================================= Instruções para entrar na lista, sair da lista e usar a lista em http://www.mat.puc-rio.br/~nicolau/olimp/obm-l.html O administrador desta lista é ========================================================================= From owner-obm-l@sucuri.mat.puc-rio.br Wed May 1 11:10:02 2002 Return-Path: Received: (from majordom@localhost) by sucuri.mat.puc-rio.br (8.9.3/8.9.3) id LAA01328 for obm-l-list; Wed, 1 May 2002 11:08:42 -0300 Received: from smtp-5.ig.com.br (smtp-5.ig.com.br [200.226.132.154] (may be forged)) by sucuri.mat.puc-rio.br (8.9.3/8.9.3) with SMTP id LAA01325 for ; Wed, 1 May 2002 11:08:41 -0300 Received: (qmail 13893 invoked from network); 1 May 2002 13:56:51 -0000 Received: from rj230088.user.veloxzone.com.br (HELO jigglypuff) (200.165.230.88) by smtp-5.ig.com.br with SMTP; 1 May 2002 13:56:51 -0000 Message-ID: <008401c1f118$4c3d9d20$019da8c0@dummy.net> From: "Juliana Freire" To: References: <3CCDAC9600000B95@www.zipmail.com.br> Subject: [obm-l] =?iso-8859-1?Q?Re:_=5Bobm-l=5D_Problemas_do_2=B0grau?= Date: Wed, 1 May 2002 10:58:39 -0300 MIME-Version: 1.0 Content-Type: text/plain; charset="iso-8859-1" Content-Transfer-Encoding: 8bit X-Priority: 3 X-MSMail-Priority: Normal X-Mailer: Microsoft Outlook Express 5.50.4522.1200 X-MimeOLE: Produced By Microsoft MimeOLE V5.50.4522.1200 Sender: owner-obm-l@sucuri.mat.puc-rio.br Precedence: bulk Reply-To: obm-l@mat.puc-rio.br > 1-Um ministro brasileiro organiza uma recepção . Metade dos convidados são > estrangeiros cuja língua não é o português e ,por delicadeza ,cada um deles > diz bom dia a cada um dos outros na língua oficial de a quem se dirige . > O ministro responde seja bem vindo a cada convidado . Sabendo que no total > foram ditos 78 bons dias em português o número de convidados na recepção > foi : Eu acho que você está certo, eu também achei 13, mas eu fiz um pouco diferente de você: Cada um dos brasileiros escuta um bom-dia em português dos outros convidados, não só dos estrangeiros, mas dos brasileiros também. Quando dois brasileiros se encontram, os dois bom-dias são em portugues. Cada pessoa ouve T-1 bom dias (não diz bom dia para ela mesma) então o total de bom-dias em portugues ouvidos são (T/2)*(T-1) = 78. T^2 - T = 156 T = (1 + sqrt (1 + 4*156)) / 2 T = 13 > 2- Um comerciante comprou n rádios por d cruzeiros , onde d é um inteiro > positivo .Ele contribuiu com a comunidade vendendo para o bazar da mesma > dois rádios pela metade do seu custo .O restante ele vendeu com um lucro > de 8 cruzeiros em cada rádio .Se o lucro total foi de 72 cruzeiros , então > o menor valor possível de n é: Nesta eu encontrei a resposta 12. Fiz assim: Se ele comprou n rádios por d dinheiros, o preço de um rádio é d/n. O dinheiro total que ele gastou foi d. O dinheiro total que ele ganhou foi: d/n (dois rádios vendidos para o bazar) (n-2)(d/n + 8) (o restante dos rádios vendidos com lucro de 8 dinheiros em cada um) Lucro = dinheiro que entrou - dinheiro que saiu, portanto 72 = d/n + (n-2)(d/n + 8) - d 72n = d + (n-2)(d+8n) - dn 72n = d + nd - 2d + 8n^2 - 16n - dn 72n = -d + 8n^2 - 16n 8n^2 - 88n - d = 0 n = (88 + sqrt(7744 + 32d) ) / 16 Temos que encontrar um d, inteiro, que colocando na conta acima dê um n também inteiro. Por tentativa, encontrei que o menor d que faz isso é 96, dando n=12. ========================================================================= Instruções para entrar na lista, sair da lista e usar a lista em http://www.mat.puc-rio.br/~nicolau/olimp/obm-l.html O administrador desta lista é ========================================================================= From owner-obm-l@sucuri.mat.puc-rio.br Wed May 1 11:18:39 2002 Return-Path: Received: (from majordom@localhost) by sucuri.mat.puc-rio.br (8.9.3/8.9.3) id LAA01609 for obm-l-list; Wed, 1 May 2002 11:17:19 -0300 Received: from www.zipmail.com.br (smtp.zipmail.com.br [200.187.242.10]) by sucuri.mat.puc-rio.br (8.9.3/8.9.3) with ESMTP id LAA01606 for ; Wed, 1 May 2002 11:17:17 -0300 From: ghaeser@zipmail.com.br Received: from [200.158.6.226] by www.zipmail.com.br with HTTP; Wed, 1 May 2002 09:42:40 -0300 Message-ID: <3CC83D560000E715@www.zipmail.com.br> Date: Wed, 1 May 2002 09:42:40 -0300 In-Reply-To: <32C9AF7F.26101.16BF58@localhost> Subject: [obm-l] =?iso-8859-1?Q?Re=3A=20=5Bobm=2Dl=5D=20Equacao=20do=20Universo?= To: obm-l@mat.puc-rio.br MIME-Version: 1.0 Content-Type: text/plain; charset="iso-8859-1" Content-Transfer-Encoding: 8bit X-MIME-Autoconverted: from quoted-printable to 8bit by sucuri.mat.puc-rio.br id LAA01607 Sender: owner-obm-l@sucuri.mat.puc-rio.br Precedence: bulk Reply-To: obm-l@mat.puc-rio.br i^i=exp(i*logi)=exp(i*(ln|i|+iarg(i))=exp(i*(ln1+i*pi/2))= =exp(i*(i*pi/2))= i^i=exp(-pi/2) se elevarmos a i novamente temos: i^i^i=exp(i*(-pi/2))=cos(pi/2)-i*sen(pi/2)=-i quanto a outra pergunta .. nem imagino .. -- Mensagem original -- >Olah a todos, > >O que eh Equacao do Universo? (se eh que isso existe) > >Quanto vale i^i? (i = sqrt(-1)) > > >Desde jah agradeco, > > > >Ezer F. da Silva >========================================================================= >Instruções para entrar na lista, sair da lista e usar a lista em >http://www.mat.puc-rio.br/~nicolau/olimp/obm-l.html >O administrador desta lista é >========================================================================= > "Mathematicus nascitur, non fit" Matemáticos não são feitos, eles nascem --------------------------------------- Gabriel Haeser www.gabas.cjb.net ------------------------------------------ Use o melhor sistema de busca da Internet Radar UOL - http://www.radaruol.com.br ========================================================================= Instruções para entrar na lista, sair da lista e usar a lista em http://www.mat.puc-rio.br/~nicolau/olimp/obm-l.html O administrador desta lista é ========================================================================= From owner-obm-l@sucuri.mat.puc-rio.br Wed May 1 12:16:18 2002 Return-Path: Received: (from majordom@localhost) by sucuri.mat.puc-rio.br (8.9.3/8.9.3) id MAA03014 for obm-l-list; Wed, 1 May 2002 12:15:55 -0300 Received: from [200.244.126.37] (bridge3.bridge.com.br [200.244.126.37]) by sucuri.mat.puc-rio.br (8.9.3/8.9.3) with ESMTP id MAA03007 for ; Wed, 1 May 2002 12:15:49 -0300 Received: from fulano (unverified [200.191.181.205]) by (Vircom SMTPRS 4.0.179) with ESMTP id for ; Wed, 1 May 2002 11:29:12 -0300 Message-ID: <003a01c1f11c$4f7795a0$cdb5bfc8@fulano> From: "Eduardo Azevedo" To: References: <3CC477B400011F7C@www.zipmail.com.br> Subject: Re: [obm-l] Algumas da Iberoamericana.SEGUNDO PROBLEMA PARA A LISTA Date: Wed, 1 May 2002 11:27:18 -0300 MIME-Version: 1.0 Content-Type: text/plain; charset="iso-8859-1" Content-Transfer-Encoding: 8bit X-Priority: 3 X-MSMail-Priority: Normal X-Mailer: Microsoft Outlook Express 5.50.4133.2400 X-MimeOLE: Produced By Microsoft MimeOLE V5.50.4133.2400 Sender: owner-obm-l@sucuri.mat.puc-rio.br Precedence: bulk Reply-To: obm-l@mat.puc-rio.br ----- Original Message ----- From: To: "Carlos Shine" ; "Celso" ; "Edmilson" ; "JP" ; "Lista de Discussao" ; "Ralph" ; "Nicolau" ; Sent: Tuesday, April 30, 2002 5:07 PM Subject: [obm-l] Algumas da Iberoamericana.SEGUNDO PROBLEMA PARA A LISTA > Ah.turma,to com a prova da Iberoamericana aquoi na mao,e tenho problemas > serios neles.Ai vai!!! > 1.Temos 98 pontos sobre uma circunferencia.Maria e Jose fazem um jogo assim:cada > um deles traça uma corda ligando dois dos pontos dados que nao tenham sido > ligados entre si antes.O jogo acaba quandoos 98 pontos forem usados como > extremos de segmentos pelo menos 1 vez.Quem fizer o ultimo traço ganha.Defina > uma estrategia vencedora se ela existir. O primeiro jogador, jogando certo, sempre vence. Isso vale para qualquer n = 2 (mod 4) Seja k o número de pontos não ligados a ninguém, j o número de jogadas ja feitas e n=98 o número de pontos. Em cada jogada pode-se ligar dois pontos livres, fazendo k diminuir 2, ligar um livre e um marcado, fazendo k baixar 1, ou ligar dois pontos marcados, mantendo o mesmo k. Chamemos isso de enrolada. Seja "e" o número de opções de enrolada num dado cenário É fácil ver que em qualquer momento do jogo e = (n-k)(n-k-1)/2 - j Numa dada jogada, o jogador com "e" ímpar está em vantagem, pois pode enrolar o quanto quiser, até o jogador com "e" par ficar sem opções de enrolação, e ser forcado a jogar. Agora, (n-k)(n-k-1)/2 é par se e somente se (n-k)(n-k-1) = 0 ou seja: (n-k)(n-k-1)/2 é par se e somente se k = n (mod 4) ou k = n+1 (mod 4) Ja j é par para o jogador A e ímpar para o jogador B. Logo, para o jogador B, temos: e par quando k= n ou n+1 (mod 4) Para A: e par quando k= n+2 ou n+3 (mod 4) Isso mostra que a paridade de e depende apenas de k, e é alternada para cada jogador. Ou seja, a abilidade de um jogador de forcar uma jogada depende apenas de k. Agora vejamos algumas situações de vitória: Se k = 1 ou k = 2, quem tiver a vez ganha. Se k=3, comer mais uma peca significa a derrota, pois leva o adversário ao cenário acima. Assim, quando k é igual a 3 é necessário enrolar. Mas como 3 = 99 = n+1 (mod 4), o jogador A tem "e" ímpar em 3, e B tem "e" par. Portanto, A sempre pode enrolar, enquanto B será eventualmente forçado a jogar. Logo se k=3, o jogador A sempre ganha. Portanto A precisa apenas garantir que o jogo caia em k =3 e ganhará sempre. Se fosse n=3 (mod 4), A continuaria podendo ganhar sempre. Se fosse n=+-1(mod4), B poderia ganhar sempre, exceto se n=5, onde A pode ganhar sempre. Esse jogo não parece muito justo ..... ========================================================================= Instruções para entrar na lista, sair da lista e usar a lista em http://www.mat.puc-rio.br/~nicolau/olimp/obm-l.html O administrador desta lista é ========================================================================= From owner-obm-l@sucuri.mat.puc-rio.br Wed May 1 12:16:18 2002 Return-Path: Received: (from majordom@localhost) by sucuri.mat.puc-rio.br (8.9.3/8.9.3) id MAA03011 for obm-l-list; Wed, 1 May 2002 12:15:53 -0300 Received: from [200.244.126.37] (bridge3.bridge.com.br [200.244.126.37]) by sucuri.mat.puc-rio.br (8.9.3/8.9.3) with ESMTP id MAA03005 for ; Wed, 1 May 2002 12:15:48 -0300 Received: from fulano (unverified [200.191.181.205]) by (Vircom SMTPRS 4.0.179) with ESMTP id for ; Wed, 1 May 2002 10:04:01 -0300 Message-ID: <002801c1f110$69484800$cdb5bfc8@fulano> From: "Eduardo Azevedo" To: References: <5.1.0.14.0.20020430204757.00ab1120@pop.sao.terra.com.br> Subject: Re: [obm-l] Algumas da Iberoamericana.SEGUNDO PROBLEMA PARA A LISTA Date: Wed, 1 May 2002 10:00:16 -0300 MIME-Version: 1.0 Content-Type: text/plain; charset="iso-8859-1" Content-Transfer-Encoding: 8bit X-Priority: 3 X-MSMail-Priority: Normal X-Mailer: Microsoft Outlook Express 5.50.4133.2400 X-MimeOLE: Produced By Microsoft MimeOLE V5.50.4133.2400 Sender: owner-obm-l@sucuri.mat.puc-rio.br Precedence: bulk Reply-To: obm-l@mat.puc-rio.br 2) Bom, como 11|11 e 11|77, acho que o problema quis dizer maior ou igual a 11. De fato B termina em um impar diferente de 5, logo nao tem fator 2 nem 5. Sobraram 3 e 7, e o Bruno cantou certinho: i) 21^k = (20 +1)^k= ..... + 20k + 1, logo termina em 1 e tem o algarismo das dezenas par. ii)Podemos provar que as potencias de 7 tem o algarismo das dezenas par: As potencias de 7 terminam em 1, 3 ,9 ou 7. Supondo, por inducao que 7^n tem a dezena par, 7^n+1=7*7^n terá dezena par, pois na multiplicacao, ou vai 0 ou vai 2 ou vai 6. iii)Analogamente 3 tem dezena par. iv)supondo B=3^x * 7^y = 21^m * (3^n) ou 21^m * (7^n). Logo tem dezena par. Contradicao. ----- Original Message ----- From: "Bruno F. C. Leite" To: Sent: Tuesday, April 30, 2002 8:52 PM Subject: Re: [obm-l] Algumas da Iberoamericana.SEGUNDO PROBLEMA PARA A LISTA > At 17:07 30/04/02 -0300, you wrote: > >Ah.turma,to com a prova da Iberoamericana aquoi na mao,e tenho problemas > >serios neles.Ai vai!!! > >1.Temos 98 pontos sobre uma circunferencia.Maria e Jose fazem um jogo > >assim:cada > >um deles traça uma corda ligando dois dos pontos dados que nao tenham sido > >ligados entre si antes.O jogo acaba quandoos 98 pontos forem usados como > >extremos de segmentos pelo menos 1 vez.Quem fizer o ultimo traço ganha.Defina > >uma estrategia vencedora se ela existir. > >2.B e um inteiro maior que 10 que so tem 1,3,7,9 como digitos decimais.Prove > >que B tem um fator primo maior que 11.(o que consegui chegar foi nisso:se > >p>11 acarretasse B/p nao inteiro,entao os unicos fatores primos de B sao > >3 e 7.Oras,5 nao pode,ja que 5*K==0 ou 5(mod 10).E 2 tambem nao,ja que > >2*k==0,2,4,6,8 > >(mod 10). > > Já resolvi este problema uma vez (faz tempo) e, pelo que me lembro, vc > supoe que os unicos primos que dividem B são 3 e 7, logo B=3^a 7^b. Aí > (tomara que seja isso mesmo) vc prova que todo numero > da forma 3^a 7^b e maior que 10 tem o digito das dezenas par! > > Bruno Leite > http://www.ime.usp.br/~brleite > > > >Problema:seja ABC um triangulo de incentro I.Nele se desenha o circuncirculo > >de BIC.Ache o ponto I sabendo desse circuncirculo > >e do ABC usando uma regua lisa. > > > > > > > >------------------------------------------ > >Use o melhor sistema de busca da Internet > >Radar UOL - http://www.radaruol.com.br > > > > > > > >========================================================================= > >Instruções para entrar na lista, sair da lista e usar a lista em > >http://www.mat.puc-rio.br/~nicolau/olimp/obm-l.html > >O administrador desta lista é > >========================================================================= > > ========================================================================= > Instruções para entrar na lista, sair da lista e usar a lista em > http://www.mat.puc-rio.br/~nicolau/olimp/obm-l.html > O administrador desta lista é > ========================================================================= ========================================================================= Instruções para entrar na lista, sair da lista e usar a lista em http://www.mat.puc-rio.br/~nicolau/olimp/obm-l.html O administrador desta lista é ========================================================================= From owner-obm-l@sucuri.mat.puc-rio.br Wed May 1 12:16:19 2002 Return-Path: Received: (from majordom@localhost) by sucuri.mat.puc-rio.br (8.9.3/8.9.3) id MAA03016 for obm-l-list; Wed, 1 May 2002 12:15:57 -0300 Received: from [200.244.126.37] (bridge3.bridge.com.br [200.244.126.37]) by sucuri.mat.puc-rio.br (8.9.3/8.9.3) with ESMTP id MAA03009 for ; Wed, 1 May 2002 12:15:51 -0300 Received: from fulano (unverified [200.191.181.205]) by (Vircom SMTPRS 4.0.179) with ESMTP id for ; Wed, 1 May 2002 11:35:40 -0300 Message-ID: <004a01c1f11d$37173d20$cdb5bfc8@fulano> From: "Eduardo Azevedo" To: References: <5.1.0.14.0.20020430204757.00ab1120@pop.sao.terra.com.br> Subject: Re: [obm-l] Algumas da Iberoamericana.CORRECAO Date: Wed, 1 May 2002 11:33:49 -0300 MIME-Version: 1.0 Content-Type: text/plain; charset="iso-8859-1" Content-Transfer-Encoding: 8bit X-Priority: 3 X-MSMail-Priority: Normal X-Mailer: Microsoft Outlook Express 5.50.4133.2400 X-MimeOLE: Produced By Microsoft MimeOLE V5.50.4133.2400 Sender: owner-obm-l@sucuri.mat.puc-rio.br Precedence: bulk Reply-To: obm-l@mat.puc-rio.br Na verdade, para B poder ganhar sempre é n = 1 ou n= 0 (mod 4) , com n diferente de 5. ========================================================================= Instruções para entrar na lista, sair da lista e usar a lista em http://www.mat.puc-rio.br/~nicolau/olimp/obm-l.html O administrador desta lista é ========================================================================= From owner-obm-l@sucuri.mat.puc-rio.br Wed May 1 14:47:03 2002 Return-Path: Received: (from majordom@localhost) by sucuri.mat.puc-rio.br (8.9.3/8.9.3) id OAA05011 for obm-l-list; Wed, 1 May 2002 14:46:53 -0300 Received: from web14804.mail.yahoo.com (web14804.mail.yahoo.com [216.136.224.220]) by sucuri.mat.puc-rio.br (8.9.3/8.9.3) with SMTP id OAA05008 for ; Wed, 1 May 2002 14:46:50 -0300 Message-ID: <20020501173516.76230.qmail@web14804.mail.yahoo.com> Received: from [200.243.206.160] by web14804.mail.yahoo.com via HTTP; Wed, 01 May 2002 14:35:16 ART Date: Wed, 1 May 2002 14:35:16 -0300 (ART) From: =?iso-8859-1?q?Ricardo=20Miranda?= Subject: Re: [obm-l] Re: [obm-l] Equacao do Universo To: obm-l@mat.puc-rio.br In-Reply-To: <3CC83D560000E715@www.zipmail.com.br> MIME-Version: 1.0 Content-Type: text/plain; charset=iso-8859-1 Content-Transfer-Encoding: 8bit Sender: owner-obm-l@sucuri.mat.puc-rio.br Precedence: bulk Reply-To: obm-l@mat.puc-rio.br > >Olah a todos, > > > >O que eh Equacao do Universo? (se eh que isso existe) > > Se é o que estou pensando, é a "Equacao do Tudo", que alguns acreditam que, qdo for encontrada, será a Lei Geral para tudo que acontece no Universo, tudo poderá ser previsto pelos conjuntos de solucoes desta equacao. Na minha opiniao é mais uma questao filosofica do que matematica. ===== []s -- Ricardo Miranda ricardomirandabr@yahoo.com.br http://rm2.hpg.ig.com.br/ _______________________________________________________________________ Yahoo! Encontros O lugar certo para você encontrar aquela pessoa que falta na sua vida. Cadastre-se hoje mesmo! http://br.encontros.yahoo.com/ ========================================================================= Instruções para entrar na lista, sair da lista e usar a lista em http://www.mat.puc-rio.br/~nicolau/olimp/obm-l.html O administrador desta lista é ========================================================================= From owner-obm-l@sucuri.mat.puc-rio.br Wed May 1 18:15:49 2002 Return-Path: Received: (from majordom@localhost) by sucuri.mat.puc-rio.br (8.9.3/8.9.3) id SAA06536 for obm-l-list; Wed, 1 May 2002 18:15:18 -0300 Received: from mta-cl.homeshopping.com.br (webmail.vento.com.br [200.170.17.71]) by sucuri.mat.puc-rio.br (8.9.3/8.9.3) with ESMTP id SAA06533 for ; Wed, 1 May 2002 18:15:16 -0300 Received: from vento.com.br (200.170.42.60) by mta-cl.homeshopping.com.br (5.1.065) (authenticated as tessa@vento.com.br) id 3CCE8A5000082500 for obm-l@mat.puc-rio.br; Wed, 1 May 2002 18:02:51 -0300 Message-ID: <3CD05800.DB78CD92@vento.com.br> Date: Wed, 01 May 2002 18:02:56 -0300 From: Alexandre Tessarollo X-Mailer: Mozilla 4.7 [en] (Win98; I) X-Accept-Language: en MIME-Version: 1.0 To: obm-l@mat.puc-rio.br Subject: Re: [obm-l] Re: ??? References: <3CCE8A5700000516@mta-cl.homeshopping.com.br> <001101c1f105$d32f2fa0$55f7fea9@p5f3e6> Content-Type: multipart/alternative; boundary="------------37013D61026CE4407B997E49" Sender: owner-obm-l@sucuri.mat.puc-rio.br Precedence: bulk Reply-To: obm-l@mat.puc-rio.br --------------37013D61026CE4407B997E49 Content-Type: text/plain; charset=iso-8859-1 Content-Transfer-Encoding: 8bit Discordo da sua resposta... Observe o trecho do enunciado: "...o caminhão descarregou as mudas no canteiro central,no local onde seria plantada a primeira muda... partindo do lugar onde as mudas foram colocadas,ele [o jardineiro] pegou 3 mudas de cada vez,plantou-as nos locais designados,enfileirando-as uma após a outra..." Portanto, as mudas estvam no lugar da primeira e o jardineiro parte necessariamente dali. O seu raciocínio me lembra um problema que o Gugu e o Nicolau criaram a partir de uma situação real. Se não me engano, eles estariam em Berlim, numa rua principal querendo chegar em uma determinada transversal, percorrendo a menor distância possível. Como não sabiam p/q lado ficava a transversal, caminharam um quarteirão p/a direita, voltaram, foram um p/a esquerda, voltaram, 2 p/a dir, volta, etc.... Assim, admitindo que os dois não pudessem se separar, eles percorreriam a menor distância possível até achar a rua. Fazendo um paralelo, o seu jardinerio estaria percorrendo a menor distância possível p/plantar as mudas. Jardineiro esperto esse, hein? :-)) []'s Alexandre Tessarollo PS: N e Gugu, desculpem se deturpei a história. Creio q vcs usaram a questão em alguma olimpíada e depois publicaram numa Eureka contando a historinha... Mas isso diz a minha [péssima] memória :-) Eder wrote: > Alguns colegas meus acharam 67.Ninguém achou 65,ainda.Eu considerei > que o jardineiro partia do centro da fila de mudas e que ia colocando > 3 mudas no lado direito,3 no lado esquerdo,...,e cheguei a uma > distância total de 35 hectômetros.Porém,eu somei a distância da última > viagem,o que não seria necessário,pela sua resolução.Vamos ver se > alguém esclarece isso... ----- Original Message ----- > > From:Alexandre Tessarollo > To: obm-l@mat.puc-rio.br > Sent: Wednesday, May 01, 2002 4:26 AM > Subject: [obm-l] Re: ??? > > > > Bem, ele planta a primeira muda, anda 5m, planta a 2a, > anda +5m, planta > a terceira e volta 10m até a > origem. Logo, sendo d[n] a distância percorrida p/plantar o > n-ésimo trio > de árvores, temos d[1]=20m. > > P/plantar a 4a muda, ele anda 10+5m, depois +5 p/a 5a, > +5 p/a sexta > e volta +25m até a origem. Ou > seja, d[2]=50m. > > Se vc observar, d[n]=2d[n-1]+2*(5+5+5) ou seja: > d[n]=2*(d[n-1]+15). > > Assim, temos > d[1]=20 > d[2]=50 > d[3]=130 > d[4]=290 > d[5]=610 > d[6]=1250 > d[7]=2550 ... > > Confesso que não consegui achar nenhum padrão nessa > sequencia, a não ser > que > d[n]=3d[n-1]-d[n-2] para n>2 > d[n]=8d[n-2]-3d[n-3] para n>3 > > Então, tentemos de uma forma mais simples: > os primeiros 10m ele percorre 2n vezes no trio n de mudas > (10*2*n); > os 15m seguintes ele percorre 2(n-1) vezes (15*2*(n-1)); > os 15m depois, 2(n-2) vezes (15*2*(n-2)); > e assim sucessivamente. > > Lembrando que há 21 trios de mudas, a distância total > percorrida seria > de > D=10*2*21+15*2*20+15*2*19+...15*2*1 > D=420+15*2*(20+19+18+...+1) > D=6720 > > Vale lembra que, após plantar a última muda o jardineiro > NÃO precisa > voltar a origem. Portanto, > devemos excluir a distância entre a última e a primeira > muda, ou seja, precisamos > excluir 100m da conta - > se alguém não entedeu pq 100m, imagine 21 MARCOS numa > estrada, distantes > 5m uns dos outros e > tente achar a distância total. > > Assim, a distância percorrida pelo nosso jardineiro do > plantio da primeira > muda até o plantio da última > muda é de 6620m ou, se vc preferir, 66,2 hectômetros(a > abreviação é hm ou > Hm?) > > Se você puder colocar a resoluçào daquelas pessoas que > acharm 65 seria > legal. Obviamente, uma das > 2 está errada e a outra talvez esteja certa. De qq forma, > comentem a minha > resposta e vamos ver como > se acha 65... > > []'s > > Alexandre Tessarollo > > PS: NÃO tenho certeza dessa solução, ainda vou revisá-la com > a devida calma. > Achei melhor publicá-la > logo pq sei q depois de revisá-la num papel não vou ter a > necessária paciência > p/digitar... :-)) A > propósito., minhas resolução e resposta bateram com a sua? > Caso não, publique-as > vc tb! > > Eder wrote: > > Olá colegas de lista, O seguinte problema,proposto em um > vestibular da > UNB,está causando uma > controvérsia lá no colégio...Tenho um gabarito dizendo que > a resposta > é 65,porém eu não consigo chegar a > esse resultado,nem alguns colegas de sala.Se alguém puder > resolver,agradeço. > > No projeto urbanístico de > uma cidade ,o paisagista previu a urbanização do canteiro > central de uma > das avenidas,com o plantio de > 63 mudas de Flamboyant,todas dispostas em linha reta e > distantes 5m uma > da outra.No dia do plantio,o > caminhão descarregou as mudas no canteiro central,no local > onde seria > plantada a primeira muda.Um > jardineiro foi designado para executar o serviço.Para > isso,partindo do > lugar onde as mudas foram > colocadas,ele pegou 3 mudas de cada vez,plantou-as nos > locais designados,enfileirando-as > uma após a > outra.Calcule ,em hectômetros,a distância total mínima > percorrida pelo > jardineiro após finalizar o trabalho. > --------------37013D61026CE4407B997E49 Content-Type: text/html; charset=us-ascii Content-Transfer-Encoding: 7bit  
    Discordo da sua resposta... Observe o trecho do enunciado: "...o  caminhão descarregou as mudas no canteiro central,no local onde seria plantada a primeira muda... partindo do lugar onde as mudas foram colocadas,ele [o jardineiro] pegou 3 mudas de cada vez,plantou-as nos locais designados,enfileirando-as uma após a outra..." Portanto, as mudas estvam no lugar da primeira e o jardineiro parte necessariamente dali.

    O seu raciocínio me lembra um problema que o Gugu e o Nicolau criaram a partir de uma situação real. Se não me engano, eles estariam em Berlim, numa rua principal querendo chegar em uma determinada transversal, percorrendo a menor distância possível. Como não sabiam p/q lado ficava a transversal, caminharam um quarteirão p/a direita, voltaram, foram um p/a esquerda, voltaram, 2 p/a dir, volta, etc.... Assim, admitindo que os dois não pudessem se separar, eles percorreriam a menor distância possível até achar a rua.

    Fazendo um paralelo, o seu jardinerio estaria percorrendo a menor distância possível p/plantar as mudas. Jardineiro esperto esse, hein? :-))
 

[]'s

Alexandre Tessarollo

PS: N e Gugu, desculpem se deturpei a história. Creio q vcs usaram a questão em alguma olimpíada e depois publicaram numa Eureka contando a historinha... Mas isso diz a minha [péssima] memória :-)
 

Eder wrote:

 Alguns colegas meus acharam 67.Ninguém achou 65,ainda.Eu considerei que o jardineiro partia do centro da fila de mudas e que ia colocando 3 mudas no lado direito,3 no lado esquerdo,...,e cheguei a uma distância total de 35 hectômetros.Porém,eu somei a distância da última viagem,o que não seria necessário,pela sua resolução.Vamos ver se alguém esclarece isso...  ----- Original Message -----
Sent: Wednesday, May 01, 2002 4:26 AM
Subject: [obm-l] Re: ???

 

    Bem, ele planta a primeira muda, anda 5m, planta a 2a, anda +5m, planta
a terceira e volta 10m até a
origem. Logo, sendo d[n] a distância percorrida p/plantar o n-ésimo trio
de árvores, temos d[1]=20m.

    P/plantar a 4a muda, ele anda 10+5m, depois +5 p/a 5a, +5 p/a sexta
e volta +25m até a origem. Ou
seja, d[2]=50m.

    Se vc observar, d[n]=2d[n-1]+2*(5+5+5) ou seja:
d[n]=2*(d[n-1]+15).

    Assim, temos
d[1]=20
d[2]=50
d[3]=130
d[4]=290
d[5]=610
d[6]=1250
d[7]=2550  ...

Confesso que não consegui achar nenhum padrão nessa sequencia, a não ser
que
d[n]=3d[n-1]-d[n-2]      para n>2
d[n]=8d[n-2]-3d[n-3]    para n>3

    Então, tentemos de uma forma mais simples:
os primeiros 10m ele percorre 2n vezes no trio n de mudas (10*2*n);
os 15m seguintes ele percorre 2(n-1) vezes (15*2*(n-1));
os 15m depois, 2(n-2) vezes (15*2*(n-2));
e assim sucessivamente.

    Lembrando que há 21 trios de mudas, a distância total percorrida seria
de
D=10*2*21+15*2*20+15*2*19+...15*2*1
D=420+15*2*(20+19+18+...+1)
D=6720

    Vale lembra que, após plantar a última muda o jardineiro NÃO precisa
voltar a origem. Portanto,
devemos excluir a distância entre a última e a primeira muda, ou seja, precisamos
excluir 100m da conta -
se alguém não entedeu pq 100m, imagine 21 MARCOS numa estrada, distantes
5m uns dos outros e
tente achar a distância total.

    Assim, a distância percorrida pelo nosso jardineiro do plantio da primeira
muda até o plantio da última
muda é de 6620m ou, se vc preferir, 66,2 hectômetros(a abreviação é hm ou
Hm?)

    Se você puder colocar a resoluçào daquelas pessoas que acharm 65 seria
legal. Obviamente, uma das
2 está errada e a outra talvez esteja certa. De qq forma, comentem a minha
resposta e vamos ver como
se acha 65...

[]'s

Alexandre Tessarollo

PS: NÃO tenho certeza dessa solução, ainda vou revisá-la com a devida calma.
Achei melhor publicá-la
logo pq sei q depois de revisá-la num papel não vou ter a necessária paciência
p/digitar... :-)) A
propósito., minhas resolução e resposta bateram com a sua? Caso não, publique-as
vc tb!

Eder wrote:

   Olá colegas de lista, O seguinte problema,proposto em um vestibular da
UNB,está causando uma
  controvérsia lá no colégio...Tenho um gabarito dizendo que a resposta
é 65,porém eu não consigo chegar a
  esse resultado,nem alguns colegas de sala.Se alguém puder resolver,agradeço.

No projeto urbanístico de
  uma cidade ,o paisagista previu a urbanização do canteiro central de uma
das avenidas,com o plantio de
  63 mudas de Flamboyant,todas dispostas em linha reta e distantes 5m uma
da outra.No dia do plantio,o
  caminhão descarregou as mudas no canteiro central,no local onde seria
plantada a primeira muda.Um
  jardineiro foi designado para executar o serviço.Para isso,partindo do
lugar onde as mudas foram
  colocadas,ele pegou 3 mudas de cada vez,plantou-as nos locais designados,enfileirando-as
uma após a
  outra.Calcule ,em hectômetros,a distância total mínima percorrida pelo
jardineiro após finalizar o trabalho.

--------------37013D61026CE4407B997E49-- ========================================================================= Instruções para entrar na lista, sair da lista e usar a lista em http://www.mat.puc-rio.br/~nicolau/olimp/obm-l.html O administrador desta lista é ========================================================================= From owner-obm-l@sucuri.mat.puc-rio.br Wed May 1 20:24:45 2002 Return-Path: Received: (from majordom@localhost) by sucuri.mat.puc-rio.br (8.9.3/8.9.3) id UAA07361 for obm-l-list; Wed, 1 May 2002 20:23:35 -0300 Received: from br.inter.net (opt-0-15.br.inter.net [200.185.56.15] (may be forged)) by sucuri.mat.puc-rio.br (8.9.3/8.9.3) with ESMTP id UAA07358 for ; Wed, 1 May 2002 20:23:33 -0300 Received: from [200.222.210.215] (helo=ig) by br.inter.net with esmtp (Exim 3.36 #1) id 1733Fq-0004ec-00 for obm-l@mat.puc-rio.br; Wed, 01 May 2002 20:11:22 -0300 Message-ID: <013501c1f166$49e531c0$e01ed8c8@ig> From: "Josimar" To: References: <00E50A093F50D511BBED00104BC54194394CD8@EXCHANGE> <3CCDCEE8.4090404@centroin.com.br> Subject: Re: RES: [obm-l] desigualdades.... Date: Wed, 1 May 2002 19:40:34 -0300 MIME-Version: 1.0 Content-Type: text/plain; charset="iso-8859-1" Content-Transfer-Encoding: 8bit X-Priority: 3 X-MSMail-Priority: Normal X-Mailer: Microsoft Outlook Express 5.00.2615.200 X-MIMEOLE: Produced By Microsoft MimeOLE V5.00.2615.200 Sender: owner-obm-l@sucuri.mat.puc-rio.br Precedence: bulk Reply-To: obm-l@mat.puc-rio.br Este problema é realmente muito interessante. Mas há mais coisas interessantes que podem ser tiradas dele: 1) Como foi feito com o número 36, de quantas maneiras podemos escrever um número natural N como o produto de 3 inteiros positivos? (Este problema já surgiu aqui na lista, porém há muito tempo). 2) A+B+C=S e A*B*C=P. Se S e P são inteiros, verifique se forçosamente A, B e C também serão inteiros. (No caso do problema, S é o número da casa em frente e P=36). ----- Original Message ----- From: Augusto César Morgado To: Sent: Monday, April 29, 2002 7:53 PM Subject: Re: RES: [obm-l] desigualdades.... Faça a lista de todas as possibilidades sabendo que o produto é 36. Lembre-se que o cara sabe a soma das idades. Pense! Se não consegiur, a solução está abaixo. Se o produto das três idades é 36, as idades podem ser 1,1,36 (soma 38) 1,2,18 (soma 21) 2,2,9 (soma 13) 1,4,9 (soma 14) 1,3,12 (soma 16) 2,3,6 (soma 11) 3,3,4 (soma 10) 1,6,6 (soma 13) Como o cara conhece a soma das idades, ele pode determinar essas idades, a não ser que a soma seja 13, caso em que as idades poderiam ser 1,6,6 ou 2, 2, 9. então, a soma é 13 e, como há uma mais velha, as idades são 2, 2 e 9. Fabio Nogueira wrote: >Concordo plenamente. A grande dificuldade que vejo nos problemas é >agarrar-se a determinada ótica e morrer com ela até o final, principalmente >para pessoas que como eu não são matemáticos. Alternar diversas resoluções >em um curto espaço de tempo não é tarefa das mais fáceis como nosso amigo >quer fazer crer. >Segue problema que estou com dificuldades, apesar da relativa facilidade do >enunciado > >Dois homens estavam conversando num bar quando um virou para o outro e disse > - Tenho 3 filhas, a soma de suas idades é igual ao número da casa em frente >e o produto é 36 > - Posso determinar as idades de suas filhas apenas com esse dados? > - Não. Dar-lhe-ei um dado fundamental:minha filha mais velha toca piano >DETERMINE AS IDADES DAS FILHAS E O NÚMERO DA CASA EM FRENTE >DESDE JÁ AGRADEÇO > > >-----Mensagem original----- >De: DEOLIVEIRASOU@aol.com [mailto:DEOLIVEIRASOU@aol.com] >Enviada em: Segunda-feira, 29 de Abril de 2002 14:48 >Para: obm-l@mat.puc-rio.br >Assunto: Re: [obm-l] desigualdades.... > > >"Caro" Dirichilet....Quando coloco uma questão dessas na lista, na verdade >estou tentando ver se alguem fez de outro modo, sem indução. O uso da >indução é meio óbvio nesse contexto....o que quero é ver se alguem consegue >resolver essas desigualdades em termos daquelas desigualdades "elementares" >do Eureka. >ps- Vc me ajuda se vc quiser, eu não te obrigo.... > Grato > Crom > >========================================================================= >Instruções para entrar na lista, sair da lista e usar a lista em >http://www.mat.puc-rio.br/~nicolau/olimp/obm-l.html >O administrador desta lista é >========================================================================= > > ========================================================================= Instruções para entrar na lista, sair da lista e usar a lista em http://www.mat.puc-rio.br/~nicolau/olimp/obm-l.html O administrador desta lista é ========================================================================= ========================================================================= Instruções para entrar na lista, sair da lista e usar a lista em http://www.mat.puc-rio.br/~nicolau/olimp/obm-l.html O administrador desta lista é ========================================================================= From owner-obm-l@sucuri.mat.puc-rio.br Wed May 1 20:59:19 2002 Return-Path: Received: (from majordom@localhost) by sucuri.mat.puc-rio.br (8.9.3/8.9.3) id UAA07738 for obm-l-list; Wed, 1 May 2002 20:59:12 -0300 Received: from matinhos.terra.com.br (matinhos.terra.com.br [200.176.3.21]) by sucuri.mat.puc-rio.br (8.9.3/8.9.3) with ESMTP id UAA07735 for ; Wed, 1 May 2002 20:59:10 -0300 Received: from srv11-sao.terra.com.br (srv11-sao.terra.com.br [200.176.3.38]) by matinhos.terra.com.br (Postfix) with ESMTP id F401D4817B for ; Wed, 1 May 2002 23:47:39 +0000 (GMT) Received: from terra.com.br (200-206-243-51.dsl.telesp.net.br [200.206.243.51]) (authenticated user lponce) by srv11-sao.terra.com.br (Postfix) with ESMTP id 31ED1C94E2 for ; Wed, 1 May 2002 20:47:39 -0300 (EST) Message-ID: <3CD07E7C.6A104BC@terra.com.br> Date: Wed, 01 May 2002 20:47:09 -0300 From: Luiz Antonio Ponce Alonso X-Mailer: Mozilla 4.79 [en] (Windows NT 5.0; U) X-Accept-Language: en MIME-Version: 1.0 To: "obm-l@mat.puc-rio.br" Subject: [obm-l] Re:(Obm 1)Geometria Content-Type: multipart/alternative; boundary="------------390BAD813CED457345BB4408" Sender: owner-obm-l@sucuri.mat.puc-rio.br Precedence: bulk Reply-To: obm-l@mat.puc-rio.br --------------390BAD813CED457345BB4408 Content-Type: text/plain; charset=iso-8859-1 Content-Transfer-Encoding: 8bit Olá Raul, Vai abaixo uma solução para o seguinte problema proposto por você: Problema: O diâmetro de uma circunferência está cortado por uma corda que faz 45 graus com ele e a corda fica dividida em partes iguais a 2sqtr3 e 2sqtr15. Qual a medida do raio da circunferência ? [Image] Uma solução possível Seja MN diametro e O centro de uma circunferência (C). Seja AB uma corda de (C) e T o seu ponto médio, onde S a intersecção desta corda com o diametro MN e 45 graus a medida do ângulo OST . Nestas condições, decorre do enunciado que BS = 2sqrt15 e AS = 2sqrt3 Assim, TB =AT= (BS + AS)/2 = sqrt(15) + sqrt(3) ......(1) Por outro lado, sendo 45 graus a medida do ângulo OST, conclui-se que o triângulo retângulo OTS é isosceles, consequentemente OT = ST = AT - AS = sqrt(15)+sqrt(3) - 2sqrt(3) = sqrt(15) - sqrt(3) ....(2) Aplicando o teorema de Pitagoras no triângulo retângulo OTB, tem-se de (1) e (2): (TB)^2 = (OT)^2 + (TB)^2 = 2.[ 15 + 3 ] = 36 Portanto, TB = 6, isto é, a medida do raio da circunferência (C) é 6 PONCE --------------390BAD813CED457345BB4408 Content-Type: multipart/related; boundary="------------CDC6D12FA4138D4F48C6F22F" --------------CDC6D12FA4138D4F48C6F22F Content-Type: text/html; charset=us-ascii Content-Transfer-Encoding: 7bit  
Olá Raul,
Vai abaixo uma solução para o seguinte problema proposto por você:
Problema:
O diâmetro de uma circunferência está cortado por uma corda que faz 45 graus com ele e a corda fica dividida em
partes iguais a 2sqtr3 e 2sqtr15. Qual a medida do raio da circunferência ?

Uma solução possível
Seja MN  diametro e O centro de uma circunferência (C).
Seja AB uma corda  de (C) e T  o seu ponto médio, onde S a intersecção desta corda com o diametro MN
e 45 graus a medida do  ângulo OST .

Nestas condições, decorre do enunciado que
 BS = 2sqrt15   e  AS = 2sqrt3

Assim,  TB =AT= (BS + AS)/2 = sqrt(15) + sqrt(3)                                  ......(1)

Por outro lado, sendo 45 graus a medida do ângulo OST, conclui-se que
o triângulo retângulo OTS é isosceles, consequentemente
OT = ST = AT - AS = sqrt(15)+sqrt(3) - 2sqrt(3) = sqrt(15) - sqrt(3)        ....(2)

Aplicando o teorema de Pitagoras no triângulo retângulo OTB, tem-se  de (1) e (2):
(TB)^2 = (OT)^2 + (TB)^2
            = 2.[ 15 + 3 ]
            = 36
Portanto, TB = 6, isto é,  a medida do raio da circunferência (C) é 6
PONCE --------------CDC6D12FA4138D4F48C6F22F Content-Type: image/jpeg Content-ID: Content-Transfer-Encoding: base64 Content-Disposition: inline; filename="C:\DOCUME~1\LUIZPO~1\CONFIG~1\Temp\nsmailOJ.jpeg" /9j/4AAQSkZJRgABAQAAAQABAAD/2wBDACAWGBwYFCAcGhwkIiAmMFA0MCwsMGJGSjpQdGZ6 eHJmcG6AkLicgIiuim5woNqirr7EztDOfJri8uDI8LjKzsb/2wBDASIkJDAqMF40NF7GhHCE xsbGxsbGxsbGxsbGxsbGxsbGxsbGxsbGxsbGxsbGxsbGxsbGxsbGxsbGxsbGxsbGxsb/wAAR CAC1AMADASIAAhEBAxEB/8QAHwAAAQUBAQEBAQEAAAAAAAAAAAECAwQFBgcICQoL/8QAtRAA AgEDAwIEAwUFBAQAAAF9AQIDAAQRBRIhMUEGE1FhByJxFDKBkaEII0KxwRVS0fAkM2JyggkK FhcYGRolJicoKSo0NTY3ODk6Q0RFRkdISUpTVFVWV1hZWmNkZWZnaGlqc3R1dnd4eXqDhIWG h4iJipKTlJWWl5iZmqKjpKWmp6ipqrKztLW2t7i5usLDxMXGx8jJytLT1NXW19jZ2uHi4+Tl 5ufo6erx8vP09fb3+Pn6/8QAHwEAAwEBAQEBAQEBAQAAAAAAAAECAwQFBgcICQoL/8QAtREA AgECBAQDBAcFBAQAAQJ3AAECAxEEBSExBhJBUQdhcRMiMoEIFEKRobHBCSMzUvAVYnLRChYk NOEl8RcYGRomJygpKjU2Nzg5OkNERUZHSElKU1RVVldYWVpjZGVmZ2hpanN0dXZ3eHl6goOE hYaHiImKkpOUlZaXmJmaoqOkpaanqKmqsrO0tba3uLm6wsPExcbHyMnK0tPU1dbX2Nna4uPk 5ebn6Onq8vP09fb3+Pn6/9oADAMBAAIRAxEAPwDoKKKKACiiigAooooAKKKKACiiigAooooA KKKKACiioL5ilhcMpKsImIIPIODQBPRWIkkiRae4+0RtI6BpJJSyuCORjJ6/T8q26ACiiigA ooooAKKKKACiiigAooooAKKKKACio5ZkiwGOXb7qD7zfQf5xUfkyTf8AHyV2/wDPJCcfieN3 0wBzzmgBWu49xSLM0gOCkeCQfc9B+JFJm7k5AihHYMC5P1wRj8zU6qEUKoCqBgADgCloAr/Z mPD3U7r3GVXP4qAf1qOfTYJ4WiZ5wG/6bMf0JIq5RQBVjsIY41QPPhQAP37j+Rp32eVfuXco x0VgrD6HjJ/PPvViigCvuuo/vJHMPWP5G/AE4/Uf4smubWWGSGeUQF1KsshCsMjHGev1GRVu s+4/4mEiW4/49j87n++B0+gJ6euDjpyAPjsI2S3zcSSxRFWjUlccDg5ABP51dqssUlqoWAB4 VGBGxO5R7E9fofzAqaKWOZd0bBgDg+oPoR2PtQA+iiigAooooAKKKKACiiigAooooAKilkIb yogDKRnnoo9T/h3/ADIWeXykBA3MxCquepP+cn2Bohi8sMSdzudztjGTjHTtwAKACKLZlmO6 RvvNjr7D0Ht/XJqSiigAooooAKKKKACiiq//AB9/9e//AKM/+x/n9OoAjMLhSzELagZJJ4kH /wAT/P6dXWqkq0zgh5TuwRyq9h7cdR6k0lx+9kS3HQ/NJ/ujt+J49wGqxQAVFLEWbzImCSgY yRkEehHGf8++ZaKAI4pd+VYbZF+8uenuPUe/9cipKimjLMssZAlQEDPQg4yD+Q59vwLoZBNE siggMOh6g9wfcUAPooooAKKKKACiiigAooqvefOi245MxwR/sfxfpxn1IoALf985uT91wBF/ udc+2T+gXuKsUUUAFFFFABRSNkqdpAbHBIyAao2891LfTwM8O2ApkiM5YEZ/vcfrQBfooqv/ AMff/Xv/AOjP/sf5/TqAH/H3/wBe/wD6M/8Asf5/TrOzBFLMQqgZJJ4Apar3H72RLcdD80n+ 6O34nj3AagBbVSVaZwQ8p3YI5Vew9uOo9SanoooAKKKKACq7/uLgS/8ALOTCv7NnCn8c4P8A wHsKsU2RFljaNxlXBUj1BoAdRUNo7PAokOZU+ST/AHh1P49R7EVNQAUUUUAFFFFABVeP95ey v2iAjHsThm/Tb+R/GxVey5hZ+u+R23f3huOD78Yx7YoAsUUUUAFFFFABVOOL7NeXl1NJGsUu zBJxjAxzVyqsiLfoUcZtj/5E/wDrfz+nUAVgbxSAxWAjqOsn/wBj/P6dZIpCW8qUASgZ46MP Uf4dvyJjVhZqI2z5IGEYDJX0U/0Pfp16vjRncSzDBH3E/uf/AF/5dB3JAJWYIpZiFUDJJPAF Q2qkq0zgh5TuwRyq9h7cdR6k0lx+9kS3HQ/NJ/ujt+J49wGqxQAUUUUAFFFFABRRRQBXX93f OvaZN4HuuAT+RX8j+Niq9z8s1s/TEhUt7FTx+J2/jirFABRRRQAVXtb2G6tftCNtTnO4gbce vp61Yrm7OyvFt4rcLIkV3nzsggx4Jz9MjA560Abttdx3NotyuUjIJ+fAwAec/lTdNYNptsVI I8pRwe4HNQaTAf7GjgnjZchlZWBBwSaNPsbU2FuTbxMWjViWUEkkZ6mgDQoqv9htP+fWD/v2 KPsNp/z6wf8AfsU9ALFNkkSJC8jqijqzHAFQ/YbT/n1g/wC/YqvJY2tyTGlvEsan53VQCSD9 0Efqfw65waATqwvRlSDbZI4P+sI6/wDAf5/TrZrHXyAkph0yKSKCTyjxlzjAJAwc9fWtD7Da f8+sH/fsUaATsodSrAMpGCCOCKg8z7LxM/7n+GRj932Y/wAj+B56n2G0/wCfWD/v2KbJp1lI hRrWIA/3V2n8xRoA6z/eRm4PWbDD2X+Eflz9SasVhWLQyNZhrO28u5DhR5fzJt6ZJ+9+lav2 G0/59YP+/Yo0AsUVX+w2n/PrB/37FH2G0/59YP8Av2KNALFFV/sNp/z6wf8AfsUfYbT/AJ9Y P+/Yo0AsUVX+w2n/AD6wf9+xR9htP+fWD/v2KNAG6jIkdupd1UebHyxx/GD/ACBqprEt1HdW n2RiHxI2znD4AOCO/epL+wtDAv8Ao8a/vUGVXacFgD09iaty2yS3MM7Ft0O7aB0ORg5pAU9M uvtV9eOrs0WIyik525Xke3vWlVa1sYLSWZ4AV80glewxnp+dWaACiiigAqvY8WwXsjuij0Cs QB+QFWKrxfJeTof+WmJAfXgKR+G0f99CgCxRRUDMZ2McZKxg4dweSf7o/qe3Qc9AAZjOxjjJ WMHDuDyT/dH9T26DnpMqhFCqAqgYAA4AoVQihVAVQMAAcAUtAGVcwXTvK0duEud+I542CjZk fe5yeB6GtWiigAooooAy7PTZoJLQO0ey18zBBOX3e2OPzNalFFABRRRQAUUUUAFFFFAFe65e 2Q/daXkeuFLD9QKsVXH7y/JHSKPaT6liDj8Ao/76FWKACiiigAooooAKr3X7sx3A/wCWZw3+ 4SAfpjg5/wBmn3U4traSZsYRScE4yewqrp7y3NrLb3wBmQlJBkZIIyOnTg4/CgCfzPtXEL/u f4pFP3vZT/M/gOek6qEUKoCqBgADgCq1ii2yNaKMCHlf9pTkg/nkfh2zVqgAooooAKKKKACi iigAooooAKKKKACiiigApGYIpZiFUDJJPAFLVef99KLf+HAeT6Z4X8cHPsCO9AC2anyfMcES SnzGBGCCRwPwGB+FT0UUAFFFFABRRRQBXvbX7XCsRfam8FxjO4DnHt9abBZC3u3mjkbbIgDI xLEkdDuJz04xVqigCKeMuFeMgSocqT3HcH2P/wBftTopBKuQCCDhlPVT6Gn1DIjI5lhGSfvp /f8A/r/z6HsQATUUyKQSrkAgg4ZT1U+hp9ABRRRQAUUUUAFFFFABRRRQAUUVHLKI8AKzu33U XGT69aACWXZhVG6Rvurnr7n0Hv8A1wKIIvKjAJ3OeXfH3j3P+enSkijIbzZSDKRjjoo9B/j3 /ICWgAooooAKKKKACiiigAooooAKKKKAIpYQ7eYjFJQMBh0PsR3HX8zjFNW4KsEuEETE4U7s qx9AfX2IHtnFT0jKHUqwDKRggjgigBaKr/ZjHzbStH/stlk/Inj8CKPNuE4e38z0MTDn6hsY /M0AWKKr/bYB94yIP70kbIo+pIxR9vs/+fuD/v4KALFFV/t9n/z9wf8AfwUfbIj9xZXz02xN hvocY/HOKALFFV99zJ9yFYR/elOSP+Ar1/Mf4n2RX5uGaYnqGOE+m3p+eT70AH2hpuLVVdf+ epPyfhjr/L3zUkUKxZILMzfeZjkn/D6DjmpKKACiiigAooooAKKKKACiiigAooooAKKKKACi iigAooooAKKKKACiiigAooooAKKKKACiiigAooooAKKKKACiiigAooooAKKKKACiiigAoooo AKKKKACiiigAooooAKKKKACiiigAooooAKKKKAP/2Q== --------------CDC6D12FA4138D4F48C6F22F-- --------------390BAD813CED457345BB4408-- ========================================================================= Instruções para entrar na lista, sair da lista e usar a lista em http://www.mat.puc-rio.br/~nicolau/olimp/obm-l.html O administrador desta lista é ========================================================================= From owner-obm-l@sucuri.mat.puc-rio.br Wed May 1 22:31:49 2002 Return-Path: Received: (from majordom@localhost) by sucuri.mat.puc-rio.br (8.9.3/8.9.3) id WAA08531 for obm-l-list; Wed, 1 May 2002 22:30:14 -0300 Received: from sr1.terra.com.br (sr1.terra.com.br [200.176.3.16]) by sucuri.mat.puc-rio.br (8.9.3/8.9.3) with ESMTP id WAA08528 for ; Wed, 1 May 2002 22:30:12 -0300 Received: from srv11-sao.terra.com.br (srv11-sao.terra.com.br [200.176.3.38]) by sr1.terra.com.br (Postfix) with ESMTP id 937E46F0C0 for ; Wed, 1 May 2002 22:18:42 -0300 (EST) Received: from terra.com.br (200-206-243-51.dsl.telesp.net.br [200.206.243.51]) (authenticated user lponce) by srv11-sao.terra.com.br (Postfix) with ESMTP id 2CB4EC94DB for ; Wed, 1 May 2002 22:18:42 -0300 (EST) Message-ID: <3CD093C5.228387CC@terra.com.br> Date: Wed, 01 May 2002 22:17:58 -0300 From: Luiz Antonio Ponce Alonso X-Mailer: Mozilla 4.79 [en] (Windows NT 5.0; U) X-Accept-Language: en MIME-Version: 1.0 To: obm-l@mat.puc-rio.br Subject: Re: [obm-l] Flanders Olympiad novamente! References: Content-Type: text/plain; charset=iso-8859-1 Content-Transfer-Encoding: 8bit Sender: owner-obm-l@sucuri.mat.puc-rio.br Precedence: bulk Reply-To: obm-l@mat.puc-rio.br caros amigos, uma pequena ressalva para o problema e solução dada. O valor (-2(p+q))/pq é uma diferença possível para o problema. Entretanto, 2(p+q))/pq, poderia ser também uma das diferenças entre dois termos consecutivos da sequência, apesar de não constar do enunciado. PONCE Paulo Santa Rita wrote: > Ola Felipe e demais > colegas desta lista, > > A questao e muito simples e deve haver um montao de maneiras de faze-la. > Tenho certeza que voce encontraria um caminho se pensasse um pouco mais ... > Todavia, como voce parece estar aflito, atendendo seu apelo, uma forma > talvez razoavel seja a seguinte : > > Sp=q e Sq=p > (p(a1 + ap))/2 = q e (q(a1 + aq))/2 = p > a1 + ap = (2q)/p e a1 + aq = (2p)/q > > subtraindo a segunda equacao da primeira : > > ap - aq = (2q)/p - (2p)/q > mas : ap - aq = (p-q)R. Logo > (p-q)R = (2q)/p - (2p)/q > (p-q)R = (2(q-p)(q+p))/pq. Supondo p diferende de q > > R = (-2(p+q))/pq > > esse deve ser o valor que voce procura. Eu nao conferi. Da uma olhada. > > Um abraco > Paulo Santa Rita > 3,1821,300402 > > >From: "Felipe Marinho" > >Reply-To: obm-l@mat.puc-rio.br > >To: obm-l@mat.puc-rio.br > >Subject: [obm-l] Flanders Olympiad novamente! > >Date: Tue, 30 Apr 2002 16:24:17 -0400 > > > >Caros amigos da lista, > > > >Semana passada eu postei um questão aqui... pedindo uma ajuda de vocês em > >como resolvê-la. > >Porem, não foi retorno por parte de ninguem e nenhum tipo de comentário > >tambem foi feito. > > > >Por isso, venho aqui mais uma vez pedir a ajuda de vocês na resolução da > >mesmo. > > > >PS: Desculpe a insistência, mas estou realmente precisando de ajuda. > > > >Desculpe qualquer coisa, > >E agradeço desde já qualquer tipo de ajuda. > > > >Segue a questão: > > > >1) A soma dos p primeiros termos de uma sequencia aritmética é igual a q e > >a > >soma dos q primeiros termos dessa sequencia é dada por p, > >onde (p diferente de q). Qual das opções expressa o valor da diferença v > >entre 2 termos sucessivos dessa sequencia é: > > > >a) v= -2/(p-q) > >b) v= -2(p+q)/pq > >c) v= 2/(p+q) > >d) v= 2(p-q)/pq > >e) v= 2(p+q) > > > >------ FIM ------- > > > >Abraços > > > >Felipe Marinho > > > >_________________________________________________________________ > >Chegou o novo MSN Explorer. Instale já. É gratuito: > >http://explorer.msn.com.br > > > >========================================================================= > >Instruções para entrar na lista, sair da lista e usar a lista em > >http://www.mat.puc-rio.br/~nicolau/olimp/obm-l.html > >O administrador desta lista é > >========================================================================= > > colegas desta lista, > > _________________________________________________________________ > Chegou o novo MSN Explorer. Instale já. É gratuito: > http://explorer.msn.com.br > > ========================================================================= > Instruções para entrar na lista, sair da lista e usar a lista em > http://www.mat.puc-rio.br/~nicolau/olimp/obm-l.html > O administrador desta lista é > ========================================================================= ========================================================================= Instruções para entrar na lista, sair da lista e usar a lista em http://www.mat.puc-rio.br/~nicolau/olimp/obm-l.html O administrador desta lista é ========================================================================= From owner-obm-l@sucuri.mat.puc-rio.br Thu May 2 11:29:50 2002 Return-Path: Received: (from majordom@localhost) by sucuri.mat.puc-rio.br (8.9.3/8.9.3) id LAA14658 for obm-l-list; Thu, 2 May 2002 11:26:18 -0300 Received: from hotmail.com (law2-f103.hotmail.com [216.32.181.103]) by sucuri.mat.puc-rio.br (8.9.3/8.9.3) with ESMTP id LAA14654 for ; Thu, 2 May 2002 11:26:15 -0300 Received: from mail pickup service by hotmail.com with Microsoft SMTPSVC; Thu, 2 May 2002 07:26:45 -0700 Received: from 32.94.119.254 by lw2fd.hotmail.msn.com with HTTP; Thu, 02 May 2002 14:26:38 GMT X-Originating-IP: [32.94.119.254] From: "Paulo Santa Rita" To: obm-l@mat.puc-rio.br Subject: Re: [obm-l] Flanders Olympiad novamente! Date: Thu, 02 May 2002 14:26:38 +0000 Mime-Version: 1.0 Content-Type: text/plain; charset=iso-8859-1; format=flowed Message-ID: X-OriginalArrivalTime: 02 May 2002 14:26:45.0745 (UTC) FILETIME=[63372E10:01C1F1E5] Sender: owner-obm-l@sucuri.mat.puc-rio.br Precedence: bulk Reply-To: obm-l@mat.puc-rio.br Ola Felipe e demais colegas desta lista, Nao ha do que agradecer, foi um prazer. Tambem achei estranho voce nao estar conseguindo fazer. Mas agora voce explicou. Realmente, a expressao "Sequencia Aritmetica" e Sui Generis e pode levar a confusao. Mas, numa computacao final, nesta sua mensagem de agradecimento ha um problema interessante : Seja A1, A2, ..., An uma Progressao Aritmetica e Ap o P-esimo termo. Caracterize todos os naturais P ( Diga quem sao eles ) tais que existe um natural N ( N < P )tal que Sn=Ap. Ou seja : Em uma progressao aritmetica, quais sao os termos que sao iguais a soma de termos anteriores ? >From: "Felipe Marinho" >Reply-To: obm-l@mat.puc-rio.br >To: obm-l@mat.puc-rio.br >Subject: Re: [obm-l] Flanders Olympiad novamente! >Date: Wed, 01 May 2002 01:33:37 -0400 > >Caro Paulo, > >Muito obrigado por ter me ajudado na resolução do tal exercicio. >Porem, nas horas em que eu tentei resolver a questão... eu não tratava a >sequencia que a questão deu, como uma progressão aritmetica. O exercicio >disse "sequencia aritmetica" .. e pra mim isso poderia ser qualquer >sequencia numérica sem seguir uma determinada razão. Por isso eu estava >sentindo uma grande dificuldade na resolução. E é verdade, tratando a >sequencia como um progressão aritmetica... não há nada de dificil na >questão. > >Novamente, obrigado pela ajuda. > >Tudo de bom pra você, e para todos os amigos da lista. > >Até a próxima. > >Abraços >Felipe Marinho > >>From: "Paulo Santa Rita" >>Reply-To: obm-l@mat.puc-rio.br >>To: obm-l@mat.puc-rio.br >>Subject: Re: [obm-l] Flanders Olympiad novamente! >>Date: Tue, 30 Apr 2002 21:14:47 +0000 >> >>Ola Felipe e demais >>colegas desta lista, >> >>A questao e muito simples e deve haver um montao de maneiras de faze-la. >>Tenho certeza que voce encontraria um caminho se pensasse um pouco mais >>... >>Todavia, como voce parece estar aflito, atendendo seu apelo, uma forma >>talvez razoavel seja a seguinte : >> >>Sp=q e Sq=p >>(p(a1 + ap))/2 = q e (q(a1 + aq))/2 = p >>a1 + ap = (2q)/p e a1 + aq = (2p)/q >> >>subtraindo a segunda equacao da primeira : >> >>ap - aq = (2q)/p - (2p)/q >>mas : ap - aq = (p-q)R. Logo >>(p-q)R = (2q)/p - (2p)/q >>(p-q)R = (2(q-p)(q+p))/pq. Supondo p diferende de q >> >>R = (-2(p+q))/pq >> >>esse deve ser o valor que voce procura. Eu nao conferi. Da uma olhada. >> >>Um abraco >>Paulo Santa Rita >>3,1821,300402 >> >> >> >> >> >>>From: "Felipe Marinho" >>>Reply-To: obm-l@mat.puc-rio.br >>>To: obm-l@mat.puc-rio.br >>>Subject: [obm-l] Flanders Olympiad novamente! >>>Date: Tue, 30 Apr 2002 16:24:17 -0400 >>> >>>Caros amigos da lista, >>> >>>Semana passada eu postei um questão aqui... pedindo uma ajuda de vocês em >>>como resolvê-la. >>>Porem, não foi retorno por parte de ninguem e nenhum tipo de comentário >>>tambem foi feito. >>> >>>Por isso, venho aqui mais uma vez pedir a ajuda de vocês na resolução da >>>mesmo. >>> >>>PS: Desculpe a insistência, mas estou realmente precisando de ajuda. >>> >>>Desculpe qualquer coisa, >>>E agradeço desde já qualquer tipo de ajuda. >>> >>>Segue a questão: >>> >>>1) A soma dos p primeiros termos de uma sequencia aritmética é igual a q >>>e >>>a >>>soma dos q primeiros termos dessa sequencia é dada por p, >>>onde (p diferente de q). Qual das opções expressa o valor da diferença v >>>entre 2 termos sucessivos dessa sequencia é: >>> >>>a) v= -2/(p-q) >>>b) v= -2(p+q)/pq >>>c) v= 2/(p+q) >>>d) v= 2(p-q)/pq >>>e) v= 2(p+q) >>> >>>------ FIM ------- >>> >>>Abraços >>> >>>Felipe Marinho >>> >>>_________________________________________________________________ >>>Chegou o novo MSN Explorer. Instale já. É gratuito: >>>http://explorer.msn.com.br >>> >>>========================================================================= >>>Instruções para entrar na lista, sair da lista e usar a lista em >>>http://www.mat.puc-rio.br/~nicolau/olimp/obm-l.html >>>O administrador desta lista é >>>========================================================================= >> >> >>colegas desta lista, >> >> >>_________________________________________________________________ >>Chegou o novo MSN Explorer. Instale já. É gratuito: >>http://explorer.msn.com.br >> >>========================================================================= >>Instruções para entrar na lista, sair da lista e usar a lista em >>http://www.mat.puc-rio.br/~nicolau/olimp/obm-l.html >>O administrador desta lista é >>========================================================================= > > >_________________________________________________________________ >Chegou o novo MSN Explorer. Instale já. É gratuito: >http://explorer.msn.com.br > >========================================================================= >Instruções para entrar na lista, sair da lista e usar a lista em >http://www.mat.puc-rio.br/~nicolau/olimp/obm-l.html >O administrador desta lista é >========================================================================= _________________________________________________________________ Una-se ao maior serviço de email do mundo: o MSN Hotmail. http://www.hotmail.com ========================================================================= Instruções para entrar na lista, sair da lista e usar a lista em http://www.mat.puc-rio.br/~nicolau/olimp/obm-l.html O administrador desta lista é ========================================================================= From owner-obm-l@sucuri.mat.puc-rio.br Thu May 2 11:59:42 2002 Return-Path: Received: (from majordom@localhost) by sucuri.mat.puc-rio.br (8.9.3/8.9.3) id LAA15355 for obm-l-list; Thu, 2 May 2002 11:57:46 -0300 Received: from hotmail.com (law2-f45.hotmail.com [216.32.181.45]) by sucuri.mat.puc-rio.br (8.9.3/8.9.3) with ESMTP id LAA15348 for ; Thu, 2 May 2002 11:57:43 -0300 Received: from mail pickup service by hotmail.com with Microsoft SMTPSVC; Thu, 2 May 2002 07:58:13 -0700 Received: from 32.94.119.254 by lw2fd.hotmail.msn.com with HTTP; Thu, 02 May 2002 14:58:10 GMT X-Originating-IP: [32.94.119.254] From: "Paulo Santa Rita" To: obm-l@mat.puc-rio.br Subject: Re: [obm-l] Re: [obm-l] Equacao do Universo Date: Thu, 02 May 2002 14:58:10 +0000 Mime-Version: 1.0 Content-Type: text/plain; charset=iso-8859-1; format=flowed Message-ID: X-OriginalArrivalTime: 02 May 2002 14:58:13.0695 (UTC) FILETIME=[C8857CF0:01C1F1E9] Sender: owner-obm-l@sucuri.mat.puc-rio.br Precedence: bulk Reply-To: obm-l@mat.puc-rio.br Ola Ezer e demais colegas desta lista, As ( talvez mais famosas ) Equacoes da Fisica que tratam o Universo como um todo sao as equacoes da Teoria da Relatividade Geral. Esta teoria e uma aplicao da Teoria da Relatividade Restrita a movimentos dotados de aceleracao. Elas modificao os conceitos basicos Newtonianos, em especial o conceito de forca. Nesta teoria um corpo nao se movimenta porque uma forca atuou nele,mas sim em virtude das distorcoes no espaco que sao causadas pela presenca de um corpo dotado de massa ... Uma analogia talvez ajude. Se um corpo desce um plano inclinado, como voce explica tal movimento ? Bom, voce diz que a componente da forca peso paralela ao plano puxa o corpo para baixo. Einstein diria que ele desce porque a presenca de alguma massa curvou o espaco ( no caso, o plano )e corpo naturalmente seguiu o caminho da curvatura. Num espaco curvo os corpos seguem o caminho de menor consumo de energia, o que, geometricamente, equivale a certas curvas numa superficie que chamamos de geodesicas ou geodesias. Um a geodesia e o caminho mais curto entre dois pontos sobre uma superficie. No plano a geodesia e um segmento de reta. Na teoria de Einstein, diz-se que os planetas seguem as linhas geodesicas que existem na superficie do espaco-tempo curvo causado pela imensa massa do sol. Na teoria de Newton, os planetas tracam elipses. com o sol em um dos focos. Qual esta correta ? So a experiencia poderia dizer. E ela disse ... O perielio de Mercurio ( ponto mais proximo do sol ) avanca ( se inclina ) de uma quantidade nao prevista pela teoria Newtoniana, mesmo se considerando a pertubacao que os demais planetas produzem. Tal efeito e previsto pela teoria de Einstein. Assim, tudo leva a crer que as suposicoes de Einstein consistem numa visao mais correta sobre a real natureza do Universo. As Equacoes de Einstein sao Equacoes Diferenciais-Tensoriais, vale dizer, nao sao formulas como as da gravitacao newtoniana. Segue que existem infinitas possibilidades de solucoes. Uma solucao singular pode ser um buraco negro. Em verdade, os buracso negros so existem hoje como singulares das equacoes de Einstein. As singularidades, em geral, levam a implicacoes sensacionais. Voce ja ouviu falar em singularidades de Whitney ? Quantas solucoes tem a equacao z^7 - 5*(conj(z))^4 + z=0 ? Quando da comemoracao de um aniversario de Einstein, o Godel mostrou ( pro Einstein e pro resto do pessoal de Princeton ) uma solucao que incluia o movimento de rotacao dos planetas ( as outras solucoes nao tinham esta propriedade, vale dizer, nelas os planetas so transladam ) e o Einstein ficou, a principio, muito contente. Mas logo depois ficou chateado. Por que ? Porque na solucao mais completas de Godel deveriamos admitir que ha regioes no espaco em que o principio da causalidade e quebrado, isto e, locais onde o efeito ocorre antes da causa ... Ocorre uma coisa e a razao dela ocorrer esta no futuro ! Ora, se ocorrer um tal fenomeno, ele sera simplesmente inexplicavel ... Pois a explicacao de um fenomeno e absolutamente identico a demonstracao de um teorema : e como se existisse um teorema ou proposicao que nos verificamos que ela e correta mas jamais conseguimos demontra-la : VERDADES INDEMONSTRAVEIS. Interessante ( Sera que e so interessante ? ) que tambem foi o Godel que provou a incompletude dos sistemas formais ... Um abraco Paulo Santa Rita 5,1156,020502 >From: ghaeser@zipmail.com.br >Reply-To: obm-l@mat.puc-rio.br >To: obm-l@mat.puc-rio.br >Subject: [obm-l] Re: [obm-l] Equacao do Universo >Date: Wed, 1 May 2002 09:42:40 -0300 > >i^i=exp(i*logi)=exp(i*(ln|i|+iarg(i))=exp(i*(ln1+i*pi/2))= >=exp(i*(i*pi/2))= > >i^i=exp(-pi/2) > >se elevarmos a i novamente temos: >i^i^i=exp(i*(-pi/2))=cos(pi/2)-i*sen(pi/2)=-i > >quanto a outra pergunta .. nem imagino .. > >-- Mensagem original -- > > >Olah a todos, > > > >O que eh Equacao do Universo? (se eh que isso existe) > > > >Quanto vale i^i? (i = sqrt(-1)) > > > > > >Desde jah agradeco, > > > > > > > >Ezer F. da Silva > >========================================================================= > >Instruções para entrar na lista, sair da lista e usar a lista em > >http://www.mat.puc-rio.br/~nicolau/olimp/obm-l.html > >O administrador desta lista é > >========================================================================= > > > >"Mathematicus nascitur, non fit" >Matemáticos não são feitos, eles nascem >--------------------------------------- >Gabriel Haeser >www.gabas.cjb.net > > >------------------------------------------ >Use o melhor sistema de busca da Internet >Radar UOL - http://www.radaruol.com.br > > > >========================================================================= >Instruções para entrar na lista, sair da lista e usar a lista em >http://www.mat.puc-rio.br/~nicolau/olimp/obm-l.html >O administrador desta lista é >========================================================================= _________________________________________________________________ Una-se ao maior serviço de email do mundo: o MSN Hotmail. http://www.hotmail.com ========================================================================= Instruções para entrar na lista, sair da lista e usar a lista em http://www.mat.puc-rio.br/~nicolau/olimp/obm-l.html O administrador desta lista é ========================================================================= From owner-obm-l@sucuri.mat.puc-rio.br Thu May 2 17:14:15 2002 Return-Path: Received: (from majordom@localhost) by sucuri.mat.puc-rio.br (8.9.3/8.9.3) id RAA22111 for obm-l-list; Thu, 2 May 2002 17:13:13 -0300 Received: from smtp-4.ig.com.br (smtp-4.ig.com.br [200.226.132.153] (may be forged)) by sucuri.mat.puc-rio.br (8.9.3/8.9.3) with SMTP id RAA22107 for ; Thu, 2 May 2002 17:13:07 -0300 From: ezer@ig.com.br Received: (qmail 3022 invoked from network); 2 May 2002 20:13:23 -0000 Received: from shasta056187.ig.com.br (HELO house) (200.151.56.187) by smtp-4.ig.com.br with SMTP; 2 May 2002 20:13:23 -0000 To: obm-l@mat.puc-rio.br Date: Wed, 1 Jan 1997 01:20:09 -0200 MIME-Version: 1.0 Subject: [obm-l] questao - geom analitica Message-ID: <32C9BBC9.8689.482E4B@localhost> X-mailer: Pegasus Mail for Windows (v4.01) Content-type: text/plain; charset=ISO-8859-1 Content-description: Mail message body Content-Transfer-Encoding: 8bit X-MIME-Autoconverted: from Quoted-printable to 8bit by sucuri.mat.puc-rio.br id RAA22108 Sender: owner-obm-l@sucuri.mat.puc-rio.br Precedence: bulk Reply-To: obm-l@mat.puc-rio.br Olah amigos da lista, Eu gostaria que vcs vissem essa questao, para eu saber onde errei. Um triângulo tem seus vertices nos pontos A(p,q), B(2p,3q) e C(3p,2q). Em que pontos corta a ordenada e abscissa a reta que passa no baricentro deste triângulo e eh paralela ao lado BC? Eu achei como resposta os pontos (4p,0) e (0,4q), porem no gabarito estava (p,0) e (0,q). Desde jah agradeco, Ezer F. da Silva ========================================================================= Instruções para entrar na lista, sair da lista e usar a lista em http://www.mat.puc-rio.br/~nicolau/olimp/obm-l.html O administrador desta lista é ========================================================================= From owner-obm-l@sucuri.mat.puc-rio.br Thu May 2 18:33:06 2002 Return-Path: Received: (from majordom@localhost) by sucuri.mat.puc-rio.br (8.9.3/8.9.3) id SAA23411 for obm-l-list; Thu, 2 May 2002 18:32:46 -0300 Received: from hotmail.com (law2-f61.hotmail.com [216.32.181.61]) by sucuri.mat.puc-rio.br (8.9.3/8.9.3) with ESMTP id SAA23406 for ; Thu, 2 May 2002 18:32:43 -0300 Received: from mail pickup service by hotmail.com with Microsoft SMTPSVC; Thu, 2 May 2002 14:33:15 -0700 Received: from 32.94.119.254 by lw2fd.hotmail.msn.com with HTTP; Thu, 02 May 2002 21:33:15 GMT X-Originating-IP: [32.94.119.254] From: "Paulo Santa Rita" To: obm-l@mat.puc-rio.br Subject: Re: [obm-l] Flanders Olympiad novamente! Date: Thu, 02 May 2002 21:33:15 +0000 Mime-Version: 1.0 Content-Type: text/plain; charset=iso-8859-1; format=flowed Message-ID: X-OriginalArrivalTime: 02 May 2002 21:33:15.0539 (UTC) FILETIME=[F7EBDE30:01C1F220] Sender: owner-obm-l@sucuri.mat.puc-rio.br Precedence: bulk Reply-To: obm-l@mat.puc-rio.br Ola Ponce e demais colegas desta lista, E verdade. A Observacao do Colega Ponce e procedente. Olhando agora a mensagem original do Felipe vejo que havia um conjunto de respostas. Do enunciado nao se pode inferir quem e maior, P ou Q : dai as duas possiveis solucoes. Valeu Ponce ! Um abraco Paulo Santa Rita 5,1831,020502 >From: Luiz Antonio Ponce Alonso >Reply-To: obm-l@mat.puc-rio.br >To: obm-l@mat.puc-rio.br >Subject: Re: [obm-l] Flanders Olympiad novamente! >Date: Wed, 01 May 2002 22:17:58 -0300 > >caros amigos, uma pequena ressalva para o problema e solução dada. >O valor (-2(p+q))/pq é uma diferença possível para o problema. >Entretanto, 2(p+q))/pq, poderia ser também uma das diferenças entre dois >termos >consecutivos >da sequência, apesar de não constar do enunciado. >PONCE > >Paulo Santa Rita wrote: > > > Ola Felipe e demais > > colegas desta lista, > > > > A questao e muito simples e deve haver um montao de maneiras de faze-la. > > Tenho certeza que voce encontraria um caminho se pensasse um pouco mais >... > > Todavia, como voce parece estar aflito, atendendo seu apelo, uma forma > > talvez razoavel seja a seguinte : > > > > Sp=q e Sq=p > > (p(a1 + ap))/2 = q e (q(a1 + aq))/2 = p > > a1 + ap = (2q)/p e a1 + aq = (2p)/q > > > > subtraindo a segunda equacao da primeira : > > > > ap - aq = (2q)/p - (2p)/q > > mas : ap - aq = (p-q)R. Logo > > (p-q)R = (2q)/p - (2p)/q > > (p-q)R = (2(q-p)(q+p))/pq. Supondo p diferende de q > > > > R = (-2(p+q))/pq > > > > esse deve ser o valor que voce procura. Eu nao conferi. Da uma olhada. > > > > Um abraco > > Paulo Santa Rita > > 3,1821,300402 > > > > >From: "Felipe Marinho" > > >Reply-To: obm-l@mat.puc-rio.br > > >To: obm-l@mat.puc-rio.br > > >Subject: [obm-l] Flanders Olympiad novamente! > > >Date: Tue, 30 Apr 2002 16:24:17 -0400 > > > > > >Caros amigos da lista, > > > > > >Semana passada eu postei um questão aqui... pedindo uma ajuda de vocês >em > > >como resolvê-la. > > >Porem, não foi retorno por parte de ninguem e nenhum tipo de comentário > > >tambem foi feito. > > > > > >Por isso, venho aqui mais uma vez pedir a ajuda de vocês na resolução >da > > >mesmo. > > > > > >PS: Desculpe a insistência, mas estou realmente precisando de ajuda. > > > > > >Desculpe qualquer coisa, > > >E agradeço desde já qualquer tipo de ajuda. > > > > > >Segue a questão: > > > > > >1) A soma dos p primeiros termos de uma sequencia aritmética é igual a >q e > > >a > > >soma dos q primeiros termos dessa sequencia é dada por p, > > >onde (p diferente de q). Qual das opções expressa o valor da diferença >v > > >entre 2 termos sucessivos dessa sequencia é: > > > > > >a) v= -2/(p-q) > > >b) v= -2(p+q)/pq > > >c) v= 2/(p+q) > > >d) v= 2(p-q)/pq > > >e) v= 2(p+q) > > > > > >------ FIM ------- > > > > > >Abraços > > > > > >Felipe Marinho > > > > > >_________________________________________________________________ > > >Chegou o novo MSN Explorer. Instale já. É gratuito: > > >http://explorer.msn.com.br > > > > > > >========================================================================= > > >Instruções para entrar na lista, sair da lista e usar a lista em > > >http://www.mat.puc-rio.br/~nicolau/olimp/obm-l.html > > >O administrador desta lista é > > > >========================================================================= > > > > colegas desta lista, > > > > _________________________________________________________________ > > Chegou o novo MSN Explorer. Instale já. É gratuito: > > http://explorer.msn.com.br > > > > >========================================================================= > > Instruções para entrar na lista, sair da lista e usar a lista em > > http://www.mat.puc-rio.br/~nicolau/olimp/obm-l.html > > O administrador desta lista é > > >========================================================================= > > >========================================================================= >Instruções para entrar na lista, sair da lista e usar a lista em >http://www.mat.puc-rio.br/~nicolau/olimp/obm-l.html >O administrador desta lista é >========================================================================= _________________________________________________________________ Converse com amigos on-line, conheça o MSN Messenger: http://messenger.msn.com ========================================================================= Instruções para entrar na lista, sair da lista e usar a lista em http://www.mat.puc-rio.br/~nicolau/olimp/obm-l.html O administrador desta lista é ========================================================================= From owner-obm-l@sucuri.mat.puc-rio.br Fri May 3 00:45:28 2002 Return-Path: Received: (from majordom@localhost) by sucuri.mat.puc-rio.br (8.9.3/8.9.3) id AAA26043 for obm-l-list; Fri, 3 May 2002 00:45:11 -0300 Received: from web10107.mail.yahoo.com (web10107.mail.yahoo.com [216.136.130.57]) by sucuri.mat.puc-rio.br (8.9.3/8.9.3) with SMTP id AAA26039 for ; Fri, 3 May 2002 00:45:09 -0300 Message-ID: <20020503034542.91489.qmail@web10107.mail.yahoo.com> Received: from [150.161.199.28] by web10107.mail.yahoo.com via HTTP; Thu, 02 May 2002 20:45:42 PDT Date: Thu, 2 May 2002 20:45:42 -0700 (PDT) From: Rafael WC Subject: [obm-l] Complicada... To: OBM MIME-Version: 1.0 Content-Type: text/plain; charset=us-ascii Sender: owner-obm-l@sucuri.mat.puc-rio.br Precedence: bulk Reply-To: obm-l@mat.puc-rio.br Pessoal, já gastei muitas folhas com isso, não aguento mais. É muito braçal! Alguém tem alguma dica pra facilitar? Se a, b, c são números reais tais que (bc - a²)-¹ + (ca - b²)-¹ +(ab - c²)-¹ = 0 então: a(bc - a²)-² + b(ac - b²)-² + c(ab -c²)-² é igual a... Resp.: 0 Abraço, Rafael. ===== Rafael Werneck Cinoto ICQ# 107011599 rwcinoto@yahoo.com rafael.caixa@gov.com.br matduvidas@yahoo.com.br http://www.rwcinoto.hpg.com.br/ __________________________________________________ Do You Yahoo!? Yahoo! Health - your guide to health and wellness http://health.yahoo.com ========================================================================= Instruções para entrar na lista, sair da lista e usar a lista em http://www.mat.puc-rio.br/~nicolau/olimp/obm-l.html O administrador desta lista é ========================================================================= From owner-obm-l@sucuri.mat.puc-rio.br Fri May 3 01:13:44 2002 Return-Path: Received: (from majordom@localhost) by sucuri.mat.puc-rio.br (8.9.3/8.9.3) id BAA26499 for obm-l-list; Fri, 3 May 2002 01:13:40 -0300 Received: from hotmail.com (f146.pav2.hotmail.com [64.4.37.146]) by sucuri.mat.puc-rio.br (8.9.3/8.9.3) with ESMTP id BAA26485 for ; Fri, 3 May 2002 01:13:37 -0300 Received: from mail pickup service by hotmail.com with Microsoft SMTPSVC; Thu, 2 May 2002 21:14:09 -0700 Received: from 200.151.187.223 by pv2fd.pav2.hotmail.msn.com with HTTP; Fri, 03 May 2002 04:14:09 GMT X-Originating-IP: [200.151.187.223] From: "Fernanda Medeiros" To: obm-l@mat.puc-rio.br Subject: [obm-l] baralho Date: Fri, 03 May 2002 04:14:09 +0000 Mime-Version: 1.0 Content-Type: text/plain; charset=iso-8859-1; format=flowed Message-ID: X-OriginalArrivalTime: 03 May 2002 04:14:09.0454 (UTC) FILETIME=[F92C04E0:01C1F258] Sender: owner-obm-l@sucuri.mat.puc-rio.br Precedence: bulk Reply-To: obm-l@mat.puc-rio.br >Olá,gostaria de ajuda nesta questão: > >Temos um baralho especial de cartas. As cartas são numeradas e estão >colocadas por ordem.A nº 1 é a que está por baixo,a nº 2 está por cima da >nº >1,e assim sucessivamente.Carta de nº mais alto é a que está por cima.O >total de cartas é inferior a 5000.Toma-se a carta superior e coloca-se sob >o >baralho.Pega-se na seguinte,q sai fora do jogo.A nova carta superior do >baralho é colocada sob o baralho e a seguinte sai fora do jogo.E >continua-se >assim até q resta apenas uma carta,que por sinal, tem o nº 2001. Quantas >cartas tinha o baralho? >Obrigada! >Fê > > >_________________________________________________________________ >O MSN Photos é o modo mais fácil de compartilhar e imprimir suas fotos: >http://photos.msn.com/support/worldwide.aspx > >========================================================================= >Instruções para entrar na lista, sair da lista e usar a lista em >http://www.mat.puc-rio.br/~nicolau/olimp/obm-l.html >O administrador desta lista é >========================================================================= _________________________________________________________________ Chegou o novo MSN Explorer. Instale já. É gratuito: http://explorer.msn.com.br ========================================================================= Instruções para entrar na lista, sair da lista e usar a lista em http://www.mat.puc-rio.br/~nicolau/olimp/obm-l.html O administrador desta lista é ========================================================================= From owner-obm-l@sucuri.mat.puc-rio.br Fri May 3 11:08:31 2002 Return-Path: Received: (from majordom@localhost) by sucuri.mat.puc-rio.br (8.9.3/8.9.3) id LAA30858 for obm-l-list; Fri, 3 May 2002 11:06:34 -0300 Received: from mat.puc-rio.br (IDENT:root@perere.mat.puc-rio.br [139.82.27.60]) by sucuri.mat.puc-rio.br (8.9.3/8.9.3) with ESMTP id LAA30854 for ; Fri, 3 May 2002 11:06:32 -0300 Received: from localhost (fredpalm@localhost) by mat.puc-rio.br (8.9.3/8.9.3) with ESMTP id KAA03392 for ; Fri, 3 May 2002 10:24:31 -0300 Date: Fri, 3 May 2002 10:24:30 -0300 (BRT) From: Carlos Frederico Borges Palmeira To: obm-l@mat.puc-rio.br Subject: Re: [obm-l] questao - geom analitica In-Reply-To: <32C9BBC9.8689.482E4B@localhost> Message-ID: MIME-Version: 1.0 Content-Type: TEXT/PLAIN; charset=X-UNKNOWN Content-Transfer-Encoding: 8bit X-MIME-Autoconverted: from QUOTED-PRINTABLE to 8bit by sucuri.mat.puc-rio.br id LAA30855 Sender: owner-obm-l@sucuri.mat.puc-rio.br Precedence: bulk Reply-To: obm-l@mat.puc-rio.br oi eder, ou o gabarito esta' errado ou erramos igual. O que entendi do enunciado e' que e' para obter o baricentro, a equacao da reta paralela ao lado BC que passa pelo baricentro e achar os pontos onde esta reta corta os eixos. E tal como voce achei (4p,0) e (0,4q). Fred palmeira On Wed, 1 Jan 1997 ezer@ig.com.br wrote: > Olah amigos da lista, > > Eu gostaria que vcs vissem essa questao, > para eu saber onde errei. > > Um triângulo tem seus vertices nos pontos A(p,q), B(2p,3q) e C(3p,2q). > Em que pontos corta a ordenada e abscissa a reta que passa no baricentro deste > triângulo e eh paralela ao lado BC? > > Eu achei como resposta os pontos (4p,0) e (0,4q), porem no gabarito > estava (p,0) e (0,q). > > > Desde jah agradeco, > > Ezer F. da Silva > ========================================================================= > Instruções para entrar na lista, sair da lista e usar a lista em > http://www.mat.puc-rio.br/~nicolau/olimp/obm-l.html > O administrador desta lista é > ========================================================================= > ========================================================================= Instruções para entrar na lista, sair da lista e usar a lista em http://www.mat.puc-rio.br/~nicolau/olimp/obm-l.html O administrador desta lista é ========================================================================= From owner-obm-l@sucuri.mat.puc-rio.br Fri May 3 13:10:47 2002 Return-Path: Received: (from majordom@localhost) by sucuri.mat.puc-rio.br (8.9.3/8.9.3) id NAA32496 for obm-l-list; Fri, 3 May 2002 13:08:10 -0300 Received: from hotmail.com (law2-f89.hotmail.com [216.32.181.89]) by sucuri.mat.puc-rio.br (8.9.3/8.9.3) with ESMTP id NAA32492 for ; Fri, 3 May 2002 13:08:07 -0300 Received: from mail pickup service by hotmail.com with Microsoft SMTPSVC; Fri, 3 May 2002 09:08:37 -0700 Received: from 200.229.244.100 by lw2fd.hotmail.msn.com with HTTP; Fri, 03 May 2002 16:08:36 GMT X-Originating-IP: [200.229.244.100] From: "Paulo Santa Rita" To: obm-l@mat.puc-rio.br Subject: Re: [obm-l] baralho Date: Fri, 03 May 2002 16:08:36 +0000 Mime-Version: 1.0 Content-Type: text/plain; charset=iso-8859-1; format=flowed Message-ID: X-OriginalArrivalTime: 03 May 2002 16:08:37.0450 (UTC) FILETIME=[C87D1AA0:01C1F2BC] Sender: owner-obm-l@sucuri.mat.puc-rio.br Precedence: bulk Reply-To: obm-l@mat.puc-rio.br Ola Fernanda e demais colegas desta lista, Achei o problema legal. Vou pensar contigo sobre ele. Ve se voce descobre algo de util em minha reflexao. ABRE PARENTESES : Nao sei se e o seu caso, mas eu observo que muitas pessoas tem dificuldades em resolver alguns problemas nao pela dificuldade intrinseca que porventura os problemas tenham, mas, sobretudo, porque procuram enquadrar a questao que abordam em alguma teoria ou conjunto de tecnicas ja conhecidas. Isso nao me parece uma atitude saudavel ... Um problema e um problema ... A linguagem que nos vamos usar para resolve-lo e uma instancia da questao que se apresentara a posteriori, tao somente por ser a mais conveniente ou a unica que conhecemos naquele momento. Nos estudamos para compreender os objetos matematicos e suas relacoes mutuas, nao para compreender as tecnicas e as linguagens que sao os suportes desta compreensao. FECHA PARENTESES. 1) Na primeira operacao, a carta que esta no topo do monte sera conduzida a base ( base do monte ). Se N for o numero desta carta, olhando as cartas da base para o topo apos esta primeira operacao, veremos a sequencia : N, 1, 2, 3, ..., N-2, N-1 A carta de topo, agora, e N-1. Ela sera retirada. N-2 sera entao conduzida a base do monte. Olhando as cartas da base para topo apos a terceira operacao, veremos a sequencia : N-2, N, 1, 2, 3, ..., N-3 Claramente que N esta subindo e fatalmente chegara ao topo do monte apos um numero finito de operacoes... E igualmente evidente que se N for um numero impar(par), quando N chegar novamente ao topo do monte teremos retirado todas as cartas com numero par(impar) ... Portanto, quando N for, pela segunda vez, a carta do topo do monte, olhando as cartas da base para o topo, veremos a sequencia : (1, 3, 5, 7, ..., N) se N for impar ou (2, 4, 6, 8, ..., N) se N for par. Esta simples observacao torna evidente que a carta final tem a mesma paridade que N. Assim, sendo 2001 a carta final, concluimos que N deve ser, necessariamente, um numero impar. Seja, portanto, N impar. Bom, o processo continua ... Que operacao devera ser efetuada sobre a carta N, quando ela retornar ao topo ? Ela sera CONDUZIDA A BASE DO MONTE ou sera RETIRADA ? Toda carta que for conduzida a base do monte, evidentemente, retornara ao topo. Quando ela retornar, a operacao que sera efetuada sobre ela depende unica e exclusivamente da PARIDADE DO NUMERO DE CARTAS QUE HAVIA NO MONTE QUANDO ELA ESTEVE NO TOPO PELA ULTIMA VEZ. Para ver isso claramente, sejam : B -> Operacao de CONDUZIR A CARTA A BASE DO MONTE R -> Operacao de RETIRAR A CARTA DO MONTE No monte original, ocorrem as operacoes : B->N; R->N-1; B->N-2; ...; B->1(se N impar) ou R->1(se N par) Apos operar sobre 1 surge N no topo. Logo, sera aplicada : R->N(se N impar) ou B->N(se N par) Sera que fui claro ? Uma carta que em determinado momento do processo sair do topo do monte para a base do monte inevitavelmente retornara ao topo. Neste instante, ela sera reconduzida a base do monte se, e somente se, no vez anterior o numero de cartas do monte era par. Bom, voce deve estar se perguntando porque estou acompanhando com tanta meticulosidade as possiveis cartas que ficam no topo do monte, certo ? Ora, faco assim porque o que ha de mais obvio no problema, diria mesmo obvio ululante, e que o processo que voce descreve implica na retirada sucessiva de cartas, mas esta bendita, a carta 2001, passou incolume por todo este processo, terminando por ficar no topo do ultimo monte que e precisamente o monte com uma carta ! Dai eu concluo, imediatamente, que a carta 2001 esteve no topo de um monte com duas cartas, todas impares. Mas, duas cartas, nao podem formar uma sequencia de N cartas, comecando do 1 e indo ate, pelo menos, 2003. Logo, concluo que a carta 2001 este no topo de um monte de 4 carta, todas impares. Mas, quatro cartas, todas impares, nao podem formar uma sequencia de N cartas, comecando do 1 e indo ate, pelo menos, 2003. E assim sucessivamente... Este raciocinio vai permitir voce galgar montes cada vez mais altos ( com maior numero de cartas ). Mas ele deve parar em algum ponto... Em que ponto, ja que sabemos que temos menos de 5000 cartas ? Observe que quando a carta N atinge o topo do monte pela segunda vez, todos as cartas com numeros pares foram retiradas ... Ate onde eu posso dizer UM MONTE COM CARTAS TODAS IMPARES ? Bom, eu dei o passe. Cabe a voce, agora, fazer o gol ! Com os melhores votos de paz profunda, sou Paulo Santa Rita 6,1307,030502 >From: "Fernanda Medeiros" >Reply-To: obm-l@mat.puc-rio.br >To: obm-l@mat.puc-rio.br >Subject: [obm-l] baralho >Date: Fri, 03 May 2002 04:14:09 +0000 > > > >>Olá,gostaria de ajuda nesta questão: >> >>Temos um baralho especial de cartas. As cartas são numeradas e estão >>colocadas por ordem.A nº 1 é a que está por baixo,a nº 2 está por cima da >>nº >>1,e assim sucessivamente.Carta de nº mais alto é a que está por cima.O >>total de cartas é inferior a 5000.Toma-se a carta superior e coloca-se sob >>o >>baralho.Pega-se na seguinte,q sai fora do jogo.A nova carta superior do >>baralho é colocada sob o baralho e a seguinte sai fora do jogo.E >>continua-se >>assim até q resta apenas uma carta,que por sinal, tem o nº 2001. Quantas >>cartas tinha o baralho? >>Obrigada! >>Fê >> >> >>_________________________________________________________________ >>O MSN Photos é o modo mais fácil de compartilhar e imprimir suas fotos: >>http://photos.msn.com/support/worldwide.aspx >> >>========================================================================= >>Instruções para entrar na lista, sair da lista e usar a lista em >>http://www.mat.puc-rio.br/~nicolau/olimp/obm-l.html >>O administrador desta lista é >>========================================================================= > > >_________________________________________________________________ >Chegou o novo MSN Explorer. Instale já. É gratuito: >http://explorer.msn.com.br > >========================================================================= >Instruções para entrar na lista, sair da lista e usar a lista em >http://www.mat.puc-rio.br/~nicolau/olimp/obm-l.html >O administrador desta lista é >========================================================================= _________________________________________________________________ Envie e receba emails com o Hotmail no seu dispositivo móvel: http://mobile.msn.com ========================================================================= Instruções para entrar na lista, sair da lista e usar a lista em http://www.mat.puc-rio.br/~nicolau/olimp/obm-l.html O administrador desta lista é ========================================================================= From owner-obm-l@sucuri.mat.puc-rio.br Fri May 3 15:22:23 2002 Return-Path: Received: (from majordom@localhost) by sucuri.mat.puc-rio.br (8.9.3/8.9.3) id PAA02075 for obm-l-list; Fri, 3 May 2002 15:21:09 -0300 Received: from hotmail.com (f119.pav1.hotmail.com [64.4.31.119]) by sucuri.mat.puc-rio.br (8.9.3/8.9.3) with ESMTP id PAA02071 for ; Fri, 3 May 2002 15:21:06 -0300 Received: from mail pickup service by hotmail.com with Microsoft SMTPSVC; Fri, 3 May 2002 11:21:34 -0700 Received: from 200.242.50.4 by pv1fd.pav1.hotmail.msn.com with HTTP; Fri, 03 May 2002 18:21:30 GMT X-Originating-IP: [200.242.50.4] From: "Felipe Marinho" To: obm-l@mat.puc-rio.br Subject: Re: [obm-l] questao - geom analitica Date: Fri, 03 May 2002 14:21:30 -0400 Mime-Version: 1.0 Content-Type: text/plain; charset=iso-8859-1; format=flowed Message-ID: X-OriginalArrivalTime: 03 May 2002 18:21:34.0242 (UTC) FILETIME=[5B06F820:01C1F2CF] Sender: owner-obm-l@sucuri.mat.puc-rio.br Precedence: bulk Reply-To: obm-l@mat.puc-rio.br Caro Ezer, e demais amigos da lista. Vamos a resolução do problema: A coordenada da abcissa e da ordenada do Baricentro de um triangulo é dada por: Pb = [(x1+x2+x3)/3, (y1+y2+y3)/3] --> Onde Pb (Ponto do Baricentro) Com isso, temos que: Pb = [(p+2p+3p)/3, (q+3q+2q)/3] Pb = [(6p)/3, (6q)/3] Pb = (2p,2q) Achado a abcissa e a ordenada do ponto do Baricentro do triangulo, temos que analisar o coeficiente angular da reta BC. Tal coeficiente é dado por: m = (y3-y2)/(x3-x2) m = (2q - 3q)/(3p - 2p) m = (-q)/p m = -q/p Como a reta que vai passar pelo Baricentro é parelela a BC, temos que seu coeficiente angular será igual ao coeficente angular da reta BC, com isso, temos que -q/p é o coeficente da reta que passa por Pb (Ponto do Baricentro). Equação da reta que passa por Pb é: y - yo = m(x - xo) y - 2q = -q/p(x - 2p) Analisando a última equação , e igualando x=0, acharemos o ponto em que a reta corta o eixo das ordenadas, com isso, teremos: y - 2q = -q/p(0 - 2p) y - 2q = -q/p(-2p) y - 2q = 2q [ y = 4q ] (I) Agora, fazendo y=0, acharemos o ponto em que a reta corta o eixo das abcissas, com isso, teremos: 0 - 2q = -q/p(x - 2p) -2q = -q/p(x - 2p) 2q = q/p(x - 2p) 2pq/q = (x - 2p) 2p = x - 2p x = 4p (II) Em (I) e (II), temos os pontos que a reta que passa pelo Baricentro e é paralela a BC, corta os eixos das abcissas e das ordenadas. Resposta: (4p,0) e (0,4q). Caro Ezer, a resolução que encontrei foi esta. Espero ter ajudado e Espero não ter errado conta ;) E com um grande abraço a todos, vou fechando mais este e-mail. Felipe Marinho. >From: ezer@ig.com.br >Reply-To: obm-l@mat.puc-rio.br >To: obm-l@mat.puc-rio.br >Subject: [obm-l] questao - geom analitica >Date: Wed, 1 Jan 1997 01:20:09 -0200 > >Olah amigos da lista, > >Eu gostaria que vcs vissem essa questao, >para eu saber onde errei. > >Um triângulo tem seus vertices nos pontos A(p,q), B(2p,3q) e C(3p,2q). >Em que pontos corta a ordenada e abscissa a reta que passa no baricentro >deste >triângulo e eh paralela ao lado BC? > >Eu achei como resposta os pontos (4p,0) e (0,4q), porem no gabarito >estava (p,0) e (0,q). > > >Desde jah agradeco, > >Ezer F. da Silva >========================================================================= >Instruções para entrar na lista, sair da lista e usar a lista em >http://www.mat.puc-rio.br/~nicolau/olimp/obm-l.html >O administrador desta lista é >========================================================================= _________________________________________________________________ Una-se ao maior serviço de email do mundo: o MSN Hotmail. http://www.hotmail.com ========================================================================= Instruções para entrar na lista, sair da lista e usar a lista em http://www.mat.puc-rio.br/~nicolau/olimp/obm-l.html O administrador desta lista é ========================================================================= From owner-obm-l@sucuri.mat.puc-rio.br Fri May 3 15:32:08 2002 Return-Path: Received: (from majordom@localhost) by sucuri.mat.puc-rio.br (8.9.3/8.9.3) id PAA02256 for obm-l-list; Fri, 3 May 2002 15:31:59 -0300 Received: from shannon.bol.com.br (shannon.bol.com.br [200.221.24.13]) by sucuri.mat.puc-rio.br (8.9.3/8.9.3) with ESMTP id PAA02251 for ; Fri, 3 May 2002 15:31:36 -0300 Received: from bol.com.br (200.221.24.82) by shannon.bol.com.br (5.1.071) id 3C6F230B00FDD4C5 for obm-l@mat.puc-rio.br; Fri, 3 May 2002 15:31:10 -0300 Date: Fri, 3 May 2002 15:31:10 -0300 Message-Id: Subject: [obm-l] =?iso-8859-1?q?d=FAvidas?= MIME-Version: 1.0 Content-Type: text/plain;charset="iso-8859-1" From: "rafaelc.l" To: obm-l@mat.puc-rio.br X-XaM3-API-Version: 2.4.3.4.4 X-SenderIP: 200.176.166.238 Content-Transfer-Encoding: 8bit X-MIME-Autoconverted: from quoted-printable to 8bit by sucuri.mat.puc-rio.br id PAA02253 Sender: owner-obm-l@sucuri.mat.puc-rio.br Precedence: bulk Reply-To: obm-l@mat.puc-rio.br (IME-96)Determine o termo máximo do desenvolvimento da expressão: (1+1/3)^65 (IME-2001) a) Sejam x,y,z números reais positivos, prove que: x+y+z/3 maior ou igual a (x.y.z)^1/3 Em que condições a igualdade se verifica? b) Seja um paralelepípedo de lados a,b e c e área total S.Ache o volume máximo desse paralelepípedo em função de S. Qual a relação entre a, b e c para que esse volume seja máximo? demonstre seus resultados. OBS: ponham o maior número de soluções possíveis. Obrigado __________________________________________________________________________ Quer ter seu próprio endereço na Internet? Garanta já o seu e ainda ganhe cinco e-mails personalizados. DomíniosBOL - http://dominios.bol.com.br ========================================================================= Instruções para entrar na lista, sair da lista e usar a lista em http://www.mat.puc-rio.br/~nicolau/olimp/obm-l.html O administrador desta lista é ========================================================================= From owner-obm-l@sucuri.mat.puc-rio.br Fri May 3 22:28:00 2002 Return-Path: Received: (from majordom@localhost) by sucuri.mat.puc-rio.br (8.9.3/8.9.3) id WAA06538 for obm-l-list; Fri, 3 May 2002 22:27:33 -0300 Received: from imo-m03.mx.aol.com (imo-m03.mx.aol.com [64.12.136.6]) by sucuri.mat.puc-rio.br (8.9.3/8.9.3) with ESMTP id WAA06534 for ; Fri, 3 May 2002 22:27:31 -0300 From: DEOLIVEIRASOU@aol.com Received: from DEOLIVEIRASOU@aol.com by imo-m03.mx.aol.com (mail_out_v32.5.) id z.12f.10dce056 (4186) for ; Fri, 3 May 2002 21:27:59 -0400 (EDT) Message-ID: <12f.10dce056.2a04931e@aol.com> Date: Fri, 3 May 2002 21:27:58 EDT Subject: [obm-l] conferir.... To: obm-l@mat.puc-rio.br MIME-Version: 1.0 Content-Type: multipart/alternative; boundary="part1_12f.10dce056.2a04931e_boundary" X-Mailer: AOL 7.0 for Windows BR sub 10501 Sender: owner-obm-l@sucuri.mat.puc-rio.br Precedence: bulk Reply-To: obm-l@mat.puc-rio.br --part1_12f.10dce056.2a04931e_boundary Content-Type: text/plain; charset="ISO-8859-1" Content-Transfer-Encoding: quoted-printable 1)Determine o menor inteiro positivo a para o qual a equa=E7=E3o=20 1001x+770y=3D1000000+a possui solu=E7=E3o inteira e mostre que que h=E1 100=20= solu=E7=F5es=20 inteiras positivas. 2)Calcule o valor de1/1*2+1/2*3+....+1/(n-1)*n Valeu --part1_12f.10dce056.2a04931e_boundary Content-Type: text/html; charset="ISO-8859-1" Content-Transfer-Encoding: quoted-printable 1)Determine o menor inteiro positivo a para o qual a e= qua=E7=E3o 1001x+770y=3D1000000+a possui solu=E7=E3o inteira e mostre que qu= e h=E1 100 solu=E7=F5es inteiras positivas.
2)Calcule o valor de1/1*2+1/2*3+....+1/(n-1)*n
    Valeu
--part1_12f.10dce056.2a04931e_boundary-- ========================================================================= Instruções para entrar na lista, sair da lista e usar a lista em http://www.mat.puc-rio.br/~nicolau/olimp/obm-l.html O administrador desta lista é ========================================================================= From owner-obm-l@sucuri.mat.puc-rio.br Fri May 3 23:19:39 2002 Return-Path: Received: (from majordom@localhost) by sucuri.mat.puc-rio.br (8.9.3/8.9.3) id XAA07346 for obm-l-list; Fri, 3 May 2002 23:19:10 -0300 Received: from smtp-6.ig.com.br (smtp-6.ig.com.br [200.226.132.155] (may be forged)) by sucuri.mat.puc-rio.br (8.9.3/8.9.3) with SMTP id XAA07342 for ; Fri, 3 May 2002 23:19:08 -0300 Received: (qmail 14505 invoked from network); 4 May 2002 02:19:23 -0000 Received: from 157.68.226.200.in-addr.arpa.ig.com.br (HELO oemcomputer) (200.226.68.157) by smtp-6.ig.com.br with SMTP; 4 May 2002 02:19:23 -0000 Message-ID: <002801c1f314$04265100$9d44e2c8@oemcomputer> From: "Daniel" To: References: Subject: [obm-l] =?iso-8859-1?Q?Re:_=5Bobm-l=5D_d=FAvidas?= Date: Fri, 3 May 2002 23:32:59 -0300 MIME-Version: 1.0 Content-Type: text/plain; charset="iso-8859-1" Content-Transfer-Encoding: 8bit X-Priority: 3 X-MSMail-Priority: Normal X-Mailer: Microsoft Outlook Express 5.00.2615.200 X-MimeOLE: Produced By Microsoft MimeOLE V5.00.2615.200 Sender: owner-obm-l@sucuri.mat.puc-rio.br Precedence: bulk Reply-To: obm-l@mat.puc-rio.br (IME-2001) a) Sejam x,y,z números reais positivos, prove > que: > x+y+z/3 maior ou igual a (x.y.z)^1/3 > > Em que condições a igualdade se verifica? Ai vai a resposta do item a: É obvio que: x^3 + y^3 + z^3 -3xyz = (x+y+z)(x^2 + y^2 + z^2 -xy - xz - yz) e ainda o segundo termo da fatoração clássica pode ser escrito como: [(x - y)^2 + (x - z)^2 + (y - z)^2] / 2 Sendo x, y z reias positivos, agora fica fácil perceber que o primeiro termo da fatoração é positivo e o segundo também, logo o lado esquerdo da igualdade é maior que zero: x^3 + y^3 +z^3 -3xyz > 0 x^3 + y^3 + z^3 > 3xyz logo: (x + y +z)/3 > (xyz)^1/3 A igualde se verifica para x=y=z. Trata-se da desigualdade das médias para o caso particular de três termos. Se quiser a demonstração da fatoração acima é só pedir. Dica: tente completar um cubo perfeito que sai. Daniel O. Costa ========================================================================= Instruções para entrar na lista, sair da lista e usar a lista em http://www.mat.puc-rio.br/~nicolau/olimp/obm-l.html O administrador desta lista é ========================================================================= From owner-obm-l@sucuri.mat.puc-rio.br Sat May 4 00:54:11 2002 Return-Path: Received: (from majordom@localhost) by sucuri.mat.puc-rio.br (8.9.3/8.9.3) id AAA08286 for obm-l-list; Sat, 4 May 2002 00:53:40 -0300 Received: from hotmail.com (f77.sea1.hotmail.com [207.68.163.77]) by sucuri.mat.puc-rio.br (8.9.3/8.9.3) with ESMTP id AAA08282 for ; Sat, 4 May 2002 00:53:38 -0300 Received: from mail pickup service by hotmail.com with Microsoft SMTPSVC; Fri, 3 May 2002 20:54:13 -0700 Received: from 200.227.149.164 by sea1fd.sea1.hotmail.msn.com with HTTP; Sat, 04 May 2002 03:54:13 GMT X-Originating-IP: [200.227.149.164] From: "Frederico Reis Marques de Brito" To: obm-l@mat.puc-rio.br Subject: Re: [obm-l] conferir.... Date: Sat, 04 May 2002 00:54:13 -0300 Mime-Version: 1.0 Content-Type: text/plain; charset=iso-8859-1; format=flowed Message-ID: X-OriginalArrivalTime: 04 May 2002 03:54:13.0946 (UTC) FILETIME=[5B01B9A0:01C1F31F] Sender: owner-obm-l@sucuri.mat.puc-rio.br Precedence: bulk Reply-To: obm-l@mat.puc-rio.br 1) uma solução não muito técnica é a seguinte: Observe que mdc(1001, 770) = 77 , pois 1001 = 7 x 11 x 13 e 770 = 2 x 5 x 7 x 11. Daí, a equação admite soluções inteiras se, e somente se ( 1000000 + a ) for múltiplo de 77, isto é, de 11 e 7 simultaneamente. Desde que 1000000 = 12987 x 77 + 1 , resulta que o próximo múltiplo de 77 é 1000076. Portanto, o valor mínimo de a é 76. 2) De imediato, não consegui nenhuma solução. >From: DEOLIVEIRASOU@aol.com >Reply-To: obm-l@mat.puc-rio.br >To: obm-l@mat.puc-rio.br >Subject: [obm-l] conferir.... >Date: Fri, 3 May 2002 21:27:58 EDT > >1)Determine o menor inteiro positivo a para o qual a equação >1001x+770y=1000000+a possui solução inteira e mostre que que há 100 >soluções >inteiras positivas. >2)Calcule o valor de 1/1*2+1/2*3+....+1/(n-1)*n > Valeu _________________________________________________________________ Una-se ao maior serviço de email do mundo: o MSN Hotmail. http://www.hotmail.com ========================================================================= Instruções para entrar na lista, sair da lista e usar a lista em http://www.mat.puc-rio.br/~nicolau/olimp/obm-l.html O administrador desta lista é ========================================================================= From owner-obm-l@sucuri.mat.puc-rio.br Sat May 4 09:06:08 2002 Return-Path: Received: (from majordom@localhost) by sucuri.mat.puc-rio.br (8.9.3/8.9.3) id JAA11654 for obm-l-list; Sat, 4 May 2002 09:05:52 -0300 Received: from hotmail.com (f233.law9.hotmail.com [64.4.9.233]) by sucuri.mat.puc-rio.br (8.9.3/8.9.3) with ESMTP id JAA11650 for ; Sat, 4 May 2002 09:05:50 -0300 Received: from mail pickup service by hotmail.com with Microsoft SMTPSVC; Sat, 4 May 2002 05:06:26 -0700 Received: from 200.151.153.4 by lw9fd.law9.hotmail.msn.com with HTTP; Sat, 04 May 2002 12:06:26 GMT X-Originating-IP: [200.151.153.4] From: "Henrique Lima Santana" To: obm-l@mat.puc-rio.br Subject: Re: [obm-l] conferir.... Date: Sat, 04 May 2002 12:06:26 +0000 Mime-Version: 1.0 Content-Type: text/plain; format=flowed Message-ID: X-OriginalArrivalTime: 04 May 2002 12:06:26.0855 (UTC) FILETIME=[1DFE0370:01C1F364] Sender: owner-obm-l@sucuri.mat.puc-rio.br Precedence: bulk Reply-To: obm-l@mat.puc-rio.br E ae Crom,blz? 2.Note q 1/x(x+1)=1/x -1/(x+1) dae chamando de E a expressão abaixo temos: E=1 -1/2 +1/2 -1/3 +...+1/(n-1) -1/n = 1-1/n= (n-1)/n Falou []´s Henrique >From: DEOLIVEIRASOU@aol.com >Reply-To: obm-l@mat.puc-rio.br >To: obm-l@mat.puc-rio.br >Subject: [obm-l] conferir.... >Date: Fri, 3 May 2002 21:27:58 EDT > >1)Determine o menor inteiro positivo a para o qual a equação >1001x+770y=1000000+a possui solução inteira e mostre que que há 100 >soluções >inteiras positivas. >2)Calcule o valor de1/1*2+1/2*3+....+1/(n-1)*n > Valeu _________________________________________________________________ Get your FREE download of MSN Explorer at http://explorer.msn.com/intl.asp. ========================================================================= Instruções para entrar na lista, sair da lista e usar a lista em http://www.mat.puc-rio.br/~nicolau/olimp/obm-l.html O administrador desta lista é ========================================================================= From owner-obm-l@sucuri.mat.puc-rio.br Sat May 4 14:23:52 2002 Return-Path: Received: (from majordom@localhost) by sucuri.mat.puc-rio.br (8.9.3/8.9.3) id OAA14154 for obm-l-list; Sat, 4 May 2002 14:23:25 -0300 Received: from www.zipmail.com.br (smtp.zipmail.com.br [200.187.242.10]) by sucuri.mat.puc-rio.br (8.9.3/8.9.3) with ESMTP id OAA14150 for ; Sat, 4 May 2002 14:23:23 -0300 From: peterdirichlet@zipmail.com.br Received: from [200.144.43.149] by www.zipmail.com.br with HTTP; Sat, 4 May 2002 14:22:19 -0300 Message-ID: <3CD27704000001AC@www.zipmail.com.br> Date: Sat, 4 May 2002 14:22:19 -0300 In-Reply-To: <12f.10dce056.2a04931e@aol.com> Subject: [obm-l] =?iso-8859-1?Q?Re=3A=20=5Bobm=2Dl=5D=20conferir=2E=2E=2E=2E?= To: obm-l@mat.puc-rio.br MIME-Version: 1.0 Content-Type: text/plain; charset="iso-8859-1" Content-Transfer-Encoding: 8bit X-MIME-Autoconverted: from quoted-printable to 8bit by sucuri.mat.puc-rio.br id OAA14151 Sender: owner-obm-l@sucuri.mat.puc-rio.br Precedence: bulk Reply-To: obm-l@mat.puc-rio.br ANSWER:1)tente ver divisibilidade por 77 e desigualdades elementares. -- Mensagem original -- >1)Determine o menor inteiro positivo a para o qual a equação >1001x+770y=1000000+a possui solução inteira e mostre que que há 100 soluções > >inteiras positivas. >2)Calcule o valor de 1/1*2+1/2*3+....+1/(n-1)*n > Valeu > ------------------------------------------ Use o melhor sistema de busca da Internet Radar UOL - http://www.radaruol.com.br ========================================================================= Instruções para entrar na lista, sair da lista e usar a lista em http://www.mat.puc-rio.br/~nicolau/olimp/obm-l.html O administrador desta lista é ========================================================================= From owner-obm-l@sucuri.mat.puc-rio.br Sat May 4 14:33:56 2002 Return-Path: Received: (from majordom@localhost) by sucuri.mat.puc-rio.br (8.9.3/8.9.3) id OAA14311 for obm-l-list; Sat, 4 May 2002 14:33:50 -0300 Received: from shannon.bol.com.br (shannon.bol.com.br [200.221.24.13]) by sucuri.mat.puc-rio.br (8.9.3/8.9.3) with ESMTP id OAA14307 for ; Sat, 4 May 2002 14:33:48 -0300 Received: from bol.com.br (200.221.24.79) by shannon.bol.com.br (5.1.071) id 3C6F230B010167A7 for obm-l@mat.puc-rio.br; Sat, 4 May 2002 14:33:36 -0300 Date: Sat, 4 May 2002 14:33:36 -0300 Message-Id: Subject: [obm-l] =?iso-8859-1?q?Re=3A=5Bobm=2Dl=5D_Re=3A_=5Bobm=2Dl=5D_d=FAvidas?= MIME-Version: 1.0 Content-Type: text/plain;charset="iso-8859-1" From: "rafaelc.l" To: obm-l@mat.puc-rio.br X-XaM3-API-Version: 2.4.3.4.4 X-SenderIP: 200.176.166.64 Content-Transfer-Encoding: 8bit X-MIME-Autoconverted: from quoted-printable to 8bit by sucuri.mat.puc-rio.br id OAA14308 Sender: owner-obm-l@sucuri.mat.puc-rio.br Precedence: bulk Reply-To: obm-l@mat.puc-rio.br valeu Daniel. Mas de onde vc tira essas fatorações? isso eu nunca aprendi, aliás, esse tipo de questão abrange álgebra básica, que não tem em nenhum livro..não é? __________________________________________________________________________ Quer ter seu próprio endereço na Internet? Garanta já o seu e ainda ganhe cinco e-mails personalizados. DomíniosBOL - http://dominios.bol.com.br ========================================================================= Instruções para entrar na lista, sair da lista e usar a lista em http://www.mat.puc-rio.br/~nicolau/olimp/obm-l.html O administrador desta lista é ========================================================================= From owner-obm-l@sucuri.mat.puc-rio.br Sat May 4 15:53:38 2002 Return-Path: Received: (from majordom@localhost) by sucuri.mat.puc-rio.br (8.9.3/8.9.3) id PAA15492 for obm-l-list; Sat, 4 May 2002 15:53:10 -0300 Received: from hotmail.com (f122.pav1.hotmail.com [64.4.31.122]) by sucuri.mat.puc-rio.br (8.9.3/8.9.3) with ESMTP id PAA15488 for ; Sat, 4 May 2002 15:53:07 -0300 Received: from mail pickup service by hotmail.com with Microsoft SMTPSVC; Sat, 4 May 2002 11:53:45 -0700 Received: from 200.151.50.96 by pv1fd.pav1.hotmail.msn.com with HTTP; Sat, 04 May 2002 18:53:45 GMT X-Originating-IP: [200.151.50.96] From: "Adriano Almeida Faustino" To: obm-l@mat.puc-rio.br Subject: [obm-l] =?iso-8859-1?B?UmU6IFtvYm0tbF0gZPp2aWRhcw==?= Date: Sat, 04 May 2002 18:53:45 +0000 Mime-Version: 1.0 Content-Type: text/plain; charset=iso-8859-1; format=flowed Message-ID: X-OriginalArrivalTime: 04 May 2002 18:53:45.0373 (UTC) FILETIME=[047BECD0:01C1F39D] Sender: owner-obm-l@sucuri.mat.puc-rio.br Precedence: bulk Reply-To: obm-l@mat.puc-rio.br (IME-96) sendo T(k+1) o termo de ordem (k+1) T(k+1) = C (65,k) * (1/3)^k * 1^(65-k) = 1/3^k * C(65,k) se T (k+1) é o maior : T (k+1) >= T(k) 1/3^k * C(65,k) >= 1/3(k-1) * C(65,(k-1)) efetuando as contas : 1/3k >= 1/(66-k) k =< 66/4 = 16,5 e também : T(k+1) >= T(k+2) 1/3^k * C(65,k) >= 1/3^(k+1) * C(65,(k+1)) 1/(65-k) >= 1/(3(k+1)) k >= 62/4 =15,5 15,5 =< k =< 16,5 -> k=16 , já que k é inteiro T(16+1) =T(17) = C(65,16)/3^16 Adriano. >From: "rafaelc.l" >Reply-To: obm-l@mat.puc-rio.br >To: obm-l@mat.puc-rio.br >Subject: [obm-l] dúvidas >Date: Fri, 3 May 2002 15:31:10 -0300 > > >(IME-96)Determine o termo máximo do desenvolvimento da >expressão: > (1+1/3)^65 > > > >(IME-2001) a) Sejam x,y,z números reais positivos, prove >que: > x+y+z/3 maior ou igual a (x.y.z)^1/3 > >Em que condições a igualdade se verifica? > > b) Seja um paralelepípedo de lados a,b e c e área total >S.Ache o volume máximo desse paralelepípedo em função de >S. Qual a relação entre a, b e c para que esse volume >seja máximo? demonstre seus resultados. > > >OBS: ponham o maior número de soluções possíveis. > > > Obrigado > > >__________________________________________________________________________ >Quer ter seu próprio endereço na Internet? >Garanta já o seu e ainda ganhe cinco e-mails personalizados. >DomíniosBOL - http://dominios.bol.com.br > > >========================================================================= >Instruções para entrar na lista, sair da lista e usar a lista em >http://www.mat.puc-rio.br/~nicolau/olimp/obm-l.html >O administrador desta lista é >========================================================================= _________________________________________________________________ Una-se ao maior serviço de email do mundo: o MSN Hotmail. http://www.hotmail.com ========================================================================= Instruções para entrar na lista, sair da lista e usar a lista em http://www.mat.puc-rio.br/~nicolau/olimp/obm-l.html O administrador desta lista é ========================================================================= From owner-obm-l@sucuri.mat.puc-rio.br Sat May 4 21:30:10 2002 Return-Path: Received: (from majordom@localhost) by sucuri.mat.puc-rio.br (8.9.3/8.9.3) id VAA17846 for obm-l-list; Sat, 4 May 2002 21:28:24 -0300 Received: from shrek2.uol.com.br (shrek2.uol.com.br [200.221.4.15]) by sucuri.mat.puc-rio.br (8.9.3/8.9.3) with ESMTP id VAA17842 for ; Sat, 4 May 2002 21:28:22 -0300 Received: from ginsberg.uol.com.br (172.26.5.9) by shrek2.uol.com.br (5.1.071) id 3CD431BA0002B93D for obm-l@mat.puc-rio.br; Sat, 4 May 2002 21:36:02 -0300 Received: from cui.uol.com.br ([200.160.246.82]) by ginsberg.uol.com.br (8.9.1/8.9.1) with ESMTP id VAA06526 for ; Sat, 4 May 2002 21:27:53 -0300 (BRT) Message-Id: <5.1.0.14.2.20020504212545.01d59f70@pop3.uol.com.br> X-Sender: fnicks@uol.com.br@pop3.uol.com.br (Unverified) X-Mailer: QUALCOMM Windows Eudora Version 5.1 Date: Sat, 04 May 2002 21:28:13 -0300 To: obm-l@mat.puc-rio.br From: Nicks Subject: [obm-l] SAC Mime-Version: 1.0 Content-Type: text/plain; charset="iso-8859-1"; format=flowed Content-Transfer-Encoding: 8bit X-MIME-Autoconverted: from quoted-printable to 8bit by sucuri.mat.puc-rio.br id VAA17843 Sender: owner-obm-l@sucuri.mat.puc-rio.br Precedence: bulk Reply-To: obm-l@mat.puc-rio.br Olá pessoal , Alguém poderia me dizer o que significa Sistema de Amortização Constante para um empréstimo ? Agradeço desde já Nick ========================================================================= Instruções para entrar na lista, sair da lista e usar a lista em http://www.mat.puc-rio.br/~nicolau/olimp/obm-l.html O administrador desta lista é ========================================================================= From owner-obm-l@sucuri.mat.puc-rio.br Sat May 4 22:29:14 2002 Return-Path: Received: (from majordom@localhost) by sucuri.mat.puc-rio.br (8.9.3/8.9.3) id WAA18519 for obm-l-list; Sat, 4 May 2002 22:28:04 -0300 Received: from panther.unisys.com.br (panther.unisys.com.br [200.220.64.10]) by sucuri.mat.puc-rio.br (8.9.3/8.9.3) with ESMTP id WAA18515 for ; Sat, 4 May 2002 22:28:01 -0300 Received: from itacon (riopm04p10.uninet.com.br [200.220.2.42]) by panther.unisys.com.br (8.12.2/8.12.2) with SMTP id g451SRrL002037 for ; Sat, 4 May 2002 22:28:33 -0300 (EST) X-Spam-Filter: check_local@panther.unisys.com.br by digitalanswers.org Message-ID: <000801c1f3d4$0e5d5ae0$2a02dcc8@itacon> From: "Jose Francisco Guimaraes Costa" To: References: <5.1.0.14.2.20020504212545.01d59f70@pop3.uol.com.br> Subject: Re: [obm-l] SAC Date: Sat, 4 May 2002 22:27:29 -0300 MIME-Version: 1.0 Content-Type: text/plain; charset="iso-8859-1" Content-Transfer-Encoding: 8bit X-Priority: 3 X-MSMail-Priority: Normal X-Mailer: Microsoft Outlook Express 5.00.2314.1300 X-MimeOLE: Produced By Microsoft MimeOLE V5.00.2314.1300 Sender: owner-obm-l@sucuri.mat.puc-rio.br Precedence: bulk Reply-To: obm-l@mat.puc-rio.br Quando V faz um empréstimo V se compromete a pagar a quantia que foi emprestada (capital) acrescida de juros. Dessa forma, as parcelas do pagamento são compostas de duas partes: uma amortiza, isto é, paga, o capital que foi emprestado, e a outra paga os juros. A forma mais usual de pagamento é utilizando a Tabela Price, onde o valor das parcelas é constante, de forma que os componentes de cada parcela - capital e juros - variam ao longo do tempo. A outra forma de pagamento é o SAC (Sistema de Amortização Constante), onde a parte de amortização do capital é a mesma em todas as parcelas, variando a parte correspondente aos juros, fazendo com que o valor total das parcelas diminua com o tempo. O SAC é utilizado aqui no Brasil para pagamento de hipotecas, que são empréstimos com prazos de pagamento muito longos. O problema nosso é que existe correção monetária do capital (hoje disfarçada sob o nome de TR), com critérios diferentes para o reajuste do saldo devedor e do valor da prestação, fazendo com que, ao se contratar um empréstimo, não se tenha a menor idéia do que vai ser o valor da prestação uns poucos anos depois. Alem disso, ao se pagar a última parcela em geral ainda existe um saldo devedor - capital não amortizado - impossível de ser pago. Se V vai comprar um imóvel com financiamento da Caixa Econômica - que é o único agente que está fazendo empréstimos para compra de imóveis - faça simulações muito realistas para ver se vai poder pagá-lo. Em geral não poderá! JF -----Mensagem Original----- De: Nicks Para: Enviada em: Sábado, 4 de Maio de 2002 21:28 Assunto: [obm-l] SAC > Olá pessoal , > > Alguém poderia me dizer o que significa Sistema de Amortização > Constante para um empréstimo ? > > Agradeço desde já > > Nick > > > ========================================================================= > Instruções para entrar na lista, sair da lista e usar a lista em > http://www.mat.puc-rio.br/~nicolau/olimp/obm-l.html > O administrador desta lista é > ========================================================================= > ========================================================================= Instruções para entrar na lista, sair da lista e usar a lista em http://www.mat.puc-rio.br/~nicolau/olimp/obm-l.html O administrador desta lista é ========================================================================= From owner-obm-l@sucuri.mat.puc-rio.br Sat May 4 22:48:24 2002 Return-Path: Received: (from majordom@localhost) by sucuri.mat.puc-rio.br (8.9.3/8.9.3) id WAA18873 for obm-l-list; Sat, 4 May 2002 22:46:56 -0300 Received: from hotmail.com (f36.pav1.hotmail.com [64.4.31.36]) by sucuri.mat.puc-rio.br (8.9.3/8.9.3) with ESMTP id WAA18869 for ; Sat, 4 May 2002 22:46:54 -0300 Received: from mail pickup service by hotmail.com with Microsoft SMTPSVC; Sat, 4 May 2002 18:47:32 -0700 Received: from 200.151.30.56 by pv1fd.pav1.hotmail.msn.com with HTTP; Sun, 05 May 2002 01:47:32 GMT X-Originating-IP: [200.151.30.56] From: "Adriano Almeida Faustino" To: obm-l@mat.puc-rio.br Subject: [obm-l] =?iso-8859-1?B?UmU6IFtvYm0tbF0gZPp2aWRhcw==?= Date: Sun, 05 May 2002 01:47:32 +0000 Mime-Version: 1.0 Content-Type: text/plain; charset=iso-8859-1; format=flowed Message-ID: X-OriginalArrivalTime: 05 May 2002 01:47:32.0727 (UTC) FILETIME=[D2BD8070:01C1F3D6] Sender: owner-obm-l@sucuri.mat.puc-rio.br Precedence: bulk Reply-To: obm-l@mat.puc-rio.br (IME-2001) a)sendo a e b reais positivos temos que : (a^(1/2)-b(^1/2))^2 >= 0 temos que a+b >= 2(ab)^(1/2) fazendo a = (c+d)/2 , b= (e+f)/2 (c+d+e+f)/4 >= ( (c+d)/2 *(e+f)/2 )^(1/2) utilizando a desigualdade entre as médias provada no começo: (c+d+f+e)/4 >= (cdef)^(1/4) fazendo um termo qualquer como a média arit. dos outros temos que: ( (c+d+e)+ (c+d+e)/3 )/4 >= ( cde(c+d+e)/3 )~(1/4) (c+d+e)/3 >= ( cde(c+d+e)/3 )^(1/4) elevando a quarta pot. e simplificando nos dois lados temos: (c+d+e)/3 >= (cde)^(1/3) b)comoS = 2(ab+bc+ac), fazendo x=ab ,y=ac, z=bc temos: (ab+ac+bc)/3 >= ( (ab)(ac)(bc) )^(1/3) S/6 >= ( (abc)^2) ^(1/3) S/6 >= ( V^2 ) ^(1/3) V =< S/6 ( (S/6)^(1/2) ) então o valor máximo de V é igual a S/6 ( (S/6) ^(1/2) ) o que implica que a média arit. é igual a média geom. então a=b=c ou seja o paral. é um cubo. Adriano >From: "rafaelc.l" >Reply-To: obm-l@mat.puc-rio.br >To: obm-l@mat.puc-rio.br >Subject: [obm-l] dúvidas >Date: Fri, 3 May 2002 15:31:10 -0300 > > >(IME-96)Determine o termo máximo do desenvolvimento da >expressão: > (1+1/3)^65 > > > >(IME-2001) a) Sejam x,y,z números reais positivos, prove >que: > x+y+z/3 maior ou igual a (x.y.z)^1/3 > >Em que condições a igualdade se verifica? > > b) Seja um paralelepípedo de lados a,b e c e área total >S.Ache o volume máximo desse paralelepípedo em função de >S. Qual a relação entre a, b e c para que esse volume >seja máximo? demonstre seus resultados. > > >OBS: ponham o maior número de soluções possíveis. > > > Obrigado > > >__________________________________________________________________________ >Quer ter seu próprio endereço na Internet? >Garanta já o seu e ainda ganhe cinco e-mails personalizados. >DomíniosBOL - http://dominios.bol.com.br > > >========================================================================= >Instruções para entrar na lista, sair da lista e usar a lista em >http://www.mat.puc-rio.br/~nicolau/olimp/obm-l.html >O administrador desta lista é >========================================================================= _________________________________________________________________ Envie e receba emails com o Hotmail no seu dispositivo móvel: http://mobile.msn.com ========================================================================= Instruções para entrar na lista, sair da lista e usar a lista em http://www.mat.puc-rio.br/~nicolau/olimp/obm-l.html O administrador desta lista é ========================================================================= From owner-obm-l@sucuri.mat.puc-rio.br Sun May 5 09:11:36 2002 Return-Path: Received: (from majordom@localhost) by sucuri.mat.puc-rio.br (8.9.3/8.9.3) id JAA24550 for obm-l-list; Sun, 5 May 2002 09:10:05 -0300 Received: from shrek3.uol.com.br (shrek3.uol.com.br [200.221.4.18]) by sucuri.mat.puc-rio.br (8.9.3/8.9.3) with ESMTP id JAA24546 for ; Sun, 5 May 2002 09:10:03 -0300 Received: from traven.uol.com.br (172.26.5.16) by shrek3.uol.com.br (5.1.071) id 3CD316B6000E205B for obm-l@mat.puc-rio.br; Sun, 5 May 2002 09:10:16 -0300 Received: from cui.uol.com.br ([200.160.246.230]) by traven.uol.com.br (8.9.1/8.9.1) with ESMTP id IAA02900; Sun, 5 May 2002 08:59:50 -0300 (BRT) Message-Id: <5.1.0.14.2.20020505090418.01c02ec0@pop3.uol.com.br> X-Sender: fnicks@uol.com.br@pop3.uol.com.br X-Mailer: QUALCOMM Windows Eudora Version 5.1 Date: Sun, 05 May 2002 09:10:35 -0300 To: obm-l@mat.puc-rio.br, From: Nicks Subject: Re: [obm-l] SAC In-Reply-To: <000801c1f3d4$0e5d5ae0$2a02dcc8@itacon> References: <5.1.0.14.2.20020504212545.01d59f70@pop3.uol.com.br> Mime-Version: 1.0 Content-Type: text/plain; charset="iso-8859-1"; format=flowed Content-Transfer-Encoding: 8bit X-MIME-Autoconverted: from quoted-printable to 8bit by sucuri.mat.puc-rio.br id JAA24547 Sender: owner-obm-l@sucuri.mat.puc-rio.br Precedence: bulk Reply-To: obm-l@mat.puc-rio.br Obrigado pela resposta , Mais precisamente a questão é a seguinte : Num empréstimo de 300.000 reais que será pago em 3 anos com juros de 10% ao ano pelo SAC ( em que a primeira parcela será paga daqui um ano) ; qual o juros pago e qual o valor da terceira prestação ? []´s Nick At 22:27 4/5/2002 -0300, Jose Francisco Guimaraes Costa wrote: >Quando V faz um empréstimo V se compromete a pagar a quantia que foi >emprestada (capital) acrescida de juros. Dessa forma, as parcelas do >pagamento são compostas de duas partes: uma amortiza, isto é, paga, o >capital que foi emprestado, e a outra paga os juros. A forma mais usual de >pagamento é utilizando a Tabela Price, onde o valor das parcelas é >constante, de forma que os componentes de cada parcela - capital e juros - >variam ao longo do tempo. A outra forma de pagamento é o SAC (Sistema de >Amortização Constante), onde a parte de amortização do capital é a mesma em >todas as parcelas, variando a parte correspondente aos juros, fazendo com >que o valor total das parcelas diminua com o tempo. > >O SAC é utilizado aqui no Brasil para pagamento de hipotecas, que são >empréstimos com prazos de pagamento muito longos. O problema nosso é que >existe correção monetária do capital (hoje disfarçada sob o nome de TR), com >critérios diferentes para o reajuste do saldo devedor e do valor da >prestação, fazendo com que, ao se contratar um empréstimo, não se tenha a >menor idéia do que vai ser o valor da prestação uns poucos anos depois. Alem >disso, ao se pagar a última parcela em geral ainda existe um saldo devedor - >capital não amortizado - impossível de ser pago. > >Se V vai comprar um imóvel com financiamento da Caixa Econômica - que é o >único agente que está fazendo empréstimos para compra de imóveis - faça >simulações muito realistas para ver se vai poder pagá-lo. Em geral não >poderá! > >JF > > >-----Mensagem Original----- >De: Nicks >Para: >Enviada em: Sábado, 4 de Maio de 2002 21:28 >Assunto: [obm-l] SAC > > > > Olá pessoal , > > > > Alguém poderia me dizer o que significa Sistema de Amortização > > Constante para um empréstimo ? > > > > Agradeço desde já > > > > Nick > > > > > > ========================================================================= > > Instruções para entrar na lista, sair da lista e usar a lista em > > http://www.mat.puc-rio.br/~nicolau/olimp/obm-l.html > > O administrador desta lista é > > ========================================================================= > > > >========================================================================= >Instruções para entrar na lista, sair da lista e usar a lista em >http://www.mat.puc-rio.br/~nicolau/olimp/obm-l.html >O administrador desta lista é >========================================================================= ========================================================================= Instruções para entrar na lista, sair da lista e usar a lista em http://www.mat.puc-rio.br/~nicolau/olimp/obm-l.html O administrador desta lista é ========================================================================= From owner-obm-l@sucuri.mat.puc-rio.br Sun May 5 09:11:36 2002 Return-Path: Received: (from majordom@localhost) by sucuri.mat.puc-rio.br (8.9.3/8.9.3) id JAA24550 for obm-l-list; Sun, 5 May 2002 09:10:05 -0300 Received: from shrek3.uol.com.br (shrek3.uol.com.br [200.221.4.18]) by sucuri.mat.puc-rio.br (8.9.3/8.9.3) with ESMTP id JAA24546 for ; Sun, 5 May 2002 09:10:03 -0300 Received: from traven.uol.com.br (172.26.5.16) by shrek3.uol.com.br (5.1.071) id 3CD316B6000E205B for obm-l@mat.puc-rio.br; Sun, 5 May 2002 09:10:16 -0300 Received: from cui.uol.com.br ([200.160.246.230]) by traven.uol.com.br (8.9.1/8.9.1) with ESMTP id IAA02900; Sun, 5 May 2002 08:59:50 -0300 (BRT) Message-Id: <5.1.0.14.2.20020505090418.01c02ec0@pop3.uol.com.br> X-Sender: fnicks@uol.com.br@pop3.uol.com.br X-Mailer: QUALCOMM Windows Eudora Version 5.1 Date: Sun, 05 May 2002 09:10:35 -0300 To: obm-l@mat.puc-rio.br, From: Nicks Subject: Re: [obm-l] SAC In-Reply-To: <000801c1f3d4$0e5d5ae0$2a02dcc8@itacon> References: <5.1.0.14.2.20020504212545.01d59f70@pop3.uol.com.br> Mime-Version: 1.0 Content-Type: text/plain; charset="iso-8859-1"; format=flowed Content-Transfer-Encoding: 8bit X-MIME-Autoconverted: from quoted-printable to 8bit by sucuri.mat.puc-rio.br id JAA24547 Sender: owner-obm-l@sucuri.mat.puc-rio.br Precedence: bulk Reply-To: obm-l@mat.puc-rio.br Obrigado pela resposta , Mais precisamente a questão é a seguinte : Num empréstimo de 300.000 reais que será pago em 3 anos com juros de 10% ao ano pelo SAC ( em que a primeira parcela será paga daqui um ano) ; qual o juros pago e qual o valor da terceira prestação ? []´s Nick At 22:27 4/5/2002 -0300, Jose Francisco Guimaraes Costa wrote: >Quando V faz um empréstimo V se compromete a pagar a quantia que foi >emprestada (capital) acrescida de juros. Dessa forma, as parcelas do >pagamento são compostas de duas partes: uma amortiza, isto é, paga, o >capital que foi emprestado, e a outra paga os juros. A forma mais usual de >pagamento é utilizando a Tabela Price, onde o valor das parcelas é >constante, de forma que os componentes de cada parcela - capital e juros - >variam ao longo do tempo. A outra forma de pagamento é o SAC (Sistema de >Amortização Constante), onde a parte de amortização do capital é a mesma em >todas as parcelas, variando a parte correspondente aos juros, fazendo com >que o valor total das parcelas diminua com o tempo. > >O SAC é utilizado aqui no Brasil para pagamento de hipotecas, que são >empréstimos com prazos de pagamento muito longos. O problema nosso é que >existe correção monetária do capital (hoje disfarçada sob o nome de TR), com >critérios diferentes para o reajuste do saldo devedor e do valor da >prestação, fazendo com que, ao se contratar um empréstimo, não se tenha a >menor idéia do que vai ser o valor da prestação uns poucos anos depois. Alem >disso, ao se pagar a última parcela em geral ainda existe um saldo devedor - >capital não amortizado - impossível de ser pago. > >Se V vai comprar um imóvel com financiamento da Caixa Econômica - que é o >único agente que está fazendo empréstimos para compra de imóveis - faça >simulações muito realistas para ver se vai poder pagá-lo. Em geral não >poderá! > >JF > > >-----Mensagem Original----- >De: Nicks >Para: >Enviada em: Sábado, 4 de Maio de 2002 21:28 >Assunto: [obm-l] SAC > > > > Olá pessoal , > > > > Alguém poderia me dizer o que significa Sistema de Amortização > > Constante para um empréstimo ? > > > > Agradeço desde já > > > > Nick > > > > > > ========================================================================= > > Instruções para entrar na lista, sair da lista e usar a lista em > > http://www.mat.puc-rio.br/~nicolau/olimp/obm-l.html > > O administrador desta lista é > > ========================================================================= > > > >========================================================================= >Instruções para entrar na lista, sair da lista e usar a lista em >http://www.mat.puc-rio.br/~nicolau/olimp/obm-l.html >O administrador desta lista é >========================================================================= ========================================================================= Instruções para entrar na lista, sair da lista e usar a lista em http://www.mat.puc-rio.br/~nicolau/olimp/obm-l.html O administrador desta lista é ========================================================================= From owner-obm-l@sucuri.mat.puc-rio.br Sun May 5 13:31:18 2002 Return-Path: Received: (from majordom@localhost) by sucuri.mat.puc-rio.br (8.9.3/8.9.3) id NAA26005 for obm-l-list; Sun, 5 May 2002 13:28:46 -0300 Received: from hotmail.com (f92.sea1.hotmail.com [207.68.163.92]) by sucuri.mat.puc-rio.br (8.9.3/8.9.3) with ESMTP id NAA26001 for ; Sun, 5 May 2002 13:28:43 -0300 Received: from mail pickup service by hotmail.com with Microsoft SMTPSVC; Sun, 5 May 2002 09:29:23 -0700 Received: from 200.227.148.103 by sea1fd.sea1.hotmail.msn.com with HTTP; Sun, 05 May 2002 16:29:22 GMT X-Originating-IP: [200.227.148.103] From: "Frederico Reis Marques de Brito" To: obm-l@mat.puc-rio.br Subject: [obm-l] =?iso-8859-1?B?UmU6IFtvYm0tbF0gUmU6IFtvYm0tbF0gUmU6IFtvYm0tbF0gbvptZXJv?= =?iso-8859-1?B?cw==?= Date: Sun, 05 May 2002 13:29:22 -0300 Mime-Version: 1.0 Content-Type: text/plain; charset=iso-8859-1; format=flowed Message-ID: X-OriginalArrivalTime: 05 May 2002 16:29:23.0199 (UTC) FILETIME=[03D6D0F0:01C1F452] Sender: owner-obm-l@sucuri.mat.puc-rio.br Precedence: bulk Reply-To: obm-l@mat.puc-rio.br É. Vc tem razão , na construção aprersentada por Peano para os naturais, o 1o número natural é chamado de 1, mas bem poderia ser chamado de 0 , serm perda de coerência da axiomática. Esta é uma construção possível, majoritariamente usada pelos analistas. Quando se trata de usar o conjunto IN algebriocamente, faz falta relevante o elemento neutro da soma, sem o qual IN não teria nenhuma estrutura alg´ébrica. Daí, em alguns casops considera-se 0 como natural. Mas tudo se resume a um problema de convençao. >From: "adr.scr.m" >Reply-To: obm-l@mat.puc-rio.br >To: obm-l@mat.puc-rio.br >Subject: [obm-l] Re: [obm-l] Re: [obm-l] números >Date: Sun, 28 Apr 2002 00:27:22 -0300 > > > mas e o axioma de Peano que diz que 1 é o número >natural que não tem antecessor ? >Adriano. > > > > ---------- > > >From: "Nicolau C. Saldanha" rio.br> > > >To: obm-l@mat.puc-rio.br > > >Subject: [obm-l] Re: [obm-l] números > > >Date: Sat, Apr 27, 2002, 3:24 PM > > > > > > > > On Sat, Apr 27, 2002 at 02:38:19PM -0300, adr.scr.m >wrote: > > >> Tenho algumas dúvidas sobre certos números que > > >> normalmente ninguém sabe responder e ficam >enrolando: > > >> 1)O nº 0 é natural ? > > > > > > Isto é puramente uma questão de definição/linguagem. > > > A definição que me parece ser a mais útil e talvez a >mais comum > > > é incluir 0 entre os naturais. Se você desejar >evitar ambiguidades > > > fale do conjunto dos inteiros positivos ou dos >inteiros não negativos. > > >> > > Os livros didaticos brasileiros incluem o 0 no >conjunto dos naturais. > > Mas nao sao coerentes com essa definicao. Quando >abordam as progressoes, > > aritmetica e geometrica, falam de uma sequencia cujo >termo geral eh > > a_n onde n eh natural. Mas sempre comecam a sequencia >por a_1. > > Como disse o Nicolau, isto eh apenas uma questao de >definicao. > > A duvida dos alunos se deve ao fato que os livros >consideram os > > naturais ora com o zero, ora sem o zero. > > > > >> 2)vi no livro "curso de análise vol.1" do Elon Lages > > >> Lima pág.164 falando que os números +infinito e > > >> -infinito não são reais.Eu queria saber o que que >eles > > >> são então ? > > > > > > De fato, +infinito e -infinito não são números reais. > > > Para algumas perguntas é entretanto conveniente >aumentar > > > o conjunto dos reais com estes dois objetos. > > > > > > []s, N. > > > >========================================================= >================ > > > Instruções para entrar na lista, sair da lista e >usar a lista em > > > http://www.mat.puc-rio.br/~nicolau/olimp/obm-l.html > > > O administrador desta lista é rio.br> > > > >========================================================= >================ > > >========================================================= >================ > > Instruções para entrar na lista, sair da lista e usar >a lista em > > http://www.mat.puc-rio.br/~nicolau/olimp/obm-l.html > > O administrador desta lista é > > >========================================================= >================ > > > > >__________________________________________________________________________ >Quer ter seu próprio endereço na Internet? >Garanta já o seu e ainda ganhe cinco e-mails personalizados. >DomíniosBOL - http://dominios.bol.com.br > > >========================================================================= >Instruções para entrar na lista, sair da lista e usar a lista em >http://www.mat.puc-rio.br/~nicolau/olimp/obm-l.html >O administrador desta lista é >========================================================================= _________________________________________________________________ Converse com amigos on-line, conheça o MSN Messenger: http://messenger.msn.com ========================================================================= Instruções para entrar na lista, sair da lista e usar a lista em http://www.mat.puc-rio.br/~nicolau/olimp/obm-l.html O administrador desta lista é ========================================================================= From owner-obm-l@sucuri.mat.puc-rio.br Sun May 5 16:34:51 2002 Return-Path: Received: (from majordom@localhost) by sucuri.mat.puc-rio.br (8.9.3/8.9.3) id QAA27262 for obm-l-list; Sun, 5 May 2002 16:32:47 -0300 Received: from web13709.mail.yahoo.com (web13709.mail.yahoo.com [216.136.175.251]) by sucuri.mat.puc-rio.br (8.9.3/8.9.3) with SMTP id QAA27258 for ; Sun, 5 May 2002 16:32:45 -0300 Message-ID: <20020505193325.28097.qmail@web13709.mail.yahoo.com> Received: from [200.213.88.186] by web13709.mail.yahoo.com via HTTP; Sun, 05 May 2002 16:33:25 ART Date: Sun, 5 May 2002 16:33:25 -0300 (ART) From: =?iso-8859-1?q?pichurin?= Subject: [obm-l] sebos To: obm-l@mat.puc-rio.br MIME-Version: 1.0 Content-Type: text/plain; charset=iso-8859-1 Content-Transfer-Encoding: 8bit Sender: owner-obm-l@sucuri.mat.puc-rio.br Precedence: bulk Reply-To: obm-l@mat.puc-rio.br Por favor, qual é o sebo mais próximo da região de Bauru?Alguém conhece algum? _______________________________________________________________________ Yahoo! Encontros O lugar certo para você encontrar aquela pessoa que falta na sua vida. Cadastre-se hoje mesmo! http://br.encontros.yahoo.com/ ========================================================================= Instruções para entrar na lista, sair da lista e usar a lista em http://www.mat.puc-rio.br/~nicolau/olimp/obm-l.html O administrador desta lista é ========================================================================= From owner-obm-l@sucuri.mat.puc-rio.br Sun May 5 17:16:39 2002 Return-Path: Received: (from majordom@localhost) by sucuri.mat.puc-rio.br (8.9.3/8.9.3) id RAA27653 for obm-l-list; Sun, 5 May 2002 17:15:08 -0300 Received: from shannon.bol.com.br (shannon.bol.com.br [200.221.24.13]) by sucuri.mat.puc-rio.br (8.9.3/8.9.3) with ESMTP id RAA27649 for ; Sun, 5 May 2002 17:15:06 -0300 Received: from bol.com.br (200.221.24.68) by shannon.bol.com.br (5.1.071) id 3C6F230B0104BE7A for obm-l@mat.puc-rio.br; Sun, 5 May 2002 17:15:02 -0300 Date: Sun, 5 May 2002 17:15:03 -0300 Message-Id: Subject: [obm-l] =?iso-8859-1?q?Re=3A=5Bobm=2Dl=5D_Re=3A_=5Bobm=2Dl=5D_d=FAvidas?= MIME-Version: 1.0 Content-Type: text/plain;charset="iso-8859-1" From: "rafaelc.l" To: obm-l@mat.puc-rio.br X-XaM3-API-Version: 2.4.3.4.4 X-SenderIP: 200.176.167.31 Content-Transfer-Encoding: 8bit X-MIME-Autoconverted: from quoted-printable to 8bit by sucuri.mat.puc-rio.br id RAA27650 Sender: owner-obm-l@sucuri.mat.puc-rio.br Precedence: bulk Reply-To: obm-l@mat.puc-rio.br Interessante vc ter se baseado na letra A para fazer a letra B..não é a toa que elas estão na mesma questão. Não sei se vc concorda comigo, mas essa questão foi pra mim a mais difícil da prova e esse tipo de questão que chamam de "álgebra especulativa" eu acho realmente mais complicado que o resto em matemática. Vc concorda? __________________________________________________________________________ Quer ter seu próprio endereço na Internet? Garanta já o seu e ainda ganhe cinco e-mails personalizados. DomíniosBOL - http://dominios.bol.com.br ========================================================================= Instruções para entrar na lista, sair da lista e usar a lista em http://www.mat.puc-rio.br/~nicolau/olimp/obm-l.html O administrador desta lista é ========================================================================= From owner-obm-l@sucuri.mat.puc-rio.br Sun May 5 22:50:22 2002 Return-Path: Received: (from majordom@localhost) by sucuri.mat.puc-rio.br (8.9.3/8.9.3) id WAA30023 for obm-l-list; Sun, 5 May 2002 22:47:45 -0300 Received: from smtp-5.ig.com.br (smtp-5.ig.com.br [200.226.132.154] (may be forged)) by sucuri.mat.puc-rio.br (8.9.3/8.9.3) with SMTP id WAA30019 for ; Sun, 5 May 2002 22:47:43 -0300 Received: (qmail 20758 invoked from network); 6 May 2002 01:48:10 -0000 Received: from 21.206.226.200.in-addr.arpa.ig.com.br (HELO oemcomputer) (200.226.206.21) by smtp-5.ig.com.br with SMTP; 6 May 2002 01:48:10 -0000 Message-ID: <006501c1f4a2$027e5e80$42e7e2c8@oemcomputer> From: "Daniel" To: Subject: [obm-l] =?iso-8859-1?B?RmF0b3Jh5+NvIENs4XNzaWNh?= Date: Sun, 5 May 2002 22:58:57 -0300 MIME-Version: 1.0 Content-Type: multipart/alternative; boundary="----=_NextPart_000_0061_01C1F488.6FD145C0" X-Priority: 3 X-MSMail-Priority: Normal X-Mailer: Microsoft Outlook Express 5.00.2615.200 X-MIMEOLE: Produced By Microsoft MimeOLE V5.00.2615.200 Sender: owner-obm-l@sucuri.mat.puc-rio.br Precedence: bulk Reply-To: obm-l@mat.puc-rio.br This is a multi-part message in MIME format. ------=_NextPart_000_0061_01C1F488.6FD145C0 Content-Type: text/plain; charset="iso-8859-1" Content-Transfer-Encoding: quoted-printable Atendendo a dois pedidos vamos l=E1: Tese: x^3 + y^3 + z^3 - 3xyz =3D (x + y + z)(x^2 + y^2 + z^2 - xy = - xz - yz) Demonstra=E7=E3o: Completando e tirando um cubo perfeito em x e y, e usando o lado = esquerdo da tese, temos: x^3 + 3x^2 y + 3x y^2 + y^3 - 3x^2 - 3xy^2 - 3xyz + z^3 (x + y)^3 + z^3 - 3xy(x + y +z) (x+ y +z) [ (x + y)^2 - (x + y)z + z^2 - 3xy] (x + y + z)(x^2 + y^2 + 2xy - xz -yz + c^2 - 3xy) (x + y + z)(x^2 + y^2 + z^2 -xy -xz -yz ) CQD Espero que ajude Daniel ------=_NextPart_000_0061_01C1F488.6FD145C0 Content-Type: text/html; charset="iso-8859-1" Content-Transfer-Encoding: quoted-printable

       =20     Atendendo a dois pedidos vamos l=E1:
 
Tese:
       =20     x^3 + y^3 + z^3 - 3xyz =3D (x + y + z)(x^2 + y^2 + = z^2 - xy -=20 xz - yz)
 
Demonstra=E7=E3o:
 
Completando e tirando um cubo perfeito = em x e y, e=20 usando o lado esquerdo da tese, temos:
 
 x^3 + 3x^2 y + 3x y^2 + y^3 - = 3x^2 - 3xy^2 -=20 3xyz + z^3
 
(x + y)^3 + z^3 - 3xy(x + y = +z)
 
(x+ y +z) [ (x + y)^2 - (x + y)z + z^2 = -=20 3xy]
 
(x + y + z)(x^2 + y^2 + 2xy - xz -yz + = c^2 -=20 3xy)
 
(x + y + z)(x^2 + y^2 + z^2 -xy -xz -yz = )
 
       =20         CQD
Espero que ajude
 
       =20             =    =20         Daniel
------=_NextPart_000_0061_01C1F488.6FD145C0-- ========================================================================= Instruções para entrar na lista, sair da lista e usar a lista em http://www.mat.puc-rio.br/~nicolau/olimp/obm-l.html O administrador desta lista é ========================================================================= From owner-obm-l@sucuri.mat.puc-rio.br Mon May 6 01:39:26 2002 Return-Path: Received: (from majordom@localhost) by sucuri.mat.puc-rio.br (8.9.3/8.9.3) id BAA31107 for obm-l-list; Mon, 6 May 2002 01:37:23 -0300 Received: from shrek3.uol.com.br (shrek3.uol.com.br [200.221.4.18]) by sucuri.mat.puc-rio.br (8.9.3/8.9.3) with ESMTP id BAA31103 for ; Mon, 6 May 2002 01:37:21 -0300 Received: from ginsberg.uol.com.br (172.26.5.9) by shrek3.uol.com.br (5.1.071) id 3CD316B600155945 for obm-l@mat.puc-rio.br; Mon, 6 May 2002 01:37:36 -0300 Received: from computador ([200.213.211.193]) by ginsberg.uol.com.br (8.9.1/8.9.1) with SMTP id BAA28098 for ; Mon, 6 May 2002 01:37:04 -0300 (BRT) From: =?iso-8859-1?Q?Moacyr_Rodrigues_J=FAnior?= To: Subject: RES: [obm-l] Me pegou!!! Ajuda Date: Mon, 6 May 2002 01:37:51 -0300 Message-ID: MIME-Version: 1.0 Content-Type: multipart/alternative; boundary="----=_NextPart_000_000B_01C1F49E.A2E373F0" X-Priority: 3 (Normal) X-MSMail-Priority: Normal X-Mailer: Microsoft Outlook IMO, Build 9.0.2416 (9.0.2910.0) X-MimeOLE: Produced By Microsoft MimeOLE V6.00.2600.0000 Importance: Normal In-Reply-To: <86.19f95e64.2a014393@aol.com> Sender: owner-obm-l@sucuri.mat.puc-rio.br Precedence: bulk Reply-To: obm-l@mat.puc-rio.br This is a multi-part message in MIME format. ------=_NextPart_000_000B_01C1F49E.A2E373F0 Content-Type: text/plain; charset="iso-8859-1" Content-Transfer-Encoding: 8bit Se a rifa custou 300 reais e o prêmio pago foi de 300 000 reais então o lucro líquido foi de 299 700 reais ou seja 99,90% do total do prêmio. Coube a Maria 39,96% e portanto sobrou a André e João a diferença, que é de 59,94% do total do prêmio. Como André recebeu o dobro de João, este receberá apenas um terço daquilo 19,98% do total do prêmio. Veja que 19,98% corresponde a um quinto de 99,90%, logo como a compra foi em parceria João deve ter pago um quinto do valor do bilhete, ou seja 60 reais. Abraço a todos. Moacyr Rodrigues Junior -----Mensagem original----- De: owner-obm-l@sucuri.mat.puc-rio.br [mailto:owner-obm-l@sucuri.mat.puc-rio.br]Em nome de Lltmdrtm@aol.com Enviada em: quarta-feira, 1 de maio de 2002 10:12 Para: obm-l@mat.puc-rio.br Assunto: [obm-l] Me pegou!!! Ajuda João, Maria e André compraram em parceria um bilhete de rifa por 300 reais, que sorteado pagou um prêmio de 300.000 reais. No rateio do prêmio coube a André o dobro da quantia paga a João. O lucro líquido de Maria correspondeu a 39,96% do total do prêmio. Calcule a quantia, em reais, paga por João, na compra do bilhete. ------=_NextPart_000_000B_01C1F49E.A2E373F0 Content-Type: text/html; charset="iso-8859-1" Content-Transfer-Encoding: quoted-printable

Se a rifa custou 300 reais e o pr=EAmio pago foi de = 300 000 reais ent=E3o o lucro l=EDquido foi de 299 700 reais ou seja 99,90% do total = do pr=EAmio.

 

Coube a Maria 39,96% e portanto sobrou a Andr=E9 e Jo=E3o a diferen=E7a, que =E9 = de 59,94% do total do pr=EAmio.

 

Como Andr=E9 recebeu o dobro de Jo=E3o, este receber=E1 apenas um ter=E7o = daquilo 19,98% do total do pr=EAmio. Veja que 19,98% corresponde a um quinto de 99,90%, = logo como a compra foi em parceria Jo=E3o deve ter pago um quinto do valor do = bilhete, ou seja 60 reais.

 

Abra=E7o a todos.

Moacyr Rodrigues Junior

 

-----Mensagem original-----
De: owner-obm-l@sucuri.mat.puc-rio.br = [mailto:owner-obm-l@sucuri.mat.puc-rio.br]Em nome de Lltmdrtm@aol.com
Enviada em: quarta-feira, = 1 de maio de 2002 10:12
Para: = obm-l@mat.puc-rio.br
Assunto: [obm-l] Me = pegou!!! Ajuda

 


Jo=E3o, Maria e Andr=E9 compraram em parceria um bilhete de rifa por 300 = reais, que sorteado pagou um pr=EAmio de 300.000 reais. No rateio do pr=EAmio coube = a Andr=E9 o dobro da quantia paga a Jo=E3o. O lucro l=EDquido de Maria correspondeu = a 39,96% do total do pr=EAmio. Calcule a quantia, em reais, paga por Jo=E3o, na = compra do bilhete.

------=_NextPart_000_000B_01C1F49E.A2E373F0-- ========================================================================= Instruções para entrar na lista, sair da lista e usar a lista em http://www.mat.puc-rio.br/~nicolau/olimp/obm-l.html O administrador desta lista é ========================================================================= From owner-obm-l@sucuri.mat.puc-rio.br Mon May 6 13:44:03 2002 Return-Path: Received: (from majordom@localhost) by sucuri.mat.puc-rio.br (8.9.3/8.9.3) id NAA05718 for obm-l-list; Mon, 6 May 2002 13:41:03 -0300 Received: from hotmail.com (f80.pav1.hotmail.com [64.4.31.80]) by sucuri.mat.puc-rio.br (8.9.3/8.9.3) with ESMTP id NAA05714 for ; Mon, 6 May 2002 13:41:00 -0300 Received: from mail pickup service by hotmail.com with Microsoft SMTPSVC; Mon, 6 May 2002 09:41:43 -0700 Received: from 200.151.212.28 by pv1fd.pav1.hotmail.msn.com with HTTP; Mon, 06 May 2002 16:41:43 GMT X-Originating-IP: [200.151.212.28] From: "Adherbal Rocha Filho" To: obm-l@mat.puc-rio.br Subject: [obm-l] ajuda importante Date: Mon, 06 May 2002 16:41:43 +0000 Mime-Version: 1.0 Content-Type: text/plain; charset=iso-8859-1; format=flowed Message-ID: X-OriginalArrivalTime: 06 May 2002 16:41:43.0611 (UTC) FILETIME=[E79278B0:01C1F51C] Sender: owner-obm-l@sucuri.mat.puc-rio.br Precedence: bulk Reply-To: obm-l@mat.puc-rio.br Gostaria muito de ajuda nestas questões: 1.Como o médico me recomendou caminhadas,todo dia de manhã dou uma volta(com velocidde constante) na quadra em que resido.Minha mulher aproveita pra correr (com velocidade constante) em volta do quarteirão.Saímos juntos e chegamos juntos.Ela percorre a quadra no mesmo sentido que eu e me ultrapassa 2 vezes durante o percurso.Se ela corresse no sentido contrário ao meu,quantas vezes ela cruzaria comigo? 2.Um gramado tem a forma de um quadrado de lado 10m.Uma corda tem um dos extremos fixado em um dos vértices, e no outro lado extremo está amarrado um bode.Se o bode consegue comer metade da grama,então o comprimento da corda é aproximadamente qnt? 3.Um ladrilho em forma de um polígono regular ,foi retirado do lugar que ocupava em um painel.Observou-se então que esse ladrilho,se sofresse uma rotação de 40º ou de 60º em torno do seu centro,poderia ser encaixado perfeitamente no lugar que ficou vago no painel.O menor nº de lados que pode ter esse ladrilho é? Muito grato por qualquer ajuda! []´s Adherbal _________________________________________________________________ O MSN Photos é o modo mais fácil de compartilhar e imprimir suas fotos: http://photos.msn.com/support/worldwide.aspx ========================================================================= Instruções para entrar na lista, sair da lista e usar a lista em http://www.mat.puc-rio.br/~nicolau/olimp/obm-l.html O administrador desta lista é ========================================================================= From owner-obm-l@sucuri.mat.puc-rio.br Mon May 6 14:03:24 2002 Return-Path: Received: (from majordom@localhost) by sucuri.mat.puc-rio.br (8.9.3/8.9.3) id OAA06094 for obm-l-list; Mon, 6 May 2002 14:02:58 -0300 Received: from hotmail.com (law2-f15.hotmail.com [216.32.181.15]) by sucuri.mat.puc-rio.br (8.9.3/8.9.3) with ESMTP id OAA06090 for ; Mon, 6 May 2002 14:02:55 -0300 Received: from mail pickup service by hotmail.com with Microsoft SMTPSVC; Mon, 6 May 2002 10:03:35 -0700 Received: from 32.94.119.254 by lw2fd.hotmail.msn.com with HTTP; Mon, 06 May 2002 17:03:34 GMT X-Originating-IP: [32.94.119.254] From: "Paulo Santa Rita" To: obm-l@mat.puc-rio.br Subject: Re: [obm-l] Re: [obm-l] conferir.... Date: Mon, 06 May 2002 17:03:34 +0000 Mime-Version: 1.0 Content-Type: text/plain; charset=iso-8859-1; format=flowed Message-ID: X-OriginalArrivalTime: 06 May 2002 17:03:35.0062 (UTC) FILETIME=[F5420360:01C1F51F] Sender: owner-obm-l@sucuri.mat.puc-rio.br Precedence: bulk Reply-To: obm-l@mat.puc-rio.br Ola Pessoal, 1) A primeira Equacao Diofantina que se estuda e : Ax + By = C E existe um teorema que afirma que uma tal equacao so tem solucao se, e somente se, MDC(A,B) divide C. ( MDC(A,B) | C ). No seu problema, abaixo, devemos ter : MDC(1001,770) | 1000000 + a Como o lado esquerdo e conhecido, fica facil encontrar o "a" do lado direito. 2) Se A1, A2, A3, ..., An e uma PA entao : 1/A1*A2 = (1/R)*(1/A1 - 1/A2) 1/A2*A3 = (1/R)*(1/A2 - 1/A3) ... 1/An-1*An = (1/R)*(1/An-1 - 1/An) somando e simplificando : 1/A1*A2 + 1/A2*A3 + ... + 1/An-1*An = (1/R)*(1/A1 - 1/An) Esta expressao nao so nos permite obter uma formula "fechadinha" para a soma de N termos como calcular para onde tende a soma quando N -> +INF. Em verdade, esta expressao e apenas um caso particular de algo muito mais amplo ... Para ver isso, a titulo de exemplificacao, pergunto : como fariamos para calcular : 1/A1*A2*A3 + 1/A2*A3*A4 + ... + 1/An-2*An-1*An ? Aqui observe que : 1/A1*A2*A3 = (1/(2*(R^2))) * (1/A1 - 2/A2 + 1/A3) 1/A2*A3*A4 = (1/(2*(R^2))) * (1/A2 - 2/A3 + 1/A4) 1/A3*A4*A5 = (1/(2*(R^2))) * (1/A3 - 2/A4 + 1/A5) ... 1/An-2*An-1*An = (1/(2*(R^2))) * (1/An-2 - 2/An-1 + 1/An) Mais uma vez : Esta expressao, nao so permite determinar uma formula fechadinha para a soma de N termos como avaliar para onde tende a soma quando N tende ao infinito. Esta semelhanca sera mera coincidencia ? Nao ! Para 4 termos verifique que : 1/A1*A2*A3*A4 = K*(1/R^3)*(1/A1 - 3/A2 + 3/A3 - 1/A4), K=1/(3!) O que voce vai observar de notavel e o seguinte : O INVERSO DE UM PRODUTO ORDENADO DE n TERMOS TENDE SEMPRE PARA O INVERSO DE UM PRODUTO ORDENADO DE n-1 TERMOS. Prove isso por inducao ! Assim ... 1/AiAi+1 vai convergir para K*(1/A1) 1/A1Ai+1Ai+2 vai convergir para K*(1/A1*A2) e assim sucessivamente. Em verdade tudo isso sao consequencias do TRIANGULO HARMONICO onde ficam os BRACOS NEGATIVOS ( ou Progressoes Aritmeticas de ordem negativa ) do Triangulo de Pascal. As relacoes esporadicas que vimos acima sao meras aplicacoes da generalizacao do teorema das colunas ... Uma Pergunta : Por que nos nao conseguimos uma simplificacao semelhante quando Ai=Ai+1=Ai+2 = Ai^3. Uma resposta a essa pergunta pode levar ao calculo da serie : 1 + 1/8 + 1/27 + 1/64 + 1/125 + ... Mas isso era um problema em aberto, onde fracassaram Euler e Gauss e, portanto, muito dificil, sendo assim desonesto propo-lo como uma mera questao olimpica. Um abraco Paulo Santa Rita 2,1401,060502 >1)Determine o menor inteiro positivo a para o qual a equação >1001x+770y=1000000+a possui solução inteira e mostre que que há 100 > >soluções inteiras positivas. >2)Calcule o valor de 1/1*2+1/2*3+....+1/(n-1)*n > Valeu >========================================================================= >Instruções para entrar na lista, sair da lista e usar a lista em >http://www.mat.puc-rio.br/~nicolau/olimp/obm-l.html >O administrador desta lista é >======================================================================= _________________________________________________________________ Una-se ao maior serviço de email do mundo: o MSN Hotmail. http://www.hotmail.com ========================================================================= Instruções para entrar na lista, sair da lista e usar a lista em http://www.mat.puc-rio.br/~nicolau/olimp/obm-l.html O administrador desta lista é ========================================================================= From owner-obm-l@sucuri.mat.puc-rio.br Mon May 6 15:10:53 2002 Return-Path: Received: (from majordom@localhost) by sucuri.mat.puc-rio.br (8.9.3/8.9.3) id PAA07420 for obm-l-list; Mon, 6 May 2002 15:09:05 -0300 Received: from imo-r05.mx.aol.com (imo-r05.mx.aol.com [152.163.225.101]) by sucuri.mat.puc-rio.br (8.9.3/8.9.3) with ESMTP id PAA07416 for ; Mon, 6 May 2002 15:09:00 -0300 From: DEOLIVEIRASOU@aol.com Received: from DEOLIVEIRASOU@aol.com by imo-r05.mx.aol.com (mail_out_v32.5.) id z.15a.d720c99 (3998) for ; Mon, 6 May 2002 14:09:36 -0400 (EDT) Message-ID: <15a.d720c99.2a0820e0@aol.com> Date: Mon, 6 May 2002 14:09:36 EDT Subject: [obm-l] =?ISO-8859-1?Q?Ser=E1=3F=3F?= To: obm-l@mat.puc-rio.br MIME-Version: 1.0 Content-Type: multipart/alternative; boundary="part1_15a.d720c99.2a0820e0_boundary" X-Mailer: AOL 7.0 for Windows BR sub 10501 Sender: owner-obm-l@sucuri.mat.puc-rio.br Precedence: bulk Reply-To: obm-l@mat.puc-rio.br --part1_15a.d720c99.2a0820e0_boundary Content-Type: text/plain; charset="ISO-8859-1" Content-Transfer-Encoding: quoted-printable Resolvi , mas achei muito longo....ser=E1 que tem um segredinho que encurta=20 esses exercicios de olimp=EDadas Russas de 1940 e 1950? As olimp=EDadas hoje= em=20 dia s=E3o muito mais dificeis ou =E9 impress=E3o minha? Se forem mais=20 dificeis...por que? 1) Com quantos zeros termina o numero que =E9 produto de todos os inteiros d= e 1=20 a 100, inclusive?? 2)Dados 27 pesos de valores 1^2, 2^2, 3^2,....27^2 unidades. Agrupe esses=20 pesos em tres conjuntos que tenham peso igual.( obs. o mesmo problema poderi= a=20 ser resolvido para o caso de 1998 pesos e valores 1^2, 2^2,3^2,....,1998^2.)= . Valeu rapaziada! Crom --part1_15a.d720c99.2a0820e0_boundary Content-Type: text/html; charset="ISO-8859-1" Content-Transfer-Encoding: quoted-printable Resolvi , mas achei muito longo....ser=E1 que tem um s= egredinho que encurta esses exercicios de olimp=EDadas Russas de 1940 e 1950= ? As olimp=EDadas hoje em dia s=E3o muito mais dificeis ou =E9 impress=E3o m= inha? Se forem mais dificeis...por que?
1) Com quantos zeros termina o numero que =E9 produto de todos os inteiros d= e 1 a 100, inclusive??
2)Dados 27 pesos de valores 1^2, 2^2, 3^2,....27^2 unidades. Agrupe esses p= esos em tres conjuntos que tenham peso igual.( obs. o mesmo problema poderia= ser resolvido para o caso de 1998 pesos e valores 1^2, 2^2,3^2,....,1998^2.= ).
             Val= eu rapaziada!
             Cro= m
--part1_15a.d720c99.2a0820e0_boundary-- ========================================================================= Instruções para entrar na lista, sair da lista e usar a lista em http://www.mat.puc-rio.br/~nicolau/olimp/obm-l.html O administrador desta lista é ========================================================================= From owner-obm-l@sucuri.mat.puc-rio.br Mon May 6 15:32:46 2002 Return-Path: Received: (from majordom@localhost) by sucuri.mat.puc-rio.br (8.9.3/8.9.3) id PAA07917 for obm-l-list; Mon, 6 May 2002 15:32:28 -0300 Received: from hotmail.com (f121.pav1.hotmail.com [64.4.31.121]) by sucuri.mat.puc-rio.br (8.9.3/8.9.3) with ESMTP id PAA07913 for ; Mon, 6 May 2002 15:32:25 -0300 Received: from mail pickup service by hotmail.com with Microsoft SMTPSVC; Mon, 6 May 2002 11:33:05 -0700 Received: from 200.241.125.214 by pv1fd.pav1.hotmail.msn.com with HTTP; Mon, 06 May 2002 18:33:05 GMT X-Originating-IP: [200.241.125.214] From: "Felipe Marinho" To: obm-l@mat.puc-rio.br Subject: [obm-l] Provar desigualdade. Date: Mon, 06 May 2002 14:33:05 -0400 Mime-Version: 1.0 Content-Type: text/plain; charset=iso-8859-1; format=flowed Message-ID: X-OriginalArrivalTime: 06 May 2002 18:33:05.0531 (UTC) FILETIME=[764E8CB0:01C1F52C] Sender: owner-obm-l@sucuri.mat.puc-rio.br Precedence: bulk Reply-To: obm-l@mat.puc-rio.br Olá pessoal da lista, É com imensa esperança que venho aqui pedir ajuda a vocês na resolução deste exercício: 1) Se a e b são números inteiros positivos tais que a + b = 1, prove que ab² <= 4/27. (onde <= significa menor ou igual.) -- Oque eu devo usar para provar isto ? Já tentei usar MA e MG, Desigualdade de Cauchy-Schwarz, porem não consigo sair do lugar. Conta com a ajuda de vocês e com a resolução do exercício. Obrigado, Abraços _________________________________________________________________ Chegou o novo MSN Explorer. Instale já. É gratuito: http://explorer.msn.com.br ========================================================================= Instruções para entrar na lista, sair da lista e usar a lista em http://www.mat.puc-rio.br/~nicolau/olimp/obm-l.html O administrador desta lista é ========================================================================= From owner-obm-l@sucuri.mat.puc-rio.br Mon May 6 15:55:04 2002 Return-Path: Received: (from majordom@localhost) by sucuri.mat.puc-rio.br (8.9.3/8.9.3) id PAA08762 for obm-l-list; Mon, 6 May 2002 15:54:33 -0300 Received: from hotmail.com (law2-f93.hotmail.com [216.32.181.93]) by sucuri.mat.puc-rio.br (8.9.3/8.9.3) with ESMTP id PAA08758 for ; Mon, 6 May 2002 15:54:31 -0300 Received: from mail pickup service by hotmail.com with Microsoft SMTPSVC; Mon, 6 May 2002 11:55:10 -0700 Received: from 32.94.119.254 by lw2fd.hotmail.msn.com with HTTP; Mon, 06 May 2002 18:55:09 GMT X-Originating-IP: [32.94.119.254] From: "Paulo Santa Rita" To: obm-l@mat.puc-rio.br Subject: Re: [obm-l] Provar desigualdade. Date: Mon, 06 May 2002 18:55:09 +0000 Mime-Version: 1.0 Content-Type: text/plain; charset=iso-8859-1; format=flowed Message-ID: X-OriginalArrivalTime: 06 May 2002 18:55:10.0118 (UTC) FILETIME=[8BD27C60:01C1F52F] Sender: owner-obm-l@sucuri.mat.puc-rio.br Precedence: bulk Reply-To: obm-l@mat.puc-rio.br Ola Felipe e demais colegas desta lista, O problema e que os expoentes que voce usa nao da pra ver legal ... Se for a*(b^2) e sendo a+b=1 tire "b" em funcao de "a" ( ou "a" em funcao de "b" ). substitua e a expressao sera um trinomio bem conhecido seu. Ele tem um maximo ! Dai ... >From: "Felipe Marinho" >Reply-To: obm-l@mat.puc-rio.br >To: obm-l@mat.puc-rio.br >Subject: [obm-l] Provar desigualdade. >Date: Mon, 06 May 2002 14:33:05 -0400 > >Olá pessoal da lista, > >É com imensa esperança que venho aqui pedir ajuda a vocês na resolução >deste exercício: > >1) Se a e b são números inteiros positivos tais que a + b = 1, prove que >ab² ><= 4/27. (onde <= significa menor ou igual.) > >-- Oque eu devo usar para provar isto ? Já tentei usar MA e MG, >Desigualdade >de Cauchy-Schwarz, porem não consigo sair do lugar. Conta com a ajuda de >vocês e com a resolução do exercício. > >Obrigado, > >Abraços > >_________________________________________________________________ >Chegou o novo MSN Explorer. Instale já. É gratuito: >http://explorer.msn.com.br > >========================================================================= >Instruções para entrar na lista, sair da lista e usar a lista em >http://www.mat.puc-rio.br/~nicolau/olimp/obm-l.html >O administrador desta lista é >========================================================================= _________________________________________________________________ Una-se ao maior serviço de email do mundo: o MSN Hotmail. http://www.hotmail.com ========================================================================= Instruções para entrar na lista, sair da lista e usar a lista em http://www.mat.puc-rio.br/~nicolau/olimp/obm-l.html O administrador desta lista é ========================================================================= From owner-obm-l@sucuri.mat.puc-rio.br Mon May 6 16:03:49 2002 Return-Path: Received: (from majordom@localhost) by sucuri.mat.puc-rio.br (8.9.3/8.9.3) id QAA09146 for obm-l-list; Mon, 6 May 2002 16:03:42 -0300 Received: from hotmail.com (law2-f23.hotmail.com [216.32.181.23]) by sucuri.mat.puc-rio.br (8.9.3/8.9.3) with ESMTP id QAA09139 for ; Mon, 6 May 2002 16:03:39 -0300 Received: from mail pickup service by hotmail.com with Microsoft SMTPSVC; Mon, 6 May 2002 12:04:23 -0700 Received: from 200.229.244.100 by lw2fd.hotmail.msn.com with HTTP; Mon, 06 May 2002 19:04:23 GMT X-Originating-IP: [200.229.244.100] From: "Paulo Santa Rita" To: obm-l@mat.puc-rio.br Subject: [obm-l] =?iso-8859-1?B?UmU6IFtvYm0tbF0gU2Vy4T8/?= Date: Mon, 06 May 2002 19:04:23 +0000 Mime-Version: 1.0 Content-Type: text/plain; charset=iso-8859-1; format=flowed Message-ID: X-OriginalArrivalTime: 06 May 2002 19:04:23.0531 (UTC) FILETIME=[D5AE9BB0:01C1F530] Sender: owner-obm-l@sucuri.mat.puc-rio.br Precedence: bulk Reply-To: obm-l@mat.puc-rio.br Ola Oliveiras e demais colegas desta lista, 1)Colocando um numero apos o outro 1*2*3*...*100 e imaginando eles como tivessem sido decompostos em fatores primos, para cada par de fatores 2 e 5 surgira um zero no final de 100 ! Como ha muito mais fatores 2 do que 5, o problema se resume a contar quantos fatores 5 aparecem. E um problema de PA. 2)Se os pessos sao iguais e porque existem duas tres sequencias de quadrados que somam o mesmo valor. Em (a+b)^2=a^2 + 2*ab + b^2 surgem tres. Excolhendo convenientemente "a" e "b" em funcao de uma mesma variavel ... >From: DEOLIVEIRASOU@aol.com >Reply-To: obm-l@mat.puc-rio.br >To: obm-l@mat.puc-rio.br >Subject: [obm-l] Será?? >Date: Mon, 6 May 2002 14:09:36 EDT > >Resolvi , mas achei muito longo....será que tem um segredinho que encurta >esses exercicios de olimpíadas Russas de 1940 e 1950? As olimpíadas hoje em >dia são muito mais dificeis ou é impressão minha? Se forem mais >dificeis...por que? >1) Com quantos zeros termina o numero que é produto de todos os inteiros de >1 >a 100, inclusive?? > 2)Dados 27 pesos de valores 1^2, 2^2, 3^2,....27^2 unidades. Agrupe esses >pesos em tres conjuntos que tenham peso igual.( obs. o mesmo problema >poderia >ser resolvido para o caso de 1998 pesos e valores 1^2, >2^2,3^2,....,1998^2.). > Valeu rapaziada! > Crom _________________________________________________________________ O MSN Photos é o modo mais fácil de compartilhar e imprimir suas fotos: http://photos.msn.com/support/worldwide.aspx ========================================================================= Instruções para entrar na lista, sair da lista e usar a lista em http://www.mat.puc-rio.br/~nicolau/olimp/obm-l.html O administrador desta lista é ========================================================================= From owner-obm-l@sucuri.mat.puc-rio.br Mon May 6 16:28:30 2002 Return-Path: Received: (from majordom@localhost) by sucuri.mat.puc-rio.br (8.9.3/8.9.3) id QAA10105 for obm-l-list; Mon, 6 May 2002 16:27:31 -0300 Received: from hotmail.com (law2-f80.hotmail.com [216.32.181.80]) by sucuri.mat.puc-rio.br (8.9.3/8.9.3) with ESMTP id QAA10097 for ; Mon, 6 May 2002 16:27:29 -0300 Received: from mail pickup service by hotmail.com with Microsoft SMTPSVC; Mon, 6 May 2002 12:28:12 -0700 Received: from 32.94.119.254 by lw2fd.hotmail.msn.com with HTTP; Mon, 06 May 2002 19:28:11 GMT X-Originating-IP: [32.94.119.254] From: "Paulo Santa Rita" To: obm-l@mat.puc-rio.br Subject: Re: [obm-l] Provar desigualdade.( corrigindo um cochilo ) Date: Mon, 06 May 2002 19:28:11 +0000 Mime-Version: 1.0 Content-Type: text/plain; charset=iso-8859-1; format=flowed Message-ID: X-OriginalArrivalTime: 06 May 2002 19:28:12.0184 (UTC) FILETIME=[2939C180:01C1F534] Sender: owner-obm-l@sucuri.mat.puc-rio.br Precedence: bulk Reply-To: obm-l@mat.puc-rio.br Ola Pessoal, Agora que fui perceber um detalhe ... sendo "a" e "b" INTEIROS POSITIVOS e "a + b = 1" segue a=1 e b=1. O SEU PROBLEMA ESTA MAL FORMULADO !!!! Uma formulacao consistente seria : Prove que se "a" e "b" sao REAIS POSITIVOS e "a+b=1" entao a*(b^2) =< 4/27. >From: "Paulo Santa Rita" >Reply-To: obm-l@mat.puc-rio.br >To: obm-l@mat.puc-rio.br >Subject: Re: [obm-l] Provar desigualdade. >Date: Mon, 06 May 2002 18:55:09 +0000 > >Ola Felipe e demais >colegas desta lista, > >O problema e que os expoentes que voce usa nao da pra ver legal ... >Se for a*(b^2) e sendo a+b=1 tire "b" em funcao de "a" ( ou "a" em funcao >de "b" ). substitua e a expressao sera um trinomio bem conhecido seu. Ele >tem um maximo ! Dai ... > > > >>From: "Felipe Marinho" >>Reply-To: obm-l@mat.puc-rio.br >>To: obm-l@mat.puc-rio.br >>Subject: [obm-l] Provar desigualdade. >>Date: Mon, 06 May 2002 14:33:05 -0400 >> >>Olá pessoal da lista, >> >>É com imensa esperança que venho aqui pedir ajuda a vocês na resolução >>deste exercício: >> >>1) Se a e b são números inteiros positivos tais que a + b = 1, prove que >>ab² >><= 4/27. (onde <= significa menor ou igual.) >> >>-- Oque eu devo usar para provar isto ? Já tentei usar MA e MG, >>Desigualdade >>de Cauchy-Schwarz, porem não consigo sair do lugar. Conta com a ajuda de >>vocês e com a resolução do exercício. >> >>Obrigado, >> >>Abraços >> >>_________________________________________________________________ >>Chegou o novo MSN Explorer. Instale já. É gratuito: >>http://explorer.msn.com.br >> >>========================================================================= >>Instruções para entrar na lista, sair da lista e usar a lista em >>http://www.mat.puc-rio.br/~nicolau/olimp/obm-l.html >>O administrador desta lista é >>========================================================================= > > > > >_________________________________________________________________ >Una-se ao maior serviço de email do mundo: o MSN Hotmail. >http://www.hotmail.com > >========================================================================= >Instruções para entrar na lista, sair da lista e usar a lista em >http://www.mat.puc-rio.br/~nicolau/olimp/obm-l.html >O administrador desta lista é >========================================================================= _________________________________________________________________ Converse com amigos on-line, conheça o MSN Messenger: http://messenger.msn.com ========================================================================= Instruções para entrar na lista, sair da lista e usar a lista em http://www.mat.puc-rio.br/~nicolau/olimp/obm-l.html O administrador desta lista é ========================================================================= From owner-obm-l@sucuri.mat.puc-rio.br Mon May 6 17:01:20 2002 Return-Path: Received: (from majordom@localhost) by sucuri.mat.puc-rio.br (8.9.3/8.9.3) id RAA11118 for obm-l-list; Mon, 6 May 2002 17:00:57 -0300 Received: from www.zipmail.com.br (smtp.zipmail.com.br [200.187.242.10]) by sucuri.mat.puc-rio.br (8.9.3/8.9.3) with ESMTP id RAA11114 for ; Mon, 6 May 2002 17:00:51 -0300 From: peterdirichlet@zipmail.com.br Received: from [200.206.103.3] by www.zipmail.com.br with HTTP; Mon, 6 May 2002 17:01:02 -0300 Message-ID: <3CD6A4FE00001699@www.zipmail.com.br> Date: Mon, 6 May 2002 17:01:02 -0300 In-Reply-To: <15a.d720c99.2a0820e0@aol.com> Subject: [obm-l] =?iso-8859-1?Q?Re=3A=20=5Bobm=2Dl=5D=20Ser=E1=3F=3F?= To: obm-l@mat.puc-rio.br MIME-Version: 1.0 Content-Type: text/plain; charset="iso-8859-1" Content-Transfer-Encoding: 8bit X-MIME-Autoconverted: from quoted-printable to 8bit by sucuri.mat.puc-rio.br id RAA11115 Sender: owner-obm-l@sucuri.mat.puc-rio.br Precedence: bulk Reply-To: obm-l@mat.puc-rio.br E la vou eu de novo...Para o primeiro veja que a maior potencia de um certo primo p que divide n! e:[n/p]+[n/p^2]+[n/p^3]+... Para o segundo,uma ideia e agrupar os 9 primeiros e ver no que da.Depois meio que de PIF voce continua. FALOWS!!!!!!Peterdirichlet -- Mensagem original -- >Resolvi , mas achei muito longo....será que tem um segredinho que encurta > >esses exercicios de olimpíadas Russas de 1940 e 1950? As olimpíadas hoje >em >dia são muito mais dificeis ou é impressão minha? Se forem mais >dificeis...por que? >1) Com quantos zeros termina o numero que é produto de todos os inteiros >de 1 >a 100, inclusive?? > 2)Dados 27 pesos de valores 1^2, 2^2, 3^2,....27^2 unidades. Agrupe esses > >pesos em tres conjuntos que tenham peso igual.( obs. o mesmo problema poderia > >ser resolvido para o caso de 1998 pesos e valores 1^2, 2^2,3^2,....,1998^2.). > Valeu rapaziada! > Crom > ------------------------------------------ Use o melhor sistema de busca da Internet Radar UOL - http://www.radaruol.com.br ========================================================================= Instruções para entrar na lista, sair da lista e usar a lista em http://www.mat.puc-rio.br/~nicolau/olimp/obm-l.html O administrador desta lista é ========================================================================= From owner-obm-l@sucuri.mat.puc-rio.br Mon May 6 18:52:36 2002 Return-Path: Received: (from majordom@localhost) by sucuri.mat.puc-rio.br (8.9.3/8.9.3) id SAA12929 for obm-l-list; Mon, 6 May 2002 18:52:14 -0300 Received: from hotmail.com (f186.pav1.hotmail.com [64.4.31.186]) by sucuri.mat.puc-rio.br (8.9.3/8.9.3) with ESMTP id SAA12925 for ; Mon, 6 May 2002 18:52:12 -0300 Received: from mail pickup service by hotmail.com with Microsoft SMTPSVC; Mon, 6 May 2002 14:52:56 -0700 Received: from 200.241.125.137 by pv1fd.pav1.hotmail.msn.com with HTTP; Mon, 06 May 2002 21:52:56 GMT X-Originating-IP: [200.241.125.137] From: "Felipe Marinho" To: obm-l@mat.puc-rio.br Subject: Re: [obm-l] Provar desigualdade.( corrigindo um cochilo ) Date: Mon, 06 May 2002 17:52:56 -0400 Mime-Version: 1.0 Content-Type: text/plain; charset=iso-8859-1; format=flowed Message-ID: X-OriginalArrivalTime: 06 May 2002 21:52:56.0624 (UTC) FILETIME=[618E2300:01C1F548] Sender: owner-obm-l@sucuri.mat.puc-rio.br Precedence: bulk Reply-To: obm-l@mat.puc-rio.br Caro amigo Paulo, Desculpe-me por mais este enunciado enviado de maneira errada. A questão fala na verdade que a e b são REAIS POSITIVOS. Peço desculpas aqui. E Obrigado desde já, Felipe Marinho >From: "Paulo Santa Rita" >Reply-To: obm-l@mat.puc-rio.br >To: obm-l@mat.puc-rio.br >Subject: Re: [obm-l] Provar desigualdade.( corrigindo um cochilo ) >Date: Mon, 06 May 2002 19:28:11 +0000 > >Ola Pessoal, > >Agora que fui perceber um detalhe ... sendo "a" e "b" INTEIROS POSITIVOS e >"a + b = 1" segue a=1 e b=1. O SEU PROBLEMA ESTA MAL FORMULADO !!!! > >Uma formulacao consistente seria : > >Prove que se "a" e "b" sao REAIS POSITIVOS e "a+b=1" entao a*(b^2) =< 4/27. > > >>From: "Paulo Santa Rita" >>Reply-To: obm-l@mat.puc-rio.br >>To: obm-l@mat.puc-rio.br >>Subject: Re: [obm-l] Provar desigualdade. >>Date: Mon, 06 May 2002 18:55:09 +0000 >> >>Ola Felipe e demais >>colegas desta lista, >> >>O problema e que os expoentes que voce usa nao da pra ver legal ... >>Se for a*(b^2) e sendo a+b=1 tire "b" em funcao de "a" ( ou "a" em funcao >>de "b" ). substitua e a expressao sera um trinomio bem conhecido seu. Ele >>tem um maximo ! Dai ... >> >> >> >>>From: "Felipe Marinho" >>>Reply-To: obm-l@mat.puc-rio.br >>>To: obm-l@mat.puc-rio.br >>>Subject: [obm-l] Provar desigualdade. >>>Date: Mon, 06 May 2002 14:33:05 -0400 >>> >>>Olá pessoal da lista, >>> >>>É com imensa esperança que venho aqui pedir ajuda a vocês na resolução >>>deste exercício: >>> >>>1) Se a e b são números inteiros positivos tais que a + b = 1, prove que >>>ab² >>><= 4/27. (onde <= significa menor ou igual.) >>> >>>-- Oque eu devo usar para provar isto ? Já tentei usar MA e MG, >>>Desigualdade >>>de Cauchy-Schwarz, porem não consigo sair do lugar. Conta com a ajuda de >>>vocês e com a resolução do exercício. >>> >>>Obrigado, >>> >>>Abraços >>> >>>_________________________________________________________________ >>>Chegou o novo MSN Explorer. Instale já. É gratuito: >>>http://explorer.msn.com.br >>> >>>========================================================================= >>>Instruções para entrar na lista, sair da lista e usar a lista em >>>http://www.mat.puc-rio.br/~nicolau/olimp/obm-l.html >>>O administrador desta lista é >>>========================================================================= >> >> >> >> >>_________________________________________________________________ >>Una-se ao maior serviço de email do mundo: o MSN Hotmail. >>http://www.hotmail.com >> >>========================================================================= >>Instruções para entrar na lista, sair da lista e usar a lista em >>http://www.mat.puc-rio.br/~nicolau/olimp/obm-l.html >>O administrador desta lista é >>========================================================================= > > > > >_________________________________________________________________ >Converse com amigos on-line, conheça o MSN Messenger: >http://messenger.msn.com > >========================================================================= >Instruções para entrar na lista, sair da lista e usar a lista em >http://www.mat.puc-rio.br/~nicolau/olimp/obm-l.html >O administrador desta lista é >========================================================================= _________________________________________________________________ Chegou o novo MSN Explorer. Instale já. É gratuito: http://explorer.msn.com.br ========================================================================= Instruções para entrar na lista, sair da lista e usar a lista em http://www.mat.puc-rio.br/~nicolau/olimp/obm-l.html O administrador desta lista é ========================================================================= From owner-obm-l@sucuri.mat.puc-rio.br Mon May 6 21:32:33 2002 Return-Path: Received: (from majordom@localhost) by sucuri.mat.puc-rio.br (8.9.3/8.9.3) id VAA14930 for obm-l-list; Mon, 6 May 2002 21:32:16 -0300 Received: from hotmail.com (f58.pav1.hotmail.com [64.4.31.58]) by sucuri.mat.puc-rio.br (8.9.3/8.9.3) with ESMTP id VAA14926 for ; Mon, 6 May 2002 21:32:14 -0300 Received: from mail pickup service by hotmail.com with Microsoft SMTPSVC; Mon, 6 May 2002 17:32:54 -0700 Received: from 200.241.125.171 by pv1fd.pav1.hotmail.msn.com with HTTP; Tue, 07 May 2002 00:32:54 GMT X-Originating-IP: [200.241.125.171] From: "Felipe Marinho" To: obm-l@mat.puc-rio.br Subject: Re: [obm-l] Provar desigualdade.( corrigindo um cochilo ) Date: Mon, 06 May 2002 20:32:54 -0400 Mime-Version: 1.0 Content-Type: text/plain; charset=iso-8859-1; format=flowed Message-ID: X-OriginalArrivalTime: 07 May 2002 00:32:54.0936 (UTC) FILETIME=[BA985180:01C1F55E] Sender: owner-obm-l@sucuri.mat.puc-rio.br Precedence: bulk Reply-To: obm-l@mat.puc-rio.br Caro Paulo, Eu ia fazer um comentário aqui tambem, porem mais uma vez acabei esquecendo. ;) Olha, é o seguinte, essa questão, no caso, teríamos que ab² = a(1-a)², e derivando a função podemos analisar em quais intervalos a função é crescente ou decresente. Porem, eu evitei ao máximo resolver essa questão analisando o gráfico da derivada, pois a mesma é uma questão do nível II da OBM (Nível II - 7a. e 8a séries). E com certeza, os alunos de tais séries ainda não tiveram uma iniciação em assuntos como derivadas, limites, etc. Por isso que vim até aqui a lista, para "procurar" uma outra solução para o mesmo problema. E mais uma vez, peço a ajuda aqui de vocês. Paulo, obrigado por tudo. Valeu mesmo.. de coração ! :) Um grande abraço Felipe Marinho >From: "Felipe Marinho" >Reply-To: obm-l@mat.puc-rio.br >To: obm-l@mat.puc-rio.br >Subject: Re: [obm-l] Provar desigualdade.( corrigindo um cochilo ) >Date: Mon, 06 May 2002 17:52:56 -0400 > >Caro amigo Paulo, > >Desculpe-me por mais este enunciado enviado de maneira errada. > >A questão fala na verdade que a e b são REAIS POSITIVOS. > >Peço desculpas aqui. > >E Obrigado desde já, > >Felipe Marinho > >>From: "Paulo Santa Rita" >>Reply-To: obm-l@mat.puc-rio.br >>To: obm-l@mat.puc-rio.br >>Subject: Re: [obm-l] Provar desigualdade.( corrigindo um cochilo ) >>Date: Mon, 06 May 2002 19:28:11 +0000 >> >>Ola Pessoal, >> >>Agora que fui perceber um detalhe ... sendo "a" e "b" INTEIROS POSITIVOS >>e >>"a + b = 1" segue a=1 e b=1. O SEU PROBLEMA ESTA MAL FORMULADO !!!! >> >>Uma formulacao consistente seria : >> >>Prove que se "a" e "b" sao REAIS POSITIVOS e "a+b=1" entao a*(b^2) =< >>4/27. >> >> >>>From: "Paulo Santa Rita" >>>Reply-To: obm-l@mat.puc-rio.br >>>To: obm-l@mat.puc-rio.br >>>Subject: Re: [obm-l] Provar desigualdade. >>>Date: Mon, 06 May 2002 18:55:09 +0000 >>> >>>Ola Felipe e demais >>>colegas desta lista, >>> >>>O problema e que os expoentes que voce usa nao da pra ver legal ... >>>Se for a*(b^2) e sendo a+b=1 tire "b" em funcao de "a" ( ou "a" em >>>funcao >>>de "b" ). substitua e a expressao sera um trinomio bem conhecido seu. Ele >>>tem um maximo ! Dai ... >>> >>> >>> >>>>From: "Felipe Marinho" >>>>Reply-To: obm-l@mat.puc-rio.br >>>>To: obm-l@mat.puc-rio.br >>>>Subject: [obm-l] Provar desigualdade. >>>>Date: Mon, 06 May 2002 14:33:05 -0400 >>>> >>>>Olá pessoal da lista, >>>> >>>>É com imensa esperança que venho aqui pedir ajuda a vocês na resolução >>>>deste exercício: >>>> >>>>1) Se a e b são números inteiros positivos tais que a + b = 1, prove que >>>>ab² >>>><= 4/27. (onde <= significa menor ou igual.) >>>> >>>>-- Oque eu devo usar para provar isto ? Já tentei usar MA e MG, >>>>Desigualdade >>>>de Cauchy-Schwarz, porem não consigo sair do lugar. Conta com a ajuda de >>>>vocês e com a resolução do exercício. >>>> >>>>Obrigado, >>>> >>>>Abraços >>>> >>>>_________________________________________________________________ >>>>Chegou o novo MSN Explorer. Instale já. É gratuito: >>>>http://explorer.msn.com.br >>>> >>>>========================================================================= >>>>Instruções para entrar na lista, sair da lista e usar a lista em >>>>http://www.mat.puc-rio.br/~nicolau/olimp/obm-l.html >>>>O administrador desta lista é >>>>========================================================================= >>> >>> >>> >>> >>>_________________________________________________________________ >>>Una-se ao maior serviço de email do mundo: o MSN Hotmail. >>>http://www.hotmail.com >>> >>>========================================================================= >>>Instruções para entrar na lista, sair da lista e usar a lista em >>>http://www.mat.puc-rio.br/~nicolau/olimp/obm-l.html >>>O administrador desta lista é >>>========================================================================= >> >> >> >> >>_________________________________________________________________ >>Converse com amigos on-line, conheça o MSN Messenger: >>http://messenger.msn.com >> >>========================================================================= >>Instruções para entrar na lista, sair da lista e usar a lista em >>http://www.mat.puc-rio.br/~nicolau/olimp/obm-l.html >>O administrador desta lista é >>========================================================================= > > >_________________________________________________________________ >Chegou o novo MSN Explorer. Instale já. É gratuito: >http://explorer.msn.com.br > >========================================================================= >Instruções para entrar na lista, sair da lista e usar a lista em >http://www.mat.puc-rio.br/~nicolau/olimp/obm-l.html >O administrador desta lista é >========================================================================= _________________________________________________________________ Chegou o novo MSN Explorer. Instale já. É gratuito: http://explorer.msn.com.br ========================================================================= Instruções para entrar na lista, sair da lista e usar a lista em http://www.mat.puc-rio.br/~nicolau/olimp/obm-l.html O administrador desta lista é ========================================================================= From owner-obm-l@sucuri.mat.puc-rio.br Tue May 7 00:05:20 2002 Return-Path: Received: (from majordom@localhost) by sucuri.mat.puc-rio.br (8.9.3/8.9.3) id AAA16307 for obm-l-list; Tue, 7 May 2002 00:02:51 -0300 Received: from shannon.bol.com.br (shannon.bol.com.br [200.221.24.13]) by sucuri.mat.puc-rio.br (8.9.3/8.9.3) with ESMTP id AAA16303 for ; Tue, 7 May 2002 00:02:50 -0300 Received: from bol.com.br (200.221.24.67) by shannon.bol.com.br (5.1.071) id 3C6F230B0109C4DA for obm-l@mat.puc-rio.br; Tue, 7 May 2002 00:03:11 -0300 Date: Tue, 7 May 2002 00:03:11 -0300 Message-Id: Subject: [obm-l] bibliografia indicada IME/ITA MIME-Version: 1.0 Content-Type: text/plain;charset="iso-8859-1" From: "rafaelc.l" To: obm-l@mat.puc-rio.br X-XaM3-API-Version: 2.4.3.4.4 X-SenderIP: 200.176.167.71 Content-Transfer-Encoding: 8bit X-MIME-Autoconverted: from quoted-printable to 8bit by sucuri.mat.puc-rio.br id AAA16304 Sender: owner-obm-l@sucuri.mat.puc-rio.br Precedence: bulk Reply-To: obm-l@mat.puc-rio.br Já foi dito aqui na lista os livros mais indicados para quem se prepara pro IME e ITA. Estou com fundamentos da Matemática Elementar de Gelson Iezzi, Física Clássica do Calçadas e química é que eu queria saber..falaram que era o Feltre, mas é que tem o Feltre sozinho e outro que é Feltre e Yoshinaga, um bem antigo.....qual deles é o indicado? Se puderem me indicar uma boa gramática de português para esse tipo de vestibular, tbm ficarei grato..... Obrigado __________________________________________________________________________ Quer ter seu próprio endereço na Internet? Garanta já o seu e ainda ganhe cinco e-mails personalizados. DomíniosBOL - http://dominios.bol.com.br ========================================================================= Instruções para entrar na lista, sair da lista e usar a lista em http://www.mat.puc-rio.br/~nicolau/olimp/obm-l.html O administrador desta lista é ========================================================================= From owner-obm-l@sucuri.mat.puc-rio.br Tue May 7 00:11:40 2002 Return-Path: Received: (from majordom@localhost) by sucuri.mat.puc-rio.br (8.9.3/8.9.3) id AAA16447 for obm-l-list; Tue, 7 May 2002 00:09:40 -0300 Received: from web13709.mail.yahoo.com (web13709.mail.yahoo.com [216.136.175.251]) by sucuri.mat.puc-rio.br (8.9.3/8.9.3) with SMTP id AAA16443 for ; Tue, 7 May 2002 00:09:38 -0300 Message-ID: <20020507031023.20970.qmail@web13709.mail.yahoo.com> Received: from [200.213.88.160] by web13709.mail.yahoo.com via HTTP; Tue, 07 May 2002 00:10:23 ART Date: Tue, 7 May 2002 00:10:23 -0300 (ART) From: =?iso-8859-1?q?pichurin?= Subject: [obm-l] porcentagem To: obm-l@mat.puc-rio.br MIME-Version: 1.0 Content-Type: text/plain; charset=iso-8859-1 Content-Transfer-Encoding: 8bit Sender: owner-obm-l@sucuri.mat.puc-rio.br Precedence: bulk Reply-To: obm-l@mat.puc-rio.br Por favor, alguém poderia me enviar alguns exercícios que envolvam porcentegagem?Ou então falem o endereço de algum site onde posso encontrar isso.Mas exercícios bons......Valeeeeeu! _______________________________________________________________________ Yahoo! Encontros O lugar certo para você encontrar aquela pessoa que falta na sua vida. Cadastre-se hoje mesmo! http://br.encontros.yahoo.com/ ========================================================================= Instruções para entrar na lista, sair da lista e usar a lista em http://www.mat.puc-rio.br/~nicolau/olimp/obm-l.html O administrador desta lista é ========================================================================= From owner-obm-l@sucuri.mat.puc-rio.br Tue May 7 00:21:16 2002 Return-Path: Received: (from majordom@localhost) by sucuri.mat.puc-rio.br (8.9.3/8.9.3) id AAA16579 for obm-l-list; Tue, 7 May 2002 00:18:40 -0300 Received: from www.zipmail.com.br (smtp.zipmail.com.br [200.187.242.10]) by sucuri.mat.puc-rio.br (8.9.3/8.9.3) with ESMTP id AAA16575 for ; Tue, 7 May 2002 00:18:38 -0300 From: ciceroth@zipmail.com.br Received: from [200.253.200.134] by www.zipmail.com.br with HTTP; Tue, 7 May 2002 00:17:53 -0300 Message-ID: <3CD1900000025B65@www.zipmail.com.br> Date: Tue, 7 May 2002 00:17:53 -0300 In-Reply-To: Subject: [obm-l] =?iso-8859-1?Q?Re=3A=20=5Bobm=2Dl=5D=20Provar=20desigualdade=2E?= To: obm-l@mat.puc-rio.br MIME-Version: 1.0 Content-Type: text/plain; charset="iso-8859-1" Content-Transfer-Encoding: 8bit X-MIME-Autoconverted: from quoted-printable to 8bit by sucuri.mat.puc-rio.br id AAA16576 Sender: owner-obm-l@sucuri.mat.puc-rio.br Precedence: bulk Reply-To: obm-l@mat.puc-rio.br como a + b = 1, usando MA => MG, temos 2a + b + b/3 =>(2ab²)^1/3 2/3=>(2ab²)^1/3 <=> 8/27=>2ab² <=> ab²<=4/27 um abraço Cicero Thiago Ai ai um problema que eu achei muito interessante Seja ABCD um quadrilátero convexo inscrito em um semicirculo de diametro AB. Os segmentos AC e BD se intersectam em E e os segmentos AD e BC em F. O segmento EF intersecta o semicirculo em G e o segmento AB em H. Prove que E é o ponto médio do segmento GH se e somente se G é o ponto médio do segmento FH. ------------------------------------------ Use o melhor sistema de busca da Internet Radar UOL - http://www.radaruol.com.br ========================================================================= Instruções para entrar na lista, sair da lista e usar a lista em http://www.mat.puc-rio.br/~nicolau/olimp/obm-l.html O administrador desta lista é ========================================================================= From owner-obm-l@sucuri.mat.puc-rio.br Tue May 7 00:32:11 2002 Return-Path: Received: (from majordom@localhost) by sucuri.mat.puc-rio.br (8.9.3/8.9.3) id AAA17113 for obm-l-list; Tue, 7 May 2002 00:30:53 -0300 Received: from www.zipmail.com.br (smtp.zipmail.com.br [200.187.242.10]) by sucuri.mat.puc-rio.br (8.9.3/8.9.3) with ESMTP id AAA17103 for ; Tue, 7 May 2002 00:30:51 -0300 From: ciceroth@zipmail.com.br Received: from [200.253.200.134] by www.zipmail.com.br with HTTP; Tue, 7 May 2002 00:30:07 -0300 Message-ID: <3CD1900000025CB6@www.zipmail.com.br> Date: Tue, 7 May 2002 00:30:07 -0300 In-Reply-To: Subject: [obm-l] =?iso-8859-1?Q?Re=3A=20=5Bobm=2Dl=5D=20Provar=20desigualdade=2E?= To: obm-l@mat.puc-rio.br MIME-Version: 1.0 Content-Type: text/plain; charset="iso-8859-1" Content-Transfer-Encoding: 8bit X-MIME-Autoconverted: from quoted-printable to 8bit by sucuri.mat.puc-rio.br id AAA17110 Sender: owner-obm-l@sucuri.mat.puc-rio.br Precedence: bulk Reply-To: obm-l@mat.puc-rio.br olá se a + b = 1 usando o fato que MA => MG, então 2a + b + b/3 => (2ab²)^1/3 2/3 => (2ab²)^1/3 <=> 8/27 => 2ab² <=> ab²<= 4/27 um abraço Cicero Thiago Fortaleza CE ------------------------------------------ Use o melhor sistema de busca da Internet Radar UOL - http://www.radaruol.com.br ========================================================================= Instruções para entrar na lista, sair da lista e usar a lista em http://www.mat.puc-rio.br/~nicolau/olimp/obm-l.html O administrador desta lista é ========================================================================= From owner-obm-l@sucuri.mat.puc-rio.br Tue May 7 12:01:42 2002 Return-Path: Received: (from majordom@localhost) by sucuri.mat.puc-rio.br (8.9.3/8.9.3) id LAA22681 for obm-l-list; Tue, 7 May 2002 11:51:28 -0300 Received: from mail.terraempresas.com.br (mail.terraempresas.com.br [200.177.96.20]) by sucuri.mat.puc-rio.br (8.9.3/8.9.3) with ESMTP id LAA22677 for ; Tue, 7 May 2002 11:51:24 -0300 Received: from cemag.com.br (ipcorp-C8B17502.terraempresas.com.br [200.177.117.2] (may be forged)) by mail.terraempresas.com.br (8.12.2/8.11.2) with SMTP id g47Enj7h020816 for ; Tue, 7 May 2002 11:52:03 -0300 Received: from 192.168.3.2 ([192.168.3.2]) by cemag.com.br (WinRoute Pro 4.1) with SMTP; Tue, 7 May 2002 09:05:45 -0300 Message-ID: <001801c1f5be$c9dd4f00$0203a8c0@secrel.com.br> From: "Davidson Estanislau" To: "obm" Subject: [obm-l] =?iso-8859-1?Q?=C1lgebra_Linear?= Date: Tue, 7 May 2002 09:00:30 -0300 Organization: Cemag MIME-Version: 1.0 Content-Type: multipart/alternative; boundary="----=_NextPart_000_0015_01C1F5A5.A3BD84C0" X-Priority: 3 X-MSMail-Priority: Normal X-Mailer: Microsoft Outlook Express 6.00.2600.0000 X-MimeOLE: Produced By Microsoft MimeOLE V6.00.2600.0000 Sender: owner-obm-l@sucuri.mat.puc-rio.br Precedence: bulk Reply-To: obm-l@mat.puc-rio.br This is a multi-part message in MIME format. ------=_NextPart_000_0015_01C1F5A5.A3BD84C0 Content-Type: text/plain; charset="iso-8859-1" Content-Transfer-Encoding: quoted-printable Bom dia! Estou precisando da ajuda de voc=EAs, nestes dois problemas: 1. Determine uma transforma=E7=E3o linear T: R^3 -> R^3, cuja imagem = e n=FAcleo s=E3o, respectivamente, os subspa=E7os E =3D [(1, 1, 1), (1, = -1, 1)] e F =3D [(1, 0, -1)]. 2. Determine uma base para o n=FAcleo da transforma=E7=E3o linear = T(x, y, z, w) =3D (x + y + 2z + 2w, x - y + 2z - 2w, x + y + 2z + 2w, x = + y + 2z + 2w) Davidson Estanislau ------=_NextPart_000_0015_01C1F5A5.A3BD84C0 Content-Type: text/html; charset="iso-8859-1" Content-Transfer-Encoding: quoted-printable
 
    Bom dia!
 
    Estou precisando da ajuda de voc=EAs, nestes = dois=20 problemas:
 
   1. Determine uma transforma=E7=E3o linear T: R^3 -> = R^3, cuja=20 imagem e n=FAcleo s=E3o, respectivamente, os subspa=E7os E =3D [(1, 1, = 1), (1, -1, 1)] e=20 F =3D [(1, 0, -1)].
 
   2. Determine uma base para o n=FAcleo da = transforma=E7=E3o linear=20 T(x, y, z, w) =3D (x + y + 2z + 2w, x - y + 2z - 2w, x + y + 2z + 2w, x = + y + 2z +=20 2w)
 
   Davidson Estanislau
------=_NextPart_000_0015_01C1F5A5.A3BD84C0-- ========================================================================= Instruções para entrar na lista, sair da lista e usar a lista em http://www.mat.puc-rio.br/~nicolau/olimp/obm-l.html O administrador desta lista é ========================================================================= From owner-obm-l@sucuri.mat.puc-rio.br Tue May 7 13:09:36 2002 Return-Path: Received: (from majordom@localhost) by sucuri.mat.puc-rio.br (8.9.3/8.9.3) id NAA23746 for obm-l-list; Tue, 7 May 2002 13:07:47 -0300 Received: from hotmail.com (law2-f10.hotmail.com [216.32.181.10]) by sucuri.mat.puc-rio.br (8.9.3/8.9.3) with ESMTP id NAA23742 for ; Tue, 7 May 2002 13:07:43 -0300 Received: from mail pickup service by hotmail.com with Microsoft SMTPSVC; Tue, 7 May 2002 08:49:53 -0700 Received: from 200.229.244.100 by lw2fd.hotmail.msn.com with HTTP; Tue, 07 May 2002 15:49:53 GMT X-Originating-IP: [200.229.244.100] From: "Paulo Santa Rita" To: obm-l@mat.puc-rio.br Subject: Re: [obm-l] Provar desigualdade.( corrigindo um cochilo ) Date: Tue, 07 May 2002 15:49:53 +0000 Mime-Version: 1.0 Content-Type: text/plain; charset=iso-8859-1; format=flowed Message-ID: X-OriginalArrivalTime: 07 May 2002 15:49:53.0965 (UTC) FILETIME=[D47DE5D0:01C1F5DE] Sender: owner-obm-l@sucuri.mat.puc-rio.br Precedence: bulk Reply-To: obm-l@mat.puc-rio.br Ola Felipe e demais colegas desta lista, Meu camarada, nao ha do que agradecer e nem do que se desculpar... Nos, estudantes, aprendemos muito aqui. E eu me sinto honrado e feliz em poder estar sendo util, mesmo que infimamente, em uma lista orientada e moderada por Grandes Mestes tais como os Prof's Nicolau, Eduardo Wagner, Morgado, Jose Paulo, Ralph e muitos outros. Infelizmente, confesso que nao sei como explicar a um aluno de 7 ou 8 serie estas coisas... Em verdade, eu acho que sao os caras que conseguem fazer este trabalho - e parece que voce faz - os Profs mais importantes, porque eles pegam o garoto em tenra idade e despertam a inteligencia e interesse deles, fazendo problemas bonitos. Nos aqui temos um colega - o Josimar - que publicou um livro que parece ser a verdadeira solucao para tornar a Matematica atraente. O Livro e "E divertido resolver problemas". Nao sei de qual editora. Mas o certo e que ele consegue colocar problemas bonitos, sem a mediocridade que parece ser a regra da imensa maioria dos outros, e mostrar formas criativas e inteligentes de aborda-los. Ele faz a Matematica ser empolgante. E isso : genial, por que e simples, simples porque e genial. Com os melhores votos de Paz Profunda, sou Paulo Santa Rita 3,1248,070502 >From: "Felipe Marinho" >Reply-To: obm-l@mat.puc-rio.br >To: obm-l@mat.puc-rio.br >Subject: Re: [obm-l] Provar desigualdade.( corrigindo um cochilo ) >Date: Mon, 06 May 2002 20:32:54 -0400 > > >Caro Paulo, > >Eu ia fazer um comentário aqui tambem, porem mais uma vez acabei >esquecendo. >;) > >Olha, é o seguinte, essa questão, no caso, teríamos que ab² = a(1-a)², e >derivando a função podemos analisar em quais intervalos a função é >crescente >ou decresente. > >Porem, eu evitei ao máximo resolver essa questão analisando o gráfico da >derivada, pois a mesma é uma questão do nível II da OBM (Nível II - 7a. e >8a >séries). E com certeza, os alunos de tais séries ainda não tiveram uma >iniciação em assuntos como derivadas, limites, etc. > >Por isso que vim até aqui a lista, para "procurar" uma outra solução para o >mesmo problema. > >E mais uma vez, peço a ajuda aqui de vocês. > >Paulo, obrigado por tudo. Valeu mesmo.. de coração ! :) > >Um grande abraço >Felipe Marinho > >>From: "Felipe Marinho" >>Reply-To: obm-l@mat.puc-rio.br >>To: obm-l@mat.puc-rio.br >>Subject: Re: [obm-l] Provar desigualdade.( corrigindo um cochilo ) >>Date: Mon, 06 May 2002 17:52:56 -0400 >> >>Caro amigo Paulo, >> >>Desculpe-me por mais este enunciado enviado de maneira errada. >> >>A questão fala na verdade que a e b são REAIS POSITIVOS. >> >>Peço desculpas aqui. >> >>E Obrigado desde já, >> >>Felipe Marinho >> >>>From: "Paulo Santa Rita" >>>Reply-To: obm-l@mat.puc-rio.br >>>To: obm-l@mat.puc-rio.br >>>Subject: Re: [obm-l] Provar desigualdade.( corrigindo um cochilo ) >>>Date: Mon, 06 May 2002 19:28:11 +0000 >>> >>>Ola Pessoal, >>> >>>Agora que fui perceber um detalhe ... sendo "a" e "b" INTEIROS POSITIVOS >>>e >>>"a + b = 1" segue a=1 e b=1. O SEU PROBLEMA ESTA MAL FORMULADO !!!! >>> >>>Uma formulacao consistente seria : >>> >>>Prove que se "a" e "b" sao REAIS POSITIVOS e "a+b=1" entao a*(b^2) =< >>>4/27. >>> >>> >>>>From: "Paulo Santa Rita" >>>>Reply-To: obm-l@mat.puc-rio.br >>>>To: obm-l@mat.puc-rio.br >>>>Subject: Re: [obm-l] Provar desigualdade. >>>>Date: Mon, 06 May 2002 18:55:09 +0000 >>>> >>>>Ola Felipe e demais >>>>colegas desta lista, >>>> >>>>O problema e que os expoentes que voce usa nao da pra ver legal ... >>>>Se for a*(b^2) e sendo a+b=1 tire "b" em funcao de "a" ( ou "a" em >>>>funcao >>>>de "b" ). substitua e a expressao sera um trinomio bem conhecido seu. >>>>Ele >>>>tem um maximo ! Dai ... >>>> >>>> >>>> >>>>>From: "Felipe Marinho" >>>>>Reply-To: obm-l@mat.puc-rio.br >>>>>To: obm-l@mat.puc-rio.br >>>>>Subject: [obm-l] Provar desigualdade. >>>>>Date: Mon, 06 May 2002 14:33:05 -0400 >>>>> >>>>>Olá pessoal da lista, >>>>> >>>>>É com imensa esperança que venho aqui pedir ajuda a vocês na resolução >>>>>deste exercício: >>>>> >>>>>1) Se a e b são números inteiros positivos tais que a + b = 1, prove >>>>>que >>>>>ab² >>>>><= 4/27. (onde <= significa menor ou igual.) >>>>> >>>>>-- Oque eu devo usar para provar isto ? Já tentei usar MA e MG, >>>>>Desigualdade >>>>>de Cauchy-Schwarz, porem não consigo sair do lugar. Conta com a ajuda >>>>>de >>>>>vocês e com a resolução do exercício. >>>>> >>>>>Obrigado, >>>>> >>>>>Abraços >>>>> >>>>>_________________________________________________________________ >>>>>Chegou o novo MSN Explorer. Instale já. É gratuito: >>>>>http://explorer.msn.com.br >>>>> >>>>>========================================================================= >>>>>Instruções para entrar na lista, sair da lista e usar a lista em >>>>>http://www.mat.puc-rio.br/~nicolau/olimp/obm-l.html >>>>>O administrador desta lista é >>>>>========================================================================= >>>> >>>> >>>> >>>> >>>>_________________________________________________________________ >>>>Una-se ao maior serviço de email do mundo: o MSN Hotmail. >>>>http://www.hotmail.com >>>> >>>>========================================================================= >>>>Instruções para entrar na lista, sair da lista e usar a lista em >>>>http://www.mat.puc-rio.br/~nicolau/olimp/obm-l.html >>>>O administrador desta lista é >>>>========================================================================= >>> >>> >>> >>> >>>_________________________________________________________________ >>>Converse com amigos on-line, conheça o MSN Messenger: >>>http://messenger.msn.com >>> >>>========================================================================= >>>Instruções para entrar na lista, sair da lista e usar a lista em >>>http://www.mat.puc-rio.br/~nicolau/olimp/obm-l.html >>>O administrador desta lista é >>>========================================================================= >> >> >>_________________________________________________________________ >>Chegou o novo MSN Explorer. Instale já. É gratuito: >>http://explorer.msn.com.br >> >>========================================================================= >>Instruções para entrar na lista, sair da lista e usar a lista em >>http://www.mat.puc-rio.br/~nicolau/olimp/obm-l.html >>O administrador desta lista é >>========================================================================= > > >_________________________________________________________________ >Chegou o novo MSN Explorer. Instale já. É gratuito: >http://explorer.msn.com.br > >========================================================================= >Instruções para entrar na lista, sair da lista e usar a lista em >http://www.mat.puc-rio.br/~nicolau/olimp/obm-l.html >O administrador desta lista é >========================================================================= _________________________________________________________________ Converse com amigos on-line, conheça o MSN Messenger: http://messenger.msn.com ========================================================================= Instruções para entrar na lista, sair da lista e usar a lista em http://www.mat.puc-rio.br/~nicolau/olimp/obm-l.html O administrador desta lista é ========================================================================= From owner-obm-l@sucuri.mat.puc-rio.br Tue May 7 13:23:42 2002 Return-Path: Received: (from majordom@localhost) by sucuri.mat.puc-rio.br (8.9.3/8.9.3) id NAA24086 for obm-l-list; Tue, 7 May 2002 13:22:07 -0300 Received: from hotmail.com (law2-f25.hotmail.com [216.32.181.25]) by sucuri.mat.puc-rio.br (8.9.3/8.9.3) with ESMTP id NAA24082 for ; Tue, 7 May 2002 13:22:04 -0300 Received: from mail pickup service by hotmail.com with Microsoft SMTPSVC; Tue, 7 May 2002 09:04:44 -0700 Received: from 200.229.244.100 by lw2fd.hotmail.msn.com with HTTP; Tue, 07 May 2002 16:04:43 GMT X-Originating-IP: [200.229.244.100] From: "Paulo Santa Rita" To: obm-l@mat.puc-rio.br Subject: Re: [obm-l] Re: [obm-l] Provar desigualdade. Date: Tue, 07 May 2002 16:04:43 +0000 Mime-Version: 1.0 Content-Type: text/plain; charset=iso-8859-1; format=flowed Message-ID: X-OriginalArrivalTime: 07 May 2002 16:04:44.0171 (UTC) FILETIME=[E7188DB0:01C1F5E0] Sender: owner-obm-l@sucuri.mat.puc-rio.br Precedence: bulk Reply-To: obm-l@mat.puc-rio.br Ola Thiago, Bonita a sua solucao. Acho mesmo a mais bonita. Mas, fica o problema : Voce nao pode ensinar assim pra um aluno de 7 serie ... Como fazer ? Eu, confesso, que nao sei. Um abraco Paulo Santa Rita 3,1303,070502 >From: ciceroth@zipmail.com.br >Reply-To: obm-l@mat.puc-rio.br >To: obm-l@mat.puc-rio.br >Subject: [obm-l] Re: [obm-l] Provar desigualdade. >Date: Tue, 7 May 2002 00:17:53 -0300 > > > >como a + b = 1, usando MA => MG, temos >2a + b + b/3 =>(2ab²)^1/3 >2/3=>(2ab²)^1/3 <=> 8/27=>2ab² <=> ab²<=4/27 > >um abraço Cicero Thiago >Use o melhor sistema de busca da Internet >Radar UOL - http://www.radaruol.com.br > > > >========================================================================= >Instruções para entrar na lista, sair da lista e usar a lista em >http://www.mat.puc-rio.br/~nicolau/olimp/obm-l.html >O administrador desta lista é >========================================================================= _________________________________________________________________ Envie e receba emails com o Hotmail no seu dispositivo móvel: http://mobile.msn.com ========================================================================= Instruções para entrar na lista, sair da lista e usar a lista em http://www.mat.puc-rio.br/~nicolau/olimp/obm-l.html O administrador desta lista é ========================================================================= From owner-obm-l@sucuri.mat.puc-rio.br Tue May 7 13:49:10 2002 Return-Path: Received: (from majordom@localhost) by sucuri.mat.puc-rio.br (8.9.3/8.9.3) id NAA24770 for obm-l-list; Tue, 7 May 2002 13:47:11 -0300 Received: from fgvrj23.fgv.br (fgvrj23.fgv.br [200.20.164.23]) by sucuri.mat.puc-rio.br (8.9.3/8.9.3) with ESMTP id NAA24766 for ; Tue, 7 May 2002 13:47:09 -0300 Received: from E01055 ([10.250.4.147]) by fgvrj23.fgv.br with SMTP (Microsoft Exchange Internet Mail Service Version 5.5.2653.13) id 20VA4SWX; Tue, 7 May 2002 13:51:20 -0300 Message-ID: <001901c1f5e7$02359100$9304fa0a@fgv.br> From: "Ralph Teixeira" To: References: Subject: Re: [obm-l] Re: [obm-l] Provar desigualdade. Date: Tue, 7 May 2002 13:48:26 -0300 MIME-Version: 1.0 Content-Type: text/plain; charset="iso-8859-1" Content-Transfer-Encoding: 8bit X-Priority: 3 X-MSMail-Priority: Normal X-Mailer: Microsoft Outlook Express 6.00.2600.0000 X-MimeOLE: Produced By Microsoft MimeOLE V6.00.2600.0000 Sender: owner-obm-l@sucuri.mat.puc-rio.br Precedence: bulk Reply-To: obm-l@mat.puc-rio.br Bom, dá para provar MA >= MG para o caso particular de três números como o pessoal fez por aqui, usando uma fatoração mágica acessível ao pessoal de 7a série. Eu não disse que é fácil para a 7a série, pois não é, mas é *possível* se feito com carinho e vários exemplos numéricos. Fora isso, só se você for direto na álgebra, tipo: a=1-b então S=4/27-ab^2 = 4/27-(1-b)b^2 =4/27-b^2+b^3 Quero mostrar que S=b^3-b^2+4/27 >= 0 para qualquer b entre 0 e 1, certo? Procuro uma raiz.... Tento b=1/3 ou b=2/3, já de olho no 27 no denominador... Faço as contas, descubro que b=2/3 é raiz (o que era de se esperar, pois tomando a=1/3 e b=2/3 tem-se 1/3.2/3.2/3 = 4/27 certinho)... Então faço: S=(b-2/3)(b^2-b/3-2/9) Quero fatorar o outro termo.... Ih, resolvendo a quadrática descubro que 2/3 é raiz de novo.... Fica assim: S=(b-2/3)(b-2/3)(b+1/3) S=(b+1/3)(b-2/3)^2 Bom, como 0<=b<=1, claramente o primeiro termo eh positivo e o segundo eh nao-negativo. Assim, S>=0, com a igualdade acontecendo somente para b=2/3. Que tal? Abraço, Ralph ----- Original Message ----- From: "Paulo Santa Rita" To: Sent: Tuesday, May 07, 2002 1:04 PM Subject: Re: [obm-l] Re: [obm-l] Provar desigualdade. Ola Thiago, Bonita a sua solucao. Acho mesmo a mais bonita. Mas, fica o problema : Voce nao pode ensinar assim pra um aluno de 7 serie ... Como fazer ? Eu, confesso, que nao sei. Um abraco Paulo Santa Rita 3,1303,070502 >From: ciceroth@zipmail.com.br >Reply-To: obm-l@mat.puc-rio.br >To: obm-l@mat.puc-rio.br >Subject: [obm-l] Re: [obm-l] Provar desigualdade. >Date: Tue, 7 May 2002 00:17:53 -0300 > > > >como a + b = 1, usando MA => MG, temos >2a + b + b/3 =>(2ab²)^1/3 >2/3=>(2ab²)^1/3 <=> 8/27=>2ab² <=> ab²<=4/27 > >um abraço Cicero Thiago >Use o melhor sistema de busca da Internet >Radar UOL - http://www.radaruol.com.br > > > >========================================================================= >Instruções para entrar na lista, sair da lista e usar a lista em >http://www.mat.puc-rio.br/~nicolau/olimp/obm-l.html >O administrador desta lista é >========================================================================= _________________________________________________________________ Envie e receba emails com o Hotmail no seu dispositivo móvel: http://mobile.msn.com ========================================================================= Instruções para entrar na lista, sair da lista e usar a lista em http://www.mat.puc-rio.br/~nicolau/olimp/obm-l.html O administrador desta lista é ========================================================================= ========================================================================= Instruções para entrar na lista, sair da lista e usar a lista em http://www.mat.puc-rio.br/~nicolau/olimp/obm-l.html O administrador desta lista é ========================================================================= From owner-obm-l@sucuri.mat.puc-rio.br Tue May 7 13:52:48 2002 Return-Path: Received: (from majordom@localhost) by sucuri.mat.puc-rio.br (8.9.3/8.9.3) id NAA24885 for obm-l-list; Tue, 7 May 2002 13:52:46 -0300 Received: from www.zipmail.com.br (smtp.zipmail.com.br [200.187.242.10]) by sucuri.mat.puc-rio.br (8.9.3/8.9.3) with ESMTP id NAA24881 for ; Tue, 7 May 2002 13:52:42 -0300 From: ciceroth@zipmail.com.br Received: from [200.253.207.2] by www.zipmail.com.br with HTTP; Tue, 7 May 2002 13:53:58 -0300 Message-ID: <3CD7FB2E00000714@www.zipmail.com.br> Date: Tue, 7 May 2002 13:53:58 -0300 In-Reply-To: Subject: [obm-l] =?iso-8859-1?Q?Re=3A=20=5Bobm=2Dl=5D=20bibliografia=20indicada=20IME/ITA?= To: obm-l@mat.puc-rio.br MIME-Version: 1.0 Content-Type: text/plain; charset="iso-8859-1" Content-Transfer-Encoding: 8bit X-MIME-Autoconverted: from quoted-printable to 8bit by sucuri.mat.puc-rio.br id NAA24882 Sender: owner-obm-l@sucuri.mat.puc-rio.br Precedence: bulk Reply-To: obm-l@mat.puc-rio.br na minha opinião bons livros de quimica pro ITA e IME são: (irei indicar o nome dos autores) Kotz (preferido) Solomons Peter Atkins apesar de não ter prestado vestibular para o ITA e nem para o IME estive muito em contato com estes livros pois tenho muitos amigos tanto no ITA quanto no IME e estes eram o os livros preferidos. abraço Cicero Thiago ------------------------------------------ Use o melhor sistema de busca da Internet Radar UOL - http://www.radaruol.com.br ========================================================================= Instruções para entrar na lista, sair da lista e usar a lista em http://www.mat.puc-rio.br/~nicolau/olimp/obm-l.html O administrador desta lista é ========================================================================= From owner-obm-l@sucuri.mat.puc-rio.br Tue May 7 16:07:04 2002 Return-Path: Received: (from majordom@localhost) by sucuri.mat.puc-rio.br (8.9.3/8.9.3) id QAA27588 for obm-l-list; Tue, 7 May 2002 16:05:54 -0300 Received: from hotmail.com (law2-f79.hotmail.com [216.32.181.79]) by sucuri.mat.puc-rio.br (8.9.3/8.9.3) with ESMTP id QAA27581 for ; Tue, 7 May 2002 16:05:44 -0300 Received: from mail pickup service by hotmail.com with Microsoft SMTPSVC; Tue, 7 May 2002 12:06:01 -0700 Received: from 200.229.244.100 by lw2fd.hotmail.msn.com with HTTP; Tue, 07 May 2002 19:06:00 GMT X-Originating-IP: [200.229.244.100] From: "Paulo Santa Rita" To: obm-l@mat.puc-rio.br Subject: Re: [obm-l] Re: [obm-l] Provar desigualdade. Date: Tue, 07 May 2002 19:06:00 +0000 Mime-Version: 1.0 Content-Type: text/plain; charset=iso-8859-1; format=flowed Message-ID: X-OriginalArrivalTime: 07 May 2002 19:06:01.0046 (UTC) FILETIME=[3A37F760:01C1F5FA] Sender: owner-obm-l@sucuri.mat.puc-rio.br Precedence: bulk Reply-To: obm-l@mat.puc-rio.br Legal, muito legal mesmo ! E se o Prof antes fizer umas contas para motivar os(as) garotos(as), transformando a busca da prova em uma verdadeira batalha, deve ficar mais emocionante ainda ... A proposito. Neste sentido, lembro que no ano passado eu andava lendo umas pesquisas historicas sobre o egito antigo e, em particular, sobre os Keri-Hari. Eles constituiam uma Casta Superior, so se admitindo em sua Ordem os melhores individuos da epoca ( Melhores do ponto de vista deles, logico ). O Historiador deixa claro que acredita que esta casta tinha, efetivamente, alguma forma de conhecimento secreto e superior porque eles se antecipavam aos fatos culturais que iriam ocorrer em dezenas de anos. Todos eram Instrutores e/ou Academicos( Professores ). O ensino interno dos Keri-Hari era iniciatico ... ????? !!!!! Iniciático : Eles primeiro condicionavam as pessoas EMOCIONALMENTE, fazendo-as passar por uma encenacao teatral terrivel. So depois de mexer(preparar) o emocional eles passavam a ministrar os cursos intelectuais propriamente ditos. E antes de cada etapa intelectual havia uma iniciacao ou preparacao emocional. Por outro lado, o ensino intelectual era meramente fagulhar, vale dizer, o Mestre dava as bases e o emocional do aluno, ja preparado, fazia com que o proprio aluno descobrisse a maior parte das coisas. Um abraco Paulo Santa Rita 3,1604,070502 >From: "Ralph Teixeira" >Reply-To: obm-l@mat.puc-rio.br >To: >Subject: Re: [obm-l] Re: [obm-l] Provar desigualdade. >Date: Tue, 7 May 2002 13:48:26 -0300 > > Bom, dá para provar MA >= MG para o caso particular de três números >como >o pessoal fez por aqui, usando uma fatoração mágica acessível ao pessoal de >7a série. Eu não disse que é fácil para a 7a série, pois não é, mas é >*possível* se feito com carinho e vários exemplos numéricos. > > Fora isso, só se você for direto na álgebra, tipo: > > a=1-b então S=4/27-ab^2 = 4/27-(1-b)b^2 =4/27-b^2+b^3 > > Quero mostrar que S=b^3-b^2+4/27 >= 0 para qualquer b entre 0 e 1, >certo? Procuro uma raiz.... Tento b=1/3 ou b=2/3, já de olho no 27 no >denominador... Faço as contas, descubro que b=2/3 é raiz (o que era de se >esperar, pois tomando a=1/3 e b=2/3 tem-se 1/3.2/3.2/3 = 4/27 certinho)... >Então faço: > > S=(b-2/3)(b^2-b/3-2/9) > > Quero fatorar o outro termo.... Ih, resolvendo a quadrática descubro >que >2/3 é raiz de novo.... Fica assim: > > S=(b-2/3)(b-2/3)(b+1/3) > > S=(b+1/3)(b-2/3)^2 > > Bom, como 0<=b<=1, claramente o primeiro termo eh positivo e o segundo >eh nao-negativo. Assim, S>=0, com a igualdade acontecendo somente para >b=2/3. > > Que tal? > > Abraço, > Ralph > >----- Original Message ----- >From: "Paulo Santa Rita" >To: >Sent: Tuesday, May 07, 2002 1:04 PM >Subject: Re: [obm-l] Re: [obm-l] Provar desigualdade. > > >Ola Thiago, > >Bonita a sua solucao. Acho mesmo a mais bonita. Mas, fica o problema : Voce >nao pode ensinar assim pra um aluno de 7 serie ... Como fazer ? Eu, >confesso, que nao sei. > >Um abraco >Paulo Santa Rita >3,1303,070502 > > > >From: ciceroth@zipmail.com.br > >Reply-To: obm-l@mat.puc-rio.br > >To: obm-l@mat.puc-rio.br > >Subject: [obm-l] Re: [obm-l] Provar desigualdade. > >Date: Tue, 7 May 2002 00:17:53 -0300 > > > > > > > >como a + b = 1, usando MA => MG, temos > >2a + b + b/3 =>(2ab²)^1/3 > >2/3=>(2ab²)^1/3 <=> 8/27=>2ab² <=> ab²<=4/27 > > > >um abraço Cicero Thiago > >Use o melhor sistema de busca da Internet > >Radar UOL - http://www.radaruol.com.br > > > > > > > >========================================================================= > >Instruções para entrar na lista, sair da lista e usar a lista em > >http://www.mat.puc-rio.br/~nicolau/olimp/obm-l.html > >O administrador desta lista é > >========================================================================= > > > > >_________________________________________________________________ >Envie e receba emails com o Hotmail no seu dispositivo móvel: >http://mobile.msn.com > >========================================================================= >Instruções para entrar na lista, sair da lista e usar a lista em >http://www.mat.puc-rio.br/~nicolau/olimp/obm-l.html >O administrador desta lista é >========================================================================= > > >========================================================================= >Instruções para entrar na lista, sair da lista e usar a lista em >http://www.mat.puc-rio.br/~nicolau/olimp/obm-l.html >O administrador desta lista é >========================================================================= _________________________________________________________________ Una-se ao maior serviço de email do mundo: o MSN Hotmail. http://www.hotmail.com ========================================================================= Instruções para entrar na lista, sair da lista e usar a lista em http://www.mat.puc-rio.br/~nicolau/olimp/obm-l.html O administrador desta lista é ========================================================================= From owner-obm-l@sucuri.mat.puc-rio.br Tue May 7 22:52:14 2002 Return-Path: Received: (from majordom@localhost) by sucuri.mat.puc-rio.br (8.9.3/8.9.3) id WAA03816 for obm-l-list; Tue, 7 May 2002 22:51:57 -0300 Received: from web13301.mail.yahoo.com (web13301.mail.yahoo.com [216.136.175.37]) by sucuri.mat.puc-rio.br (8.9.3/8.9.3) with SMTP id WAA03812 for ; Tue, 7 May 2002 22:51:54 -0300 Message-ID: <20020508014010.42144.qmail@web13301.mail.yahoo.com> Received: from [200.190.8.68] by web13301.mail.yahoo.com via HTTP; Tue, 07 May 2002 18:40:10 PDT Date: Tue, 7 May 2002 18:40:10 -0700 (PDT) From: Carlos Yuzo Shine Subject: [obm-l] Website e curso da OPM To: teoremalista@yahoogrupos.com.br, ocmlista@yahoogrupos.com.br, obm-l@mat.puc-rio.br MIME-Version: 1.0 Content-Type: text/plain; charset=us-ascii Sender: owner-obm-l@sucuri.mat.puc-rio.br Precedence: bulk Reply-To: obm-l@mat.puc-rio.br Olá a todos! O novo e moderno site da OPM já está no ar. Chequem em http://www.opm.mat.br/ Lá tem, entre outras informações, a história do símbolo da OPM: o grafo de Petersen. Para quem está pertinho de São Paulo: E além disso, a OPM está oferecendo mais um curso de aperfeiçoamento em Matemática, e desta vez o tema será Análise Combinatória. Este curso é direcionado para professores e estudantes. Chequem em http://www.opm.mat.br/ []'s Shine __________________________________________________ Do You Yahoo!? Yahoo! Health - your guide to health and wellness http://health.yahoo.com ========================================================================= Instruções para entrar na lista, sair da lista e usar a lista em http://www.mat.puc-rio.br/~nicolau/olimp/obm-l.html O administrador desta lista é ========================================================================= From owner-obm-l@sucuri.mat.puc-rio.br Tue May 7 23:54:58 2002 Return-Path: Received: (from majordom@localhost) by sucuri.mat.puc-rio.br (8.9.3/8.9.3) id XAA04618 for obm-l-list; Tue, 7 May 2002 23:54:38 -0300 Received: from escelsa.com.br (svcoc05.escelsanet.com.br [200.242.30.218]) by sucuri.mat.puc-rio.br (8.9.3/8.9.3) with ESMTP id XAA04614 for ; Tue, 7 May 2002 23:54:36 -0300 Received: from smdksbcr ([200.242.14.35]) by escelsa.com.br ; Tue, 07 May 2002 23:42:52 -0300 Message-ID: <004301c1f63a$9199db00$230ef2c8@smdksbcr> From: "Rodrigo Freitas" To: Subject: [obm-l] Cilindro reto Date: Tue, 7 May 2002 23:46:31 -0300 MIME-Version: 1.0 Content-Type: multipart/alternative; boundary="----=_NextPart_000_0040_01C1F621.6A5E4080" X-Priority: 3 X-MSMail-Priority: Normal X-Mailer: Microsoft Outlook Express 5.50.4133.2400 X-MimeOLE: Produced By Microsoft MimeOLE V5.50.4133.2400 Sender: owner-obm-l@sucuri.mat.puc-rio.br Precedence: bulk Reply-To: obm-l@mat.puc-rio.br This is a multi-part message in MIME format. ------=_NextPart_000_0040_01C1F621.6A5E4080 Content-Type: text/plain; charset="iso-8859-1" Content-Transfer-Encoding: quoted-printable Ol=E1 pessoal! Eu encontrei um problema e gostaria que algu=E9m me ajudasse se = poss=EDvel. Pede-se para provar que um cilindro reto de determinado volume = possui =E1rea superficial m=EDnima quando a sua altura =E9 igual ao = di=E2metro de sua base. (Halliday, vol.1, cap.1) Eu cheguei a comentar com um professor e ele me disse que conseguria = fazer por c=E1lculos n=E3o compat=EDveis com o segundo grau. Algu=E9m = consegue resolver usando mat=E9rias do segundo grau? Se n=E3o, e se = n=E3o for muito dif=EDcil, pediria pra responderem mesmo usando = c=E1lculo integral. Obrigado! ------=_NextPart_000_0040_01C1F621.6A5E4080 Content-Type: text/html; charset="iso-8859-1" Content-Transfer-Encoding: quoted-printable
Ol=E1 pessoal!
 
 Eu encontrei um problema e = gostaria que=20 algu=E9m me ajudasse se poss=EDvel.
 
    Pede-se para provar = que um=20 cilindro reto de determinado volume possui =E1rea superficial m=EDnima = quando a sua=20 altura =E9 igual ao di=E2metro de sua base. (Halliday, vol.1, = cap.1)
 
Eu cheguei a comentar com um professor = e ele me=20 disse que conseguria fazer por c=E1lculos n=E3o compat=EDveis com o = segundo grau.=20 Algu=E9m consegue resolver usando mat=E9rias do segundo grau? Se n=E3o, = e se n=E3o for=20 muito dif=EDcil, pediria pra responderem mesmo usando c=E1lculo=20 integral.
 
   =20 Obrigado!
------=_NextPart_000_0040_01C1F621.6A5E4080-- ========================================================================= Instruções para entrar na lista, sair da lista e usar a lista em http://www.mat.puc-rio.br/~nicolau/olimp/obm-l.html O administrador desta lista é ========================================================================= From owner-obm-l@sucuri.mat.puc-rio.br Wed May 8 00:40:39 2002 Return-Path: Received: (from majordom@localhost) by sucuri.mat.puc-rio.br (8.9.3/8.9.3) id AAA05253 for obm-l-list; Wed, 8 May 2002 00:40:06 -0300 Received: from hotmail.com (f41.pav1.hotmail.com [64.4.31.41]) by sucuri.mat.puc-rio.br (8.9.3/8.9.3) with ESMTP id AAA05249 for ; Wed, 8 May 2002 00:40:04 -0300 Received: from mail pickup service by hotmail.com with Microsoft SMTPSVC; Tue, 7 May 2002 20:28:20 -0700 Received: from 200.151.12.115 by pv1fd.pav1.hotmail.msn.com with HTTP; Wed, 08 May 2002 03:28:20 GMT X-Originating-IP: [200.151.12.115] From: "Adriano Almeida Faustino" To: obm-l@mat.puc-rio.br Subject: [obm-l] =?iso-8859-1?B?UmU6IFtvYm0tbF0gUmU6W29ibS1sXSBSZTogW29ibS1sXSBk+nZpZGFz?= Date: Wed, 08 May 2002 03:28:20 +0000 Mime-Version: 1.0 Content-Type: text/plain; charset=iso-8859-1; format=flowed Message-ID: X-OriginalArrivalTime: 08 May 2002 03:28:20.0756 (UTC) FILETIME=[66E2BD40:01C1F640] Sender: owner-obm-l@sucuri.mat.puc-rio.br Precedence: bulk Reply-To: obm-l@mat.puc-rio.br Não. 1º-Eu achei a questão mais difícil,a questão do grafo (questão nº7,dos caminhos).Essa questão das médias foi "difícil",porque ninguém estava esperando que caísse esse tipo de questão,e praticamente ninguém ensinou. 2º-Não acho essa parte a mais difícil da Matemática,acho Geometria(Sintética) ,porque sou completamente "cego".Eu só conheço algumas coisinhas,como Teorema de Pitágoras (e olhe lá).Prefiro fazer os problemas por Geometria Analítica (quando consigo),mas muitas vezes é muito trabalhoso. []´s. Adriano. >From: "rafaelc.l" >Reply-To: obm-l@mat.puc-rio.br >To: obm-l@mat.puc-rio.br >Subject: [obm-l] Re:[obm-l] Re: [obm-l] dúvidas >Date: Sun, 5 May 2002 17:15:03 -0300 > > >Interessante vc ter se baseado na letra A para fazer a >letra B..não é a toa que elas estão na mesma questão. > Não sei se vc concorda comigo, mas essa questão foi pra >mim a mais difícil da prova e esse tipo de questão que >chamam de "álgebra especulativa" eu acho realmente mais >complicado que o resto em matemática. Vc concorda? > > >__________________________________________________________________________ >Quer ter seu próprio endereço na Internet? >Garanta já o seu e ainda ganhe cinco e-mails personalizados. >DomíniosBOL - http://dominios.bol.com.br > > >========================================================================= >Instruções para entrar na lista, sair da lista e usar a lista em >http://www.mat.puc-rio.br/~nicolau/olimp/obm-l.html >O administrador desta lista é >========================================================================= _________________________________________________________________ Chegou o novo MSN Explorer. Instale já. É gratuito: http://explorer.msn.com.br ========================================================================= Instruções para entrar na lista, sair da lista e usar a lista em http://www.mat.puc-rio.br/~nicolau/olimp/obm-l.html O administrador desta lista é ========================================================================= From owner-obm-l@sucuri.mat.puc-rio.br Wed May 8 00:44:40 2002 Return-Path: Received: (from majordom@localhost) by sucuri.mat.puc-rio.br (8.9.3/8.9.3) id AAA05340 for obm-l-list; Wed, 8 May 2002 00:44:35 -0300 Received: from bidu.ime.usp.br (bidu.ime.usp.br [143.107.45.12]) by sucuri.mat.puc-rio.br (8.9.3/8.9.3) with SMTP id AAA05336 for ; Wed, 8 May 2002 00:44:33 -0300 Received: (qmail 4095 invoked from network); 8 May 2002 03:32:51 -0000 Received: from mafalda.ime.usp.br (143.107.45.13) by bidu.ime.usp.br with SMTP; 8 May 2002 03:32:51 -0000 Received: (qmail 28857 invoked by uid 1604); 8 May 2002 03:32:50 -0000 Date: Wed, 8 May 2002 00:32:49 -0300 (EST) From: Salvador Addas Zanata X-Sender: sazanata@mafalda To: obm-l@mat.puc-rio.br Subject: Re: [obm-l] Cilindro reto In-Reply-To: <004301c1f63a$9199db00$230ef2c8@smdksbcr> Message-ID: MIME-Version: 1.0 Content-Type: TEXT/PLAIN; charset=ISO-8859-1 Content-Transfer-Encoding: 8bit X-MIME-Autoconverted: from QUOTED-PRINTABLE to 8bit by sucuri.mat.puc-rio.br id AAA05337 Sender: owner-obm-l@sucuri.mat.puc-rio.br Precedence: bulk Reply-To: obm-l@mat.puc-rio.br V=pi.r^2.h (1) A=2.pi.r.h+2.pi.r^2 (2) Subst. (1) em (2), A=2.V/r+2.pi.r^2 Agora voce tem que achar o min. dessa funcao, supondo V constante. Quando r fica pequeno, prox. de 0, A estoura, o mesmo ocorrendo pra r muito grande, logo existe o minimo. A area A e uma funcao de r, entao um dos jeitos de achar esse minimo e derivando com rel. a r e igualando a zero: dA/dr=-2.V/r^2+4.pi.r=0 => V=pi.r^2.(2r), logo a altura e 2r. Abraco, Salvador On Tue, 7 May 2002, Rodrigo Freitas wrote: > Olá pessoal! > > Eu encontrei um problema e gostaria que alguém me ajudasse se possível. > > Pede-se para provar que um cilindro reto de determinado volume possui área superficial mínima quando a sua altura é igual ao diâmetro de sua base. (Halliday, vol.1, cap.1) > > Eu cheguei a comentar com um professor e ele me disse que conseguria fazer por cálculos não compatíveis com o segundo grau. Alguém consegue resolver usando matérias do segundo grau? Se não, e se não for muito difícil, pediria pra responderem mesmo usando cálculo integral. > > Obrigado! > ========================================================================= Instruções para entrar na lista, sair da lista e usar a lista em http://www.mat.puc-rio.br/~nicolau/olimp/obm-l.html O administrador desta lista é ========================================================================= From owner-obm-l@sucuri.mat.puc-rio.br Wed May 8 02:22:02 2002 Return-Path: Received: (from majordom@localhost) by sucuri.mat.puc-rio.br (8.9.3/8.9.3) id CAA06702 for obm-l-list; Wed, 8 May 2002 02:21:52 -0300 Received: from sr2.terra.com.br (sr2.terra.com.br [200.176.3.17]) by sucuri.mat.puc-rio.br (8.9.3/8.9.3) with ESMTP id CAA06698 for ; Wed, 8 May 2002 02:21:50 -0300 Received: from srv11-sao.terra.com.br (srv11-sao.terra.com.br [200.176.3.38]) by sr2.terra.com.br (Postfix) with ESMTP id B7BF0931AC for ; Wed, 8 May 2002 02:10:08 -0300 (EST) Received: from terra.com.br (unknown [200.206.243.51]) (authenticated user lponce) by srv11-sao.terra.com.br (Postfix) with ESMTP id 71EABC94C3 for ; Wed, 8 May 2002 02:10:08 -0300 (EST) Message-ID: <3CD8B300.E8288C63@terra.com.br> Date: Wed, 08 May 2002 02:09:21 -0300 From: Luiz Antonio Ponce Alonso X-Mailer: Mozilla 4.79 [en] (Windows NT 5.0; U) X-Accept-Language: en MIME-Version: 1.0 To: obm-l@mat.puc-rio.br Subject: [obm-l] Um =?iso-8859-1?Q?endere=E7o?= de interesse de todos Content-Type: text/plain; charset=iso-8859-1 Content-Transfer-Encoding: 8bit Sender: owner-obm-l@sucuri.mat.puc-rio.br Precedence: bulk Reply-To: obm-l@mat.puc-rio.br Aos grandes amigos desta lista, Faço parte de outra lista, na qual temos um arquivo com muito material de olimpiadas. Neste material você encontrará varias shortlist da IMO inclusive de 2001 vale a pena conferir. O que o Nicolau acha de aproveitar algumas e colocar em sua homepage ?? O endereço é http://groups.yahoo.com/group/imo-problems/files/ Um abraço e bom proveito PONCE ========================================================================= Instruções para entrar na lista, sair da lista e usar a lista em http://www.mat.puc-rio.br/~nicolau/olimp/obm-l.html O administrador desta lista é ========================================================================= From owner-obm-l@sucuri.mat.puc-rio.br Wed May 8 11:54:21 2002 Return-Path: Received: (from majordom@localhost) by sucuri.mat.puc-rio.br (8.9.3/8.9.3) id LAA11258 for obm-l-list; Wed, 8 May 2002 11:52:03 -0300 Received: from www.zipmail.com.br (smtp.zipmail.com.br [200.187.242.10]) by sucuri.mat.puc-rio.br (8.9.3/8.9.3) with ESMTP id LAA11254 for ; Wed, 8 May 2002 11:52:01 -0300 From: ciceroth@zipmail.com.br Received: from [200.253.207.2] by www.zipmail.com.br with HTTP; Wed, 8 May 2002 11:40:59 -0300 Message-ID: <3CD7FB2E0000622C@www.zipmail.com.br> Date: Wed, 8 May 2002 11:40:59 -0300 In-Reply-To: Subject: [obm-l] =?iso-8859-1?Q?Re=3A=20=5Bobm=2Dl=5D=20Re=3A=20=5Bobm=2Dl=5D=20Provar=20desigualdade=2E?= To: obm-l@mat.puc-rio.br MIME-Version: 1.0 Content-Type: text/plain; charset="iso-8859-1" Content-Transfer-Encoding: 8bit X-MIME-Autoconverted: from quoted-printable to 8bit by sucuri.mat.puc-rio.br id LAA11255 Sender: owner-obm-l@sucuri.mat.puc-rio.br Precedence: bulk Reply-To: obm-l@mat.puc-rio.br olá Paulo Esse assunto de medias me foi ensinado quando eu estava exatamente na setima serie( nas minhas aulas de olimpiada). Na minha opinoão essa eh a forma mais facil de se resolver para um aluno de setima serie. abraço Cicero Thiago ------------------------------------------ Use o melhor sistema de busca da Internet Radar UOL - http://www.radaruol.com.br ========================================================================= Instruções para entrar na lista, sair da lista e usar a lista em http://www.mat.puc-rio.br/~nicolau/olimp/obm-l.html O administrador desta lista é ========================================================================= From owner-obm-l@sucuri.mat.puc-rio.br Wed May 8 12:43:01 2002 Return-Path: Received: (from majordom@localhost) by sucuri.mat.puc-rio.br (8.9.3/8.9.3) id MAA12151 for obm-l-list; Wed, 8 May 2002 12:42:17 -0300 Received: from hotmail.com (f199.pav1.hotmail.com [64.4.31.199]) by sucuri.mat.puc-rio.br (8.9.3/8.9.3) with ESMTP id MAA12146 for ; Wed, 8 May 2002 12:42:14 -0300 Received: from mail pickup service by hotmail.com with Microsoft SMTPSVC; Wed, 8 May 2002 08:30:33 -0700 Received: from 200.242.76.12 by pv1fd.pav1.hotmail.msn.com with HTTP; Wed, 08 May 2002 15:30:32 GMT X-Originating-IP: [200.242.76.12] From: "Felipe Marinho" To: obm-l@mat.puc-rio.br Subject: [obm-l] =?iso-8859-1?B?QW5hbGlzZW0gaXN0bywgc2UgcG9zc+12ZWwu?= Date: Wed, 08 May 2002 11:30:32 -0400 Mime-Version: 1.0 Content-Type: text/plain; charset=iso-8859-1; format=flowed Message-ID: X-OriginalArrivalTime: 08 May 2002 15:30:33.0231 (UTC) FILETIME=[4B0DB1F0:01C1F6A5] Sender: owner-obm-l@sucuri.mat.puc-rio.br Precedence: bulk Reply-To: obm-l@mat.puc-rio.br Olá mais uma vez amigos da lista, Um grande abraço especial para o Paulo, Tiago e Ralph... que me ajudaram na questão da desigualdade. A ajuda de você foi de grande importância aqui para mim. Agora é o seguinte, peço que vocês analisem a minha resolução para um tal exercício... se tiver algum erro, ou alguma outra solução mais rápida e inteligente... gostaria que vocês me indicassem. Lá vai: Seja a, b e c números reais positivos, provar que: P = a/(b+c) + b/(a+c) + c(a+b) >= 3/2 ----Resolução---- De MA >= MG, temos: [a/(b+c) + b(a+c) + c(a+b)] /3 >= [abc/(b+c).(a+c).(a+b)]^(1/3) Como P = a/(b+c) + b/(a+c) + c(a+b), temos: P >= 3[abc/(b+c).(a+c).(a+b)]^(1/3) (I) (a+b+c)/3 >= (abc)^(1/3) (II) [(b+c) + (a+c) + (a+b)]/3 >= [(b+c).(a+c).(a+b)]^(1/3) [2(a+b+c)]/3 >= [(b+c).(a+c).(a+b)]^(1/3) (III) Multiplicando (II) por 2, temos: 2(a+b+c)/3 >= 2(abc)^(1/3) (IV) Analisando (III) e (IV), temos: 2(a+b+c)/3 >= [(b+c).(a+c).(a+b)]^(1/3) 2(a+b+c)/3 >= 2(abc)^(1/3) ----- Bem, até aí, sem problemas, porem aqui começa a minha dúvida. Para finalizar o exercício, o único passo que consegui dar, foi achar uma igualdade entre as 2 expressões do lado direito das 2 desigualdades. Basei-me no fato de que, se 2(a+b+c)/3 é maior ou igual a [(b+c).(a+c).(a+b)]^(1/3) e tambem 2(a+b+c)/3 é maior ou igual a 2(abc)^(1/3) é porque [(b+c).(a+c).(a+b)]^(1/3) = 2(abc)^(1/3). Baseando-se nesta igualdade, fica fácil provar que P >= 3/2, pois: Como P >= 3[abc/(b+c).(a+c).(a+b)]^(1/3), teríamos: P >= 3(abc)^(1/3) x [(1/(b+c).(a+c).(a+b)]^(1/3) E substituindo os valores achados, teríamos: P>= [3(abc)^(1/3)] / [2(abc)^(1/3)] e cancelando o termo em (abc)^(1/3), temos: P>= 3/2 (cqd) -----FIM------ Bem pessoal, a questão é essa aí. Foi a única maneira que achei para resolvê-la. Desculpe se não a resolvi usando algo melhor. Porem, a minha pergunta aqui, é a seguinte: Essa igualdade que eu achei entre as 2 expressões, eu realmente poderia afirmar isto ? Eu errei em algum procedimento tomado para solucionar o problema ? Existe algum método mais rápido ou inteligente ? Obrigado desde já, Conto com a ajudade de vocês mais uma vez. Um grande abraço a todos os amigos, Felipe Marinho _________________________________________________________________ Chegou o novo MSN Explorer. Instale já. É gratuito: http://explorer.msn.com.br ========================================================================= Instruções para entrar na lista, sair da lista e usar a lista em http://www.mat.puc-rio.br/~nicolau/olimp/obm-l.html O administrador desta lista é ========================================================================= From owner-obm-l@sucuri.mat.puc-rio.br Wed May 8 14:28:38 2002 Return-Path: Received: (from majordom@localhost) by sucuri.mat.puc-rio.br (8.9.3/8.9.3) id OAA14167 for obm-l-list; Wed, 8 May 2002 14:25:34 -0300 Received: from www.zipmail.com.br (smtp.zipmail.com.br [200.187.242.10]) by sucuri.mat.puc-rio.br (8.9.3/8.9.3) with ESMTP id OAA14163 for ; Wed, 8 May 2002 14:25:32 -0300 From: peterdirichlet@zipmail.com.br Received: from [200.206.103.3] by www.zipmail.com.br with HTTP; Wed, 8 May 2002 14:13:09 -0300 Message-ID: <3CD7FA3F000060BC@www.zipmail.com.br> Date: Wed, 8 May 2002 14:13:09 -0300 Subject: [obm-l] =?iso-8859-1?Q?Re=3A=20=5Bobm=2Dl=5D=20Analisem=20isto=2C=20se=20poss=EDvel=2E?= To: obm-l@mat.puc-rio.br MIME-Version: 1.0 Content-Type: text/plain; charset="iso-8859-1" Content-Transfer-Encoding: 8bit X-MIME-Autoconverted: from quoted-printable to 8bit by sucuri.mat.puc-rio.br id OAA14164 Sender: owner-obm-l@sucuri.mat.puc-rio.br Precedence: bulk Reply-To: obm-l@mat.puc-rio.br ANSWER:Bem cara,resolvi isso por Chebychev:se as sequencias reais positivas a(i) e b(i) sao tais que a(i)>a(j) se e e so se b(i)>b(j),entao a MA dos produtos a(i)*b(n-i) a(i) vezes a MA dos a(j) nao e maior que a MA dos produtos a(i)*b(i).Na pratica supondo a>=b>=c,por simetria,entao a/(b+c)-b/(c+a)=(ac+aa-bb-cb)/(b+c)(c+a)=(a-b)c+(a-b)(a+b)/(b+c)(c+a)>=0.LOGO:a/(b+c)>=b/(c+a)>=c/(a+b) e b+c<=c+a<=a+b.Chebychev,P/3*(b+c+c+a+a+b)/3>=(a+b+c)/3.E pronto!!!!!! Falouzes!!!!Peterdirichlet. -- Mensagem original -- >Olá mais uma vez amigos da lista, > >Um grande abraço especial para o Paulo, Tiago e Ralph... que me ajudaram >na >questão da desigualdade. A ajuda de você foi de grande importância aqui para > >mim. > >Agora é o seguinte, peço que vocês analisem a minha resolução para um tal > >exercício... se tiver algum erro, ou alguma outra solução mais rápida e >inteligente... gostaria que vocês me indicassem. > >Lá vai: > >Seja a, b e c números reais positivos, provar que: >P = a/(b+c) + b/(a+c) + c(a+b) >= 3/2 > >----Resolução---- >De MA >= MG, temos: > >[a/(b+c) + b(a+c) + c(a+b)] /3 >= [abc/(b+c).(a+c).(a+b)]^(1/3) > >Como P = a/(b+c) + b/(a+c) + c(a+b), temos: > >P >= 3[abc/(b+c).(a+c).(a+b)]^(1/3) (I) > >(a+b+c)/3 >= (abc)^(1/3) (II) > >[(b+c) + (a+c) + (a+b)]/3 >= [(b+c).(a+c).(a+b)]^(1/3) >[2(a+b+c)]/3 >= [(b+c).(a+c).(a+b)]^(1/3) (III) > >Multiplicando (II) por 2, temos: > >2(a+b+c)/3 >= 2(abc)^(1/3) (IV) > >Analisando (III) e (IV), temos: > >2(a+b+c)/3 >= [(b+c).(a+c).(a+b)]^(1/3) >2(a+b+c)/3 >= 2(abc)^(1/3) > >----- >Bem, até aí, sem problemas, porem aqui começa a minha dúvida. Para finalizar > >o exercício, o único passo que consegui dar, foi achar uma igualdade entre > >as 2 expressões do lado direito das 2 desigualdades. > >Basei-me no fato de que, se 2(a+b+c)/3 é maior ou igual a >[(b+c).(a+c).(a+b)]^(1/3) e tambem 2(a+b+c)/3 é maior ou igual a >2(abc)^(1/3) é porque [(b+c).(a+c).(a+b)]^(1/3) = 2(abc)^(1/3). > >Baseando-se nesta igualdade, fica fácil provar que P >= 3/2, pois: > >Como P >= 3[abc/(b+c).(a+c).(a+b)]^(1/3), teríamos: > >P >= 3(abc)^(1/3) x [(1/(b+c).(a+c).(a+b)]^(1/3) >E substituindo os valores achados, teríamos: > >P>= [3(abc)^(1/3)] / [2(abc)^(1/3)] >e cancelando o termo em (abc)^(1/3), temos: >P>= 3/2 (cqd) > >-----FIM------ > >Bem pessoal, a questão é essa aí. Foi a única maneira que achei para >resolvê-la. Desculpe se não a resolvi usando algo melhor. Porem, a minha > >pergunta aqui, é a seguinte: Essa igualdade que eu achei entre as 2 >expressões, eu realmente poderia afirmar isto ? Eu errei em algum >procedimento tomado para solucionar o problema ? Existe algum método mais > >rápido ou inteligente ? > >Obrigado desde já, >Conto com a ajudade de vocês mais uma vez. >Um grande abraço a todos os amigos, > >Felipe Marinho > >_________________________________________________________________ >Chegou o novo MSN Explorer. Instale já. É gratuito: >http://explorer.msn.com.br > >========================================================================= >Instruções para entrar na lista, sair da lista e usar a lista em >http://www.mat.puc-rio.br/~nicolau/olimp/obm-l.html >O administrador desta lista é >========================================================================= > ------------------------------------------ Use o melhor sistema de busca da Internet Radar UOL - http://www.radaruol.com.br ========================================================================= Instruções para entrar na lista, sair da lista e usar a lista em http://www.mat.puc-rio.br/~nicolau/olimp/obm-l.html O administrador desta lista é ========================================================================= From owner-obm-l@sucuri.mat.puc-rio.br Wed May 8 15:32:34 2002 Return-Path: Received: (from majordom@localhost) by sucuri.mat.puc-rio.br (8.9.3/8.9.3) id PAA15454 for obm-l-list; Wed, 8 May 2002 15:30:39 -0300 Received: from hotmail.com (f146.pav1.hotmail.com [64.4.31.146]) by sucuri.mat.puc-rio.br (8.9.3/8.9.3) with ESMTP id PAA15449 for ; Wed, 8 May 2002 15:30:36 -0300 Received: from mail pickup service by hotmail.com with Microsoft SMTPSVC; Wed, 8 May 2002 11:29:17 -0700 Received: from 200.151.217.144 by pv1fd.pav1.hotmail.msn.com with HTTP; Wed, 08 May 2002 18:29:17 GMT X-Originating-IP: [200.151.217.144] From: "Adherbal Rocha Filho" To: obm-l@mat.puc-rio.br Subject: [obm-l] ajuda importante Date: Wed, 08 May 2002 18:29:17 +0000 Mime-Version: 1.0 Content-Type: text/plain; charset=iso-8859-1; format=flowed Message-ID: X-OriginalArrivalTime: 08 May 2002 18:29:17.0898 (UTC) FILETIME=[437402A0:01C1F6BE] Sender: owner-obm-l@sucuri.mat.puc-rio.br Precedence: bulk Reply-To: obm-l@mat.puc-rio.br >Gostaria muito de ajuda nestas questões: >1.Como o médico me recomendou caminhadas,todo dia de manhã dou uma >volta(com >velocidde constante) na quadra em que resido.Minha mulher aproveita pra >correr (com velocidade constante) em volta do quarteirão.Saímos juntos e >chegamos juntos.Ela percorre a quadra no mesmo sentido que eu e me >ultrapassa 2 vezes durante o percurso.Se ela corresse no sentido contrário >ao meu,quantas vezes ela cruzaria comigo? > >2.Um gramado tem a forma de um quadrado de lado 10m.Uma corda tem um dos >extremos fixado em um dos vértices, e no outro lado extremo está amarrado >um >bode.Se o bode consegue comer metade da grama,então o comprimento da corda >é >aproximadamente qnt? > >3.Um ladrilho em forma de um polígono regular ,foi retirado do lugar que >ocupava em um painel.Observou-se então que esse ladrilho,se sofresse uma >rotação de 40º ou de 60º em torno do seu centro,poderia ser encaixado >perfeitamente no lugar que ficou vago no painel.O menor nº de lados que >pode >ter esse ladrilho é? > >Muito grato por qualquer ajuda! > []´s > Adherbal > >_________________________________________________________________ >O MSN Photos é o modo mais fácil de compartilhar e imprimir suas fotos: >http://photos.msn.com/support/worldwide.aspx > >========================================================================= >Instruções para entrar na lista, sair da lista e usar a lista em >http://www.mat.puc-rio.br/~nicolau/olimp/obm-l.html >O administrador desta lista é >========================================================================= _________________________________________________________________ Envie e receba emails com o Hotmail no seu dispositivo móvel: http://mobile.msn.com ========================================================================= Instruções para entrar na lista, sair da lista e usar a lista em http://www.mat.puc-rio.br/~nicolau/olimp/obm-l.html O administrador desta lista é ========================================================================= From owner-obm-l@sucuri.mat.puc-rio.br Wed May 8 16:22:05 2002 Return-Path: Received: (from majordom@localhost) by sucuri.mat.puc-rio.br (8.9.3/8.9.3) id QAA16467 for obm-l-list; Wed, 8 May 2002 16:21:05 -0300 Received: from fgvrj23.fgv.br (fgvrj23.fgv.br [200.20.164.23]) by sucuri.mat.puc-rio.br (8.9.3/8.9.3) with ESMTP id QAA16463 for ; Wed, 8 May 2002 16:21:01 -0300 Received: by FGVRJ23 with Internet Mail Service (5.5.2653.19) id <20VAVDV3>; Wed, 8 May 2002 16:00:19 -0300 Message-ID: <765A72978645D4118B1C0000E229806D05B6B36F@FGVRJ23> From: Ralph Teixeira To: "'obm-l@mat.puc-rio.br '" Subject: RE: [obm-l] Cilindro reto Date: Wed, 8 May 2002 16:00:09 -0300 MIME-Version: 1.0 X-Mailer: Internet Mail Service (5.5.2653.19) Content-Type: text/plain; charset="iso-8859-1" Sender: owner-obm-l@sucuri.mat.puc-rio.br Precedence: bulk Reply-To: obm-l@mat.puc-rio.br A solucao do Salvador eh perfeita. Ele comecou: V=pi.r^2.h (1) A=2.pi.r.h+2.pi.r^2 (2) Subst. (1) em (2), A=2.V/r+2.pi.r^2 E o jeito mais facil e mais natural de terminar eh usando derivada como o Salvador fez. Se voce nao quiser usar derivada, terah de usar uma magica no lugar... Uma possivel magica vem da desigualdade das medias (media aritmetica > media geometrica) que a gente anda comentando aqui na lista: A = V/r + V/r + 2Pi r^2 >= 3 . raiz_cubica[(V/r).(V/r).(2Pir^2)] = = 3*(2PiV^2)^(1/3) (que nao depende de r!) Em outras palavras, se o V estah fixo, esta eh a area minima (se eh que ela pode chegar a este valor!). Por outro lado, este minimo seria atingido quando os 3 numeros da "media" sao iguais, isto eh, quando: V/r = V/r = 2Pir^2 A primeira igualdade eh redundante, mas a segunda nos indica o r que dah o tal minimo r^3=V/(2Pi) ou r=(V/(2Pi))^(1/3) A gente pode agora achar o h correspondente fazendo umas contas, mas eu prefiro agora voltar ao modo do Salvador: desta ultima tiramos V=2.Pi.r^3; mas sabiamos que V=Pi.r^2.h; comparando estas duas, ve-se que h=2r. Abraco, Ralph ========================================================================= Instruções para entrar na lista, sair da lista e usar a lista em http://www.mat.puc-rio.br/~nicolau/olimp/obm-l.html O administrador desta lista é ========================================================================= From owner-obm-l@sucuri.mat.puc-rio.br Wed May 8 16:41:46 2002 Return-Path: Received: (from majordom@localhost) by sucuri.mat.puc-rio.br (8.9.3/8.9.3) id QAA17201 for obm-l-list; Wed, 8 May 2002 16:41:17 -0300 Received: from hotmail.com (law2-f19.hotmail.com [216.32.181.19]) by sucuri.mat.puc-rio.br (8.9.3/8.9.3) with ESMTP id QAA17197 for ; Wed, 8 May 2002 16:41:14 -0300 Received: from mail pickup service by hotmail.com with Microsoft SMTPSVC; Wed, 8 May 2002 12:39:55 -0700 Received: from 200.229.244.100 by lw2fd.hotmail.msn.com with HTTP; Wed, 08 May 2002 19:39:54 GMT X-Originating-IP: [200.229.244.100] From: "Paulo Santa Rita" To: obm-l@mat.puc-rio.br Subject: [obm-l] =?iso-8859-1?B?UmU6IFtvYm0tbF0gQW5hbGlzZW0gaXN0bywgc2UgcG9zc+12ZWwu?= Date: Wed, 08 May 2002 19:39:54 +0000 Mime-Version: 1.0 Content-Type: text/plain; charset=iso-8859-1; format=flowed Message-ID: X-OriginalArrivalTime: 08 May 2002 19:39:55.0296 (UTC) FILETIME=[2123A200:01C1F6C8] Sender: owner-obm-l@sucuri.mat.puc-rio.br Precedence: bulk Reply-To: obm-l@mat.puc-rio.br Ola Felipe e demais colegas desta lista, Sim, existe um erro em seu raciocinio ... Se X > a e X > b isto nao implica que a=b. Contra-Exemplo : 3 > 2 e 3 > 1 e, no entanto, 2 # 1 ! Eu destaquei pra voce a passagem errada. E a seguinte : >Basei-me no fato de que, se 2(a+b+c)/3 é maior ou igual a >[(b+c).(a+c).(a+b)]^(1/3) e tambem 2(a+b+c)/3 é maior ou igual a >2(abc)^(1/3) é porque [(b+c).(a+c).(a+b)]^(1/3) = 2(abc)^(1/3). Mas o seu esforco e valido ! Quer uma dica ? Verifique que a expressao E(a,b,c)= a/(b+c) + b/(a+c) + c/(a+b) e simetrica ... Um outro caminho podia ser : Seja L=a+b+c. Entao nao pode ser a < L/3 e b < L/3 e c < L/3 pois aconteceria que a + b + c < L. Logo, algum deles, digamos "a", e tal que a >= L/3. Mas se a >= L/3 entao b+c =< 2L/3. E dai segue que a/(b+c) >= (L/3)/(2L/3) isto e a/(b+c) >= 1/2. E por ai vai ... Em verdade existe um "caminhao" de FORMAS SIMPLES de se provar isso, inclusive com as medias que voce usou. E so mexer um pouco mais ... AVANTE ! Um abraco Paulo Santa Rita 4,1638,080502 >From: "Felipe Marinho" >Reply-To: obm-l@mat.puc-rio.br >To: obm-l@mat.puc-rio.br >Subject: [obm-l] Analisem isto, se possível. >Date: Wed, 08 May 2002 11:30:32 -0400 > >Olá mais uma vez amigos da lista, > >Um grande abraço especial para o Paulo, Tiago e Ralph... que me ajudaram na >questão da desigualdade. A ajuda de você foi de grande importância aqui >para >mim. > >Agora é o seguinte, peço que vocês analisem a minha resolução para um tal >exercício... se tiver algum erro, ou alguma outra solução mais rápida e >inteligente... gostaria que vocês me indicassem. > >Lá vai: > >Seja a, b e c números reais positivos, provar que: >P = a/(b+c) + b/(a+c) + c(a+b) >= 3/2 > >----Resolução---- >De MA >= MG, temos: > >[a/(b+c) + b(a+c) + c(a+b)] /3 >= [abc/(b+c).(a+c).(a+b)]^(1/3) > >Como P = a/(b+c) + b/(a+c) + c(a+b), temos: > >P >= 3[abc/(b+c).(a+c).(a+b)]^(1/3) (I) > >(a+b+c)/3 >= (abc)^(1/3) (II) > >[(b+c) + (a+c) + (a+b)]/3 >= [(b+c).(a+c).(a+b)]^(1/3) >[2(a+b+c)]/3 >= [(b+c).(a+c).(a+b)]^(1/3) (III) > >Multiplicando (II) por 2, temos: > >2(a+b+c)/3 >= 2(abc)^(1/3) (IV) > >Analisando (III) e (IV), temos: > >2(a+b+c)/3 >= [(b+c).(a+c).(a+b)]^(1/3) >2(a+b+c)/3 >= 2(abc)^(1/3) > >----- >Bem, até aí, sem problemas, porem aqui começa a minha dúvida. Para >finalizar >o exercício, o único passo que consegui dar, foi achar uma igualdade entre >as 2 expressões do lado direito das 2 desigualdades. > >Basei-me no fato de que, se 2(a+b+c)/3 é maior ou igual a >[(b+c).(a+c).(a+b)]^(1/3) e tambem 2(a+b+c)/3 é maior ou igual a >2(abc)^(1/3) é porque [(b+c).(a+c).(a+b)]^(1/3) = 2(abc)^(1/3). > >Baseando-se nesta igualdade, fica fácil provar que P >= 3/2, pois: > >Como P >= 3[abc/(b+c).(a+c).(a+b)]^(1/3), teríamos: > >P >= 3(abc)^(1/3) x [(1/(b+c).(a+c).(a+b)]^(1/3) >E substituindo os valores achados, teríamos: > >P>= [3(abc)^(1/3)] / [2(abc)^(1/3)] >e cancelando o termo em (abc)^(1/3), temos: >P>= 3/2 (cqd) > >-----FIM------ > >Bem pessoal, a questão é essa aí. Foi a única maneira que achei para >resolvê-la. Desculpe se não a resolvi usando algo melhor. Porem, a minha >pergunta aqui, é a seguinte: Essa igualdade que eu achei entre as 2 >expressões, eu realmente poderia afirmar isto ? Eu errei em algum >procedimento tomado para solucionar o problema ? Existe algum método mais >rápido ou inteligente ? > >Obrigado desde já, >Conto com a ajudade de vocês mais uma vez. >Um grande abraço a todos os amigos, > >Felipe Marinho > >_________________________________________________________________ >Chegou o novo MSN Explorer. Instale já. É gratuito: >http://explorer.msn.com.br > >========================================================================= >Instruções para entrar na lista, sair da lista e usar a lista em >http://www.mat.puc-rio.br/~nicolau/olimp/obm-l.html >O administrador desta lista é >========================================================================= _________________________________________________________________ Una-se ao maior serviço de email do mundo: o MSN Hotmail. http://www.hotmail.com ========================================================================= Instruções para entrar na lista, sair da lista e usar a lista em http://www.mat.puc-rio.br/~nicolau/olimp/obm-l.html O administrador desta lista é ========================================================================= From owner-obm-l@sucuri.mat.puc-rio.br Wed May 8 16:49:55 2002 Return-Path: Received: (from majordom@localhost) by sucuri.mat.puc-rio.br (8.9.3/8.9.3) id QAA17639 for obm-l-list; Wed, 8 May 2002 16:49:44 -0300 Received: from bidu.ime.usp.br (bidu.ime.usp.br [143.107.45.12]) by sucuri.mat.puc-rio.br (8.9.3/8.9.3) with SMTP id QAA17635 for ; Wed, 8 May 2002 16:49:41 -0300 Received: (qmail 241 invoked from network); 8 May 2002 19:48:23 -0000 Received: from fradim.ime.usp.br (143.107.45.37) by bidu.ime.usp.br with SMTP; 8 May 2002 19:48:23 -0000 Received: (qmail 4876 invoked by uid 1604); 8 May 2002 19:45:55 -0000 Date: Wed, 8 May 2002 16:45:55 -0300 (EST) From: Salvador Addas Zanata X-Sender: sazanata@fradim To: "'obm-l@mat.puc-rio.br '" Subject: RE: [obm-l] Cilindro reto In-Reply-To: <765A72978645D4118B1C0000E229806D05B6B36F@FGVRJ23> Message-ID: MIME-Version: 1.0 Content-Type: TEXT/PLAIN; charset=ISO-8859-1 Content-Transfer-Encoding: 8bit X-MIME-Autoconverted: from QUOTED-PRINTABLE to 8bit by sucuri.mat.puc-rio.br id QAA17636 Sender: owner-obm-l@sucuri.mat.puc-rio.br Precedence: bulk Reply-To: obm-l@mat.puc-rio.br Muito legal, a utilidade dessas desigualdades e impressionante. Abraco, Salvador On Wed, 8 May 2002, Ralph Teixeira wrote: > A solucao do Salvador eh perfeita. Ele comecou: > > V=pi.r^2.h (1) > A=2.pi.r.h+2.pi.r^2 (2) > > Subst. (1) em (2), > > A=2.V/r+2.pi.r^2 > > E o jeito mais facil e mais natural de terminar eh usando derivada como o > Salvador fez. Se voce nao quiser usar derivada, terah de usar uma magica no > lugar... Uma possivel magica vem da desigualdade das medias (media > aritmetica > media geometrica) que a gente anda comentando aqui na lista: > > A = V/r + V/r + 2Pi r^2 >= 3 . raiz_cubica[(V/r).(V/r).(2Pir^2)] = > = 3*(2PiV^2)^(1/3) (que nao depende de r!) > > Em outras palavras, se o V estah fixo, esta eh a area minima (se eh que ela > pode chegar a este valor!). Por outro lado, este minimo seria atingido > quando os 3 numeros da "media" sao iguais, isto eh, quando: > > V/r = V/r = 2Pir^2 > > A primeira igualdade eh redundante, mas a segunda nos indica o r que dah o > tal minimo > > r^3=V/(2Pi) ou r=(V/(2Pi))^(1/3) > > A gente pode agora achar o h correspondente fazendo umas contas, mas eu > prefiro agora voltar ao modo do Salvador: desta ultima tiramos V=2.Pi.r^3; > mas sabiamos que V=Pi.r^2.h; comparando estas duas, ve-se que h=2r. > > Abraco, > Ralph > ========================================================================= > Instruções para entrar na lista, sair da lista e usar a lista em > http://www.mat.puc-rio.br/~nicolau/olimp/obm-l.html > O administrador desta lista é > ========================================================================= > ========================================================================= Instruções para entrar na lista, sair da lista e usar a lista em http://www.mat.puc-rio.br/~nicolau/olimp/obm-l.html O administrador desta lista é ========================================================================= From owner-obm-l@sucuri.mat.puc-rio.br Wed May 8 17:37:48 2002 Return-Path: Received: (from majordom@localhost) by sucuri.mat.puc-rio.br (8.9.3/8.9.3) id RAA18865 for obm-l-list; Wed, 8 May 2002 17:37:26 -0300 Received: from www.zipmail.com.br (smtp.zipmail.com.br [200.187.242.10]) by sucuri.mat.puc-rio.br (8.9.3/8.9.3) with ESMTP id RAA18847 for ; Wed, 8 May 2002 17:37:20 -0300 From: peterdirichlet@zipmail.com.br Received: from [200.206.103.3] by www.zipmail.com.br with HTTP; Wed, 8 May 2002 17:35:27 -0300 Message-ID: <3CD981A6000003F4@www.zipmail.com.br> Date: Wed, 8 May 2002 17:35:27 -0300 Subject: [obm-l] =?iso-8859-1?Q?Anderson=20fala=3APROBLEMAS=20RUSSOS=20PARA=20VOCES=21=21=21=21=21=21?= To: =?iso-8859-1?Q?Carlos=20Shine?= , =?iso-8859-1?Q?Lista=20de=20Discussao?= MIME-Version: 1.0 Content-Type: text/plain; charset="iso-8859-1" Content-Transfer-Encoding: 8bit X-MIME-Autoconverted: from quoted-printable to 8bit by sucuri.mat.puc-rio.br id RAA18861 Sender: owner-obm-l@sucuri.mat.puc-rio.br Precedence: bulk Reply-To: obm-l@mat.puc-rio.br Ola turma da Lista!!!!!Tenho uns problemas aqui de um livro da Dover. 1)Prove que se temos as sequencias de reais a(i) e b(i),entao 4* (a(1)^2+...+a(n)^2)(b(1)^2+...+b(n)^2)/(a(1)b(1)+...+a(n)b(n))^2 <=(k^1/2+(1/k)^1/2)^2 em que k e a fraçao feita assim:pega-se o produto dos maiores a e b,divide-se pelos menores a e b. 2)Prove que xy<=x^p/p+y^q/q,em que p+q=pq com p e q IRRACIONAIS e x e y reais positivos. ATE!!!!!!!Peterdirichlet ------------------------------------------ Use o melhor sistema de busca da Internet Radar UOL - http://www.radaruol.com.br ========================================================================= Instruções para entrar na lista, sair da lista e usar a lista em http://www.mat.puc-rio.br/~nicolau/olimp/obm-l.html O administrador desta lista é ========================================================================= From owner-obm-l@sucuri.mat.puc-rio.br Wed May 8 17:53:47 2002 Return-Path: Received: (from majordom@localhost) by sucuri.mat.puc-rio.br (8.9.3/8.9.3) id RAA19187 for obm-l-list; Wed, 8 May 2002 17:52:32 -0300 Received: from www.zipmail.com.br (smtp.zipmail.com.br [200.187.242.10]) by sucuri.mat.puc-rio.br (8.9.3/8.9.3) with ESMTP id RAA19183 for ; Wed, 8 May 2002 17:52:25 -0300 From: peterdirichlet@zipmail.com.br Received: from [200.206.103.3] by www.zipmail.com.br with HTTP; Wed, 8 May 2002 17:50:31 -0300 Message-ID: <3CD981A60000054B@www.zipmail.com.br> Date: Wed, 8 May 2002 17:50:31 -0300 In-Reply-To: <3CD6A4FE0000157E@www.zipmail.com.br> Subject: [obm-l] =?iso-8859-1?Q?Re=3A=20oi=21=28PROBLEMAS=20PARA=20A=20LISTA=29?= To: =?iso-8859-1?Q?Lista=20de=20Discussao?= MIME-Version: 1.0 Content-Type: text/plain; charset="iso-8859-1" Content-Transfer-Encoding: 8bit X-MIME-Autoconverted: from quoted-printable to 8bit by sucuri.mat.puc-rio.br id RAA19184 Sender: owner-obm-l@sucuri.mat.puc-rio.br Precedence: bulk Reply-To: obm-l@mat.puc-rio.br -- Mensagem original -- >ANSWER:Mas e claro que eu envio!!!!Na verdade a ideia e de que a lista inteira >se junte para resolver esses problemas.Aqui vao alguns: >01)Prove que as raizes cubicas de 3 primos diferentes nao podem ser termos(nao >necessariamente consecutivos) de uma mesma PA. >02)Prove que na asequencia de Fermat F(n)=1+2^(2^n)dois termos quaisquer >ou sao iguais ou sao primos entre si. >O primeiro nao sei responder.Mas o 02) se voce tiver duvida me avise!!!!!Ass.:Peterdirichlet > >-- Mensagem original -- > >> >> >>Oi! >>Vc perguntou(em algum email aí...) se alguém queria q vc mandasse questões >> >>q vc jah tinha resolvido pra treinar pra OBM,lembra? Então,será que vc > >>poderia mandar pra mim?? >>Thanx! >>[]´s >>Fê >> >> >>_________________________________________________________________ >>Envie e receba emails com o Hotmail no seu dispositivo móvel: >>http://mobile.msn.com >> >> > > > >------------------------------------------ >Use o melhor sistema de busca da Internet >Radar UOL - http://www.radaruol.com.br > > > > ------------------------------------------ Use o melhor sistema de busca da Internet Radar UOL - http://www.radaruol.com.br ========================================================================= Instruções para entrar na lista, sair da lista e usar a lista em http://www.mat.puc-rio.br/~nicolau/olimp/obm-l.html O administrador desta lista é ========================================================================= From owner-obm-l@sucuri.mat.puc-rio.br Wed May 8 18:29:59 2002 Return-Path: Received: (from majordom@localhost) by sucuri.mat.puc-rio.br (8.9.3/8.9.3) id SAA20350 for obm-l-list; Wed, 8 May 2002 18:29:43 -0300 Received: from hotmail.com (law2-f103.hotmail.com [216.32.181.103]) by sucuri.mat.puc-rio.br (8.9.3/8.9.3) with ESMTP id SAA20346 for ; Wed, 8 May 2002 18:29:40 -0300 Received: from mail pickup service by hotmail.com with Microsoft SMTPSVC; Wed, 8 May 2002 14:28:22 -0700 Received: from 32.94.119.254 by lw2fd.hotmail.msn.com with HTTP; Wed, 08 May 2002 21:28:22 GMT X-Originating-IP: [32.94.119.254] From: "Paulo Santa Rita" To: obm-l@mat.puc-rio.br Subject: Re: [obm-l] Re: [obm-l] Re: [obm-l] Provar desigualdade. Date: Wed, 08 May 2002 21:28:22 +0000 Mime-Version: 1.0 Content-Type: text/plain; charset=iso-8859-1; format=flowed Message-ID: X-OriginalArrivalTime: 08 May 2002 21:28:22.0444 (UTC) FILETIME=[47B39EC0:01C1F6D7] Sender: owner-obm-l@sucuri.mat.puc-rio.br Precedence: bulk Reply-To: obm-l@mat.puc-rio.br Ola Cicero, Tudo Legal ? E que faz alguns anos - cerca de 6 - que terminei o 1 grau e nao me lembro mais de tudo que se ensina neste nivel. Deve ser por isso. De qualquer forma, sua solucao foi muito bonita. Um abracao Paulo Santa Rita 4,1826,080502 >From: ciceroth@zipmail.com.br >Reply-To: obm-l@mat.puc-rio.br >To: obm-l@mat.puc-rio.br >Subject: [obm-l] Re: [obm-l] Re: [obm-l] Provar desigualdade. >Date: Wed, 8 May 2002 11:40:59 -0300 > > >olá Paulo > >Esse assunto de medias me foi ensinado quando eu estava exatamente na >setima >serie( nas minhas aulas de olimpiada). Na minha opinoão essa eh a forma >mais facil de se resolver para um aluno de setima serie. > >abraço >Cicero Thiago > > > > >------------------------------------------ >Use o melhor sistema de busca da Internet >Radar UOL - http://www.radaruol.com.br > > > >========================================================================= >Instruções para entrar na lista, sair da lista e usar a lista em >http://www.mat.puc-rio.br/~nicolau/olimp/obm-l.html >O administrador desta lista é >========================================================================= _________________________________________________________________ O MSN Photos é o modo mais fácil de compartilhar e imprimir suas fotos: http://photos.msn.com/support/worldwide.aspx ========================================================================= Instruções para entrar na lista, sair da lista e usar a lista em http://www.mat.puc-rio.br/~nicolau/olimp/obm-l.html O administrador desta lista é ========================================================================= From owner-obm-l@sucuri.mat.puc-rio.br Wed May 8 18:56:07 2002 Return-Path: Received: (from majordom@localhost) by sucuri.mat.puc-rio.br (8.9.3/8.9.3) id SAA21040 for obm-l-list; Wed, 8 May 2002 18:55:25 -0300 Received: from hotmail.com (law2-f77.hotmail.com [216.32.181.77]) by sucuri.mat.puc-rio.br (8.9.3/8.9.3) with ESMTP id SAA21036 for ; Wed, 8 May 2002 18:55:22 -0300 Received: from mail pickup service by hotmail.com with Microsoft SMTPSVC; Wed, 8 May 2002 14:54:04 -0700 Received: from 200.229.244.100 by lw2fd.hotmail.msn.com with HTTP; Wed, 08 May 2002 21:54:03 GMT X-Originating-IP: [200.229.244.100] From: "Paulo Santa Rita" To: obm-l@mat.puc-rio.br Subject: Re: [obm-l] ajuda importante Date: Wed, 08 May 2002 21:54:03 +0000 Mime-Version: 1.0 Content-Type: text/plain; charset=iso-8859-1; format=flowed Message-ID: X-OriginalArrivalTime: 08 May 2002 21:54:04.0226 (UTC) FILETIME=[DEACE220:01C1F6DA] Sender: owner-obm-l@sucuri.mat.puc-rio.br Precedence: bulk Reply-To: obm-l@mat.puc-rio.br Ola Adherbal e demais colegas desta lista, 1) Se, ao longo do percurso, a esposa encontrou o marido duas vezes E eles chegaram juntos em casa e porque a velocidade dela e 4 vezes a do marido ... eles saem juntos, quando ela encontra ele pela primeira vez e porque ja deu uma volta e mais um pouquinho. Quando ela ela encontra ele pela segunda vez, e porque ja deu mais uma volta ( que ja perfazem duas ) e mais um pouquindo. Como eles chegam juntos, ela deu precisamente 4 voltas. Logo, se V e a velocidade do homem, 4V e a velocidade da esposa dele. Quando ela anda no sentido contrario a velocidade de aproximacao sera 5V, dai ... 2) O bode vai pastar um setor de 90 graus em um circulo cujo raio e precisamente o comprimento da corda. Como o que ele pasta a metade do quadrado, se L for o lado deste quadrado, ele pastara (L^2)/2, logo: Setor de 90= (L^2)/2. Dai ... 3) Significa que o ladrilho e invariante sobe uma rotacao de 40 ou 60 graus. Num poligono regular de N lados, 2pi/N e o angulo central e, portanto, uma rotacao de K*(2pi/N), K inteiro, deixa o poligono invariante. Dai ... >From: "Adherbal Rocha Filho" >Reply-To: obm-l@mat.puc-rio.br >To: obm-l@mat.puc-rio.br >Subject: [obm-l] ajuda importante >Date: Wed, 08 May 2002 18:29:17 +0000 > > > >>Gostaria muito de ajuda nestas questões: > >>1.Como o médico me recomendou caminhadas,todo dia de manhã dou uma >>volta(com >>velocidde constante) na quadra em que resido.Minha mulher aproveita pra >>correr (com velocidade constante) em volta do quarteirão.Saímos juntos e >>chegamos juntos.Ela percorre a quadra no mesmo sentido que eu e me >>ultrapassa 2 vezes durante o percurso.Se ela corresse no sentido contrário >>ao meu,quantas vezes ela cruzaria comigo? >> >>2.Um gramado tem a forma de um quadrado de lado 10m.Uma corda tem um dos >>extremos fixado em um dos vértices, e no outro lado extremo está amarrado >>um >>bode.Se o bode consegue comer metade da grama,então o comprimento da corda >>é >>aproximadamente qnt? >> >>3.Um ladrilho em forma de um polígono regular ,foi retirado do lugar que >>ocupava em um painel.Observou-se então que esse ladrilho,se sofresse uma >>rotação de 40º ou de 60º em torno do seu centro,poderia ser encaixado >>perfeitamente no lugar que ficou vago no painel.O menor nº de lados que >>pode >>ter esse ladrilho é? >> >>Muito grato por qualquer ajuda! >> []´s >> Adherbal >> >>_________________________________________________________________ >>O MSN Photos é o modo mais fácil de compartilhar e imprimir suas fotos: >>http://photos.msn.com/support/worldwide.aspx >> >>========================================================================= >>Instruções para entrar na lista, sair da lista e usar a lista em >>http://www.mat.puc-rio.br/~nicolau/olimp/obm-l.html >>O administrador desta lista é >>========================================================================= > > >_________________________________________________________________ >Envie e receba emails com o Hotmail no seu dispositivo móvel: >http://mobile.msn.com > >========================================================================= >Instruções para entrar na lista, sair da lista e usar a lista em >http://www.mat.puc-rio.br/~nicolau/olimp/obm-l.html >O administrador desta lista é >========================================================================= _________________________________________________________________ O MSN Photos é o modo mais fácil de compartilhar e imprimir suas fotos: http://photos.msn.com/support/worldwide.aspx ========================================================================= Instruções para entrar na lista, sair da lista e usar a lista em http://www.mat.puc-rio.br/~nicolau/olimp/obm-l.html O administrador desta lista é ========================================================================= From owner-obm-l@sucuri.mat.puc-rio.br Wed May 8 20:00:42 2002 Return-Path: Received: (from majordom@localhost) by sucuri.mat.puc-rio.br (8.9.3/8.9.3) id UAA22021 for obm-l-list; Wed, 8 May 2002 20:00:07 -0300 Received: from sr1.terra.com.br (sr1.terra.com.br [200.176.3.16]) by sucuri.mat.puc-rio.br (8.9.3/8.9.3) with ESMTP id UAA22017 for ; Wed, 8 May 2002 20:00:05 -0300 Received: from bica.terra.com.br (bica.terra.com.br [200.176.3.32]) by sr1.terra.com.br (Postfix) with ESMTP id 371086F4D4 for ; Wed, 8 May 2002 19:58:48 -0300 (EST) Received: from Lucelindo (unknown [200.177.64.180]) (authenticated user ldias1) by bica.terra.com.br (Postfix) with ESMTP id 43474D5445 for ; Wed, 8 May 2002 19:58:47 -0300 (EST) Message-ID: <014501c1f6e3$660383c0$b440b1c8@Lucelindo> From: "Lucelindo D. Ferreira" To: References: Subject: Re: [obm-l] ajuda importante Date: Wed, 8 May 2002 19:55:05 -0300 MIME-Version: 1.0 Content-Type: text/plain; charset="iso-8859-1" Content-Transfer-Encoding: 8bit X-Priority: 3 X-MSMail-Priority: Normal X-Mailer: Microsoft Outlook Express 5.00.2615.200 X-MimeOLE: Produced By Microsoft MimeOLE V5.00.2615.200 Sender: owner-obm-l@sucuri.mat.puc-rio.br Precedence: bulk Reply-To: obm-l@mat.puc-rio.br Fala Adherbal! td bom! Se vc já tiver resolvido este problema por favor desconsidere esta solução. 2-Supondo que a grama está distribuida uniformemente pelo campo. Então se o bode come metade da grama, ele tem acesso a metade do campo.A área varrida pelo bode é 1/4 do círculo de raio igual ao comprimento da corda.Ou seja, A = (1/4)piL^2 = (10^2)/2 Abraços. Fabio ----- Original Message ----- From: Adherbal Rocha Filho To: Sent: Monday, May 06, 2002 1:41 PM Subject: [obm-l] ajuda importante > > Gostaria muito de ajuda nestas questões: > 1.Como o médico me recomendou caminhadas,todo dia de manhã dou uma volta(com > velocidde constante) na quadra em que resido.Minha mulher aproveita pra > correr (com velocidade constante) em volta do quarteirão.Saímos juntos e > chegamos juntos.Ela percorre a quadra no mesmo sentido que eu e me > ultrapassa 2 vezes durante o percurso.Se ela corresse no sentido contrário > ao meu,quantas vezes ela cruzaria comigo? > > 2.Um gramado tem a forma de um quadrado de lado 10m.Uma corda tem um dos > extremos fixado em um dos vértices, e no outro lado extremo está amarrado um > bode.Se o bode consegue comer metade da grama,então o comprimento da corda é > aproximadamente qnt? > > 3.Um ladrilho em forma de um polígono regular ,foi retirado do lugar que > ocupava em um painel.Observou-se então que esse ladrilho,se sofresse uma > rotação de 40º ou de 60º em torno do seu centro,poderia ser encaixado > perfeitamente no lugar que ficou vago no painel.O menor nº de lados que pode > ter esse ladrilho é? > > Muito grato por qualquer ajuda! > []´s > Adherbal > > _________________________________________________________________ > O MSN Photos é o modo mais fácil de compartilhar e imprimir suas fotos: > http://photos.msn.com/support/worldwide.aspx > > ========================================================================= > Instruções para entrar na lista, sair da lista e usar a lista em > http://www.mat.puc-rio.br/~nicolau/olimp/obm-l.html > O administrador desta lista é > ========================================================================= > ========================================================================= Instruções para entrar na lista, sair da lista e usar a lista em http://www.mat.puc-rio.br/~nicolau/olimp/obm-l.html O administrador desta lista é ========================================================================= From owner-obm-l@sucuri.mat.puc-rio.br Thu May 9 00:51:19 2002 Return-Path: Received: (from majordom@localhost) by sucuri.mat.puc-rio.br (8.9.3/8.9.3) id AAA24655 for obm-l-list; Thu, 9 May 2002 00:49:22 -0300 Received: from salvatore4.bol.com.br ([200.221.24.52]) by sucuri.mat.puc-rio.br (8.9.3/8.9.3) with ESMTP id AAA24651 for ; Thu, 9 May 2002 00:49:20 -0300 From: curupirazinho@bol.com.br Received: from bol.com.br (200.221.24.80) by salvatore4.bol.com.br (5.1.071) id 3C43690800FC5681 for obm-l@mat.puc-rio.br; Thu, 9 May 2002 00:48:12 -0300 Date: Thu, 9 May 2002 00:47:19 -0300 Message-Id: Subject: Re:[obm-l] ajuda importante MIME-Version: 1.0 Content-Type: text/plain;charset="iso-8859-1" To: obm-l@mat.puc-rio.br X-XaM3-API-Version: 2.4.3.4.4 X-SenderIP: 200.249.244.32 Content-Transfer-Encoding: 8bit X-MIME-Autoconverted: from quoted-printable to 8bit by sucuri.mat.puc-rio.br id AAA24652 Sender: owner-obm-l@sucuri.mat.puc-rio.br Precedence: bulk Reply-To: obm-l@mat.puc-rio.br Olá pra tdos da lista, essa é a primeira vez q eu escrevo. Adherbal, se encontrar qqer erro, perdoe-me. 1) primeiro eu supus q a quadra eh um quadrado d lado "a". Dai achei uma relacao entre as velocidades do homem e da mulher: Vh - velocidade do homem Vm - " da mulher t - tempo pra dar uma volta completa na quadra Vh = 4*a/t Como a mulher ultrapassa o homem 2 vezes e chega no mesmo instante q ele no ponto inicial, concluimos q ela dah 3 voltas na quadra. Vm = 12*a/t Tirando o valor d "t" em qqer das expressoes e jogando na outra expressao, temos q "Vm = 3.Vh" . Dai chegamos facilmente a conclusao q a mulher ultrapassa o homem 3 vezes qdo ela corre em sentido contrario. 2) Como estah dito no problema, o bode come metade da grama, entao a situacao eh a seguinte: jah q o bode estah preso a uma corda e esta está presa a um dos vertices do quadrado, a area de atuaçao do bode (entenda-se a area q o bode pode chegar e comer a grama) eh um quarto de circulo. E como está enunciado no problema: o bode come metade da grama, logo: A - area q o bode come r - tamanho da corda em q o bode está preso A=(pi*r^2)/4 e como A=50, chegamos à aproximaçao r=6,37m 3) Eh facil aceitar q para o poligono poder ser rotacionado de 40 graus e depois de 60 graus, sabendo q isso se refere ao angulo interno, eh facil de aceitar q o angulo interno real desse poligono eh divisor tanto de 40 qto de 60. Dai, para sabermos o numero minimo de lados do poligono, basta saber o mdc de 40 e 60, q eh 20, logo, o angulo interno do poligono eh 20 graus, q o força a ser o poligono regular de 18 lados. __________________________________________________________________________ Quer ter seu próprio endereço na Internet? Garanta já o seu e ainda ganhe cinco e-mails personalizados. DomíniosBOL - http://dominios.bol.com.br ========================================================================= Instruções para entrar na lista, sair da lista e usar a lista em http://www.mat.puc-rio.br/~nicolau/olimp/obm-l.html O administrador desta lista é ========================================================================= From owner-obm-l@sucuri.mat.puc-rio.br Thu May 9 09:55:10 2002 Return-Path: Received: (from majordom@localhost) by sucuri.mat.puc-rio.br (8.9.3/8.9.3) id JAA28639 for obm-l-list; Thu, 9 May 2002 09:52:39 -0300 Received: from chacal.cpd.eesc.sc.usp.br (chacal.cpd.eesc.usp.br [143.107.182.10] (may be forged)) by sucuri.mat.puc-rio.br (8.9.3/8.9.3) with ESMTP id JAA28629 for ; Thu, 9 May 2002 09:52:30 -0300 Received: from localhost ([127.0.0.1] helo=CentrodeProcessamentodeDados) by chacal.cpd.eesc.sc.usp.br with smtp (Exim 3.12 #1 (Debian)) id 175kZB-0004Fe-00 for ; Thu, 09 May 2002 09:50:29 +0000 Date: Thu, 9 May 2002 9:50:29 From: Claudio Antonio Teixeira Bastos To: obm-l Subject: [obm-l] geometria X-Priority: 3 X-Mailer: Centro de Processamento de Dados EESC USP Content-Transfer-Encoding: 8bit X-MSMail-Priority: Medium Importance: Medium Content-Type: text/html; charset="iso-8859-1"; MIME-Version: 1.0 Message-Id: Sender: owner-obm-l@sucuri.mat.puc-rio.br Precedence: bulk Reply-To: obm-l@mat.puc-rio.br

Uma vaca fica presa num pasto em forma de círculo de raio R, qual o comprimento da corda que devemos prendê-la em um ponto do círculo para que ela possa comer metade da área do mesmo?

[]´s Claudio Bastos



________________________________________________
Centro de Processamento de Dados - EESC USP
========================================================================= Instruções para entrar na lista, sair da lista e usar a lista em http://www.mat.puc-rio.br/~nicolau/olimp/obm-l.html O administrador desta lista é ========================================================================= From owner-obm-l@sucuri.mat.puc-rio.br Thu May 9 10:01:39 2002 Return-Path: Received: (from majordom@localhost) by sucuri.mat.puc-rio.br (8.9.3/8.9.3) id KAA28856 for obm-l-list; Thu, 9 May 2002 10:00:19 -0300 Received: from www.zipmail.com.br (smtp.zipmail.com.br [200.187.242.10]) by sucuri.mat.puc-rio.br (8.9.3/8.9.3) with ESMTP id KAA28852 for ; Thu, 9 May 2002 10:00:17 -0300 From: ciceroth@zipmail.com.br Received: from [200.19.178.12] by www.zipmail.com.br with HTTP; Thu, 9 May 2002 09:58:19 -0300 Message-ID: <3CD27D0C0000363B@www.zipmail.com.br> Date: Thu, 9 May 2002 09:58:19 -0300 In-Reply-To: Subject: [obm-l] =?iso-8859-1?Q?Re=3A=20=5Bobm=2Dl=5D=20Re=3A=20=5Bobm=2Dl=5D=20Re=3A=20=5Bobm=2Dl=5D=20Provar=20desigualdade?= =?iso-8859-1?Q?=2E?= To: obm-l@mat.puc-rio.br MIME-Version: 1.0 Content-Type: text/plain; charset="iso-8859-1" Content-Transfer-Encoding: 8bit X-MIME-Autoconverted: from quoted-printable to 8bit by sucuri.mat.puc-rio.br id KAA28853 Sender: owner-obm-l@sucuri.mat.puc-rio.br Precedence: bulk Reply-To: obm-l@mat.puc-rio.br ola Paulo Bom, no meu caso nao eh muito diferente, terminei meu primeiro grau em 1997, tenho apenas 19 anos e ainda me lembro das minhas primeiras aulas de olimpiada(que saudade,hehehehe). A todos da lista queria saber se tem alguma livraria nacional que venda o livro Principles and tequiniques in combinatorics, pois estou interessado em compra-lo. abraco Cicero Thiago ------------------------------------------ Use o melhor sistema de busca da Internet Radar UOL - http://www.radaruol.com.br ========================================================================= Instruções para entrar na lista, sair da lista e usar a lista em http://www.mat.puc-rio.br/~nicolau/olimp/obm-l.html O administrador desta lista é ========================================================================= From owner-obm-l@sucuri.mat.puc-rio.br Thu May 9 11:47:07 2002 Return-Path: Received: (from majordom@localhost) by sucuri.mat.puc-rio.br (8.9.3/8.9.3) id LAA30644 for obm-l-list; Thu, 9 May 2002 11:45:55 -0300 Received: from bidu.ime.usp.br (bidu.ime.usp.br [143.107.45.12]) by sucuri.mat.puc-rio.br (8.9.3/8.9.3) with SMTP id LAA30638 for ; Thu, 9 May 2002 11:45:47 -0300 Received: (qmail 478 invoked from network); 9 May 2002 14:44:31 -0000 Received: from fradim.ime.usp.br (143.107.45.37) by bidu.ime.usp.br with SMTP; 9 May 2002 14:44:31 -0000 Received: (qmail 19167 invoked by uid 1604); 9 May 2002 14:42:03 -0000 Date: Thu, 9 May 2002 11:42:03 -0300 (EST) From: Salvador Addas Zanata X-Sender: sazanata@fradim To: obm-l Subject: Re: [obm-l] geometria In-Reply-To: Message-ID: MIME-Version: 1.0 Content-Type: TEXT/PLAIN; charset=ISO-8859-1 Content-Transfer-Encoding: 8bit X-MIME-Autoconverted: from QUOTED-PRINTABLE to 8bit by sucuri.mat.puc-rio.br id LAA30641 Sender: owner-obm-l@sucuri.mat.puc-rio.br Precedence: bulk Reply-To: obm-l@mat.puc-rio.br Se nao me engano, esse problema foi dado ao Nicolau quando ele ganhou a IMO, pelo Figueiredo, entao presidente na epoca. So que no "original" era um cavalo. Manchete da folha do dia seguinte: "Nicolau resolve problema do cavalo do presidente"... Pra quem e novo, o Figueiredo era realmente meio estupido, pelo menos muitas declaracoes dele eram. Abraco, Salvador PS: Ao inves de dar uma solucao, escrevi essas bobagens... Desculpem. On Thu, 9 May 2002, Claudio Antonio Teixeira Bastos wrote: > > Uma vaca fica presa num pasto em forma de círculo de raio R, qual o > comprimento da corda que devemos prendê-la em um ponto do círculo para > que ela possa comer metade da área do mesmo? > > []´s Claudio Bastos > > > > ________________________________________________ > Centro de Processamento de Dados - EESC USP > ========================================================================= > Instruções para entrar na lista, sair da lista e usar a lista em > http://www.mat.puc-rio.br/~nicolau/olimp/obm-l.html O administrador > desta lista é > ========================================================================= > ========================================================================= Instruções para entrar na lista, sair da lista e usar a lista em http://www.mat.puc-rio.br/~nicolau/olimp/obm-l.html O administrador desta lista é ========================================================================= From owner-obm-l@sucuri.mat.puc-rio.br Thu May 9 12:20:34 2002 Return-Path: Received: (from majordom@localhost) by sucuri.mat.puc-rio.br (8.9.3/8.9.3) id MAA31159 for obm-l-list; Thu, 9 May 2002 12:19:51 -0300 Received: from aquarius.ime.eb.br (aquarius.ime.eb.br [200.20.120.44]) by sucuri.mat.puc-rio.br (8.9.3/8.9.3) with ESMTP id MAA31149 for ; Thu, 9 May 2002 12:19:47 -0300 Received: from epq.ime.eb.br (epq.ime.eb.br [200.20.120.217]) by aquarius.ime.eb.br (8.9.1a/8.9.3) with ESMTP id MAA29913 for ; Thu, 9 May 2002 12:21:03 -0300 Received: from EPQ/SpoolDir by epq.ime.eb.br (Mercury 1.48); 9 May 02 12:20:31 -0300 Received: from SpoolDir by EPQ (Mercury 1.48); 9 May 02 12:20:10 -0300 Received: from LAB36 (200.20.123.166) by epq.ime.eb.br (Mercury 1.48); 9 May 02 12:20:09 -0300 Message-ID: <007201c185a2$da2a83a0$a67b14c8@LAB36> From: "Hamilton Rodrigues" To: "obm" Subject: [obm-l] =?iso-8859-1?Q?An=E1lise?= Date: Sat, 15 Dec 2001 17:58:23 -0200 MIME-Version: 1.0 Content-Type: multipart/alternative; boundary="----=_NextPart_000_006F_01C18592.169B9920" X-Priority: 3 X-MSMail-Priority: Normal X-Mailer: Microsoft Outlook Express 5.00.3018.1300 X-MimeOLE: Produced By Microsoft MimeOLE V5.00.3018.1300 Sender: owner-obm-l@sucuri.mat.puc-rio.br Precedence: bulk Reply-To: obm-l@mat.puc-rio.br This is a multi-part message in MIME format. ------=_NextPart_000_006F_01C18592.169B9920 Content-Type: text/plain; charset="iso-8859-1" Content-Transfer-Encoding: quoted-printable Algu=E9m pode me ajudar com esta? Seja uma fun=E7=E3o f, deriv=E1vel no intervalo (a,b). =20 Definimos uma nova fun=E7=E3o g(x)=3D[f(x)-f(a)]/(x-a) , x<>a, = g(a)=3Df =B4(a).=20 =20 Demonstrar que f =B4 toma qualquer valor compreendido entre f =B4(a) = e g(b) no intervalo (a,b). ------=_NextPart_000_006F_01C18592.169B9920 Content-Type: text/html; charset="iso-8859-1" Content-Transfer-Encoding: quoted-printable
    Algu=E9m pode me ajudar com=20 esta?
 
    Seja uma fun=E7=E3o f, = deriv=E1vel no=20 intervalo (a,b).
   
    Definimos uma nova fun=E7=E3o = g(x)=3D[f(x)-f(a)]/(x-a) , x<>a, g(a)=3Df =B4(a).
   
    Demonstrar que f =B4 toma = qualquer valor=20 compreendido entre f =B4(a) e g(b) no intervalo = (a,b).
------=_NextPart_000_006F_01C18592.169B9920-- ========================================================================= Instruções para entrar na lista, sair da lista e usar a lista em http://www.mat.puc-rio.br/~nicolau/olimp/obm-l.html O administrador desta lista é ========================================================================= From owner-obm-l@sucuri.mat.puc-rio.br Thu May 9 12:59:32 2002 Return-Path: Received: (from majordom@localhost) by sucuri.mat.puc-rio.br (8.9.3/8.9.3) id MAA31923 for obm-l-list; Thu, 9 May 2002 12:58:08 -0300 Received: from hotmail.com (law2-f15.hotmail.com [216.32.181.15]) by sucuri.mat.puc-rio.br (8.9.3/8.9.3) with ESMTP id MAA31919 for ; Thu, 9 May 2002 12:58:05 -0300 Received: from mail pickup service by hotmail.com with Microsoft SMTPSVC; Thu, 9 May 2002 08:56:40 -0700 Received: from 32.94.119.254 by lw2fd.hotmail.msn.com with HTTP; Thu, 09 May 2002 15:56:40 GMT X-Originating-IP: [32.94.119.254] From: "Paulo Santa Rita" To: obm-l@mat.puc-rio.br Subject: [obm-l] =?iso-8859-1?B?UmU6IFtvYm0tbF0gQW7hbGlzZQ==?= Date: Thu, 09 May 2002 15:56:40 +0000 Mime-Version: 1.0 Content-Type: text/plain; charset=iso-8859-1; format=flowed Message-ID: X-OriginalArrivalTime: 09 May 2002 15:56:40.0648 (UTC) FILETIME=[1BB84C80:01C1F772] Sender: owner-obm-l@sucuri.mat.puc-rio.br Precedence: bulk Reply-To: obm-l@mat.puc-rio.br Ola Hamilton, Para demonstrar basta observar coisas basicas : 1) Se f e derivavel em (a,b) entao ela e continua em (a,b) 2) g(x) e a inclinacao da reta que liga dois pontos em (a,b) 3) pelo teorema do valor intermediario ha uma tangente com o valor da inclinacao expresso em 2) 4) a inclinacao da tangente dita em 3 e a derivada de F no ponto. Um abraco Paulo Santa Rita 5,1254,090502 >From: "Hamilton Rodrigues" >Reply-To: obm-l@mat.puc-rio.br >To: "obm" >Subject: [obm-l] Análise >Date: Sat, 15 Dec 2001 17:58:23 -0200 > > Alguém pode me ajudar com esta? > > Seja uma função f, derivável no intervalo (a,b). > > Definimos uma nova função g(x)=[f(x)-f(a)]/(x-a) , x<>a, g(a)=f ´(a). > > Demonstrar que f ´ toma qualquer valor compreendido entre f ´(a) e >g(b) no intervalo (a,b). _________________________________________________________________ Converse com amigos on-line, conheça o MSN Messenger: http://messenger.msn.com ========================================================================= Instruções para entrar na lista, sair da lista e usar a lista em http://www.mat.puc-rio.br/~nicolau/olimp/obm-l.html O administrador desta lista é ========================================================================= From owner-obm-l@sucuri.mat.puc-rio.br Thu May 9 13:55:15 2002 Return-Path: Received: (from majordom@localhost) by sucuri.mat.puc-rio.br (8.9.3/8.9.3) id NAA00448 for obm-l-list; Thu, 9 May 2002 13:54:17 -0300 Received: from bidu.ime.usp.br (bidu.ime.usp.br [143.107.45.12]) by sucuri.mat.puc-rio.br (8.9.3/8.9.3) with SMTP id NAA00443 for ; Thu, 9 May 2002 13:54:13 -0300 Received: (qmail 18845 invoked from network); 9 May 2002 16:52:58 -0000 Received: from fradim.ime.usp.br (143.107.45.37) by bidu.ime.usp.br with SMTP; 9 May 2002 16:52:58 -0000 Received: (qmail 24116 invoked by uid 1604); 9 May 2002 16:50:30 -0000 Date: Thu, 9 May 2002 13:50:30 -0300 (EST) From: Salvador Addas Zanata X-Sender: sazanata@fradim To: obm Subject: Re: [obm-l] =?iso-8859-1?Q?An=E1lise?= In-Reply-To: <007201c185a2$da2a83a0$a67b14c8@LAB36> Message-ID: MIME-Version: 1.0 Content-Type: TEXT/PLAIN; charset=ISO-8859-1 Content-Transfer-Encoding: 8bit X-MIME-Autoconverted: from QUOTED-PRINTABLE to 8bit by sucuri.mat.puc-rio.br id NAA00444 Sender: owner-obm-l@sucuri.mat.puc-rio.br Precedence: bulk Reply-To: obm-l@mat.puc-rio.br Se f e derivavel em (a,b), entao vale o seguinte teorema, conhecido como do valor medio: Dado x,y em (a,b), com x<>y, entao existe z em (x,y), tal que : f(x)-f(y)=f'(z)(x-y) Aplicando isso ao seu problema, dado x em (a,b], entao existe z=z(x), tal que: f(x)-f(a)=f'(z)(x-a), com z em (a,x) Entao f'(z)=g(x). Agora, se o que voce queria mesmo era ver uma demonstracao do teorema do valor medio, e so olhar qualquer livro de calculo 1. Uma ideia besta e a seguinte: Primeiro voce pode provar que se f(a)=f(b), entao existe c em (a,b) com f'(c)=0, se f e derivavel. O teorema geral sai a partir desse com um truquinho besta. Pra "provar" esse, observe que em [a,b], f tem maximo e minimo, pelo teorema de Weierstrass. Se f' nao se anula, entao o maximo e minimo sao os pontos do bordo (aqui esta o ponto onde eu troco 6 por meia duzia), mas como f(a)=f(b), entao a funcao e constante. Isso e so uma ideia infame, mas a prova tem em qualquer livro. Abraco, Salvador On Sat, 15 Dec 2001, Hamilton Rodrigues wrote: > Alguém pode me ajudar com esta? > > Seja uma função f, derivável no intervalo (a,b). > > Definimos uma nova função g(x)=[f(x)-f(a)]/(x-a) , x<>a, g(a)=f ´(a). > > Demonstrar que f ´ toma qualquer valor compreendido entre f ´(a) e g(b) no intervalo (a,b). > ========================================================================= Instruções para entrar na lista, sair da lista e usar a lista em http://www.mat.puc-rio.br/~nicolau/olimp/obm-l.html O administrador desta lista é ========================================================================= From owner-obm-l@sucuri.mat.puc-rio.br Thu May 9 13:57:27 2002 Return-Path: Received: (from majordom@localhost) by sucuri.mat.puc-rio.br (8.9.3/8.9.3) id NAA00528 for obm-l-list; Thu, 9 May 2002 13:57:25 -0300 Received: from ginsberg.uol.com.br (ginsberg.uol.com.br [200.231.206.26]) by sucuri.mat.puc-rio.br (8.9.3/8.9.3) with ESMTP id NAA00509 for ; Thu, 9 May 2002 13:57:21 -0300 Received: from oemcomputer ([200.227.70.49]) by ginsberg.uol.com.br (8.9.1/8.9.1) with SMTP id NAA23563 for ; Thu, 9 May 2002 13:54:59 -0300 (BRT) Message-ID: <00ba01c1f77a$74ac3c40$bd44e3c8@oemcomputer> From: "Paulo Rodrigues" To: References: Subject: Re: [obm-l] geometria Date: Thu, 9 May 2002 13:56:23 -0300 MIME-Version: 1.0 Content-Type: text/plain; charset="iso-8859-1" Content-Transfer-Encoding: 8bit X-Priority: 3 X-MSMail-Priority: Normal X-Mailer: Microsoft Outlook Express 5.00.2615.200 X-MimeOLE: Produced By Microsoft MimeOLE V5.00.2615.200 Sender: owner-obm-l@sucuri.mat.puc-rio.br Precedence: bulk Reply-To: obm-l@mat.puc-rio.br : : Se nao me engano, esse problema foi dado ao Nicolau quando ele ganhou a : IMO, pelo Figueiredo, entao presidente na epoca. So que no "original" era : um cavalo. Manchete da folha do dia seguinte: "Nicolau resolve problema do : cavalo do presidente"... Pra quem e novo, o Figueiredo era realmente meio : estupido, pelo menos muitas declaracoes dele eram. : : Abraco, : : Salvador Acho que foi o Ralph e não o Nicolau. abraços, Paulo --- esta mensagem não contém vírus! Checked by AVG anti-virus system (http://www.grisoft.com). Version: 6.0.351 / Virus Database: 197 - Release Date: 19/04/2002 ========================================================================= Instruções para entrar na lista, sair da lista e usar a lista em http://www.mat.puc-rio.br/~nicolau/olimp/obm-l.html O administrador desta lista é ========================================================================= From owner-obm-l@sucuri.mat.puc-rio.br Thu May 9 16:27:31 2002 Return-Path: Received: (from majordom@localhost) by sucuri.mat.puc-rio.br (8.9.3/8.9.3) id QAA03642 for obm-l-list; Thu, 9 May 2002 16:25:30 -0300 Received: (from nicolau@localhost) by sucuri.mat.puc-rio.br (8.9.3/8.9.3) id QAA03637 for obm-l@mat.puc-rio.br; Thu, 9 May 2002 16:25:29 -0300 Date: Thu, 9 May 2002 16:25:29 -0300 From: "Nicolau C. Saldanha" To: obm-l@mat.puc-rio.br Subject: Re: [obm-l] geometria Message-ID: <20020509162529.A3429@sucuri.mat.puc-rio.br> References: <00ba01c1f77a$74ac3c40$bd44e3c8@oemcomputer> Mime-Version: 1.0 Content-Type: text/plain; charset=iso-8859-1 Content-Disposition: inline Content-Transfer-Encoding: 8bit User-Agent: Mutt/1.2.5i In-Reply-To: <00ba01c1f77a$74ac3c40$bd44e3c8@oemcomputer>; from pauloemanu@uol.com.br on Thu, May 09, 2002 at 01:56:23PM -0300 Sender: owner-obm-l@sucuri.mat.puc-rio.br Precedence: bulk Reply-To: obm-l@mat.puc-rio.br On Thu, May 09, 2002 at 01:56:23PM -0300, Paulo Rodrigues wrote: > > : > : Se nao me engano, esse problema foi dado ao Nicolau quando ele ganhou a > : IMO, pelo Figueiredo, entao presidente na epoca. So que no "original" era > : um cavalo. Manchete da folha do dia seguinte: "Nicolau resolve problema do > : cavalo do presidente"... Pra quem e novo, o Figueiredo era realmente meio > : estupido, pelo menos muitas declaracoes dele eram. > : > : Abraco, > : > : Salvador > > Acho que foi o Ralph e não o Nicolau. Foi o Nicolau sim, mas não foi quando ganhei a IMO (foi mais tarde). E acho que a tal manchete não saiu realmente assim, acho que só rolou a piada. O Figueiredo adorava cavalos. O problema não admite solução explícita. Você deve escrever a fórmula para a área da região atacada pelo herbívoro em função do comprimento da corda e escrever uma equação. []s, N. ========================================================================= Instruções para entrar na lista, sair da lista e usar a lista em http://www.mat.puc-rio.br/~nicolau/olimp/obm-l.html O administrador desta lista é ========================================================================= From owner-obm-l@sucuri.mat.puc-rio.br Thu May 9 16:32:07 2002 Return-Path: Received: (from majordom@localhost) by sucuri.mat.puc-rio.br (8.9.3/8.9.3) id QAA03776 for obm-l-list; Thu, 9 May 2002 16:31:49 -0300 Received: from shannon.bol.com.br (shannon.bol.com.br [200.221.24.13]) by sucuri.mat.puc-rio.br (8.9.3/8.9.3) with ESMTP id QAA03771 for ; Thu, 9 May 2002 16:31:26 -0300 Received: from bol.com.br (200.221.24.76) by shannon.bol.com.br (5.1.071) id 3CDA9EC50000CD04 for obm-l@mat.puc-rio.br; Thu, 9 May 2002 16:29:16 -0300 Date: Thu, 9 May 2002 16:29:16 -0300 Message-Id: Subject: [obm-l] =?iso-8859-1?q?Re=3A=5Bobm=2Dl=5D_Re=3A_=5Bobm=2Dl=5D_Re=3A=5Bobm=2Dl=5D?= =?iso-8859-1?q?_Re=3A_=5Bobm=2Dl=5D_d=FAvidas?= MIME-Version: 1.0 Content-Type: text/plain;charset="iso-8859-1" From: "rafaelc.l" To: obm-l@mat.puc-rio.br X-XaM3-API-Version: 2.4.3.4.4 X-SenderIP: 200.176.166.241 Content-Transfer-Encoding: 8bit X-MIME-Autoconverted: from quoted-printable to 8bit by sucuri.mat.puc-rio.br id QAA03772 Sender: owner-obm-l@sucuri.mat.puc-rio.br Precedence: bulk Reply-To: obm-l@mat.puc-rio.br É, a numero 7 do IME de análise combinatória foi realmente difícil por ser tbm muito enganosa... por exemplo: eu achei que caminhos fossem auqeles em que não se pudesse repetir nem voltar trechos, mas quando vi a resolução, percibi que estava completamente errado e ainda por cima, não entendi a resolução. __________________________________________________________________________ Quer ter seu próprio endereço na Internet? Garanta já o seu e ainda ganhe cinco e-mails personalizados. DomíniosBOL - http://dominios.bol.com.br ========================================================================= Instruções para entrar na lista, sair da lista e usar a lista em http://www.mat.puc-rio.br/~nicolau/olimp/obm-l.html O administrador desta lista é ========================================================================= From owner-obm-l@sucuri.mat.puc-rio.br Thu May 9 19:52:49 2002 Return-Path: Received: (from majordom@localhost) by sucuri.mat.puc-rio.br (8.9.3/8.9.3) id TAA07075 for obm-l-list; Thu, 9 May 2002 19:52:15 -0300 Received: from intercon.interconect.com.br (intercon.interconect.com.br [200.241.255.65]) by sucuri.mat.puc-rio.br (8.9.3/8.9.3) with SMTP id TAA07067 for ; Thu, 9 May 2002 19:52:11 -0300 Received: (qmail 24717 invoked by uid 105); 9 May 2002 22:40:33 -0000 Received: from unknown (HELO t6o7e2) (200.242.250.102) by intercon.interconect.com.br with SMTP; 9 May 2002 22:40:32 -0000 Message-ID: <000001c1f7ac$248de880$66faf2c8@t6o7e2> From: "Odelir Maria Casanova dos Santos" To: Subject: [obm-l] Maio01 Date: Thu, 9 May 2002 13:00:43 -0300 MIME-Version: 1.0 Content-Type: multipart/alternative; boundary="----=_NextPart_000_0007_01C1F759.876B7100" X-Priority: 3 X-MSMail-Priority: Normal X-Mailer: Microsoft Outlook Express 6.00.2600.0000 X-MimeOLE: Produced By Microsoft MimeOLE V6.00.2600.0000 X-Virus-Scanned: by AMaViS perl-11 Sender: owner-obm-l@sucuri.mat.puc-rio.br Precedence: bulk Reply-To: obm-l@mat.puc-rio.br This is a multi-part message in MIME format. ------=_NextPart_000_0007_01C1F759.876B7100 Content-Type: text/plain; charset="iso-8859-1" Content-Transfer-Encoding: quoted-printable Pessoal voc=EAs poderiam me ajudar nessas duas quest=F5es da olimp=EDada = de maio de 2001 ? 1-Em volta de um c=EDrculo situam-se dez moedas de 1 cm de raio . Cada = moeda =E9 tangente ao c=EDrculo e =E0s duas moedas vizinhas. Demonstre = que a soma das =E1reas das dez moedas =E9 o dobro da =E1rea do = c=EDrculo.=20 2-No trap=E9zio ABCD, o lado DA =E9 perpendicular =E0s bases AB e CD. A = base AB mede 45, a base CD mede 20 e o lado BC mede 65. Seja P no lado = BC tal que BP mede 45 e seja M o ponto m=E9dio de DA.=20 Calcule a medida do segmento PM.=20 Obrigado=20 Marcus Dimitri ------=_NextPart_000_0007_01C1F759.876B7100 Content-Type: text/html; charset="iso-8859-1" Content-Transfer-Encoding: quoted-printable
Pessoal voc=EAs poderiam me = ajudar nessas=20 duas quest=F5es da olimp=EDada de maio de 2001 ?

1-Em volta de um c=EDrculo situam-se = dez moedas de 1=20 cm de raio . Cada moeda =E9 tangente ao c=EDrculo e =E0s duas moedas = vizinhas.=20 Demonstre que a soma das =E1reas das dez moedas = =E9 o dobro da=20 =E1rea do c=EDrculo.

2-No trap=E9zio = ABCD, o=20 lado DA =E9 perpendicular =E0s bases AB e CD. A = base AB=20 mede 45, a base CD mede 20 e o lado BC mede 65. Seja = P no=20 lado BC tal que BP mede 45 e seja M o ponto m=E9dio = de=20 DA.

Calcule a medida do segmento = PM.=20

Obrigado

Marcus=20 Dimitri

------=_NextPart_000_0007_01C1F759.876B7100-- ========================================================================= Instruções para entrar na lista, sair da lista e usar a lista em http://www.mat.puc-rio.br/~nicolau/olimp/obm-l.html O administrador desta lista é ========================================================================= From owner-obm-l@sucuri.mat.puc-rio.br Fri May 10 00:03:42 2002 Return-Path: Received: (from majordom@localhost) by sucuri.mat.puc-rio.br (8.9.3/8.9.3) id AAA08953 for obm-l-list; Fri, 10 May 2002 00:02:05 -0300 Received: from prointer.net (smtp.prointernet.com.br [200.194.176.12]) by sucuri.mat.puc-rio.br (8.9.3/8.9.3) with SMTP id AAA08949 for ; Fri, 10 May 2002 00:02:03 -0300 Received: (qmail 27681 invoked from network); 10 May 2002 03:03:30 -0000 Received: from pib-dial-1591.access.prointernet.com.br (HELO globalx) (200.194.181.21) by smtp.prointernet.com.br with SMTP; 10 May 2002 03:03:30 -0000 Message-ID: <011901c1f7ce$c9c77420$15b5c2c8@globalx> From: "Anderson" To: "Lista - OBM" Subject: [obm-l] Inducao Date: Thu, 9 May 2002 23:26:25 -0300 MIME-Version: 1.0 Content-Type: text/plain; charset="iso-8859-1" Content-Transfer-Encoding: 7bit X-Priority: 3 X-MSMail-Priority: Normal X-Mailer: Microsoft Outlook Express 5.00.2919.6600 X-MimeOLE: Produced By Microsoft MimeOLE V5.00.2919.6600 X-AntiVirus: scanned for viruses by Prointernet Brasil (http://www.pib.com.br/) Sender: owner-obm-l@sucuri.mat.puc-rio.br Precedence: bulk Reply-To: obm-l@mat.puc-rio.br Oi, Estou com problemas nos conceitos do metodo de prova da inducao matematica, alguem poderia ajduar? Vejam os exemplos abaixo e por favor tentem me explicar o q esta errado ... ah, os problemas foram tirados do livro do knuth... 1) Let "a" be any positive number. For all positive integers "n" we have a^(n-1) = 1. Proof: If n = 1, a^(n-1) = 1. And by induction, assuming that the theorem is true for 1, 2, 3 ..., n, we have: a^[(n+1) - 1] = a^n = a^(n-1) * a^(n-1) / a^(n-2) = 1*1/1 = 1 Onde esta o erro da prova de acordo com a definicao de inducao? Parece claro q a hipotese a^(n-1) nao e valida para todo n, mas pela definicao de inducao e necessario tambem provar para n=2? Ha tb o problema do termo a^(n-2) nao estar definido para n=1, mas se ele estivesse definido como a^(n-2) = 1 a prova estaria correta? 2) The following proof by induction seems correct, but for some reason the equation for n = 6 gives 1/2 + 1/6 + 1/12 / + 1/20 + 1/30 = 5/6 on the left-hand sid, and 3/2 - 1/6 = 4/3 on the right-hand side. Find a mistake: Theorem: 1/1*2 + 1/2*3 + 1/3*4 + ... + 1/(n-1)*n = 3/2 - 1/n Proof: We use induction on n. For n = 1, 3/2 - 1/n = 1/1*2 and, assuming the theorem is true for n, 1/1*2 + 1/2*3 + 1/3*4 + ... + 1/(n-1)*n + 1/n*(n+1) = 3/2 - 1/n + 1/n*(n+1) = 3/2 -1/n + [1/n - 1/(n+1)] = 3/2 - 1/(n+1) Nesse eu so vi o problema do termo (n-1) nao estar definido para todo "n" ... sera so este o problema? Obrigado, Anderson ========================================================================= Instruções para entrar na lista, sair da lista e usar a lista em http://www.mat.puc-rio.br/~nicolau/olimp/obm-l.html O administrador desta lista é ========================================================================= From owner-obm-l@sucuri.mat.puc-rio.br Fri May 10 05:44:54 2002 Return-Path: Received: (from majordom@localhost) by sucuri.mat.puc-rio.br (8.9.3/8.9.3) id FAA11939 for obm-l-list; Fri, 10 May 2002 05:40:47 -0300 Received: from smtp-3.ig.com.br (smtp-3.ig.com.br [200.226.132.152] (may be forged)) by sucuri.mat.puc-rio.br (8.9.3/8.9.3) with SMTP id FAA11935 for ; Fri, 10 May 2002 05:40:46 -0300 From: ezer@ig.com.br Received: (qmail 8212 invoked from network); 10 May 2002 08:39:20 -0000 Received: from shasta003069.ig.com.br (HELO house) (200.151.3.69) by smtp-3.ig.com.br with SMTP; 10 May 2002 08:39:20 -0000 To: obm-l@mat.puc-rio.br Date: Wed, 1 Jan 1997 00:28:12 -0200 MIME-Version: 1.0 Subject: Re: [obm-l] bibliografia indicada IME/ITA Message-ID: <32C9AF9C.15124.18A48F@localhost> In-reply-to: X-mailer: Pegasus Mail for Windows (v4.01) Content-type: text/plain; charset=ISO-8859-1 Content-description: Mail message body Content-Transfer-Encoding: 8bit X-MIME-Autoconverted: from Quoted-printable to 8bit by sucuri.mat.puc-rio.br id FAA11936 Sender: owner-obm-l@sucuri.mat.puc-rio.br Precedence: bulk Reply-To: obm-l@mat.puc-rio.br Um outro livro mto bom eh o do Russerl, que eh um livro de ensino superior de Quimica, mas contem a mesma materia dos outros livros, soh que bem mais aprofundada e apresentada de maneira cientifica. Na verdade sao tres livros, e o preco eh meio salgado... :/ Se vc comprar, pode emprestar pra mim como forma de retribuicao... ;) hehehe T+ Ezer F. da Silva On 7 May 2002 at 0:03, rafaelc.l wrote: > > Já foi dito aqui na lista os livros mais indicados para > quem se prepara pro IME e ITA. Estou com fundamentos da > Matemática Elementar de Gelson Iezzi, Física Clássica do > Calçadas e química é que eu queria saber..falaram que era > o Feltre, mas é que tem o Feltre sozinho e outro que é > Feltre e Yoshinaga, um bem antigo.....qual deles é o > indicado? > Se puderem me indicar uma boa gramática de português para > esse tipo de vestibular, tbm ficarei grato..... > > > Obrigado > > > __________________________________________________________________________ > Quer ter seu próprio endereço na Internet? > Garanta já o seu e ainda ganhe cinco e-mails personalizados. > DomíniosBOL - http://dominios.bol.com.br > > > ========================================================================= > Instruções para entrar na lista, sair da lista e usar a lista em > http://www.mat.puc-rio.br/~nicolau/olimp/obm-l.html > O administrador desta lista é > ========================================================================= ========================================================================= Instruções para entrar na lista, sair da lista e usar a lista em http://www.mat.puc-rio.br/~nicolau/olimp/obm-l.html O administrador desta lista é ========================================================================= From owner-obm-l@sucuri.mat.puc-rio.br Fri May 10 09:59:01 2002 Return-Path: Received: (from majordom@localhost) by sucuri.mat.puc-rio.br (8.9.3/8.9.3) id JAA14319 for obm-l-list; Fri, 10 May 2002 09:55:53 -0300 Received: from cairu.terra.com.br (cairu.terra.com.br [200.176.3.19]) by sucuri.mat.puc-rio.br (8.9.3/8.9.3) with ESMTP id JAA14313 for ; Fri, 10 May 2002 09:55:50 -0300 Received: from srv9-sao.terra.com.br (srv9-sao.terra.com.br [200.176.3.37]) by cairu.terra.com.br (Postfix) with ESMTP id B29F647278 for ; Fri, 10 May 2002 09:54:37 +0000 (GMT) Received: from Itautec.terra.com.br (200-158-60-215.dsl.telesp.net.br [200.158.60.215]) (authenticated user bruleite) by srv9-sao.terra.com.br (Postfix) with ESMTP id 27A1EC87B5 for ; Fri, 10 May 2002 09:54:37 -0300 (EST) Message-Id: <5.1.0.14.0.20020510094130.00ac0b90@pop.sao.terra.com.br> X-Sender: bruleite@pop.sao.terra.com.br X-Mailer: QUALCOMM Windows Eudora Version 5.1 Date: Fri, 10 May 2002 09:54:56 -0300 To: obm-l@mat.puc-rio.br From: "Bruno F. C. Leite" Subject: Re: [obm-l] Inducao In-Reply-To: <011901c1f7ce$c9c77420$15b5c2c8@globalx> Mime-Version: 1.0 Content-Type: text/plain; charset="iso-8859-1"; format=flowed Content-Transfer-Encoding: 8bit X-MIME-Autoconverted: from quoted-printable to 8bit by sucuri.mat.puc-rio.br id JAA14315 Sender: owner-obm-l@sucuri.mat.puc-rio.br Precedence: bulk Reply-To: obm-l@mat.puc-rio.br At 23:26 09/05/02 -0300, you wrote: >Oi, > Estou com problemas nos conceitos do metodo de prova da inducao >matematica, alguem poderia ajduar? Vejam os exemplos abaixo e por favor >tentem me explicar o q esta errado ... ah, os problemas foram tirados do >livro do knuth... Que livro do Knuth? >1) Let "a" be any positive number. For all positive integers "n" we have >a^(n-1) = 1. >Proof: If n = 1, a^(n-1) = 1. And by induction, assuming that the theorem is >true for 1, 2, 3 ..., n, we have: >a^[(n+1) - 1] = a^n = a^(n-1) * a^(n-1) / a^(n-2) = 1*1/1 = 1 > >Onde esta o erro da prova de acordo com a definicao de inducao? Parece claro >q a hipotese a^(n-1) nao e valida para todo n, mas pela definicao de inducao >e necessario tambem provar para n=2? Ha tb o problema do termo a^(n-2) nao >estar definido para n=1, mas se ele estivesse definido como a^(n-2) = 1 a >prova estaria correta? O problema parece ser o seguinte: a^(n-2) não é inteiro positivo se n =1 logo o começo da indução está errado. (vc está usando uma hipótese que NAO é a hipotese de indução) >2) The following proof by induction seems correct, but for some reason the >equation for n = 6 gives >1/2 + 1/6 + 1/12 / + 1/20 + 1/30 = 5/6 on the left-hand sid, and 3/2 - 1/6 >= 4/3 on the right-hand side. Find a mistake: > >Theorem: >1/1*2 + 1/2*3 + 1/3*4 + ... + 1/(n-1)*n = 3/2 - 1/n > >Proof: We use induction on n. For n = 1, 3/2 - 1/n = 1/1*2 and, assuming >the theorem is true for n, > >1/1*2 + 1/2*3 + 1/3*4 + ... + 1/(n-1)*n + 1/n*(n+1) > >= 3/2 - 1/n + 1/n*(n+1) = 3/2 -1/n + [1/n - 1/(n+1)] = 3/2 - 1/(n+1) > >Nesse eu so vi o problema do termo (n-1) nao estar definido para todo "n" >... sera so este o problema? Sim, mas este já é um problema grave: 1/1*2 + 1/2*3 + 1/3*4 + ... + 1/(n-1)*n = 3/2 - 1/n NAO VALE para n=1. Por outro lado, na indução vc usa que vale para 1 para provar que vale para 2 para provar que vale para 3...se vc partir de uma bobagem, pode chegar em bobagem. Por exemplo. é fácil provar por indução que para todo n inteiro positivo, n+10=n, se assumirmos que vale para n=1. Afinal, n+1+10=n+1 se e só se n+10=n. Isso é diferente de começarmos a indução com um número que não seja 1. Por exemplo: "prove que se n>=4, n!>2^n" Aqui, se n=1,2 ou 3, o que queremos provar é falso, mas isso não atrapalha pq nem usaremos estes casos para completar a indução. Bruno Leite http://www.ime.usp.br/~brleite >Obrigado, >Anderson > > > > > > >========================================================================= >Instruções para entrar na lista, sair da lista e usar a lista em >http://www.mat.puc-rio.br/~nicolau/olimp/obm-l.html >O administrador desta lista é >========================================================================= ========================================================================= Instruções para entrar na lista, sair da lista e usar a lista em http://www.mat.puc-rio.br/~nicolau/olimp/obm-l.html O administrador desta lista é ========================================================================= From owner-obm-l@sucuri.mat.puc-rio.br Fri May 10 10:17:21 2002 Return-Path: Received: (from majordom@localhost) by sucuri.mat.puc-rio.br (8.9.3/8.9.3) id KAA14739 for obm-l-list; Fri, 10 May 2002 10:16:58 -0300 Received: from calhau.terra.com.br (calhau.terra.com.br [200.176.3.20]) by sucuri.mat.puc-rio.br (8.9.3/8.9.3) with ESMTP id KAA14735 for ; Fri, 10 May 2002 10:16:56 -0300 Received: from pacuiba.terra.com.br (pacuiba.terra.com.br [200.176.3.40]) by calhau.terra.com.br (Postfix) with ESMTP id 3B67E48A52 for ; Fri, 10 May 2002 13:15:44 +0000 (GMT) Received: from terra.com.br (webmail4.terra.com.br [200.176.3.179]) by pacuiba.terra.com.br (Postfix) with ESMTP id 288A38015 for ; Fri, 10 May 2002 10:15:44 -0300 (EST) Date: Fri, 10 May 2002 11:15:44 -0200 Message-Id: Subject: [obm-l] Logica de Primeira Ordem MIME-Version: 1.0 Content-Type: text/plain;charset="iso-8859-1" From: "Juliana Borsari" To: obm-l@mat.puc-rio.br X-XaM3-API-Version: 2.4.3.2.7 X-SenderIP: 63.77.55.6 Content-Transfer-Encoding: 8bit X-MIME-Autoconverted: from quoted-printable to 8bit by sucuri.mat.puc-rio.br id KAA14736 Sender: owner-obm-l@sucuri.mat.puc-rio.br Precedence: bulk Reply-To: obm-l@mat.puc-rio.br Alguém poderia ajudar uma pessoa desesperada e me passar exemplos de lógica de primeira ordem resolvidos URGENTE HOJE AINDA!? Muitissimo Obrigada! ========================================================================= Instruções para entrar na lista, sair da lista e usar a lista em http://www.mat.puc-rio.br/~nicolau/olimp/obm-l.html O administrador desta lista é ========================================================================= From owner-obm-l@sucuri.mat.puc-rio.br Fri May 10 12:37:20 2002 Return-Path: Received: (from majordom@localhost) by sucuri.mat.puc-rio.br (8.9.3/8.9.3) id MAA17181 for obm-l-list; Fri, 10 May 2002 12:36:23 -0300 Received: from mls10.cmb.com.br (mls10.mls.com.br [200.152.96.21]) by sucuri.mat.puc-rio.br (8.9.3/8.9.3) with SMTP id MAA17177 for ; Fri, 10 May 2002 12:36:21 -0300 Received: from mlsrj200152101p149.mls.com.br (mlsrj200152101p149.mls.com.br [200.152.101.149]) by mls10.cmb.com.br (NTMail 3.03.0018/1.a4zv) with ESMTP id da246327 for ; Fri, 10 May 2002 12:37:11 -0200 Received: by troy.RIPE.NET with Internet Mail Service (5.5.2653.19) id ; Fri, 10 May 2002 12:04:20 -0300 Message-ID: <00E50A093F50D511BBED00104BC54194394D96@troy.RIPE.NET> From: Fabio Nogueira To: "'obm-l@mat.puc-rio.br'" Subject: RES: [obm-l] Logica de Primeira Ordem Date: Fri, 10 May 2002 12:03:29 -0300 MIME-Version: 1.0 X-Mailer: Internet Mail Service (5.5.2653.19) Content-Type: text/plain; charset="iso-8859-1" Content-Transfer-Encoding: 8bit X-MIME-Autoconverted: from quoted-printable to 8bit by sucuri.mat.puc-rio.br id MAA17178 Sender: owner-obm-l@sucuri.mat.puc-rio.br Precedence: bulk Reply-To: obm-l@mat.puc-rio.br Tenho muitos dê exemplos -----Mensagem original----- De: Juliana Borsari [mailto:jborsari@terra.com.br] Enviada em: Sexta-feira, 10 de Maio de 2002 10:16 Para: obm-l@mat.puc-rio.br Assunto: [obm-l] Logica de Primeira Ordem Alguém poderia ajudar uma pessoa desesperada e me passar exemplos de lógica de primeira ordem resolvidos URGENTE HOJE AINDA!? Muitissimo Obrigada! ========================================================================= Instruções para entrar na lista, sair da lista e usar a lista em http://www.mat.puc-rio.br/~nicolau/olimp/obm-l.html O administrador desta lista é ========================================================================= ========================================================================= Instruções para entrar na lista, sair da lista e usar a lista em http://www.mat.puc-rio.br/~nicolau/olimp/obm-l.html O administrador desta lista é ========================================================================= From owner-obm-l@sucuri.mat.puc-rio.br Fri May 10 17:20:53 2002 Return-Path: Received: (from majordom@localhost) by sucuri.mat.puc-rio.br (8.9.3/8.9.3) id RAA20943 for obm-l-list; Fri, 10 May 2002 17:19:20 -0300 Received: from mat.puc-rio.br (IDENT:root@perere.mat.puc-rio.br [139.82.27.60]) by sucuri.mat.puc-rio.br (8.9.3/8.9.3) with ESMTP id RAA20937 for ; Fri, 10 May 2002 17:19:18 -0300 Received: from localhost (fredpalm@localhost) by mat.puc-rio.br (8.9.3/8.9.3) with ESMTP id QAA01692 for ; Fri, 10 May 2002 16:34:51 -0300 Date: Fri, 10 May 2002 16:34:50 -0300 (BRT) From: Carlos Frederico Borges Palmeira To: obm Subject: Re: [obm-l] =?iso-8859-1?Q?=C1lgebra_Linear?= In-Reply-To: <001801c1f5be$c9dd4f00$0203a8c0@secrel.com.br> Message-ID: MIME-Version: 1.0 Content-Type: TEXT/PLAIN; charset=X-UNKNOWN Content-Transfer-Encoding: 8bit X-MIME-Autoconverted: from QUOTED-PRINTABLE to 8bit by sucuri.mat.puc-rio.br id RAA20938 Sender: owner-obm-l@sucuri.mat.puc-rio.br Precedence: bulk Reply-To: obm-l@mat.puc-rio.br oi davidson, como ate' agora ninguem se manifestou, ai' vai um esboco de solucao. On Tue, 7 May 2002, Davidson Estanislau wrote: > > Bom dia! > > Estou precisando da ajuda de vocês, nestes dois problemas: > > 1. Determine uma transformação linear T: R^3 -> R^3, cuja imagem e núcleo são, respectivamente, os subspaços E = [(1, 1, 1), (1, -1, 1)] e F = [(1, 0, - 1)]. defina T por sua matriz com 9 incognitas e escreva que T(1,1,1)=0, T(1,-1,1}=0 e TV=(1,0,1) onde V e' um vetor arbitrario linearmente independente com os 2 anteriores. Acho que (1,0,0) serve. Na verdade nao e' um sistema 9x9 mas 3 sistemas 3x3 com mesmo determinante, de modo que fica facil. > > 2. Determine uma base para o núcleo da transformação linear T(x, y, z, w) = (x + y + 2z + 2w, x - y + 2z - 2w, x + y + 2z + 2w, x + y + 2z + 2w) > resolva o sistema linear definido por cada coordenada acima igual a zero. 3 equacoes sao iguais, logo so sao 2 de fato.. resolva o sistema de 2 eq. como um sistema em x e y, acho que da': x=-2z ;y=-2w. O nucleo e' formado por vetores da forma (-2z,-2w,z,w) ou seja z(-2,0,1,0)+w(0,-2,0,1). Ai esta' a base que se quer. acho que com isso voce completa a solucao. Fred palmeira > Davidson Estanislau > ========================================================================= Instruções para entrar na lista, sair da lista e usar a lista em http://www.mat.puc-rio.br/~nicolau/olimp/obm-l.html O administrador desta lista é ========================================================================= From owner-obm-l@sucuri.mat.puc-rio.br Fri May 10 18:31:38 2002 Return-Path: Received: (from majordom@localhost) by sucuri.mat.puc-rio.br (8.9.3/8.9.3) id SAA21925 for obm-l-list; Fri, 10 May 2002 18:30:14 -0300 Received: from hotmail.com (f63.law9.hotmail.com [64.4.9.63]) by sucuri.mat.puc-rio.br (8.9.3/8.9.3) with ESMTP id SAA21921 for ; Fri, 10 May 2002 18:30:11 -0300 Received: from mail pickup service by hotmail.com with Microsoft SMTPSVC; Fri, 10 May 2002 14:28:58 -0700 Received: from 200.190.92.89 by lw9fd.law9.hotmail.msn.com with HTTP; Fri, 10 May 2002 21:28:58 GMT X-Originating-IP: [200.190.92.89] From: "Rogerio Fajardo" To: obm-l@mat.puc-rio.br Subject: Re: [obm-l] Problemão que circulou em outra lista Date: Fri, 10 May 2002 21:28:58 +0000 Mime-Version: 1.0 Content-Type: text/plain; format=flowed Message-ID: X-OriginalArrivalTime: 10 May 2002 21:28:58.0777 (UTC) FILETIME=[B22F1890:01C1F869] Sender: owner-obm-l@sucuri.mat.puc-rio.br Precedence: bulk Reply-To: obm-l@mat.puc-rio.br Alguém já sabe a solução deste problema? Não consegui fazer e estou curioso, pois não me parece ter solução. Pelo que percebi, cada vez que um fala "Não sei", ele dá uma dica para o outro. Isto significa que, em cada instante, dependendo do número que ele estivesse pensando, poderia saber. Por exemplo, se o Sr. P soubesse que o produto era 121, quando o Sr. S disse que a soma é menor que 99, ele já mataria o problema, pois 121=121*1 ou 121=11*11, e não há outra opção. Sabendo que a soma é menor que 99, ele já eliminaria a primeira opção e saberia quais eram os números. Quando Sr. P disse "Eu não sei", o Sr. S já percebeu que os números não podiam ser dois primos cujo produto era maior ou igual a 98, pois, se fosse, pelo argumento acima o Sr. P resolveria o problema. Mas, mesmo assim, Sr. S não resolveu o problema, e diss "Eu não sei". Se o problema tem solução, isso significa que, dependendo da soma que o Sr. S conhecesse, ele teria dito "Eu sei". A pergunta é: para quais números ele mataria o problema só do Sr. P dizer: "Eu não sei"? Para mim, isso paree não ter solução, pois as possibilidades para a soma são muitas, ao contrário do produto. >From: "Marcos Melo" >Reply-To: obm-l@mat.puc-rio.br >To: "obm-l" >Subject: [obm-l] Problemão que circulou em outra lista >Date: Thu, 25 Apr 2002 20:46:21 -0200 > >Para o caso de não ter circulado por esta lista: > >******************* Texto do Problema ***************************** >Dois amigos se encontram. Um tem o produto de dois numeros (Sr. P) e >o outro >tem a soma dos dois numeros (Sr. S). Nenhum dos dois sabe quais sao >os >numeros. Entao eles desenvolvem o seguinte dialogo: > >Sr. S: A soma eh menor que 99. >Sr. P: Deste jeito, eu nao sei quais sao os numeros. >Sr. S: Entao eu tambem nao sei quais sao os numeros. >Sr. P: Se voce nao sabe ainda, eu tambem nao sei. >Sr. S: Como voce nao sabe, eu tambem nao sei. > >Sr. P: Agora, eu sei quais sao os numeros. >Sr. S: Eu tambem sei. > >Quais sao os numeros? >************************************************************* > >========================================================================= >Instruções para entrar na lista, sair da lista e usar a lista em >http://www.mat.puc-rio.br/~nicolau/olimp/obm-l.html >O administrador desta lista é >========================================================================= _________________________________________________________________ Chat with friends online, try MSN Messenger: http://messenger.msn.com ========================================================================= Instruções para entrar na lista, sair da lista e usar a lista em http://www.mat.puc-rio.br/~nicolau/olimp/obm-l.html O administrador desta lista é ========================================================================= From owner-obm-l@sucuri.mat.puc-rio.br Fri May 10 20:20:30 2002 Return-Path: Received: (from majordom@localhost) by sucuri.mat.puc-rio.br (8.9.3/8.9.3) id UAA23426 for obm-l-list; Fri, 10 May 2002 20:18:55 -0300 Received: from fgvrj23.fgv.br (fgvrj23.fgv.br [200.20.164.23]) by sucuri.mat.puc-rio.br (8.9.3/8.9.3) with ESMTP id UAA23422 for ; Fri, 10 May 2002 20:18:53 -0300 Received: by FGVRJ23 with Internet Mail Service (5.5.2653.19) id <20VAWNM4>; Fri, 10 May 2002 20:19:28 -0300 Message-ID: <765A72978645D4118B1C0000E229806D05B6B372@FGVRJ23> From: Ralph Teixeira To: "'Rogerio Fajardo '" , "'obm-l@mat.puc-rio.br '" Subject: [obm-l] =?iso-8859-1?Q?RE=3A_=5Bobm-l=5D_Problem=E3o_que_circulou_em_o?= =?iso-8859-1?Q?utra_lista?= Date: Fri, 10 May 2002 20:19:27 -0300 MIME-Version: 1.0 X-Mailer: Internet Mail Service (5.5.2653.19) Content-Type: text/plain; charset="iso-8859-1" Sender: owner-obm-l@sucuri.mat.puc-rio.br Precedence: bulk Reply-To: obm-l@mat.puc-rio.br As ideias do Rogerio estao no caminho certo... Agora eh soh organizar tudo. Eu conheco esse problema sem a restricao do "menor que 99", mas o enunciado costuma deixar claro que sao dois numeros inteiros POSITIVOS -- creio ser esta a ideia, nao? Eu costumo fazer assim... Faca um tabelao mostrando todas as hipoteses para os dois numeros (ou pense em todos os pontos do plano cujas coordenadas sejam inteiros positivos (x,y) com x<=y -- como eu disse, vou jogar fora a condicao do 99, pois acho que nao faz TANTA diferenca na IDEIA). Para cada par (ponto), escreva o "dialogo" entre Sr. S e Sr. P usando D para "descobri!" e N para "nao descobri"... Por exemplo, se os numeros iniciais fossem (1,1), teriamos S=2 e P=1. Senhor P ve o produto e diz "D!". Sr. S tambem diz "D!". Em outras palavras, o par (1,1) leva ao dialogo DD. Assim, na posicao (1,1) eu tenho "DD". Bom, agora monte os dialogos letra a letra, assim: i) O Sr. P ve uma tabela vazia. Se o produto que ele tem for primo (ou 1), ele sabera quais sao os numeros; senao, ele nao tem como adivinhar. Assim, ponha "D" (de descobri!) como primeira letra na tabela para todos os pares da forma (1,p) com p primo (e em (1,1)); nas outras, claramente a primeira letra do dialogo eh "N" (de Nao descobri!). ii) O Sr. S olha a tabela, agora separada em dois conjuntos de possibilidades, aquelas que comecam com D e aquelas que comecam com N. Ele soh tem a soma, isto eh, ele estah restrito a uma especie de "diagonal" do tipo x+y=S no tabelao. Ele olha todos os pontos no tabelao com aquela soma, e percebe que: -- Em (1,1), Sr. S claramente diz "D" (eh a unica possibilidade) e temos "DD" -- fim de papo. -- Em (1,p), idem, pois o Sr. S verah que (1,p) eh o unico par com aquela soma S particular que comeca por "D", entao ele conclui que os numeor sao 1 e p=S-1. Assim, estas casas levam ao dialogo "DD", e fim de papo (lembre-se que eu supus x<=y, jah que a ordem nao interessa). Note que nestes casos, nos aqui de fora nao temos como saber se foi (1,1), ou se foi (1,p), ou que "p" foi esse. -- O mesmo ocorre para (2,2); de fato, o Sr. S olha para a diagonal x+y=4 no tabelao e ve apenas duas possibilidades para o dialogo ateh entao proferido pelo Sr. P: "D" (em (1,3)) e "N" (em (2,2)). Dependendo do que o Sr. P falou, o Sr. S saberah a resposta. Assim temos "DD" em (1,3) e "ND" em (2,2). -- Qualquer outra casa do meu tabelao pertence a uma diagonal x+y=S com varias casas marcadas ateh aqui com "N". Assim, nestas casa, acresente mais um N e fique com "NN" -- o sr. S nao tem como saber que "N" foi o que o Sr. P proferiu. iii) Estah acompanhando o tabelao? O Sr. P estah limitado a uma curva do tipo xy=P (uma especie de hiperbole nos inteiros, limitada pela condicao x<=y). Veja as possibilidades para esta curva... Muitas delas teem um bando de "NN" e, portanto, o Sr. P diria mais um N, incapaz de decidir qual daqueles pontos NN eh o do momento. As excecoes sao: -- Se o produto eh 4, ha apenas os pontos (1,4) e (2,2), que no momento teem dialogos distintos (NN e ND repsectivamente!). Assim, o Sr. P eh capaz de separa-los. Portanto, (1,4) fica com NND e (2,2)=NDD. -- Todos os outros pontos tipo NN viram NNN -- ha varios NN em cada uma das "hiperboles", e o Sr. P nao tem como decidir nada. iv) Agora, o Sr. S ve a sua "diagonal". Quase todas teem um bando de NNN ateh aqui e o Sr. S eh incapaz de dizer qualquer coisa. A excecao notavel eh a reta x+y=5, com apenas (1,4)=NND e (2,3)=NNN. Assim, se a soma for 5, o Sr. S eh capaz de decidir qual dos dois eh o correto pelo dialogo. Ficamos com (1,4)=NNDD e fim de papo, (2,3)=NNND, e todos os outros que tinham NNN ficam NNNN. v) Agora, o Sr. P olha a sua hiperbole. Note que todas elas estao lotadas de NNNN como no passo (iii); a unica diferenca notavel estah na curva xy=6, que agora tem (1,6)=NNNN e (2,3)=NNND. Sr. P eh agora capaz de separa-los pelo dialogo, isto eh, (1,6)=NNNND e (2,3)=NNNDD. O resto que tinha NNNN fica com NNNNN. vi) Note que nao ha mudancas significativas desde o passo (iv) para o Sr. S. A unica esperanca seria a troca de status do ponto (1,6), mas infelizmente ainda ha DOIS pontos na reta x+y=7 com status NNNNN, e o Sr. S seria incapaz de ditingui-los. Em suma, (1,6)=NNNNDD e fim de papo, o resto leva NNNNNN. vii) Daqui para a frente, a tabela nao ganha nenhum D, e portanto nem Sr. S nem Sr. P serah capaz de distinguir pelo dialogo algo que estava confuso depois. A tabela fica assim: y/x 1 2 3 4 5 6 7 8 9 10... 1 DD DD DD NNDD DD NNNNDD DD NNNNNN NNNNNN NNNNNN 2 NDD NNNDD NNNNNN NNNNNN NNNNNN NNNNNN NNNNNN NNNNNN NNNNNN 3 NNNNNN NNNNNN NNNNNN NNNNNN NNNNNN NNNNNN NNNNNN NNNNNN 4 NNNNNN NNNNNN NNNNNN NNNNNN NNNNNN NNNNNN NNNNNN 5 NNNNNN NNNNNN NNNNNN NNNNNN NNNNNN NNNNNN 6 NNNNNN NNNNNN NNNNNN NNNNNN NNNNNN ... Onde NNNNNN de fato significa NNNNNNNNNNN... :) Em suma, pra nois coitadinhos inqui de fora que num tem numuro na mao: - Se o dialogo for DD, era (1,1) ou (1,p) com p primo; - Se o dialogo for NDD, era (2,2) - Se o dialogo for NNDD, era (1,4) - Se o dialogo for NNNDD, era (2,3) - Se o dialogo for NNNNDD, era (1,6) -- o caso do problema! - Fora isso, o dialogo TEM DE SER NNNNN..., e o par pode ser qualquer outro! Fique aa vontade para adaptar este tabelao para o caso da questao, onde a soma eh limitada por 99. Havera varias diferencas do lado direito da tabela, perto do 99, com uma onda de D's do lado de lah... :) De uma certa maneira este metodo eh "infalivel" -- sim, pode dar um trabalho de cao, mas este metodo dah um "algoritmo" para gerar todos os dialogos possiveis, mesmo que a situacao envolvesse inteiros negativos ou outras restricoes. Abraco, Ralph P.S.: Hmmmm... Mas serah que 0,NNNNNN...=D,000000... :) ;P :) >From: "Marcos Melo" >Reply-To: obm-l@mat.puc-rio.br >To: "obm-l" >Subject: [obm-l] Problemão que circulou em outra lista >Date: Thu, 25 Apr 2002 20:46:21 -0200 > >Para o caso de não ter circulado por esta lista: > >******************* Texto do Problema ***************************** >Dois amigos se encontram. Um tem o produto de dois numeros (Sr. P) e >o outro >tem a soma dos dois numeros (Sr. S). Nenhum dos dois sabe quais sao >os >numeros. Entao eles desenvolvem o seguinte dialogo: > >Sr. S: A soma eh menor que 99. >Sr. P: Deste jeito, eu nao sei quais sao os numeros. >Sr. S: Entao eu tambem nao sei quais sao os numeros. >Sr. P: Se voce nao sabe ainda, eu tambem nao sei. >Sr. S: Como voce nao sabe, eu tambem nao sei. > >Sr. P: Agora, eu sei quais sao os numeros. >Sr. S: Eu tambem sei. > >Quais sao os numeros? ========================================================================= Instruções para entrar na lista, sair da lista e usar a lista em http://www.mat.puc-rio.br/~nicolau/olimp/obm-l.html O administrador desta lista é ========================================================================= From owner-obm-l@sucuri.mat.puc-rio.br Fri May 10 23:24:50 2002 Return-Path: Received: (from majordom@localhost) by sucuri.mat.puc-rio.br (8.9.3/8.9.3) id XAA24978 for obm-l-list; Fri, 10 May 2002 23:24:25 -0300 Received: from imo-m02.mx.aol.com (imo-m02.mx.aol.com [64.12.136.5]) by sucuri.mat.puc-rio.br (8.9.3/8.9.3) with ESMTP id XAA24974 for ; Fri, 10 May 2002 23:24:22 -0300 From: DEOLIVEIRASOU@aol.com Received: from DEOLIVEIRASOU@aol.com by imo-m02.mx.aol.com (mail_out_v32.5.) id z.183.830bf96 (4248) for ; Fri, 10 May 2002 22:23:06 -0400 (EDT) Message-ID: <183.830bf96.2a0dda8a@aol.com> Date: Fri, 10 May 2002 22:23:06 EDT Subject: [obm-l] treino.... To: obm-l@mat.puc-rio.br MIME-Version: 1.0 Content-Type: multipart/alternative; boundary="part1_183.830bf96.2a0dda8a_boundary" X-Mailer: AOL 7.0 for Windows BR sub 10501 Sender: owner-obm-l@sucuri.mat.puc-rio.br Precedence: bulk Reply-To: obm-l@mat.puc-rio.br --part1_183.830bf96.2a0dda8a_boundary Content-Type: text/plain; charset="ISO-8859-1" Content-Transfer-Encoding: quoted-printable 1)prove que na P.A 5, 11, 17, 23, 29, 35,...... , h=E1 infinitos n=FAmeros=20 primos.... 2)Mostre que qualquer P.A n=E3o constante, de n=FAmeros inteiros possui uma=20 infinidade de valores compostos. 3) Se a^n-1 =E9 primo, com n>1, ent=E3o a=3D2 e n =E9 primo. 4) Calcular a soma de todos os Divisores positivos de n. Valeu. --part1_183.830bf96.2a0dda8a_boundary Content-Type: text/html; charset="ISO-8859-1" Content-Transfer-Encoding: quoted-printable 1)prove que na P.A  5, 11, 17, 23, 29, 35,......=20= , h=E1 infinitos n=FAmeros primos....
2)Mostre que qualquer P.A n=E3o constante, de n=FAmeros inteiros possui uma=20= infinidade de valores compostos.
3) Se a^n-1 =E9 primo, com n>1, ent=E3o a=3D2 e n =E9 primo.
4) Calcular a soma de todos os Divisores positivos de n.
             Val= eu.
--part1_183.830bf96.2a0dda8a_boundary-- ========================================================================= Instruções para entrar na lista, sair da lista e usar a lista em http://www.mat.puc-rio.br/~nicolau/olimp/obm-l.html O administrador desta lista é ========================================================================= From owner-obm-l@sucuri.mat.puc-rio.br Sat May 11 02:15:11 2002 Return-Path: Received: (from majordom@localhost) by sucuri.mat.puc-rio.br (8.9.3/8.9.3) id CAA26399 for obm-l-list; Sat, 11 May 2002 02:14:57 -0300 Received: from shannon.bol.com.br (shannon.bol.com.br [200.221.24.13]) by sucuri.mat.puc-rio.br (8.9.3/8.9.3) with ESMTP id CAA26395 for ; Sat, 11 May 2002 02:14:55 -0300 Received: from bol.com.br (200.221.24.79) by shannon.bol.com.br (5.1.071) id 3CDAE6F70006286D for obm-l@mat.puc-rio.br; Sat, 11 May 2002 02:13:00 -0300 Date: Sat, 11 May 2002 02:13:00 -0300 Message-Id: Subject: Re:[obm-l] Maio01 MIME-Version: 1.0 Content-Type: text/plain;charset="iso-8859-1" From: "rafaelc.l" To: obm-l@mat.puc-rio.br X-XaM3-API-Version: 2.4.3.4.4 X-SenderIP: 200.176.166.190 Content-Transfer-Encoding: 8bit X-MIME-Autoconverted: from quoted-printable to 8bit by sucuri.mat.puc-rio.br id CAA26396 Sender: owner-obm-l@sucuri.mat.puc-rio.br Precedence: bulk Reply-To: obm-l@mat.puc-rio.br Por acaso a resposta do 2 não é: 10.3^1/2.(9-14^1/2)^1/2 ? OBS: eu fiz só usando báskara __________________________________________________________________________ Quer ter seu próprio endereço na Internet? Garanta já o seu e ainda ganhe cinco e-mails personalizados. DomíniosBOL - http://dominios.bol.com.br ========================================================================= Instruções para entrar na lista, sair da lista e usar a lista em http://www.mat.puc-rio.br/~nicolau/olimp/obm-l.html O administrador desta lista é ========================================================================= From owner-obm-l@sucuri.mat.puc-rio.br Sat May 11 02:17:15 2002 Return-Path: Received: (from majordom@localhost) by sucuri.mat.puc-rio.br (8.9.3/8.9.3) id CAA26438 for obm-l-list; Sat, 11 May 2002 02:17:14 -0300 Received: from shannon.bol.com.br (shannon.bol.com.br [200.221.24.13]) by sucuri.mat.puc-rio.br (8.9.3/8.9.3) with ESMTP id CAA26434 for ; Sat, 11 May 2002 02:17:12 -0300 Received: from bol.com.br (200.221.24.79) by shannon.bol.com.br (5.1.071) id 3CDAE6F700062A0A for obm-l@mat.puc-rio.br; Sat, 11 May 2002 02:15:17 -0300 Date: Sat, 11 May 2002 02:15:17 -0300 Message-Id: Subject: Re: [obm-l] bibliografia indicada IME/ITA MIME-Version: 1.0 Content-Type: text/plain;charset="iso-8859-1" From: "rafaelc.l" To: obm-l@mat.puc-rio.br X-XaM3-API-Version: 2.4.3.4.4 X-SenderIP: 200.176.166.190 Content-Transfer-Encoding: 8bit X-MIME-Autoconverted: from quoted-printable to 8bit by sucuri.mat.puc-rio.br id CAA26435 Sender: owner-obm-l@sucuri.mat.puc-rio.br Precedence: bulk Reply-To: obm-l@mat.puc-rio.br Russel é pesado demias pra mim.....vou usa-lo só pra pegar alguns tópicos que não tem no livro do Feltre e Setsu.... __________________________________________________________________________ Quer ter seu próprio endereço na Internet? Garanta já o seu e ainda ganhe cinco e-mails personalizados. DomíniosBOL - http://dominios.bol.com.br ========================================================================= Instruções para entrar na lista, sair da lista e usar a lista em http://www.mat.puc-rio.br/~nicolau/olimp/obm-l.html O administrador desta lista é ========================================================================= From owner-obm-l@sucuri.mat.puc-rio.br Sat May 11 05:30:12 2002 Return-Path: Received: (from majordom@localhost) by sucuri.mat.puc-rio.br (8.9.3/8.9.3) id FAA28328 for obm-l-list; Sat, 11 May 2002 05:29:25 -0300 Received: from mta1-3.us4.outblaze.com (205-158-62-44.outblaze.com [205.158.62.44]) by sucuri.mat.puc-rio.br (8.9.3/8.9.3) with ESMTP id FAA28324 for ; Sat, 11 May 2002 05:29:22 -0300 Received: from ws1-2.us4.outblaze.com (205-158-62-54.outblaze.com [205.158.62.54]) by mta1-3.us4.outblaze.com (8.12.3/8.12.3/us4-srs) with SMTP id g4B8SC3O009811 for ; Sat, 11 May 2002 08:28:12 GMT Received: (qmail 75671 invoked by uid 1001); 11 May 2002 08:28:12 -0000 Message-ID: <20020511082812.75670.qmail@iname.com> Content-Type: text/plain; charset="iso-8859-1" Content-Disposition: inline Content-Transfer-Encoding: 7bit MIME-Version: 1.0 X-Mailer: MIME-tools 5.41 (Entity 5.404) Received: from [200.191.178.29] by ws1-2.us4.outblaze.com with http for tessa@mail.com; Sat, 11 May 2002 05:28:12 -0300 From: "Alexandre Tessarollo" To: obm-l@mat.puc-rio.br Date: Sat, 11 May 2002 05:28:12 -0300 Subject: [obm-l] Re: Maio01 X-Originating-Ip: 200.191.178.29 X-Originating-Server: ws1-2.us4.outblaze.com Sender: owner-obm-l@sucuri.mat.puc-rio.br Precedence: bulk Reply-To: obm-l@mat.puc-rio.br > Pessoal vocês poderiam me ajudar nessas duas questões da olimpíada de maio de 2001 ? > 1-Em volta de um círculo situam-se dez moedas de 1 cm de raio . Cada moeda é tangente ao círculo e > às duas moedas vizinhas. Demonstre que a soma das áreas das dez moedas é o dobro da área do > círculo. > Junte os centros das moedas e vc terá um decágono regular de lado 2cm (2r para generalizar). Junte os ponto de tangência das moedas com o círculo e vc terá outro decágono regular, de lado L e inscrito numa circunferência de raio R. Observe o trapézio formado pelos centros (O1 e O2) de duas moedas consecutivas e seus respectivos pontos de tangência com o círculo (T1 e T2). Temos o trapézio O1T1T2O2. Sabemos que o ângulo interno de um decágono regular é 144º. (Odeio essa frase, mas) é fácil ver que T1O1 e T2O2 bissectam os ângulos O1 e O2 do decágono maior. Logo, NO TRAPÉZIO, os ângulos O1 e O2 são de 72º. A base maior O1O2 é 2cm (2r). Projetando T1T2 em O1O2 e usando cosseno nos dois triângulos retângulos que "sobram", vemos que T1T2 é igual à 2-2cos(72) [2r-2cos(72), na nossa generalização]. Assim nós temos o valor do lado do decágono menor. Pelas propriedades de polígonos regulares, sabemos que L/2R=sen(180/n), onde R é o raio da circunscrita e n é o número de lados. No decágono menor, L=2-2cos(72) [2r-2cos(72)]. Aplicando essa fórmula e fazendo as contas, temos R=(r-sen(18))/sen(18). Queremos provar que 2*Pi*R^2=10*Pi*r^2. Ou seja, queremos mostrar que R^2=5*r^2. Fazendo as devidas substituições e simplificando, só precisaremos saber o seno de 18º. Só que esse vale (sqrt(5)-1)/4. Simplificando devidamente, veremos que, para r=1, a afirmação é válida, CQD. Talvez eu não tenha sido claro o suficiente em alguns pontos pq estou com sono agora, mas os resultados que eu não msotrei são facilmente comprováveis - vide a relação eentre o lado do polígono regular e o raio da circunscrita. O único resultado que eu usei e não sei mostrar é o valor do sen(18). Esse eu realmente "colei" :))) > 2-No trapézio ABCD, o lado DA é perpendicular às bases AB e CD. A base AB mede 45, a base CD mede > 20 e o lado BC mede 65. Seja P no lado BC tal que BP mede 45 e seja M o ponto médio de DA. > > Calcule a medida do segmento PM. > Tá, DA=65. Mas cadê o ponto P? Meu e-mail atual é meio esquisitão, então nem sei se vc mandou um attach. Dei uma olhada no arquivo da lista e lá não tem anexo nenhum... Será q vc poderia descrever a posição de P? > Obrigado > > Marcus Dimitri []'s Alexandre Tessarollo -- _______________________________________________ Sign-up for your own FREE Personalized E-mail at Mail.com http://www.mail.com/?sr=signup ========================================================================= Instruções para entrar na lista, sair da lista e usar a lista em http://www.mat.puc-rio.br/~nicolau/olimp/obm-l.html O administrador desta lista é ========================================================================= From owner-obm-l@sucuri.mat.puc-rio.br Sat May 11 09:46:41 2002 Return-Path: Received: (from majordom@localhost) by sucuri.mat.puc-rio.br (8.9.3/8.9.3) id JAA29606 for obm-l-list; Sat, 11 May 2002 09:45:16 -0300 Received: from hotmail.com (f52.law8.hotmail.com [216.33.241.52]) by sucuri.mat.puc-rio.br (8.9.3/8.9.3) with ESMTP id JAA29602 for ; Sat, 11 May 2002 09:45:14 -0300 Received: from mail pickup service by hotmail.com with Microsoft SMTPSVC; Sat, 11 May 2002 05:44:04 -0700 Received: from 200.202.96.10 by lw8fd.law8.hotmail.msn.com with HTTP; Sat, 11 May 2002 12:44:03 GMT X-Originating-IP: [200.202.96.10] From: "Antonio Neto" To: obm-l@mat.puc-rio.br Subject: Re: [obm-l] Re: Maio01 Date: Sat, 11 May 2002 12:44:03 +0000 Mime-Version: 1.0 Content-Type: text/plain; format=flowed Message-ID: X-OriginalArrivalTime: 11 May 2002 12:44:04.0187 (UTC) FILETIME=[885BCAB0:01C1F8E9] Sender: owner-obm-l@sucuri.mat.puc-rio.br Precedence: bulk Reply-To: obm-l@mat.puc-rio.br > Talvez eu não tenha sido claro o suficiente em alguns pontos pq estou >com sono agora, mas os resultados que eu não msotrei são facilmente >comprováveis - vide a relação eentre o lado do polígono regular e o raio da >circunscrita. O único resultado que eu usei e não sei mostrar é o valor do >sen(18). Esse eu realmente "colei" :))) Alexandre Tessarollo Sem figura eh mais dificil, mas trace um triangulo isosceles de lados 1, 1 e x, e angulos de 36, 72 e 72. tracando a bissetriz de um dos angulos de 72, vc encontra outro triangulo semelhante ao primeiro, e acha x por semelhanca. Dahi, a bissetriz do angulo de 36 intercepta x no ponto medio, e... Abracos, olavo _________________________________________________________________ Get your FREE download of MSN Explorer at http://explorer.msn.com/intl.asp. ========================================================================= Instruções para entrar na lista, sair da lista e usar a lista em http://www.mat.puc-rio.br/~nicolau/olimp/obm-l.html O administrador desta lista é ========================================================================= From owner-obm-l@sucuri.mat.puc-rio.br Sat May 11 11:14:01 2002 Return-Path: Received: (from majordom@localhost) by sucuri.mat.puc-rio.br (8.9.3/8.9.3) id LAA30434 for obm-l-list; Sat, 11 May 2002 11:12:32 -0300 Received: (from nicolau@localhost) by sucuri.mat.puc-rio.br (8.9.3/8.9.3) id LAA30429 for obm-l@mat.puc-rio.br; Sat, 11 May 2002 11:12:32 -0300 Date: Sat, 11 May 2002 11:12:32 -0300 From: "Nicolau C. Saldanha" To: obm-l@mat.puc-rio.br Subject: Re: [obm-l] treino.... Message-ID: <20020511111232.A30413@sucuri.mat.puc-rio.br> References: <183.830bf96.2a0dda8a@aol.com> Mime-Version: 1.0 Content-Type: text/plain; charset=iso-8859-1 Content-Disposition: inline Content-Transfer-Encoding: 8bit User-Agent: Mutt/1.2.5i In-Reply-To: <183.830bf96.2a0dda8a@aol.com>; from DEOLIVEIRASOU@aol.com on Fri, May 10, 2002 at 10:23:06PM -0400 Sender: owner-obm-l@sucuri.mat.puc-rio.br Precedence: bulk Reply-To: obm-l@mat.puc-rio.br On Fri, May 10, 2002 at 10:23:06PM -0400, DEOLIVEIRASOU@aol.com wrote: > 1)prove que na P.A 5, 11, 17, 23, 29, 35,...... , há infinitos números > primos.... Suponha por absurdo que fossem apenas p1, p2, ..., pn. Considere N = 6*p1*p2*...*pn - 1. Pense em quais podem ser os fatores primos de N... []s, N. ========================================================================= Instruções para entrar na lista, sair da lista e usar a lista em http://www.mat.puc-rio.br/~nicolau/olimp/obm-l.html O administrador desta lista é ========================================================================= From owner-obm-l@sucuri.mat.puc-rio.br Sat May 11 15:29:10 2002 Return-Path: Received: (from majordom@localhost) by sucuri.mat.puc-rio.br (8.9.3/8.9.3) id PAA32023 for obm-l-list; Sat, 11 May 2002 15:26:37 -0300 Received: from intercon.interconect.com.br (intercon.interconect.com.br [200.241.255.65]) by sucuri.mat.puc-rio.br (8.9.3/8.9.3) with SMTP id PAA32019 for ; Sat, 11 May 2002 15:26:35 -0300 Received: (qmail 24817 invoked by uid 105); 11 May 2002 18:15:00 -0000 Received: from unknown (HELO t6o7e2) (200.242.250.212) by intercon.interconect.com.br with SMTP; 11 May 2002 18:14:59 -0000 Message-ID: <003401c1f919$67bca740$d4faf2c8@t6o7e2> From: "Odelir Maria Casanova dos Santos" To: References: <20020511082812.75670.qmail@iname.com> Subject: Re: [obm-l] Re: Maio01 Date: Sat, 11 May 2002 15:26:43 -0300 MIME-Version: 1.0 Content-Type: text/plain; charset="iso-8859-1" Content-Transfer-Encoding: 8bit X-Priority: 3 X-MSMail-Priority: Normal X-Mailer: Microsoft Outlook Express 6.00.2600.0000 X-MimeOLE: Produced By Microsoft MimeOLE V6.00.2600.0000 X-Virus-Scanned: by AMaViS perl-11 Sender: owner-obm-l@sucuri.mat.puc-rio.br Precedence: bulk Reply-To: obm-l@mat.puc-rio.br Desculpe, mas o problema não fornece figura alguma, eu o passei como me foi fornecido. A única coisa que se sabe sobre o ponto P é que ele se localiza em BC, tal que BP meça 45, por conseguinte PC 20 ----- Original Message ----- From: "Alexandre Tessarollo" To: Sent: Saturday, May 11, 2002 5:28 AM Subject: [obm-l] Re: Maio01 > > 2-No trapézio ABCD, o lado DA é perpendicular às bases AB e CD. A base AB mede 45, a base CD mede > > 20 e o lado BC mede 65. Seja P no lado BC tal que BP mede 45 e seja M o ponto médio de DA. > > > > Calcule a medida do segmento PM. > > > > Tá, DA=65. Mas cadê o ponto P? Meu e-mail atual é meio esquisitão, então nem sei se vc mandou um attach. Dei uma olhada no arquivo da lista e lá não tem anexo nenhum... Será q vc poderia descrever a posição de P? > > Obrigado > _______________________________________________ > Sign-up for your own FREE Personalized E-mail at Mail.com > http://www.mail.com/?sr=signup > > ========================================================================= > Instruções para entrar na lista, sair da lista e usar a lista em > http://www.mat.puc-rio.br/~nicolau/olimp/obm-l.html > O administrador desta lista é > ========================================================================= > ========================================================================= Instruções para entrar na lista, sair da lista e usar a lista em http://www.mat.puc-rio.br/~nicolau/olimp/obm-l.html O administrador desta lista é ========================================================================= From owner-obm-l@sucuri.mat.puc-rio.br Mon May 13 10:17:38 2002 Return-Path: Received: (from majordom@localhost) by sucuri.mat.puc-rio.br (8.9.3/8.9.3) id KAA01531 for obm-l-list; Mon, 13 May 2002 10:13:43 -0300 Received: from hotmail.com (f135.pav1.hotmail.com [64.4.31.135]) by sucuri.mat.puc-rio.br (8.9.3/8.9.3) with ESMTP id KAA01527 for ; Mon, 13 May 2002 10:13:41 -0300 Received: from mail pickup service by hotmail.com with Microsoft SMTPSVC; Sun, 12 May 2002 15:53:49 -0700 Received: from 200.151.212.33 by pv1fd.pav1.hotmail.msn.com with HTTP; Sun, 12 May 2002 22:53:49 GMT X-Originating-IP: [200.151.212.33] From: "Adherbal Rocha Filho" To: obm-l@mat.puc-rio.br Subject: [obm-l] ajuda por favor Date: Sun, 12 May 2002 22:53:49 +0000 Mime-Version: 1.0 Content-Type: text/plain; charset=iso-8859-1; format=flowed Message-ID: X-OriginalArrivalTime: 12 May 2002 22:53:49.0857 (UTC) FILETIME=[E187C110:01C1FA07] Sender: owner-obm-l@sucuri.mat.puc-rio.br Precedence: bulk Reply-To: obm-l@mat.puc-rio.br Oi pessoal, como resolvo: 1.determine as soluções inteiras positivas de abc=a+b+c 2.sendo a+b-c=1,(a,b,c nºs positivos) prove q [(1/a)-1][(1/b)-1][(1/c)-1]>=8 3. sejam a e b as raizes da equação x^2 +x+1=0 determine 1/a^3 +1/b^3 Muito grato! Adherbal _________________________________________________________________ Una-se ao maior serviço de email do mundo: o MSN Hotmail. http://www.hotmail.com ========================================================================= Instruções para entrar na lista, sair da lista e usar a lista em http://www.mat.puc-rio.br/~nicolau/olimp/obm-l.html O administrador desta lista é ========================================================================= From owner-obm-l@sucuri.mat.puc-rio.br Mon May 13 10:44:24 2002 Return-Path: Received: (from majordom@localhost) by sucuri.mat.puc-rio.br (8.9.3/8.9.3) id KAA03471 for obm-l-list; Mon, 13 May 2002 10:43:46 -0300 Received: from spf2.outblaze.com (202-77-181-64.outblaze.com [202.77.181.64]) by sucuri.mat.puc-rio.br (8.9.3/8.9.3) with ESMTP id KAA03466 for ; Mon, 13 May 2002 10:43:41 -0300 Received: from mg.hk5.outblaze.com (202-77-181-23.outblaze.com [202.77.181.23]) by spf2.outblaze.com (8.11.6/8.11.6/srs) with ESMTP id g4DBUgr25377 for ; Mon, 13 May 2002 12:30:42 +0100 (BST) (envelope-from tessa@mail.com) Received: from mta1-3.us4.outblaze.com (205-158-62-44.outblaze.com [205.158.62.44]) by mg.hk5.outblaze.com (8.11.2/8.11.2) with ESMTP id g4D9TMx14264 for ; Mon, 13 May 2002 09:29:22 GMT Received: from ws1-4.us4.outblaze.com (205-158-62-50.outblaze.com [205.158.62.50]) by mta1-3.us4.outblaze.com (8.12.3/8.12.3/us4-srs) with SMTP id g4D7IW3O024661 for ; Mon, 13 May 2002 07:18:32 GMT Received: (qmail 15734 invoked by uid 1001); 13 May 2002 07:18:32 -0000 Message-ID: <20020513071832.15733.qmail@iname.com> Content-Type: text/plain; charset="iso-8859-1" Content-Disposition: inline Content-Transfer-Encoding: 7bit MIME-Version: 1.0 X-Mailer: MIME-tools 5.41 (Entity 5.404) Received: from [200.191.168.63] by ws1-4.us4.outblaze.com with http for tessa@mail.com; Mon, 13 May 2002 04:18:32 -0300 From: "Alexandre Tessarollo" To: obm-l@mat.puc-rio.br Date: Mon, 13 May 2002 04:18:32 -0300 Subject: [obm-l] Re: Maio01 X-Originating-Ip: 200.191.168.63 X-Originating-Server: ws1-4.us4.outblaze.com Sender: owner-obm-l@sucuri.mat.puc-rio.br Precedence: bulk Reply-To: obm-l@mat.puc-rio.br Ok, mil desclupas. As palavras e os números se embolaram na minha frente... Agora lendo o enunciado com a devida calma, temos, no sentido anti-horário: AB=45; BP=45; PC=20; CD=20; DM=30; MA=30. Trace PD. Observe que o triângulo PCD é isósceles. Como o angPCD+angABP=180º, temos cos(PCD)=-cos(ABP). Projetando C ortogonalmente em AB, temos um triângulo retângulo do qula tiramos cos(ABP)=5/13, logo, cos(PCD)=-5/13. Aplicando Lei do Cossenos no triângulo PCD, temos PD=120/sqrt(5). Aplicando Lei dos Senos em PCD, temos CP/sen(PDC)=PD/sen(PCD). Daí tiramos que sen(PDC)=sqrt(5)/5. Como angPDC+angPDM=90º, temos cos(ODM)=sen(PDC)=sqrt(5)/5. Aplicando Lei dos Cossenos em PDM, teremos PM=6sqrt(145). Se eu não errei nenhuma conta no caminho, o resultado é esse. Outro opção seria tomaro o triângulo isósceles ABP ao invés do PCD. Depois, em vez de observar PDM, trabalha-se com PAM, mas o raciocínio todo é basicamente o mesmo. []'s Alexandre Tessarollo PS: Obrigado ao Antônio pelo sen(18). Já tinha ouvido o valor várias vezes mas nunca tinha parado p/pensar a respeito nem nunca tinha visto nenhuma demonstração. Valeu mesmo. >Desculpe, mas o problema não fornece figura alguma, eu o passei como me foi fornecido. A única coisa que se sabe sobre o >ponto P é que ele se localiza em BC, tal que BP meça 45, por conseguinte PC 20. ----- Original Message ----- From: "Alexandre Tessarollo" To: Sent: Saturday, May 11, 2002 5:28 AM Subject: [obm-l] Re: Maio01 > > 2-No trapézio ABCD, o lado DA é perpendicular às bases AB e CD. A base AB mede 45, a base CD mede > > 20 e o lado BC mede 65. Seja P no lado BC tal que BP mede 45 e seja M o ponto médio de DA. > > > > Calcule a medida do segmento PM. -- _______________________________________________ Sign-up for your own FREE Personalized E-mail at Mail.com http://www.mail.com/?sr=signup ========================================================================= Instruções para entrar na lista, sair da lista e usar a lista em http://www.mat.puc-rio.br/~nicolau/olimp/obm-l.html O administrador desta lista é ========================================================================= From owner-obm-l@sucuri.mat.puc-rio.br Mon May 13 11:58:10 2002 Return-Path: Received: (from majordom@localhost) by sucuri.mat.puc-rio.br (8.9.3/8.9.3) id LAA06723 for obm-l-list; Mon, 13 May 2002 11:56:13 -0300 Received: from shannon.bol.com.br (shannon.bol.com.br [200.221.24.13]) by sucuri.mat.puc-rio.br (8.9.3/8.9.3) with ESMTP id LAA06719 for ; Mon, 13 May 2002 11:56:04 -0300 Received: from bol.com.br (200.221.24.84) by shannon.bol.com.br (5.1.071) id 3CDAE6F70008F7F7 for obm-l@mat.puc-rio.br; Sat, 11 May 2002 22:46:53 -0300 Date: Sat, 11 May 2002 22:46:53 -0300 Message-Id: Subject: [obm-l] .......... MIME-Version: 1.0 Content-Type: text/plain;charset="iso-8859-1" From: "rafaelc.l" To: obm-l@mat.puc-rio.br X-XaM3-API-Version: 2.4.3.4.4 X-SenderIP: 200.176.167.101 Content-Transfer-Encoding: 8bit X-MIME-Autoconverted: from quoted-printable to 8bit by sucuri.mat.puc-rio.br id LAA06720 Sender: owner-obm-l@sucuri.mat.puc-rio.br Precedence: bulk Reply-To: obm-l@mat.puc-rio.br 1) Considere um pentágono regular de lado L inscrito numa circunferência de raio R. Existe alguma maneira de se obter o valor de L em função de R sem usar a tebela dos arcos? 2)(IME-2001) Resolva a equação [5-(5-x)^1/2]^1/2=x sendo x>0. Obrigado __________________________________________________________________________ Quer ter seu próprio endereço na Internet? Garanta já o seu e ainda ganhe cinco e-mails personalizados. DomíniosBOL - http://dominios.bol.com.br ========================================================================= Instruções para entrar na lista, sair da lista e usar a lista em http://www.mat.puc-rio.br/~nicolau/olimp/obm-l.html O administrador desta lista é ========================================================================= From owner-obm-l@sucuri.mat.puc-rio.br Mon May 13 12:04:25 2002 Return-Path: Received: (from majordom@localhost) by sucuri.mat.puc-rio.br (8.9.3/8.9.3) id MAA06902 for obm-l-list; Mon, 13 May 2002 12:03:56 -0300 Received: from mail.terraempresas.com.br (mail.terraempresas.com.br [200.177.96.20]) by sucuri.mat.puc-rio.br (8.9.3/8.9.3) with ESMTP id MAA06898 for ; Mon, 13 May 2002 12:03:54 -0300 Received: from cemag.com.br (ipcorp-C8B17502.terraempresas.com.br [200.177.117.2] (may be forged)) by mail.terraempresas.com.br (8.12.2/8.11.2) with SMTP id g4DEnbdR029692 for ; Mon, 13 May 2002 11:51:58 -0300 Received: from 192.168.3.2 ([192.168.3.2]) by cemag.com.br (WinRoute Pro 4.1) with SMTP; Mon, 13 May 2002 11:21:22 -0300 Message-ID: <002901c1fa88$c13d4980$0203a8c0@secrel.com.br> From: "Davidson Estanislau" To: "obm" Subject: Re: [obm-l] ajuda por favor Date: Mon, 13 May 2002 11:16:05 -0300 Organization: Cemag MIME-Version: 1.0 Content-Type: text/plain; charset="iso-8859-1" Content-Transfer-Encoding: 8bit X-Priority: 3 X-MSMail-Priority: Normal X-Mailer: Microsoft Outlook Express 6.00.2600.0000 X-MimeOLE: Produced By Microsoft MimeOLE V6.00.2600.0000 Sender: owner-obm-l@sucuri.mat.puc-rio.br Precedence: bulk Reply-To: obm-l@mat.puc-rio.br Para a questão 3: 1/a^3 + 1/b^3 = (a^3 + b^3)/(ab)^3 = (a + b)(a^2 - ab + b^2)/(ab)^3 = (a + b)((a + b)^2 - 3ab)/(ab)^3 (I) Pela equação, x^2 +x+1=0, temos que: a + b = -1 ab = 1 Substitundo em (I), teremos que: 1/a^3 + 1/b^3 = 2 Felicidades! Davidson Estanislau -----Mensagem Original----- De: "Adherbal Rocha Filho" Para: Enviada em: Domingo, 12 de Maio de 2002 19:53 Assunto: [obm-l] ajuda por favor Oi pessoal, como resolvo: 1.determine as soluções inteiras positivas de abc=a+b+c 2.sendo a+b-c=1,(a,b,c nºs positivos) prove q [(1/a)-1][(1/b)-1][(1/c)-1]>=8 3. sejam a e b as raizes da equação x^2 +x+1=0 determine 1/a^3 +1/b^3 Muito grato! Adherbal ========================================================================= Instruções para entrar na lista, sair da lista e usar a lista em http://www.mat.puc-rio.br/~nicolau/olimp/obm-l.html O administrador desta lista é ========================================================================= From owner-obm-l@sucuri.mat.puc-rio.br Mon May 13 12:09:34 2002 Return-Path: Received: (from majordom@localhost) by sucuri.mat.puc-rio.br (8.9.3/8.9.3) id MAA07043 for obm-l-list; Mon, 13 May 2002 12:09:04 -0300 Received: from shannon.bol.com.br (shannon.bol.com.br [200.221.24.13]) by sucuri.mat.puc-rio.br (8.9.3/8.9.3) with ESMTP id MAA07039 for ; Mon, 13 May 2002 12:09:02 -0300 From: rafaelc.l@bol.com.br Received: from bol.com.br (200.221.24.82) by shannon.bol.com.br (5.1.071) id 3CDAE6F7000D942A for obm-l@mat.puc-rio.br; Mon, 13 May 2002 11:56:11 -0300 Date: Mon, 13 May 2002 11:56:11 -0300 Message-Id: Subject: [obm-l] provas IME e ITA MIME-Version: 1.0 Content-Type: text/plain;charset="iso-8859-1" To: obm-l@mat.puc-rio.br X-XaM3-API-Version: 2.4.3.4.4 X-SenderIP: 200.176.166.167 Content-Transfer-Encoding: 8bit X-MIME-Autoconverted: from quoted-printable to 8bit by sucuri.mat.puc-rio.br id MAA07040 Sender: owner-obm-l@sucuri.mat.puc-rio.br Precedence: bulk Reply-To: obm-l@mat.puc-rio.br Algúem aí sabe onde posso encontrar as provas passadas( década de 80 e 90)do ITA e do IME? mas sem a resolucão delas e se tiver gabarito que seja inteligente( posto numa tabela no final da prova e não na questão como tenho visto). Obrigado __________________________________________________________________________ Quer ter seu próprio endereço na Internet? Garanta já o seu e ainda ganhe cinco e-mails personalizados. DomíniosBOL - http://dominios.bol.com.br ========================================================================= Instruções para entrar na lista, sair da lista e usar a lista em http://www.mat.puc-rio.br/~nicolau/olimp/obm-l.html O administrador desta lista é ========================================================================= From owner-obm-l@sucuri.mat.puc-rio.br Mon May 13 12:30:55 2002 Return-Path: Received: (from majordom@localhost) by sucuri.mat.puc-rio.br (8.9.3/8.9.3) id MAA08018 for obm-l-list; Mon, 13 May 2002 12:30:18 -0300 Received: from candeias.terra.com.br (candeias.terra.com.br [200.176.3.18]) by sucuri.mat.puc-rio.br (8.9.3/8.9.3) with ESMTP id MAA08006 for ; Mon, 13 May 2002 12:30:16 -0300 Received: from guarapari.terra.com.br (guarapari.terra.com.br [200.176.3.36]) by candeias.terra.com.br (Postfix) with ESMTP id F379F442DC for ; Mon, 13 May 2002 12:18:19 -0300 (EST) Received: from Lucelindo (unknown [200.177.64.29]) (authenticated user ldias1) by guarapari.terra.com.br (Postfix) with ESMTP id A3010ECD00 for ; Mon, 13 May 2002 12:18:18 -0300 (EST) Message-ID: <001d01c1fa90$dff38e40$1d40b1c8@Lucelindo> From: "Lucelindo D. Ferreira" To: References: Subject: Re: [obm-l] ajuda por favor Date: Mon, 13 May 2002 12:14:25 -0300 MIME-Version: 1.0 Content-Type: text/plain; charset="iso-8859-1" Content-Transfer-Encoding: 8bit X-Priority: 3 X-MSMail-Priority: Normal X-Mailer: Microsoft Outlook Express 5.00.2615.200 X-MimeOLE: Produced By Microsoft MimeOLE V5.00.2615.200 Sender: owner-obm-l@sucuri.mat.puc-rio.br Precedence: bulk Reply-To: obm-l@mat.puc-rio.br ----- Original Message ----- From: Adherbal Rocha Filho To: Sent: Sunday, May 12, 2002 7:53 PM Subject: [obm-l] ajuda por favor > > > Oi pessoal, > como resolvo: > 1.determine as soluções inteiras positivas de abc=a+b+c > 2.sendo a+b-c=1,(a,b,c nºs positivos) prove q [(1/a)-1][(1/b)-1][(1/c)-1]>=8 Holla Adherbal, td bem? Eu acho que este problema está com o enunciado errado, pois se vc fizer a = b = c = 1, [(1/a)-1][(1/b)-1][(1/c)-1] = 0 < 8 o que é um contraexemplo. Talvez a hipótese do problema seja a + b + c = 1 (a,b,c positivos). Sendo assim ( se for), [(1/a)-1][(1/b)-1][(1/c)-1] = = [(1 - a)/a][(1 - b)/b][(1 - c)/c] = [(b + c)/a][(a + c)/b][(a + b)/c] = (b + c)(a + c)(a + b)/abc . Pela desigualdade entre as médias AM-GM: (b + c)(a + c)(a + b)>= [2sqrt(bc)][2sqrt(ac)][2aqrt(ab)] = 8abc. Então (b + c)(a + c)(a + b)/abc >= 8abc/abc = 8 > 3. sejam a e b as raizes da equação x^2 +x+1=0 determine 1/a^3 +1/b^3 3-dica: efetue a soma e use a^3 + b^3 = (a + b)[(a + b)^2 - 3ab]. Aí é com vc. Até!! > Muito grato! > Adherbal > > > _________________________________________________________________ > Una-se ao maior serviço de email do mundo: o MSN Hotmail. > http://www.hotmail.com > > ========================================================================= > Instruções para entrar na lista, sair da lista e usar a lista em > http://www.mat.puc-rio.br/~nicolau/olimp/obm-l.html > O administrador desta lista é > ========================================================================= > ========================================================================= Instruções para entrar na lista, sair da lista e usar a lista em http://www.mat.puc-rio.br/~nicolau/olimp/obm-l.html O administrador desta lista é ========================================================================= From owner-obm-l@sucuri.mat.puc-rio.br Mon May 13 14:21:37 2002 Return-Path: Received: (from majordom@localhost) by sucuri.mat.puc-rio.br (8.9.3/8.9.3) id OAA10508 for obm-l-list; Mon, 13 May 2002 14:20:26 -0300 Received: from bidu.ime.usp.br (bidu.ime.usp.br [143.107.45.12]) by sucuri.mat.puc-rio.br (8.9.3/8.9.3) with SMTP id OAA10504 for ; Mon, 13 May 2002 14:20:24 -0300 Received: (qmail 24755 invoked from network); 13 May 2002 17:08:31 -0000 Received: from mafalda.ime.usp.br (143.107.45.13) by bidu.ime.usp.br with SMTP; 13 May 2002 17:08:31 -0000 Received: (qmail 18257 invoked by uid 1604); 13 May 2002 17:08:29 -0000 Date: Mon, 13 May 2002 14:08:29 -0300 (EST) From: Salvador Addas Zanata X-Sender: sazanata@mafalda To: obm-l@mat.puc-rio.br Subject: Re: [obm-l] .......... In-Reply-To: Message-ID: MIME-Version: 1.0 Content-Type: TEXT/PLAIN; charset=ISO-8859-1 Content-Transfer-Encoding: 8bit X-MIME-Autoconverted: from QUOTED-PRINTABLE to 8bit by sucuri.mat.puc-rio.br id OAA10505 Sender: owner-obm-l@sucuri.mat.puc-rio.br Precedence: bulk Reply-To: obm-l@mat.puc-rio.br Para o 1), tente fazer pro decagono, pegue um dos triangulos isoceles de angulo 36,72,72 tome a bissetriz de um dos angulos de 72 e observe relacoes de semelhanca. Ai, observe que dado o lado de um poligono de n lados, uma conta padrao permite obter o lado do poligono de 2n lados. Ai e so resolver isso as avessas. Existem jeitos mais bonitos de resolver esse prob., mas acho que isso funciona. Abraco, Salvador On Sat, 11 May 2002, rafaelc.l wrote: > > 1) Considere um pentágono regular de lado L inscrito numa > circunferência de raio R. > Existe alguma maneira de se obter o valor de L em função > de R sem usar a tebela dos arcos? > > 2)(IME-2001) Resolva a equação [5-(5-x)^1/2]^1/2=x sendo > x>0. > > > > > Obrigado > > > __________________________________________________________________________ > Quer ter seu próprio endereço na Internet? > Garanta já o seu e ainda ganhe cinco e-mails personalizados. > DomíniosBOL - http://dominios.bol.com.br > > > ========================================================================= > Instruções para entrar na lista, sair da lista e usar a lista em > http://www.mat.puc-rio.br/~nicolau/olimp/obm-l.html > O administrador desta lista é > ========================================================================= > ========================================================================= Instruções para entrar na lista, sair da lista e usar a lista em http://www.mat.puc-rio.br/~nicolau/olimp/obm-l.html O administrador desta lista é ========================================================================= From owner-obm-l@sucuri.mat.puc-rio.br Mon May 13 18:36:30 2002 Return-Path: Received: (from majordom@localhost) by sucuri.mat.puc-rio.br (8.9.3/8.9.3) id SAA15750 for obm-l-list; Mon, 13 May 2002 18:35:57 -0300 Received: from web10105.mail.yahoo.com (web10105.mail.yahoo.com [216.136.130.55]) by sucuri.mat.puc-rio.br (8.9.3/8.9.3) with SMTP id SAA15746 for ; Mon, 13 May 2002 18:35:54 -0300 Message-ID: <20020513212401.83458.qmail@web10105.mail.yahoo.com> Received: from [150.161.4.45] by web10105.mail.yahoo.com via HTTP; Mon, 13 May 2002 14:24:01 PDT Date: Mon, 13 May 2002 14:24:01 -0700 (PDT) From: Rafael WC Subject: Re: [obm-l] treino... To: OBM MIME-Version: 1.0 Content-Type: text/plain; charset=us-ascii Sender: owner-obm-l@sucuri.mat.puc-rio.br Precedence: bulk Reply-To: obm-l@mat.puc-rio.br --- "Nicolau C. Saldanha" wrote: > On Fri, May 10, 2002 at 10:23:06PM -0400, > DEOLIVEIRASOU@aol.com wrote: > > 1)prove que na P.A 5, 11, 17, 23, 29, 35,...... , > há infinitos números > > primos.... > Suponha por absurdo que fossem apenas p1, p2, ..., > pn. > Considere N = 6*p1*p2*...*pn - 1. > Pense em quais podem ser os fatores primos de N... > > []s, N. Por que 7 não pode dividir N? Rafael. ===== Rafael Werneck Cinoto ICQ# 107011599 rwcinoto@yahoo.com rafael.caixa@gov.com.br matduvidas@yahoo.com.br http://www.rwcinoto.hpg.com.br/ __________________________________________________ Do You Yahoo!? LAUNCH - Your Yahoo! Music Experience http://launch.yahoo.com ========================================================================= Instruções para entrar na lista, sair da lista e usar a lista em http://www.mat.puc-rio.br/~nicolau/olimp/obm-l.html O administrador desta lista é ========================================================================= From owner-obm-l@sucuri.mat.puc-rio.br Mon May 13 18:54:54 2002 Return-Path: Received: (from majordom@localhost) by sucuri.mat.puc-rio.br (8.9.3/8.9.3) id SAA16072 for obm-l-list; Mon, 13 May 2002 18:54:32 -0300 Received: from web10103.mail.yahoo.com (web10103.mail.yahoo.com [216.136.130.53]) by sucuri.mat.puc-rio.br (8.9.3/8.9.3) with SMTP id SAA16068 for ; Mon, 13 May 2002 18:54:29 -0300 Message-ID: <20020513214235.70767.qmail@web10103.mail.yahoo.com> Received: from [150.161.4.45] by web10103.mail.yahoo.com via HTTP; Mon, 13 May 2002 14:42:35 PDT Date: Mon, 13 May 2002 14:42:35 -0700 (PDT) From: Rafael WC Subject: [obm-l] círculos To: OBM MIME-Version: 1.0 Content-Type: multipart/mixed; boundary="0-1804611008-1021326155=:70311" Sender: owner-obm-l@sucuri.mat.puc-rio.br Precedence: bulk Reply-To: obm-l@mat.puc-rio.br --0-1804611008-1021326155=:70311 Content-Type: text/plain; charset=us-ascii Content-Disposition: inline Oi Pessoal! Me perguntaram como se resolve esta questão, (que parece que é de um livro do Machado) só que eu nunca ouvi falar de eixo radical: "Considere os círculos da figura de raios 10 e 4 e seu eixo radical. Se AT é tangente em J ao círculo menor, calcule a área do triângulo ATH. AB = 5." (segue uma figura anexada) Pela resposta do problema, que é 11881/150, eu descobri que a distância da reta TH até o círculo maior (CH) é 0,9. Se eu soubesse essa distância CH eu saberia resolver, mas parece que para isso preciso saber a definição de eixo radical. Alguém sabe? Um abraço, Rafael. ===== Rafael Werneck Cinoto ICQ# 107011599 rwcinoto@yahoo.com rafael.caixa@gov.com.br matduvidas@yahoo.com.br http://www.rwcinoto.hpg.com.br/ __________________________________________________ Do You Yahoo!? LAUNCH - Your Yahoo! Music Experience http://launch.yahoo.com --0-1804611008-1021326155=:70311 Content-Type: image/gif; name="eixoradical.gif" Content-Transfer-Encoding: base64 Content-Description: eixoradical.gif Content-Disposition: inline; filename="eixoradical.gif" R0lGODdhAwEBAfcAAAAAAAAAQAAAgAAA/wAgAAAgQAAggAAg/wBAAABAQABA gABA/wBgAABgQABggABg/wCAAACAQACAgACA/wCgAACgQACggACg/wDAAADA QADAgADA/wD/AAD/QAD/gAD//yAAACAAQCAAgCAA/yAgACAgQCAggCAg/yBA ACBAQCBAgCBA/yBgACBgQCBggCBg/yCAACCAQCCAgCCA/yCgACCgQCCggCCg /yDAACDAQCDAgCDA/yD/ACD/QCD/gCD//0AAAEAAQEAAgEAA/0AgAEAgQEAg gEAg/0BAAEBAQEBAgEBA/0BgAEBgQEBggEBg/0CAAECAQECAgECA/0CgAECg QECggECg/0DAAEDAQEDAgEDA/0D/AED/QED/gED//2AAAGAAQGAAgGAA/2Ag AGAgQGAggGAg/2BAAGBAQGBAgGBA/2BgAGBgQGBggGBg/2CAAGCAQGCAgGCA /2CgAGCgQGCggGCg/2DAAGDAQGDAgGDA/2D/AGD/QGD/gGD//4AAAIAAQIAA gIAA/4AgAIAgQIAggIAg/4BAAIBAQIBAgIBA/4BgAIBgQIBggIBg/4CAAICA QICAgICA/4CgAICgQICggICg/4DAAIDAQIDAgIDA/4D/AID/QID/gID//6AA AKAAQKAAgKAA/6AgAKAgQKAggKAg/6BAAKBAQKBAgKBA/6BgAKBgQKBggKBg /6CAAKCAQKCAgKCA/6CgAKCgQKCggKCg/6DAAKDAQKDAgKDA/6D/AKD/QKD/ gKD//8AAAMAAQMAAgMAA/8AgAMAgQMAggMAg/8BAAMBAQMBAgMBA/8BgAMBg QMBggMBg/8CAAMCAQMCAgMCA/8CgAMCgQMCggMCg/8DAAMDAQMDAgMDA/8D/ AMD/QMD/gMD///8AAP8AQP8AgP8A//8gAP8gQP8ggP8g//9AAP9AQP9AgP9A //9gAP9gQP9ggP9g//+AAP+AQP+AgP+A//+gAP+gQP+ggP+g///AAP/AQP/A gP/A////AP//QP//gP///yH5BAAAAAAALAAAAAADAQEBAAj/AP8JHEiwoMGD CBMqXMiwocOHECNKnEixosWLGDNq3Mixo8ePIEOKHEmypMmTKFOqXMmypcuX MGOuBECzJgCZOHPqjHlzp8+fQEf2DEq0qFGJQ48qXcr0X9KmUKPqfCq1qtWZ V7NqPUl1q9evEGka7Aq2rNmCNtGeXct2YNq2cNveFBu37tmeZO3qlTo0796/ SpP6BUwY6NPBhRPjpIpYseOWXRs/nswVoWTKmEPmvZy580a/nD2Ltgh6tGnN C0OfXp2aoWrWsNW6jk0bqcPXtVdfxp17NGfevTuHBh4c8/DiyAWqJp488Wvm zQHjhh5dL2/q1eMCx56dLXHu3c0yGgcf/it08uW3nk8vXCN69lWxv4cPlft8 +kvB/9/Hb5T8fv5BofcfgD69NyCBU4F0IIIyzbcggy/d9yCELO03IYUp/Xch hiYNuCGHQnUIolcLfjiiRw+aeCJHKa5o1YQqunjRhTG+aNONONZkHlas5TiR j1F9WGOAb310Y2CQZVYkSkvupOKQDdLFk5SLRaiYjj9hyZOV0lFJlJYVwgRl hmMiJeSWdYHJl5choilXmZ95WKV3cKIoYU51spinggImKN5f+jlZFptpUpfn nhURupeiYRVIonGkZakVo4VRmhCiiDbqW0SZZnqbbg956mlqoy6G2KilWpZb aUWlOlZwZKXqqluzFngYkvlVJxiuR9XaqnK59lqepYL2lxwesX5+eSywvBq2 rFvNOjrsq8Y6O+1BpWLK3ma//0p77aXKFvutQp16251k5SZ7rqbmurkup5LO Gd507XL57o/1JjnvjOLae2+i6uq7b0ZlDulrtQTjKea4CTfor65GOuzSweGi KHGY/1q8sMDRWehuhhkruLFKFAco4sQYQ3wyx5VlByPKJLscM8sllZxvzTST ZHPATOYs0s4Kwzxzy8nVeGbPzRmdMs6bNgTlhi023drFSDMtNbiW5agmi0Pr fNpgjG2NltiRgryyaKwqh+xYa89mttdfY002p21jTXSbpsXKbEffvY23ktC+ WjfdAFf9s9SRyUlR1H8LR+jgpD1nOGqxiQW1bXeLXPnccLPbOeWwNem305lH TBvnVjiTfrbmoQfuM7al8326bK9T+7npPUo5l9Bu365xj8A+/WnquMft+seq Ev973mDXfvzhoDOvOo9TN/8u+9e7Oe+U8lznvX2oSydvffdo7z386LD7Tn57 3yMsPvSss+9q8+p/Vr758E5uO/zLQ4o/5rEjiIaiR5ld8SuAzyve9fyXQHyt bn/xW+BkGMMU+vFPgo6hYFPSRsD1XSl99qvf//rnwUq9zz0ibF8HQ5jBE6Lw grRb4Qu90zsIslCBMYxgCcGyO2xRykEGupSGgkjDxGntiEhMohKXyMQmOvGJ UDwiueSiQlr9kE+Qm97Yshgq1C2OhiO0FBB1SCudEfEuDdRitHKIwxvyEEcO rOAUxzfDQbHxfMKao9oCRcZJgdBzZhJdGwXoOvvk8DpotKEa85fGHRKyivI5 ZOHGvDfJEWKQWg6SZCXvF0dLutF2mXxkIxfJvh8diY6FJCCQvmi8TlaRhLQL 5fO2AzxX3ql6t6ojJMvWSkZ6smF6bJ8hRfnLGl4NkLcMplO4aDf80bKWdBPk J/+oyjsC8phBqx4qdYlNeSlThtPkZDa1CcNLUtKb5AQnN89JtXT20ZHifNg3 1blOdmqPmNuspz27Rsp3hrOX6DNmOWEJKn7205//xB5XJIdAeEJzoAcdJEFD N0ZWNtShrZsoHrkn0dMxU4AfLaZG/80Jqyx6caMc7WhJxXjSa6aQpLpCIj1V ClMZ+fKiNbUp+AyaT526M6UI9ek8gUpTobrwgS81qiKR2lOl4lN/RHWqSCFK ValOdaZBVWqMjkafPvEuoDz8KjDvidUXifWA8oTqjsi607SqVT2JkqZAeelW nJrVoptEa13type8AhAjYzJYIv/qV1fula/1UWLWyMZEfVLvsH3tJL0AyzO2 BkuvLr0p8s56V8y2Fa/ohOwG8TrZwor2rUGKKzOf2a92dvaxmg3tZlcF2sqO DEPpai2CRGUtBqHKZLt1380Y1tvibmuNw+3YZSs2MOTGK2TdyqPK5Lhc5FDM VyG9UsmuCxur11J3dt6tINAWmhWgjTepzv2uZ8571VaxV09lTn3vWLUq38XV dy3ZXZNVj0rF/QqxUP51zX1BGuAuDmrAj2npl/LrXwbPBMHMm5+DC8w2TE2Y wuS6cOQgHFMNO03BGBaKXAE74hCLCY6BLLGJifTEFbv4xTCOsYxnTOMa23gh AQEAOw== --0-1804611008-1021326155=:70311-- ========================================================================= Instruções para entrar na lista, sair da lista e usar a lista em http://www.mat.puc-rio.br/~nicolau/olimp/obm-l.html O administrador desta lista é ========================================================================= From owner-obm-l@sucuri.mat.puc-rio.br Mon May 13 18:57:58 2002 Return-Path: Received: (from majordom@localhost) by sucuri.mat.puc-rio.br (8.9.3/8.9.3) id SAA16194 for obm-l-list; Mon, 13 May 2002 18:57:53 -0300 Received: (from nicolau@localhost) by sucuri.mat.puc-rio.br (8.9.3/8.9.3) id SAA16189 for obm-l@mat.puc-rio.br; Mon, 13 May 2002 18:57:52 -0300 Date: Mon, 13 May 2002 18:57:52 -0300 From: "Nicolau C. Saldanha" To: obm-l@mat.puc-rio.br Subject: Re: [obm-l] treino... Message-ID: <20020513185752.C16042@sucuri.mat.puc-rio.br> References: <20020513212401.83458.qmail@web10105.mail.yahoo.com> Mime-Version: 1.0 Content-Type: text/plain; charset=iso-8859-1 Content-Disposition: inline Content-Transfer-Encoding: 8bit User-Agent: Mutt/1.2.5i In-Reply-To: <20020513212401.83458.qmail@web10105.mail.yahoo.com>; from rwcinoto@yahoo.com on Mon, May 13, 2002 at 02:24:01PM -0700 Sender: owner-obm-l@sucuri.mat.puc-rio.br Precedence: bulk Reply-To: obm-l@mat.puc-rio.br On Mon, May 13, 2002 at 02:24:01PM -0700, Rafael WC wrote: > --- "Nicolau C. Saldanha" > wrote: > > On Fri, May 10, 2002 at 10:23:06PM -0400, > > DEOLIVEIRASOU@aol.com wrote: > > > 1)prove que na P.A 5, 11, 17, 23, 29, 35,...... , > > há infinitos números > > > primos.... > > Suponha por absurdo que fossem apenas p1, p2, ..., > > pn. > > Considere N = 6*p1*p2*...*pn - 1. > > Pense em quais podem ser os fatores primos de N... > > > > []s, N. > > Por que 7 não pode dividir N? Pode, mas se *todos* os fatores de N forem da forma 6k+1 temos uma contradição... []s, N. ========================================================================= Instruções para entrar na lista, sair da lista e usar a lista em http://www.mat.puc-rio.br/~nicolau/olimp/obm-l.html O administrador desta lista é ========================================================================= From owner-obm-l@sucuri.mat.puc-rio.br Mon May 13 19:36:39 2002 Return-Path: Received: (from majordom@localhost) by sucuri.mat.puc-rio.br (8.9.3/8.9.3) id TAA17391 for obm-l-list; Mon, 13 May 2002 19:36:29 -0300 Received: from Euler.impa.br (euler.impa.br [147.65.1.3]) by sucuri.mat.puc-rio.br (8.9.3/8.9.3) with ESMTP id TAA17387 for ; Mon, 13 May 2002 19:36:26 -0300 Received: from [147.65.11.10] (dial10.impa.br [147.65.11.10]) by Euler.impa.br (8.11.6/8.11.6) with ESMTP id g4DMOXh25150 for ; Mon, 13 May 2002 19:24:33 -0300 (EST) Message-Id: <200205132224.g4DMOXh25150@Euler.impa.br> X-Mailer: Microsoft Outlook Express Macintosh Edition - 4.5 (0410) Date: Wed, 15 May 2002 19:22:35 -0300 Subject: Re: [obm-l] c=?ISO-8859-1?B?zA==?=rculos From: "Eduardo Wagner" To: obm-l@mat.puc-rio.br Mime-version: 1.0 X-Priority: 3 Content-type: text/plain; charset="ISO-8859-1" Content-Transfer-Encoding: 8bit X-MIME-Autoconverted: from quoted-printable to 8bit by sucuri.mat.puc-rio.br id TAA17388 Sender: owner-obm-l@sucuri.mat.puc-rio.br Precedence: bulk Reply-To: obm-l@mat.puc-rio.br O eixo radical de duas circunferências eh o conjunto dos pontos que possuem mesma potencia em relacao a essas circunferencias. Dada uma circunferencia de centro O e raio R, e um ponto P do seu plano, a potencia de P em relacao a essa circunferencia eh definida por OP^2 - R^2. ---------- >From: Rafael WC >To: OBM >Subject: [obm-l] cÌrculos >Date: Mon, May 13, 2002, 6:42 PM > > Oi Pessoal! > > Me perguntaram como se resolve esta quest“o, (que > parece que È de um livro do Machado) sÛ que eu nunca > ouvi falar de eixo radical: > > "Considere os cÌrculos da figura de raios 10 e 4 e seu > eixo radical. Se AT È tangente em J ao cÌrculo menor, > calcule a ·rea do tri’ngulo ATH. AB = 5." (segue uma > figura anexada) > > Pela resposta do problema, que È 11881/150, eu > descobri que a dist’ncia da reta TH atÈ o cÌrculo > maior (CH) È 0,9. > > Se eu soubesse essa dist’ncia CH eu saberia resolver, > mas parece que para isso preciso saber a definiÁ“o de > eixo radical. AlguÈm sabe? > > Um abraÁo, > > Rafael. > > ===== > Rafael Werneck Cinoto > ICQ# 107011599 > rwcinoto@yahoo.com > rafael.caixa@gov.com.br > matduvidas@yahoo.com.br > http://www.rwcinoto.hpg.com.br/ > > __________________________________________________ > Do You Yahoo!? > LAUNCH - Your Yahoo! Music Experience > http://launch.yahoo.com ========================================================================= Instruções para entrar na lista, sair da lista e usar a lista em http://www.mat.puc-rio.br/~nicolau/olimp/obm-l.html O administrador desta lista é ========================================================================= From owner-obm-l@sucuri.mat.puc-rio.br Mon May 13 20:38:48 2002 Return-Path: Received: (from majordom@localhost) by sucuri.mat.puc-rio.br (8.9.3/8.9.3) id UAA18417 for obm-l-list; Mon, 13 May 2002 20:38:23 -0300 Received: from imo-r05.mx.aol.com (imo-r05.mx.aol.com [152.163.225.101]) by sucuri.mat.puc-rio.br (8.9.3/8.9.3) with ESMTP id UAA18413 for ; Mon, 13 May 2002 20:38:21 -0300 From: DEOLIVEIRASOU@aol.com Received: from DEOLIVEIRASOU@aol.com by imo-r05.mx.aol.com (mail_out_v32.5.) id z.176.8327865 (4116) for ; Mon, 13 May 2002 19:26:21 -0400 (EDT) Message-ID: <176.8327865.2a11a59d@aol.com> Date: Mon, 13 May 2002 19:26:21 EDT Subject: [obm-l] Lista para treino.... To: obm-l@mat.puc-rio.br MIME-Version: 1.0 Content-Type: multipart/alternative; boundary="part1_176.8327865.2a11a59d_boundary" X-Mailer: AOL 7.0 for Windows BR sub 10501 Sender: owner-obm-l@sucuri.mat.puc-rio.br Precedence: bulk Reply-To: obm-l@mat.puc-rio.br --part1_176.8327865.2a11a59d_boundary Content-Type: text/plain; charset="ISO-8859-1" Content-Transfer-Encoding: quoted-printable 1)Em uma dessas listas pra treino para olimp=EDadas, o sujeito pede para=20 calcularmos a soma de todos os divisores positivos de n? Existe alguma=20 f=F3rmula para isso? 2)Sendo N o n=FAmero de divisores positivos de n, determine, em fun=E7=E3o d= e n e N=20 o produto de todos os divisores de n. 3)Mostre que qualquer P.A, n=E3o constante, de inteiros possui uma infinidad= e=20 de valores compostos. Agrade=E7o de antem=E3o a quem resolver. Crom --part1_176.8327865.2a11a59d_boundary Content-Type: text/html; charset="ISO-8859-1" Content-Transfer-Encoding: quoted-printable 1)Em uma dessas listas pra treino para olimp=EDadas, o= sujeito pede para calcularmos a soma de todos os divisores positivos de n?=20= Existe alguma f=F3rmula para isso?
2)Sendo N o n=FAmero de divisores positivos de n, determine, em fun=E7=E3o d= e n e N o produto de todos os divisores de n.
3)Mostre que qualquer P.A, n=E3o constante, de inteiros possui uma infinidad= e de valores compostos.
   Agrade=E7o de antem=E3o a quem resolver.
            &nbs= p;             C= rom
--part1_176.8327865.2a11a59d_boundary-- ========================================================================= Instruções para entrar na lista, sair da lista e usar a lista em http://www.mat.puc-rio.br/~nicolau/olimp/obm-l.html O administrador desta lista é ========================================================================= From owner-obm-l@sucuri.mat.puc-rio.br Mon May 13 22:57:24 2002 Return-Path: Received: (from majordom@localhost) by sucuri.mat.puc-rio.br (8.9.3/8.9.3) id WAA20586 for obm-l-list; Mon, 13 May 2002 22:56:55 -0300 Received: from home.iis.com.br (mail.iis.com.br [200.202.96.2]) by sucuri.mat.puc-rio.br (8.9.3/8.9.3) with ESMTP id WAA20581 for ; Mon, 13 May 2002 22:56:53 -0300 Received: from Marcio (rio-tc0-tty44.iis.com.br [200.202.98.44]) by home.iis.com.br (8.11.6/8.11.6/1.1.1.16) with SMTP id g4E1ixV01820 for ; Mon, 13 May 2002 22:45:00 -0300 Message-ID: <002501c1fae8$fda525e0$2c62cac8@epq.ime.eb.br> From: "Marcio" To: References: <176.8327865.2a11a59d@aol.com> Subject: Re: [obm-l] Lista para treino.... Date: Mon, 13 May 2002 22:45:11 -0300 MIME-Version: 1.0 Content-Type: multipart/alternative; boundary="----=_NextPart_000_0022_01C1FACF.D743BEC0" X-Priority: 3 X-MSMail-Priority: Normal X-Mailer: Microsoft Outlook Express 5.50.4133.2400 X-MimeOLE: Produced By Microsoft MimeOLE V5.50.4133.2400 X-AntiVirus: Antivirus for sendmail by Petr Rehor Sender: owner-obm-l@sucuri.mat.puc-rio.br Precedence: bulk Reply-To: obm-l@mat.puc-rio.br This is a multi-part message in MIME format. ------=_NextPart_000_0022_01C1FACF.D743BEC0 Content-Type: text/plain; charset="iso-8859-1" Content-Transfer-Encoding: quoted-printable 1) Sim, existe (pelo menos qdo vc conhece a fatoracao de n). eh o = problema proposto 40(a) da Eureka (eureka 8) em obm.org.br. 2) se n nao for quadrado perfeito, pra cada a|n existe um outro divisor = (n/a)|n. Juntando esses (N/2) pares vc nota que o produto dos divisores = eh n^(N/2). Se n for quadrado perfeito (N impar), entao vc consegue = (N-1)/2 pares e sobra apenas o proprio n^(1/2) como divisor, de modo = que o produto de todo mundo eh n^[(N+1)/2]. Juntando, vc pode dizer que = a resposta eh n^[teto(N/2)]. 3) Se a_n =3D a_0 + n*r, entao infinitos termos de indice n =3D k*a_0 = sao compostos (a_n =3D a_0 * (1+k) ). (Se a_0 for 1, olhe apenas para os = (infinitos) naturais tais que 1+k eh composto). Marcio ----- Original Message -----=20 From: DEOLIVEIRASOU@aol.com=20 To: obm-l@mat.puc-rio.br=20 Sent: Monday, May 13, 2002 8:26 PM Subject: [obm-l] Lista para treino.... 1)Em uma dessas listas pra treino para olimp=EDadas, o sujeito pede = para calcularmos a soma de todos os divisores positivos de n? Existe = alguma f=F3rmula para isso? 2)Sendo N o n=FAmero de divisores positivos de n, determine, em = fun=E7=E3o de n e N o produto de todos os divisores de n. 3)Mostre que qualquer P.A, n=E3o constante, de inteiros possui uma = infinidade de valores compostos. Agrade=E7o de antem=E3o a quem resolver. Crom=20 ------=_NextPart_000_0022_01C1FACF.D743BEC0 Content-Type: text/html; charset="iso-8859-1" Content-Transfer-Encoding: quoted-printable
1) Sim, existe (pelo menos qdo vc = conhece a=20 fatoracao de n). eh o problema proposto 40(a) da Eureka (eureka 8)  = em=20 obm.org.br.
2) se n nao for quadrado perfeito, pra = cada a|n=20 existe um outro divisor (n/a)|n. Juntando esses (N/2) pares vc nota = que o=20 produto dos divisores eh n^(N/2). Se n for quadrado perfeito (N impar), = entao vc=20 consegue (N-1)/2 pares e  sobra apenas o proprio n^(1/2) como = divisor, de=20 modo que o produto de todo mundo eh n^[(N+1)/2]. Juntando, vc pode dizer = que a=20 resposta eh n^[teto(N/2)].
3) Se a_n =3D a_0 + n*r, entao = infinitos termos=20 de indice n =3D k*a_0 sao compostos (a_n =3D a_0 * (1+k) ). (Se a_0 for = 1, olhe=20 apenas para os (infinitos) naturais tais que 1+k eh = composto).
 
Marcio
 
----- Original Message -----
From:=20 DEOLIVEIRASOU@aol.com
Sent: Monday, May 13, 2002 8:26 = PM
Subject: [obm-l] Lista para=20 treino....

1)Em uma dessas listas pra treino para = olimp=EDadas, o=20 sujeito pede para calcularmos a soma de todos os divisores positivos = de n?=20 Existe alguma f=F3rmula para isso?
2)Sendo N o n=FAmero de = divisores positivos=20 de n, determine, em fun=E7=E3o de n e N o produto de todos os = divisores de=20 n.
3)Mostre que qualquer P.A, n=E3o constante, de inteiros possui = uma=20 infinidade de valores compostos.
   Agrade=E7o de = antem=E3o a quem=20 = resolver.
          =             &= nbsp;  =20 Crom
------=_NextPart_000_0022_01C1FACF.D743BEC0-- ========================================================================= Instruções para entrar na lista, sair da lista e usar a lista em http://www.mat.puc-rio.br/~nicolau/olimp/obm-l.html O administrador desta lista é ========================================================================= From owner-obm-l@sucuri.mat.puc-rio.br Mon May 13 23:07:41 2002 Return-Path: Received: (from majordom@localhost) by sucuri.mat.puc-rio.br (8.9.3/8.9.3) id XAA20726 for obm-l-list; Mon, 13 May 2002 23:07:39 -0300 Received: from calhau.terra.com.br (calhau.terra.com.br [200.176.3.20]) by sucuri.mat.puc-rio.br (8.9.3/8.9.3) with ESMTP id XAA20722 for ; Mon, 13 May 2002 23:07:37 -0300 Received: from taipe.terra.com.br (taipe.terra.com.br [200.176.3.34]) by calhau.terra.com.br (Postfix) with ESMTP id 071B747F5D for ; Tue, 14 May 2002 01:55:46 +0000 (GMT) Received: from Itautec.terra.com.br (200-158-60-251.dsl.telesp.net.br [200.158.60.251]) (authenticated user bruleite) by taipe.terra.com.br (Postfix) with ESMTP id 5DD1B1B40A3 for ; Mon, 13 May 2002 22:55:45 -0300 (EST) Message-Id: <5.1.0.14.0.20020513223129.00ac0d70@pop.sao.terra.com.br> X-Sender: bruleite@pop.sao.terra.com.br X-Mailer: QUALCOMM Windows Eudora Version 5.1 Date: Mon, 13 May 2002 22:55:51 -0300 To: obm-l@mat.puc-rio.br From: "Bruno F. C. Leite" Subject: Re: [obm-l] Lista para treino.... In-Reply-To: <176.8327865.2a11a59d@aol.com> Mime-Version: 1.0 Content-Type: text/plain; charset="iso-8859-1"; format=flowed Content-Transfer-Encoding: 8bit X-MIME-Autoconverted: from quoted-printable to 8bit by sucuri.mat.puc-rio.br id XAA20723 Sender: owner-obm-l@sucuri.mat.puc-rio.br Precedence: bulk Reply-To: obm-l@mat.puc-rio.br At 19:26 13/05/02 -0400, you wrote: >1)Em uma dessas listas pra treino para olimpíadas, o sujeito pede para >calcularmos a soma de todos os divisores positivos de n? Existe alguma >fórmula para isso? existe, em termos da fatoração de n em primos. Fica fácil se vc provar que se mdc(a,b)=1, então soma_divisores(a) soma_divisores(b)=soma_divisores(ab). (e aí só falta saber soma_divisores(p^n), onde p é primo- mas isso é fácil mesmo) >2)Sendo N o número de divisores positivos de n, determine, em função de n >e N o produto de todos os divisores de n. Se d divide n, n/d divide n. Agrupe os divisores dessa forma...(não esqueça o caso em que n é quadrado) >3)Mostre que qualquer P.A, não constante, de inteiros possui uma >infinidade de valores compostos. Suponha que só possua finitos valores compostos. Então, a partir de um ponto, todos os valores da PA são primos. Seja a+kb a sua PA, com k=0,1,2.... Se k>=k_0, então a+bk é primo. Temos que a deve ser ímpar e b par. Tome k=ak_0>=k_0. Então a+bak_0 é primo, logo a=1. Agora tomando k=2b^(n-1)+b^(2n-1), a+bk=1+2b^n+b^(2n)=(b^n+1)^2 é primo se n for suficientemente grande, o que é absurdo. Está confuso (e deve ter solução mais simples) mas acho que está certo. Bruno Leite http://www.ime.usp.br/~brleite > Agradeço de antemão a quem resolver. > Crom ========================================================================= Instruções para entrar na lista, sair da lista e usar a lista em http://www.mat.puc-rio.br/~nicolau/olimp/obm-l.html O administrador desta lista é ========================================================================= From owner-obm-l@sucuri.mat.puc-rio.br Mon May 13 23:27:44 2002 Return-Path: Received: (from majordom@localhost) by sucuri.mat.puc-rio.br (8.9.3/8.9.3) id XAA21408 for obm-l-list; Mon, 13 May 2002 23:27:36 -0300 Received: from smtp-5.ig.com.br (smtp-5.ig.com.br [200.226.132.154] (may be forged)) by sucuri.mat.puc-rio.br (8.9.3/8.9.3) with SMTP id XAA21404 for ; Mon, 13 May 2002 23:27:35 -0300 Received: (qmail 3606 invoked from network); 14 May 2002 02:15:28 -0000 Received: from 109.154.225.200.in-addr.arpa.ig.com.br (HELO oemcomputer) (200.225.154.109) by smtp-5.ig.com.br with SMTP; 14 May 2002 02:15:28 -0000 Message-ID: <004301c1faef$4eb691c0$6d9ae1c8@oemcomputer> From: "Daniel" To: References: Subject: Re: [obm-l] .......... Date: Mon, 13 May 2002 23:27:47 -0300 MIME-Version: 1.0 Content-Type: text/plain; charset="iso-8859-1" Content-Transfer-Encoding: 8bit X-Priority: 3 X-MSMail-Priority: Normal X-Mailer: Microsoft Outlook Express 5.00.2615.200 X-MIMEOLE: Produced By Microsoft MimeOLE V5.00.2615.200 Sender: owner-obm-l@sucuri.mat.puc-rio.br Precedence: bulk Reply-To: obm-l@mat.puc-rio.br > > 2)(IME-2001) Resolva a equação [5-(5-x)^1/2]^1/2=x sendo > x>0. Chame sqrt (5 - x) = x Daí temos: 5 - x = x^2 x^2 + x - 5 = 0 x = [-1 + sqrt(21)]/2 ou x = [-1- sqrt(21)]/2 Daniel ========================================================================= Instruções para entrar na lista, sair da lista e usar a lista em http://www.mat.puc-rio.br/~nicolau/olimp/obm-l.html O administrador desta lista é ========================================================================= From owner-obm-l@sucuri.mat.puc-rio.br Tue May 14 00:23:28 2002 Return-Path: Received: (from majordom@localhost) by sucuri.mat.puc-rio.br (8.9.3/8.9.3) id AAA22308 for obm-l-list; Tue, 14 May 2002 00:23:02 -0300 Received: from imo-m06.mx.aol.com (imo-m06.mx.aol.com [64.12.136.161]) by sucuri.mat.puc-rio.br (8.9.3/8.9.3) with ESMTP id AAA22304 for ; Tue, 14 May 2002 00:22:59 -0300 From: DEOLIVEIRASOU@aol.com Received: from DEOLIVEIRASOU@aol.com by imo-m06.mx.aol.com (mail_out_v32.5.) id z.4e.b6ada3a (4363) for ; Mon, 13 May 2002 23:10:58 -0400 (EDT) Message-ID: <4e.b6ada3a.2a11da41@aol.com> Date: Mon, 13 May 2002 23:10:57 EDT Subject: [obm-l] primos To: obm-l@mat.puc-rio.br MIME-Version: 1.0 Content-Type: multipart/alternative; boundary="part1_4e.b6ada3a.2a11da41_boundary" X-Mailer: AOL 7.0 for Windows BR sub 10501 Sender: owner-obm-l@sucuri.mat.puc-rio.br Precedence: bulk Reply-To: obm-l@mat.puc-rio.br --part1_4e.b6ada3a.2a11da41_boundary Content-Type: text/plain; charset="ISO-8859-1" Content-Transfer-Encoding: quoted-printable Se a^n-1 =E9 primo, com n>1, ent=E3o a=3D2 e n =E9 primo. Acho que vi algo p= arecido,=20 mas era uma varia=E7=E3o dessa proposi=E7=E3o aqui na lista....Como posso=20 demonstrar?? Valeu Crom --part1_4e.b6ada3a.2a11da41_boundary Content-Type: text/html; charset="ISO-8859-1" Content-Transfer-Encoding: quoted-printable Se a^n-1 =E9 primo, com n>1, ent=E3o a=3D2 e n =E9=20= primo. Acho que vi algo parecido, mas era uma varia=E7=E3o dessa proposi=E7= =E3o aqui na lista....Como posso demonstrar??
            &nbs= p;           Valeu
            &nbs= p;            &n= bsp;      Crom
--part1_4e.b6ada3a.2a11da41_boundary-- ========================================================================= Instruções para entrar na lista, sair da lista e usar a lista em http://www.mat.puc-rio.br/~nicolau/olimp/obm-l.html O administrador desta lista é ========================================================================= From owner-obm-l@sucuri.mat.puc-rio.br Tue May 14 09:27:56 2002 Return-Path: Received: (from majordom@localhost) by sucuri.mat.puc-rio.br (8.9.3/8.9.3) id JAA26016 for obm-l-list; Tue, 14 May 2002 09:27:33 -0300 Received: from hotmail.com (f82.sea1.hotmail.com [207.68.163.82]) by sucuri.mat.puc-rio.br (8.9.3/8.9.3) with ESMTP id JAA26012 for ; Tue, 14 May 2002 09:27:31 -0300 Received: from mail pickup service by hotmail.com with Microsoft SMTPSVC; Tue, 14 May 2002 05:15:39 -0700 Received: from 200.227.148.43 by sea1fd.sea1.hotmail.msn.com with HTTP; Tue, 14 May 2002 12:15:39 GMT X-Originating-IP: [200.227.148.43] From: "Frederico Reis Marques de Brito" To: obm-l@mat.puc-rio.br Subject: Re: [obm-l] primos Date: Tue, 14 May 2002 09:15:39 -0300 Mime-Version: 1.0 Content-Type: text/plain; charset=iso-8859-1; format=flowed Message-ID: X-OriginalArrivalTime: 14 May 2002 12:15:39.0414 (UTC) FILETIME=[0F795360:01C1FB41] Sender: owner-obm-l@sucuri.mat.puc-rio.br Precedence: bulk Reply-To: obm-l@mat.puc-rio.br Vou te dar umas dicas que facilitarão bastante. Para ver que n é primo, observe que se n = b . c então a^(bc) - 1 = (a^b)^c - 1 = ( a^b - 1 ) . ( **** ) , pode ser convenientemente fatorado.( Lembre-se que da fórmula da soma dos termos de uma PG de razão y e termo inicial 1 : 1 + y + y^2 +... + y^k = [ y^(n+1) -1]/ y - 1 ). Bom, agora resta provar que a = 2 . Novamente usando a relação acima, vemos que : a^n - 1 = ( a - 1 ). ( 1 + a + a^2 + ... + a^(n-1)). Para que seja primo, a - 1 = 1 => a = 2 ou 1 + a + ... + a^(n-1) = 1 => a = 0 , e neste caso??? BOa sorte. Fred. >From: DEOLIVEIRASOU@aol.com >Reply-To: obm-l@mat.puc-rio.br >To: obm-l@mat.puc-rio.br >Subject: [obm-l] primos >Date: Mon, 13 May 2002 23:10:57 EDT > >Se a^n-1 é primo, com n>1, então a=2 e n é primo. Acho que vi algo >parecido, >mas era uma variação dessa proposição aqui na lista....Como posso >demonstrar?? > Valeu > Crom _________________________________________________________________ Converse com amigos on-line, conheça o MSN Messenger: http://messenger.msn.com ========================================================================= Instruções para entrar na lista, sair da lista e usar a lista em http://www.mat.puc-rio.br/~nicolau/olimp/obm-l.html O administrador desta lista é ========================================================================= From owner-obm-l@sucuri.mat.puc-rio.br Tue May 14 09:59:51 2002 Return-Path: Received: (from majordom@localhost) by sucuri.mat.puc-rio.br (8.9.3/8.9.3) id JAA27041 for obm-l-list; Tue, 14 May 2002 09:59:35 -0300 Received: from Euler.impa.br (euler.impa.br [147.65.1.3]) by sucuri.mat.puc-rio.br (8.9.3/8.9.3) with ESMTP id JAA27037 for ; Tue, 14 May 2002 09:59:33 -0300 Received: from obm-01 (obm-01.impa.br [147.65.2.170]) by Euler.impa.br (8.11.6/8.11.6) with SMTP id g4EClgh06293 for ; Tue, 14 May 2002 09:47:42 -0300 (EST) Message-Id: <3.0.5.32.20020514094839.0081a6d0@pop.impa.br> X-Sender: obm@pop.impa.br X-Mailer: QUALCOMM Windows Eudora Light Version 3.0.5 (32) Date: Tue, 14 May 2002 09:48:39 -0300 To: obm-l@mat.puc-rio.br From: Olimpiada Brasileira de Matematica Subject: [obm-l] Revista Olimpica on-line. Mime-Version: 1.0 Content-Type: text/plain; charset="us-ascii" Sender: owner-obm-l@sucuri.mat.puc-rio.br Precedence: bulk Reply-To: obm-l@mat.puc-rio.br Caros(as) amigos(as) da lista: Esta on-line o numero 1 da Revista Escolar da Olimpiada Iberoamericana de Matematica, o endereco para dar uma olhada e' o seguinte: http://www.campus-oei.org/oim/revistaoim/numero1.htm A revista esta em espanhol e seus conteudos sao os seguintes: Articulos, Notas y Lecciones de preparacion olimpica F.Bellot : Los teoremas de Ptolomeo y su generalizacion por Casey. Aplicaciones Problemas para alumnos de Educacion Media y de Olimpiadas Problemas de la Olimpiada Pan Africana 2000 Problemas para los mas jovenes Problemas del Concurso Canguro Matematico 2002 (Nivel 1, 12-13 anos de edad) Problemas propuestos En este apartado se invita a los lectores a resolver cinco problemas y enviarnos sus soluciones. Las mas originales seran publicadas. Divertimentos matematicos El beso preciso, poema de Sir Frederic Soddy (1937) Abracos, Nelly. ========================================================================= Instruções para entrar na lista, sair da lista e usar a lista em http://www.mat.puc-rio.br/~nicolau/olimp/obm-l.html O administrador desta lista é ========================================================================= From owner-obm-l@sucuri.mat.puc-rio.br Tue May 14 10:54:56 2002 Return-Path: Received: (from majordom@localhost) by sucuri.mat.puc-rio.br (8.9.3/8.9.3) id KAA28286 for obm-l-list; Tue, 14 May 2002 10:52:05 -0300 Received: from sr1.terra.com.br (sr1.terra.com.br [200.176.3.16]) by sucuri.mat.puc-rio.br (8.9.3/8.9.3) with ESMTP id KAA28273 for ; Tue, 14 May 2002 10:52:03 -0300 Received: from bica.terra.com.br (bica.terra.com.br [200.176.3.32]) by sr1.terra.com.br (Postfix) with ESMTP id 14BE07150B for ; Tue, 14 May 2002 10:40:13 -0300 (EST) Received: from Lucelindo (unknown [200.177.64.158]) (authenticated user ldias1) by bica.terra.com.br (Postfix) with ESMTP id F36FCD5473 for ; Tue, 14 May 2002 10:40:11 -0300 (EST) Message-ID: <001801c1fb4c$540cdd20$9e40b1c8@Lucelindo> From: "Lucelindo D. Ferreira" To: Subject: [obm-l] algebrismos Date: Tue, 14 May 2002 10:36:16 -0300 MIME-Version: 1.0 Content-Type: multipart/alternative; boundary="----=_NextPart_000_0015_01C1FB33.2D453A20" X-Priority: 3 X-MSMail-Priority: Normal X-Mailer: Microsoft Outlook Express 5.00.2615.200 X-MimeOLE: Produced By Microsoft MimeOLE V5.00.2615.200 Sender: owner-obm-l@sucuri.mat.puc-rio.br Precedence: bulk Reply-To: obm-l@mat.puc-rio.br This is a multi-part message in MIME format. ------=_NextPart_000_0015_01C1FB33.2D453A20 Content-Type: text/plain; charset="iso-8859-1" Content-Transfer-Encoding: quoted-printable Bom dia galera. Eu queria uma m=E3o nesse problema da olimp=EDada russa. = Eu come=E7ei a resolver... Prove que=20 a^2 + b^2 + b^2 + c^2 + a^2 + c^2 =3D< 6R 2mc 2ma 2mb Nota=E7=E3o: a,b e c s=E3o lados do tri=E2ngulo inscrito numa = circunfer=EAncia de raio R. ma, mb e mc s=E3o as medianas relativas a a,b e c. Pelo teorema isoperim=E9trico eu sei que 3Rsqrt3 >=3D a + b + c , 3R^3sqrt3 >=3D abc e 3R^2sqrt3 >=3D sqrt[(a + = b + c)(b + c - a)(a + b - c)(a + c - b)]=20 =20 a^2 + b^2 + b^2 + c^2 + a^2 + c^2 =3D< 6R 2mc 2ma 2mb sabendo que ma =3D 1/2sqrt[2(b^2 + c^2) - a^2] mb =3D 1/2sqrt[2(a^2 + c^2) - b^2] e mc =3D 1/2sqrt[2(b^2 + a^2) - = c^2]... Empaquei a=ED e gostaria de sugest=F5es para continuar a solu=E7=E3o =20 Agrade=E7o qualquer sugest=E3o ------=_NextPart_000_0015_01C1FB33.2D453A20 Content-Type: text/html; charset="iso-8859-1" Content-Transfer-Encoding: quoted-printable
Bom dia galera. Eu queria uma m=E3o = nesse problema da=20 olimp=EDada russa. Eu come=E7ei a resolver...
Prove que=20
a^2 + b^2 + b^2 + = c^2  + a^2=20 + c^2 =3D< 6R
    =20 2mc        =20 2ma           &nbs= p; =20 2mb
Nota=E7=E3o: a,b e c s=E3o lados do tri=E2ngulo inscrito numa = circunfer=EAncia de=20 raio R.
ma, mb e mc s=E3o as medianas relativas a a,b e = c.
Pelo teorema isoperim=E9trico eu sei = que
3Rsqrt3 >=3D a + b + c , = 3R^3sqrt3 =20 >=3D abc e 3R^2sqrt3 >=3D sqrt[(a + b + c)(b + c - a)(a + b - = c)(a + c -=20 b)]
          &nbs= p;            = ;            =             &= nbsp;           &n= bsp;         =20
a^2 + b^2 + b^2 + = c^2  + a^2=20 + c^2 =3D< 6R
    =20 2mc        =20 2ma           &nbs= p; =20 2mb
sabendo que ma =3D 1/2sqrt[2(b^2 + c^2) = -=20 a^2]
mb =3D 1/2sqrt[2(a^2 + c^2) - b^2] e mc = =3D=20 1/2sqrt[2(b^2 + a^2) - c^2]...
Empaquei a=ED e gostaria de = sugest=F5es para=20 continuar a solu=E7=E3o
Agrade=E7o qualquer=20 sugest=E3o
------=_NextPart_000_0015_01C1FB33.2D453A20-- ========================================================================= Instruções para entrar na lista, sair da lista e usar a lista em http://www.mat.puc-rio.br/~nicolau/olimp/obm-l.html O administrador desta lista é ========================================================================= From owner-obm-l@sucuri.mat.puc-rio.br Tue May 14 15:34:38 2002 Return-Path: Received: (from majordom@localhost) by sucuri.mat.puc-rio.br (8.9.3/8.9.3) id PAA32569 for obm-l-list; Tue, 14 May 2002 15:33:01 -0300 Received: from prointer.net (smtp.prointernet.com.br [200.194.176.12]) by sucuri.mat.puc-rio.br (8.9.3/8.9.3) with SMTP id PAA32558 for ; Tue, 14 May 2002 15:32:47 -0300 Received: (qmail 27490 invoked from network); 14 May 2002 18:21:45 -0000 Received: from pib-dial-1482.access.prointernet.com.br (HELO globalx) (200.194.181.171) by smtp.prointernet.com.br with SMTP; 14 May 2002 18:21:45 -0000 Message-ID: <000401c1fb73$896e42c0$abb5c2c8@globalx> From: "Anderson" To: References: <5.1.0.14.0.20020510094130.00ac0b90@pop.sao.terra.com.br> Subject: Re: [obm-l] Inducao Date: Mon, 13 May 2002 10:52:23 -0300 MIME-Version: 1.0 Content-Type: text/plain; charset="iso-8859-1" Content-Transfer-Encoding: 7bit X-Priority: 3 X-MSMail-Priority: Normal X-Mailer: Microsoft Outlook Express 5.00.2919.6600 X-MIMEOLE: Produced By Microsoft MimeOLE V5.00.2919.6600 X-AntiVirus: scanned for viruses by Prointernet Brasil (http://www.pib.com.br/) Sender: owner-obm-l@sucuri.mat.puc-rio.br Precedence: bulk Reply-To: obm-l@mat.puc-rio.br ----- Original Message ----- From: "Bruno F. C. Leite" To: Sent: Friday, May 10, 2002 9:54 AM Subject: Re: [obm-l] Inducao > At 23:26 09/05/02 -0300, you wrote: > >Oi, > > Estou com problemas nos conceitos do metodo de prova da inducao > >matematica, alguem poderia ajduar? Vejam os exemplos abaixo e por favor > >tentem me explicar o q esta errado ... ah, os problemas foram tirados do > >livro do knuth... > > Que livro do Knuth? Fundamental Algorithms .... Obrigado pelas respostas! []s Anderson ========================================================================= Instruções para entrar na lista, sair da lista e usar a lista em http://www.mat.puc-rio.br/~nicolau/olimp/obm-l.html O administrador desta lista é ========================================================================= From owner-obm-l@sucuri.mat.puc-rio.br Tue May 14 17:30:46 2002 Return-Path: Received: (from majordom@localhost) by sucuri.mat.puc-rio.br (8.9.3/8.9.3) id RAA02339 for obm-l-list; Tue, 14 May 2002 17:29:49 -0300 Received: from www.zipmail.com.br (smtp.zipmail.com.br [200.187.242.10]) by sucuri.mat.puc-rio.br (8.9.3/8.9.3) with ESMTP id RAA02335 for ; Tue, 14 May 2002 17:29:47 -0300 From: peterdirichlet@zipmail.com.br Received: from [200.206.103.3] by www.zipmail.com.br with HTTP; Tue, 14 May 2002 17:27:04 -0300 Message-ID: <3CE14E0300000DD4@www.zipmail.com.br> Date: Tue, 14 May 2002 17:27:04 -0300 Subject: [obm-l] =?iso-8859-1?Q?Mais=20da=20Iberoamericana?= To: =?iso-8859-1?Q?Antonio=20Neto?= , =?iso-8859-1?Q?JP?= , =?iso-8859-1?Q?Lista=20de=20Discussao?= , =?iso-8859-1?Q?Celso?= MIME-Version: 1.0 Content-Type: text/plain; charset="iso-8859-1" Content-Transfer-Encoding: 8bit X-MIME-Autoconverted: from quoted-printable to 8bit by sucuri.mat.puc-rio.br id RAA02336 Sender: owner-obm-l@sucuri.mat.puc-rio.br Precedence: bulk Reply-To: obm-l@mat.puc-rio.br Alo turma!!!!!!Tenho mais perguntas a fazer(da Iberoamericana): 1)Ache todos os naturais n de 3,2 ou 1 digito tal que o quadrado de n seja o cubo da soma dos digitos. 2)Encontre o menor n tal que se pegarmos n dos 999 primeiros inteiros positivos sempre se acham 4 numeros diferentes a,b,c,d com a+2*b+3*c-4*d=0. 3)L>0 e tal que -L^2+1998*L+1=0.Seja a recorrencia a(0)=1 e a(n+1)=parte inteira de L*a(n)=[L*a(n)].Calcule a(1998)mod 1998(x mod y e o resto de x por y). 4)na mesa da banca de lideres da OIM estao lideres de P paises de modo que se dois lideres quaisquer sao de mesmo pais entao seus vizinhos direitos nao sao.Quantos lideres ha no maximo? 5)Sabe-se que num conjunto de primos se p e q sao elementos(iguais ou nao)entao p*q+r,em que r e constante.Quantos elementos tem S com n=4?Generalize o r. TRANSIRE SVVM PECTVS MVNDOQUE POTIRE CONGREGATI EX TOTO ORBE MATHEMATICI OB SCRIPTA INSIGNIA TRIBVERE Medalha Fields(John Charles Fields) ------------------------------------------ Use o melhor sistema de busca da Internet Radar UOL - http://www.radaruol.com.br ========================================================================= Instruções para entrar na lista, sair da lista e usar a lista em http://www.mat.puc-rio.br/~nicolau/olimp/obm-l.html O administrador desta lista é ========================================================================= From owner-obm-l@sucuri.mat.puc-rio.br Tue May 14 21:52:55 2002 Return-Path: Received: (from majordom@localhost) by sucuri.mat.puc-rio.br (8.9.3/8.9.3) id VAA05176 for obm-l-list; Tue, 14 May 2002 21:52:00 -0300 Received: from calhau.terra.com.br (calhau.terra.com.br [200.176.3.20]) by sucuri.mat.puc-rio.br (8.9.3/8.9.3) with ESMTP id VAA05172 for ; Tue, 14 May 2002 21:51:58 -0300 Received: from guarapari.terra.com.br (guarapari.terra.com.br [200.176.3.36]) by calhau.terra.com.br (Postfix) with ESMTP id 3089C4A6BA for ; Wed, 15 May 2002 00:50:07 +0000 (GMT) Received: from Lucelindo (unknown [200.177.64.180]) (authenticated user ldias1) by guarapari.terra.com.br (Postfix) with ESMTP id 479DCECCFA for ; Tue, 14 May 2002 21:50:06 -0300 (EST) Message-ID: <001a01c1fba9$e92a68c0$b440b1c8@Lucelindo> From: "Lucelindo D. Ferreira" To: Subject: [obm-l] CURSO Date: Tue, 14 May 2002 21:46:09 -0300 MIME-Version: 1.0 Content-Type: multipart/alternative; boundary="----=_NextPart_000_0017_01C1FB90.C1F7F600" X-Priority: 3 X-MSMail-Priority: Normal X-Mailer: Microsoft Outlook Express 5.00.2615.200 X-MimeOLE: Produced By Microsoft MimeOLE V5.00.2615.200 Sender: owner-obm-l@sucuri.mat.puc-rio.br Precedence: bulk Reply-To: obm-l@mat.puc-rio.br This is a multi-part message in MIME format. ------=_NextPart_000_0017_01C1FB90.C1F7F600 Content-Type: text/plain; charset="iso-8859-1" Content-Transfer-Encoding: quoted-printable Ol=E1 pessoal gostaria de saber onde s=E3o ministrados cursos de = olimp=EDadas aqui em Fortaleza. Pois j=E1 n=E3o sou aluno secund=E1rio e = n=E3o tive a oportunidade de participar desde cedo de cursos de = olimpiadas. Tenho interesse de participar das olimp=EDadas a n=EDvel = universit=E1rio.=20 Agrade=E7o desde j=E1!! Fabio ------=_NextPart_000_0017_01C1FB90.C1F7F600 Content-Type: text/html; charset="iso-8859-1" Content-Transfer-Encoding: quoted-printable
Ol=E1 pessoal gostaria de saber onde = s=E3o ministrados=20 cursos de olimp=EDadas aqui em Fortaleza. Pois j=E1 n=E3o sou aluno = secund=E1rio e n=E3o=20 tive a oportunidade de participar desde cedo de cursos de = olimpiadas. Tenho=20 interesse de participar das olimp=EDadas a n=EDvel universit=E1rio. =
Agrade=E7o desde j=E1!!
Fabio
------=_NextPart_000_0017_01C1FB90.C1F7F600-- ========================================================================= Instruções para entrar na lista, sair da lista e usar a lista em http://www.mat.puc-rio.br/~nicolau/olimp/obm-l.html O administrador desta lista é ========================================================================= From owner-obm-l@sucuri.mat.puc-rio.br Wed May 15 00:29:41 2002 Return-Path: Received: (from majordom@localhost) by sucuri.mat.puc-rio.br (8.9.3/8.9.3) id AAA06682 for obm-l-list; Wed, 15 May 2002 00:28:06 -0300 Received: from shannon.bol.com.br (shannon.bol.com.br [200.221.24.13]) by sucuri.mat.puc-rio.br (8.9.3/8.9.3) with ESMTP id AAA06678 for ; Wed, 15 May 2002 00:28:05 -0300 Received: from bol.com.br (200.221.24.66) by shannon.bol.com.br (5.1.071) id 3CE1A947000095C7 for obm-l@mat.puc-rio.br; Wed, 15 May 2002 00:25:29 -0300 Date: Wed, 15 May 2002 00:25:29 -0300 Message-Id: Subject: Re: [obm-l] .......... MIME-Version: 1.0 Content-Type: text/plain;charset="iso-8859-1" From: "rafaelc.l" To: obm-l@mat.puc-rio.br X-XaM3-API-Version: 2.4.3.4.4 X-SenderIP: 200.176.165.233 Content-Transfer-Encoding: 8bit X-MIME-Autoconverted: from quoted-printable to 8bit by sucuri.mat.puc-rio.br id AAA06679 Sender: owner-obm-l@sucuri.mat.puc-rio.br Precedence: bulk Reply-To: obm-l@mat.puc-rio.br > > > > > 2)(IME-2001) Resolva a equação [5-(5-x)^1/2]^1/2=x sendo > > x>0. > Chame sqrt (5 - x) = x > > Daí temos: > > 5 - x = x^2 > x^2 + x - 5 = 0 > > x = [-1 + sqrt(21)]/2 ou x = [-1- sqrt(21)]/2 > Tá, mas pq sqrt(5-x)=x se x=sqrt 5- sqrt(5-x)? Me diz mais uma coisa: Não daria pra resolver esta questão elevando os membros ao quadrado até tirar os radicais e depois achando as raízes do polinômio por girard? > > > ========================================================== =============== > Instruções para entrar na lista, sair da lista e usar a lista em > http://www.mat.puc-rio.br/~nicolau/olimp/obm-l.html > O administrador desta lista é > ========================================================== =============== > __________________________________________________________________________ Quer ter seu próprio endereço na Internet? Garanta já o seu e ainda ganhe cinco e-mails personalizados. DomíniosBOL - http://dominios.bol.com.br ========================================================================= Instruções para entrar na lista, sair da lista e usar a lista em http://www.mat.puc-rio.br/~nicolau/olimp/obm-l.html O administrador desta lista é ========================================================================= From owner-obm-l@sucuri.mat.puc-rio.br Wed May 15 00:53:27 2002 Return-Path: Received: (from majordom@localhost) by sucuri.mat.puc-rio.br (8.9.3/8.9.3) id AAA07123 for obm-l-list; Wed, 15 May 2002 00:52:25 -0300 Received: from traven.uol.com.br (traven.uol.com.br [200.231.206.184]) by sucuri.mat.puc-rio.br (8.9.3/8.9.3) with ESMTP id AAA07119 for ; Wed, 15 May 2002 00:52:23 -0300 Received: from fabiogianniniuol ([200.191.92.34]) by traven.uol.com.br (8.9.1/8.9.1) with SMTP id AAA22968 for ; Wed, 15 May 2002 00:38:20 -0300 (BRT) Received: by localhost with Microsoft MAPI; Wed, 15 May 2002 00:50:27 -0300 Message-ID: <01C1FBAA.8112F580.fabiogiannini@uol.com.br> From: =?us-ascii?Q?Fabio_Giannini?= To: "'obm-l@mat.puc-rio.br'" Subject: [obm-l] Problemas de Conjuntos Date: Wed, 15 May 2002 00:50:16 -0300 Organization: Particular X-Mailer: Microsoft Internet E-mail/MAPI - 8.0.0.4211 MIME-Version: 1.0 Content-Type: text/plain; charset="us-ascii" Content-Transfer-Encoding: 7bit Sender: owner-obm-l@sucuri.mat.puc-rio.br Precedence: bulk Reply-To: obm-l@mat.puc-rio.br Estou cursando a FATEC no periodo noturno e me deparei com os seguintes problemas no curso de MAT-1 Obtenha uma descricao geometrica para os seguintes lugares geometricos: 1- Origem de RxR (Reais produto catesiano Reais) 2- Reta que contem A,B Pertencentes a RxR A diferente de B 3- Circunferencia de centro no ponto C=(a,b) pertence a RxR e raio r > 0. Idem para o circulo que esta circunferencia delimita. 4- Parabola de foco F=(xf,yf) pertence a RxR e diretriz dada pela reta d: ax+by+c=0 sendo que F nao pertence a d. Desde ja , agradeco qualquer dica para iniciar as solucoes. Um abraco Giannini --- Outgoing mail is certified Virus Free. Checked by AVG anti-virus system (http://www.grisoft.com). Version: 6.0.325 / Virus Database: 182 - Release Date: 19/02/02 ========================================================================= Instruções para entrar na lista, sair da lista e usar a lista em http://www.mat.puc-rio.br/~nicolau/olimp/obm-l.html O administrador desta lista é ========================================================================= From owner-obm-l@sucuri.mat.puc-rio.br Wed May 15 01:10:27 2002 Return-Path: Received: (from majordom@localhost) by sucuri.mat.puc-rio.br (8.9.3/8.9.3) id BAA07546 for obm-l-list; Wed, 15 May 2002 01:10:18 -0300 Received: from hotmail.com (f165.pav1.hotmail.com [64.4.31.165]) by sucuri.mat.puc-rio.br (8.9.3/8.9.3) with ESMTP id BAA07542 for ; Wed, 15 May 2002 01:10:16 -0300 Received: from mail pickup service by hotmail.com with Microsoft SMTPSVC; Tue, 14 May 2002 21:08:23 -0700 Received: from 200.151.190.121 by pv1fd.pav1.hotmail.msn.com with HTTP; Wed, 15 May 2002 04:08:23 GMT X-Originating-IP: [200.151.190.121] From: "Adherbal Rocha Filho" To: obm-l@mat.puc-rio.br Subject: [obm-l] dificuldades Date: Wed, 15 May 2002 04:08:23 +0000 Mime-Version: 1.0 Content-Type: text/plain; charset=iso-8859-1; format=flowed Message-ID: X-OriginalArrivalTime: 15 May 2002 04:08:23.0940 (UTC) FILETIME=[282FC440:01C1FBC6] Sender: owner-obm-l@sucuri.mat.puc-rio.br Precedence: bulk Reply-To: obm-l@mat.puc-rio.br Ae pessoal, valeu pela ajuda nas questões Agora,tem mais estas 2 aqui,se alguem puder ajudar,blza! 1.Os nºs naturais 1,2...,1998 são escritos em um imenso quadro negro.Em seguida ,um aluno apaga dois quaisquer colocando no lugar sua diferença (não negativa).Depois de muitas operações,um nº ficará escrito no quadro.É possível que esse nº seja 0? 2.Em uma ilha plana existem 11 cidades numeradas de 1 a 11.Estradas retas ligam 1 a 2,2 a 3,3 a 4,...,10 a 11 e 11 a 1.É possível que uma reta corte todas as estradas? Obrigadão! []´s Adherbal _________________________________________________________________ Una-se ao maior serviço de email do mundo: o MSN Hotmail. http://www.hotmail.com ========================================================================= Instruções para entrar na lista, sair da lista e usar a lista em http://www.mat.puc-rio.br/~nicolau/olimp/obm-l.html O administrador desta lista é ========================================================================= From owner-obm-l@sucuri.mat.puc-rio.br Wed May 15 13:06:11 2002 Return-Path: Received: (from majordom@localhost) by sucuri.mat.puc-rio.br (8.9.3/8.9.3) id NAA14834 for obm-l-list; Wed, 15 May 2002 13:01:46 -0300 Received: from matinhos.terra.com.br (matinhos.terra.com.br [200.176.3.21]) by sucuri.mat.puc-rio.br (8.9.3/8.9.3) with ESMTP id NAA14830 for ; Wed, 15 May 2002 13:01:44 -0300 Received: from smtp2-bra.terra.com.br (smtp2-bra.terra.com.br [200.176.3.33]) by matinhos.terra.com.br (Postfix) with ESMTP id E33EB48D8E for ; Wed, 15 May 2002 15:59:54 +0000 (GMT) Received: from Lucelindo (unknown [200.177.64.32]) (authenticated user ldias1) by smtp2-bra.terra.com.br (Postfix) with ESMTP id DBE6A885D1 for ; Wed, 15 May 2002 12:59:53 -0300 (EST) Message-ID: <002f01c1fc29$010ef260$9a40b1c8@Lucelindo> From: "Lucelindo D. Ferreira" To: References: Subject: Re: [obm-l] dificuldades Date: Wed, 15 May 2002 12:55:56 -0300 MIME-Version: 1.0 Content-Type: text/plain; charset="iso-8859-1" Content-Transfer-Encoding: 8bit X-Priority: 3 X-MSMail-Priority: Normal X-Mailer: Microsoft Outlook Express 5.00.2615.200 X-MimeOLE: Produced By Microsoft MimeOLE V5.00.2615.200 Sender: owner-obm-l@sucuri.mat.puc-rio.br Precedence: bulk Reply-To: obm-l@mat.puc-rio.br Fala! td OK! Aqui eu te dou uma dica para a 1. 1-Dica: Note q ñ importa a ordem em q vc faz as diferenças, a paridade do último número permanece a mesma(Uma invariante no problema).Esse é um típico problema de paridade( nesses problemas a paridade é usada para mostrar que uma coisa ñ é possível). Vc pode achar a paridade do último número(subdica: Tanto faz vc achar a paridade das diferenças como das somas, é a mesma) e daí, bom daí vc vai ver. Valeu! Fabio ----- Original Message ----- From: Adherbal Rocha Filho To: Sent: Wednesday, May 15, 2002 1:08 AM Subject: [obm-l] dificuldades > > > Ae pessoal, valeu pela ajuda nas questões > Agora,tem mais estas 2 aqui,se alguem puder ajudar,blza! > 1.Os nºs naturais 1,2...,1998 são escritos em um imenso quadro negro.Em > seguida ,um aluno apaga dois quaisquer colocando no lugar sua diferença (não > negativa).Depois de muitas operações,um nº ficará escrito no quadro.É > possível que esse nº seja 0? > > 2.Em uma ilha plana existem 11 cidades numeradas de 1 a 11.Estradas retas > ligam 1 a 2,2 a 3,3 a 4,...,10 a 11 e 11 a 1.É possível que uma reta corte > todas as estradas? > Obrigadão! > []´s > Adherbal > > > _________________________________________________________________ > Una-se ao maior serviço de email do mundo: o MSN Hotmail. > http://www.hotmail.com > > ========================================================================= > Instruções para entrar na lista, sair da lista e usar a lista em > http://www.mat.puc-rio.br/~nicolau/olimp/obm-l.html > O administrador desta lista é > ========================================================================= > ========================================================================= Instruções para entrar na lista, sair da lista e usar a lista em http://www.mat.puc-rio.br/~nicolau/olimp/obm-l.html O administrador desta lista é ========================================================================= From owner-obm-l@sucuri.mat.puc-rio.br Wed May 15 16:11:18 2002 Return-Path: Received: (from majordom@localhost) by sucuri.mat.puc-rio.br (8.9.3/8.9.3) id QAA17837 for obm-l-list; Wed, 15 May 2002 16:08:52 -0300 Received: from web10104.mail.yahoo.com (web10104.mail.yahoo.com [216.136.130.54]) by sucuri.mat.puc-rio.br (8.9.3/8.9.3) with SMTP id QAA17827 for ; Wed, 15 May 2002 16:08:41 -0300 Message-ID: <20020515190642.95877.qmail@web10104.mail.yahoo.com> Received: from [150.161.4.45] by web10104.mail.yahoo.com via HTTP; Wed, 15 May 2002 12:06:42 PDT Date: Wed, 15 May 2002 12:06:42 -0700 (PDT) From: Rafael WC Subject: [obm-l] triângulo To: OBM MIME-Version: 1.0 Content-Type: text/plain; charset=us-ascii Sender: owner-obm-l@sucuri.mat.puc-rio.br Precedence: bulk Reply-To: obm-l@mat.puc-rio.br Olá Pessoal! Alguém conseguiria resolver essa pra mim? Não tô conseguindo... AB = 8,AC = 5 e BC = 7 são os lados de um triangulo ABC. Inscreve-se neste triangulo uma circunferencia e traça-se-lhe a tangente paralela ao lado BC, cujos pontos de interceção com os lados AB e AC são D e E. Calcular a razão ID/IE, sendo I o ponto de contato da tangente DE com a circunferencia inscrita no ABC. Obrigado! Rafael. ===== Rafael Werneck Cinoto ICQ# 107011599 rwcinoto@yahoo.com rafael.caixa@gov.com.br matduvidas@yahoo.com.br http://www.rwcinoto.hpg.com.br/ __________________________________________________ Do You Yahoo!? LAUNCH - Your Yahoo! Music Experience http://launch.yahoo.com ========================================================================= Instruções para entrar na lista, sair da lista e usar a lista em http://www.mat.puc-rio.br/~nicolau/olimp/obm-l.html O administrador desta lista é ========================================================================= From owner-obm-l@sucuri.mat.puc-rio.br Wed May 15 17:09:43 2002 Return-Path: Received: (from majordom@localhost) by sucuri.mat.puc-rio.br (8.9.3/8.9.3) id RAA19111 for obm-l-list; Wed, 15 May 2002 17:07:52 -0300 Received: from hotmail.com (f184.law15.hotmail.com [64.4.23.184]) by sucuri.mat.puc-rio.br (8.9.3/8.9.3) with ESMTP id RAA19107 for ; Wed, 15 May 2002 17:07:50 -0300 Received: from mail pickup service by hotmail.com with Microsoft SMTPSVC; Wed, 15 May 2002 13:05:52 -0700 Received: from 200.206.103.3 by lw15fd.law15.hotmail.msn.com with HTTP; Wed, 15 May 2002 20:05:52 GMT X-Originating-IP: [200.206.103.3] From: "RICARDO CHAVES" To: obm-l@mat.puc-rio.br Subject: Re: [obm-l] dificuldades Date: Wed, 15 May 2002 20:05:52 +0000 Mime-Version: 1.0 Content-Type: text/html; charset=iso-8859-1 Message-ID: X-OriginalArrivalTime: 15 May 2002 20:05:52.0768 (UTC) FILETIME=[EA5B4000:01C1FC4B] Sender: owner-obm-l@sucuri.mat.puc-rio.br Precedence: bulk Reply-To: obm-l@mat.puc-rio.br

ANSWER:Para a primeira,veja que x+y=x-y(mod 2).Some tudo e veja se da par.Ja a 2 eu nao sei direito...

FALOWZIS,Peterdirichlet.

>From: "Adherbal Rocha Filho"
>Reply-To: obm-l@mat.puc-rio.br
>To: obm-l@mat.puc-rio.br
>Subject: [obm-l] dificuldades
>Date: Wed, 15 May 2002 04:08:23 +0000
>
>
>
>Ae pessoal, valeu pela ajuda nas questões
>Agora,tem mais estas 2 aqui,se alguem puder ajudar,blza!
>1.Os nºs naturais 1,2...,1998 são escritos em um imenso quadro
>negro.Em
>seguida ,um aluno apaga dois quaisquer colocando no lugar sua
>diferença (não
>negativa).Depois de muitas operações,um nº ficará escrito no
>quadro.É
>possível que esse nº seja 0?
>
>2.Em uma ilha plana existem 11 cidades numeradas de 1 a 11.Estradas
>retas
>ligam 1 a 2,2 a 3,3 a 4,...,10 a 11 e 11 a 1.É possível que uma reta
>corte
>todas as estradas?
>Obrigadão!
>[]´s
>Adherbal
>
>
>_________________________________________________________________
>Una-se ao maior serviço de email do mundo: o MSN Hotmail.
>http://www.hotmail.com
>
>=========================================================================
>Instruções para entrar na lista, sair da lista e usar a lista em
>http://www.mat.puc-rio.br/~nicolau/olimp/obm-l.html
>O administrador desta lista é
>=========================================================================


Faça o download GRATUITO do MSN Explorer no endereço http://explorer.msn.com.
========================================================================= Instruções para entrar na lista, sair da lista e usar a lista em http://www.mat.puc-rio.br/~nicolau/olimp/obm-l.html O administrador desta lista é ========================================================================= From owner-obm-l@sucuri.mat.puc-rio.br Wed May 15 17:21:54 2002 Return-Path: Received: (from majordom@localhost) by sucuri.mat.puc-rio.br (8.9.3/8.9.3) id RAA19367 for obm-l-list; Wed, 15 May 2002 17:20:13 -0300 Received: from hotmail.com (f8.law15.hotmail.com [64.4.23.8]) by sucuri.mat.puc-rio.br (8.9.3/8.9.3) with ESMTP id RAA19363 for ; Wed, 15 May 2002 17:20:10 -0300 Received: from mail pickup service by hotmail.com with Microsoft SMTPSVC; Wed, 15 May 2002 13:18:19 -0700 Received: from 200.206.103.3 by lw15fd.law15.hotmail.msn.com with HTTP; Wed, 15 May 2002 20:18:19 GMT X-Originating-IP: [200.206.103.3] From: "RICARDO CHAVES" To: obm-l@mat.puc-rio.br Subject: Re: [obm-l] Lista para treino.... Date: Wed, 15 May 2002 20:18:19 +0000 Mime-Version: 1.0 Content-Type: text/html; charset=iso-8859-1 Message-ID: X-OriginalArrivalTime: 15 May 2002 20:18:19.0991 (UTC) FILETIME=[A7BC6E70:01C1FC4D] Sender: owner-obm-l@sucuri.mat.puc-rio.br Precedence: bulk Reply-To: obm-l@mat.puc-rio.br

Para esse dos infinitos compostos tente refinar o resultado:prove que esses termos(nao todos,so alguns...) sao multiplos com o primeiro termo se o dito cujo for diferente de 1.Se o primeiro termo e 1 pegue o segundo e pense que ele e o 1°.

>From: "Bruno F. C. Leite"
>Reply-To: obm-l@mat.puc-rio.br
>To: obm-l@mat.puc-rio.br
>Subject: Re: [obm-l] Lista para treino....
>Date: Mon, 13 May 2002 22:55:51 -0300
>
>At 19:26 13/05/02 -0400, you wrote:
>>1)Em uma dessas listas pra treino para olimpíadas, o sujeito pede
>>para
>>calcularmos a soma de todos os divisores positivos de n? Existe
>>alguma
>>fórmula para isso?
>
>existe, em termos da fatoração de n em primos. Fica fácil se vc
>provar que
>se mdc(a,b)=1, então soma_divisores(a)
>soma_divisores(b)=soma_divisores(ab). (e aí só falta saber
>soma_divisores(p^n), onde p é primo- mas isso é fácil mesmo)
>
>>2)Sendo N o número de divisores positivos de n, determine, em
>>função de n
>>e N o produto de todos os divisores de n.
>
>Se d divide n, n/d divide n. Agrupe os divisores dessa forma...(não
>esqueça
>o caso em que n é quadrado)
>
>>3)Mostre que qualquer P.A, não constante, de inteiros possui uma
>>infinidade de valores compostos.
>
>Suponha que só possua finitos valores compostos. Então, a partir de
>um
>ponto, todos os valores da PA são primos. Seja a+kb a sua PA, com
>k=0,1,2.... Se k>=k_0, então a+bk é primo. Temos que a deve ser
>ímpar e b
>par. Tome k=ak_0>=k_0. Então a+bak_0 é primo, logo a=1. Agora
>tomando
>k=2b^(n-1)+b^(2n-1), a+bk=1+2b^n+b^(2n)=(b^n+1)^2 é primo se n for
>suficientemente grande, o que é absurdo.
>
>Está confuso (e deve ter solução mais simples) mas acho que está
>certo.
>
>Bruno Leite
>http://www.ime.usp.br/~brleite
>
>
>> Agradeço de antemão a quem resolver.
>> Crom
>
>=========================================================================
>Instruções para entrar na lista, sair da lista e usar a lista em
>http://www.mat.puc-rio.br/~nicolau/olimp/obm-l.html
>O administrador desta lista é
>=========================================================================


Faça o download GRATUITO do MSN Explorer no endereço http://explorer.msn.com.
========================================================================= Instruções para entrar na lista, sair da lista e usar a lista em http://www.mat.puc-rio.br/~nicolau/olimp/obm-l.html O administrador desta lista é ========================================================================= From owner-obm-l@sucuri.mat.puc-rio.br Wed May 15 22:08:50 2002 Return-Path: Received: (from majordom@localhost) by sucuri.mat.puc-rio.br (8.9.3/8.9.3) id WAA22463 for obm-l-list; Wed, 15 May 2002 22:07:22 -0300 Received: from traven.uol.com.br (traven.uol.com.br [200.231.206.184]) by sucuri.mat.puc-rio.br (8.9.3/8.9.3) with ESMTP id WAA22459 for ; Wed, 15 May 2002 22:07:20 -0300 Received: from fabiogianniniuol ([200.191.200.99]) by traven.uol.com.br (8.9.1/8.9.1) with SMTP id VAA12143 for ; Wed, 15 May 2002 21:54:28 -0300 (BRT) Received: by localhost with Microsoft MAPI; Wed, 15 May 2002 22:06:55 -0300 Message-ID: <01C1FC5C.D35F1820.fabiogiannini@uol.com.br> From: =?us-ascii?Q?Fabio_Giannini?= To: =?us-ascii?Q?=22Lista_da_OBM=2DOlimpiadas_Brasileira_de_Matematica_?= =?us-ascii?Q?=28Correio_eletronico=29=22?= Subject: [obm-l] Conjuntos Date: Wed, 15 May 2002 22:06:47 -0300 Organization: Particular X-Mailer: Microsoft Internet E-mail/MAPI - 8.0.0.4211 MIME-Version: 1.0 Content-Type: text/plain; charset="us-ascii" Content-Transfer-Encoding: 7bit Sender: owner-obm-l@sucuri.mat.puc-rio.br Precedence: bulk Reply-To: obm-l@mat.puc-rio.br Estou cursando a FATEC no periodo noturno e me deparei com os seguintes problemas no curso de MAT-1 Obtenha uma descricao geometrica para os seguintes lugares geometricos: 1- Origem de RxR (Reais produto catesiano Reais) 2- Reta que contem A,B Pertencentes a RxR A diferente de B 3- Circunferencia de centro no ponto C=(a,b) pertence a RxR e raio r > 0. Idem para o circulo que esta circunferencia delimita. 4- Parabola de foco F=(xf,yf) pertence a RxR e diretriz dada pela reta d: ax+by+c=0 sendo que F nao pertence a d. Desde ja , agradeco qualquer dica para iniciar as solucoes. Um abraco Giannini --- Outgoing mail is certified Virus Free. Checked by AVG anti-virus system (http://www.grisoft.com). Version: 6.0.325 / Virus Database: 182 - Release Date: 19/02/02 ========================================================================= Instruções para entrar na lista, sair da lista e usar a lista em http://www.mat.puc-rio.br/~nicolau/olimp/obm-l.html O administrador desta lista é ========================================================================= From owner-obm-l@sucuri.mat.puc-rio.br Thu May 16 01:30:57 2002 Return-Path: Received: (from majordom@localhost) by sucuri.mat.puc-rio.br (8.9.3/8.9.3) id BAA24230 for obm-l-list; Thu, 16 May 2002 01:28:03 -0300 Received: from imo-r08.mx.aol.com (imo-r08.mx.aol.com [152.163.225.104]) by sucuri.mat.puc-rio.br (8.9.3/8.9.3) with ESMTP id BAA24226 for ; Thu, 16 May 2002 01:28:00 -0300 From: Euraul@aol.com Received: from Euraul@aol.com by imo-r08.mx.aol.com (mail_out_v32.5.) id z.7d.2755d8b1 (4412) for ; Thu, 16 May 2002 00:26:02 -0400 (EDT) Message-ID: <7d.2755d8b1.2a148eda@aol.com> Date: Thu, 16 May 2002 00:26:02 EDT Subject: [obm-l] Por favor... To: obm-l@mat.puc-rio.br MIME-Version: 1.0 Content-Type: multipart/alternative; boundary="part1_7d.2755d8b1.2a148eda_boundary" X-Mailer: AOL 6.0 for Windows BR sub 10516 Sender: owner-obm-l@sucuri.mat.puc-rio.br Precedence: bulk Reply-To: obm-l@mat.puc-rio.br --part1_7d.2755d8b1.2a148eda_boundary Content-Type: text/plain; charset="ISO-8859-1" Content-Transfer-Encoding: quoted-printable Sejam AB e BC dois lados adjacentes de um pol=EDgono regular de 9 lados,=20 inscrito em uma circunfer=EAncia de lado O. Seja M o ponto m=E9dio do lado A= B e N=20 o ponto m=E9dio do raio perpendicular a BC. Qual a medida do =E2ngulo OMN ? Obrigado Raul --part1_7d.2755d8b1.2a148eda_boundary Content-Type: text/html; charset="ISO-8859-1" Content-Transfer-Encoding: quoted-printable  Sejam AB e BC doi= s lados adjacentes de um pol=EDgono regular de 9 lados, inscrito em uma circ= unfer=EAncia de lado O. Seja M o ponto m=E9dio do lado AB e N o ponto m=E9di= o do raio perpendicular a BC. Qual a medida do =E2ngulo OMN ?
      Obrigado
            = ;Raul
--part1_7d.2755d8b1.2a148eda_boundary-- ========================================================================= Instruções para entrar na lista, sair da lista e usar a lista em http://www.mat.puc-rio.br/~nicolau/olimp/obm-l.html O administrador desta lista é ========================================================================= From owner-obm-l@sucuri.mat.puc-rio.br Thu May 16 02:41:39 2002 Return-Path: Received: (from majordom@localhost) by sucuri.mat.puc-rio.br (8.9.3/8.9.3) id CAA25039 for obm-l-list; Thu, 16 May 2002 02:39:00 -0300 Received: from hotmail.com (f103.pav1.hotmail.com [64.4.31.103]) by sucuri.mat.puc-rio.br (8.9.3/8.9.3) with ESMTP id CAA25035 for ; Thu, 16 May 2002 02:38:57 -0300 Received: from mail pickup service by hotmail.com with Microsoft SMTPSVC; Wed, 15 May 2002 22:37:08 -0700 Received: from 200.174.155.137 by pv1fd.pav1.hotmail.msn.com with HTTP; Thu, 16 May 2002 05:37:08 GMT X-Originating-IP: [200.174.155.137] From: "Felipe Marinho" To: obm-l@mat.puc-rio.br Subject: [obm-l] Exercicios - Olimpiada. Date: Thu, 16 May 2002 01:37:08 -0400 Mime-Version: 1.0 Content-Type: text/plain; charset=iso-8859-1; format=flowed Message-ID: X-OriginalArrivalTime: 16 May 2002 05:37:08.0616 (UTC) FILETIME=[B85A9880:01C1FC9B] Sender: owner-obm-l@sucuri.mat.puc-rio.br Precedence: bulk Reply-To: obm-l@mat.puc-rio.br Olá pessoal da lista, Venho aqui pedir uma grande ajuda a vocês na resolução destes problemas. Encontrei-os numa lista de preparação para Olimpíadas, porem, estes 2 eu realmente não consegui resolvê-los. Por isso, conto com vocês mais uma vez. 1) Considere os números formados por 2 dígitos tais que a multiplicação deles pela soma do seus dígitos seja igual a soma do cubo dos digitos. Quantos e quais são esses números ? 2) 40 bolas são numeradas de 1 a 40. Elas então são colocadas em caixas. Se uma caixa contem n bolas, então a caixa não poderá conter uma bola numerada com um múltiplo de n. No mínimo quantas caixas serão precisas para guardar as bolas, considerando todas as possibilidades possíveis ? Pessoal, agradeço desde já qualquer tipo de ajuda. E com um grande abraço a todos, vou fechando mais este e-mail. Felipe Marinho _________________________________________________________________ Envie e receba emails com o Hotmail no seu dispositivo móvel: http://mobile.msn.com ========================================================================= Instruções para entrar na lista, sair da lista e usar a lista em http://www.mat.puc-rio.br/~nicolau/olimp/obm-l.html O administrador desta lista é ========================================================================= From owner-obm-l@sucuri.mat.puc-rio.br Thu May 16 03:54:19 2002 Return-Path: Received: (from majordom@localhost) by sucuri.mat.puc-rio.br (8.9.3/8.9.3) id DAA25771 for obm-l-list; Thu, 16 May 2002 03:51:43 -0300 Received: from toole.uol.com.br (toole.uol.com.br [200.231.206.186]) by sucuri.mat.puc-rio.br (8.9.3/8.9.3) with ESMTP id DAA25767 for ; Thu, 16 May 2002 03:51:41 -0300 Received: from afonso ([200.191.190.98]) by toole.uol.com.br (8.9.1/8.9.1) with SMTP id DAA23215 for ; Thu, 16 May 2002 03:45:28 -0300 (BRT) From: "Guilherme Pimentel" To: Subject: RES: [obm-l] Exercicios - Olimpiada. Date: Thu, 16 May 2002 03:53:27 -0300 Message-ID: MIME-Version: 1.0 Content-Type: text/plain; charset="iso-8859-1" Content-Transfer-Encoding: 8bit X-Priority: 3 (Normal) X-MSMail-Priority: Normal X-Mailer: Microsoft Outlook IMO, Build 9.0.2416 (9.0.2910.0) X-MimeOLE: Produced By Microsoft MimeOLE V5.00.2615.200 In-Reply-To: Importance: Normal Sender: owner-obm-l@sucuri.mat.puc-rio.br Precedence: bulk Reply-To: obm-l@mat.puc-rio.br Para o primeiro note que, sendo ab o numero de dois digitos: a*b*(a + b) = a^3 + b^3 e que a^3 + b^3 = (a + b)^3 - 3a*b*(a + b) logo a*b*(a + b) = (a + b)^3 - 3a*b*(a + b) (a + b)^3 = 4a*b*(a + b) supondo que a ou b sejam diferentes de zero: (a + b)^2 = 4a*b (a - b)^2 = 0 ou seja a = b agora vc conta quantos são... -----Mensagem original----- De: owner-obm-l@sucuri.mat.puc-rio.br [mailto:owner-obm-l@sucuri.mat.puc-rio.br]Em nome de Felipe Marinho Enviada em: quinta-feira, 16 de maio de 2002 02:37 Para: obm-l@mat.puc-rio.br Assunto: [obm-l] Exercicios - Olimpiada. Olá pessoal da lista, Venho aqui pedir uma grande ajuda a vocês na resolução destes problemas. Encontrei-os numa lista de preparação para Olimpíadas, porem, estes 2 eu realmente não consegui resolvê-los. Por isso, conto com vocês mais uma vez. 1) Considere os números formados por 2 dígitos tais que a multiplicação deles pela soma do seus dígitos seja igual a soma do cubo dos digitos. Quantos e quais são esses números ? 2) 40 bolas são numeradas de 1 a 40. Elas então são colocadas em caixas. Se uma caixa contem n bolas, então a caixa não poderá conter uma bola numerada com um múltiplo de n. No mínimo quantas caixas serão precisas para guardar as bolas, considerando todas as possibilidades possíveis ? Pessoal, agradeço desde já qualquer tipo de ajuda. E com um grande abraço a todos, vou fechando mais este e-mail. Felipe Marinho _________________________________________________________________ Envie e receba emails com o Hotmail no seu dispositivo móvel: http://mobile.msn.com ========================================================================= Instruções para entrar na lista, sair da lista e usar a lista em http://www.mat.puc-rio.br/~nicolau/olimp/obm-l.html O administrador desta lista é ========================================================================= ========================================================================= Instruções para entrar na lista, sair da lista e usar a lista em http://www.mat.puc-rio.br/~nicolau/olimp/obm-l.html O administrador desta lista é ========================================================================= From owner-obm-l@sucuri.mat.puc-rio.br Thu May 16 05:11:52 2002 Return-Path: Received: (from majordom@localhost) by sucuri.mat.puc-rio.br (8.9.3/8.9.3) id FAA26623 for obm-l-list; Thu, 16 May 2002 05:08:36 -0300 Received: from hotmail.com (f29.pav1.hotmail.com [64.4.31.29]) by sucuri.mat.puc-rio.br (8.9.3/8.9.3) with ESMTP id FAA26619 for ; Thu, 16 May 2002 05:08:33 -0300 Received: from mail pickup service by hotmail.com with Microsoft SMTPSVC; Thu, 16 May 2002 01:06:44 -0700 Received: from 200.174.155.137 by pv1fd.pav1.hotmail.msn.com with HTTP; Thu, 16 May 2002 08:06:43 GMT X-Originating-IP: [200.174.155.137] From: "Felipe Marinho" To: obm-l@mat.puc-rio.br Subject: Re: [obm-l] Exercicios - Olimpiada. Date: Thu, 16 May 2002 04:06:43 -0400 Mime-Version: 1.0 Content-Type: text/plain; charset=iso-8859-1; format=flowed Message-ID: X-OriginalArrivalTime: 16 May 2002 08:06:44.0978 (UTC) FILETIME=[9EAEBD20:01C1FCB0] Sender: owner-obm-l@sucuri.mat.puc-rio.br Precedence: bulk Reply-To: obm-l@mat.puc-rio.br Olá Guilherme, Obrigado pela sua ajuda. Resolvendo a questão do modo como você apresentou, a resposta seria 10 números (00,11,22,33,44,55,66,77,88,99). Porem, tal resposta não bate com a resposta do gabarito da prova. O enunciado da questão deve ser entendido como: (10a+b)(a+b) = a³+b³ (Multiplicacao do número formado por a e b pela soma dos dígitos) Na sua resolução você utilizou-se da multiplicação dos dígitos pela soma dos mesmos. [ a.b(a+b)=a³+b³ ]. As opções dadas pelo problema é: a) 0 b) 1 c) 2 d) 3 e) 4 Guilherme, e com isso, utilizando-se de seu raciocínio, não obteríamos a resposta do problema. Acho eu, então, que a solução é fazendo (10a+b)(a+b)=a³+b³. Porem, a resposta disto eu não consigo achar. Por isso vim até aqui, vocês, amigos da lista, para me dar uma ajuda... no que for possível, é claro. Agradeço desde já mais uma vez, Abraços Felipe Marinho >From: "Guilherme Pimentel" >Reply-To: obm-l@mat.puc-rio.br >To: >Subject: RES: [obm-l] Exercicios - Olimpiada. >Date: Thu, 16 May 2002 03:53:27 -0300 > >Para o primeiro note que, sendo ab o numero de dois digitos: >a*b*(a + b) = a^3 + b^3 >e que >a^3 + b^3 = (a + b)^3 - 3a*b*(a + b) >logo >a*b*(a + b) = (a + b)^3 - 3a*b*(a + b) >(a + b)^3 = 4a*b*(a + b) >supondo que a ou b sejam diferentes de zero: >(a + b)^2 = 4a*b >(a - b)^2 = 0 >ou seja >a = b >agora vc conta quantos são... > >-----Mensagem original----- >De: owner-obm-l@sucuri.mat.puc-rio.br >[mailto:owner-obm-l@sucuri.mat.puc-rio.br]Em nome de Felipe Marinho >Enviada em: quinta-feira, 16 de maio de 2002 02:37 >Para: obm-l@mat.puc-rio.br >Assunto: [obm-l] Exercicios - Olimpiada. > > >Olá pessoal da lista, > >Venho aqui pedir uma grande ajuda a vocês na resolução destes problemas. >Encontrei-os numa lista de preparação para Olimpíadas, porem, estes 2 eu >realmente não consegui resolvê-los. Por isso, conto com vocês mais uma vez. > >1) Considere os números formados por 2 dígitos tais que a multiplicação >deles pela soma do seus dígitos seja igual a soma do cubo dos digitos. >Quantos e quais são esses números ? > >2) 40 bolas são numeradas de 1 a 40. Elas então são colocadas em caixas. Se >uma caixa contem n bolas, então a caixa não poderá conter uma bola numerada >com um múltiplo de n. No mínimo quantas caixas serão precisas para guardar >as bolas, considerando todas as possibilidades possíveis ? > >Pessoal, agradeço desde já qualquer tipo de ajuda. > >E com um grande abraço a todos, >vou fechando mais este e-mail. > >Felipe Marinho > >_________________________________________________________________ >Envie e receba emails com o Hotmail no seu dispositivo móvel: >http://mobile.msn.com > >========================================================================= >Instruções para entrar na lista, sair da lista e usar a lista em >http://www.mat.puc-rio.br/~nicolau/olimp/obm-l.html >O administrador desta lista é >========================================================================= > >========================================================================= >Instruções para entrar na lista, sair da lista e usar a lista em >http://www.mat.puc-rio.br/~nicolau/olimp/obm-l.html >O administrador desta lista é >========================================================================= _________________________________________________________________ Converse com amigos on-line, conheça o MSN Messenger: http://messenger.msn.com ========================================================================= Instruções para entrar na lista, sair da lista e usar a lista em http://www.mat.puc-rio.br/~nicolau/olimp/obm-l.html O administrador desta lista é ========================================================================= From owner-obm-l@sucuri.mat.puc-rio.br Thu May 16 11:54:23 2002 Return-Path: Received: (from majordom@localhost) by sucuri.mat.puc-rio.br (8.9.3/8.9.3) id LAA31364 for obm-l-list; Thu, 16 May 2002 11:53:47 -0300 Received: from web14805.mail.yahoo.com (web14805.mail.yahoo.com [216.136.224.221]) by sucuri.mat.puc-rio.br (8.9.3/8.9.3) with SMTP id LAA31360 for ; Thu, 16 May 2002 11:53:43 -0300 Message-ID: <20020516145152.84334.qmail@web14805.mail.yahoo.com> Received: from [200.243.206.168] by web14805.mail.yahoo.com via HTTP; Thu, 16 May 2002 11:51:52 ART Date: Thu, 16 May 2002 11:51:52 -0300 (ART) From: =?iso-8859-1?q?Ricardo=20Miranda?= Subject: Re: [obm-l] Por favor... To: obm-l@mat.puc-rio.br In-Reply-To: <7d.2755d8b1.2a148eda@aol.com> MIME-Version: 1.0 Content-Type: text/plain; charset=iso-8859-1 Content-Transfer-Encoding: 8bit Sender: owner-obm-l@sucuri.mat.puc-rio.br Precedence: bulk Reply-To: obm-l@mat.puc-rio.br --- Euraul@aol.com escreveu: > Sejam AB e BC dois lados adjacentes de um polígono regular de 9 > lados, > inscrito em uma circunferência de lado O. Seja M o ponto médio do lado > AB e N > o ponto médio do raio perpendicular a BC. Qual a medida do ângulo OMN ? > Obrigado > Raul Onde lê-se "cincunferencia de lado O", creio que é "circunferencia de centro O", nao eh mesmo? No triangulo ABO temos que, angulo OAB=OBA=70 e AOB=40 graus. Se tracarmos MO, dividiremos o angulo AOB em 2, e ficaremos entao com MOB=20, OBM=70. Tracando MN, temos que OMN=ONM, e já que MON é 40, entao OMN=ONM=70. ===== []s -- Ricardo Miranda ricardomirandabr@yahoo.com.br http://rm2.hpg.ig.com.br/ _______________________________________________________________________ Yahoo! Encontros O lugar certo para você encontrar aquela pessoa que falta na sua vida. Cadastre-se hoje mesmo! http://br.encontros.yahoo.com/ ========================================================================= Instruções para entrar na lista, sair da lista e usar a lista em http://www.mat.puc-rio.br/~nicolau/olimp/obm-l.html O administrador desta lista é ========================================================================= From owner-obm-l@sucuri.mat.puc-rio.br Thu May 16 11:54:26 2002 Return-Path: Received: (from majordom@localhost) by sucuri.mat.puc-rio.br (8.9.3/8.9.3) id LAA31340 for obm-l-list; Thu, 16 May 2002 11:53:09 -0300 Received: from hotmail.com (law2-f91.hotmail.com [216.32.181.91]) by sucuri.mat.puc-rio.br (8.9.3/8.9.3) with ESMTP id LAA31335 for ; Thu, 16 May 2002 11:53:04 -0300 Received: from mail pickup service by hotmail.com with Microsoft SMTPSVC; Thu, 16 May 2002 07:51:10 -0700 Received: from 200.229.244.100 by lw2fd.hotmail.msn.com with HTTP; Thu, 16 May 2002 14:51:09 GMT X-Originating-IP: [200.229.244.100] From: "Paulo Santa Rita" To: obm-l@mat.puc-rio.br Subject: Re: [obm-l] Exercicios - Olimpiada. Date: Thu, 16 May 2002 14:51:09 +0000 Mime-Version: 1.0 Content-Type: text/plain; charset=iso-8859-1; format=flowed Message-ID: X-OriginalArrivalTime: 16 May 2002 14:51:10.0525 (UTC) FILETIME=[1E135ED0:01C1FCE9] Sender: owner-obm-l@sucuri.mat.puc-rio.br Precedence: bulk Reply-To: obm-l@mat.puc-rio.br Ola Felipe, Guilherme e demais colegas desta lista, Bom, isso deve ser um daqueles problemas que precisam ser explicados a alunos do 1 grau, certo ? Dificil ! Vou tentar. Perdao se nao conseguir atingir os objetivos ! Devemos encontrar "a" e "b" inteiros nao-negativos. Como (10a + b)*(a+b) = a^3 + b^3 posso FATORAR o segundo membro assim : a^3 + b^3 =(a+b)*(a^2- ab + b^2) colocando essa fatoracao na primeira equacao, fica : (10a + b)*(a+b) = (a+b)*(a^2 -ab + b^2) como a + b > 0 (pois 00=0 nao e um numero de dois digitos ) posso dividir tudo por a+b. Vai ficar : 10a + b = a^2 - ab + b^2 reduzindo os termos semelhantes a^2 - (10 - b)*a + b^2 - b=0 E isto e UMA EQUACAO DO 2 GRAU LITERAL em "a". O Discriminante e : (10 - b)^2 - 4*(b^2 - b) Simplificando fica : -3b^2 - 16b + 100 = 0 Como b varia de 0 ate 9, posso verificar os casos em que o discriminante e um quadrado perfeito e que implicam num "a" inteiro positivo. Isso vai me fornecer a quantidade de numeros que satisfazem o problema. E ai Felipe ? Que tal ? Eu acho que so usei coisas que uma aluno serio do 1 grau sabe ... ou nao ? Se errei, foi uma tentativa e a culpa exclusivamente minha. Mas, se acertei, e porque sou um aluno aplicado do Jardim do Tio Ralph. Um abracao Paulo Santa Rita 5,1147,160502 >From: "Felipe Marinho" >Reply-To: obm-l@mat.puc-rio.br >To: obm-l@mat.puc-rio.br >Subject: Re: [obm-l] Exercicios - Olimpiada. >Date: Thu, 16 May 2002 04:06:43 -0400 > >Olá Guilherme, > >Obrigado pela sua ajuda. > >Resolvendo a questão do modo como você apresentou, a resposta seria 10 >números (00,11,22,33,44,55,66,77,88,99). >Porem, tal resposta não bate com a resposta do gabarito da prova. > >O enunciado da questão deve ser entendido como: > >(10a+b)(a+b) = a³+b³ (Multiplicacao do número formado por a e b pela soma >dos dígitos) > >Na sua resolução você utilizou-se da multiplicação dos dígitos pela soma >dos mesmos. [ a.b(a+b)=a³+b³ ]. > >As opções dadas pelo problema é: >a) 0 >b) 1 >c) 2 >d) 3 >e) 4 > >Guilherme, e com isso, utilizando-se de seu raciocínio, não obteríamos a >resposta do problema. > >Acho eu, então, que a solução é fazendo (10a+b)(a+b)=a³+b³. Porem, a >resposta disto eu não consigo achar. Por isso vim até aqui, vocês, amigos >da lista, para me dar uma ajuda... no que for possível, é claro. > >Agradeço desde já mais uma vez, >Abraços > >Felipe Marinho > >>From: "Guilherme Pimentel" >>Reply-To: obm-l@mat.puc-rio.br >>To: >>Subject: RES: [obm-l] Exercicios - Olimpiada. >>Date: Thu, 16 May 2002 03:53:27 -0300 >> >>Para o primeiro note que, sendo ab o numero de dois digitos: >>a*b*(a + b) = a^3 + b^3 >>e que >>a^3 + b^3 = (a + b)^3 - 3a*b*(a + b) >>logo >>a*b*(a + b) = (a + b)^3 - 3a*b*(a + b) >>(a + b)^3 = 4a*b*(a + b) >>supondo que a ou b sejam diferentes de zero: >>(a + b)^2 = 4a*b >>(a - b)^2 = 0 >>ou seja >>a = b >>agora vc conta quantos são... >> >>-----Mensagem original----- >>De: owner-obm-l@sucuri.mat.puc-rio.br >>[mailto:owner-obm-l@sucuri.mat.puc-rio.br]Em nome de Felipe Marinho >>Enviada em: quinta-feira, 16 de maio de 2002 02:37 >>Para: obm-l@mat.puc-rio.br >>Assunto: [obm-l] Exercicios - Olimpiada. >> >> >>Olá pessoal da lista, >> >>Venho aqui pedir uma grande ajuda a vocês na resolução destes problemas. >>Encontrei-os numa lista de preparação para Olimpíadas, porem, estes 2 eu >>realmente não consegui resolvê-los. Por isso, conto com vocês mais uma >>vez. >> >>1) Considere os números formados por 2 dígitos tais que a multiplicação >>deles pela soma do seus dígitos seja igual a soma do cubo dos digitos. >>Quantos e quais são esses números ? >> >>2) 40 bolas são numeradas de 1 a 40. Elas então são colocadas em caixas. >>Se >>uma caixa contem n bolas, então a caixa não poderá conter uma bola >>numerada >>com um múltiplo de n. No mínimo quantas caixas serão precisas para guardar >>as bolas, considerando todas as possibilidades possíveis ? >> >>Pessoal, agradeço desde já qualquer tipo de ajuda. >> >>E com um grande abraço a todos, >>vou fechando mais este e-mail. >> >>Felipe Marinho >> >>_________________________________________________________________ >>Envie e receba emails com o Hotmail no seu dispositivo móvel: >>http://mobile.msn.com >> >>========================================================================= >>Instruções para entrar na lista, sair da lista e usar a lista em >>http://www.mat.puc-rio.br/~nicolau/olimp/obm-l.html >>O administrador desta lista é >>========================================================================= >> >>========================================================================= >>Instruções para entrar na lista, sair da lista e usar a lista em >>http://www.mat.puc-rio.br/~nicolau/olimp/obm-l.html >>O administrador desta lista é >>========================================================================= > > >_________________________________________________________________ >Converse com amigos on-line, conheça o MSN Messenger: >http://messenger.msn.com > >========================================================================= >Instruções para entrar na lista, sair da lista e usar a lista em >http://www.mat.puc-rio.br/~nicolau/olimp/obm-l.html >O administrador desta lista é >========================================================================= _________________________________________________________________ Chegou o novo MSN Explorer. Instale já. É gratuito: http://explorer.msn.com.br ========================================================================= Instruções para entrar na lista, sair da lista e usar a lista em http://www.mat.puc-rio.br/~nicolau/olimp/obm-l.html O administrador desta lista é ========================================================================= From owner-obm-l@sucuri.mat.puc-rio.br Thu May 16 11:56:47 2002 Return-Path: Received: (from majordom@localhost) by sucuri.mat.puc-rio.br (8.9.3/8.9.3) id LAA31444 for obm-l-list; Thu, 16 May 2002 11:56:44 -0300 Received: from sr1.terra.com.br (sr1.terra.com.br [200.176.3.16]) by sucuri.mat.puc-rio.br (8.9.3/8.9.3) with ESMTP id LAA31440 for ; Thu, 16 May 2002 11:56:42 -0300 Received: from bica.terra.com.br (bica.terra.com.br [200.176.3.32]) by sr1.terra.com.br (Postfix) with ESMTP id 341957058D for ; Thu, 16 May 2002 11:54:56 -0300 (EST) Received: from Lucelindo (unknown [200.177.64.77]) (authenticated user ldias1) by bica.terra.com.br (Postfix) with ESMTP id 18D93D5446 for ; Thu, 16 May 2002 11:54:55 -0300 (EST) Message-ID: <003b01c1fce9$1635f920$4d40b1c8@Lucelindo> From: "Lucelindo D. Ferreira" To: Subject: [obm-l] CURSO Date: Thu, 16 May 2002 11:50:54 -0300 MIME-Version: 1.0 Content-Type: multipart/alternative; boundary="----=_NextPart_000_0038_01C1FCCF.EF12C8A0" X-Priority: 3 X-MSMail-Priority: Normal X-Mailer: Microsoft Outlook Express 5.00.2615.200 X-MimeOLE: Produced By Microsoft MimeOLE V5.00.2615.200 Sender: owner-obm-l@sucuri.mat.puc-rio.br Precedence: bulk Reply-To: obm-l@mat.puc-rio.br This is a multi-part message in MIME format. ------=_NextPart_000_0038_01C1FCCF.EF12C8A0 Content-Type: text/plain; charset="iso-8859-1" Content-Transfer-Encoding: quoted-printable Ol=E1 pessoal gostaria de saber onde s=E3o ministrados cursos de = olimp=EDadas aqui em Fortaleza. Pois j=E1 n=E3o sou aluno secund=E1rio e = n=E3o tive a oportunidade de participar desde cedo de cursos de = olimpiadas. Tenho interesse de participar das olimp=EDadas a n=EDvel = universit=E1rio.=20 Agrade=E7o desde j=E1!! Fabio ------=_NextPart_000_0038_01C1FCCF.EF12C8A0 Content-Type: text/html; charset="iso-8859-1" Content-Transfer-Encoding: quoted-printable
Ol=E1 pessoal gostaria de saber onde = s=E3o ministrados=20 cursos de olimp=EDadas aqui em Fortaleza. Pois j=E1 n=E3o sou aluno = secund=E1rio e n=E3o=20 tive a oportunidade de participar desde cedo de cursos de = olimpiadas. Tenho=20 interesse de participar das olimp=EDadas a n=EDvel universit=E1rio. =
Agrade=E7o desde j=E1!!
Fabio
------=_NextPart_000_0038_01C1FCCF.EF12C8A0-- ========================================================================= Instruções para entrar na lista, sair da lista e usar a lista em http://www.mat.puc-rio.br/~nicolau/olimp/obm-l.html O administrador desta lista é ========================================================================= From owner-obm-l@sucuri.mat.puc-rio.br Thu May 16 11:57:08 2002 Return-Path: Received: (from majordom@localhost) by sucuri.mat.puc-rio.br (8.9.3/8.9.3) id LAA31487 for obm-l-list; Thu, 16 May 2002 11:57:07 -0300 Received: from sr1.terra.com.br (sr1.terra.com.br [200.176.3.16]) by sucuri.mat.puc-rio.br (8.9.3/8.9.3) with ESMTP id LAA31482 for ; Thu, 16 May 2002 11:57:05 -0300 Received: from guarapari.terra.com.br (guarapari.terra.com.br [200.176.3.36]) by sr1.terra.com.br (Postfix) with ESMTP id 9095C6FCCF for ; Thu, 16 May 2002 11:55:15 -0300 (EST) Received: from Lucelindo (unknown [200.177.64.77]) (authenticated user ldias1) by guarapari.terra.com.br (Postfix) with ESMTP id DFBE0ECD30 for ; Thu, 16 May 2002 11:55:14 -0300 (EST) Message-ID: <004401c1fce9$21c4a700$4d40b1c8@Lucelindo> From: "Lucelindo D. Ferreira" To: Subject: [obm-l] CURSO Date: Thu, 16 May 2002 11:51:15 -0300 MIME-Version: 1.0 Content-Type: multipart/alternative; boundary="----=_NextPart_000_0041_01C1FCCF.FBB72C40" X-Priority: 3 X-MSMail-Priority: Normal X-Mailer: Microsoft Outlook Express 5.00.2615.200 X-MimeOLE: Produced By Microsoft MimeOLE V5.00.2615.200 Sender: owner-obm-l@sucuri.mat.puc-rio.br Precedence: bulk Reply-To: obm-l@mat.puc-rio.br This is a multi-part message in MIME format. ------=_NextPart_000_0041_01C1FCCF.FBB72C40 Content-Type: text/plain; charset="iso-8859-1" Content-Transfer-Encoding: quoted-printable Ol=E1 pessoal gostaria de saber onde s=E3o ministrados cursos de = olimp=EDadas aqui em Fortaleza. Pois j=E1 n=E3o sou aluno secund=E1rio e = n=E3o tive a oportunidade de participar desde cedo de cursos de = olimpiadas. Tenho interesse de participar das olimp=EDadas a n=EDvel = universit=E1rio.=20 Agrade=E7o desde j=E1!! Fabio ------=_NextPart_000_0041_01C1FCCF.FBB72C40 Content-Type: text/html; charset="iso-8859-1" Content-Transfer-Encoding: quoted-printable
Ol=E1 pessoal gostaria de saber onde = s=E3o ministrados=20 cursos de olimp=EDadas aqui em Fortaleza. Pois j=E1 n=E3o sou aluno = secund=E1rio e n=E3o=20 tive a oportunidade de participar desde cedo de cursos de = olimpiadas. Tenho=20 interesse de participar das olimp=EDadas a n=EDvel universit=E1rio. =
Agrade=E7o desde j=E1!!
Fabio
------=_NextPart_000_0041_01C1FCCF.FBB72C40-- ========================================================================= Instruções para entrar na lista, sair da lista e usar a lista em http://www.mat.puc-rio.br/~nicolau/olimp/obm-l.html O administrador desta lista é ========================================================================= From owner-obm-l@sucuri.mat.puc-rio.br Thu May 16 12:19:27 2002 Return-Path: Received: (from majordom@localhost) by sucuri.mat.puc-rio.br (8.9.3/8.9.3) id MAA32657 for obm-l-list; Thu, 16 May 2002 12:19:12 -0300 Received: from smtp-3.ig.com.br (smtp-3.ig.com.br [200.226.132.152] (may be forged)) by sucuri.mat.puc-rio.br (8.9.3/8.9.3) with SMTP id MAA32653 for ; Thu, 16 May 2002 12:19:08 -0300 Received: (qmail 30491 invoked from network); 16 May 2002 15:17:07 -0000 Received: from 253.170.226.200.in-addr.arpa.ig.com.br (HELO oemcomputer) (200.226.170.253) by smtp-3.ig.com.br with SMTP; 16 May 2002 15:17:07 -0000 Message-ID: <004401c1fcee$e21186e0$fdaae2c8@oemcomputer> From: "Daniel" To: References: Subject: Re: [obm-l] .......... Date: Thu, 16 May 2002 12:31:20 -0300 MIME-Version: 1.0 Content-Type: text/plain; charset="iso-8859-1" Content-Transfer-Encoding: 8bit X-Priority: 3 X-MSMail-Priority: Normal X-Mailer: Microsoft Outlook Express 5.00.2615.200 X-MIMEOLE: Produced By Microsoft MimeOLE V5.00.2615.200 Sender: owner-obm-l@sucuri.mat.puc-rio.br Precedence: bulk Reply-To: obm-l@mat.puc-rio.br > > > > > > > Tá, mas pq sqrt(5-x)=x se x=sqrt 5- sqrt(5-x)? > > Me diz mais uma coisa: Não daria pra resolver esta > questão elevando os membros ao quadrado até tirar os > radicais e depois achando as raízes do polinômio por > girard? > > > É só substituir que vc pode observar que a igualdade vale. Elevar ao quadrado requer muita força bruta, pode tentar, se conseguir me avisa!!! > > Daniel> ========================================================== > =============== > > Instruções para entrar na lista, sair da lista e usar a > lista em > > http://www.mat.puc-rio.br/~nicolau/olimp/obm-l.html > > O administrador desta lista é > > > ========================================================== > =============== > > > > > __________________________________________________________________________ > Quer ter seu próprio endereço na Internet? > Garanta já o seu e ainda ganhe cinco e-mails personalizados. > DomíniosBOL - http://dominios.bol.com.br > > > ========================================================================= > Instruções para entrar na lista, sair da lista e usar a lista em > http://www.mat.puc-rio.br/~nicolau/olimp/obm-l.html > O administrador desta lista é > ========================================================================= > ========================================================================= Instruções para entrar na lista, sair da lista e usar a lista em http://www.mat.puc-rio.br/~nicolau/olimp/obm-l.html O administrador desta lista é ========================================================================= From owner-obm-l@sucuri.mat.puc-rio.br Thu May 16 12:35:41 2002 Return-Path: Received: (from majordom@localhost) by sucuri.mat.puc-rio.br (8.9.3/8.9.3) id MAA00870 for obm-l-list; Thu, 16 May 2002 12:35:13 -0300 Received: from ns-3.idc.dglnet.com.br (ns-3.idc.dglnet.com.br [200.218.161.4]) by sucuri.mat.puc-rio.br (8.9.3/8.9.3) with ESMTP id MAA00864 for ; Thu, 16 May 2002 12:35:10 -0300 Received: from user (200-205-219-162.dial-up.telesp.net.br [200.205.219.162]) by ns-3.idc.dglnet.com.br (Postfix) with SMTP id 5254F150025 for ; Thu, 16 May 2002 12:33:22 -0300 (BRT) Message-ID: <003901c1fcef$0ae44580$a2dbcdc8@user> From: "Claudio" To: References: <7d.2755d8b1.2a148eda@aol.com> Subject: Re: [obm-l] Por favor... Date: Thu, 16 May 2002 12:33:30 -0300 MIME-Version: 1.0 Content-Type: multipart/alternative; boundary="----=_NextPart_000_0036_01C1FCD5.E300D240" X-Priority: 3 X-MSMail-Priority: Normal X-Mailer: Microsoft Outlook Express 5.00.2314.1300 X-MimeOLE: Produced By Microsoft MimeOLE V5.00.2314.1300 Sender: owner-obm-l@sucuri.mat.puc-rio.br Precedence: bulk Reply-To: obm-l@mat.puc-rio.br This is a multi-part message in MIME format. ------=_NextPart_000_0036_01C1FCD5.E300D240 Content-Type: text/plain; charset="iso-8859-1" Content-Transfer-Encoding: quoted-printable Ol=E1 Raul. Chame de N o ponto m=E9dio do arco BC. Chame de L o ponto m=E9dio do = raio ON. Note que o tri=E2ngulo ANO '=E9 equil=E1tero ! A circunfer=EAncia de di=E2metro AO cont=E9m os pontos M e L. Conclua = que o =E2ngulo procurado mede 30 =BA . Saludos. Claudio Casemiro. ----- Original Message -----=20 From: Euraul@aol.com=20 To: obm-l@mat.puc-rio.br=20 Sent: Thursday, May 16, 2002 1:26 AM Subject: [obm-l] Por favor... Sejam AB e BC dois lados adjacentes de um pol=EDgono regular de 9 = lados, inscrito em uma circunfer=EAncia de lado O. Seja M o ponto = m=E9dio do lado AB e N o ponto m=E9dio do raio perpendicular a BC. Qual = a medida do =E2ngulo OMN ?=20 Obrigado=20 Raul=20 ------=_NextPart_000_0036_01C1FCD5.E300D240 Content-Type: text/html; charset="iso-8859-1" Content-Transfer-Encoding: quoted-printable
Ol=E1 Raul.
Chame de N o ponto m=E9dio do arco BC. = Chame de L o=20 ponto m=E9dio do raio ON.
Note que o tri=E2ngulo ANO '=E9 = equil=E1tero=20 !
A circunfer=EAncia de di=E2metro AO = cont=E9m os pontos M=20 e L. Conclua que o =E2ngulo procurado mede 30 =BA .
Saludos.
Claudio Casemiro.
----- Original Message -----
From:=20 Euraul@aol.com
Sent: Thursday, May 16, 2002 = 1:26=20 AM
Subject: [obm-l] Por = favor...

 Sejam = AB e BC=20 dois lados adjacentes de um pol=EDgono regular de 9 lados, inscrito em = uma=20 circunfer=EAncia de lado O. Seja M o ponto m=E9dio do lado AB e N o = ponto m=E9dio do=20 raio perpendicular a BC. Qual a medida do =E2ngulo OMN ?=20
      Obrigado=20 =
           &nb= sp;Raul
=20
------=_NextPart_000_0036_01C1FCD5.E300D240-- ========================================================================= Instruções para entrar na lista, sair da lista e usar a lista em http://www.mat.puc-rio.br/~nicolau/olimp/obm-l.html O administrador desta lista é ========================================================================= From owner-obm-l@sucuri.mat.puc-rio.br Thu May 16 14:25:56 2002 Return-Path: Received: (from majordom@localhost) by sucuri.mat.puc-rio.br (8.9.3/8.9.3) id OAA03597 for obm-l-list; Thu, 16 May 2002 14:23:31 -0300 Received: from www.zipmail.com.br (smtp.zipmail.com.br [200.187.242.10]) by sucuri.mat.puc-rio.br (8.9.3/8.9.3) with ESMTP id OAA03593 for ; Thu, 16 May 2002 14:23:28 -0300 From: peterdirichlet@zipmail.com.br Received: from [200.206.103.3] by www.zipmail.com.br with HTTP; Thu, 16 May 2002 14:21:45 -0300 Message-ID: <3CE14E4D0000491A@www.zipmail.com.br> Date: Thu, 16 May 2002 14:21:45 -0300 In-Reply-To: <3CE14E0300000DD4@www.zipmail.com.br> Subject: [obm-l] =?iso-8859-1?Q?Apelo=3A=20Mais=20da=20Iberoamericana?= To: obm-l@mat.puc-rio.br MIME-Version: 1.0 Content-Type: text/plain; charset="iso-8859-1" Content-Transfer-Encoding: 8bit X-MIME-Autoconverted: from quoted-printable to 8bit by sucuri.mat.puc-rio.br id OAA03594 Sender: owner-obm-l@sucuri.mat.puc-rio.br Precedence: bulk Reply-To: obm-l@mat.puc-rio.br Meu,que coisa e essa?Ate agora ninguem me respondeu!!!!!!!! -- Mensagem original -- >Alo turma!!!!!!Tenho mais perguntas a fazer(da Iberoamericana): >1)Ache todos os naturais n de 3,2 ou 1 digito tal que o quadrado de n seja >o cubo da soma dos digitos. >2)Encontre o menor n tal que se pegarmos n dos 999 primeiros inteiros positivos >sempre se acham 4 numeros diferentes a,b,c,d com a+2*b+3*c-4*d=0. >3)L>0 e tal que -L^2+1998*L+1=0.Seja a recorrencia a(0)=1 e a(n+1)=parte >inteira de L*a(n)=[L*a(n)].Calcule a(1998)mod 1998(x mod y e o resto de >x por y). >4)na mesa da banca de lideres da OIM estao lideres de P paises de modo que >se dois lideres quaisquer sao de mesmo pais entao seus vizinhos direitos >nao sao.Quantos lideres ha no maximo? >5)Sabe-se que num conjunto de primos se p e q sao elementos(iguais ou nao)entao >p*q+r,em que r e constante.Quantos elementos tem S com n=4?Generalize o >r. > >TRANSIRE SVVM PECTVS MVNDOQUE POTIRE >CONGREGATI EX TOTO ORBE MATHEMATICI OB SCRIPTA INSIGNIA TRIBVERE >Medalha Fields(John Charles Fields) > > >------------------------------------------ >Use o melhor sistema de busca da Internet >Radar UOL - http://www.radaruol.com.br > > > >========================================================================= >Instruções para entrar na lista, sair da lista e usar a lista em >http://www.mat.puc-rio.br/~nicolau/olimp/obm-l.html >O administrador desta lista é >========================================================================= > TRANSIRE SVVM PECTVS MVNDOQUE POTIRE CONGREGATI EX TOTO ORBE MATHEMATICI OB SCRIPTA INSIGNIA TRIBVERE Medalha Fields(John Charles Fields) ------------------------------------------ Use o melhor sistema de busca da Internet Radar UOL - http://www.radaruol.com.br ========================================================================= Instruções para entrar na lista, sair da lista e usar a lista em http://www.mat.puc-rio.br/~nicolau/olimp/obm-l.html O administrador desta lista é ========================================================================= From owner-obm-l@sucuri.mat.puc-rio.br Thu May 16 15:53:29 2002 Return-Path: Received: (from majordom@localhost) by sucuri.mat.puc-rio.br (8.9.3/8.9.3) id PAA05300 for obm-l-list; Thu, 16 May 2002 15:52:22 -0300 Received: from fgvrj23.fgv.br (fgvrj23.fgv.br [200.20.164.23]) by sucuri.mat.puc-rio.br (8.9.3/8.9.3) with ESMTP id PAA05296 for ; Thu, 16 May 2002 15:52:19 -0300 Received: by FGVRJ23 with Internet Mail Service (5.5.2653.19) id <20VAYNZY>; Thu, 16 May 2002 15:53:44 -0300 Message-ID: <765A72978645D4118B1C0000E229806D05B6B381@FGVRJ23> From: Ralph Teixeira To: "'obm-l@mat.puc-rio.br'" Subject: RES: [obm-l] .......... Date: Thu, 16 May 2002 15:53:43 -0300 MIME-Version: 1.0 X-Mailer: Internet Mail Service (5.5.2653.19) Content-Type: text/plain; charset="iso-8859-1" Content-Transfer-Encoding: 8bit X-MIME-Autoconverted: from quoted-printable to 8bit by sucuri.mat.puc-rio.br id PAA05297 Sender: owner-obm-l@sucuri.mat.puc-rio.br Precedence: bulk Reply-To: obm-l@mat.puc-rio.br Hmmm, eu já vi esta questão antes, há de se tomar cuidado. A equação é x=sqrt(5-sqrt(5-x)). Concordo que: "SE x=sqrt(5-x) ENTÃO x é raiz da equação original." Mas isso NÃO GARANTE que estas são as únicas soluções.... Então, eu faria no braço elevando coisas ao quadrado (e tomando cuidado com raízes estranhas): x^2=5-sqrt(5-x) sqrt(5-x)=5-x^2 5-x=25-10x^2+x^4 x^4-10x^2+x+20=0 Como resolver isto? Se eu pelo menos achasse algumas raízes eu podia fatorar... Mas pera aí, eu sei algumas raízes! Afinal as raízes de x = sqrt(5-x), ou seja, x^2+x-5=0 são raízes da minha equação. Isso sugere a fatoração: x^4-10x^2+x+20=0 (x^2+x-5)(x^2-x-4)=0 E as raízes são quatro: x=(-1+-sqrt(21))/2 x=(1+-sqrt(17))/2 Agora basta conferir e ver se alguma delas é raiz estranha introduzida quando elevamos ao quadrado. De fato, note que podemos reverter os passos acima (tirando a raiz quadrada nos passos-chave) exatamente quando 5-x^2>=0 (isto é) -sqrt(5)<=x<=sqrt(5) E x>=0 (para que sqrt(x^2)=x de fato) Quais das quatro raízes estão entre 0 e 5? Concluímos que as raízes verdadeiras da equação são: (sqrt(21)-1)/2 ~= 1.791 e (sqrt(17)-1)/2 ~= 1.562 Abraço, Ralph ========================================================================= Instruções para entrar na lista, sair da lista e usar a lista em http://www.mat.puc-rio.br/~nicolau/olimp/obm-l.html O administrador desta lista é ========================================================================= From owner-obm-l@sucuri.mat.puc-rio.br Thu May 16 17:45:15 2002 Return-Path: Received: (from majordom@localhost) by sucuri.mat.puc-rio.br (8.9.3/8.9.3) id RAA07040 for obm-l-list; Thu, 16 May 2002 17:44:12 -0300 Received: from web10103.mail.yahoo.com (web10103.mail.yahoo.com [216.136.130.53]) by sucuri.mat.puc-rio.br (8.9.3/8.9.3) with SMTP id RAA07036 for ; Thu, 16 May 2002 17:44:08 -0300 Message-ID: <20020516204209.7102.qmail@web10103.mail.yahoo.com> Received: from [150.161.4.45] by web10103.mail.yahoo.com via HTTP; Thu, 16 May 2002 13:42:09 PDT Date: Thu, 16 May 2002 13:42:09 -0700 (PDT) From: Rafael WC Subject: Re: [obm-l] triângulo To: obm-l@mat.puc-rio.br In-Reply-To: <20020515190642.95877.qmail@web10104.mail.yahoo.com> MIME-Version: 1.0 Content-Type: multipart/mixed; boundary="0-2094278421-1021581729=:5848" Sender: owner-obm-l@sucuri.mat.puc-rio.br Precedence: bulk Reply-To: obm-l@mat.puc-rio.br --0-2094278421-1021581729=:5848 Content-Type: text/plain; charset=us-ascii Content-Disposition: inline Oi de novo! Já que ninguém respondeu, estou mandando a minha resolução que achei horrível! Por isso quero saber se alguém tem alguma idéia de fazer de uma maneira mais simples do que isso. --- Rafael WC wrote: > Olá Pessoal! > > Alguém conseguiria resolver essa pra mim? Não tô > conseguindo... > > AB = 8,AC = 5 e BC = 7 são os lados de um triangulo > ABC. Inscreve-se neste triangulo uma circunferencia > e > traça-se-lhe a tangente paralela ao lado BC, cujos > pontos de interceção com os lados AB e AC são D e E. > Calcular a razão ID/IE, sendo I o ponto de contato > da tangente DE com a circunferencia inscrita no ABC. Estou enviando uma figura pra ver se ajuda. Na figura desenhei o triângulo ABC, inscrevi uma circunferência de centro O, cujos pontos de tangência aos lados AAC, BC e AB são respectivamente F, G, H. Depois tracei uma tangente ao círculo paralela ao lado BC, com ponto de tangência I e cruzando os lados AB e AC em D e E. Ainda marquei dois ângulos que iremos precisar, os ângulos ABC e ACB, que chamei de b e c respectivamente. Primeiro vamos calcular a altura do triângulo para sabermos o seno e cosseno dos ângulos b e c. Poderíamos usar aqui a lei dos cossenos, mas também podemos usar a fórmula de Herão para a área, que é dada por: área = raiz[p.(p - AB).(p - AC).(p - BC)] onde p = semi-perímetro Como AB = 8, AC = 5 e BC = 7, o semi perímetro é: 2p = 8 + 5 + 7 2p = 20 p = 10 E a área será: área = raiz[p.(p - AB).(p - AC).(p - BC)] área = raiz[10.(10 - 8).(10 - 5).(10 - 7)] área = raiz(10.2.5.3) área = raiz(10.10.3) área = 10.raiz(3) Então podemos encontrar a altura AK do triângulo, relativa ao lado BC por exemplo: área = base x altura/2 área = BC x AK/2 10.raiz(3) = 7 x AK/2 20.raiz(3) = 7 x AK AK = 20.raiz(3)/7 Então podemos achar seno, cosseno e tangente de b e c: sen b = AK/AB sen b = [20.raiz(3)/7]/8 sen b = [20.raiz(3)/7].(1/8) sen b = [5.raiz(3)/7].(1/2) sen b = 5.raiz(3)/14 cos² b + sen² b = 1 cos² b + [5.raiz(3)/14]² = 1 cos² b = 1 - [5.raiz(3)/14]² cos² b = 1 - 75/196 cos² b = (196 - 75)/196 cos² b = 121/196 cos b = 11/14 tg b = sen b/cos b tg b = [5.raiz(3)/14]/(11/14) tg b = 5.raiz(3)/11 sen c = AK/AC sen c = [20.raiz(3)/7]/5 sen c = [20.raiz(3)/7].(1/5) sen c = 4.raiz(3)/7 cos² c + sen² c = 1 cos² c + [4.raiz(3)/7]² = 1 cos² c = 1 - [4.raiz(3)/7]² cos² c = 1 - 48/49 cos² c = (49 - 48)/49 cos² c = 1/49 cos c = 1/7 tg c = sen c/cos c tg c = [4.raiz(3)/7]/(1/7) tg c = 4.raiz(3) Como ED é paralela a BC, quando traçamos os raios OI e OG até os pontos de tangência, eles formam um segmento de reta GI, pois os dois raios são perpendiculares a duas paralelas por um mesmo ponto. No quadrilátero BGOH, como a soma dos ângulos internos tem que dar 360°, sabemos que o ângulo GOH = 180° - b. Como GOI = 180° (pois vimos que é uma reta), concluímos que HOI = b (pois é suplementar de GOH). Os triângulos ODH e ODI são congruentes, pois são triângulos retângulos com dois lados congruentes: o lado OD comum e os lados OH e OI, que são raios da circunferência. Assim, o ângulo entre esses lados é congruente. Isso quer dizer que OD divide o ângulo HOI (que vale b) em dois ângulos congruentes, de medida b/2. Com isso, podemos achar o lado DI em função de OI, pela tangente de b/2. Pela fórmula da tangente do arco duplo temos: tg 2x = 2.tg x/(1 - tg² x) (tg 2x).(1 - tg² x) = 2.tg x tg 2x - (tg 2x).(tg² x) = 2.tg x (tg 2x).(tg² x) - tg 2x + 2.tg x = 0 Fazendo x = b/2, temos: (tg 2x).(tg² x) - tg 2x + 2.tg x = 0 (tg 2.b/2).(tg² b/2) - tg 2.b/2 + 2.tg b/2 = 0 (tg b).(tg² b/2) - tg b + 2.tg b/2 = 0 Como sabemos o valor de tg b: [5.raiz(3)/11].(tg² b/2) - 5.raiz(3)/11 + 2.tg b/2 = 0 multiplica tudo por 11, [5.raiz(3)].(tg² b/2) - 5.raiz(3) + 11.2.tg b/2 = 0 [5.raiz(3)].(tg² b/2) - 5.raiz(3) + 22.tg b/2 = 0 multiplica tudo por raiz(3), [5.raiz(3)].(tg² b/2) - 5.raiz(3) + 22.tg b/2 = 0 (5.3).(tg² b/2) - 5.3 + 22.raiz(3).tg b/2 = 0 15.(tg² b/2) - 15 + 22.raiz(3).tg b/2 = 0 Mas iso é uma equação do segundo grau. Vamos chamar tg b/2 de y, para facilitar: 15.(tg² b/2) - 15 + 22.raiz(3).tg b/2 = 0 15.y² - 15 + 22.raiz(3).y = 0 15.y² + 22.raiz(3).y - 15 = 0 E pela fórmula de Báskara encontramos que: y = [-11.raiz(3) +- 17.raiz(3)]/15 tg b/2 = [-11.raiz(3) +- 14.raiz(3)]/15 Como b/2 é um ângulo do primeiro quadrante, sua tangente é positiva: tg b/2 = [-11.raiz(3) + 14.raiz(3)]/15 tg b/2 = 3.raiz(3)/15 tg b/2 = raiz(3)/5 E finalmente encontramos: tg b/2 = ID/OI raiz(3)/5 = ID/OI ID = OI.raiz(3)/5 E agora faremos as mesmas contas para encontrar IE em função de OI. No quadrilátero CGOF, como a soma dos ângulos internos tem que dar 360°, sabemos que o ângulo GOF = 180° - c. Como GOI = 180° (pois vimos que é uma reta), concluímos que FOI = c (pois é suplementar de GOF). Da mesma forma são congruentes os triângulos OEI e OEF: são retângulos e têm dois lados congruentes, OE (comum) e OI = OF (raios). Assim, o ângulo entre esses lados é congruente. Isso quer dizer que OE divide o ângulo FOI (que vale c) em dois ângulos congruentes, de medida c/2. Com isso, podemos achar o lado IE em função de OI, pela tangente de c/2. Pela fórmula da tangente do arco duplo, como já vimos, temos: (tg 2x).(tg² x) - tg 2x + 2.tg x = 0 Fazendo x = c/2, temos: (tg 2x).(tg² x) - tg 2x + 2.tg x = 0 (tg 2.c/2).(tg² c/2) - tg 2.c/2 + 2.tg c/2 = 0 (tg c).(tg² c/2) - tg c + 2.tg c/2 = 0 Como sabemos o valor de tg c: 4.raiz(3).(tg² c/2) - 4.raiz(3) + 2.tg c/2 = 0 multiplica tudo por raiz(3), 4.3.(tg² c/2) - 4.3 + 2.raiz(3).tg c/2 = 0 12.(tg² c/2) - 12 + 2.raiz(3).tg c/2 = 0 divide tudo por 2, 6.(tg² c/2) - 6 + raiz(3).tg c/2 = 0 Mas iso é uma equação do segundo grau. Vamos chamar tg c/2 de y, para facilitar: 6.(tg² c/2) - 6 + raiz(3).tg c/2 = 0 6.y² - 6 + raiz(3).y = 0 6.y² + raiz(3).y - 6 = 0 E pela fórmula de Báskara encontramos que: y = [-raiz(3) +- 7.raiz(3)]/12 tg c/2 = [-raiz(3) +- 7.raiz(3)]/12 Como c/2 é um ângulo do primeiro quadrante, sua tangente é positiva: tg c/2 = [-raiz(3) + 7.raiz(3)]/12 tg c/2 = 6.raiz(3)/12 tg c/2 = raiz(3)/2 E finalmente encontramos: tg c/2 = IE/OI raiz(3)/2 = IE/OI IE = OI.raiz(3)/2 Como o problema pediu a razão ID/IE: = ID/IE = [OI.raiz(3)/5]/[OI.raiz(3)/2] = [OI.raiz(3)/5].[2/OI.raiz(3)] = [OI/5].[2/OI] = 2/5 Ufa! Resposta: ID/IE = 2/5 Abraços, Rafael ===== Rafael Werneck Cinoto ICQ# 107011599 rwcinoto@yahoo.com rafael.caixa@gov.com.br matduvidas@yahoo.com.br http://www.rwcinoto.hpg.com.br/ __________________________________________________ Do You Yahoo!? LAUNCH - Your Yahoo! Music Experience http://launch.yahoo.com --0-2094278421-1021581729=:5848 Content-Type: image/gif; name="circinsctri.gif" Content-Transfer-Encoding: base64 Content-Description: circinsctri.gif Content-Disposition: inline; filename="circinsctri.gif" R0lGODdhNwGvAPcAAAAAAAAAQAAAgAAA/wAgAAAgQAAggAAg/wBAAABAQABA gABA/wBgAABgQABggABg/wCAAACAQACAgACA/wCgAACgQACggACg/wDAAADA QADAgADA/wD/AAD/QAD/gAD//yAAACAAQCAAgCAA/yAgACAgQCAggCAg/yBA ACBAQCBAgCBA/yBgACBgQCBggCBg/yCAACCAQCCAgCCA/yCgACCgQCCggCCg /yDAACDAQCDAgCDA/yD/ACD/QCD/gCD//0AAAEAAQEAAgEAA/0AgAEAgQEAg gEAg/0BAAEBAQEBAgEBA/0BgAEBgQEBggEBg/0CAAECAQECAgECA/0CgAECg QECggECg/0DAAEDAQEDAgEDA/0D/AED/QED/gED//2AAAGAAQGAAgGAA/2Ag AGAgQGAggGAg/2BAAGBAQGBAgGBA/2BgAGBgQGBggGBg/2CAAGCAQGCAgGCA /2CgAGCgQGCggGCg/2DAAGDAQGDAgGDA/2D/AGD/QGD/gGD//4AAAIAAQIAA gIAA/4AgAIAgQIAggIAg/4BAAIBAQIBAgIBA/4BgAIBgQIBggIBg/4CAAICA QICAgICA/4CgAICgQICggICg/4DAAIDAQIDAgIDA/4D/AID/QID/gID//6AA AKAAQKAAgKAA/6AgAKAgQKAggKAg/6BAAKBAQKBAgKBA/6BgAKBgQKBggKBg /6CAAKCAQKCAgKCA/6CgAKCgQKCggKCg/6DAAKDAQKDAgKDA/6D/AKD/QKD/ gKD//8AAAMAAQMAAgMAA/8AgAMAgQMAggMAg/8BAAMBAQMBAgMBA/8BgAMBg QMBggMBg/8CAAMCAQMCAgMCA/8CgAMCgQMCggMCg/8DAAMDAQMDAgMDA/8D/ AMD/QMD/gMD///8AAP8AQP8AgP8A//8gAP8gQP8ggP8g//9AAP9AQP9AgP9A //9gAP9gQP9ggP9g//+AAP+AQP+AgP+A//+gAP+gQP+ggP+g///AAP/AQP/A gP/A////AP//QP//gP///yH5BAAAAAAALAAAAAA3Aa8AAAj/AP8JHEiwoMGD CBMqXMiwocOHECNKnEixosWLGDNq3Mixo8ePIEOKHEmypMmTKFOqXMmypcuX MGPKnCkQgE2aOHPq3FkSwECfPIMKHUo0IdCfRZMqXSrzKNOnUKP2lEq1qlWM Tq9q3cqVYNauYMNCzfpVrNmzOI+WRcu2bUuba93KnUu3rt27ePPq3cu3r9+/ gAMLHky4sOHDiBMrXmw3LuPHVH3ehEzZqtrKmMcWnAxxgEHPmUOPLOv4IOiB p0Wr7hiXM8PUqVfLzli6NOrbs3NjNQrUNULQsXULj7j2a2nPA4IPX76QdPG4 yQUqZ059M9yattfCrs7doFPfCsHHJp7efbjt5gWDky8/+/n5f+DZyyeNtOGA 9/J1F48IOn5+5vt15tV//921JqB1BFJnYE0LkYdfgo/V5h9uvEGY24MJrVef haJh+NtDE3KomIefSUTiT9eJWNeJBGlYoYkDqjgXixROhKFxMrpFo3Rw9ehj igeFuFmOd50I130g9uidQzsSqRlESv7jYnhACgkfilY6GVmSJdJ2WXhLahnW eUC2yNpNE+IoJlgSZhgSmkbFuCZXjgk5pUVRelfmnFo995pJPvLZVoANppSn oGIZd96dHAWK6Jhh2rfSdU0+qhNZDzHKGoNZWlrUd/hp2qhXknlq2YaSThrk nqYq9WWmLDnHYKtM9cZfrFcO2VultAI6K6yqYrfhq70GVeqBMcE5LK/FfnQs snlNRZprs2lN65CouJJK7UzPXpuUmtuC9OOL0KJEoqLhOttQp2YGa2OQ6W7K ELvSZUuRn/HSNi+G2G6044L54nkoqQ/2q+9uCNEb8JBUCvunuxfVtvC7YMr5 ob0Ikzvxug1b3GVVZG7MpMbWmtbnusya6t53Chm8E34p/287rmMuZywvyiIT 5xp0b71JXM5ceovxzUEDTbK2NRo6GsVGr9qcTzVHTJKRMSPaKQBR4znVvU0j /afCrgrcdbduotVk1VqSbTJblYLtqdprO+2zuRqhrSLc6e2rrtJ12w2hrS0X XbdKMftNIOAXQ8mr4XGemS6xHxM+tL/hIh73jOIyriCqeXu5lOGaLwd5u31/ O1roF3qcdOnovRQ66rKxHLlHErsOKOyhYdr55BBPbTXDpG8NH+4Vn+R2eeDW a6ijWOJO/K9OJp+1ieOiCvvzDudIn/K3q7WztCJhzzmH20s59Z7ogz/3W+IL FuD0CbPKqe6qi5usiPvBb13tvqYlH35T7etLa/Q3vKtdiX7jsx+3EjRAL92I U8BL4N64FUC80IxFAxsZBCOIugo2LjM/qpObbiQ/ENUPO8djWk5SKEATjpBJ LJSTByN4KczArGx1KiHXJNgzocTQghqMm3PQhsD7EWWGTwlhwhI3qwyOalhG /BQSwzavsmFJeL+a4RRdmJiQrY15SwOeFpO4RR9W0TRlep20xvgkw7SJVA7y HQdhUkbq/7mxeih0VBz/t0Y6nooyA9OQ3whVMsld5YeNyZOL7EbIKB6yjkNh 2ZQYuUSaQNJmbvTOnaoGMEfS6ZIrLJEXKVfJBZoFlKYknZVS1kno8e0siEzU 7Gp3sFLyUI5sc9/ssgcvTNLQlcabESqHtp5WqrCXv/QVXYb5Su4dDZg/K94J w7giZipzdbac5hmleUsFAjEvWdEUeIy0zWRS04LWXJ/5YGRObmazm7TzSzrj iU0YavOdyHTJPI23z8HVk4uFdGc+exiYfkrtn4qDJj4HyjsBGnSH6/Rn+eCp ULplEpYIvRfZJDO6hVb0iYeJ5csyWpFnseyGH1QnYh7KMWfSjvGj6hMoRbUW IZZWjIC88V/rPFpLyNgUmTjNaUdTylDW2fCnMQoqGCH4QJkedDHGrJVLQaon D9GSlIzJoVSgxsdV6TCm7bQRUvVZTioGdZp43CkWIzQ8PfHyUxH1pty++tFG jZWOltOpseIq16elqSd3jaLsEHTEs4I1TvFhnEj38ioRmtGwYY1fAe+ZR7pO NnZQ7NjLIBvZD05mlB9d7F8a29IVcraz5HIiDRUlWsAc6j2txchpm4dBwC21 rQO6LfKgZMnZvnSIIdTtf0gok/sE97jITa5yl8vc5jr3udCNrnSnS93qKten lsVt1woT2+16c/e74E2basO7v+zmkqJFXFh6V9TZ9ebLvTzdantR2yz4Limw qy0rfgmT1irul3PZ6a966XtANs2XstSyrwy7ouCS/VeXCJ7WgwfLzQcXFLVK 3IpwwWdh1244v+Ak8HYnSt6LirjEKE6xilfM4ha7+MViCQgAOw== --0-2094278421-1021581729=:5848-- ========================================================================= Instruções para entrar na lista, sair da lista e usar a lista em http://www.mat.puc-rio.br/~nicolau/olimp/obm-l.html O administrador desta lista é ========================================================================= From owner-obm-l@sucuri.mat.puc-rio.br Thu May 16 18:10:39 2002 Return-Path: Received: (from majordom@localhost) by sucuri.mat.puc-rio.br (8.9.3/8.9.3) id SAA07533 for obm-l-list; Thu, 16 May 2002 18:10:33 -0300 Received: from toole.uol.com.br (toole.uol.com.br [200.231.206.186]) by sucuri.mat.puc-rio.br (8.9.3/8.9.3) with ESMTP id SAA07529 for ; Thu, 16 May 2002 18:10:30 -0300 Received: from focus01 ([200.197.202.201]) by toole.uol.com.br (8.9.1/8.9.1) with SMTP id SAA13479 for ; Thu, 16 May 2002 18:04:14 -0300 (BRT) Message-ID: <000a01c1fd1e$0e50b120$030c100a@focus01> From: "Marcelo Roseira" To: "OBM" Subject: [obm-l] =?iso-8859-1?Q?Boa_quest=E3o_de_probabilidade?= Date: Thu, 16 May 2002 18:10:05 -0300 MIME-Version: 1.0 Content-Type: multipart/alternative; boundary="----=_NextPart_000_0007_01C1FD04.E7BCEF60" X-Priority: 3 X-MSMail-Priority: Normal X-Mailer: Microsoft Outlook Express 5.00.2615.200 X-MimeOLE: Produced By Microsoft MimeOLE V5.00.2615.200 Sender: owner-obm-l@sucuri.mat.puc-rio.br Precedence: bulk Reply-To: obm-l@mat.puc-rio.br This is a multi-part message in MIME format. ------=_NextPart_000_0007_01C1FD04.E7BCEF60 Content-Type: text/plain; charset="iso-8859-1" Content-Transfer-Encoding: quoted-printable A=ED vai galera... Uma pessoa coloca sua bicicleta na =FAnica vaga ainda vazia na grade de = um estacionamento de bicicletas de um supermercado. Observa que a sua = bicicleta est=E1 entre 9 outras e a vaga que ocupa n=E3o fica em = qualquer das duas extremidades da grade. Depois das compras a pessoa = volta e encontra, al=E9m da sua, apenas 5 das 9 bicicletas ainda = estacionadas na grade. Pede-se: 1. A probabilidade de a pessoa encontrar vazia a vaga adjacente =E0 = direita da sua bicicleta. 2. A probabilidade de a pessoa encontrar vazias as duas vagas adjacentes = =E0 da sua bicicleta. 3. A probabilidade de a pessoa encontrar vazia a vaga adjacente =E0 = esquerda da sua bicicleta ou a vaga adjacente =E0 direita da sua = bicicleta, admitindo-se que os dois eventos sejam independentes. 4. A probabilidade de a pessoa encontrar vazia a vaga da extremidade = esquerda da grade. Grato. Marcelo ------=_NextPart_000_0007_01C1FD04.E7BCEF60 Content-Type: text/html; charset="iso-8859-1" Content-Transfer-Encoding: quoted-printable
A=ED vai galera...
 
Uma pessoa coloca sua bicicleta na = =FAnica vaga ainda=20 vazia na grade de um estacionamento de bicicletas de um supermercado. = Observa=20 que a sua bicicleta est=E1 entre 9 outras e a vaga que ocupa n=E3o fica = em qualquer=20 das duas extremidades da grade. Depois das compras a pessoa volta e = encontra,=20 al=E9m da sua, apenas 5 das 9 bicicletas ainda estacionadas na = grade.
 
Pede-se:
 
1. A probabilidade de a pessoa = encontrar vazia a=20 vaga adjacente =E0 direita da sua bicicleta.
 
2. A = probabilidade de a=20 pessoa encontrar vazias as duas vagas adjacentes =E0 da sua = bicicleta.
 
3. A probabilidade de a pessoa = encontrar vazia a=20 vaga adjacente =E0 esquerda da sua bicicleta ou a vaga adjacente = =E0 direita da=20 sua bicicleta, admitindo-se que os dois eventos sejam = independentes.
 
4. A probabilidade de a pessoa = encontrar vazia a=20 vaga da extremidade esquerda da grade.
 
Grato. Marcelo
------=_NextPart_000_0007_01C1FD04.E7BCEF60-- ========================================================================= Instruções para entrar na lista, sair da lista e usar a lista em http://www.mat.puc-rio.br/~nicolau/olimp/obm-l.html O administrador desta lista é ========================================================================= From owner-obm-l@sucuri.mat.puc-rio.br Thu May 16 18:14:48 2002 Return-Path: Received: (from majordom@localhost) by sucuri.mat.puc-rio.br (8.9.3/8.9.3) id SAA07704 for obm-l-list; Thu, 16 May 2002 18:14:45 -0300 Received: from cairu.terra.com.br (cairu.terra.com.br [200.176.3.19]) by sucuri.mat.puc-rio.br (8.9.3/8.9.3) with ESMTP id SAA07700 for ; Thu, 16 May 2002 18:14:43 -0300 Received: from srv11-sao.terra.com.br (srv11-sao.terra.com.br [200.176.3.38]) by cairu.terra.com.br (Postfix) with ESMTP id 6140949762 for ; Thu, 16 May 2002 18:12:56 +0000 (GMT) Received: from Itautec.terra.com.br (200-158-60-200.dsl.telesp.net.br [200.158.60.200]) (authenticated user bruleite) by srv11-sao.terra.com.br (Postfix) with ESMTP id C3740C950C for ; Thu, 16 May 2002 18:12:56 -0300 (EST) Message-Id: <5.1.0.14.0.20020516181244.00ac82f0@pop.sao.terra.com.br> X-Sender: bruleite@pop.sao.terra.com.br X-Mailer: QUALCOMM Windows Eudora Version 5.1 Date: Thu, 16 May 2002 18:12:57 -0300 To: obm-l@mat.puc-rio.br From: "Bruno F. C. Leite" Subject: Re: [obm-l] Apelo: Mais da Iberoamericana In-Reply-To: <3CE14E4D0000491A@www.zipmail.com.br> References: <3CE14E0300000DD4@www.zipmail.com.br> Mime-Version: 1.0 Content-Type: text/plain; charset="iso-8859-1"; format=flowed Content-Transfer-Encoding: 8bit X-MIME-Autoconverted: from quoted-printable to 8bit by sucuri.mat.puc-rio.br id SAA07701 Sender: owner-obm-l@sucuri.mat.puc-rio.br Precedence: bulk Reply-To: obm-l@mat.puc-rio.br At 14:21 16/05/02 -0300, you wrote: >Meu,que coisa e essa?Ate agora ninguem me respondeu!!!!!!!! Ninguém é pago para isso. Bruno >-- Mensagem original -- > > >Alo turma!!!!!!Tenho mais perguntas a fazer(da Iberoamericana): > >1)Ache todos os naturais n de 3,2 ou 1 digito tal que o quadrado de n seja > >o cubo da soma dos digitos. > >2)Encontre o menor n tal que se pegarmos n dos 999 primeiros inteiros > positivos > >sempre se acham 4 numeros diferentes a,b,c,d com a+2*b+3*c-4*d=0. > >3)L>0 e tal que -L^2+1998*L+1=0.Seja a recorrencia a(0)=1 e a(n+1)=parte > >inteira de L*a(n)=[L*a(n)].Calcule a(1998)mod 1998(x mod y e o resto de > >x por y). > >4)na mesa da banca de lideres da OIM estao lideres de P paises de modo >que > >se dois lideres quaisquer sao de mesmo pais entao seus vizinhos direitos > >nao sao.Quantos lideres ha no maximo? > >5)Sabe-se que num conjunto de primos se p e q sao elementos(iguais ou > nao)entao > >p*q+r,em que r e constante.Quantos elementos tem S com n=4?Generalize o > >r. > > > >TRANSIRE SVVM PECTVS MVNDOQUE POTIRE > >CONGREGATI EX TOTO ORBE MATHEMATICI OB SCRIPTA INSIGNIA TRIBVERE > >Medalha Fields(John Charles Fields) > > > > > >------------------------------------------ > >Use o melhor sistema de busca da Internet > >Radar UOL - http://www.radaruol.com.br > > > > > > > >========================================================================= > >Instruções para entrar na lista, sair da lista e usar a lista em > >http://www.mat.puc-rio.br/~nicolau/olimp/obm-l.html > >O administrador desta lista é > >========================================================================= > > > >TRANSIRE SVVM PECTVS MVNDOQUE POTIRE >CONGREGATI EX TOTO ORBE MATHEMATICI OB SCRIPTA INSIGNIA TRIBVERE >Medalha Fields(John Charles Fields) > > >------------------------------------------ >Use o melhor sistema de busca da Internet >Radar UOL - http://www.radaruol.com.br > > > >========================================================================= >Instruções para entrar na lista, sair da lista e usar a lista em >http://www.mat.puc-rio.br/~nicolau/olimp/obm-l.html >O administrador desta lista é >========================================================================= ========================================================================= Instruções para entrar na lista, sair da lista e usar a lista em http://www.mat.puc-rio.br/~nicolau/olimp/obm-l.html O administrador desta lista é ========================================================================= From owner-obm-l@sucuri.mat.puc-rio.br Thu May 16 18:32:11 2002 Return-Path: Received: (from majordom@localhost) by sucuri.mat.puc-rio.br (8.9.3/8.9.3) id SAA08433 for obm-l-list; Thu, 16 May 2002 18:31:58 -0300 Received: from hotmail.com (law2-f138.hotmail.com [216.32.181.138]) by sucuri.mat.puc-rio.br (8.9.3/8.9.3) with ESMTP id SAA08425 for ; Thu, 16 May 2002 18:31:55 -0300 Received: from mail pickup service by hotmail.com with Microsoft SMTPSVC; Thu, 16 May 2002 14:30:08 -0700 Received: from 200.229.244.100 by lw2fd.hotmail.msn.com with HTTP; Thu, 16 May 2002 21:30:08 GMT X-Originating-IP: [200.229.244.100] From: "Paulo Santa Rita" To: obm-l@mat.puc-rio.br Subject: Re: [obm-l] Apelo: Mais da Iberoamericana Date: Thu, 16 May 2002 21:30:08 +0000 Mime-Version: 1.0 Content-Type: text/plain; charset=iso-8859-1; format=flowed Message-ID: X-OriginalArrivalTime: 16 May 2002 21:30:08.0421 (UTC) FILETIME=[DA2C6D50:01C1FD20] Sender: owner-obm-l@sucuri.mat.puc-rio.br Precedence: bulk Reply-To: obm-l@mat.puc-rio.br Ola Dirichlet, Ninguem respondeu, MUITO PROVAVELMENTE, porque as suas questoes, 1) Estao mal formuladas. Por exemplo, voce escreveu : >5)Sabe-se que num conjunto de primos se p e q sao elementos(iguais ou > >nao)entao p*q+r,em que r e constante.Quantos elementos tem S com >n=4?>Generalize o r. p*q+r O QUE ? E ESSE n=4, O QUE E ? Essa mal formulacao EVIDENTE impossibilita uma solucao pode ter lancado uma descrenca quanto a correcao do enunciado das demais questoes. 2) A maioria delas ja tem solucao nos arquivos de mensagens que o Prof Nicolau guarda. por exemplo, voce escreveu : >1)Ache todos os naturais n de 3,2 ou 1 digito tal que o quadrado de n >seja >o cubo da soma dos digitos. Esta questao ( ou outra semelhante ) ja foi respondida e a solucao esta no arquivos de mensagens a que me referi acima. La voce vai ver uma linha de raciocinio proxima de : (a+b)^3=(10a+b)^2 => a+b = [(10a+b)/a+b]^2 e portanto "a+b" e quadrado perfeito e a+b divide 10a+b. Como a+b =< 18, os possiveis valores quadrado de a+b serao ... >3)L>0 e tal que -L^2+1998*L+1=0.Seja a recorrencia a(0)=1 e a(n+1)>parte >inteira de L*a(n)=[L*a(n)].Calcule a(1998)mod 1998(x mod y e o >resto de x >por y). RESTO DE QUE ? ADVINHANDO ... Se x e a unica solucao positiva da equacao do 2 grau do enunciado de sua questao e [x] e a funcao maximo inteiro, o problema consiste em determinar o valor de [x...[x[x[x]]]...] com 1998 colchetes. como [x] = k se k =< x < K+1 entao [x[x]] e a anlise de [kx] e assim sucessivamente. ESTA QUESTAO JA FOI RESOLVIDA NA LISTA ! >4)na mesa da banca de lideres da OIM estao lideres de P paises de modo >que se dois lideres quaisquer sao de mesmo pais entao seus vizinhos > >direitos nao sao.Quantos lideres ha no maximo ? FALTAM INFORMACOES ! Se P=3, sejam A,B e C lideres, dispostos ao longo de uma mesa nesta ordem. Posso sempre inserir entre dois deles o terceiro, de forma que a direita de cada um nao havera duplicacao. EVIDENTEMENTE que este processo pode ser extendido AD INFINITUM ! Meu, que coisas sao essas?Ate agora ninguem ENTENDEU !!!!!! Um abraco Paulo Santa Rita 5,1827,160502 >From: peterdirichlet@zipmail.com.br >Reply-To: obm-l@mat.puc-rio.br >To: obm-l@mat.puc-rio.br >Subject: [obm-l] Apelo: Mais da Iberoamericana >Date: Thu, 16 May 2002 14:21:45 -0300 > >Meu,que coisa e essa?Ate agora ninguem me respondeu!!!!!!!! > >-- Mensagem original -- > > >Alo turma!!!!!!Tenho mais perguntas a fazer(da Iberoamericana): > >1)Ache todos os naturais n de 3,2 ou 1 digito tal que o quadrado de n >seja > >o cubo da soma dos digitos. > >2)Encontre o menor n tal que se pegarmos n dos 999 primeiros inteiros >positivos > >sempre se acham 4 numeros diferentes a,b,c,d com a+2*b+3*c-4*d=0. > >3)L>0 e tal que -L^2+1998*L+1=0.Seja a recorrencia a(0)=1 e a(n+1)=parte > >inteira de L*a(n)=[L*a(n)].Calcule a(1998)mod 1998(x mod y e o resto de > >x por y). > >4)na mesa da banca de lideres da OIM estao lideres de P paises de modo >que > >se dois lideres quaisquer sao de mesmo pais entao seus vizinhos direitos > >nao sao.Quantos lideres ha no maximo? > >5)Sabe-se que num conjunto de primos se p e q sao elementos(iguais ou >nao)entao > >p*q+r,em que r e constante.Quantos elementos tem S com n=4?Generalize o > >r. > > > >TRANSIRE SVVM PECTVS MVNDOQUE POTIRE > >CONGREGATI EX TOTO ORBE MATHEMATICI OB SCRIPTA INSIGNIA TRIBVERE > >Medalha Fields(John Charles Fields) > > > > > >------------------------------------------ > >Use o melhor sistema de busca da Internet > >Radar UOL - http://www.radaruol.com.br > > > > > > > >========================================================================= > >Instruções para entrar na lista, sair da lista e usar a lista em > >http://www.mat.puc-rio.br/~nicolau/olimp/obm-l.html > >O administrador desta lista é > >========================================================================= > > > >TRANSIRE SVVM PECTVS MVNDOQUE POTIRE >CONGREGATI EX TOTO ORBE MATHEMATICI OB SCRIPTA INSIGNIA TRIBVERE >Medalha Fields(John Charles Fields) > > >------------------------------------------ >Use o melhor sistema de busca da Internet >Radar UOL - http://www.radaruol.com.br > > > >========================================================================= >Instruções para entrar na lista, sair da lista e usar a lista em >http://www.mat.puc-rio.br/~nicolau/olimp/obm-l.html >O administrador desta lista é >========================================================================= _________________________________________________________________ Chegou o novo MSN Explorer. Instale já. É gratuito: http://explorer.msn.com.br ========================================================================= Instruções para entrar na lista, sair da lista e usar a lista em http://www.mat.puc-rio.br/~nicolau/olimp/obm-l.html O administrador desta lista é ========================================================================= From owner-obm-l@sucuri.mat.puc-rio.br Thu May 16 19:11:20 2002 Return-Path: Received: (from majordom@localhost) by sucuri.mat.puc-rio.br (8.9.3/8.9.3) id TAA09580 for obm-l-list; Thu, 16 May 2002 19:10:14 -0300 Received: from hotmail.com (law2-f63.hotmail.com [216.32.181.63]) by sucuri.mat.puc-rio.br (8.9.3/8.9.3) with ESMTP id TAA09566 for ; Thu, 16 May 2002 19:10:11 -0300 Received: from mail pickup service by hotmail.com with Microsoft SMTPSVC; Thu, 16 May 2002 15:08:20 -0700 Received: from 200.229.244.100 by lw2fd.hotmail.msn.com with HTTP; Thu, 16 May 2002 22:08:19 GMT X-Originating-IP: [200.229.244.100] From: "Paulo Santa Rita" To: obm-l@mat.puc-rio.br Subject: [obm-l] =?iso-8859-1?B?UmU6IFtvYm0tbF0gQm9hIHF1ZXN0428gZGUgcHJvYmFiaWxpZGFkZQ==?= Date: Thu, 16 May 2002 22:08:19 +0000 Mime-Version: 1.0 Content-Type: text/plain; charset=iso-8859-1; format=flowed Message-ID: X-OriginalArrivalTime: 16 May 2002 22:08:20.0092 (UTC) FILETIME=[301DABC0:01C1FD26] Sender: owner-obm-l@sucuri.mat.puc-rio.br Precedence: bulk Reply-To: obm-l@mat.puc-rio.br Ola marcelo e demais colegas desta lista, Se a vaga era a unica e a bicicleta ficou entre nove outras biciletas e porque o estacionamento comporta 10 vagas. Enumerando estas vagas da esquerda para a direita, a partir de 1, a vaga em que o protagonista colocou a bicileta so pode ser uma dentre 2,3,4,5,6,7,8 e 9. Quando ele volta, encontra apenas 5 das 9 outras biciletas que haviam. Portanto, 4 biciletas foram retiradas. De quantas maneiras e possivel retirar 4 de um total de 9 ? Evidentemente : BINOM(9,4). Dentre estas possibilidades, em quais a vaga a direita da vaga ocupada pela bicicleta do protagonista ficou vazia ? Fixando esta vaga, isto e, supondo que a bicicleta que la estava foi retirada, posso retirar 3 outras bicicletas de um total de oito de BINOM(8,3) maneiras. A probabilidade que voce procura e, portanto : P = BINOM(8,3)/BINOM(9,4) = 56/126 Os outros itens podem ser tratados como variacoes ou acrescimos a esta linha de raciocinio. Um abraco Paulo Santa Ritya 5,1904,160502 >From: "Marcelo Roseira" >Reply-To: obm-l@mat.puc-rio.br >To: "OBM" >Subject: [obm-l] Boa questão de probabilidade >Date: Thu, 16 May 2002 18:10:05 -0300 > >Aí vai galera... > >Uma pessoa coloca sua bicicleta na única vaga ainda vazia na grade de um >estacionamento de bicicletas de um supermercado. Observa que a sua >bicicleta está entre 9 outras e a vaga que ocupa não fica em qualquer das >duas extremidades da grade. Depois das compras a pessoa volta e encontra, >além da sua, apenas 5 das 9 bicicletas ainda estacionadas na grade. > >Pede-se: > >1. A probabilidade de a pessoa encontrar vazia a vaga adjacente à direita >da sua bicicleta. > >2. A probabilidade de a pessoa encontrar vazias as duas vagas adjacentes à >da sua bicicleta. > >3. A probabilidade de a pessoa encontrar vazia a vaga adjacente à esquerda >da sua bicicleta ou a vaga adjacente à direita da sua bicicleta, >admitindo-se que os dois eventos sejam independentes. > >4. A probabilidade de a pessoa encontrar vazia a vaga da extremidade >esquerda da grade. > >Grato. Marcelo _________________________________________________________________ Envie e receba emails com o Hotmail no seu dispositivo móvel: http://mobile.msn.com ========================================================================= Instruções para entrar na lista, sair da lista e usar a lista em http://www.mat.puc-rio.br/~nicolau/olimp/obm-l.html O administrador desta lista é ========================================================================= From owner-obm-l@sucuri.mat.puc-rio.br Thu May 16 19:47:49 2002 Return-Path: Received: (from majordom@localhost) by sucuri.mat.puc-rio.br (8.9.3/8.9.3) id TAA10263 for obm-l-list; Thu, 16 May 2002 19:47:43 -0300 Received: from web14810.mail.yahoo.com (web14810.mail.yahoo.com [216.136.224.231]) by sucuri.mat.puc-rio.br (8.9.3/8.9.3) with SMTP id TAA10259 for ; Thu, 16 May 2002 19:47:40 -0300 Message-ID: <20020516224553.82350.qmail@web14810.mail.yahoo.com> Received: from [200.243.206.208] by web14810.mail.yahoo.com via HTTP; Thu, 16 May 2002 19:45:53 ART Date: Thu, 16 May 2002 19:45:53 -0300 (ART) From: =?iso-8859-1?q?Ricardo=20Miranda?= Subject: Re: [obm-l] Por favor... To: obm-l@mat.puc-rio.br In-Reply-To: <003901c1fcef$0ae44580$a2dbcdc8@user> MIME-Version: 1.0 Content-Type: text/plain; charset=iso-8859-1 Content-Transfer-Encoding: 8bit Sender: owner-obm-l@sucuri.mat.puc-rio.br Precedence: bulk Reply-To: obm-l@mat.puc-rio.br --- Claudio escreveu: > Olá Raul. > Chame de N o ponto médio do arco BC. Chame de L o ponto médio do raio > ON. > Note que o triângulo ANO 'é equilátero ! > A circunferência de diâmetro AO contém os pontos M e L. Conclua que o > ângulo procurado mede 30 º . > Saludos. > Claudio Casemiro. > ----- Original Message ----- > From: Euraul@aol.com > To: obm-l@mat.puc-rio.br > Sent: Thursday, May 16, 2002 1:26 AM > Subject: [obm-l] Por favor... > > > Sejam AB e BC dois lados adjacentes de um polígono regular de 9 > lados, inscrito em uma circunferência de lado O. Seja M o ponto médio do > lado AB e N o ponto médio do raio perpendicular a BC. Qual a medida do > ângulo OMN ? > Obrigado > Raul Enviei uma outra resolucao, que está errada. Nao tinha visto ".. N o PONTO MEDIO DO RAIO PERP. A BC..". Fiz com M o médio de AB e N o medio de BC. Mals.. ===== []s -- Ricardo Miranda ricardomirandabr@yahoo.com.br http://rm2.hpg.ig.com.br/ _______________________________________________________________________ Yahoo! Encontros O lugar certo para você encontrar aquela pessoa que falta na sua vida. Cadastre-se hoje mesmo! http://br.encontros.yahoo.com/ ========================================================================= Instruções para entrar na lista, sair da lista e usar a lista em http://www.mat.puc-rio.br/~nicolau/olimp/obm-l.html O administrador desta lista é ========================================================================= From owner-obm-l@sucuri.mat.puc-rio.br Thu May 16 19:48:21 2002 Return-Path: Received: (from majordom@localhost) by sucuri.mat.puc-rio.br (8.9.3/8.9.3) id TAA10291 for obm-l-list; Thu, 16 May 2002 19:48:20 -0300 Received: from web14804.mail.yahoo.com (web14804.mail.yahoo.com [216.136.224.220]) by sucuri.mat.puc-rio.br (8.9.3/8.9.3) with SMTP id TAA10287 for ; Thu, 16 May 2002 19:48:17 -0300 Message-ID: <20020516224629.6148.qmail@web14804.mail.yahoo.com> Received: from [200.243.206.208] by web14804.mail.yahoo.com via HTTP; Thu, 16 May 2002 19:46:29 ART Date: Thu, 16 May 2002 19:46:29 -0300 (ART) From: =?iso-8859-1?q?Ricardo=20Miranda?= Subject: Re: [obm-l] Por favor... To: obm-l@mat.puc-rio.br In-Reply-To: <003901c1fcef$0ae44580$a2dbcdc8@user> MIME-Version: 1.0 Content-Type: text/plain; charset=iso-8859-1 Content-Transfer-Encoding: 8bit Sender: owner-obm-l@sucuri.mat.puc-rio.br Precedence: bulk Reply-To: obm-l@mat.puc-rio.br --- Claudio escreveu: > Olá Raul. > Chame de N o ponto médio do arco BC. Chame de L o ponto médio do raio > ON. > Note que o triângulo ANO 'é equilátero ! > A circunferência de diâmetro AO contém os pontos M e L. Conclua que o > ângulo procurado mede 30 º . > Saludos. > Claudio Casemiro. > ----- Original Message ----- > From: Euraul@aol.com > To: obm-l@mat.puc-rio.br > Sent: Thursday, May 16, 2002 1:26 AM > Subject: [obm-l] Por favor... > > > Sejam AB e BC dois lados adjacentes de um polígono regular de 9 > lados, inscrito em uma circunferência de lado O. Seja M o ponto médio do > lado AB e N o ponto médio do raio perpendicular a BC. Qual a medida do > ângulo OMN ? > Obrigado > Raul Enviei uma outra resolucao, que está errada. Nao tinha visto ".. N o PONTO MEDIO DO RAIO PERP. A BC..". Fiz com M o médio de AB e N o medio de BC. Brigado pra Fernanda M, que me perguntou de onde eu tinha tirado aquilo e me fez achar o erro =P ===== []s -- Ricardo Miranda ricardomirandabr@yahoo.com.br http://rm2.hpg.ig.com.br/ _______________________________________________________________________ Yahoo! Encontros O lugar certo para você encontrar aquela pessoa que falta na sua vida. Cadastre-se hoje mesmo! http://br.encontros.yahoo.com/ ========================================================================= Instruções para entrar na lista, sair da lista e usar a lista em http://www.mat.puc-rio.br/~nicolau/olimp/obm-l.html O administrador desta lista é ========================================================================= From owner-obm-l@sucuri.mat.puc-rio.br Thu May 16 22:27:26 2002 Return-Path: Received: (from majordom@localhost) by sucuri.mat.puc-rio.br (8.9.3/8.9.3) id WAA12258 for obm-l-list; Thu, 16 May 2002 22:26:58 -0300 Received: from shannon.bol.com.br (shannon.bol.com.br [200.221.24.13]) by sucuri.mat.puc-rio.br (8.9.3/8.9.3) with ESMTP id WAA12254 for ; Thu, 16 May 2002 22:26:56 -0300 Received: from bol.com.br (200.221.24.77) by shannon.bol.com.br (5.1.071) id 3CE1A94700092839 for obm-l@mat.puc-rio.br; Thu, 16 May 2002 22:24:21 -0300 Date: Thu, 16 May 2002 22:24:20 -0300 Message-Id: Subject: Re: [obm-l] .......... MIME-Version: 1.0 Content-Type: text/plain;charset="iso-8859-1" From: "rafaelc.l" To: obm-l@mat.puc-rio.br X-XaM3-API-Version: 2.4.3.4.4 X-SenderIP: 200.176.166.114 Content-Transfer-Encoding: 8bit X-MIME-Autoconverted: from quoted-printable to 8bit by sucuri.mat.puc-rio.br id WAA12255 Sender: owner-obm-l@sucuri.mat.puc-rio.br Precedence: bulk Reply-To: obm-l@mat.puc-rio.br > > > > > > > > > > > > Tá, mas pq sqrt(5-x)=x se x=sqrt 5- sqrt(5-x)? > > > > Me diz mais uma coisa: Não daria pra resolver esta > > questão elevando os membros ao quadrado até tirar os > > radicais e depois achando as raízes do polinômio por > > girard? > > > > > É só substituir que vc pode observar que a igualdade vale. > Elevar ao quadrado requer muita força bruta, pode tentar, se > conseguir me avisa!!! > > Vc tem razão, elevei tudo ao quadrado, depois achei um polinômio de 4 grau. Apliquei girard e achei um sistema de 4 equações e 4 incógnitas.Esgotei minha força bruta como vc diz para isolar uma variavel e quando consegui, achei o mesmo polinômio de 4 grau nessa incógnita. > Mas eu ainda não entendi seu raciocínio. Substituir o q e onde??? se vc puder me mostrar seu resultado todo, ficarei grato..... ========================================================== > > =============== > > > Instruções para entrar na lista, sair da lista e usar a > > lista em > > > http://www.mat.puc-rio.br/~nicolau/olimp/obm-l.html > > > O administrador desta lista é > > > > > ========================================================== > > =============== > > > > > > > > > __________________________________________________________ ________________ > > Quer ter seu próprio endereço na Internet? > > Garanta já o seu e ainda ganhe cinco e-mails personalizados. > > DomíniosBOL - http://dominios.bol.com.br > > > > > > ========================================================== =============== > > Instruções para entrar na lista, sair da lista e usar a lista em > > http://www.mat.puc-rio.br/~nicolau/olimp/obm-l.html > > O administrador desta lista é > > ========================================================== =============== > > > > > ========================================================== =============== > Instruções para entrar na lista, sair da lista e usar a lista em > http://www.mat.puc-rio.br/~nicolau/olimp/obm-l.html > O administrador desta lista é > ========================================================== =============== > __________________________________________________________________________ Quer ter seu próprio endereço na Internet? Garanta já o seu e ainda ganhe cinco e-mails personalizados. DomíniosBOL - http://dominios.bol.com.br ========================================================================= Instruções para entrar na lista, sair da lista e usar a lista em http://www.mat.puc-rio.br/~nicolau/olimp/obm-l.html O administrador desta lista é ========================================================================= From owner-obm-l@sucuri.mat.puc-rio.br Fri May 17 00:13:52 2002 Return-Path: Received: (from majordom@localhost) by sucuri.mat.puc-rio.br (8.9.3/8.9.3) id AAA13460 for obm-l-list; Fri, 17 May 2002 00:13:46 -0300 Received: from hotmail.com (f20.pav1.hotmail.com [64.4.31.20]) by sucuri.mat.puc-rio.br (8.9.3/8.9.3) with ESMTP id AAA13456 for ; Fri, 17 May 2002 00:13:43 -0300 Received: from mail pickup service by hotmail.com with Microsoft SMTPSVC; Thu, 16 May 2002 20:11:57 -0700 Received: from 200.174.65.115 by pv1fd.pav1.hotmail.msn.com with HTTP; Fri, 17 May 2002 03:11:57 GMT X-Originating-IP: [200.174.65.115] From: "Felipe Marinho" To: obm-l@mat.puc-rio.br Subject: Re: [obm-l] Exercicios - Olimpiada. Date: Thu, 16 May 2002 23:11:57 -0400 Mime-Version: 1.0 Content-Type: text/plain; charset=iso-8859-1; format=flowed Message-ID: X-OriginalArrivalTime: 17 May 2002 03:11:57.0453 (UTC) FILETIME=[9A825BD0:01C1FD50] Sender: owner-obm-l@sucuri.mat.puc-rio.br Precedence: bulk Reply-To: obm-l@mat.puc-rio.br Caro Paulo, Mais uma vez aqui, tento expressar toda minha gratidão para com todos os amigos aqui da lista, e especialmente.. à você. Paulo, na verdade, a questão era para ser explicada a alunos de 1o. grau mesmo (8a. série). Bem, é o seguinte, sou aluno de uma instituição pública de ensino do Estado do Amazonas (curso agora o 2o. grau)... porem, venho desde o começo do ano ajudando os alunos da 8a. série para se prepararem para as provas de Matemática do Colégio Naval. Nossas condições dentro da escola não são muito boas, e em muitas das vezes... nosso próprio professor desconhece determinado assunto. Por este motivo, tenho aqui na lista, a única saída para o esclarecimento de certas dúvidas. Muitas vezes, eu mesmo, não consigo fazer certos exercícios que venho passando aos alunos... e quando isso ocorre, eu venho até a lista aqui, procurar alguma ajuda. E Paulo, mais uma vez, Obrigado mesmo por tudo. Sua ajuda, assim como a de todos, tem sido de grande importância para nossos alunos aqui do Amazonas. E em nome de todos eles, venho aqui deixar um grande abraço a você, e a todos os demais amigos que dessa lista participam. Obrigado mesmo, Felipe Marinho >From: "Paulo Santa Rita" >Reply-To: obm-l@mat.puc-rio.br >To: obm-l@mat.puc-rio.br >Subject: Re: [obm-l] Exercicios - Olimpiada. >Date: Thu, 16 May 2002 14:51:09 +0000 > >Ola Felipe, Guilherme e >demais colegas desta lista, > >Bom, isso deve ser um daqueles problemas que precisam ser explicados a >alunos do 1 grau, certo ? Dificil ! Vou tentar. Perdao se nao conseguir >atingir os objetivos ! > > >Devemos encontrar "a" e "b" inteiros nao-negativos. Como >(10a + b)*(a+b) = a^3 + b^3 >posso FATORAR o segundo membro assim : >a^3 + b^3 =(a+b)*(a^2- ab + b^2) >colocando essa fatoracao na primeira equacao, fica : >(10a + b)*(a+b) = (a+b)*(a^2 -ab + b^2) >como a + b > 0 (pois 00=0 nao e um numero de dois digitos ) posso dividir >tudo por a+b. Vai ficar : > >10a + b = a^2 - ab + b^2 > >reduzindo os termos semelhantes > >a^2 - (10 - b)*a + b^2 - b=0 > >E isto e UMA EQUACAO DO 2 GRAU LITERAL em "a". O Discriminante e : > >(10 - b)^2 - 4*(b^2 - b) > >Simplificando fica : -3b^2 - 16b + 100 = 0 > >Como b varia de 0 ate 9, posso verificar os casos em que o discriminante e >um quadrado perfeito e que implicam num "a" inteiro positivo. Isso vai me >fornecer a quantidade de numeros que satisfazem o problema. > >E ai Felipe ? Que tal ? Eu acho que so usei coisas que uma aluno serio do 1 >grau sabe ... ou nao ? Se errei, foi uma tentativa e a culpa exclusivamente >minha. Mas, se acertei, e porque sou um aluno aplicado do Jardim do Tio >Ralph. > >Um abracao >Paulo Santa Rita >5,1147,160502 > > > > > >>From: "Felipe Marinho" >>Reply-To: obm-l@mat.puc-rio.br >>To: obm-l@mat.puc-rio.br >>Subject: Re: [obm-l] Exercicios - Olimpiada. >>Date: Thu, 16 May 2002 04:06:43 -0400 >> >>Olá Guilherme, >> >>Obrigado pela sua ajuda. >> >>Resolvendo a questão do modo como você apresentou, a resposta seria 10 >>números (00,11,22,33,44,55,66,77,88,99). >>Porem, tal resposta não bate com a resposta do gabarito da prova. >> >>O enunciado da questão deve ser entendido como: >> >>(10a+b)(a+b) = a³+b³ (Multiplicacao do número formado por a e b pela >>soma >>dos dígitos) >> >>Na sua resolução você utilizou-se da multiplicação dos dígitos pela soma >>dos mesmos. [ a.b(a+b)=a³+b³ ]. >> >>As opções dadas pelo problema é: >>a) 0 >>b) 1 >>c) 2 >>d) 3 >>e) 4 >> >>Guilherme, e com isso, utilizando-se de seu raciocínio, não obteríamos a >>resposta do problema. >> >>Acho eu, então, que a solução é fazendo (10a+b)(a+b)=a³+b³. Porem, a >>resposta disto eu não consigo achar. Por isso vim até aqui, vocês, amigos >>da lista, para me dar uma ajuda... no que for possível, é claro. >> >>Agradeço desde já mais uma vez, >>Abraços >> >>Felipe Marinho >> >>>From: "Guilherme Pimentel" >>>Reply-To: obm-l@mat.puc-rio.br >>>To: >>>Subject: RES: [obm-l] Exercicios - Olimpiada. >>>Date: Thu, 16 May 2002 03:53:27 -0300 >>> >>>Para o primeiro note que, sendo ab o numero de dois digitos: >>>a*b*(a + b) = a^3 + b^3 >>>e que >>>a^3 + b^3 = (a + b)^3 - 3a*b*(a + b) >>>logo >>>a*b*(a + b) = (a + b)^3 - 3a*b*(a + b) >>>(a + b)^3 = 4a*b*(a + b) >>>supondo que a ou b sejam diferentes de zero: >>>(a + b)^2 = 4a*b >>>(a - b)^2 = 0 >>>ou seja >>>a = b >>>agora vc conta quantos são... >>> >>>-----Mensagem original----- >>>De: owner-obm-l@sucuri.mat.puc-rio.br >>>[mailto:owner-obm-l@sucuri.mat.puc-rio.br]Em nome de Felipe Marinho >>>Enviada em: quinta-feira, 16 de maio de 2002 02:37 >>>Para: obm-l@mat.puc-rio.br >>>Assunto: [obm-l] Exercicios - Olimpiada. >>> >>> >>>Olá pessoal da lista, >>> >>>Venho aqui pedir uma grande ajuda a vocês na resolução destes problemas. >>>Encontrei-os numa lista de preparação para Olimpíadas, porem, estes 2 eu >>>realmente não consegui resolvê-los. Por isso, conto com vocês mais uma >>>vez. >>> >>>1) Considere os números formados por 2 dígitos tais que a multiplicação >>>deles pela soma do seus dígitos seja igual a soma do cubo dos digitos. >>>Quantos e quais são esses números ? >>> >>>2) 40 bolas são numeradas de 1 a 40. Elas então são colocadas em caixas. >>>Se >>>uma caixa contem n bolas, então a caixa não poderá conter uma bola >>>numerada >>>com um múltiplo de n. No mínimo quantas caixas serão precisas para >>>guardar >>>as bolas, considerando todas as possibilidades possíveis ? >>> >>>Pessoal, agradeço desde já qualquer tipo de ajuda. >>> >>>E com um grande abraço a todos, >>>vou fechando mais este e-mail. >>> >>>Felipe Marinho >>> >>>_________________________________________________________________ >>>Envie e receba emails com o Hotmail no seu dispositivo móvel: >>>http://mobile.msn.com >>> >>>========================================================================= >>>Instruções para entrar na lista, sair da lista e usar a lista em >>>http://www.mat.puc-rio.br/~nicolau/olimp/obm-l.html >>>O administrador desta lista é >>>========================================================================= >>> >>>========================================================================= >>>Instruções para entrar na lista, sair da lista e usar a lista em >>>http://www.mat.puc-rio.br/~nicolau/olimp/obm-l.html >>>O administrador desta lista é >>>========================================================================= >> >> >>_________________________________________________________________ >>Converse com amigos on-line, conheça o MSN Messenger: >>http://messenger.msn.com >> >>========================================================================= >>Instruções para entrar na lista, sair da lista e usar a lista em >>http://www.mat.puc-rio.br/~nicolau/olimp/obm-l.html >>O administrador desta lista é >>========================================================================= > > > > >_________________________________________________________________ >Chegou o novo MSN Explorer. Instale já. É gratuito: >http://explorer.msn.com.br > >========================================================================= >Instruções para entrar na lista, sair da lista e usar a lista em >http://www.mat.puc-rio.br/~nicolau/olimp/obm-l.html >O administrador desta lista é >========================================================================= _________________________________________________________________ O MSN Photos é o modo mais fácil de compartilhar e imprimir suas fotos: http://photos.msn.com/support/worldwide.aspx ========================================================================= Instruções para entrar na lista, sair da lista e usar a lista em http://www.mat.puc-rio.br/~nicolau/olimp/obm-l.html O administrador desta lista é ========================================================================= From owner-obm-l@sucuri.mat.puc-rio.br Fri May 17 00:19:43 2002 Return-Path: Received: (from majordom@localhost) by sucuri.mat.puc-rio.br (8.9.3/8.9.3) id AAA13560 for obm-l-list; Fri, 17 May 2002 00:19:40 -0300 Received: from imo-r02.mx.aol.com (imo-r02.mx.aol.com [152.163.225.98]) by sucuri.mat.puc-rio.br (8.9.3/8.9.3) with ESMTP id AAA13551 for ; Fri, 17 May 2002 00:19:37 -0300 From: Euraul@aol.com Received: from Euraul@aol.com by imo-r02.mx.aol.com (mail_out_v32.5.) id z.34.27af5c1b (4155) for ; Thu, 16 May 2002 23:17:43 -0400 (EDT) Message-ID: <34.27af5c1b.2a15d057@aol.com> Date: Thu, 16 May 2002 23:17:43 EDT Subject: [obm-l] Espacial To: obm-l@mat.puc-rio.br MIME-Version: 1.0 Content-Type: multipart/alternative; boundary="part1_34.27af5c1b.2a15d057_boundary" X-Mailer: AOL 6.0 for Windows BR sub 10516 Sender: owner-obm-l@sucuri.mat.puc-rio.br Precedence: bulk Reply-To: obm-l@mat.puc-rio.br --part1_34.27af5c1b.2a15d057_boundary Content-Type: text/plain; charset="ISO-8859-1" Content-Transfer-Encoding: quoted-printable As arestas laterais de um paralelep=EDpedo, medidas em cm, s=E3o n= =FAmeros=20 =EDmpares consecutivos e a =E1rea lateral do mesmo =E9 de 142cm quadrados. Q= ual =E9 o=20 volume do paralelep=EDpedo ? Obrigado pela aten=E7=E3o, Raul --part1_34.27af5c1b.2a15d057_boundary Content-Type: text/html; charset="ISO-8859-1" Content-Transfer-Encoding: quoted-printable     = ; As arestas laterais de um paralelep=EDpedo, medidas em cm, s=E3o n= =FAmeros =EDmpares consecutivos e a =E1rea lateral do mesmo =E9 de 142cm qua= drados. Qual =E9 o volume do paralelep=EDpedo ?
      Obrigado pela aten=E7=E3o,
            = ;Raul
--part1_34.27af5c1b.2a15d057_boundary-- ========================================================================= Instruções para entrar na lista, sair da lista e usar a lista em http://www.mat.puc-rio.br/~nicolau/olimp/obm-l.html O administrador desta lista é ========================================================================= From owner-obm-l@sucuri.mat.puc-rio.br Fri May 17 00:19:44 2002 Return-Path: Received: (from majordom@localhost) by sucuri.mat.puc-rio.br (8.9.3/8.9.3) id AAA13575 for obm-l-list; Fri, 17 May 2002 00:19:42 -0300 Received: from imo-r02.mx.aol.com (imo-r02.mx.aol.com [152.163.225.98]) by sucuri.mat.puc-rio.br (8.9.3/8.9.3) with ESMTP id AAA13552 for ; Fri, 17 May 2002 00:19:37 -0300 From: Euraul@aol.com Received: from Euraul@aol.com by imo-r02.mx.aol.com (mail_out_v32.5.) id z.b0.27001c1b (4155) for ; Thu, 16 May 2002 23:17:42 -0400 (EDT) Message-ID: Date: Thu, 16 May 2002 23:17:41 EDT Subject: Re: RES: [obm-l] .......... To: obm-l@mat.puc-rio.br MIME-Version: 1.0 Content-Type: multipart/alternative; boundary="part1_b0.27001c1b.2a15d055_boundary" X-Mailer: AOL 6.0 for Windows BR sub 10516 Sender: owner-obm-l@sucuri.mat.puc-rio.br Precedence: bulk Reply-To: obm-l@mat.puc-rio.br --part1_b0.27001c1b.2a15d055_boundary Content-Type: text/plain; charset="ISO-8859-1" Content-Transfer-Encoding: quoted-printable Que tal essa estrat=E9gia ? Ser=E1 que compliquei muito ? A equa=E7=E3o =E9 x=3Dsqrt(5-sqrt(5-x)) ; se x vale sqrt(5-sqrt(5-x)),= podemos=20 substituir tendo x =3D sqrt(5-sqrt(5-sqrt(5-sqrt(5-x)))). Se fizermos isso=20 infinitas vezes, teremos um problema cl=E1ssico que resumimos para x =3D=20 sqrt(5-x), isto =E9, x^2 =3D 5 - x. Sendo a resposta a raiz positiva :=20 (sqrt(21)-1)/2. Um abra=E7o, Raul --part1_b0.27001c1b.2a15d055_boundary Content-Type: text/html; charset="ISO-8859-1" Content-Transfer-Encoding: quoted-printable     = ; Que tal essa estrat=E9gia ? Ser=E1 que compliquei muito ?
     A equa=E7=E3o =E9 x=3Dsqrt(5-sqrt(5-x)) ;= se x vale sqrt(5-sqrt(5-x)), podemos substituir tendo x =3D sqrt(5-sqrt(5-s= qrt(5-sqrt(5-x)))). Se fizermos isso infinitas vezes, teremos um problema cl= =E1ssico que resumimos para x =3D sqrt(5-x), isto =E9, x^2 =3D 5 - x. Sendo=20= a resposta a raiz positiva : (sqrt(21)-1)/2.
      Um abra=E7o,
            = ;Raul
--part1_b0.27001c1b.2a15d055_boundary-- ========================================================================= Instruções para entrar na lista, sair da lista e usar a lista em http://www.mat.puc-rio.br/~nicolau/olimp/obm-l.html O administrador desta lista é ========================================================================= From owner-obm-l@sucuri.mat.puc-rio.br Fri May 17 01:18:31 2002 Return-Path: Received: (from majordom@localhost) by sucuri.mat.puc-rio.br (8.9.3/8.9.3) id BAA14996 for obm-l-list; Fri, 17 May 2002 01:18:25 -0300 Received: from traven.uol.com.br (traven.uol.com.br [200.231.206.184]) by sucuri.mat.puc-rio.br (8.9.3/8.9.3) with ESMTP id BAA14992 for ; Fri, 17 May 2002 01:18:23 -0300 Received: from afonso ([200.191.190.129]) by traven.uol.com.br (8.9.1/8.9.1) with SMTP id BAA10288 for ; Fri, 17 May 2002 01:05:35 -0300 (BRT) From: "Guilherme Pimentel" To: Subject: RES: [obm-l] Exercicios - Olimpiada. Date: Fri, 17 May 2002 01:20:11 -0300 Message-ID: MIME-Version: 1.0 Content-Type: text/plain; charset="iso-8859-1" Content-Transfer-Encoding: 8bit X-Priority: 3 (Normal) X-MSMail-Priority: Normal X-Mailer: Microsoft Outlook IMO, Build 9.0.2416 (9.0.2910.0) X-MimeOLE: Produced By Microsoft MimeOLE V5.00.2615.200 Importance: Normal In-Reply-To: Sender: owner-obm-l@sucuri.mat.puc-rio.br Precedence: bulk Reply-To: obm-l@mat.puc-rio.br Bom, seu enunciado tem uma ambiguidade de escopo, "a multiplicação deles" poderia ser tanto os digitos (que foi minha interpretação, pela proximidade) ou do numero inteiro que foi a sua. Mas de madrugada nem sempre da para fiacar analisando isso. Valeu... -----Mensagem original----- De: owner-obm-l@sucuri.mat.puc-rio.br [mailto:owner-obm-l@sucuri.mat.puc-rio.br]Em nome de Felipe Marinho Enviada em: quinta-feira, 16 de maio de 2002 05:07 Para: obm-l@mat.puc-rio.br Assunto: Re: [obm-l] Exercicios - Olimpiada. Olá Guilherme, Obrigado pela sua ajuda. Resolvendo a questão do modo como você apresentou, a resposta seria 10 números (00,11,22,33,44,55,66,77,88,99). Porem, tal resposta não bate com a resposta do gabarito da prova. O enunciado da questão deve ser entendido como: (10a+b)(a+b) = a³+b³ (Multiplicacao do número formado por a e b pela soma dos dígitos) Na sua resolução você utilizou-se da multiplicação dos dígitos pela soma dos mesmos. [ a.b(a+b)=a³+b³ ]. As opções dadas pelo problema é: a) 0 b) 1 c) 2 d) 3 e) 4 Guilherme, e com isso, utilizando-se de seu raciocínio, não obteríamos a resposta do problema. Acho eu, então, que a solução é fazendo (10a+b)(a+b)=a³+b³. Porem, a resposta disto eu não consigo achar. Por isso vim até aqui, vocês, amigos da lista, para me dar uma ajuda... no que for possível, é claro. Agradeço desde já mais uma vez, Abraços Felipe Marinho >From: "Guilherme Pimentel" >Reply-To: obm-l@mat.puc-rio.br >To: >Subject: RES: [obm-l] Exercicios - Olimpiada. >Date: Thu, 16 May 2002 03:53:27 -0300 > >Para o primeiro note que, sendo ab o numero de dois digitos: >a*b*(a + b) = a^3 + b^3 >e que >a^3 + b^3 = (a + b)^3 - 3a*b*(a + b) >logo >a*b*(a + b) = (a + b)^3 - 3a*b*(a + b) >(a + b)^3 = 4a*b*(a + b) >supondo que a ou b sejam diferentes de zero: >(a + b)^2 = 4a*b >(a - b)^2 = 0 >ou seja >a = b >agora vc conta quantos são... > >-----Mensagem original----- >De: owner-obm-l@sucuri.mat.puc-rio.br >[mailto:owner-obm-l@sucuri.mat.puc-rio.br]Em nome de Felipe Marinho >Enviada em: quinta-feira, 16 de maio de 2002 02:37 >Para: obm-l@mat.puc-rio.br >Assunto: [obm-l] Exercicios - Olimpiada. > > >Olá pessoal da lista, > >Venho aqui pedir uma grande ajuda a vocês na resolução destes problemas. >Encontrei-os numa lista de preparação para Olimpíadas, porem, estes 2 eu >realmente não consegui resolvê-los. Por isso, conto com vocês mais uma vez. > >1) Considere os números formados por 2 dígitos tais que a multiplicação >deles pela soma do seus dígitos seja igual a soma do cubo dos digitos. >Quantos e quais são esses números ? > >2) 40 bolas são numeradas de 1 a 40. Elas então são colocadas em caixas. Se >uma caixa contem n bolas, então a caixa não poderá conter uma bola numerada >com um múltiplo de n. No mínimo quantas caixas serão precisas para guardar >as bolas, considerando todas as possibilidades possíveis ? > >Pessoal, agradeço desde já qualquer tipo de ajuda. > >E com um grande abraço a todos, >vou fechando mais este e-mail. > >Felipe Marinho > >_________________________________________________________________ >Envie e receba emails com o Hotmail no seu dispositivo móvel: >http://mobile.msn.com > >========================================================================= >Instruções para entrar na lista, sair da lista e usar a lista em >http://www.mat.puc-rio.br/~nicolau/olimp/obm-l.html >O administrador desta lista é >========================================================================= > >========================================================================= >Instruções para entrar na lista, sair da lista e usar a lista em >http://www.mat.puc-rio.br/~nicolau/olimp/obm-l.html >O administrador desta lista é >========================================================================= _________________________________________________________________ Converse com amigos on-line, conheça o MSN Messenger: http://messenger.msn.com ========================================================================= Instruções para entrar na lista, sair da lista e usar a lista em http://www.mat.puc-rio.br/~nicolau/olimp/obm-l.html O administrador desta lista é ========================================================================= ========================================================================= Instruções para entrar na lista, sair da lista e usar a lista em http://www.mat.puc-rio.br/~nicolau/olimp/obm-l.html O administrador desta lista é ========================================================================= From owner-obm-l@sucuri.mat.puc-rio.br Fri May 17 02:34:17 2002 Return-Path: Received: (from majordom@localhost) by sucuri.mat.puc-rio.br (8.9.3/8.9.3) id CAA15748 for obm-l-list; Fri, 17 May 2002 02:33:57 -0300 Received: from web21303.mail.yahoo.com (web21303.mail.yahoo.com [216.136.129.192]) by sucuri.mat.puc-rio.br (8.9.3/8.9.3) with SMTP id CAA15744 for ; Fri, 17 May 2002 02:33:54 -0300 Message-ID: <20020517053208.91596.qmail@web21303.mail.yahoo.com> Received: from [200.227.209.79] by web21303.mail.yahoo.com via HTTP; Fri, 17 May 2002 02:32:08 ART Date: Fri, 17 May 2002 02:32:08 -0300 (ART) From: =?iso-8859-1?q?Marcos=20Reynaldo?= Subject: Re: [obm-l] Espacial To: obm-l@mat.puc-rio.br In-Reply-To: <34.27af5c1b.2a15d057@aol.com> MIME-Version: 1.0 Content-Type: text/plain; charset=iso-8859-1 Content-Transfer-Encoding: 8bit Sender: owner-obm-l@sucuri.mat.puc-rio.br Precedence: bulk Reply-To: obm-l@mat.puc-rio.br Olá Raul! Esse enunciado esta correto ? Se as arestas laterais do paralelepipedo são numeros impares consecutivos então não da um paralelogramo. Euraul@aol.com escreveu: As arestas laterais de um paralelepípedo, medidas em cm, são números ímpares consecutivos e a área lateral do mesmo é de 142cm quadrados. Qual é o volume do paralelepípedo ? Obrigado pela atenção, Raul _______________________________________________________________________ Yahoo! Encontros O lugar certo para você encontrar aquela pessoa que falta na sua vida. Cadastre-se hoje mesmo! http://br.encontros.yahoo.com/ ========================================================================= Instruções para entrar na lista, sair da lista e usar a lista em http://www.mat.puc-rio.br/~nicolau/olimp/obm-l.html O administrador desta lista é ========================================================================= From owner-obm-l@sucuri.mat.puc-rio.br Fri May 17 03:10:44 2002 Return-Path: Received: (from majordom@localhost) by sucuri.mat.puc-rio.br (8.9.3/8.9.3) id DAA16253 for obm-l-list; Fri, 17 May 2002 03:10:30 -0300 Received: from home.iis.com.br (mail.iis.com.br [200.202.96.2]) by sucuri.mat.puc-rio.br (8.9.3/8.9.3) with ESMTP id DAA16249 for ; Fri, 17 May 2002 03:10:28 -0300 Received: from Marcio (rio-tc0-tty135.iis.com.br [200.202.98.135]) by home.iis.com.br (8.11.6/8.11.6/1.1.1.16) with SMTP id g4H68g225758 for ; Fri, 17 May 2002 03:08:42 -0300 Message-ID: <000d01c1fd69$52a62c60$8762cac8@epq.ime.eb.br> From: "Marcio" To: References: <20020516204209.7102.qmail@web10103.mail.yahoo.com> Subject: [obm-l] =?iso-8859-1?Q?Re:_=5Bobm-l=5D_tri=E2ngulo?= Date: Fri, 17 May 2002 03:08:52 -0300 MIME-Version: 1.0 Content-Type: text/plain; charset="iso-8859-1" Content-Transfer-Encoding: 7bit X-Priority: 3 X-MSMail-Priority: Normal X-Mailer: Microsoft Outlook Express 5.50.4133.2400 X-MimeOLE: Produced By Microsoft MimeOLE V5.50.4133.2400 X-AntiVirus: Antivirus for sendmail by Petr Rehor Sender: owner-obm-l@sucuri.mat.puc-rio.br Precedence: bulk Reply-To: obm-l@mat.puc-rio.br Aproveitando a sua figura: Eh fato conhecido que AF = AH = semiperimetro - CB por exemplo (pois CG+CF+AF+AH+HB+BG=2(CG+BG+AH)=perimetro, e BG+CG=BC). No seu problema, AF = AH = 3. Pondo EF=EI=x e ID=DH=y, basta notar que os triangulos AED e ACB sao semelhante para escrever (3-x)/5 = (3-y)/8 = (x+y)/7 .... = 6/20=3/10 donde x = 3/2; y = 3/5 e ID/DE=2/5. t+ ========================================================================= Instruções para entrar na lista, sair da lista e usar a lista em http://www.mat.puc-rio.br/~nicolau/olimp/obm-l.html O administrador desta lista é ========================================================================= From owner-obm-l@sucuri.mat.puc-rio.br Fri May 17 03:22:58 2002 Return-Path: Received: (from majordom@localhost) by sucuri.mat.puc-rio.br (8.9.3/8.9.3) id DAA16543 for obm-l-list; Fri, 17 May 2002 03:22:56 -0300 Received: from home.iis.com.br (mail.iis.com.br [200.202.96.2]) by sucuri.mat.puc-rio.br (8.9.3/8.9.3) with ESMTP id DAA16538 for ; Fri, 17 May 2002 03:22:53 -0300 Received: from Marcio (rio-tc0-tty135.iis.com.br [200.202.98.135]) by home.iis.com.br (8.11.6/8.11.6/1.1.1.16) with SMTP id g4H6L8226936 for ; Fri, 17 May 2002 03:21:08 -0300 Message-ID: <003501c1fd6b$0f2b2ba0$8762cac8@epq.ime.eb.br> From: "Marcio" To: References: Subject: Re: RES: [obm-l] .......... Date: Fri, 17 May 2002 03:21:18 -0300 MIME-Version: 1.0 Content-Type: multipart/alternative; boundary="----=_NextPart_000_0032_01C1FD51.E903C040" X-Priority: 3 X-MSMail-Priority: Normal X-Mailer: Microsoft Outlook Express 5.50.4133.2400 X-MimeOLE: Produced By Microsoft MimeOLE V5.50.4133.2400 X-AntiVirus: Antivirus for sendmail by Petr Rehor Sender: owner-obm-l@sucuri.mat.puc-rio.br Precedence: bulk Reply-To: obm-l@mat.puc-rio.br This is a multi-part message in MIME format. ------=_NextPart_000_0032_01C1FD51.E903C040 Content-Type: text/plain; charset="iso-8859-1" Content-Transfer-Encoding: quoted-printable Do jeito que esta escrito nao me parece certo. Uma coisa eh encontrar o limite da sequencia definida por A_0 =3D x, = A_n+1 =3D sqrt(5-A_n). Esse talvez seja o problema facil (tanto a = existencia do limite qto o seu calculo) ao qual vc se refere. No caso da sua estrategia eh um pouco diferente... agora B_0 =3D x mas = B_n+1 =3D sqrt[5-sqrt(5-B_n)]. Vc consegue provar que essa sequencia tem = limite e q limB_n =3D limA_n, mas isso eh praticamente o proprio = problema da prova..=20 Marcio ----- Original Message -----=20 From: Euraul@aol.com=20 To: obm-l@mat.puc-rio.br=20 Sent: Friday, May 17, 2002 12:17 AM Subject: Re: RES: [obm-l] .......... Que tal essa estrat=E9gia ? Ser=E1 que compliquei muito ?=20 A equa=E7=E3o =E9 x=3Dsqrt(5-sqrt(5-x)) ; se x vale = sqrt(5-sqrt(5-x)), podemos substituir tendo x =3D = sqrt(5-sqrt(5-sqrt(5-sqrt(5-x)))). Se fizermos isso infinitas vezes, = teremos um problema cl=E1ssico que resumimos para x =3D sqrt(5-x), isto = =E9, x^2 =3D 5 - x. Sendo a resposta a raiz positiva : (sqrt(21)-1)/2.=20 Um abra=E7o,=20 Raul=20 ------=_NextPart_000_0032_01C1FD51.E903C040 Content-Type: text/html; charset="iso-8859-1" Content-Transfer-Encoding: quoted-printable
Do jeito que esta escrito nao me=20 parece certo.
  Uma coisa eh encontrar o limite = da sequencia=20 definida por A_0 =3D x, A_n+1 =3D sqrt(5-A_n). Esse talvez seja o = problema=20 facil (tanto a existencia do limite qto o seu calculo) ao qual vc se=20 refere.
  No caso da sua estrategia eh um = pouco=20 diferente... agora B_0 =3D x mas B_n+1 =3D sqrt[5-sqrt(5-B_n)]. Vc = consegue provar=20 que essa sequencia tem limite e q limB_n =3D limA_n, mas isso eh = praticamente o=20 proprio problema da prova..
  Marcio
 
----- Original Message -----
From:=20 Euraul@aol.com
Sent: Friday, May 17, 2002 = 12:17 AM
Subject: Re: RES: [obm-l]=20 ..........

     Que tal essa estrat=E9gia ? = Ser=E1 que=20 compliquei muito ?
     A equa=E7=E3o =E9 = x=3Dsqrt(5-sqrt(5-x)) ; se x vale sqrt(5-sqrt(5-x)), podemos = substituir tendo x=20 =3D sqrt(5-sqrt(5-sqrt(5-sqrt(5-x)))). Se fizermos isso infinitas = vezes, teremos=20 um problema cl=E1ssico que resumimos para x =3D sqrt(5-x), isto =E9, = x^2 =3D 5 - x.=20 Sendo a resposta a raiz positiva : (sqrt(21)-1)/2.=20
      Um abra=E7o,=20 =
           &nb= sp;Raul=20
------=_NextPart_000_0032_01C1FD51.E903C040-- ========================================================================= Instruções para entrar na lista, sair da lista e usar a lista em http://www.mat.puc-rio.br/~nicolau/olimp/obm-l.html O administrador desta lista é ========================================================================= From owner-obm-l@sucuri.mat.puc-rio.br Fri May 17 08:52:16 2002 Return-Path: Received: (from majordom@localhost) by sucuri.mat.puc-rio.br (8.9.3/8.9.3) id IAA19058 for obm-l-list; Fri, 17 May 2002 08:52:02 -0300 Received: from ns-3.idc.dglnet.com.br (ns-3.idc.dglnet.com.br [200.218.161.4]) by sucuri.mat.puc-rio.br (8.9.3/8.9.3) with ESMTP id IAA19052 for ; Fri, 17 May 2002 08:52:00 -0300 Received: from user (200-204-34-35.dial-up.telesp.net.br [200.204.34.35]) by ns-3.idc.dglnet.com.br (Postfix) with SMTP id 33AFC150009 for ; Fri, 17 May 2002 08:50:15 -0300 (BRT) Message-ID: <003a01c1fd99$0a6e29e0$2322ccc8@user> From: "Claudio" To: References: Subject: Re: RES: [obm-l] .......... Date: Fri, 17 May 2002 08:50:27 -0300 MIME-Version: 1.0 Content-Type: multipart/alternative; boundary="----=_NextPart_000_0037_01C1FD7F.E4791920" X-Priority: 3 X-MSMail-Priority: Normal X-Mailer: Microsoft Outlook Express 5.00.2314.1300 X-MimeOLE: Produced By Microsoft MimeOLE V5.00.2314.1300 Sender: owner-obm-l@sucuri.mat.puc-rio.br Precedence: bulk Reply-To: obm-l@mat.puc-rio.br This is a multi-part message in MIME format. ------=_NextPart_000_0037_01C1FD7F.E4791920 Content-Type: text/plain; charset="iso-8859-1" Content-Transfer-Encoding: quoted-printable H=E1 um problema s=E9rio aqui. Esse argumento mostra que se x =E9 um ponto fixo da fun=E7=E3o F, isto = =E9 F(x)=3Dx, ent=E3o esse mesmo x =E9 um ponto fixo de FoFoFo.. ..oF(x). O contr=E1rio n=E3o =E9 verdadeiro. Exemplo- seja F :R++ = ----------->R++ definida por F(x)=3D 1 / x. O =FAnico ponto fixo ocorre = em x=3D1. Agora FoF (x) =3D x para todo x em R++. O problema em quest=E3o pede o ponto fixo da segunda iterada, da=ED o = problema. Um abra=E7o. Claudio Casemiro. ----- Original Message -----=20 From: Euraul@aol.com=20 To: obm-l@mat.puc-rio.br=20 Sent: Friday, May 17, 2002 12:17 AM Subject: Re: RES: [obm-l] .......... Que tal essa estrat=E9gia ? Ser=E1 que compliquei muito ?=20 A equa=E7=E3o =E9 x=3Dsqrt(5-sqrt(5-x)) ; se x vale = sqrt(5-sqrt(5-x)), podemos substituir tendo x =3D = sqrt(5-sqrt(5-sqrt(5-sqrt(5-x)))). Se fizermos isso infinitas vezes, = teremos um problema cl=E1ssico que resumimos para x =3D sqrt(5-x), isto = =E9, x^2 =3D 5 - x. Sendo a resposta a raiz positiva : (sqrt(21)-1)/2.=20 Um abra=E7o,=20 Raul=20 ------=_NextPart_000_0037_01C1FD7F.E4791920 Content-Type: text/html; charset="iso-8859-1" Content-Transfer-Encoding: quoted-printable
H=E1 um problema s=E9rio = aqui.
Esse argumento mostra que se x =E9 um = ponto fixo da=20 fun=E7=E3o F, isto =E9 F(x)=3Dx, ent=E3o esse mesmo x =E9 um ponto fixo = de=20 FoFoFo..
..oF(x). O contr=E1rio n=E3o =E9 = verdadeiro. Exemplo-=20 seja F :R++ ----------->R++ definida por F(x)=3D 1 / x. O =FAnico = ponto fixo=20 ocorre em
x=3D1. Agora FoF (x) =3D x para todo x = em=20 R++.
O problema em quest=E3o pede o ponto = fixo da segunda=20 iterada, da=ED o problema.
Um abra=E7o.
Claudio Casemiro.
----- Original Message -----
From:=20 Euraul@aol.com
Sent: Friday, May 17, 2002 = 12:17 AM
Subject: Re: RES: [obm-l]=20 ..........

     Que tal essa estrat=E9gia ? = Ser=E1 que=20 compliquei muito ?
     A equa=E7=E3o =E9 = x=3Dsqrt(5-sqrt(5-x)) ; se x vale sqrt(5-sqrt(5-x)), podemos = substituir tendo x=20 =3D sqrt(5-sqrt(5-sqrt(5-sqrt(5-x)))). Se fizermos isso infinitas = vezes, teremos=20 um problema cl=E1ssico que resumimos para x =3D sqrt(5-x), isto =E9, = x^2 =3D 5 - x.=20 Sendo a resposta a raiz positiva : (sqrt(21)-1)/2.=20
      Um abra=E7o,=20 =
           &nb= sp;Raul=20
------=_NextPart_000_0037_01C1FD7F.E4791920-- ========================================================================= Instruções para entrar na lista, sair da lista e usar a lista em http://www.mat.puc-rio.br/~nicolau/olimp/obm-l.html O administrador desta lista é ========================================================================= From owner-obm-l@sucuri.mat.puc-rio.br Fri May 17 09:27:54 2002 Return-Path: Received: (from majordom@localhost) by sucuri.mat.puc-rio.br (8.9.3/8.9.3) id JAA19803 for obm-l-list; Fri, 17 May 2002 09:27:45 -0300 Received: from hotmail.com (f118.pav1.hotmail.com [64.4.31.118]) by sucuri.mat.puc-rio.br (8.9.3/8.9.3) with ESMTP id JAA19799 for ; Fri, 17 May 2002 09:27:43 -0300 Received: from mail pickup service by hotmail.com with Microsoft SMTPSVC; Fri, 17 May 2002 05:25:58 -0700 Received: from 200.151.157.235 by pv1fd.pav1.hotmail.msn.com with HTTP; Fri, 17 May 2002 12:25:57 GMT X-Originating-IP: [200.151.157.235] From: "Adherbal Rocha Filho" To: obm-l@mat.puc-rio.br Subject: [obm-l] a. basica Date: Fri, 17 May 2002 12:25:57 +0000 Mime-Version: 1.0 Content-Type: text/plain; charset=iso-8859-1; format=flowed Message-ID: X-OriginalArrivalTime: 17 May 2002 12:25:58.0142 (UTC) FILETIME=[FF80F9E0:01C1FD9D] Sender: owner-obm-l@sucuri.mat.puc-rio.br Precedence: bulk Reply-To: obm-l@mat.puc-rio.br Ae, olha este problema: Seja P(x,y)=5x^2 -6xy +2y^2. a)determine qnts elementos de {1,2...,100} são valores de P. b)Prove q o produto de valores de P é um valor de P. Será que alguém pode me dar uma ajuda? Valeu! []´s Adherbal _________________________________________________________________ Envie e receba emails com o Hotmail no seu dispositivo móvel: http://mobile.msn.com ========================================================================= Instruções para entrar na lista, sair da lista e usar a lista em http://www.mat.puc-rio.br/~nicolau/olimp/obm-l.html O administrador desta lista é ========================================================================= From owner-obm-l@sucuri.mat.puc-rio.br Fri May 17 13:03:00 2002 Return-Path: Received: (from majordom@localhost) by sucuri.mat.puc-rio.br (8.9.3/8.9.3) id NAA23165 for obm-l-list; Fri, 17 May 2002 13:01:21 -0300 Received: from hotmail.com (law2-f61.hotmail.com [216.32.181.61]) by sucuri.mat.puc-rio.br (8.9.3/8.9.3) with ESMTP id NAA23161 for ; Fri, 17 May 2002 13:01:17 -0300 Received: from mail pickup service by hotmail.com with Microsoft SMTPSVC; Fri, 17 May 2002 08:59:33 -0700 Received: from 32.94.119.254 by lw2fd.hotmail.msn.com with HTTP; Fri, 17 May 2002 15:59:33 GMT X-Originating-IP: [32.94.119.254] From: "Paulo Santa Rita" To: obm-l@mat.puc-rio.br Subject: Re: [obm-l] Exercicios - Olimpiada. Date: Fri, 17 May 2002 15:59:33 +0000 Mime-Version: 1.0 Content-Type: text/plain; charset=iso-8859-1; format=flowed Message-ID: X-OriginalArrivalTime: 17 May 2002 15:59:33.0617 (UTC) FILETIME=[D61F4610:01C1FDBB] Sender: owner-obm-l@sucuri.mat.puc-rio.br Precedence: bulk Reply-To: obm-l@mat.puc-rio.br Ola Felipe, tudo legal ? Nao ha do que agradecer ! Muitas outras pessoas aqui na lista tambem tem SENTIMENTOS EDUCADOS, de forma que nao raro voce as vera sendo uteis respondendo a uma questao simples com a mesma naturalidade com que demonstram brilhantismo intelectual solucionando um problema complicado. Eu li em algum lugar um problema que pode ser resolvido por uma equacao do 2 grau. Talvez voce queira propor a seus alunos de 8 serie : Num tonel ha, a principio, 100 litros de agua. Enche-se um balde de capacidade K ( K litros ) com a agua deste tonel. A agua e jogada fora. O balde e entao cheio com vinho puro e, a seguir, o vinho contido no balde e despejado no tonel. O tonel volta a ter 100 litros, agora de mistura. Repete-se os 4 passos abaixo N vezes : 1) Enche-se o balde com a mistura que ha no tonel 2) O conteudo do balde e, entao, lancado fora. 3) O Balde e cheio com vinho puro 4) O conteudo do balde e despejado no tonel. Terminada as operacoes, um quimico analisa o conteudo do tonel e descobre que ha 36 litros de agua e 64 litros de vinho puro. Qual a capacidade do balde, em funcao de N ? Com os melhores votos de paz profunda, sou Paulo Santa Rita 6,1256,170502 >From: "Felipe Marinho" >Reply-To: obm-l@mat.puc-rio.br >To: obm-l@mat.puc-rio.br >Subject: Re: [obm-l] Exercicios - Olimpiada. >Date: Thu, 16 May 2002 23:11:57 -0400 > >Caro Paulo, > >Mais uma vez aqui, tento expressar toda minha gratidão para com todos os >amigos aqui da lista, e especialmente.. à você. > >Paulo, na verdade, a questão era para ser explicada a alunos de 1o. grau >mesmo (8a. série). Bem, é o seguinte, sou aluno de uma instituição pública >de ensino do Estado do Amazonas (curso agora o 2o. grau)... porem, venho >desde o começo do ano ajudando os alunos da 8a. série para se prepararem >para as provas de Matemática do Colégio Naval. Nossas condições dentro da >escola não são muito boas, e em muitas das vezes... nosso próprio professor >desconhece determinado assunto. Por este motivo, tenho aqui na lista, a >única saída para o esclarecimento de certas dúvidas. Muitas vezes, eu >mesmo, >não consigo fazer certos exercícios que venho passando aos alunos... e >quando isso ocorre, eu venho até a lista aqui, procurar alguma ajuda. > >E Paulo, mais uma vez, Obrigado mesmo por tudo. Sua ajuda, assim como a de >todos, tem sido de grande importância para nossos alunos aqui do Amazonas. >E >em nome de todos eles, venho aqui deixar um grande abraço a você, e a todos >os demais amigos que dessa lista participam. > >Obrigado mesmo, > >Felipe Marinho > >>From: "Paulo Santa Rita" >>Reply-To: obm-l@mat.puc-rio.br >>To: obm-l@mat.puc-rio.br >>Subject: Re: [obm-l] Exercicios - Olimpiada. >>Date: Thu, 16 May 2002 14:51:09 +0000 >> >>Ola Felipe, Guilherme e >>demais colegas desta lista, >> >>Bom, isso deve ser um daqueles problemas que precisam ser explicados a >>alunos do 1 grau, certo ? Dificil ! Vou tentar. Perdao se nao conseguir >>atingir os objetivos ! >> >> >>Devemos encontrar "a" e "b" inteiros nao-negativos. Como >>(10a + b)*(a+b) = a^3 + b^3 >>posso FATORAR o segundo membro assim : >>a^3 + b^3 =(a+b)*(a^2- ab + b^2) >>colocando essa fatoracao na primeira equacao, fica : >>(10a + b)*(a+b) = (a+b)*(a^2 -ab + b^2) >>como a + b > 0 (pois 00=0 nao e um numero de dois digitos ) posso dividir >>tudo por a+b. Vai ficar : >> >>10a + b = a^2 - ab + b^2 >> >>reduzindo os termos semelhantes >> >>a^2 - (10 - b)*a + b^2 - b=0 >> >>E isto e UMA EQUACAO DO 2 GRAU LITERAL em "a". O Discriminante e : >> >>(10 - b)^2 - 4*(b^2 - b) >> >>Simplificando fica : -3b^2 - 16b + 100 = 0 >> >>Como b varia de 0 ate 9, posso verificar os casos em que o discriminante e >>um quadrado perfeito e que implicam num "a" inteiro positivo. Isso vai me >>fornecer a quantidade de numeros que satisfazem o problema. >> >>E ai Felipe ? Que tal ? Eu acho que so usei coisas que uma aluno serio do >>1 >>grau sabe ... ou nao ? Se errei, foi uma tentativa e a culpa >>exclusivamente >>minha. Mas, se acertei, e porque sou um aluno aplicado do Jardim do Tio >>Ralph. >> >>Um abracao >>Paulo Santa Rita >>5,1147,160502 >> >> >> >> >> >>>From: "Felipe Marinho" >>>Reply-To: obm-l@mat.puc-rio.br >>>To: obm-l@mat.puc-rio.br >>>Subject: Re: [obm-l] Exercicios - Olimpiada. >>>Date: Thu, 16 May 2002 04:06:43 -0400 >>> >>>Olá Guilherme, >>> >>>Obrigado pela sua ajuda. >>> >>>Resolvendo a questão do modo como você apresentou, a resposta seria 10 >>>números (00,11,22,33,44,55,66,77,88,99). >>>Porem, tal resposta não bate com a resposta do gabarito da prova. >>> >>>O enunciado da questão deve ser entendido como: >>> >>>(10a+b)(a+b) = a³+b³ (Multiplicacao do número formado por a e b pela >>>soma >>>dos dígitos) >>> >>>Na sua resolução você utilizou-se da multiplicação dos dígitos pela soma >>>dos mesmos. [ a.b(a+b)=a³+b³ ]. >>> >>>As opções dadas pelo problema é: >>>a) 0 >>>b) 1 >>>c) 2 >>>d) 3 >>>e) 4 >>> >>>Guilherme, e com isso, utilizando-se de seu raciocínio, não obteríamos a >>>resposta do problema. >>> >>>Acho eu, então, que a solução é fazendo (10a+b)(a+b)=a³+b³. Porem, a >>>resposta disto eu não consigo achar. Por isso vim até aqui, vocês, amigos >>>da lista, para me dar uma ajuda... no que for possível, é claro. >>> >>>Agradeço desde já mais uma vez, >>>Abraços >>> >>>Felipe Marinho >>> >>>>From: "Guilherme Pimentel" >>>>Reply-To: obm-l@mat.puc-rio.br >>>>To: >>>>Subject: RES: [obm-l] Exercicios - Olimpiada. >>>>Date: Thu, 16 May 2002 03:53:27 -0300 >>>> >>>>Para o primeiro note que, sendo ab o numero de dois digitos: >>>>a*b*(a + b) = a^3 + b^3 >>>>e que >>>>a^3 + b^3 = (a + b)^3 - 3a*b*(a + b) >>>>logo >>>>a*b*(a + b) = (a + b)^3 - 3a*b*(a + b) >>>>(a + b)^3 = 4a*b*(a + b) >>>>supondo que a ou b sejam diferentes de zero: >>>>(a + b)^2 = 4a*b >>>>(a - b)^2 = 0 >>>>ou seja >>>>a = b >>>>agora vc conta quantos são... >>>> >>>>-----Mensagem original----- >>>>De: owner-obm-l@sucuri.mat.puc-rio.br >>>>[mailto:owner-obm-l@sucuri.mat.puc-rio.br]Em nome de Felipe Marinho >>>>Enviada em: quinta-feira, 16 de maio de 2002 02:37 >>>>Para: obm-l@mat.puc-rio.br >>>>Assunto: [obm-l] Exercicios - Olimpiada. >>>> >>>> >>>>Olá pessoal da lista, >>>> >>>>Venho aqui pedir uma grande ajuda a vocês na resolução destes problemas. >>>>Encontrei-os numa lista de preparação para Olimpíadas, porem, estes 2 eu >>>>realmente não consegui resolvê-los. Por isso, conto com vocês mais uma >>>>vez. >>>> >>>>1) Considere os números formados por 2 dígitos tais que a multiplicação >>>>deles pela soma do seus dígitos seja igual a soma do cubo dos digitos. >>>>Quantos e quais são esses números ? >>>> >>>>2) 40 bolas são numeradas de 1 a 40. Elas então são colocadas em caixas. >>>>Se >>>>uma caixa contem n bolas, então a caixa não poderá conter uma bola >>>>numerada >>>>com um múltiplo de n. No mínimo quantas caixas serão precisas para >>>>guardar >>>>as bolas, considerando todas as possibilidades possíveis ? >>>> >>>>Pessoal, agradeço desde já qualquer tipo de ajuda. >>>> >>>>E com um grande abraço a todos, >>>>vou fechando mais este e-mail. >>>> >>>>Felipe Marinho >>>> >>>>_________________________________________________________________ >>>>Envie e receba emails com o Hotmail no seu dispositivo móvel: >>>>http://mobile.msn.com >>>> >>>>========================================================================= >>>>Instruções para entrar na lista, sair da lista e usar a lista em >>>>http://www.mat.puc-rio.br/~nicolau/olimp/obm-l.html >>>>O administrador desta lista é >>>>========================================================================= >>>> >>>>========================================================================= >>>>Instruções para entrar na lista, sair da lista e usar a lista em >>>>http://www.mat.puc-rio.br/~nicolau/olimp/obm-l.html >>>>O administrador desta lista é >>>>========================================================================= >>> >>> >>>_________________________________________________________________ >>>Converse com amigos on-line, conheça o MSN Messenger: >>>http://messenger.msn.com >>> >>>========================================================================= >>>Instruções para entrar na lista, sair da lista e usar a lista em >>>http://www.mat.puc-rio.br/~nicolau/olimp/obm-l.html >>>O administrador desta lista é >>>========================================================================= >> >> >> >> >>_________________________________________________________________ >>Chegou o novo MSN Explorer. Instale já. É gratuito: >>http://explorer.msn.com.br >> >>========================================================================= >>Instruções para entrar na lista, sair da lista e usar a lista em >>http://www.mat.puc-rio.br/~nicolau/olimp/obm-l.html >>O administrador desta lista é >>========================================================================= > > >_________________________________________________________________ >O MSN Photos é o modo mais fácil de compartilhar e imprimir suas fotos: >http://photos.msn.com/support/worldwide.aspx > >========================================================================= >Instruções para entrar na lista, sair da lista e usar a lista em >http://www.mat.puc-rio.br/~nicolau/olimp/obm-l.html >O administrador desta lista é >========================================================================= _________________________________________________________________ Envie e receba emails com o Hotmail no seu dispositivo móvel: http://mobile.msn.com ========================================================================= Instruções para entrar na lista, sair da lista e usar a lista em http://www.mat.puc-rio.br/~nicolau/olimp/obm-l.html O administrador desta lista é ========================================================================= From owner-obm-l@sucuri.mat.puc-rio.br Fri May 17 14:07:52 2002 Return-Path: Received: (from majordom@localhost) by sucuri.mat.puc-rio.br (8.9.3/8.9.3) id OAA24359 for obm-l-list; Fri, 17 May 2002 14:06:44 -0300 Received: from www.zipmail.com.br (smtp.zipmail.com.br [200.187.242.10]) by sucuri.mat.puc-rio.br (8.9.3/8.9.3) with ESMTP id OAA24355 for ; Fri, 17 May 2002 14:06:40 -0300 From: peterdirichlet@zipmail.com.br Received: from [200.206.103.3] by www.zipmail.com.br with HTTP; Fri, 17 May 2002 14:04:55 -0300 Message-ID: <3CE50D8E000007CB@www.zipmail.com.br> Date: Fri, 17 May 2002 14:04:55 -0300 In-Reply-To: Subject: [obm-l] =?iso-8859-1?Q?Corre=E7ao=3AApelo=3A=20Mais=20da=20Iberoamericana?= To: obm-l@mat.puc-rio.br MIME-Version: 1.0 Content-Type: text/plain; charset="iso-8859-1" Content-Transfer-Encoding: 8bit X-MIME-Autoconverted: from quoted-printable to 8bit by sucuri.mat.puc-rio.br id OAA24356 Sender: owner-obm-l@sucuri.mat.puc-rio.br Precedence: bulk Reply-To: obm-l@mat.puc-rio.br ANSWER:Bem,apoveito e respondo o e-mail do Bruno.Bem,acho que o intuito nao seria o de explora-los. Afinal,so porque NINGUEM RECEBE SALARIO nao significa que nao possa responder.Fiquei espantado pela demora.Certa vez o Nicolau me respondeu uma questao 1 semana depois que enviei o e-mail.E agora fiquei mo cara sem resposta.Talvez nao me interpretei direito... Mas se as questoes estavam mal-formuladas,por que nao me avisaram? Corrigindo:1)p*q+r tambem pertence ao dito conjunto.E o r e o n sao iguais(erros de grafia e pressa). 3)Resto da DIVISAO,ta? 4)Eu modifiquei os enunciados(de modo imperceptivel:no lugar de lideres da OIM estava representantes). Mas e isso mesmo,a ideia e IGUALZINHA Meu,sera que me fiz claro? Um abraço.Peterdirichlet -- Mensagem original -- >Ola Dirichlet, > >Ninguem respondeu, MUITO PROVAVELMENTE, porque as suas questoes, > >1) Estao mal formuladas. Por exemplo, voce escreveu : > >>5)Sabe-se que num conjunto de primos se p e q sao elementos(iguais ou >> >nao)entao p*q+r,em que r e constante.Quantos elementos tem S com >>n=4?>Generalize o r. > >p*q+r O QUE ? E ESSE n=4, O QUE E ? > >Essa mal formulacao EVIDENTE impossibilita uma solucao pode ter lancado uma > >descrenca quanto a correcao do enunciado das demais questoes. > >2) A maioria delas ja tem solucao nos arquivos de mensagens que o Prof >Nicolau guarda. por exemplo, voce escreveu : > >>1)Ache todos os naturais n de 3,2 ou 1 digito tal que o quadrado de n >seja > >>o cubo da soma dos digitos. > >Esta questao ( ou outra semelhante ) ja foi respondida e a solucao esta no > >arquivos de mensagens a que me referi acima. La voce vai ver uma linha de > >raciocinio proxima de : > >(a+b)^3=(10a+b)^2 => a+b = [(10a+b)/a+b]^2 e portanto "a+b" e quadrado >perfeito e a+b divide 10a+b. Como a+b =< 18, os possiveis valores quadrado > >de a+b serao ... > >>3)L>0 e tal que -L^2+1998*L+1=0.Seja a recorrencia a(0)=1 e a(n+1)>parte > >>inteira de L*a(n)=[L*a(n)].Calcule a(1998)mod 1998(x mod y e o >resto de >x >>por y). > >RESTO DE QUE ? ADVINHANDO ... Se x e a unica solucao positiva da equacao >do >2 grau do enunciado de sua questao e [x] e a funcao maximo inteiro, o >problema consiste em determinar o valor de > >[x...[x[x[x]]]...] > >com 1998 colchetes. como >[x] = k se k =< x < K+1 >entao [x[x]] e a anlise de [kx] e assim sucessivamente. ESTA QUESTAO JA FOI > >RESOLVIDA NA LISTA ! > >>4)na mesa da banca de lideres da OIM estao lideres de P paises de modo >>que se dois lideres quaisquer sao de mesmo pais entao seus vizinhos >> >direitos nao sao.Quantos lideres ha no maximo ? > >FALTAM INFORMACOES ! Se P=3, sejam A,B e C lideres, dispostos ao longo de > >uma mesa nesta ordem. Posso sempre inserir entre dois deles o terceiro, de > >forma que a direita de cada um nao havera duplicacao. EVIDENTEMENTE que este > >processo pode ser extendido AD INFINITUM ! > >Meu, que coisas sao essas?Ate agora ninguem ENTENDEU !!!!!! > >Um abraco >Paulo Santa Rita >5,1827,160502 > >>From: peterdirichlet@zipmail.com.br >>Reply-To: obm-l@mat.puc-rio.br >>To: obm-l@mat.puc-rio.br >>Subject: [obm-l] Apelo: Mais da Iberoamericana >>Date: Thu, 16 May 2002 14:21:45 -0300 >> >>Meu,que coisa e essa?Ate agora ninguem me respondeu!!!!!!!! >> >>-- Mensagem original -- >> >> >Alo turma!!!!!!Tenho mais perguntas a fazer(da Iberoamericana): >> >1)Ache todos os naturais n de 3,2 ou 1 digito tal que o quadrado de n > >>seja >> >o cubo da soma dos digitos. >> >2)Encontre o menor n tal que se pegarmos n dos 999 primeiros inteiros > >>positivos >> >sempre se acham 4 numeros diferentes a,b,c,d com a+2*b+3*c-4*d=0. >> >3)L>0 e tal que -L^2+1998*L+1=0.Seja a recorrencia a(0)=1 e a(n+1)=parte >> >inteira de L*a(n)=[L*a(n)].Calcule a(1998)mod 1998(x mod y e o resto >de >> >x por y). >> >4)na mesa da banca de lideres da OIM estao lideres de P paises de modo >>que >> >se dois lideres quaisquer sao de mesmo pais entao seus vizinhos direitos >> >nao sao.Quantos lideres ha no maximo? >> >5)Sabe-se que num conjunto de primos se p e q sao elementos(iguais ou > >>nao)entao >> >p*q+r,em que r e constante.Quantos elementos tem S com n=4?Generalize >o >> >r. >> > >> >TRANSIRE SVVM PECTVS MVNDOQUE POTIRE >> >CONGREGATI EX TOTO ORBE MATHEMATICI OB SCRIPTA INSIGNIA TRIBVERE >> >Medalha Fields(John Charles Fields) >> > >> > >> >------------------------------------------ >> >Use o melhor sistema de busca da Internet >> >Radar UOL - http://www.radaruol.com.br >> > >> > >> > >> >========================================================================= >> >Instruções para entrar na lista, sair da lista e usar a lista em >> >http://www.mat.puc-rio.br/~nicolau/olimp/obm-l.html >> >O administrador desta lista é >> >========================================================================= >> > >> >>TRANSIRE SVVM PECTVS MVNDOQUE POTIRE >>CONGREGATI EX TOTO ORBE MATHEMATICI OB SCRIPTA INSIGNIA TRIBVERE >>Medalha Fields(John Charles Fields) >> >> >>------------------------------------------ >>Use o melhor sistema de busca da Internet >>Radar UOL - http://www.radaruol.com.br >> >> >> >>========================================================================= >>Instruções para entrar na lista, sair da lista e usar a lista em >>http://www.mat.puc-rio.br/~nicolau/olimp/obm-l.html >>O administrador desta lista é >>========================================================================= > > > > >_________________________________________________________________ >Chegou o novo MSN Explorer. Instale já. É gratuito: >http://explorer.msn.com.br > >========================================================================= >Instruções para entrar na lista, sair da lista e usar a lista em >http://www.mat.puc-rio.br/~nicolau/olimp/obm-l.html >O administrador desta lista é >========================================================================= > TRANSIRE SVVM PECTVS MVNDOQUE POTIRE CONGREGATI EX TOTO ORBE MATHEMATICI OB SCRIPTA INSIGNIA TRIBVERE Medalha Fields(John Charles Fields) ------------------------------------------ Use o melhor sistema de busca da Internet Radar UOL - http://www.radaruol.com.br ========================================================================= Instruções para entrar na lista, sair da lista e usar a lista em http://www.mat.puc-rio.br/~nicolau/olimp/obm-l.html O administrador desta lista é ========================================================================= From owner-obm-l@sucuri.mat.puc-rio.br Fri May 17 14:16:14 2002 Return-Path: Received: (from majordom@localhost) by sucuri.mat.puc-rio.br (8.9.3/8.9.3) id OAA24638 for obm-l-list; Fri, 17 May 2002 14:16:02 -0300 Received: from www.zipmail.com.br (smtp.zipmail.com.br [200.187.242.10]) by sucuri.mat.puc-rio.br (8.9.3/8.9.3) with ESMTP id OAA24634 for ; Fri, 17 May 2002 14:16:00 -0300 From: peterdirichlet@zipmail.com.br Received: from [200.206.103.3] by www.zipmail.com.br with HTTP; Fri, 17 May 2002 14:14:10 -0300 Message-ID: <3CE50D8E00000846@www.zipmail.com.br> Date: Fri, 17 May 2002 14:14:10 -0300 In-Reply-To: Subject: [obm-l] =?iso-8859-1?Q?Re=3A=20=5Bobm=2Dl=5D=20a=2E=20basica?= To: obm-l@mat.puc-rio.br MIME-Version: 1.0 Content-Type: text/plain; charset="iso-8859-1" Content-Transfer-Encoding: 8bit X-MIME-Autoconverted: from quoted-printable to 8bit by sucuri.mat.puc-rio.br id OAA24635 Sender: owner-obm-l@sucuri.mat.puc-rio.br Precedence: bulk Reply-To: obm-l@mat.puc-rio.br ANSWER:Eu tenho o livrito da Iberoamericana.A dica e tentar completar o quadrado.Ai se resolvem os dois trechos.Se nao entender me avise!!!!! Um abraço.Peterdirichlet -- Mensagem original -- > >Ae, olha este problema: >Seja P(x,y)=5x^2 -6xy +2y^2. >a)determine qnts elementos de {1,2...,100} são valores de P. >b)Prove q o produto de valores de P é um valor de P. >Será que alguém pode me dar uma ajuda? >Valeu! > []´s > Adherbal > > > >_________________________________________________________________ >Envie e receba emails com o Hotmail no seu dispositivo móvel: >http://mobile.msn.com > >========================================================================= >Instruções para entrar na lista, sair da lista e usar a lista em >http://www.mat.puc-rio.br/~nicolau/olimp/obm-l.html >O administrador desta lista é >========================================================================= > TRANSIRE SVVM PECTVS MVNDOQUE POTIRE CONGREGATI EX TOTO ORBE MATHEMATICI OB SCRIPTA INSIGNIA TRIBVERE Medalha Fields(John Charles Fields) ------------------------------------------ Use o melhor sistema de busca da Internet Radar UOL - http://www.radaruol.com.br ========================================================================= Instruções para entrar na lista, sair da lista e usar a lista em http://www.mat.puc-rio.br/~nicolau/olimp/obm-l.html O administrador desta lista é ========================================================================= From owner-obm-l@sucuri.mat.puc-rio.br Fri May 17 18:19:32 2002 Return-Path: Received: (from majordom@localhost) by sucuri.mat.puc-rio.br (8.9.3/8.9.3) id SAA29466 for obm-l-list; Fri, 17 May 2002 18:17:18 -0300 Received: from aquarius.ime.eb.br (aquarius.ime.eb.br [200.20.120.44]) by sucuri.mat.puc-rio.br (8.9.3/8.9.3) with ESMTP id SAA29462 for ; Fri, 17 May 2002 18:17:15 -0300 Received: from epq.ime.eb.br (epq.ime.eb.br [200.20.120.217]) by aquarius.ime.eb.br (8.9.1a/8.9.3) with ESMTP id SAA09232 for ; Fri, 17 May 2002 18:18:13 -0300 Received: from EPQ/SpoolDir by epq.ime.eb.br (Mercury 1.48); 17 May 02 18:17:24 -0300 Received: from SpoolDir by EPQ (Mercury 1.48); 17 May 02 18:17:10 -0300 Received: from LAB35 (200.20.123.165) by epq.ime.eb.br (Mercury 1.48); 17 May 02 18:17:05 -0300 Message-ID: <004e01c1fde7$f09824e0$a57b14c8@LAB35> From: "Euclides" To: Subject: [obm-l] Derivada Date: Fri, 17 May 2002 18:15:13 -0300 MIME-Version: 1.0 Content-Type: multipart/alternative; boundary="----=_NextPart_000_0049_01C1FDCE.CA288CF0" X-Priority: 3 X-MSMail-Priority: Normal X-Mailer: Microsoft Outlook Express 5.00.3018.1300 X-MimeOLE: Produced By Microsoft MimeOLE V5.00.3018.1300 Sender: owner-obm-l@sucuri.mat.puc-rio.br Precedence: bulk Reply-To: obm-l@mat.puc-rio.br This is a multi-part message in MIME format. ------=_NextPart_000_0049_01C1FDCE.CA288CF0 Content-Type: text/plain; charset="iso-8859-1" Content-Transfer-Encoding: quoted-printable Ol=E1 amigos da lista. Algu=E9m pode me ajudar com esta? 1) Sabe-se que x est=E1 no intervalo a<=3Dx<=3Db, com a>0. Queremos = aproximar x por meio de outro n=FAmero t em [a,b] de maneira que o erro = relativo mod(t-x)/x; (mod(x) representa o valor absoluto de x); seja = t=E3o pequeno quanto poss=EDvel. Represente M(t) o valor m=E1ximo de = mod(t-x)/x quando x varia de a a b. (a) Provar que este m=E1ximo ocorre num dos pontos extremos x=3Da ou = x=3Db. (b) Provar que M(t) =E9 m=EDnimo quando t =E9 a m=E9dia harm=F4nica de a = e b. ------=_NextPart_000_0049_01C1FDCE.CA288CF0 Content-Type: text/html; charset="iso-8859-1" Content-Transfer-Encoding: quoted-printable
Ol=E1 amigos da lista. Algu=E9m pode me = ajudar com=20 esta?
 
1) Sabe-se que x est=E1 no intervalo = a<=3Dx<=3Db,=20 com a>0. Queremos aproximar x por meio de outro n=FAmero t em [a,b] = de maneira=20 que o erro relativo mod(t-x)/x; (mod(x) representa o valor absoluto de = x); seja=20 t=E3o pequeno quanto poss=EDvel. Represente M(t) o valor m=E1ximo de = mod(t-x)/x quando=20 x varia de a a b.
(a) Provar que este m=E1ximo ocorre num = dos pontos=20 extremos x=3Da ou x=3Db.
(b) Provar que M(t) =E9 m=EDnimo quando = t =E9 a m=E9dia=20 harm=F4nica de a e b.
------=_NextPart_000_0049_01C1FDCE.CA288CF0-- ========================================================================= Instruções para entrar na lista, sair da lista e usar a lista em http://www.mat.puc-rio.br/~nicolau/olimp/obm-l.html O administrador desta lista é ========================================================================= From owner-obm-l@sucuri.mat.puc-rio.br Fri May 17 19:38:47 2002 Return-Path: Received: (from majordom@localhost) by sucuri.mat.puc-rio.br (8.9.3/8.9.3) id TAA30410 for obm-l-list; Fri, 17 May 2002 19:30:48 -0300 Received: from hotmail.com ([216.32.181.16]) by sucuri.mat.puc-rio.br (8.9.3/8.9.3) with ESMTP id TAA30406 for ; Fri, 17 May 2002 19:30:45 -0300 Received: from mail pickup service by hotmail.com with Microsoft SMTPSVC; Fri, 17 May 2002 15:28:27 -0700 Received: from 200.229.244.100 by lw2fd.hotmail.msn.com with HTTP; Fri, 17 May 2002 22:28:26 GMT X-Originating-IP: [200.229.244.100] From: "Paulo Santa Rita" To: obm-l@mat.puc-rio.br Subject: Re: RES: [obm-l] .......... Date: Fri, 17 May 2002 22:28:26 +0000 Mime-Version: 1.0 Content-Type: text/plain; charset=iso-8859-1; format=flowed Message-ID: X-OriginalArrivalTime: 17 May 2002 22:28:27.0320 (UTC) FILETIME=[2A17CB80:01C1FDF2] Sender: owner-obm-l@sucuri.mat.puc-rio.br Precedence: bulk Reply-To: obm-l@mat.puc-rio.br Ola Raul e demais colegas desta lista, A sua estrategia, neste caso, esta correta, pois ela leva a um resultado correto com passos corretos. Mas me parece que voce nao entendeu em plenitude porque ela funcionou, dai nao ter podido justificar com clareza seus argumentos ... Seja Y(X)= sqrt(5-X). O que sera "sqrt(5 - sqrt(5-X))" ? Sera, sem duvida nenhuma, Y(Y(X)). Portanto, resolver a equacao sqrt(5 - sqrt(5-X)) = X Equivale a perguntar : Para quais X, Y(Y(X))=X ? Evidentemente que podemos generalizar este raciocinio... Por exemplo, uma equacao como sqrt(5 - sqrt(5 - sqrt(5 - X)))= X pode ser parafraseada na pergunta : Para quais X, Y(Y(Y(X)))=X ? O que voce observou e que a solucao de Y(X)=X e a mesma da solucao de Y(X) composta consigo mesma N vezes, certo ? Sera sempre certo isso ? Por que ocorre este fenomeno ? Para que voce possa entender em plenitude o que esta ocorrendo e num estalar de dedos ser capaz de resolver qualquer equacao desta natureza com qualquer quantidade de radicais, faca o seguinte : 1) Observe que a solucao de Y(X)= X e a interseccao do grafico de Y=X com o grafico de Y=sqrt(5 - X) 2) Trace os dois graficos mencionados acima. 3) Interprete geometricamente o processo representado pela equacao. Exemplo. Resolver : sqrt(5 - sqrt(5 - sqrt(5 - X)))= X Claramente que devemos ter 5 - X >= 0 isto e : X =< 5. Tomando um X=X1 qualquer no intervalo (0,5) o valor Y1=sqrt(5 - X1) e a ordenada do grafico de Y=sqrt(5-X) no ponto X=X1. Por este ponto trace uma paralela ao eixo OX ate encontrar a reta Y=X. Seja (X2,Y2) este ponto. Trace por este ponto uma vertical ate encontrar o grafico de Y=sqrt(5 - X). Este novo ponto Y3 sera tal que Y3=sqrt(5 - sqrt(5 - X1)). Repita o processo acima. Voce vai encontrar um ponto Y4 em Y=sqrt(5-X) tal que : Y4=sqrt(5 - sqrt(5 - sqrt(5 - X1))) O que voce procura e ONDE COMECAR, isto e, voce procura o ponto X tal que Y4=X, isto e, a solucao de : X = sqrt(5 - sqrt(5 - sqrt(5 - X1))) Nas condicoes do seu problema o ponto que satisfaz uma tal exigencia e precisamente o X de X=sqrt(5-X). POR ESTA RAZAO, quando voce passou do processo infinito para o finito as coisas funcionaram. Em Sistemas Dinamicos diriamos que o ponto de X=sqrt(5-X) e o PONTO ESTAVEL da questao. Qualquer outro ponto - NAS CONDICOES DO SEU PROBLEMA - implicaria numa divergencia e fuga, conforme voce pode verificar heuristicamente usando os graficos e as interpretacoes que dei. Observe que aqui tivemos uma motivacao EMINENTEMENTE TOPOLOGICA, mas existem teoremas que normatizam este procedimento, mas que acredito nao seria apropriado falar sobre isso aqui e agora. De qualquer forma, voce vai se sentir mais seguro quando voltar a pensar em questoes semelhantes e nao vai se intimidar com equacoes como esta, mesmo que o numero de radicais seja muito grande. Um problema irmao deste seria : Seja Xn+1= KXn(1-Xn). Discuta a convergencia de Xn, se fixados Xo=R e sendo dado K. Muitos autores chamam esta funcao de FUNCAO LOGISTICA e ela e excelente para se ver com clareza estas coisas. Um abraco Paulo Santa Rita 6,1925,170502 > From: Euraul@aol.com > To: obm-l@mat.puc-rio.br > Sent: Friday, May 17, 2002 12:17 AM > Subject: Re: RES: [obm-l] .......... > > > Que tal essa estratégia ? Será que compliquei muito ? > A equação é x=sqrt(5-sqrt(5-x)) ; se x vale sqrt(5-sqrt(5-x)), > >podemos substituir tendo x = sqrt(5-sqrt(5-sqrt(5-sqrt(5-x)))). Se > >fizermos isso infinitas vezes, teremos um problema clássico que > >resumimos para x = sqrt(5-x), isto é, x^2 = 5 - x. Sendo a resposta a > >raiz positiva : (sqrt(21)-1)/2. > Um abraço, > Raul _________________________________________________________________ Chegou o novo MSN Explorer. Instale já. É gratuito: http://explorer.msn.com.br ========================================================================= Instruções para entrar na lista, sair da lista e usar a lista em http://www.mat.puc-rio.br/~nicolau/olimp/obm-l.html O administrador desta lista é ========================================================================= From owner-obm-l@sucuri.mat.puc-rio.br Sun May 19 10:03:18 2002 Return-Path: Received: (from majordom@localhost) by sucuri.mat.puc-rio.br (8.9.3/8.9.3) id KAA00776 for obm-l-list; Sun, 19 May 2002 10:00:57 -0300 Received: from smtp.ieg.com.br (193.125.226.200.in-addr.arpa.ig.com.br [200.226.125.193] (may be forged)) by sucuri.mat.puc-rio.br (8.9.3/8.9.3) with ESMTP id KAA00773 for ; Sun, 19 May 2002 10:00:55 -0300 Received: from joaodias ([200.157.16.153]) by smtp.ieg.com.br (IeG relay/8.9.3) with SMTP id g4J3WjtJ068791 for ; Sun, 19 May 2002 00:32:47 -0300 (BRT) Message-ID: <007b01c1fee5$7dec7960$99109dc8@joaodias> From: "Fabio Dias" To: References: Subject: Re: [obm-l] dificuldades Date: Sat, 18 May 2002 23:39:35 -0300 MIME-Version: 1.0 Content-Type: text/plain; charset="iso-8859-1" Content-Transfer-Encoding: 8bit X-Priority: 3 X-MSMail-Priority: Normal X-Mailer: Microsoft Outlook Express 6.00.2600.0000 X-MimeOLE: Produced By Microsoft MimeOLE V6.00.2600.0000 Sender: owner-obm-l@sucuri.mat.puc-rio.br Precedence: bulk Reply-To: obm-l@mat.puc-rio.br ----- Original Message ----- From: "Adherbal Rocha Filho" To: Sent: Wednesday, May 15, 2002 1:08 AM Subject: [obm-l] dificuldades > 2.Em uma ilha plana existem 11 cidades numeradas de 1 a 11.Estradas retas > ligam 1 a 2,2 a 3,3 a 4,...,10 a 11 e 11 a 1.É possível que uma reta corte > todas as estradas? Se a reta corta a estrada 1-2, então um está de um lado (digamos, A) da reta, e 2 está do lado B. Se 2 está do lado B, 3 está em A. Se 3 está em A, 4 está em B. ... Se 10 está em B, 11 está em A. Como a reta corta 11-1, e 11 está em A, 1 está em B, *absurdo*. []s, Fábio Dias ========================================================================= Instruções para entrar na lista, sair da lista e usar a lista em http://www.mat.puc-rio.br/~nicolau/olimp/obm-l.html O administrador desta lista é ========================================================================= From owner-obm-l@sucuri.mat.puc-rio.br Sun May 19 10:03:18 2002 Return-Path: Received: (from majordom@localhost) by sucuri.mat.puc-rio.br (8.9.3/8.9.3) id JAA00729 for obm-l-list; Sun, 19 May 2002 09:59:41 -0300 Received: from shannon.bol.com.br (shannon.bol.com.br [200.221.24.13]) by sucuri.mat.puc-rio.br (8.9.3/8.9.3) with ESMTP id JAA00726 for ; Sun, 19 May 2002 09:59:38 -0300 Received: from bol.com.br (200.221.24.82) by shannon.bol.com.br (5.1.071) id 3CE1A94700108A26 for obm-l@mat.puc-rio.br; Sat, 18 May 2002 21:02:47 -0300 Date: Sat, 18 May 2002 21:02:47 -0300 Message-Id: Subject: Re: RES: [obm-l] .......... MIME-Version: 1.0 Content-Type: text/plain;charset="iso-8859-1" From: "rafaelc.l" To: obm-l@mat.puc-rio.br X-XaM3-API-Version: 2.4.3.4.4 X-SenderIP: 200.176.142.207 Content-Transfer-Encoding: 8bit X-MIME-Autoconverted: from quoted-printable to 8bit by sucuri.mat.puc-rio.br id JAA00727 Sender: owner-obm-l@sucuri.mat.puc-rio.br Precedence: bulk Reply-To: obm-l@mat.puc-rio.br > Que tal essa estratégia ? Será que compliquei muito ? > A equação é x=sqrt(5-sqrt(5-x)) ; se x vale sqrt (5-sqrt(5-x)), podemos > substituir tendo x = sqrt(5-sqrt(5-sqrt(5-sqrt(5- x)))). Se fizermos isso > infinitas vezes, teremos um problema clássico que resumimos para x = > sqrt(5-x), isto é, x^2 = 5 - x. Sendo a resposta a raiz positiva : > (sqrt(21)-1)/2. > Valeu Raul, caiu na OBM 2001 um questão desse tipo...vejo que é decisivo fazer as questões das olimpiadas pra treinar pro IME...... > __________________________________________________________________________ Quer ter seu próprio endereço na Internet? Garanta já o seu e ainda ganhe cinco e-mails personalizados. DomíniosBOL - http://dominios.bol.com.br ========================================================================= Instruções para entrar na lista, sair da lista e usar a lista em http://www.mat.puc-rio.br/~nicolau/olimp/obm-l.html O administrador desta lista é ========================================================================= From owner-obm-l@sucuri.mat.puc-rio.br Sun May 19 10:04:03 2002 Return-Path: Received: (from majordom@localhost) by sucuri.mat.puc-rio.br (8.9.3/8.9.3) id KAA00874 for obm-l-list; Sun, 19 May 2002 10:02:37 -0300 Received: from web10104.mail.yahoo.com (web10104.mail.yahoo.com [216.136.130.54]) by sucuri.mat.puc-rio.br (8.9.3/8.9.3) with SMTP id KAA00869 for ; Sun, 19 May 2002 10:02:32 -0300 Message-ID: <20020519014346.92568.qmail@web10104.mail.yahoo.com> Received: from [150.161.199.56] by web10104.mail.yahoo.com via HTTP; Sat, 18 May 2002 18:43:46 PDT Date: Sat, 18 May 2002 18:43:46 -0700 (PDT) From: Rafael WC Subject: [obm-l] triângulos To: OBM MIME-Version: 1.0 Content-Type: multipart/mixed; boundary="0-1012779959-1021772626=:91942" Sender: owner-obm-l@sucuri.mat.puc-rio.br Precedence: bulk Reply-To: obm-l@mat.puc-rio.br --0-1012779959-1021772626=:91942 Content-Type: text/plain; charset=us-ascii Content-Disposition: inline Pessoal, ontem mandei uma dúvida sobre contar o total de triângulos de todos os tamanhos de uma figura como a que enviei abaixo novamente. Pensei muito sobre esse problema e cheguei a uma fórmula não muito amigável, mas até que não é ruim. Já dá até pra escrever um algoritmo pra rodar no computador se quiser. Primeiro, eu chamei de x o número de lados de triângulos que temos na base. Por exemplo, se tivermos um triângulo só x = 1. /_\ Se tivermos uma figura com quatro triângulos de menor tamanho, temos: /_\ /_\ /_\ x = 2 Na figura que mandei, temos x = 4. Com isso, já que você tem triângulos de diferentes tamanhos, você deve contar separadamente os triângulos que têm como lado 1 traço, 2 traços, 3 traços...E depois tem que contar os triângulos que estão de cabeça pra baixo com esses mesmos tamanhos. Se você fizer isso em função dos traços da base não fica muito ruim. Todas as linhas vou escrever a soma de várias parcelas de x menos alguma coisa. Quando você for calcular para algum x, você vai fazer as subtrações até encontrar o valor zero, aí você para. Por exemplo, na primeira linha temos: x + (x - 1) + (x - 2) + (x - 3) + ... Se você tiver x = 2, você irá somar até x + (x - 1), porque o próximo dará zero e aí você deve parar. Bom, no final você encontra isso: triângulos de lado 1: cabeça pra cima = x + (x - 1) + (x - 2) + (x - 3) + ... cabeça pra baixo = (x - 1) + (x - 2) + (x - 3) + ... total = x + 2.[(x - 1) + (x - 2) + (x - 3) + ...] É como se o triângulo maior de todos fosse dividido em várias linhas, aí você vai contando de cada linha. triângulos de lado 2: cabeça pra cima = (x - 1) + (x - 2) + (x - 3) + (x - 4) + ... cabeça pra baixo = (x - 3) + (x - 4) + (x - 5) + ... total = (x - 1) + (x - 2) + 2.[(x - 3) + (x - 4) + ...] Por que aqui começamos a ter de cabeça pra baixo só com (x - 3)? Porque para termos um triângulo de cabeça pra baixo, o triângulo maior tem que ter o dobro de traços na base do que o tamanho do triângulo. Como esse tem lado 2, precisamos ter x = 4, que se fizermos (x - 3) dará 1. Enquanto x for menor que 4 esse número será negativo ou zero e aí não vamos contar. triângulos de lado 3: cabeça pra cima = (x - 2) + (x - 3) + (x - 4) + (x - 5) + ... cabeça pra baixo = (x - 5) + (x - 6) + (x - 7) + ... total = (x - 2) + (x - 3) + (x - 4) + 2.[(x - 5) + ...] E assim teremos sempre esse padrão. Os triângulos de cabeça pra cima começam sempre com (x - a), onde "a" é o número anterior ao tamanho do triângulo. E os triângulos de cabeça pra baixo começam sempre com x - (2a - 1). Depois os outros termos você vai tirando sempre 1. No final das contas você pode somar tudo isso. Soma os triângulo de cabeça pra cima com os de cabeça pra baixo de todos os tamanhos. O problema é que não pode desenvolver muita coisa, porque não pode misturar x - 3 com x - 4, porque se você tiver x = 4, você não terá o termo x - 4. Mas somando apenas x - 1 com x - 1 e x - 2 com x -2, você terá: total = x + 3.(x - 1) + 4.(x - 2) + 6.(x - 3) + 7.(x - 4) + 9.(x - 5) + 10.(x - 6) + 12.(x - 7) + 13.(x - 8) + ... No final você tem então todos os fatores x, x - 1, x - 2, x - 3, ... e os coeficientes de cada um têm uma ordem até boazinha: 1, (pula o 2), 3, 4, (pula o 5), 6, 7, (pula o 8), 9, 10, (pula o 11), 12, 13, (pula o 14), ... E você vai usar a fórmula até o termo em que quando fizer a diferença de x com alguma coisa dê zero. Ou você pode até fazer a seguinte regra: considere que desse valor total você vai pegar apenas os x primeiros termos. Por exemplo, vamos pegar o triângulo da figura que tem 4 traços na base, ou seja x = 4. Então vamos pegar até o quarto termo dessa fórmula e fazer x = 4: total = x + 3.(x - 1) + 4.(x - 2) + 6.(x - 3) total = 4 + 3.(4 - 1) + 4.(4 - 2) + 6.(4 - 3) total = 4 + 3.3 + 4.2 + 6.1 total = 4 + 9 + 8 + 6 total = 27 E aí você pode fazer pra qualquer x. Aquele menor que tinha x = 2, só pegamos os 2 primeiros termos: total = x + 3.(x - 1) total = 2 + 3.(2 - 1) total = 2 + 3.1 total = 2 + 3 total = 5 De qualquer jeito você não precisa ficar contando um por um e correr o risco de se perder mais facilmente. Mas o meu problema agora é o seguinte. Suspeito que ainda dê para simplificar a fórmula, considerando duas fórmulas, uma para quando x é par e outra para quando x é ímpar. Talvez simplifique, mas aí você tem duas fórmulas, não sei. Ainda não consegui. Será que alguém consegue melhorar daqui pra frente. O pior acho que já passou. Um abraço, Rafael. ===== Rafael Werneck Cinoto ICQ# 107011599 rwcinoto@yahoo.com rafael.caixa@gov.com.br matduvidas@yahoo.com.br http://www.rwcinoto.hpg.com.br/ __________________________________________________ Do You Yahoo!? LAUNCH - Your Yahoo! Music Experience http://launch.yahoo.com --0-1012779959-1021772626=:91942 Content-Type: image/gif; name="contatriang.gif" Content-Transfer-Encoding: base64 Content-Description: contatriang.gif Content-Disposition: inline; filename="contatriang.gif" R0lGODdhsQFxAfcAAAAAAAAAAAAAAAAAAAAAAAAAAAAAAAAAAAAAAAAAAAAA AAAAAAAAAAAAAAAAAAAAAAAAAAAAAAAAAAAAAAAAAAAAAAAAAAAAAAAAAAAA AAAAAAAAAAAAAAAAAAAAAAAAAAAAAAAAAAAAAAAAAAAAAAAAAAAAAAAAAAAA AAAAAAAAAAAAAAAAAAAAAAAAAAAAAAAAAAAAAAAAAAAAAAAAAAAAAAAAAAAA AAAAAAAAAAAAAAAAAAAAAAAAAAAAAAAAAAAAAAAAAAAAAAAAAAAAAAAAAAAA AAAAAAAAAAAAAAAAAAAAAAAAAAAAAAAAAAAAAAAAAAAAAAAAAAAAAAAAAAAA AAAAAAAAAAAAAAAAAAAAAAAAAAAAAAAAAAAAAAAAAAAAAAAAAAAAAAAAAAAA AAAAAAAAAAAAAAAAAAAAAAAAAAAAAAAAAAAAAAAAAAAAAAAAAAAAAAAAAAAA AAAAAAAAAAAAAAAAAAAAAAAAAAAAAAAAAAAAAAAAAAAAAAAAAAAAAAAAAAAA AAAAAAAAAAAAAAAAAAAAAAAAAAAAAAAAAAAAAAAAAAAAAAAAAAAAAAAAAAAA AAAAAAAAAAAAAAAAAAAAAAAAAAAAAAAAAAAAAAAAAAAAAAAAAAAAAAAAAAAA AAAAAAAAAAAAAAAAAAAAAAAAAAAAAAAAAAAAAAAAAAAAAAAAAAAAAAAAAAAA AAAAAAAAAAAAAAAAAAAAAAAAAAAAAAAAAAAAAAAAAAAAAAAAAAAAAAAAAAAA AAAAAAAAAAAAAAAAAAAAAAAAAAAAAAAAAAAAAAAAAAAAAAAAAAAAAAAAAAAA AAAAAAAAAAAAAAAAAAAAAAAAAAAAAAAAAAAAAAAAAAAAAAAAAAAAAAAAAAAA AAAAAAAAAAAAAAAAAAAAAAAAAAAAAAAAAAAAAAAAAAAAAAAAAAAAAAAAAAAA AAAAAAAAAAAAAAAAAAAAAAAAAAAAAAAAAAAAAAAAAAAAAAAAAAAAAAAAAAAA AAAAAAAAAAAAAAAAAP///yH5BAAAAAAALAAAAACxAXEBQAj/AP8JHEiwoMGD CBMqXMiwocOHECNKnEixosWLGDNq3Mixo8ePIEOKHEmypMmTKFOqXMmypcuX MGPKnEmzps2bOHPq3Mmzp8+fQIMKHUq0qNGjSJMqXcq0qdOnUKNKnUq1qtWr WLNq3cq1q9eHAL6KHUtWZdiDZ8uqXcsWLMS0bePKFQu3Yt25ePMyvcuRr96/ gHGe9bsxLeHAiBOTPCyQsUPH/yArnkz5MUXJAzGjrcy5c0LNDfmCtuy5dGLD IQGMljjYtGu2jlcTPCy7IOTar3MTrX37sluLuHULr6naI+qOcIMrVD68Ocrk fW0jn52auvPrMnuz/r2dIW7JzLGLGw8NfHlGxry9Xww/vj176u83R57eGGT8 9vjz61bfz7+///8ABijggAQWaOCBCCao4IIMNujggxBGKOGEFFZo4YUYZqjh hhx26OGHIIYo4ogklmjiiSimqOKKLLbo4oswxijjjDTWaOONOOao44489ujj j55ABilkc8cNGWRsRuaYXpIuxncfkwcWV9KTUPLnl5OfVRmhZktyp2WBzKFX 2Jf+UTlffSNJSSaR8ml0131Xrmkal92RFtFoZsoZVXiiuZnlmHq2pdqghBZq 6KGIJqrooow26iiigXZFWHC0nYcQe5NGWhVomMnW6Z11eqnpUXwaBKeplraZ 6qisturqq7DGKuustNZq66245qrrrrz26jnrr8AGK+ywxBZr7LHIJqvsssw2 6+yz0EYr7bTUVmvttdhmq+223Hbr7bfghivuuOSWa+656Kar7rqy7LaLbFh5 uqvWm/IKV2m9pYGHb2WrxbtvU5T+ixemAsNmX8F0LYbwVmqKBO/CVkF3sHQQ O3XvqpdWrJS+5dmpsVB4TuTpxz8FrB5G/pLcMcYUA6ryTE/WlWfKLy/36M04 57xozWa1HN2ZH8nM85TmoZyxy6oOjXTRIp9sl6hKPx3qQiG/5VvUV0v9Z9Zb r4c11Cuj6Seq9H19tHFA0xfvw2ajeWraP68tdtT0sjy312QvfV23ymJy3fXU Pmt99sefWg2q4eQ1Dfa+I1ONt+OHJ674v8rFqffekSftN7tYYj6255ODLnjb mZkUM9ykp6766qy37vrrsMcu++y012777bjnrvvuvPfu++/ABy/88MQpF2/8 8cgnr/zyzDfv/PPQRy/99NRXb/312Gev/fbcd+/99+CHL/745Jcmb/756Kev /vrst+/++/DHL//89Ndv//3456///vz37///AAygAAdtSMACWgl1Btwazdon tAT+TXQFzJQDrfPACHqMgHSyYOYA2DgOho1/lfMfwfTXufudzn5mWqDz5CY/ mqkQeTqLIaEYCEGj1RB8Egta4MpXt7JRkIeDs1vpgFjBzeUNh077YBGzV7jQ SY57nHLiBdGx168NIo6Ko2OaEaVnMsjZkHph8qIQmzdCzX3xeW8bog7RODEE /ox5M3OjD2Eox/M0bI0v5FkO4+awHw6vh5+roxLzSLggDnKOfvTdxbKYyC2q UZFPtOIRpWjI3DXxilqUZCZvF0VMinFxn6RdB0M5xSVGUnZdrCQoG6nJ1oVR lZ7coSNVV0ZWAu6GsaSlD9OIy1ua7Y5johK8WKhL+8wsZYT8FtvUZkx/LfNr RQokL1P4SKUBUomC3OAJbakxy2Gzl6QsoSy7uclccvOU51xlMoCnhSRKmrGU 4zRnNSF2SXSWM4mmJCUsGdfKScIzn+l8J0DbVUV89jOg8RSoPt3VpX3+c54P RahD05XKhLbyOwb1pbpKZVFKhnCiES3Xe/b4RnBuUpwbRVs28TZSicpTXDKM qUxnStOaonCCOM2pTnfK05769KdADapQh0rUohr1qCVITapSl8rUpjr1qVCN qlSnStWqWvWqWM2qVrfK1a569atgDatYI8dK1rKa9axoTata18rWtrr1rXCN q1znSte62vWueM2rXvfKCte++vWvgP1qQAAAOw== --0-1012779959-1021772626=:91942-- ========================================================================= Instruções para entrar na lista, sair da lista e usar a lista em http://www.mat.puc-rio.br/~nicolau/olimp/obm-l.html O administrador desta lista é ========================================================================= From owner-obm-l@sucuri.mat.puc-rio.br Sun May 19 10:04:25 2002 Return-Path: Received: (from majordom@localhost) by sucuri.mat.puc-rio.br (8.9.3/8.9.3) id KAA00910 for obm-l-list; Sun, 19 May 2002 10:03:09 -0300 Received: from hotmail.com (f12.pav1.hotmail.com [64.4.31.12]) by sucuri.mat.puc-rio.br (8.9.3/8.9.3) with ESMTP id KAA00901 for ; Sun, 19 May 2002 10:03:05 -0300 Received: from mail pickup service by hotmail.com with Microsoft SMTPSVC; Sun, 19 May 2002 04:46:46 -0700 Received: from 200.199.189.246 by pv1fd.pav1.hotmail.msn.com with HTTP; Sun, 19 May 2002 11:46:46 GMT X-Originating-IP: [200.199.189.246] From: "Adherbal Rocha Filho" To: obm-l@mat.puc-rio.br Subject: Re: [obm-l] Re: [obm-l] a. basica Date: Sun, 19 May 2002 11:46:46 +0000 Mime-Version: 1.0 Content-Type: text/plain; charset=iso-8859-1; format=flowed Message-ID: X-OriginalArrivalTime: 19 May 2002 11:46:46.0633 (UTC) FILETIME=[DAB8AD90:01C1FF2A] Sender: owner-obm-l@sucuri.mat.puc-rio.br Precedence: bulk Reply-To: obm-l@mat.puc-rio.br Oi, Ae,ainda não consegui terminar naum...cheguei a uma soma de quadrados q naum ajudou muito... > >ANSWER:Eu tenho o livrito da Iberoamericana.A dica e tentar completar o >quadrado.Ai se resolvem os dois trechos.Se nao entender me avise!!!!! >Um abraço.Peterdirichlet > >-- Mensagem original -- > > > > >Ae, olha este problema: > >Seja P(x,y)=5x^2 -6xy +2y^2. > >a)determine qnts elementos de {1,2...,100} são valores de P. > >b)Prove q o produto de valores de P é um valor de P. > >Será que alguém pode me dar uma ajuda? > >Valeu! > > []´s > > Adherbal > > > > > > > >_________________________________________________________________ > >Envie e receba emails com o Hotmail no seu dispositivo móvel: > >http://mobile.msn.com > > > >========================================================================= > >Instruções para entrar na lista, sair da lista e usar a lista em > >http://www.mat.puc-rio.br/~nicolau/olimp/obm-l.html > >O administrador desta lista é > >========================================================================= > > > >TRANSIRE SVVM PECTVS MVNDOQUE POTIRE >CONGREGATI EX TOTO ORBE MATHEMATICI OB SCRIPTA INSIGNIA TRIBVERE >Medalha Fields(John Charles Fields) > > >------------------------------------------ >Use o melhor sistema de busca da Internet >Radar UOL - http://www.radaruol.com.br > > > >========================================================================= >Instruções para entrar na lista, sair da lista e usar a lista em >http://www.mat.puc-rio.br/~nicolau/olimp/obm-l.html >O administrador desta lista é >========================================================================= _________________________________________________________________ Converse com amigos on-line, conheça o MSN Messenger: http://messenger.msn.com ========================================================================= Instruções para entrar na lista, sair da lista e usar a lista em http://www.mat.puc-rio.br/~nicolau/olimp/obm-l.html O administrador desta lista é ========================================================================= From owner-obm-l@sucuri.mat.puc-rio.br Sun May 19 10:09:00 2002 Return-Path: Received: (from majordom@localhost) by sucuri.mat.puc-rio.br (8.9.3/8.9.3) id KAA01470 for obm-l-list; Sun, 19 May 2002 10:07:43 -0300 Received: from hotmail.com (f132.pav1.hotmail.com [64.4.31.132]) by sucuri.mat.puc-rio.br (8.9.3/8.9.3) with ESMTP id KAA01466 for ; Sun, 19 May 2002 10:07:41 -0300 Received: from mail pickup service by hotmail.com with Microsoft SMTPSVC; Sun, 19 May 2002 04:51:23 -0700 Received: from 200.199.189.246 by pv1fd.pav1.hotmail.msn.com with HTTP; Sun, 19 May 2002 11:51:23 GMT X-Originating-IP: [200.199.189.246] From: "Adherbal Rocha Filho" To: obm-l@mat.puc-rio.br Date: Sun, 19 May 2002 11:51:23 +0000 Mime-Version: 1.0 Content-Type: text/plain; charset=iso-8859-1; format=flowed Message-ID: X-OriginalArrivalTime: 19 May 2002 11:51:23.0899 (UTC) FILETIME=[7FFC1CB0:01C1FF2B] Sender: owner-obm-l@sucuri.mat.puc-rio.br Precedence: bulk Reply-To: obm-l@mat.puc-rio.br mais um probleminha: suponha q a Terra eh uma esfera e que uma corda está amarrada ao redor da linha do equador.Agora suponha que esta corda eh aumentada em um metro ,formando uma circunferencia maior,qual será a distancia entre a superficie da Terra e a corda? E se eu fizesse o mesmo pra uma bola de futebol,qual seria a distancia? valeu! []´s Adherbal _________________________________________________________________ Envie e receba emails com o Hotmail no seu dispositivo móvel: http://mobile.msn.com ========================================================================= Instruções para entrar na lista, sair da lista e usar a lista em http://www.mat.puc-rio.br/~nicolau/olimp/obm-l.html O administrador desta lista é ========================================================================= From owner-obm-l@sucuri.mat.puc-rio.br Sun May 19 10:09:26 2002 Return-Path: Received: (from majordom@localhost) by sucuri.mat.puc-rio.br (8.9.3/8.9.3) id KAA01523 for obm-l-list; Sun, 19 May 2002 10:08:10 -0300 Received: from hotmail.com (f15.pav2.hotmail.com [64.4.37.15]) by sucuri.mat.puc-rio.br (8.9.3/8.9.3) with ESMTP id KAA01516 for ; Sun, 19 May 2002 10:08:07 -0300 Received: from mail pickup service by hotmail.com with Microsoft SMTPSVC; Sun, 19 May 2002 04:36:27 -0700 Received: from 200.199.189.246 by pv2fd.pav2.hotmail.msn.com with HTTP; Sun, 19 May 2002 11:36:26 GMT X-Originating-IP: [200.199.189.246] From: "Fernanda Medeiros" To: obm-l@mat.puc-rio.br Subject: [obm-l] =?iso-8859-1?B?aW52ZXJz428vZGVzaWd1YWxkYWRlcy9jb25lIHN1bA==?= Date: Sun, 19 May 2002 11:36:26 +0000 Mime-Version: 1.0 Content-Type: text/plain; charset=iso-8859-1; format=flowed Message-ID: X-OriginalArrivalTime: 19 May 2002 11:36:27.0015 (UTC) FILETIME=[69667970:01C1FF29] Sender: owner-obm-l@sucuri.mat.puc-rio.br Precedence: bulk Reply-To: obm-l@mat.puc-rio.br Oi pessoal, alguém poderia me ajudar nessas 2 questões? Bem,aé vão: 1.Sejam a,c,d e d os lados consecutivos de um quadrilátero ABCD e x e y as suas diagonais.Suponha que os círculos circunscritos aos triangulos ABC e ACD são ortogonais.Mostre que (x^2)(y^2)=(a^2)(c^2) + (b^2)(d^2) 2.Seja P um ponto no interior de um triangulo e sejam ha,hb e hc as distancias de P aos lados a,b e c,respectivamente.Mostre q o valor mínimo de a/ha +b/hb +c/hc ocorre quando P é o incentro de ABC. 3.Seja p um real positivo dado.Achar o mínimo valor de x^3 +y^3 sabendo que x e y são reais positivos tais que xy(x+y)=p Obrigada! []´s Fê _________________________________________________________________ Converse com amigos on-line, conheça o MSN Messenger: http://messenger.msn.com ========================================================================= Instruções para entrar na lista, sair da lista e usar a lista em http://www.mat.puc-rio.br/~nicolau/olimp/obm-l.html O administrador desta lista é ========================================================================= From owner-obm-l@sucuri.mat.puc-rio.br Sun May 19 10:11:53 2002 Return-Path: Received: (from majordom@localhost) by sucuri.mat.puc-rio.br (8.9.3/8.9.3) id KAA01794 for obm-l-list; Sun, 19 May 2002 10:10:38 -0300 Received: from bidu.ime.usp.br (bidu.ime.usp.br [143.107.45.12]) by sucuri.mat.puc-rio.br (8.9.3/8.9.3) with SMTP id KAA01785 for ; Sun, 19 May 2002 10:10:36 -0300 Received: (qmail 17281 invoked from network); 18 May 2002 15:11:52 -0000 Received: from mafalda.ime.usp.br (143.107.45.13) by bidu.ime.usp.br with SMTP; 18 May 2002 15:11:52 -0000 Received: (qmail 9002 invoked by uid 1604); 18 May 2002 15:11:51 -0000 Date: Sat, 18 May 2002 12:11:50 -0300 (EST) From: Salvador Addas Zanata X-Sender: sazanata@mafalda To: obm-l@mat.puc-rio.br Subject: [obm-l] =?iso-8859-1?Q?Re:_Sob_que_condi=E7oes_uma_deformacao_preserva_medidas?= In-Reply-To: Message-ID: MIME-Version: 1.0 Content-Type: TEXT/PLAIN; charset=ISO-8859-1 Content-Transfer-Encoding: 8bit X-MIME-Autoconverted: from QUOTED-PRINTABLE to 8bit by sucuri.mat.puc-rio.br id KAA01788 Sender: owner-obm-l@sucuri.mat.puc-rio.br Precedence: bulk Reply-To: obm-l@mat.puc-rio.br Caros Nicolau e demais membros, Faz um certo tempo o Nicolau mandou um e-mail que tinha o paragrafo abaixo. Ocorre que eu li isso em uma superinteressante quando estava na escola e ate hoje tenho isso na cabeca, nao sabia se tinha sonhado, ou se era besteira, etc. Se alguem souber qual e a refererencia onde isso foi provado, ou pelo menos quem provou, ia me ajudar muito. Pelo que eu me lembro, na revista falava-se algo em torno de 2^50 pedacos... Abraco a todos, Salvador On Sun, 4 Feb 2001, Nicolau C. Saldanha wrote: > Aliás um grande problema da matemática do século XX foi o da quadratura > do círculo: não aquele proposto pelos gregos e cuja demostração foi > concluída com a prova da transcendência de pi. O problema século XX > da quadratura do círculo é: será possível decompor um círculo de área 1 > em um número finito de peças e rearrumá-las para formar um quadrado > de área 1? A resposta é que sim, é possível. > > []s, N. > > ========================================================================= Instruções para entrar na lista, sair da lista e usar a lista em http://www.mat.puc-rio.br/~nicolau/olimp/obm-l.html O administrador desta lista é ========================================================================= From owner-obm-l@sucuri.mat.puc-rio.br Sun May 19 10:12:00 2002 Return-Path: Received: (from majordom@localhost) by sucuri.mat.puc-rio.br (8.9.3/8.9.3) id KAA01805 for obm-l-list; Sun, 19 May 2002 10:10:45 -0300 Received: from web10101.mail.yahoo.com (web10101.mail.yahoo.com [216.136.130.51]) by sucuri.mat.puc-rio.br (8.9.3/8.9.3) with SMTP id KAA01802 for ; Sun, 19 May 2002 10:10:42 -0300 Message-ID: <20020519015157.86648.qmail@web10101.mail.yahoo.com> Received: from [200.151.167.101] by web10101.mail.yahoo.com via HTTP; Sat, 18 May 2002 18:51:57 PDT Date: Sat, 18 May 2002 18:51:57 -0700 (PDT) From: Rafael WC Subject: [obm-l] permutações circulares com repetição To: OBM MIME-Version: 1.0 Content-Type: text/plain; charset=us-ascii Sender: owner-obm-l@sucuri.mat.puc-rio.br Precedence: bulk Reply-To: obm-l@mat.puc-rio.br Estou com problemas para resolver esse exercício: "De quantas maneiras distintas podemos dispor ao longo de um circulo, suposto fixo, 6 bolas brancas, 8 bolas azuis, 16 bolas verdes e 24 bolas amarelas?" O círculo fica fixo em nossa frente, mas as bolas ficam livres para serem rotacionadas como em uma catraca de bicicleta. Pra mim, entendi como sendo uma permutação circular com repetição. Nunca estudei isso e o que pensei que seria mais lógico não deu muito certo. Pensei que faríamos as permutações com repetições e dividiríamos pelo total de bolas por causa de ser circular. Em alguns casos até que conferiu com a resposta, mas aí coloquei um teste com apenas 4 bolas, duas brancas e duas azuis. Se eu fosse fazer como pensei, seria: PC4(2,2) = 4!/4.2!.2! = 3/2 Nem inteiro dá!!! Ao fazer escrevendo mesmo, vejo que só temos 6 permutações: 1 - AABB 2 - ABAB 3 - ABBA 4 - BAAB 5 - BABA 6 - BBAA E destas, se considerarmos como circulares, vemos que 1 = 3 = 4 = 6 e 2 = 5. O que nos dá apenas 2 permutações. Alguém sabe como resolvo esse tipo de problema? O único livro que tenho aqui sobre análise combinatória (Introdução à Análise Combinatória; Santos, J. P. O.; Mello, M. P.; Murari, I. T. C.; 2ª edição; Campinas, SP; Ed. da Unicamp, 1988) que aliás é muito bom, não fala sobre permutações circulares com combinação. Agradeço qualquer dica. Rafael. ===== Rafael Werneck Cinoto ICQ# 107011599 rwcinoto@yahoo.com rafael.caixa@gov.com.br matduvidas@yahoo.com.br http://www.rwcinoto.hpg.com.br/ __________________________________________________ Do You Yahoo!? LAUNCH - Your Yahoo! Music Experience http://launch.yahoo.com ========================================================================= Instruções para entrar na lista, sair da lista e usar a lista em http://www.mat.puc-rio.br/~nicolau/olimp/obm-l.html O administrador desta lista é ========================================================================= From owner-obm-l@sucuri.mat.puc-rio.br Sun May 19 10:40:56 2002 Return-Path: Received: (from majordom@localhost) by sucuri.mat.puc-rio.br (8.9.3/8.9.3) id KAA03669 for obm-l-list; Sun, 19 May 2002 10:39:38 -0300 Received: from traven10.uol.com.br (traven10.uol.com.br [200.231.206.211]) by sucuri.mat.puc-rio.br (8.9.3/8.9.3) with ESMTP id KAA03664 for ; Sun, 19 May 2002 10:39:36 -0300 Received: from Eder ([200.227.65.65]) by traven10.uol.com.br (8.9.1/8.9.1) with SMTP id VAB14430 for ; Fri, 17 May 2002 21:37:13 -0300 (BRT) Message-ID: <005c01c1fe03$d93264c0$82ccfea9@Eder> From: "Eder" To: Subject: [obm-l] =?iso-8859-1?Q?valor_m=EDnimo?= Date: Fri, 17 May 2002 21:34:16 -0300 MIME-Version: 1.0 Content-Type: multipart/alternative; boundary="----=_NextPart_000_0059_01C1FDEA.98C97A40" X-Priority: 3 X-MSMail-Priority: Normal X-Mailer: Microsoft Outlook Express 5.00.2314.1300 X-MimeOLE: Produced By Microsoft MimeOLE V5.00.2314.1300 Sender: owner-obm-l@sucuri.mat.puc-rio.br Precedence: bulk Reply-To: obm-l@mat.puc-rio.br This is a multi-part message in MIME format. ------=_NextPart_000_0059_01C1FDEA.98C97A40 Content-Type: text/plain; charset="iso-8859-1" Content-Transfer-Encoding: quoted-printable Ol=E1, =C9 poss=EDvel determinar para que valor de a,tem-se y=3D sqrt( 1+ = (1-a)=B2) + sqrt(1+ a=B2) m=EDnimo? ------=_NextPart_000_0059_01C1FDEA.98C97A40 Content-Type: text/html; charset="iso-8859-1" Content-Transfer-Encoding: quoted-printable
Ol=E1,
 
 
=C9 poss=EDvel determinar para que = valor de a,tem-se y=3D=20 sqrt( 1+ (1-a)=B2) + sqrt(1+ a=B2) m=EDnimo?
 
 
------=_NextPart_000_0059_01C1FDEA.98C97A40-- ========================================================================= Instruções para entrar na lista, sair da lista e usar a lista em http://www.mat.puc-rio.br/~nicolau/olimp/obm-l.html O administrador desta lista é ========================================================================= From owner-obm-l@sucuri.mat.puc-rio.br Sun May 19 10:48:19 2002 Return-Path: Received: (from majordom@localhost) by sucuri.mat.puc-rio.br (8.9.3/8.9.3) id KAA04085 for obm-l-list; Sun, 19 May 2002 10:47:03 -0300 Received: from imo-d02.mx.aol.com (imo-d02.mx.aol.com [205.188.157.34]) by sucuri.mat.puc-rio.br (8.9.3/8.9.3) with ESMTP id KAA04076 for ; Sun, 19 May 2002 10:47:00 -0300 From: DEOLIVEIRASOU@aol.com Received: from DEOLIVEIRASOU@aol.com by imo-d02.mx.aol.com (mail_out_v32.5.) id z.193.7362b7f (25508) for ; Sat, 18 May 2002 17:15:36 -0400 (EDT) Message-ID: <193.7362b7f.2a181e78@aol.com> Date: Sat, 18 May 2002 17:15:36 EDT Subject: [obm-l] (nenhum assunto) To: obm-l@mat.puc-rio.br MIME-Version: 1.0 Content-Type: multipart/alternative; boundary="part1_193.7362b7f.2a181e78_boundary" X-Mailer: AOL 7.0 for Windows BR sub 10501 Sender: owner-obm-l@sucuri.mat.puc-rio.br Precedence: bulk Reply-To: obm-l@mat.puc-rio.br --part1_193.7362b7f.2a181e78_boundary Content-Type: text/plain; charset="US-ASCII" Content-Transfer-Encoding: 7bit (IMO-1963) PROVE QUE COS(PI/7)-COS(2PI/7)+COS(3PI/7)=1/2.COMECEI A FAZER E FOI FICANDO GRANDE...CADA VEZ MAIOR...RISOS...ALGUEM CONSEGUE ACHAR UM TRUQUIINHO AI?? VALEU! CROM --part1_193.7362b7f.2a181e78_boundary Content-Type: text/html; charset="US-ASCII" Content-Transfer-Encoding: 7bit (IMO-1963) PROVE QUE COS(PI/7)-COS(2PI/7)+COS(3PI/7)=1/2.COMECEI A FAZER E FOI FICANDO GRANDE...CADA VEZ MAIOR...RISOS...ALGUEM CONSEGUE ACHAR UM TRUQUIINHO AI??
                                    VALEU!
                                              CROM
--part1_193.7362b7f.2a181e78_boundary-- ========================================================================= Instruções para entrar na lista, sair da lista e usar a lista em http://www.mat.puc-rio.br/~nicolau/olimp/obm-l.html O administrador desta lista é ========================================================================= From owner-obm-l@sucuri.mat.puc-rio.br Sun May 19 11:18:08 2002 Return-Path: Received: (from majordom@localhost) by sucuri.mat.puc-rio.br (8.9.3/8.9.3) id LAA05630 for obm-l-list; Sun, 19 May 2002 11:16:34 -0300 Received: from bogari.terra.com.br (bogari.terra.com.br [200.176.3.24]) by sucuri.mat.puc-rio.br (8.9.3/8.9.3) with ESMTP id LAA05627 for ; Sun, 19 May 2002 11:16:33 -0300 Received: from pina.terra.com.br (pina.terra.com.br [200.176.3.17]) by bogari.terra.com.br (Postfix) with ESMTP id DB1D8A60E1 for ; Sat, 18 May 2002 11:58:37 -0300 (EST) Received: from smtp4-poa.terra.com.br (smtp4-poa.terra.com.br [200.176.3.35]) by pina.terra.com.br (Postfix) with ESMTP id C1C305305C; Sat, 18 May 2002 11:58:37 -0300 (EST) Received: from nt (RJ231244.user.veloxzone.com.br [200.165.231.244]) (authenticated user ensr) by smtp4-poa.terra.com.br (Postfix) with ESMTP id 76BBEAC5AF; Sat, 18 May 2002 11:58:36 -0300 (EST) Message-ID: <003e01c1fe7c$4167b540$5400a8c0@ensrbr> From: "Luis Lopes" To: Subject: [obm-l] "german" triangle construction Date: Sat, 18 May 2002 11:56:54 -0300 MIME-Version: 1.0 Content-Type: text/plain; charset="iso-8859-1" Content-Transfer-Encoding: 8bit X-Priority: 3 X-MSMail-Priority: Normal X-Mailer: Microsoft Outlook Express 5.00.2615.200 X-MimeOLE: Produced By Microsoft MimeOLE V5.00.2615.200 Sender: owner-obm-l@sucuri.mat.puc-rio.br Precedence: bulk Reply-To: obm-l@mat.puc-rio.br Sauda,c~oes, Alguns membros da lista sabem que escrevi (e inclusive têm um exemplar) um livro em francês sobre construção de triângulos. Desde então fiz muitas descobertas que espero colocar numa possível edição em português. No momento, gostaria de mostrar a solução abaixo, que não aparece no livro. Infelizmente o arquivo .jpg que poderia acompanhá-la é muito grande. Posso enviá-lo para o email pessoal dos interessados O ponto importante da construção é o lugar geom (lg). dado pela parábola. Apreciaria se alguém pudesse dar uma outra prova para o lg ( seria possível uma prova sintética?) e também uma análise do lg e a figura feita (parábola) por um programa de geometria. Em palavras, o problema é: Qual o lg de O quando B se move na reta r, tal que BM_b = m_b, m_b (mediana) fixo? A -------|------------|--------------------------------------- r B H_a Obrigado. []'s Luis -----Mensagem Original----- De: Barukh Ziv Para: Luis Lopes Enviada em: sexta-feira, 17 de maio de 2002 15:23 Assunto: Re: En: [EMHL] "german" triangle construction > Dear Luis, > > Thank you very much for your last messages, the > solution > in German book and Sketchpad figure reprodicing > Gilles's > construction. Your notion about the Portugese book > with > construction of conics is very intriguing; may I ask > you > to scan a page or two for me (if time permits)? > > The solution in German book is along the same lines as > two other solutions we discussed (a, m_a, s_a and m_a, > s_a, R). In what follows I present my solution of the > problem h_a, s_a, m_b. Please see the attached file, > that contains four figures: two upper are the > construction > steps; two lower are for the justification. My aim is > to > explain it as thoroughly as my skills permit. Please > remember that in my figures I use 'ta' for what you > use s_a. > > The basic idea is to construct the circumcemter O of > the sought triangle ABC. I use the method of intersection > of loci. So, this approach is essentially the same as > Gilles's, but the element constructed is different. > > The analysis goes as follows: > > 1. First, we know that O lies on a line which is > reflection of AHa in ATa (since angles HaATa and TaAO are > equal). > > 2. Second, assume the segment AHa is fixed; then the > vertex B may move (on a line BC, obviously), so that the > length of the median BMb is constant and equal m_b; what > is the locus of circumcenters O of triangles ABC (one of > such triangles is shown at the left lower figure > attached)? > It turns out that the locus is a parabola with the > following parameters: > > 2a. Its axis is parallel to AHa. > 2b. Its directrix d (perpendicular to AHa, > according to 2a), is m_b^2/(4*h_a) away from Ha. > 2c. Its focus F is 9/8*h_a away from d, and m/4 > away from AHa, where m is defined as follows: in > a right triangle with hypotenuse m_b and > one side h_a/2, the other side is m (see the upper > left figure, triangle P1HaM). > > The proof of this assertion is given below, at the > end of explanation. > > The important outcomes of this analysis are: > > (*) The triangle ABC is constructible with ruler and > compass, as its circumcenter is an intersection of a > straight line and a parabola (cf. discussion of this in J. > Petersen's book on page 98, theorem 1). > > (**) the essential elements of the parabola - its > directix and focus - may be easily constructed from given > data. > > Actually, the upper left figure depicts the > construction of F and d. After the above explanations, > I hope the steps are clear. > > The second step - after F, d and also the line AE (the > reflection > of AHa in ATa) are constructed - is to find the > intersection of > the line with the parabola. Fortunately, this has a > very elegant > solution given in an excellent book "Ruler and > Compass" by H. P. > Hudson (page 85-86). It provides a construction for > ANY conic, but the case of parabola is particularly > simple. Let me guide you through the > construction (upper right figure): > > C.1. Construct AE, the reflection of AHa in ATa. > C.2. Draw line FE. > C.3. Take any point on a line AE, eg. D. > C.4. Draw a circle centered at D and tangent to > directrix d. > C.5. Let G, G' be points of intersect of FE and the > circle. > C.6. Draw FO||DG, FO'||DG' (only the first is > shown). O, O' > are the circumcenters of the 2 solutions. > > To prove the construction, notice that (see lower > right figure) > triangles OFE and DGE, ORE and DQE are similar, > therefore: > > OE OF OR OF DG > ---- = ---- = ----, or ----- = ---- = 1, > DE DG DQ OR DQ > > or OF = OR. Thus, O is equidistant from the focus F > and the directrix of the parabola, that is, lies on it > (see T4 below)! > Beautiful, isn't it?! > > Once we construct O, two vertices B and C are easily > obtained. > > As promised, I conclude the explanation by proving the > assertion > 2 above. I will use the lower left figure for > reference. For the > convinience, the following notations are used: > > h = h_a/2, m^2 = m_b^2 - h^2 (compare with 2c above). > > > In what follows, I assume the cartesian coordinates > are used, and > will use the following basic statements about straight > lines and > parabolas: > > T1. The line passing through points (x0, y0) and (x1, > y1), has a > slope L = (y1-y0)/(x1-x0). > > T2. If a line has a slope L, its perpendicular has a > slope -1/L. > > T3. The line with a slope L and passing through point > (x0, y0), > has an equation y-y0 = L*(x-x0). > > T4. Parabola is a locus of points equidistant from a > given point F > (focus) and a given line d (directrix). > > T5. If the parabola is given by an equation y-y0 = > p*(x-x0)^2, its > directrix d is parallel to x-axis; focus F is (x0, > y0+1/4p), > and the distance from F to d is 1/2p. > > Let the cartesian coordinates be chosen so that Ha is > at the origin, > and A is on y-axis. Then, BC lies on x-axis. Assume B > at (2*b, 0). > Then, it is straightforward to see that the important > points have > the following coordinates: > > Ha(0, 0); A(0, 2*h); B(2*b, 0); Mb(2*b+m, h); > Mc(b, h). > > The circumcenter O is an intersection of perpendicular > bisectors > of AB and AC. Designate them as lb and lc accordingly; > they pass > through the points Mb and Mc (see drawing). Applying > T1-T3, we > get the following equations: > > lb: y-h = (2*b+m)/h * (x - 2*b-m) > (1) > lc: y-h = b/h * (x-b) > (2) > > Solving linear system (1)-(2), we get the following > formulas for > coordinates xO, yO of circumcenter O for a specific b: > > xO = 3*b + m > yO = b/h * (2*b + m) > (3) > > The above expressions suggest that xO, yO satisfy the > following > relation: > y = p*x^2 + q*x + r, > (4) > > where p, q, r do not depend on b. Indeed, plugging (3) > into (4), > we get: > > p = 2/(9*h), q = -m/(9*h), r = h - > m^2/(9*h). (5) > > Rearranging the terms in (4) with (5), we get: > > y-y0 = p*(x-x0)^2, > > where x0 = -p/2q = m/4, y0 = r - q^2/4p = h - > m^2/(8*h). > > Applying T5, we conclude that the locus of O is indeed > parabola, > with directrix d parallel to line BC, and distant from > Ha: > > y0 - 1/4p = - (m^2 + h^2)/(8*h) = - > m_b^2/(4*h_a). > > The focus F is 1/2p = 9/8*h_a far away from d. Q.E.D. > > Let me stop here. I will write my comments about the > solution in > a separate mail. And yours are always welcome... > > Best regards, > Barukh. > ========================================================================= Instruções para entrar na lista, sair da lista e usar a lista em http://www.mat.puc-rio.br/~nicolau/olimp/obm-l.html O administrador desta lista é ========================================================================= From owner-obm-l@sucuri.mat.puc-rio.br Sun May 19 14:33:49 2002 Return-Path: Received: (from majordom@localhost) by sucuri.mat.puc-rio.br (8.9.3/8.9.3) id OAA07662 for obm-l-list; Sun, 19 May 2002 14:32:10 -0300 Received: from imo-d08.mx.aol.com (imo-d08.mx.aol.com [205.188.157.40]) by sucuri.mat.puc-rio.br (8.9.3/8.9.3) with ESMTP id OAA07659 for ; Sun, 19 May 2002 14:32:07 -0300 From: Lltmdrtm@aol.com Received: from Lltmdrtm@aol.com by imo-d08.mx.aol.com (mail_out_v32.5.) id z.15a.e0be79d (4185) for ; Sat, 18 May 2002 23:35:03 -0400 (EDT) Message-ID: <15a.e0be79d.2a187766@aol.com> Date: Sat, 18 May 2002 23:35:02 EDT Subject: [obm-l] esclarecimento To: obm-l@mat.puc-rio.br MIME-Version: 1.0 Content-Type: multipart/alternative; boundary="part1_15a.e0be79d.2a187766_boundary" X-Mailer: AOL 7.0 for Windows BR sub 10501 Sender: owner-obm-l@sucuri.mat.puc-rio.br Precedence: bulk Reply-To: obm-l@mat.puc-rio.br --part1_15a.e0be79d.2a187766_boundary Content-Type: text/plain; charset="ISO-8859-1" Content-Transfer-Encoding: quoted-printable O que s=E3o as tritangentes? --part1_15a.e0be79d.2a187766_boundary Content-Type: text/html; charset="ISO-8859-1" Content-Transfer-Encoding: quoted-printable O que s=E3o as tritangentes? --part1_15a.e0be79d.2a187766_boundary-- ========================================================================= Instruções para entrar na lista, sair da lista e usar a lista em http://www.mat.puc-rio.br/~nicolau/olimp/obm-l.html O administrador desta lista é ========================================================================= From owner-obm-l@sucuri.mat.puc-rio.br Sun May 19 15:09:36 2002 Return-Path: Received: (from majordom@localhost) by sucuri.mat.puc-rio.br (8.9.3/8.9.3) id PAA08132 for obm-l-list; Sun, 19 May 2002 15:08:10 -0300 Received: from traven10.uol.com.br (traven10.uol.com.br [200.231.206.211]) by sucuri.mat.puc-rio.br (8.9.3/8.9.3) with ESMTP id PAA08129 for ; Sun, 19 May 2002 15:08:08 -0300 Received: from oemcomputer ([200.227.64.153]) by traven10.uol.com.br (8.9.1/8.9.1) with SMTP id OAA24528 for ; Sun, 19 May 2002 14:56:08 -0300 (BRT) Message-ID: <00f001c1ff5e$85a0c840$a741e3c8@oemcomputer> From: "Paulo Rodrigues" To: References: Subject: [obm-l] =?iso-8859-1?Q?Re:_=5Bobm-l=5D_Re:_Sob_que_condi=E7oes_uma_deformacao_pre?= =?iso-8859-1?Q?serva_medidas?= Date: Sun, 19 May 2002 14:56:14 -0300 MIME-Version: 1.0 Content-Type: multipart/alternative; boundary="----=_NextPart_000_00ED_01C1FF45.52C5B700" X-Priority: 3 X-MSMail-Priority: Normal X-Mailer: Microsoft Outlook Express 5.00.2615.200 X-MimeOLE: Produced By Microsoft MimeOLE V5.00.2615.200 Sender: owner-obm-l@sucuri.mat.puc-rio.br Precedence: bulk Reply-To: obm-l@mat.puc-rio.br This is a multi-part message in MIME format. ------=_NextPart_000_00ED_01C1FF45.52C5B700 Content-Type: text/plain; charset="iso-8859-1" Content-Transfer-Encoding: quoted-printable : Caros Nicolau e demais membros, :=20 :=20 :=20 : Faz um certo tempo o Nicolau mandou um e-mail que tinha o paragrafo : abaixo. Ocorre que eu li isso em uma superinteressante quando estava = na : escola e ate hoje tenho isso na cabeca, nao sabia se tinha sonhado, ou = se : era besteira, etc. Se alguem souber qual e a refererencia onde isso = foi : provado, ou pelo menos quem provou, ia me ajudar muito. Pelo que eu me : lembro, na revista falava-se algo em torno de 2^50 pedacos... :=20 :=20 : Abraco a todos, :=20 :=20 : Salvador :=20 : On Sun, 4 Feb 2001, Nicolau C. Saldanha wrote: :=20 :=20 : > Ali=E1s um grande problema da matem=E1tica do s=E9culo XX foi o da = quadratura=20 : > do c=EDrculo: n=E3o aquele proposto pelos gregos e cuja = demostra=E7=E3o foi : > conclu=EDda com a prova da transcend=EAncia de pi. O problema = s=E9culo XX : > da quadratura do c=EDrculo =E9: ser=E1 poss=EDvel decompor um = c=EDrculo de =E1rea 1 : > em um n=FAmero finito de pe=E7as e rearrum=E1-las para formar um = quadrado : > de =E1rea 1? A resposta =E9 que sim, =E9 poss=EDvel. : >=20 : > []s, N. : >=20 : >=20 :=20 Isto foi provado por Miklos Laczkovich: M. Laczkovich, Equidecomposability and discrepancy; a solution of = Tarski's circle-squaring problem, Journal f=FCr die Reine und Angewandte = Mathematik, 403 (1990) 77-117 Veja tamb=E9m, R. J. Gardner and S. Wagon, At long last, the circle has been squared, = Notices of the American Mathematical Society, 36 (1989) - 1338-1343 =20 --- esta mensagem n=E3o cont=E9m v=EDrus! Checked by AVG anti-virus system (http://www.grisoft.com). Version: 6.0.351 / Virus Database: 197 - Release Date: 20/04/2002 ------=_NextPart_000_00ED_01C1FF45.52C5B700 Content-Type: text/html; charset="iso-8859-1" Content-Transfer-Encoding: quoted-printable
: Caros Nicolau e demais = membros,
:
:=20
:
: Faz um certo tempo o Nicolau mandou um e-mail que tinha o=20 paragrafo
: abaixo. Ocorre que eu li isso em uma superinteressante = quando=20 estava na
: escola e ate hoje tenho isso na cabeca, nao sabia se = tinha=20 sonhado, ou se
: era besteira, etc. Se alguem souber qual e a = refererencia=20 onde isso foi
: provado, ou pelo menos quem provou, ia me ajudar = muito. Pelo=20 que eu me
: lembro, na revista falava-se algo em torno de 2^50=20 pedacos...
:
:
: Abraco a todos,
:
:
: = Salvador
:
:=20 On Sun, 4 Feb 2001, Nicolau C. Saldanha wrote:
:
:
: > = Ali=E1s um=20 grande problema da matem=E1tica do s=E9culo XX foi o da quadratura
: = > do=20 c=EDrculo: n=E3o aquele proposto pelos gregos e cuja demostra=E7=E3o = foi
: >=20 conclu=EDda com a prova da transcend=EAncia de pi. O problema s=E9culo = XX
: > da=20 quadratura do c=EDrculo =E9: ser=E1 poss=EDvel decompor um c=EDrculo de = =E1rea 1
: >=20 em um n=FAmero finito de pe=E7as e rearrum=E1-las para formar um = quadrado
: > de=20 =E1rea 1? A resposta =E9 que sim, =E9 poss=EDvel.
: >
: > = []s, N.
: >=20
: >

Isto foi provado por  Miklos=20 Laczkovich:
 
M. Laczkovich, Equidecomposability = and=20 discrepancy; a solution of Tarski's circle-squaring problem, Journal = f=FCr die=20 Reine und Angewandte Mathematik, 403 (1990) = 77-117
 
Veja tamb=E9m,
 
R. J. Gardner and S. Wagon, At = long last,=20 the circle has been squared, Notices of the American Mathematical=20 Society, 36 (1989) - 1338-1343
 
 
 

---
esta mensagem n=E3o = cont=E9m=20 v=EDrus!
Checked by AVG anti-virus system (http://www.grisoft.com).
Version: = 6.0.351 /=20 Virus Database: 197 - Release Date: = 20/04/2002
------=_NextPart_000_00ED_01C1FF45.52C5B700-- ========================================================================= Instruções para entrar na lista, sair da lista e usar a lista em http://www.mat.puc-rio.br/~nicolau/olimp/obm-l.html O administrador desta lista é ========================================================================= From owner-obm-l@sucuri.mat.puc-rio.br Sun May 19 20:43:13 2002 Return-Path: Received: (from majordom@localhost) by sucuri.mat.puc-rio.br (8.9.3/8.9.3) id UAA09448 for obm-l-list; Sun, 19 May 2002 20:41:32 -0300 Received: from smtp.ieg.com.br (194.125.226.200.in-addr.arpa.ig.com.br [200.226.125.194] (may be forged)) by sucuri.mat.puc-rio.br (8.9.3/8.9.3) with ESMTP id UAA09445 for ; Sun, 19 May 2002 20:41:30 -0300 Received: from macacos (200-161-154-181.dsl.telesp.net.br [200.161.154.181]) by smtp.ieg.com.br (IeG relay/8.9.3) with SMTP id g4JNQ854078497 for ; Sun, 19 May 2002 20:26:08 -0300 (BRT) Message-ID: <001901c1ff8d$0cc08620$0200a8c0@macacos> From: =?iso-8859-1?Q?Gabriel_P=E9rgola?= To: References: Subject: [obm-l] Re: Date: Sun, 19 May 2002 20:29:40 -0300 MIME-Version: 1.0 Content-Type: text/plain; charset="iso-8859-1" Content-Transfer-Encoding: 8bit X-Priority: 3 X-MSMail-Priority: Normal X-Mailer: Microsoft Outlook Express 5.50.4807.1700 X-MimeOLE: Produced By Microsoft MimeOLE V5.50.4807.1700 Sender: owner-obm-l@sucuri.mat.puc-rio.br Precedence: bulk Reply-To: obm-l@mat.puc-rio.br As distancias vao ser iguais. Prova-se isso atraves da equacao: C=2*PI*R ----- Original Message ----- From: "Adherbal Rocha Filho" To: Sent: Sunday, May 19, 2002 8:51 AM > > mais um probleminha: > suponha q a Terra eh uma esfera e que uma corda está amarrada ao redor da > linha do equador.Agora suponha que esta corda eh aumentada em um metro > ,formando uma circunferencia maior,qual será a distancia entre a superficie > da Terra e a corda? E se eu fizesse o mesmo pra uma bola de futebol,qual > seria a distancia? > > valeu! > []´s > Adherbal > > > _________________________________________________________________ > Envie e receba emails com o Hotmail no seu dispositivo móvel: > http://mobile.msn.com > > ========================================================================= > Instruções para entrar na lista, sair da lista e usar a lista em > http://www.mat.puc-rio.br/~nicolau/olimp/obm-l.html > O administrador desta lista é > ========================================================================= ========================================================================= Instruções para entrar na lista, sair da lista e usar a lista em http://www.mat.puc-rio.br/~nicolau/olimp/obm-l.html O administrador desta lista é ========================================================================= From owner-obm-l@sucuri.mat.puc-rio.br Mon May 20 01:05:26 2002 Return-Path: Received: (from majordom@localhost) by sucuri.mat.puc-rio.br (8.9.3/8.9.3) id BAA10677 for obm-l-list; Mon, 20 May 2002 01:03:43 -0300 Received: from web10107.mail.yahoo.com (web10107.mail.yahoo.com [216.136.130.57]) by sucuri.mat.puc-rio.br (8.9.3/8.9.3) with SMTP id BAA10674 for ; Mon, 20 May 2002 01:03:40 -0300 Message-ID: <20020520035137.46986.qmail@web10107.mail.yahoo.com> Received: from [150.161.199.55] by web10107.mail.yahoo.com via HTTP; Sun, 19 May 2002 20:51:37 PDT Date: Sun, 19 May 2002 20:51:37 -0700 (PDT) From: Rafael WC Subject: Re: [obm-l] Exercicios - Olimpiada. To: OBM MIME-Version: 1.0 Content-Type: text/plain; charset=us-ascii Sender: owner-obm-l@sucuri.mat.puc-rio.br Precedence: bulk Reply-To: obm-l@mat.puc-rio.br > >Devemos encontrar "a" e "b" inteiros nao-negativos. > Como > >(10a + b)*(a+b) = a^3 + b^3 > >posso FATORAR o segundo membro assim : > >a^3 + b^3 =(a+b)*(a^2- ab + b^2) > >colocando essa fatoracao na primeira equacao, fica > : > >(10a + b)*(a+b) = (a+b)*(a^2 -ab + b^2) > >como a + b > 0 (pois 00=0 nao e um numero de dois > digitos ) posso dividir > >tudo por a+b. Vai ficar : > > > >10a + b = a^2 - ab + b^2 > > > >reduzindo os termos semelhantes > > > >a^2 - (10 - b)*a + b^2 - b=0 > > > >E isto e UMA EQUACAO DO 2 GRAU LITERAL em "a". O > Discriminante e : > > > >(10 - b)^2 - 4*(b^2 - b) > > > >Simplificando fica : -3b^2 - 16b + 100 = 0 > > > >Como b varia de 0 ate 9, posso verificar os casos > em que o discriminante e > >um quadrado perfeito e que implicam num "a" > inteiro positivo. Isso vai me > >fornecer a quantidade de numeros que satisfazem o > problema. Oi Pessoal! Na curiosidade, vou achar os tais números e percebi que tinha um pequeno equívoco na resolução, só um sinalzinho de mais trocado por um de menos. Segue a correção e já aproveito para mandar as respostas: 1) Considere os números formados por 2 dígitos tais que a multiplicacao do número formado por a e b pela soma dos dígitos seja igual a soma do cubo dos digitos. Quantos e quais são esses números ? Devemos encontrar "a" e "b" inteiros nao-negativos. Como (10a + b)*(a+b) = a^3 + b^3 posso FATORAR o segundo membro assim : a^3 + b^3 =(a+b)*(a^2- ab + b^2) colocando essa fatoracao na primeira equacao, fica : (10a + b)*(a+b) = (a+b)*(a^2 -ab + b^2) como a + b > 0 (pois 00=0 nao e um numero de dois digitos ) posso dividir tudo por a+b. Vai ficar : 10a + b = a² - ab + b² reduzindo os termos semelhantes a² - (10 + b).a + b² - b = 0 **(o sinal foi trocado aqui acima, no 10 + b)** E isto e UMA EQUACAO DO 2 GRAU LITERAL em "a". O Discriminante e : (10 + b)² - 4*(b² - b) Simplificando fica : -3b² + 24b + 100 Como b varia de 0 ate 9, posso verificar os casos em que o discriminante e um quadrado perfeito e que implicam num "a" inteiro positivo. Isso vai me fornecer a quantidade de numeros que satisfazem o problema. b = 1, 7, 8 b = 1 = -3b² + 24b + 100 = -3 + 24 + 100 = 121 a² - (10 + b).a + b² - b = 0 a² - (10 + 1).a + 1 - 1 = 0 a² - 11a = 0 a = 0, 11 b = 7 = -3b² + 24b + 100 = -3.7² + 24.7 + 100 = 121 a² - (10 + b).a + b² - b = 0 a² - (10 + 7).a + 7² - 7 = 0 a² - 17a + 42 = 0 a = 3, 14 b = 8 = -3b² + 24b + 100 = -3.8² + 24.8 + 100 = 100 a² - (10 + b).a + b² - b = 0 a² - (10 + 8).a + 8² - 8 = 0 a² - 18a + 56 = 0 a = 4, 14 Resposta: 37, 48 Um abraço, Rafael. ===== Rafael Werneck Cinoto ICQ# 107011599 rwcinoto@yahoo.com rafael.caixa@gov.com.br matduvidas@yahoo.com.br http://www.rwcinoto.hpg.com.br/ __________________________________________________ Do You Yahoo!? LAUNCH - Your Yahoo! Music Experience http://launch.yahoo.com ========================================================================= Instruções para entrar na lista, sair da lista e usar a lista em http://www.mat.puc-rio.br/~nicolau/olimp/obm-l.html O administrador desta lista é ========================================================================= From owner-obm-l@sucuri.mat.puc-rio.br Mon May 20 08:30:46 2002 Return-Path: Received: (from majordom@localhost) by sucuri.mat.puc-rio.br (8.9.3/8.9.3) id IAA12085 for obm-l-list; Mon, 20 May 2002 08:30:32 -0300 Received: from hotmail.com (oe20.law7.hotmail.com [216.33.236.124]) by sucuri.mat.puc-rio.br (8.9.3/8.9.3) with ESMTP id IAA12082 for ; Mon, 20 May 2002 08:30:29 -0300 Received: from mail pickup service by hotmail.com with Microsoft SMTPSVC; Mon, 20 May 2002 04:18:27 -0700 X-Originating-IP: [200.241.240.154] From: "Marcelo Rufino de Oliveira" To: References: Subject: [obm-l] =?iso-8859-1?Q?Re:_=5Bobm-l=5D_invers=E3o/desigualdades/cone_sul?= Date: Mon, 20 May 2002 07:42:20 -0300 MIME-Version: 1.0 Content-Type: text/plain; charset="iso-8859-1" Content-Transfer-Encoding: 8bit X-Priority: 3 X-MSMail-Priority: Normal X-Mailer: Microsoft Outlook Express 5.00.2314.1300 X-MimeOLE: Produced By Microsoft MimeOLE V5.00.2314.1300 Message-ID: X-OriginalArrivalTime: 20 May 2002 11:18:27.0138 (UTC) FILETIME=[10281220:01C1FFF0] Sender: owner-obm-l@sucuri.mat.puc-rio.br Precedence: bulk Reply-To: obm-l@mat.puc-rio.br Acho que a terceira possui a seguinte solução > > 3.Seja p um real positivo dado.Achar o mínimo valor de x^3 +y^3 sabendo que > x e y são reais positivos tais que xy(x+y)=p P = x^3 + y^3 = (x + y)(x^2 + y^2 - xy) = (x + y)(x^2 + y^2) - (x + y)xy = (x + y)(x^2 + y^2) - p Pela desigualdades entre as médias aritmética e geométrica, temos que x^2 + y^2 >= 2xy Assim: P = x^3 + y^3 >= (x + y)(2xy) - p = 2p - p = p Assim, o valor mínimo de x^3 + y^3 é p. > > Obrigada! > []´s > Fê > Até mais, Marcelo Rufino de Oliveira ========================================================================= Instruções para entrar na lista, sair da lista e usar a lista em http://www.mat.puc-rio.br/~nicolau/olimp/obm-l.html O administrador desta lista é ========================================================================= From owner-obm-l@sucuri.mat.puc-rio.br Mon May 20 08:51:31 2002 Return-Path: Received: (from majordom@localhost) by sucuri.mat.puc-rio.br (8.9.3/8.9.3) id IAA12355 for obm-l-list; Mon, 20 May 2002 08:51:28 -0300 Received: from hotmail.com (oe39.law7.hotmail.com [216.33.236.34]) by sucuri.mat.puc-rio.br (8.9.3/8.9.3) with ESMTP id IAA12352 for ; Mon, 20 May 2002 08:51:25 -0300 Received: from mail pickup service by hotmail.com with Microsoft SMTPSVC; Mon, 20 May 2002 04:39:23 -0700 X-Originating-IP: [200.241.240.154] From: "Marcelo Rufino de Oliveira" To: References: <193.7362b7f.2a181e78@aol.com> Subject: Re: [obm-l] (nenhum assunto) Date: Mon, 20 May 2002 08:03:20 -0300 MIME-Version: 1.0 Content-Type: multipart/alternative; boundary="----=_NextPart_000_0029_01C1FFD4.CE83AC50" X-Priority: 3 X-MSMail-Priority: Normal X-Mailer: Microsoft Outlook Express 5.00.2314.1300 X-MimeOLE: Produced By Microsoft MimeOLE V5.00.2314.1300 Message-ID: X-OriginalArrivalTime: 20 May 2002 11:39:23.0221 (UTC) FILETIME=[FCD72050:01C1FFF2] Sender: owner-obm-l@sucuri.mat.puc-rio.br Precedence: bulk Reply-To: obm-l@mat.puc-rio.br This is a multi-part message in MIME format. ------=_NextPart_000_0029_01C1FFD4.CE83AC50 Content-Type: text/plain; charset="iso-8859-1" Content-Transfer-Encoding: quoted-printable Considere o podlin=F4mio P(x) =3D x^7 - 1, que possui as 7 seguintes = ra=EDzes complexas: z(k) =3D cos (2.k.pi/7) + i.sen (2.k.pi/7), k =3D 0, 1, 2, 3, 4, 5, 6 Como o coeficiente de x^6 em P(x) =E9 0 ent=E3o a soma das ra=EDzes de = P(x) =E9 0, implicando que: cos 0 + cos (2.pi/7) + cos (4.pi/7) + cos (6.pi/7) + cos (8.pi/7) + cos = (10.pi/7) + cos (12.pi/7) =3D 0 =20 Como 2.pi/7 + 12.pi/7 =3D 2.pi =3D> cos (12.pi/7) =3D cos (2.pi/7) Como 4.pi/7 + 10.pi/7 =3D 2.pi =3D> cos (10.pi/7) =3D cos (4.pi/7) = =3D - cos (3.pi/7) Como 6.pi/7 + 8.pi/7 =3D 2.pi =3D> cos (8.pi/7) =3D cos (6.pi/7) = =3D - cos (pi/7) Portanto: 1 + cos (2.pi/7) - cos (3.pi/7) - cos (pi/7) - cos (pi/7) - cos (3.pi/7) = + cos (2.pi/7) =3D 0 =3D> cos (pi/7) - cos (2.pi/7) + cos (3.pi/7) =3D 1/2 At=E9 mais, Marcelo Rufino de Oliveira ----- Original Message -----=20 From: DEOLIVEIRASOU@aol.com=20 To: obm-l@mat.puc-rio.br=20 Sent: Saturday, May 18, 2002 6:15 PM Subject: [obm-l] (nenhum assunto) (IMO-1963) PROVE QUE COS(PI/7)-COS(2PI/7)+COS(3PI/7)=3D1/2.COMECEI A = FAZER E FOI FICANDO GRANDE...CADA VEZ MAIOR...RISOS...ALGUEM CONSEGUE = ACHAR UM TRUQUIINHO AI?? VALEU! CROM=20 ------=_NextPart_000_0029_01C1FFD4.CE83AC50 Content-Type: text/html; charset="iso-8859-1" Content-Transfer-Encoding: quoted-printable
Considere o podlin=F4mio P(x) =3D = x^7 - 1, =20 que possui as 7 seguintes ra=EDzes complexas:
z(k) =3D cos (2.k.pi/7) + i.sen=20 (2.k.pi/7),   k =3D 0, 1, 2, 3, 4, 5, 6
 
Como o coeficiente de x^6 em P(x) = =E9 0 ent=E3o a=20 soma das ra=EDzes de P(x) =E9 0, implicando que:
 
cos 0 + cos (2.pi/7) + cos (4.pi/7) + = cos (6.pi/7)=20 + cos (8.pi/7) + cos (10.pi/7) + cos (12.pi/7) =3D 0   =
 
Como  2.pi/7 + 12.pi/7 =3D = 2.pi  =20 =3D>   cos (12.pi/7) =3D cos (2.pi/7)
Como  4.pi/7 + 10.pi/7 =3D = 2.pi  =20 =3D>   cos (10.pi/7) =3D cos (4.pi/7) =3D - cos = (3.pi/7)
Como  6.pi/7 + 8.pi/7 =3D = 2.pi   =3D>=20   cos (8.pi/7) =3D cos (6.pi/7) =3D - cos=20 (pi/7)
Portanto:
1 + cos (2.pi/7) - cos (3.pi/7) - = cos (pi/7) -=20 cos (pi/7) - cos (3.pi/7) + cos (2.pi/7) =3D 0   = =3D>
cos (pi/7) - cos (2.pi/7) + cos = (3.pi/7) =3D=20 1/2
 
 
At=E9 mais,
Marcelo Rufino de Oliveira
 
----- Original Message -----
From:=20 DEOLIVEIRASOU@aol.com
Sent: Saturday, May 18, 2002 = 6:15=20 PM
Subject: [obm-l] (nenhum = assunto)

(IMO-1963) PROVE QUE=20 COS(PI/7)-COS(2PI/7)+COS(3PI/7)=3D1/2.COMECEI A FAZER E FOI FICANDO=20 GRANDE...CADA VEZ MAIOR...RISOS...ALGUEM CONSEGUE ACHAR UM TRUQUIINHO=20 = AI??
           = ;            =              = = VALEU!
          &nb= sp;           &nbs= p;            = ;          =20 CROM
------=_NextPart_000_0029_01C1FFD4.CE83AC50-- ========================================================================= Instruções para entrar na lista, sair da lista e usar a lista em http://www.mat.puc-rio.br/~nicolau/olimp/obm-l.html O administrador desta lista é ========================================================================= From owner-obm-l@sucuri.mat.puc-rio.br Mon May 20 10:00:17 2002 Return-Path: Received: (from majordom@localhost) by sucuri.mat.puc-rio.br (8.9.3/8.9.3) id JAA13365 for obm-l-list; Mon, 20 May 2002 09:59:24 -0300 Received: from venus.ccet.ufrn.br (venus.ccet.ufrn.br [200.19.174.45]) by sucuri.mat.puc-rio.br (8.9.3/8.9.3) with ESMTP id JAA13362 for ; Mon, 20 May 2002 09:59:21 -0300 Received: from scheelita (sheelita.ccet.ufrn.br [10.9.0.137]) by venus.ccet.ufrn.br (Postfix) with SMTP id 34183451E69 for ; Mon, 20 May 2002 09:59:31 -0300 (BRT) Message-ID: <00aa01c0e12a$8af59440$8900090a@cceta.ufrn.br> From: "bene" To: References: Subject: [obm-l] Exercicios - Olimpiada. Date: Sun, 20 May 2001 09:44:00 -0300 MIME-Version: 1.0 Content-Type: text/plain; charset="iso-8859-1" Content-Transfer-Encoding: 8bit X-Priority: 3 X-MSMail-Priority: Normal X-Mailer: Microsoft Outlook Express 5.00.2615.200 X-MimeOLE: Produced By Microsoft MimeOLE V5.00.2615.200 Sender: owner-obm-l@sucuri.mat.puc-rio.br Precedence: bulk Reply-To: obm-l@mat.puc-rio.br Um problema de Olimpíadas Argentinas: Escreve-se um número número natural em cada vértice de um cubo, sendo todos os oito números distintos. Em seguida, em cada aresta escreve-se o Máximo Divisor Comum dos números dos vértice correspondentes. É possível escolher os oito números iniciais de maneira que sua soma seja igual a soma dos 12 números das arestas? ========================================================================= Instruções para entrar na lista, sair da lista e usar a lista em http://www.mat.puc-rio.br/~nicolau/olimp/obm-l.html O administrador desta lista é ========================================================================= From owner-obm-l@sucuri.mat.puc-rio.br Mon May 20 11:03:56 2002 Return-Path: Received: (from majordom@localhost) by sucuri.mat.puc-rio.br (8.9.3/8.9.3) id LAA14179 for obm-l-list; Mon, 20 May 2002 11:01:10 -0300 Received: from ns-3.idc.dglnet.com.br (ns-3.idc.dglnet.com.br [200.218.161.4]) by sucuri.mat.puc-rio.br (8.9.3/8.9.3) with ESMTP id LAA14176 for ; Mon, 20 May 2002 11:01:07 -0300 Received: from user (200-204-37-238.dial-up.telesp.net.br [200.204.37.238]) by ns-3.idc.dglnet.com.br (Postfix) with SMTP id F1B20150021 for ; Mon, 20 May 2002 10:49:04 -0300 (BRT) Message-ID: <005301c20005$231c7820$ee25ccc8@user> From: "Claudio" To: References: Subject: [obm-l] =?iso-8859-1?Q?Re:_=5Bobm-l=5D_invers=E3o/desigualdades/cone_sul?= Date: Mon, 20 May 2002 10:49:14 -0300 MIME-Version: 1.0 Content-Type: text/plain; charset="iso-8859-1" Content-Transfer-Encoding: 8bit X-Priority: 3 X-MSMail-Priority: Normal X-Mailer: Microsoft Outlook Express 5.00.2314.1300 X-MimeOLE: Produced By Microsoft MimeOLE V5.00.2314.1300 Sender: owner-obm-l@sucuri.mat.puc-rio.br Precedence: bulk Reply-To: obm-l@mat.puc-rio.br Olá Fernanda! Veja se a seguinte idéia funciona para o 2º. Ponhamos BC=a, AC=b e AB=c, P um ponto interior, P(x,y,z) onde x é a distância de P até BC, etc. Agora ax=by=cz=2A (o dobro da área do triângulo ABC). A expressão a/x + b/y + c/z = 2A [1/x^2 + 1/y^2 + 1/z^2 ]. Fixe z. Suponha xy. Em qualquer caso é possível diminuir o valor da expressão inicial tornando x=y. Então devemos ter x=y. Idem para z. Se x=y=z então P é o incentro. Verifique. Um abraço. Claudio Casemiro. ----- Original Message ----- From: Fernanda Medeiros To: Sent: Sunday, May 19, 2002 8:36 AM Subject: [obm-l] inversão/desigualdades/cone sul > > Oi pessoal, alguém poderia me ajudar nessas 2 questões? Bem,aé vão: > 1.Sejam a,c,d e d os lados consecutivos de um quadrilátero ABCD e x e y as > suas diagonais.Suponha que os círculos circunscritos aos triangulos ABC e > ACD são ortogonais.Mostre que (x^2)(y^2)=(a^2)(c^2) + (b^2)(d^2) > > 2.Seja P um ponto no interior de um triangulo e sejam ha,hb e hc as > distancias de P aos lados a,b e c,respectivamente.Mostre q o valor mínimo de > a/ha +b/hb +c/hc ocorre quando P é o incentro de ABC. > > 3.Seja p um real positivo dado.Achar o mínimo valor de x^3 +y^3 sabendo que > x e y são reais positivos tais que xy(x+y)=p > > Obrigada! > []´s > Fê > > > _________________________________________________________________ > Converse com amigos on-line, conheça o MSN Messenger: > http://messenger.msn.com > > ========================================================================= > Instruções para entrar na lista, sair da lista e usar a lista em > http://www.mat.puc-rio.br/~nicolau/olimp/obm-l.html > O administrador desta lista é > ========================================================================= > ========================================================================= Instruções para entrar na lista, sair da lista e usar a lista em http://www.mat.puc-rio.br/~nicolau/olimp/obm-l.html O administrador desta lista é ========================================================================= From owner-obm-l@sucuri.mat.puc-rio.br Mon May 20 11:59:42 2002 Return-Path: Received: (from majordom@localhost) by sucuri.mat.puc-rio.br (8.9.3/8.9.3) id LAA15345 for obm-l-list; Mon, 20 May 2002 11:59:28 -0300 Received: from calhau.terra.com.br (calhau.terra.com.br [200.176.3.20]) by sucuri.mat.puc-rio.br (8.9.3/8.9.3) with ESMTP id LAA15341 for ; Mon, 20 May 2002 11:59:26 -0300 Received: from mucuri.terra.com.br (mucuri.terra.com.br [200.176.3.39]) by calhau.terra.com.br (Postfix) with ESMTP id 1E78A47898 for ; Mon, 20 May 2002 14:47:25 +0000 (GMT) Received: from nt (RJ231244.user.veloxzone.com.br [200.165.231.244]) (authenticated user ensr) by mucuri.terra.com.br (Postfix) with ESMTP id E255C92FAA for ; Mon, 20 May 2002 11:47:23 -0300 (EST) Message-ID: <009c01c2000d$03c03c20$5400a8c0@ensrbr> From: "Luis Lopes" To: References: <001801c1fb4c$540cdd20$9e40b1c8@Lucelindo> Subject: Re: [obm-l] algebrismos Date: Mon, 20 May 2002 11:45:40 -0300 MIME-Version: 1.0 Content-Type: multipart/alternative; boundary="----=_NextPart_000_0099_01C1FFF3.DDD2CC80" X-Priority: 3 X-MSMail-Priority: Normal X-Mailer: Microsoft Outlook Express 5.00.2615.200 X-MimeOLE: Produced By Microsoft MimeOLE V5.00.2615.200 Sender: owner-obm-l@sucuri.mat.puc-rio.br Precedence: bulk Reply-To: obm-l@mat.puc-rio.br This is a multi-part message in MIME format. ------=_NextPart_000_0099_01C1FFF3.DDD2CC80 Content-Type: text/plain; charset="iso-8859-1" Content-Transfer-Encoding: quoted-printable Sauda, c~oes, Segue uma solu=E7=E3o. []'s Luis >From: "ndergiades"=20 >Reply-To: Hyacinthos@yahoogroups.com >To: Hyacinthos@yahoogroups.com >Subject: Re: [EMHL] S and more >Date: Sat, 18 May 2002 23:53:38 -0000 > >Dear Luvs, > >You wrote: > > >And this one, taken from another list: > > > >Prove que (Show that, standard notation) > > > >(a^2 + b^2)/2m_c + (b^2 + c^2)/2m_a + (a^2 + c^2)/2m_b <=3D >6R > >****** >If m1, m2, m3 are the medians of ABC then your inequality >is written >(bb+cc)/2m1 + (cc+aa)/2m2 + (aa+bb)/2m3 <=3D 6R. > >From 9RR - (aa + bb + cc) =3D OH^2 we get >aa + bb + cc <=3D 9RR >Since (x + y + z )^2 <=3D 3(xx + yy + zz) >for the given inequality it is sufficient to prove that >3(bb + cc)^2 / (2(bb + cc) - aa) + cyclic ... <=3D 36(aa + bb >+ cc)/9 >or after calculations that > >4aabbcc -4(bb+cc-aa)(cc+aa-bb)(aa+bb-cc)+(ab-ac)^2+ >(bc-ba)^2+(ca-cb)^2>=3D0 >which is true because e.g. >a^4 >=3D a^4 - (bb-cc)^2 or >a^4 >=3D (aa+bb-cc)(cc+aa-bb). > >Another nice inequality is >1/m1 + 1/m2 + 1/m3 >=3D 2/R >Does anybody has a reference for this? > >Best regards >Nikos Dergiades >From: "ndergiades"=20 >Reply-To: Hyacinthos@yahoogroups.com >To: Hyacinthos@yahoogroups.com >Subject: Re: [EMHL] S and more >Date: Sun, 19 May 2002 09:23:55 -0000 > > >Sorry, a correction, > > > > ****** > > If m1, m2, m3 are the medians of ABC then your inequality > > is written > > (bb+cc)/2m1 + (cc+aa)/2m2 + (aa+bb)/2m3 <=3D 6R. > > > > From 9RR - (aa + bb + cc) =3D OH^2 we get > > aa + bb + cc <=3D 9RR > > Since (x + y + z )^2 <=3D 3(xx + yy + zz) > > for the given inequality it is sufficient to prove that > > 3(bb + cc)^2 / (2(bb + cc) - aa) + cyclic ... <=3D 36(aa + bb > > + cc)/9 > > or after calculations that > > > > 4aabbcc -4(bb+cc-aa)(cc+aa-bb)(aa+bb-cc)+(ab-ac)^2+ > > (bc-ba)^2+(ca-cb)^2>=3D0 > >the correct is >4aabbcc -4(bb+cc-aa)(cc+aa-bb)(aa+bb-cc)+ >(abb-acc)^2+(bcc-baa)^2+(caa-cbb)^2>=3D0 > > > > which is true because e.g. > > a^4 >=3D a^4 - (bb-cc)^2 or > > a^4 >=3D (aa+bb-cc)(cc+aa-bb). > > > > Another nice inequality is > > 1/m1 + 1/m2 + 1/m3 >=3D 2/R > >Does anybody has a reference for this? > > Best regards > Nikos Dergiades > -----Mensagem Original-----=20 De: Lucelindo D. Ferreira=20 Para: obm-l@mat.puc-rio.br=20 Enviada em: ter=E7a-feira, 14 de maio de 2002 10:36 Assunto: [obm-l] algebrismos Bom dia galera. Eu queria uma m=E3o nesse problema da olimp=EDada = russa. Eu come=E7ei a resolver... Prove que=20 a^2 + b^2 + b^2 + c^2 + a^2 + c^2 =3D< 6R 2mc 2ma 2mb Nota=E7=E3o: a,b e c s=E3o lados do tri=E2ngulo inscrito numa = circunfer=EAncia de raio R. ma, mb e mc s=E3o as medianas relativas a a,b e c. Pelo teorema isoperim=E9trico eu sei que 3Rsqrt3 >=3D a + b + c , 3R^3sqrt3 >=3D abc e 3R^2sqrt3 >=3D sqrt[(a = + b + c)(b + c - a)(a + b - c)(a + c - b)]=20 =20 a^2 + b^2 + b^2 + c^2 + a^2 + c^2 =3D< 6R 2mc 2ma 2mb sabendo que ma =3D 1/2sqrt[2(b^2 + c^2) - a^2] mb =3D 1/2sqrt[2(a^2 + c^2) - b^2] e mc =3D 1/2sqrt[2(b^2 + a^2) - = c^2]... Empaquei a=ED e gostaria de sugest=F5es para continuar a solu=E7=E3o=20 Agrade=E7o qualquer sugest=E3o ------=_NextPart_000_0099_01C1FFF3.DDD2CC80 Content-Type: text/html; charset="iso-8859-1" Content-Transfer-Encoding: quoted-printable
Sauda, c~oes,
 
Segue uma solu=E7=E3o.
 
[]'s
Luis
 

>From: "ndergiades"
>Reply-To: Hyacinthos@yahoogroups.com
>To: = Hyacinthos@yahoogroups.com

>Subject: Re:=20 [EMHL] S and more
>Date: Sat, 18 May 2002 23:53:38=20 -0000
>
>Dear Luvs,
>
>You = wrote:
>
>=20 >And this one, taken from another list:
> >
> = >Prove que=20 (Show that, standard notation)
> >
> >(a^2 + b^2)/2m_c = + (b^2=20 + c^2)/2m_a + (a^2 + c^2)/2m_b = <=3D
>6R
>
>******
>If=20 m1, m2, m3 are the medians of ABC then your inequality
>is=20 written
>(bb+cc)/2m1 + (cc+aa)/2m2 + (aa+bb)/2m3 <=3D=20 6R.
>
>From  9RR - (aa + bb + cc) =3D OH^2  we = get
>aa=20 + bb + cc <=3D 9RR
>Since (x + y + z )^2 <=3D 3(xx + yy + = zz)
>for=20 the given inequality it is sufficient to prove that
>3(bb + cc)^2 = / (2(bb=20 + cc) - aa) + cyclic ... <=3D 36(aa + bb
>+ cc)/9
>or = after=20 calculations that
>
>4aabbcc=20 -4(bb+cc-aa)(cc+aa-bb)(aa+bb-cc)+(ab-ac)^2+
>(bc-ba)^2+(ca-cb)^2>= ;=3D0
>which=20 is true because e.g.
>a^4 >=3D a^4 - (bb-cc)^2  = or
>a^4 >=3D=20 (aa+bb-cc)(cc+aa-bb).
>
>Another nice inequality = is
>1/m1 +=20 1/m2 + 1/m3 >=3D 2/R
>Does anybody has a reference for=20 this?
>
>Best regards
>Nikos = Dergiades
>From: "ndergiades"
>Reply-To: Hyacinthos@yahoogroups.com=
>To:=20 Hyacinthos@yahoogroups.com=
>Subject:=20 Re: [EMHL] S and more
>Date: Sun, 19 May 2002 09:23:55=20 -0000
>
>
>Sorry, a = correction,
>
>
> >=20 ******
> > If m1, m2, m3 are the medians of ABC then your=20 inequality
> > is written
> > (bb+cc)/2m1 + = (cc+aa)/2m2 +=20 (aa+bb)/2m3 <=3D 6R.
> >
> > From  9RR - (aa + = bb + cc)=20 =3D OH^2  we get
> > aa + bb + cc <=3D 9RR
> > = Since (x +=20 y + z )^2 <=3D 3(xx + yy + zz)
> > for the given inequality = it is=20 sufficient to prove that
> > 3(bb + cc)^2 / (2(bb + cc) - aa) + = cyclic=20 ... <=3D 36(aa + bb
> > + cc)/9
> > or after = calculations=20 that
> >
> > 4aabbcc=20 -4(bb+cc-aa)(cc+aa-bb)(aa+bb-cc)+(ab-ac)^2+
> >=20 (bc-ba)^2+(ca-cb)^2>=3D0
>
>the correct is
>4aabbcc = -4(bb+cc-aa)(cc+aa-bb)(aa+bb-cc)+
>(abb-acc)^2+(bcc-baa)^2+(caa-cbb= )^2>=3D0
>
>
>=20 > which is true because e.g.
> > a^4 >=3D a^4 - = (bb-cc)^2 =20 or
> > a^4 >=3D (aa+bb-cc)(cc+aa-bb).
> >
> = >=20 Another nice inequality is
> > 1/m1 + 1/m2 + 1/m3 >=3D=20 2/R
>
>Does anybody has a reference for = this?
>
> =20 Best regards
>  Nikos=20 Dergiades
>

-----Mensagem = Original-----
De: Lucelindo D.=20 Ferreira
Enviada em: ter=E7a-feira, 14 = de maio de=20 2002 10:36
Assunto: [obm-l] = algebrismos

Bom dia galera. Eu queria uma m=E3o = nesse problema=20 da olimp=EDada russa. Eu come=E7ei a resolver...
Prove que=20
a^2 + b^2 + b^2 + = c^2  +=20 a^2 + c^2 =3D< 6R
    =20 2mc        =20 = 2ma           &nbs= p; =20 2mb
Nota=E7=E3o: a,b e c s=E3o lados do tri=E2ngulo inscrito numa = circunfer=EAncia de=20 raio R.
ma, mb e mc s=E3o as medianas relativas a a,b e = c.
Pelo teorema isoperim=E9trico eu sei=20 que
3Rsqrt3 >=3D a + b + c , = 3R^3sqrt3 =20 >=3D abc e 3R^2sqrt3 >=3D sqrt[(a + b + c)(b + c - a)(a + b = - c)(a + c=20 - b)]
          &nbs= p;            = ;            =             &= nbsp;           &n= bsp;         =20
a^2 + b^2 + b^2 + = c^2  +=20 a^2 + c^2 =3D< 6R
    =20 2mc        =20 = 2ma           &nbs= p; =20 2mb
sabendo que ma =3D 1/2sqrt[2(b^2 + = c^2) -=20 a^2]
mb =3D 1/2sqrt[2(a^2 + c^2) - b^2] e = mc =3D=20 1/2sqrt[2(b^2 + a^2) - c^2]...
Empaquei a=ED e gostaria de = sugest=F5es para=20 continuar a solu=E7=E3o
Agrade=E7o qualquer=20 sugest=E3o
------=_NextPart_000_0099_01C1FFF3.DDD2CC80-- ========================================================================= Instruções para entrar na lista, sair da lista e usar a lista em http://www.mat.puc-rio.br/~nicolau/olimp/obm-l.html O administrador desta lista é ========================================================================= From owner-obm-l@sucuri.mat.puc-rio.br Mon May 20 12:04:17 2002 Return-Path: Received: (from majordom@localhost) by sucuri.mat.puc-rio.br (8.9.3/8.9.3) id MAA15417 for obm-l-list; Mon, 20 May 2002 12:03:58 -0300 Received: from sr1.terra.com.br (sr1.terra.com.br [200.176.3.16]) by sucuri.mat.puc-rio.br (8.9.3/8.9.3) with ESMTP id MAA15413 for ; Mon, 20 May 2002 12:03:56 -0300 Received: from mucuri.terra.com.br (mucuri.terra.com.br [200.176.3.39]) by sr1.terra.com.br (Postfix) with ESMTP id 757B36EF9F for ; Mon, 20 May 2002 11:51:52 -0300 (EST) Received: from nt (RJ231244.user.veloxzone.com.br [200.165.231.244]) (authenticated user ensr) by mucuri.terra.com.br (Postfix) with ESMTP id EC75792F5D for ; Mon, 20 May 2002 11:51:51 -0300 (EST) Message-ID: <00bf01c2000d$a37abce0$5400a8c0@ensrbr> From: "Luis Lopes" To: Subject: [obm-l] A property of X55 and X56 Date: Mon, 20 May 2002 11:50:08 -0300 MIME-Version: 1.0 Content-Type: text/plain; charset="iso-8859-1" Content-Transfer-Encoding: 8bit X-Priority: 3 X-MSMail-Priority: Normal X-Mailer: Microsoft Outlook Express 5.00.2615.200 X-MimeOLE: Produced By Microsoft MimeOLE V5.00.2615.200 Sender: owner-obm-l@sucuri.mat.puc-rio.br Precedence: bulk Reply-To: obm-l@mat.puc-rio.br Sauda,c~oes, A CRUX tem alguns arquivos públicos. Ir no site http://journals.cms.math.ca/cgi- bin/vault/public/view/CRUXv23n3/body/HTML/187?template=CRUX []'s Luis >From: "yiuatfauedu" >Reply-To: Hyacinthos@yahoogroups.com >To: Hyacinthos@yahoogroups.com >Subject: [EMHL] Re: A property of X55 and X56 >Date: Thu, 16 May 2002 22:33:47 -0000 > > >Dear Gilles and Edward: > >This was Problem 2137 of Crux Mathematicorum: > >http://journals.cms.math.ca/cgi- >bin/vault/public/view/CRUXv23n3/body/HTML/187?template=CRUX >Best regards >Sincerely >Paul > >--- In Hyacinthos@y..., Gilles Boutte wrote: > > Dear all Hyacinthists, > > > > Edward Brisse and I worked on the following problem: > > > > Let Ca, Cb, Cc be 3 circles, with the same radius, Ca tangent to AB >and > > AC, Cb tangent to BC and BA, Cc tangent to CA and CB. > > > > There are two such triad, for which the 3 circles are concurrent, >in X55 > > and X56 respectively. > > > > This result is knowm as "Berzsenyi Triple Circle Concurrency". > > > > We wrote a short note (in French) on this subject. You can download >it > > at http://g.boutte.free.fr/articles/020516.pdf > > > > Would anybody have references to supply us on this problem? > > > > Best regards, > > > > Gilles Boutte > ========================================================================= Instruções para entrar na lista, sair da lista e usar a lista em http://www.mat.puc-rio.br/~nicolau/olimp/obm-l.html O administrador desta lista é ========================================================================= From owner-obm-l@sucuri.mat.puc-rio.br Mon May 20 12:06:48 2002 Return-Path: Received: (from majordom@localhost) by sucuri.mat.puc-rio.br (8.9.3/8.9.3) id MAA15489 for obm-l-list; Mon, 20 May 2002 12:06:44 -0300 Received: from pina.terra.com.br (pina.terra.com.br [200.176.3.17]) by sucuri.mat.puc-rio.br (8.9.3/8.9.3) with ESMTP id MAA15485 for ; Mon, 20 May 2002 12:06:41 -0300 Received: from pacuiba.terra.com.br (pacuiba.terra.com.br [200.176.3.40]) by pina.terra.com.br (Postfix) with ESMTP id C0FFD52F9D for ; Mon, 20 May 2002 11:54:40 -0300 (EST) Received: from nt (RJ231244.user.veloxzone.com.br [200.165.231.244]) (authenticated user ensr) by pacuiba.terra.com.br (Postfix) with ESMTP id E95608017 for ; Mon, 20 May 2002 11:54:36 -0300 (EST) Message-ID: <00e601c2000e$06d723a0$5400a8c0@ensrbr> From: "Luis Lopes" To: Subject: [obm-l] Floor's Monthly problem Date: Mon, 20 May 2002 11:52:47 -0300 MIME-Version: 1.0 Content-Type: text/plain; charset="iso-8859-1" Content-Transfer-Encoding: 7bit X-Priority: 3 X-MSMail-Priority: Normal X-Mailer: Microsoft Outlook Express 5.00.2615.200 X-MimeOLE: Produced By Microsoft MimeOLE V5.00.2615.200 Sender: owner-obm-l@sucuri.mat.puc-rio.br Precedence: bulk Reply-To: obm-l@mat.puc-rio.br Sauda,c~oes, Mais um site de IMO. http://www.math.ust.hk/excalibur/vol_6_no_1.pdf []'s Luis >From: "fvlamoenwxs" >Reply-To: Hyacinthos@yahoogroups.com >To: Hyacinthos@yahoogroups.com >Subject: [EMHL] Re: Floor's Monthly problem >Date: Wed, 15 May 2002 11:25:57 -0000 > >Dear Hyacinthists, > > > Show that the circumcenters of the 6 triangles, into which a >triangle is divided by > > its medians, are concyclic. > > > > The editors give a nice synthetic proof about which I have a >question. > >Let me start by saying that I have not actually read the solution >published in AMM (If someone can send me text (or scan) I would be >pleased). When I submitted the proof, I included a synthetic proof. I >understand from Alexander that it has the same problem in reasoning, >so when I have a little bit more time, I will look again and see if I >can reproduce the exact reason why I concluded the proof was complete. > >Meanwhile, you can read a slightly different approach (published in >fact prematurely) in the Hong-Kong based journal "Excalibur": > >http://www.math.ust.hk/excalibur/vol_6_no_1.pdf > >Kind regards, >Sincerely, >Floor. ========================================================================= Instruções para entrar na lista, sair da lista e usar a lista em http://www.mat.puc-rio.br/~nicolau/olimp/obm-l.html O administrador desta lista é ========================================================================= From owner-obm-l@sucuri.mat.puc-rio.br Mon May 20 12:15:06 2002 Return-Path: Received: (from majordom@localhost) by sucuri.mat.puc-rio.br (8.9.3/8.9.3) id MAA15722 for obm-l-list; Mon, 20 May 2002 12:14:55 -0300 Received: from trex.centroin.com.br (trex.centroin.com.br [200.225.63.134]) by sucuri.mat.puc-rio.br (8.9.3/8.9.3) with ESMTP id MAA15687 for ; Mon, 20 May 2002 12:14:53 -0300 Received: from trex.centroin.com.br (localhost [127.0.0.1]) by trex.centroin.com.br (8.12.1/8.12.1) with ESMTP id g4KF38O8002579 for ; Mon, 20 May 2002 12:03:08 -0300 (EST) Received: (from morgado@localhost) by trex.centroin.com.br (8.12.1/8.12.1/Submit) id g4KF38Ng002574; Mon, 20 May 2002 12:03:08 -0300 (EST) Message-Id: <200205201503.g4KF38Ng002574@trex.centroin.com.br> Received: from 200.165.213.58 by trex.centroin.com.br (CIPWM versao 1.4C1) with HTTPS for ; Mon, 20 May 2002 12:03:08 -0300 (EST) Date: Mon, 20 May 2002 12:03:08 -0300 (EST) From: Augusto Cesar de Oliveira Morgado To: obm-l@mat.puc-rio.br Subject: Re: [obm-l] .......... MIME-Version: 1.0 X-Mailer: CentroIn Internet Provider WebMail v. 1.4C1 (http://www.centroin.com.br/) Content-Type: text/plain; charset="iso-8859-1" Content-Transfer-Encoding: 8bit X-MIME-Autoconverted: from quoted-printable to 8bit by sucuri.mat.puc-rio.br id MAA15718 Sender: owner-obm-l@sucuri.mat.puc-rio.br Precedence: bulk Reply-To: obm-l@mat.puc-rio.br chame sqrt(5-x) de y. teremos sqrt(5-x)= y e x=sqrt 5- y. daí, y^2 = 5-x e x^2 = 5-y (x e y maiores ou iguais a 0). Para resolver o sistema, basta subtrair. (y-x)(y+x) = (y-x). daí, y=x ou y+x=1 ................................. Em Thu, 16 May 2002 22:24:20 -0300, "rafaelc.l" disse: > > > > > > > > > > > > > > > > > Tá, mas pq sqrt(5-x)=x se x=sqrt 5- sqrt(5-x)? > > > > > > Me diz mais uma coisa: Não daria pra resolver esta > > > questão elevando os membros ao quadrado até tirar os > > > radicais e depois achando as raízes do polinômio por > > > girard? > > > > > > > É só substituir que vc pode observar que a > igualdade vale. > > Elevar ao quadrado requer muita força > bruta, pode tentar, se > > conseguir me avisa!!! > > > > > Vc tem razão, elevei tudo ao quadrado, depois achei > um polinômio de 4 grau. Apliquei girard e achei um > sistema de 4 equações e 4 incógnitas.Esgotei minha força > bruta como vc diz para isolar uma variavel e quando > consegui, achei o mesmo polinômio de 4 grau nessa > incógnita. > > > Mas eu ainda não entendi seu raciocínio. Substituir o q > e onde??? se vc puder me mostrar seu resultado todo, > ficarei grato..... > > > ========================================================== > > > =============== > > > > Instruções para entrar na lista, sair da lista e > usar a > > > lista em > > > > http://www.mat.puc-rio.br/~nicolau/olimp/obm-l.html > > > > O administrador desta lista é rio.br> > > > > > > > > ========================================================== > > > =============== > > > > > > > > > > > > > > __________________________________________________________ > ________________ > > > Quer ter seu próprio endereço na Internet? > > > Garanta já o seu e ainda ganhe cinco e-mails > personalizados. > > > DomíniosBOL - http://dominios.bol.com.br > > > > > > > > > > ========================================================== > =============== > > > Instruções para entrar na lista, sair da lista e usar > a lista em > > > http://www.mat.puc-rio.br/~nicolau/olimp/obm-l.html > > > O administrador desta lista é > > > > ========================================================== > =============== > > > > > > > > > > ========================================================== > =============== > > Instruções para entrar na lista, sair da lista e usar a > lista em > > http://www.mat.puc-rio.br/~nicolau/olimp/obm-l.html > > O administrador desta lista é > > > ========================================================== > =============== > > > > > __________________________________________________________________________ > Quer ter seu próprio endereço na Internet? > Garanta já o seu e ainda ganhe cinco e-mails personalizados. > DomíniosBOL - http://dominios.bol.com.br > > > ========================================================================= > Instruções para entrar na lista, sair da lista e usar a lista em > http://www.mat.puc-rio.br/~nicolau/olimp/obm-l.html > O administrador desta lista é > ========================================================================= > > ========================================================================= Instruções para entrar na lista, sair da lista e usar a lista em http://www.mat.puc-rio.br/~nicolau/olimp/obm-l.html O administrador desta lista é ========================================================================= From owner-obm-l@sucuri.mat.puc-rio.br Mon May 20 12:17:15 2002 Return-Path: Received: (from majordom@localhost) by sucuri.mat.puc-rio.br (8.9.3/8.9.3) id MAA15794 for obm-l-list; Mon, 20 May 2002 12:17:05 -0300 Received: from trex.centroin.com.br (trex.centroin.com.br [200.225.63.134]) by sucuri.mat.puc-rio.br (8.9.3/8.9.3) with ESMTP id MAA15790 for ; Mon, 20 May 2002 12:17:00 -0300 Received: from trex.centroin.com.br (localhost [127.0.0.1]) by trex.centroin.com.br (8.12.1/8.12.1) with ESMTP id g4KF5FO8003810 for ; Mon, 20 May 2002 12:05:15 -0300 (EST) Received: by trex.centroin.com.br (8.12.1/8.12.1/Submit) id g4KF5FKC003807; Mon, 20 May 2002 12:05:15 -0300 (EST) Message-Id: <200205201505.g4KF5FKC003807@trex.centroin.com.br> Received: from 200.165.213.58 by trex.centroin.com.br (CIPWM versao 1.4C1) with HTTPS for ; Mon, 20 May 2002 12:05:15 -0300 (EST) Date: Mon, 20 May 2002 12:05:15 -0300 (EST) From: Augusto Cesar de Oliveira Morgado To: obm-l@mat.puc-rio.br Subject: =?iso-8859-1?q?Re: [obm-l] permuta=E7=F5es circulares com repeti=E7=E3o?= MIME-Version: 1.0 X-Mailer: CentroIn Internet Provider WebMail v. 1.4C1 (http://www.centroin.com.br/) Content-Type: text/plain; charset="iso-8859-1" Content-Transfer-Encoding: 8bit X-MIME-Autoconverted: from quoted-printable to 8bit by sucuri.mat.puc-rio.br id MAA15791 Sender: owner-obm-l@sucuri.mat.puc-rio.br Precedence: bulk Reply-To: obm-l@mat.puc-rio.br Procure o Teorema de Moreau. Em Sat, 18 May 2002 18:51:57 -0700 (PDT), Rafael WC disse: > Estou com problemas para resolver esse exercício: > > "De quantas maneiras distintas podemos dispor ao longo > de um circulo, suposto fixo, 6 bolas brancas, 8 bolas > azuis, 16 bolas verdes e 24 bolas amarelas?" > O círculo fica fixo em nossa frente, mas as bolas > ficam livres para serem rotacionadas como em uma > catraca de bicicleta. > > Pra mim, entendi como sendo uma permutação circular > com repetição. Nunca estudei isso e o que pensei que > seria mais lógico não deu muito certo. Pensei que > faríamos as permutações com repetições e dividiríamos > pelo total de bolas por causa de ser circular. Em > alguns casos até que conferiu com a resposta, mas aí > coloquei um teste com apenas 4 bolas, duas brancas e > duas azuis. > > Se eu fosse fazer como pensei, seria: > PC4(2,2) = 4!/4.2!.2! = 3/2 > > Nem inteiro dá!!! > > Ao fazer escrevendo mesmo, vejo que só temos 6 > permutações: > 1 - AABB > 2 - ABAB > 3 - ABBA > 4 - BAAB > 5 - BABA > 6 - BBAA > > E destas, se considerarmos como circulares, vemos que > 1 = 3 = 4 = 6 e 2 = 5. O que nos dá apenas 2 > permutações. > > Alguém sabe como resolvo esse tipo de problema? O > único livro que tenho aqui sobre análise combinatória > (Introdução à Análise Combinatória; Santos, J. P. O.; > Mello, M. P.; Murari, I. T. C.; 2ª edição; Campinas, > SP; Ed. da Unicamp, 1988) que aliás é muito bom, não > fala sobre permutações circulares com combinação. > > Agradeço qualquer dica. > > Rafael. > > ===== > Rafael Werneck Cinoto > ICQ# 107011599 > rwcinoto@yahoo.com > rafael.caixa@gov.com.br > matduvidas@yahoo.com.br > http://www.rwcinoto.hpg.com.br/ > > __________________________________________________ > Do You Yahoo!? > LAUNCH - Your Yahoo! Music Experience > http://launch.yahoo.com > ========================================================================= > Instruções para entrar na lista, sair da lista e usar a lista em > http://www.mat.puc-rio.br/~nicolau/olimp/obm-l.html > O administrador desta lista é > ========================================================================= > > ========================================================================= Instruções para entrar na lista, sair da lista e usar a lista em http://www.mat.puc-rio.br/~nicolau/olimp/obm-l.html O administrador desta lista é ========================================================================= From owner-obm-l@sucuri.mat.puc-rio.br Mon May 20 12:42:37 2002 Return-Path: Received: (from majordom@localhost) by sucuri.mat.puc-rio.br (8.9.3/8.9.3) id MAA17162 for obm-l-list; Mon, 20 May 2002 12:42:26 -0300 Received: from hotmail.com (law2-f17.hotmail.com [216.32.181.17]) by sucuri.mat.puc-rio.br (8.9.3/8.9.3) with ESMTP id MAA17156 for ; Mon, 20 May 2002 12:42:22 -0300 Received: from mail pickup service by hotmail.com with Microsoft SMTPSVC; Mon, 20 May 2002 08:30:20 -0700 Received: from 32.94.119.254 by lw2fd.hotmail.msn.com with HTTP; Mon, 20 May 2002 15:30:20 GMT X-Originating-IP: [32.94.119.254] From: "Paulo Santa Rita" To: obm-l@mat.puc-rio.br Subject: [obm-l] =?iso-8859-1?B?UmU6IFtvYm0tbF0gdHJp4m5ndWxvcw==?= Date: Mon, 20 May 2002 15:30:20 +0000 Mime-Version: 1.0 Content-Type: text/plain; charset=iso-8859-1; format=flowed Message-ID: X-OriginalArrivalTime: 20 May 2002 15:30:20.0269 (UTC) FILETIME=[4048D9D0:01C20013] Sender: owner-obm-l@sucuri.mat.puc-rio.br Precedence: bulk Reply-To: obm-l@mat.puc-rio.br Ola Rafael e demais colegas desta lista, O problema a que voce se refere ja foi proposto ( pelo Grande e Saudoso amigo Bruno Leite ) e resolvido aqui na lista. Segue abaixo a solucao apresentada : Caro Bruno, Saudacoes ! Antes de abordarmos propriamente a questao que voce propos considero importante, para uma maior claridade na exposicao, fixar a notacao que usaremos. Assim, representaremos por : { N/R } o numero binomial de numerador N e denominador P, vale dizer: { N/P } = (N!)/(P! (N-P)!); Se N < R, { N/R } = 0 Si[1,R] F(i) o somatorio de F(i), "i" variando de 1 ate R. A solucao que obteremos esta dividida em duas partes. Na primeira, que representaremos por A(n), estarao computados todos os triangulos equilateros com vertice para cima. O total de triangulos com vertices para baixo sera representado por V(n). Nas duas funcoes "n" representa o comprimento do lado do triangulo inicial. Seja ABC um tringulo equilatero de lado "n" imaginado como se BC fosse a base e A o vertice. Entre A e B, no sentido de A para B, inserimos N-1 pontos D1, D2, ..., Dn-1 de forma que o lado AB fique dividido em N partes iguais, Por estes pontos tracamos N-1 paralelas a BC que interceptarao AC respectivamente nos pontos E1, E2, ..., En-1, Chamaremos de "base i" ao segmento DiEi. A base BC sera referenciado com "a base n". O total de triangulos equilateros com lado unitario e vertice para cima, que representaremos por T1, e facilmente computavel. Com efeito, sobre a base 1 ( D1E1) cabe um triangulo; sobre a base 2 cabem dois triangulos; sobre a base tres cabem tres triangulos ... sobre a base N cabem N triangulos. Isto e : T1 = 1 + 2 + 3 + ... + N = (N(N+1))/2 = { N+1/2 } O total de triangiulos equilateros com lado igual a 2 e vertice para cima ( T2 ) segue a mesma logica: sobre a base 2 ( D2E2 )cabe um triangulo; sobre a base 3 cabem dois triangulos; ...; sobre a base N cabem N-1 triangulos. Isto e: T2 = 1 + 2 + 3 + ... + (N-1) = ((N-1)N)/2 = { N/2 } No caso dos triangulos com lado 3 : T3 = 1 + 2 + 3 + ,,, + (N-2) = ((N-2)(N-1))/2 = { N-1/2 } E assim sucessivamente ... O que queremos e calcular T1 + T2 + ,,, + Tn. Ficara : T1+T2+T3+...+Tn={ N+1/2 }+{ N/2 }+{ N-1/2 }+...+{ 2/2 }={ N+2/3 } Portanto, o total de triangulos com vertices para cima, que convencionamos chamar A(n) e que e a primeira parte de nossa formula, e: A(n) = { N+2/3 } Precisamos agora calcular V(n), o total de triangulos equilateros com vertice para baixo. Seja P=[ N/2 ] "o maior inteiro que não supera N/2". Observe que se N for par, entao P=N/2. Se N for impar, P = (N-1)/2. A expressao E = N/2 - P = N/2 - [N/2], assume o valor zero se N e par. Se N for impar, E = 1/2 ( um meio ) Observe que num triangulo equilatero qualquer P e o lado do maior triangulo com vertice para baixo que cabe neste triangulo Quantos triangulo equilateros com lado P cabem no triangulo equilagtero com lado N ? 1 (sobre a base P=DpEp), se N e par ; 1+2 se P for impar ( 1 sobre a base P=DpEp e 2 sobre a base P+1=Dp+1Ep+1). E se o lado do triangulo for "P-1" ? 1 + 2 + 3 se N for par; 1+2+3+4 se N for impar. Esse "valor a mais" que o triangulo de lado impar tem chamarei de "Excesso" e representarei por E(p). Assim, um triangulo de lado impar K pode ser pensado como um de lado par "K-1" mais um Excesso E(k) Assim, sendo P=[N/2], "o maior inteiro que não supera N/2", fica : 1 ( Lado P) => se N e par: 1 ; se N e impar: 1+2 ( aqui E(1) = 2 ) 2 (Lado P-1) => se N e par: 1+2+3 ; se N e impar: 1+2+3+4 ( aqui E(2) = 4 ) 3 (Lado P-2) => : se N e par: 1+2+3+4+5; se N e impar: 1+2+3+4+5+6 ( aqui E(3) = 6) ................ P (Lado 1) => se N e par: 1+2+...+(2P-1); se N e impar: 1+2+...+(2P-1)+2P ( aqui E(p) = 2P ) Assim, se N e par não somamos o excesso. Se N e impar, somamos. Ora, a expressao E = ( N/2 - [N/2] ) = ( N/2 - P ) tem essas caracteristicas, sendo zero par N par e 1/2 para N impar. Logo, representando por S(n) = 1+2+3+...+N, tudo que obtivemos acima pode ser expresso por : K ( lado P-k+1) => 1+2+3+...+(2K-1) + 4K(N/2 - P), independente de N ser par ou impar !!!!!!!!!! A expressao de V(n), que representa todos os triangulos equilateros com vertice para baixo, sera: V(n) = S(1) + S(3) + S(5) + ... + S(2P-1) + 4(1+2+...+P)(N/2 - P) V(n) = S(1)+S(3)+S(5)+...+S(2p-1) + 4{ P+1/2 }( N/2 - P ) V(n) = { P/1 }*S(1) + { P/2 }*(S(3)-S(1)) + { P/3 }*(S(5) - 2S(3) + S(1)) + 4*{ P+1/2 }*(N/2 - P) Mas S(1) = 1, S(3) = 6 e S(5) = 15. Logo V(n) = { P/1 } + 5{ P/2 } + 4{ P/3 } + 4{ P+1/2 }( N/2 - P ) Nesta ultima expressao, conforme já dissemos, { N/R } representa o numero binomial (N!)/(P!(N-P)!) com { N/R } = 0 se N < R. N e o comprimento do lado do triangulo equilatero e P=[N/2] e o maior inteiro que não supera N/2. A expressao final que voce procura e A(n) + V(n), que chamaremos de T(n). Portanto : T(N)={ N+2/3 }+{ P/1 }+ 5*{ P/2 }+ 4*{ P/3 }+ 4*{ P+1/2 }*( N/2 - P ) Independente de N ser par ou impar ! E SEM PRECISAR USAR DUAS FORMULAS ! Finalmente, não sei se voce percebeu que um quadrado de lado N tambem permite enunciar uma questao semelhante. Neste figura não há quadrado menores invertidos, visto que a inversao de um quadrado gera um outro identico ao primeiro, mas há uma gama enorme de "quadrados inclinados" e eu já calculei que um raciocinio semelhante ao que desenvolvemos acima permite solucionar esta questao. Voce gostaria de tentar resolve-la ? >From: "Bruno Leite" >Reply-To: obm-l@mat.puc-rio.br >To: obm-l@mat.puc-rio.br >Subject: Problema horroroso >Date: Sun, 26 Sep 1999 10:58:34 PDT > >Seja f(n) o número de triângulos equiláteros (0triângulo equilátero de lado n (num triangulado, digamos) > >Ex: f(1)=1 pois num triângulo eq. de lado 1 há apenas um triângulo. > f(2)=5 pois num tr.eq. de lado 2 há um triangulo de ponta cabeça, três >tr. de lado 1 "certos" e um grandão. > f(3)=13 pois temos 9 pequenos, 3 médios e um grande. > f(4)=27 etc > /\ > /\/\ > /\/\/\ >/\/\/\/\ > > >A figura ilustra o caso n=4 (é preciso fingir que há também as divisões >horizontais, formando uma malha triangular.) > >É fácil ver que há: >16 triângulos do tipo /\ ou \/ (lado 1) >7 triângulos de lado 2 (6 /\ e 1 \ /) > / \ \/ >3 triângulos lado 3 e >1 de lado 4. > > >Pede-se f(n) em função de n (fórmula explícita) >Eu comecei a estudar esse problema há 2 anos mas sempre desisti por falta >de resultados. Já achei várias relações mas não acho a fórmula geral. >Gostaria MUITO que alguém falasse como se faz. > >Se alguém que se interessou não entender o enunciado muito bem, eu faço >umas figuras explicativas para ajudar. > >Bruno Leite > >From: Rafael WC >Reply-To: obm-l@mat.puc-rio.br >To: OBM >Subject: [obm-l] triângulos >Date: Sat, 18 May 2002 18:43:46 -0700 (PDT) > >Pessoal, ontem mandei uma dúvida sobre contar o total >de triângulos de todos os tamanhos de uma figura como >a que enviei abaixo novamente. Pensei muito sobre esse >problema e cheguei a uma fórmula não muito amigável, >mas até que não é ruim. Já dá até pra escrever um >algoritmo pra rodar no computador se quiser. > >Primeiro, eu chamei de x o número de lados de >triângulos que temos na base. Por exemplo, se tivermos >um triângulo só x = 1. >/_\ > >Se tivermos uma figura com quatro triângulos de menor >tamanho, temos: > /_\ >/_\ /_\ > >x = 2 > >Na figura que mandei, temos x = 4. > >Com isso, já que você tem triângulos de diferentes >tamanhos, você deve contar separadamente os triângulos >que têm como lado 1 traço, 2 traços, 3 traços...E >depois tem que contar os triângulos que estão de >cabeça pra baixo com esses mesmos tamanhos. > >Se você fizer isso em função dos traços da base não >fica muito ruim. Todas as linhas vou escrever a soma >de várias parcelas de x menos alguma coisa. Quando >você for calcular para algum x, você vai fazer as >subtrações até encontrar o valor zero, aí você para. >Por exemplo, na primeira linha temos: >x + (x - 1) + (x - 2) + (x - 3) + ... > >Se você tiver x = 2, você irá somar até x + (x - 1), >porque o próximo dará zero e aí você deve parar. > >Bom, no final você encontra isso: >triângulos de lado 1: >cabeça pra cima = x + (x - 1) + (x - 2) + (x - 3) + >... >cabeça pra baixo = (x - 1) + (x - 2) + (x - 3) + ... >total = x + 2.[(x - 1) + (x - 2) + (x - 3) + ...] >É como se o triângulo maior de todos fosse dividido em >várias linhas, aí você vai contando de cada linha. > >triângulos de lado 2: >cabeça pra cima = (x - 1) + (x - 2) + (x - 3) + (x - >4) + ... >cabeça pra baixo = (x - 3) + (x - 4) + (x - 5) + ... >total = (x - 1) + (x - 2) + 2.[(x - 3) + (x - 4) + >...] > >Por que aqui começamos a ter de cabeça pra baixo só >com (x - 3)? Porque para termos um triângulo de cabeça >pra baixo, o triângulo maior tem que ter o dobro de >traços na base do que o tamanho do triângulo. Como >esse tem lado 2, precisamos ter x = 4, que se fizermos >(x - 3) dará 1. Enquanto x for menor que 4 esse número >será negativo ou zero e aí não vamos contar. > >triângulos de lado 3: >cabeça pra cima = (x - 2) + (x - 3) + (x - 4) + (x - >5) + ... >cabeça pra baixo = (x - 5) + (x - 6) + (x - 7) + ... >total = (x - 2) + (x - 3) + (x - 4) + 2.[(x - 5) + >...] > >E assim teremos sempre esse padrão. Os triângulos de >cabeça pra cima começam sempre com (x - a), onde "a" é >o número anterior ao tamanho do triângulo. E os >triângulos de cabeça pra baixo começam sempre com x - >(2a - 1). Depois os outros termos você vai tirando >sempre 1. > >No final das contas você pode somar tudo isso. Soma os >triângulo de cabeça pra cima com os de cabeça pra >baixo de todos os tamanhos. O problema é que não pode >desenvolver muita coisa, porque não pode misturar x - >3 com x - 4, porque se você tiver x = 4, você não terá >o termo x - 4. Mas somando apenas x - 1 com x - 1 e x >- 2 com x -2, você terá: >total = x + 3.(x - 1) + 4.(x - 2) + 6.(x - 3) + 7.(x - >4) + 9.(x - 5) + 10.(x - 6) + 12.(x - 7) + 13.(x - 8) >+ ... > >No final você tem então todos os fatores x, x - 1, x - >2, x - 3, ... e os coeficientes de cada um têm uma >ordem até boazinha: >1, (pula o 2), 3, 4, (pula o 5), 6, 7, (pula o 8), 9, >10, (pula o 11), 12, 13, (pula o 14), ... > >E você vai usar a fórmula até o termo em que quando >fizer a diferença de x com alguma coisa dê zero. Ou >você pode até fazer a seguinte regra: considere que >desse valor total você vai pegar apenas os x primeiros >termos. > >Por exemplo, vamos pegar o triângulo da figura que tem >4 traços na base, ou seja x = 4. Então vamos pegar até >o quarto termo dessa fórmula e fazer x = 4: >total = x + 3.(x - 1) + 4.(x - 2) + 6.(x - 3) >total = 4 + 3.(4 - 1) + 4.(4 - 2) + 6.(4 - 3) >total = 4 + 3.3 + 4.2 + 6.1 >total = 4 + 9 + 8 + 6 >total = 27 > >E aí você pode fazer pra qualquer x. Aquele menor que >tinha x = 2, só pegamos os 2 primeiros termos: >total = x + 3.(x - 1) >total = 2 + 3.(2 - 1) >total = 2 + 3.1 >total = 2 + 3 >total = 5 > >De qualquer jeito você não precisa ficar contando um >por um e correr o risco de se perder mais facilmente. > >Mas o meu problema agora é o seguinte. Suspeito que >ainda dê para simplificar a fórmula, considerando duas >fórmulas, uma para quando x é par e outra para quando >x é ímpar. Talvez simplifique, mas aí você tem duas >fórmulas, não sei. Ainda não consegui. > >Será que alguém consegue melhorar daqui pra frente. O >pior acho que já passou. > >Um abraço, > >Rafael. > >===== >Rafael Werneck Cinoto > ICQ# 107011599 > rwcinoto@yahoo.com > rafael.caixa@gov.com.br > matduvidas@yahoo.com.br >http://www.rwcinoto.hpg.com.br/ > >__________________________________________________ >Do You Yahoo!? >LAUNCH - Your Yahoo! Music Experience >http://launch.yahoo.com ><< contatriang.gif >> _________________________________________________________________ Converse com amigos on-line, conheça o MSN Messenger: http://messenger.msn.com ========================================================================= Instruções para entrar na lista, sair da lista e usar a lista em http://www.mat.puc-rio.br/~nicolau/olimp/obm-l.html O administrador desta lista é ========================================================================= From owner-obm-l@sucuri.mat.puc-rio.br Mon May 20 13:03:27 2002 Return-Path: Received: (from majordom@localhost) by sucuri.mat.puc-rio.br (8.9.3/8.9.3) id NAA18189 for obm-l-list; Mon, 20 May 2002 13:02:46 -0300 Received: from hotmail.com (law2-f135.hotmail.com [216.32.181.135]) by sucuri.mat.puc-rio.br (8.9.3/8.9.3) with ESMTP id NAA18185 for ; Mon, 20 May 2002 13:02:43 -0300 Received: from mail pickup service by hotmail.com with Microsoft SMTPSVC; Mon, 20 May 2002 08:50:40 -0700 Received: from 32.94.119.254 by lw2fd.hotmail.msn.com with HTTP; Mon, 20 May 2002 15:50:36 GMT X-Originating-IP: [32.94.119.254] From: "Paulo Santa Rita" To: obm-l@mat.puc-rio.br Subject: [obm-l] =?iso-8859-1?B?UmU6IFtvYm0tbF0gcGVybXV0Yef1ZXMgY2lyY3VsYXJlcyBjb20gcmVw?= =?iso-8859-1?B?ZXRp5+Nv?= Date: Mon, 20 May 2002 15:50:36 +0000 Mime-Version: 1.0 Content-Type: text/plain; charset=iso-8859-1; format=flowed Message-ID: X-OriginalArrivalTime: 20 May 2002 15:50:40.0468 (UTC) FILETIME=[17947140:01C20016] Sender: owner-obm-l@sucuri.mat.puc-rio.br Precedence: bulk Reply-To: obm-l@mat.puc-rio.br Ola Rafael e demais colegas desta lista, Voce pode ver o TEOREMA DE MOREAU em : http://anduril.eupvg.upc.es/~joan/documents/moreau.htm Se mesmo assim voce nao se esclarecer eu conheco um caminho alternativo a este teorema. Um abraco Paulo Santa Rita 2,1245,200502 >From: Augusto Cesar de Oliveira Morgado >Reply-To: obm-l@mat.puc-rio.br >To: obm-l@mat.puc-rio.br >Subject: Re: [obm-l] permutações circulares com repetição >Date: Mon, 20 May 2002 12:05:15 -0300 (EST) > >Procure o Teorema de Moreau. > >Em Sat, 18 May 2002 18:51:57 -0700 (PDT), Rafael WC >disse: > > > Estou com problemas para resolver esse exercício: > > > > "De quantas maneiras distintas podemos dispor ao longo > > de um circulo, suposto fixo, 6 bolas brancas, 8 bolas > > azuis, 16 bolas verdes e 24 bolas amarelas?" > > O círculo fica fixo em nossa frente, mas as bolas > > ficam livres para serem rotacionadas como em uma > > catraca de bicicleta. > > > > Pra mim, entendi como sendo uma permutação circular > > com repetição. Nunca estudei isso e o que pensei que > > seria mais lógico não deu muito certo. Pensei que > > faríamos as permutações com repetições e dividiríamos > > pelo total de bolas por causa de ser circular. Em > > alguns casos até que conferiu com a resposta, mas aí > > coloquei um teste com apenas 4 bolas, duas brancas e > > duas azuis. > > > > Se eu fosse fazer como pensei, seria: > > PC4(2,2) = 4!/4.2!.2! = 3/2 > > > > Nem inteiro dá!!! > > > > Ao fazer escrevendo mesmo, vejo que só temos 6 > > permutações: > > 1 - AABB > > 2 - ABAB > > 3 - ABBA > > 4 - BAAB > > 5 - BABA > > 6 - BBAA > > > > E destas, se considerarmos como circulares, vemos que > > 1 = 3 = 4 = 6 e 2 = 5. O que nos dá apenas 2 > > permutações. > > > > Alguém sabe como resolvo esse tipo de problema? O > > único livro que tenho aqui sobre análise combinatória > > (Introdução à Análise Combinatória; Santos, J. P. O.; > > Mello, M. P.; Murari, I. T. C.; 2ª edição; Campinas, > > SP; Ed. da Unicamp, 1988) que aliás é muito bom, não > > fala sobre permutações circulares com combinação. > > > > Agradeço qualquer dica. > > > > Rafael. > > > > ===== > > Rafael Werneck Cinoto > > ICQ# 107011599 > > rwcinoto@yahoo.com > > rafael.caixa@gov.com.br > > matduvidas@yahoo.com.br > > http://www.rwcinoto.hpg.com.br/ > > > > __________________________________________________ > > Do You Yahoo!? > > LAUNCH - Your Yahoo! Music Experience > > http://launch.yahoo.com > > >========================================================================= > > Instruções para entrar na lista, sair da lista e usar a lista em > > http://www.mat.puc-rio.br/~nicolau/olimp/obm-l.html > > O administrador desta lista é > > >========================================================================= > > > > > >========================================================================= >Instruções para entrar na lista, sair da lista e usar a lista em >http://www.mat.puc-rio.br/~nicolau/olimp/obm-l.html >O administrador desta lista é >========================================================================= _________________________________________________________________ Envie e receba emails com o Hotmail no seu dispositivo móvel: http://mobile.msn.com ========================================================================= Instruções para entrar na lista, sair da lista e usar a lista em http://www.mat.puc-rio.br/~nicolau/olimp/obm-l.html O administrador desta lista é ========================================================================= From owner-obm-l@sucuri.mat.puc-rio.br Mon May 20 16:02:37 2002 Return-Path: Received: (from majordom@localhost) by sucuri.mat.puc-rio.br (8.9.3/8.9.3) id QAA22501 for obm-l-list; Mon, 20 May 2002 16:00:52 -0300 Received: from panther.unisys.com.br (panther.unisys.com.br [200.220.64.10]) by sucuri.mat.puc-rio.br (8.9.3/8.9.3) with ESMTP id QAA22495 for ; Mon, 20 May 2002 16:00:19 -0300 Received: from josefran (riopm12p23.uninet.com.br [200.220.2.215]) by panther.unisys.com.br (8.12.2/8.12.2) with SMTP id g4KIlk3n011172 for ; Mon, 20 May 2002 15:47:52 -0300 (EST) X-Spam-Filter: check_local@panther.unisys.com.br by digitalanswers.org Message-ID: <00f801c2002f$1413cca0$2902dcc8@josefran> From: "Jose Francisco Guimaraes Costa" To: "obm-l" Subject: [obm-l] =?iso-8859-1?Q?En:_=5Bobm-l=5D_Re:_=5Bobm-l=5D_Re:_Sob_que_condi=E7oe?= =?iso-8859-1?Q?s_uma_deformacao_preserva_medidas?= Date: Mon, 20 May 2002 15:48:43 -0300 MIME-Version: 1.0 Content-Type: multipart/alternative; boundary="----=_NextPart_000_00F3_01C20015.D231FDC0" X-Priority: 3 X-MSMail-Priority: Normal X-Mailer: Microsoft Outlook Express 5.00.2314.1300 X-MimeOLE: Produced By Microsoft MimeOLE V5.00.2314.1300 Sender: owner-obm-l@sucuri.mat.puc-rio.br Precedence: bulk Reply-To: obm-l@mat.puc-rio.br This is a multi-part message in MIME format. ------=_NextPart_000_00F3_01C20015.D231FDC0 Content-Type: text/plain; charset="iso-8859-1" Content-Transfer-Encoding: quoted-printable Rearrum=E1-las sem deform=E1-las? V poderia dar uma id=E9ia da demonstra=E7=E3o, isto =E9, a linha geral = seguida na demonstra=E7=E3o? Algu=E9m saberia dizer se as publica=E7=F5es citadas podem ser = encontradas em alguma biblioteca no Rio, que seja aberta a p=FAblico = externo? JF -----Mensagem Original-----=20 De: Paulo Rodrigues=20 Para: obm-l@mat.puc-rio.br=20 Enviada em: Domingo, 19 de Maio de 2002 14:56 Assunto: [obm-l] Re: [obm-l] Re: Sob que condi=E7oes uma deformacao = preserva medidas : Caros Nicolau e demais membros, :=20 :=20 :=20 : Faz um certo tempo o Nicolau mandou um e-mail que tinha o paragrafo : abaixo. Ocorre que eu li isso em uma superinteressante quando estava = na : escola e ate hoje tenho isso na cabeca, nao sabia se tinha sonhado, ou = se : era besteira, etc. Se alguem souber qual e a refererencia onde isso = foi : provado, ou pelo menos quem provou, ia me ajudar muito. Pelo que eu me : lembro, na revista falava-se algo em torno de 2^50 pedacos... :=20 :=20 : Abraco a todos, :=20 :=20 : Salvador :=20 : On Sun, 4 Feb 2001, Nicolau C. Saldanha wrote: :=20 :=20 : > Ali=E1s um grande problema da matem=E1tica do s=E9culo XX foi o da = quadratura=20 : > do c=EDrculo: n=E3o aquele proposto pelos gregos e cuja = demostra=E7=E3o foi : > conclu=EDda com a prova da transcend=EAncia de pi. O problema = s=E9culo XX : > da quadratura do c=EDrculo =E9: ser=E1 poss=EDvel decompor um = c=EDrculo de =E1rea 1 : > em um n=FAmero finito de pe=E7as e rearrum=E1-las para formar um = quadrado : > de =E1rea 1? A resposta =E9 que sim, =E9 poss=EDvel. : >=20 : > []s, N. : >=20 : >=20 :=20 Isto foi provado por Miklos Laczkovich: =20 M. Laczkovich, Equidecomposability and discrepancy; a solution of = Tarski's circle-squaring problem, Journal f=FCr die Reine und Angewandte = Mathematik, 403 (1990) 77-117 =20 Veja tamb=E9m, =20 R. J. Gardner and S. Wagon, At long last, the circle has been squared, = Notices of the American Mathematical Society, 36 (1989) - 1338-1343 =20 ------=_NextPart_000_00F3_01C20015.D231FDC0 Content-Type: text/html; charset="iso-8859-1" Content-Transfer-Encoding: quoted-printable
Rearrum=E1-las sem deform=E1-las?
 
V poderia dar uma id=E9ia da demonstra=E7=E3o, isto =E9, a linha = geral seguida na=20 demonstra=E7=E3o?
 
Algu=E9m saberia dizer se as publica=E7=F5es citadas podem ser = encontradas em=20 alguma biblioteca no Rio, que seja aberta a p=FAblico externo?
 
JF
 
-----Mensagem Original-----=20
De: Paulo=20 Rodrigues
Enviada em: Domingo, 19 de Maio de 2002 14:56
Assunto: [obm-l] Re: [obm-l] Re: Sob que condi=E7oes uma = deformacao=20 preserva medidas

: Caros Nicolau e demais = membros,
:
:=20
:
: Faz um certo tempo o Nicolau mandou um e-mail que tinha o=20 paragrafo
: abaixo. Ocorre que eu li isso em uma superinteressante = quando=20 estava na
: escola e ate hoje tenho isso na cabeca, nao sabia se = tinha=20 sonhado, ou se
: era besteira, etc. Se alguem souber qual e a = refererencia=20 onde isso foi
: provado, ou pelo menos quem provou, ia me ajudar = muito. Pelo=20 que eu me
: lembro, na revista falava-se algo em torno de 2^50=20 pedacos...
:
:
: Abraco a todos,
:
:
: = Salvador
:
:=20 On Sun, 4 Feb 2001, Nicolau C. Saldanha wrote:
:
:
: > = Ali=E1s um=20 grande problema da matem=E1tica do s=E9culo XX foi o da quadratura
: = > do=20 c=EDrculo: n=E3o aquele proposto pelos gregos e cuja demostra=E7=E3o = foi
: >=20 conclu=EDda com a prova da transcend=EAncia de pi. O problema s=E9culo = XX
: > da=20 quadratura do c=EDrculo =E9: ser=E1 poss=EDvel decompor um c=EDrculo de = =E1rea 1
: >=20 em um n=FAmero finito de pe=E7as e rearrum=E1-las para formar um = quadrado
: > de=20 =E1rea 1? A resposta =E9 que sim, =E9 poss=EDvel.
: >
: > = []s, N.
: >=20
: >

Isto foi provado por  Miklos=20 Laczkovich:
 
M. Laczkovich, Equidecomposability = and=20 discrepancy; a solution of Tarski's circle-squaring problem, Journal = f=FCr die=20 Reine und Angewandte Mathematik, 403 (1990) = 77-117
 
Veja tamb=E9m,
 
R. J. Gardner and S. Wagon, At = long last,=20 the circle has been squared, Notices of the American Mathematical=20 Society, 36 (1989) - 1338-1343
 
------=_NextPart_000_00F3_01C20015.D231FDC0-- ========================================================================= Instruções para entrar na lista, sair da lista e usar a lista em http://www.mat.puc-rio.br/~nicolau/olimp/obm-l.html O administrador desta lista é ========================================================================= From owner-obm-l@sucuri.mat.puc-rio.br Mon May 20 16:15:46 2002 Return-Path: Received: (from majordom@localhost) by sucuri.mat.puc-rio.br (8.9.3/8.9.3) id QAA22799 for obm-l-list; Mon, 20 May 2002 16:15:38 -0300 Received: from fgvrj23.fgv.br (fgvrj23.fgv.br [200.20.164.23]) by sucuri.mat.puc-rio.br (8.9.3/8.9.3) with ESMTP id QAA22786 for ; Mon, 20 May 2002 16:15:35 -0300 Received: by FGVRJ23 with Internet Mail Service (5.5.2653.19) id <20VAZPY7>; Mon, 20 May 2002 16:04:48 -0300 Message-ID: <765A72978645D4118B1C0000E229806D05B6B394@FGVRJ23> From: Ralph Teixeira To: "'obm-l@mat.puc-rio.br'" Subject: RES: [obm-l] Espacial Date: Mon, 20 May 2002 16:04:38 -0300 MIME-Version: 1.0 X-Mailer: Internet Mail Service (5.5.2653.19) Content-Type: text/plain; charset="iso-8859-1" Content-Transfer-Encoding: 8bit X-MIME-Autoconverted: from quoted-printable to 8bit by sucuri.mat.puc-rio.br id QAA22787 Sender: owner-obm-l@sucuri.mat.puc-rio.br Precedence: bulk Reply-To: obm-l@mat.puc-rio.br Acho que o enunciado quer dizer que as três arestas *distintas* são ímpares consecutivos, digamos, x-2, x e x+2. Vamos supor que o paralelepípedo é retângulo (não creio que dá para fazer de outro jeito; se der, a resposta devia ser a mesma de qualquer forma). Então temos: Área Lateral = 2(x-2)x+2(x+2)x+2(x-2)(x+2)=6x^2-8=142 6x^2=150 x^2=25 x=5 (pois x>0) Então as arestas são 3cm, 5cm e 7cm e o volume dá 105cm^3. Abraço, Ralph -----Mensagem original----- De: Marcos Reynaldo [mailto:marc_reybr@yahoo.com.br] Enviada em: sexta-feira, 17 de maio de 2002 02:32 Para: obm-l@mat.puc-rio.br Assunto: Re: [obm-l] Espacial Olá Raul! Esse enunciado esta correto ? Se as arestas laterais do paralelepipedo são numeros impares consecutivos então não da um paralelogramo. Euraul@aol.com escreveu: As arestas laterais de um paralelepípedo, medidas em cm, são números ímpares consecutivos e a área lateral do mesmo é de 142cm quadrados. Qual é o volume do paralelepípedo ? Obrigado pela atenção, Raul _______________________________________________________________________ Yahoo! Encontros O lugar certo para você encontrar aquela pessoa que falta na sua vida. Cadastre-se hoje mesmo! http://br.encontros.yahoo.com/ ========================================================================= Instruções para entrar na lista, sair da lista e usar a lista em http://www.mat.puc-rio.br/~nicolau/olimp/obm-l.html O administrador desta lista é ========================================================================= ========================================================================= Instruções para entrar na lista, sair da lista e usar a lista em http://www.mat.puc-rio.br/~nicolau/olimp/obm-l.html O administrador desta lista é ========================================================================= From owner-obm-l@sucuri.mat.puc-rio.br Mon May 20 16:15:48 2002 Return-Path: Received: (from majordom@localhost) by sucuri.mat.puc-rio.br (8.9.3/8.9.3) id QAA22790 for obm-l-list; Mon, 20 May 2002 16:15:37 -0300 Received: from fgvrj23.fgv.br (fgvrj23.fgv.br [200.20.164.23]) by sucuri.mat.puc-rio.br (8.9.3/8.9.3) with ESMTP id QAA22784 for ; Mon, 20 May 2002 16:15:34 -0300 Received: by FGVRJ23 with Internet Mail Service (5.5.2653.19) id <20VAZP4J>; Mon, 20 May 2002 15:55:25 -0300 Message-ID: <765A72978645D4118B1C0000E229806D05B6B393@FGVRJ23> From: Ralph Teixeira To: "'obm-l@mat.puc-rio.br'" Subject: [obm-l] =?iso-8859-1?Q?RES=3A_=5Bobm-l=5D_valor_m=EDnimo?= Date: Mon, 20 May 2002 15:55:17 -0300 MIME-Version: 1.0 X-Mailer: Internet Mail Service (5.5.2653.19) Content-Type: multipart/alternative; boundary="----_=_NextPart_001_01C2002F.E21B7B70" Sender: owner-obm-l@sucuri.mat.puc-rio.br Precedence: bulk Reply-To: obm-l@mat.puc-rio.br This message is in MIME format. Since your mail reader does not understand this format, some or all of this message may not be legible. ------_=_NextPart_001_01C2002F.E21B7B70 Content-Type: text/plain; charset="iso-8859-1" Content-Transfer-Encoding: quoted-printable =C9 poss=EDvel sim. =20 1) Via c=E1lculo =20 Derive a express=E3o com rela=E7=E3o a "a", iguale a zero, d=E1 uma = equa=E7=E3o meio feia mas sai que a=3D1/2; =20 1.1) C=E1lculo incrementado Note que, se f(x)=3Dsqrt(1+(1-x)^2)+sqrt(1+x^2), ent=E3o = f(x)=3Df(1-x), isto =E9, o gr=E1fico da fun=E7=E3o =E9 sim=E9trico com rela=E7=E3o =E0 reta = x=3D1/2. Isto sugere fazer y=3Dx-1/2, e ent=E3o f(y)=3Dsqrt(1+(y-1/2)^2)+sqrt(1+(y+1/2)^2). = Os c=E1lculos aqui j=E1 s=E3o um pouco mais simples... D=E1 at=E9 para fazer a partir = daqui sem c=E1lculo, com m=E1gica....=20 =20 2) Por geometria =20 ...mas se voc=EA quer uma maneira BEM m=E1gica de fazer, pense assim: =20 f(a)=3Dsqrt(1+(1-a)^2)+sqrt(1+a^2) =E9 a soma das dist=E2ncias do ponto = (1,a) aos pontos (0,1) e (2,0). Em outras palavras, queremos o ponto P(1,a) na = reta x=3D1 que minimiza as somas das dist=E2ncias aos pontos B(0,1) e C(2,0) = -- que s=E3o fixos e se encontram um de cada lado da reta! Ora, o menor = caminho BPC =E9 o segmento de reta que liga B a C! Assim, o m=EDnimo se d=E1 quando B,P = e C est=E3o alinhados; note que, ent=E3o, P ser=E1 o ponto m=E9dio de BC, = isto =E9, a=3D1/2. =20 Legal? =20 Abra=E7o, Ralph =20 -----Mensagem original----- De: Eder [mailto:edalbuquerque@uol.com.br] Enviada em: sexta-feira, 17 de maio de 2002 21:34 Para: obm-l@mat.puc-rio.br Assunto: [obm-l] valor m=EDnimo Ol=E1, =C9 poss=EDvel determinar para que valor de a,tem-se y=3D sqrt( 1+ = (1-a)=B2) + sqrt(1+ a=B2) m=EDnimo? =20 =20 ------_=_NextPart_001_01C2002F.E21B7B70 Content-Type: text/html; charset="iso-8859-1" Content-Transfer-Encoding: quoted-printable
=C9=20 poss=EDvel sim.
 
1) Via=20 c=E1lculo
 
Derive=20 a express=E3o com rela=E7=E3o a "a", iguale a zero, d=E1 uma = equa=E7=E3o meio feia mas sai=20 que a=3D1/2;
 
    1.1) C=E1lculo = incrementado
    Note que, se = f(x)=3Dsqrt(1+(1-x)^2)+sqrt(1+x^2),=20 ent=E3o f(x)=3Df(1-x), isto =E9, o gr=E1fico da fun=E7=E3o =E9 = sim=E9trico com rela=E7=E3o =E0 reta=20 x=3D1/2. Isto sugere fazer y=3Dx-1/2, e ent=E3o f(y)=3Dsqrt(1+(y-1/2)^2)+sqrt(1+(y+1/2)^2). = Os c=E1lculos aqui=20 j=E1 s=E3o um pouco mais simples... D=E1 at=E9 para fazer a partir = daqui sem c=E1lculo,=20 com m=E1gica....
 
2) Por=20 geometria
 
...mas se voc=EA quer uma maneira BEM m=E1gica = de fazer, pense=20 assim:
 
f(a)=3Dsqrt(1+(1-a)^2)+sqrt(1+a^2) =E9 a soma = das dist=E2ncias do=20 ponto (1,a) aos pontos (0,1) e (2,0). Em outras palavras, queremos o = ponto=20 P(1,a) na reta x=3D1 que minimiza as somas das dist=E2ncias aos pontos = B(0,1) e=20 C(2,0) -- que s=E3o fixos e se encontram um de cada lado da reta! Ora, = o menor=20 caminho BPC =E9 o segmento de reta que liga B a C! Assim, o m=EDnimo se = d=E1 quando=20 B,P e C est=E3o alinhados; note que, ent=E3o, P ser=E1 o ponto m=E9dio = de BC, isto =E9,=20 a=3D1/2.
 
Legal?
 
Abra=E7o,
       =20 Ralph
 
 -----M= ensagem=20 original-----
De: Eder = [mailto:edalbuquerque@uol.com.br]
Enviada=20 em: sexta-feira, 17 de maio de 2002 21:34
Para:=20 obm-l@mat.puc-rio.br
Assunto: [obm-l] valor=20 m=EDnimo
Ol=E1,
 =C9 poss=EDvel=20 determinar para que valor de a,tem-se y=3D sqrt( 1+ (1-a)=B2) + sqrt(1+ = a=B2)=20 m=EDnimo?
 
 
------_=_NextPart_001_01C2002F.E21B7B70-- ========================================================================= Instruções para entrar na lista, sair da lista e usar a lista em http://www.mat.puc-rio.br/~nicolau/olimp/obm-l.html O administrador desta lista é ========================================================================= From owner-obm-l@sucuri.mat.puc-rio.br Mon May 20 16:39:55 2002 Return-Path: Received: (from majordom@localhost) by sucuri.mat.puc-rio.br (8.9.3/8.9.3) id QAA23533 for obm-l-list; Mon, 20 May 2002 16:39:17 -0300 Received: from fgvrj23.fgv.br (fgvrj23.fgv.br [200.20.164.23]) by sucuri.mat.puc-rio.br (8.9.3/8.9.3) with ESMTP id QAA23529 for ; Mon, 20 May 2002 16:39:15 -0300 Received: by FGVRJ23 with Internet Mail Service (5.5.2653.19) id <20VAZQCY>; Mon, 20 May 2002 16:23:05 -0300 Message-ID: <765A72978645D4118B1C0000E229806D05B6B395@FGVRJ23> From: Ralph Teixeira To: "'obm-l@mat.puc-rio.br'" Subject: RES: RES: [obm-l] .......... Date: Mon, 20 May 2002 16:23:02 -0300 MIME-Version: 1.0 X-Mailer: Internet Mail Service (5.5.2653.19) Content-Type: text/plain; charset="iso-8859-1" Content-Transfer-Encoding: 8bit X-MIME-Autoconverted: from quoted-printable to 8bit by sucuri.mat.puc-rio.br id QAA23530 Sender: owner-obm-l@sucuri.mat.puc-rio.br Precedence: bulk Reply-To: obm-l@mat.puc-rio.br Oi, galera. Só para destacar: a equação x=sqrt(5-sqrt(5-x)) tem DUAS soluções, uma das quais é a que sai dos argumentos tipo "x=sqrt(5-x)". Mas há uma outra, que não sairá por este argumento pois ela simplesmente não satisfaz x=sqrt(5-x). De fato, ao se construir a seqüência do Raul para a segunda solução, ela ficará alternando entre dois valores distintos do tipo a e 1-a; a seqüência não terá limite e o argumento do Raul não pode ser feito. Eu ainda prefiro a minha solução, que é mais braçal, sim, mas que acha as duas respostas corretas (vejam minha outra mensagem). Note que ela usa sim o fato de que SE x=sqrt(5-x) ENTÃO x=sqrt(5-sqrt(5-x)), que é em suma o que está sendo dito em várias mensagens -- mas algumas mensagens que eu li parecem querer reverter a implicação, o que simplesmente não pode ser feito. Você não pode supor que x=sqrt(5-x) neste problema -- se você fizé-lo, raízes perfeitamente válidas serão perdidas. Se você não quiser usar meu método braçal, a outra opção é a de fazer y=sqrt(5-x) e trabalhar por aí -- alguém escreveu isso, eu só não me lembro quem foi. Você chegará a y=x **OU** y+x=1. A opção y=x leva a x=sqrt(5-x), tudo bem, mas y+x=1 também é possível! Abraço, Ralph ========================================================================= Instruções para entrar na lista, sair da lista e usar a lista em http://www.mat.puc-rio.br/~nicolau/olimp/obm-l.html O administrador desta lista é ========================================================================= From owner-obm-l@sucuri.mat.puc-rio.br Mon May 20 19:08:27 2002 Return-Path: Received: (from majordom@localhost) by sucuri.mat.puc-rio.br (8.9.3/8.9.3) id TAA26275 for obm-l-list; Mon, 20 May 2002 19:08:02 -0300 Received: from web10104.mail.yahoo.com (web10104.mail.yahoo.com [216.136.130.54]) by sucuri.mat.puc-rio.br (8.9.3/8.9.3) with SMTP id TAA26271 for ; Mon, 20 May 2002 19:07:59 -0300 Message-ID: <20020520215557.78210.qmail@web10104.mail.yahoo.com> Received: from [150.161.4.41] by web10104.mail.yahoo.com via HTTP; Mon, 20 May 2002 14:55:57 PDT Date: Mon, 20 May 2002 14:55:57 -0700 (PDT) From: Rafael WC Subject: Re: [obm-l] permutações circulares com repetição To: obm-l@mat.puc-rio.br In-Reply-To: MIME-Version: 1.0 Content-Type: text/plain; charset=us-ascii Sender: owner-obm-l@sucuri.mat.puc-rio.br Precedence: bulk Reply-To: obm-l@mat.puc-rio.br Olá Pessoal! Obrigado Morgado e Paulo pela ajuda. Paulo, entrei na página que você indicou. Encontrei a demonstração de um teorema de Moreau que sinceramente não consegui associar nem de longe com permutações circulares com repetição. Como você falou que conhece um caminho alternativo, acho que vou abusar da sua boa vontade e perguntar qual é. Se puder me ajudar, agradeço muito. Hoje fui à biblioteca da faculdade e andei procurando alguns livros, mas todos param nas permutações circulares simples!! Muito obrigado, Rafael. --- Paulo Santa Rita wrote: > Ola Rafael e demais > colegas desta lista, > > Voce pode ver o TEOREMA DE MOREAU em : > > http://anduril.eupvg.upc.es/~joan/documents/moreau.htm > > Se mesmo assim voce nao se esclarecer eu conheco um > caminho alternativo a > este teorema. > > Um abraco > Paulo Santa Rita > 2,1245,200502 > > > >From: Augusto Cesar de Oliveira Morgado > > >Reply-To: obm-l@mat.puc-rio.br > >To: obm-l@mat.puc-rio.br > >Subject: Re: [obm-l] permutações circulares com > repetição > >Date: Mon, 20 May 2002 12:05:15 -0300 (EST) > > > >Procure o Teorema de Moreau. > > > >Em Sat, 18 May 2002 18:51:57 -0700 (PDT), Rafael WC > > >disse: > > > > > Estou com problemas para resolver esse > exercício: > > > > > > "De quantas maneiras distintas podemos dispor ao > longo > > > de um circulo, suposto fixo, 6 bolas brancas, 8 > bolas > > > azuis, 16 bolas verdes e 24 bolas amarelas?" > > > O círculo fica fixo em nossa frente, mas as > bolas > > > ficam livres para serem rotacionadas como em uma > > > catraca de bicicleta. > > > > > > Pra mim, entendi como sendo uma permutação > circular > > > com repetição. Nunca estudei isso e o que pensei > que > > > seria mais lógico não deu muito certo. Pensei > que > > > faríamos as permutações com repetições e > dividiríamos > > > pelo total de bolas por causa de ser circular. > Em > > > alguns casos até que conferiu com a resposta, > mas aí > > > coloquei um teste com apenas 4 bolas, duas > brancas e > > > duas azuis. > > > > > > Se eu fosse fazer como pensei, seria: > > > PC4(2,2) = 4!/4.2!.2! = 3/2 > > > > > > Nem inteiro dá!!! > > > > > > Ao fazer escrevendo mesmo, vejo que só temos 6 > > > permutações: > > > 1 - AABB > > > 2 - ABAB > > > 3 - ABBA > > > 4 - BAAB > > > 5 - BABA > > > 6 - BBAA > > > > > > E destas, se considerarmos como circulares, > vemos que > > > 1 = 3 = 4 = 6 e 2 = 5. O que nos dá apenas 2 > > > permutações. > > > > > > Alguém sabe como resolvo esse tipo de problema? > O > > > único livro que tenho aqui sobre análise > combinatória > > > (Introdução à Análise Combinatória; Santos, J. > P. O.; > > > Mello, M. P.; Murari, I. T. C.; 2ª edição; > Campinas, > > > SP; Ed. da Unicamp, 1988) que aliás é muito bom, > não > > > fala sobre permutações circulares com > combinação. > > > > > > Agradeço qualquer dica. > > > > > > Rafael. > > > > > > ===== > > > Rafael Werneck Cinoto > > > ICQ# 107011599 > > > rwcinoto@yahoo.com > > > rafael.caixa@gov.com.br > > > matduvidas@yahoo.com.br > > > http://www.rwcinoto.hpg.com.br/ > > > > > > > __________________________________________________ > > > Do You Yahoo!? > > > LAUNCH - Your Yahoo! Music Experience > > > http://launch.yahoo.com > > > > >========================================================================= > > > Instruções para entrar na lista, sair da lista e > usar a lista em > > > > http://www.mat.puc-rio.br/~nicolau/olimp/obm-l.html > > > O administrador desta lista é > > > > > >========================================================================= > > > > > > > > > >========================================================================= > >Instruções para entrar na lista, sair da lista e > usar a lista em > >http://www.mat.puc-rio.br/~nicolau/olimp/obm-l.html > >O administrador desta lista é > > >========================================================================= > > > > > _________________________________________________________________ > Envie e receba emails com o Hotmail no seu > dispositivo móvel: > http://mobile.msn.com > > ========================================================================= > Instruções para entrar na lista, sair da lista e > usar a lista em > http://www.mat.puc-rio.br/~nicolau/olimp/obm-l.html > O administrador desta lista é > > ========================================================================= ===== Rafael Werneck Cinoto ICQ# 107011599 rwcinoto@yahoo.com rafael.caixa@gov.com.br matduvidas@yahoo.com.br http://www.rwcinoto.hpg.com.br/ __________________________________________________ Do You Yahoo!? LAUNCH - Your Yahoo! Music Experience http://launch.yahoo.com ========================================================================= Instruções para entrar na lista, sair da lista e usar a lista em http://www.mat.puc-rio.br/~nicolau/olimp/obm-l.html O administrador desta lista é ========================================================================= From owner-obm-l@sucuri.mat.puc-rio.br Mon May 20 20:21:36 2002 Return-Path: Received: (from majordom@localhost) by sucuri.mat.puc-rio.br (8.9.3/8.9.3) id UAA27290 for obm-l-list; Mon, 20 May 2002 20:20:56 -0300 Received: from web10104.mail.yahoo.com (web10104.mail.yahoo.com [216.136.130.54]) by sucuri.mat.puc-rio.br (8.9.3/8.9.3) with SMTP id UAA27286 for ; Mon, 20 May 2002 20:20:53 -0300 Message-ID: <20020520230852.88886.qmail@web10104.mail.yahoo.com> Received: from [150.161.4.42] by web10104.mail.yahoo.com via HTTP; Mon, 20 May 2002 16:08:52 PDT Date: Mon, 20 May 2002 16:08:52 -0700 (PDT) From: Rafael WC Subject: [obm-l] logaritmo de (-10)^2 To: OBM MIME-Version: 1.0 Content-Type: text/plain; charset=us-ascii Sender: owner-obm-l@sucuri.mat.puc-rio.br Precedence: bulk Reply-To: obm-l@mat.puc-rio.br Oi Pessoal! Caiu uma questão num concurso só para professores de matemática ontem que me deixou intrigado: Dada a função f: f(x) = x + raiz(x^2) - log(base 10)(x^2) Calcule f(-10). A resposta foi -2. Mas depois da prova surgiu a maior discussão porque existia uma alternativa que era "f(-10) não está definida". O pessoal questinou que estando f(-10) definida, devia valer a propriedade do expoente de logaritmo e poderíamos escrever: f(x) = x + raiz(x^2) - log(base 10)(x^2) f(x) = x + raiz(x^2) - 2.log(base 10)(x) E aí vemos claramente que não podemos tirar o log de -10. Mas como o gabarito da comissão organizadora foi -2, ficamos todos na dúvida: está definida f(-10)??? Um abraço, Rafael. ===== Rafael Werneck Cinoto ICQ# 107011599 rwcinoto@yahoo.com rafael.caixa@gov.com.br matduvidas@yahoo.com.br http://www.rwcinoto.hpg.com.br/ __________________________________________________ Do You Yahoo!? LAUNCH - Your Yahoo! Music Experience http://launch.yahoo.com ========================================================================= Instruções para entrar na lista, sair da lista e usar a lista em http://www.mat.puc-rio.br/~nicolau/olimp/obm-l.html O administrador desta lista é ========================================================================= From owner-obm-l@sucuri.mat.puc-rio.br Mon May 20 20:50:53 2002 Return-Path: Received: (from majordom@localhost) by sucuri.mat.puc-rio.br (8.9.3/8.9.3) id UAA27736 for obm-l-list; Mon, 20 May 2002 20:50:28 -0300 Received: from sr1.terra.com.br (sr1.terra.com.br [200.176.3.16]) by sucuri.mat.puc-rio.br (8.9.3/8.9.3) with ESMTP id UAA27732 for ; Mon, 20 May 2002 20:50:26 -0300 Received: from smtp4-poa.terra.com.br (smtp4-poa.terra.com.br [200.176.3.35]) by sr1.terra.com.br (Postfix) with ESMTP id 9101A6EF98 for ; Mon, 20 May 2002 20:38:26 -0300 (EST) Received: from stabel (dl-nas1-poa-C89A0132.p001.terra.com.br [200.154.1.50]) (authenticated user dudasta) by smtp4-poa.terra.com.br (Postfix) with ESMTP id 139E9AC5C1 for ; Mon, 20 May 2002 20:38:25 -0300 (EST) Message-ID: <002d01c20057$6dfc20f0$32019ac8@stabel> From: "Eduardo Casagrande Stabel" To: References: <20020520230852.88886.qmail@web10104.mail.yahoo.com> Subject: Re: [obm-l] logaritmo de (-10)^2 Date: Mon, 20 May 2002 20:38:20 -0300 MIME-Version: 1.0 Content-Type: text/plain; charset="iso-8859-1" Content-Transfer-Encoding: 8bit X-Priority: 3 X-MSMail-Priority: Normal X-Mailer: Microsoft Outlook Express 6.00.2600.0000 X-MimeOLE: Produced By Microsoft MimeOLE V6.00.2600.0000 Sender: owner-obm-l@sucuri.mat.puc-rio.br Precedence: bulk Reply-To: obm-l@mat.puc-rio.br Oi Rafael. A função f está definida em x=-10, pois como x^2>0 existe o log(x^2). A propriedade do expoente vale se x>0. No seu caso, escreva assim: log(x^2) = log(|x|^2) = 2*log( |x| ). Eduardo Casagrande Stabel. From: "Rafael WC" > Oi Pessoal! > > Caiu uma questão num concurso só para professores de > matemática ontem que me deixou intrigado: > Dada a função f: > f(x) = x + raiz(x^2) - log(base 10)(x^2) > > Calcule f(-10). > > A resposta foi -2. Mas depois da prova surgiu a maior > discussão porque existia uma alternativa que era > "f(-10) não está definida". > > O pessoal questinou que estando f(-10) definida, devia > valer a propriedade do expoente de logaritmo e > poderíamos escrever: > f(x) = x + raiz(x^2) - log(base 10)(x^2) > f(x) = x + raiz(x^2) - 2.log(base 10)(x) > > E aí vemos claramente que não podemos tirar o log de > -10. Mas como o gabarito da comissão organizadora foi > -2, ficamos todos na dúvida: está definida f(-10)??? > > Um abraço, > > Rafael. > > ===== > Rafael Werneck Cinoto > ICQ# 107011599 > rwcinoto@yahoo.com > rafael.caixa@gov.com.br > matduvidas@yahoo.com.br > http://www.rwcinoto.hpg.com.br/ > > __________________________________________________ > Do You Yahoo!? > LAUNCH - Your Yahoo! Music Experience > http://launch.yahoo.com > ========================================================================= > Instruções para entrar na lista, sair da lista e usar a lista em > http://www.mat.puc-rio.br/~nicolau/olimp/obm-l.html > O administrador desta lista é > ========================================================================= > > ========================================================================= Instruções para entrar na lista, sair da lista e usar a lista em http://www.mat.puc-rio.br/~nicolau/olimp/obm-l.html O administrador desta lista é ========================================================================= From owner-obm-l@sucuri.mat.puc-rio.br Mon May 20 21:07:46 2002 Return-Path: Received: (from majordom@localhost) by sucuri.mat.puc-rio.br (8.9.3/8.9.3) id VAA28274 for obm-l-list; Mon, 20 May 2002 21:07:35 -0300 Received: from traven10.uol.com.br (traven10.uol.com.br [200.231.206.211]) by sucuri.mat.puc-rio.br (8.9.3/8.9.3) with ESMTP id VAA28270 for ; Mon, 20 May 2002 21:07:33 -0300 Received: from Eder ([200.227.65.53]) by traven10.uol.com.br (8.9.1/8.9.1) with SMTP id UAA12943 for ; Mon, 20 May 2002 20:55:46 -0300 (BRT) Message-ID: <007901c20059$8a4a2700$b826fea9@Eder> From: "Eder" To: References: <765A72978645D4118B1C0000E229806D05B6B393@FGVRJ23> Subject: [obm-l] =?iso-8859-1?Q?Re:_=5Bobm-l=5D_RES:_=5Bobm-l=5D_valor_m=EDnimo?= Date: Mon, 20 May 2002 18:58:24 -0300 MIME-Version: 1.0 Content-Type: multipart/alternative; boundary="----=_NextPart_000_0021_01C20030.514FA700" X-Priority: 3 X-MSMail-Priority: Normal X-Mailer: Microsoft Outlook Express 5.00.2314.1300 X-MimeOLE: Produced By Microsoft MimeOLE V5.00.2314.1300 Sender: owner-obm-l@sucuri.mat.puc-rio.br Precedence: bulk Reply-To: obm-l@mat.puc-rio.br This is a multi-part message in MIME format. ------=_NextPart_000_0021_01C20030.514FA700 Content-Type: text/plain; charset="iso-8859-1" Content-Transfer-Encoding: quoted-printable Valeu Ralph, Essa express=E3o surgiu do seguinte problema: detrerminar o menor = caminho que uma formiguinha pode fazer por sobre a superf=EDcie de um = cubo de aresta 1,de um v=E9rtice a outro "diagonalmente oposto". Eu admiti uma trajet=F3ria gen=E9rica e cheguei a esse valor para o = caminho.Eu j=E1 imaginava que a=3D1/2,mas queria provar algebricamente. ----- Original Message -----=20 From: Ralph Teixeira=20 To: 'obm-l@mat.puc-rio.br'=20 Sent: Monday, May 20, 2002 3:55 PM Subject: [obm-l] RES: [obm-l] valor m=EDnimo =C9 poss=EDvel sim. 1) Via c=E1lculo Derive a express=E3o com rela=E7=E3o a "a", iguale a zero, d=E1 uma = equa=E7=E3o meio feia mas sai que a=3D1/2; 1.1) C=E1lculo incrementado Note que, se f(x)=3Dsqrt(1+(1-x)^2)+sqrt(1+x^2), ent=E3o = f(x)=3Df(1-x), isto =E9, o gr=E1fico da fun=E7=E3o =E9 sim=E9trico com = rela=E7=E3o =E0 reta x=3D1/2. Isto sugere fazer y=3Dx-1/2, e ent=E3o = f(y)=3Dsqrt(1+(y-1/2)^2)+sqrt(1+(y+1/2)^2). Os c=E1lculos aqui j=E1 = s=E3o um pouco mais simples... D=E1 at=E9 para fazer a partir daqui sem = c=E1lculo, com m=E1gica....=20 2) Por geometria ...mas se voc=EA quer uma maneira BEM m=E1gica de fazer, pense assim: f(a)=3Dsqrt(1+(1-a)^2)+sqrt(1+a^2) =E9 a soma das dist=E2ncias do = ponto (1,a) aos pontos (0,1) e (2,0). Em outras palavras, queremos o = ponto P(1,a) na reta x=3D1 que minimiza as somas das dist=E2ncias aos = pontos B(0,1) e C(2,0) -- que s=E3o fixos e se encontram um de cada lado = da reta! Ora, o menor caminho BPC =E9 o segmento de reta que liga B a C! = Assim, o m=EDnimo se d=E1 quando B,P e C est=E3o alinhados; note que, = ent=E3o, P ser=E1 o ponto m=E9dio de BC, isto =E9, a=3D1/2. Legal? Abra=E7o, Ralph -----Mensagem original----- De: Eder [mailto:edalbuquerque@uol.com.br] Enviada em: sexta-feira, 17 de maio de 2002 21:34 Para: obm-l@mat.puc-rio.br Assunto: [obm-l] valor m=EDnimo Ol=E1, =C9 poss=EDvel determinar para que valor de a,tem-se y=3D sqrt( 1+ = (1-a)=B2) + sqrt(1+ a=B2) m=EDnimo? ------=_NextPart_000_0021_01C20030.514FA700 Content-Type: text/html; charset="iso-8859-1" Content-Transfer-Encoding: quoted-printable
Valeu Ralph,
 
 
Essa express=E3o surgiu do seguinte = problema:=20 detrerminar o menor caminho que uma formiguinha pode fazer por sobre a=20 superf=EDcie de um cubo de aresta 1,de um v=E9rtice a outro = "diagonalmente=20 oposto".
 
Eu admiti uma trajet=F3ria gen=E9rica e = cheguei a esse=20 valor para o caminho.Eu j=E1 imaginava que a=3D1/2,mas queria provar=20 algebricamente.
----- Original Message -----
From:=20 Ralph = Teixeira
Sent: Monday, May 20, 2002 3:55 = PM
Subject: [obm-l] RES: [obm-l] = valor=20 m=EDnimo

=C9=20 poss=EDvel sim.
 
1)=20 Via c=E1lculo
 
Derive a express=E3o com rela=E7=E3o a "a", iguale a zero, = d=E1 uma equa=E7=E3o=20 meio feia mas sai que a=3D1/2;
 
    1.1) C=E1lculo = incrementado
    Note que, se = f(x)=3Dsqrt(1+(1-x)^2)+sqrt(1+x^2),=20 ent=E3o f(x)=3Df(1-x), isto =E9, o gr=E1fico da fun=E7=E3o =E9 = sim=E9trico com rela=E7=E3o =E0 reta=20 x=3D1/2. Isto sugere fazer y=3Dx-1/2, e ent=E3o f(y)=3Dsqrt(1+(y-1/2)^2)+sqrt(1+(y+1/2)^2). = Os c=E1lculos=20 aqui j=E1 s=E3o um pouco mais simples... D=E1 at=E9 para fazer a = partir daqui sem=20 c=E1lculo, com m=E1gica....
 
2)=20 Por geometria
 
...mas se voc=EA quer uma maneira BEM = m=E1gica de fazer,=20 pense assim:
 
f(a)=3Dsqrt(1+(1-a)^2)+sqrt(1+a^2) =E9 a soma = das dist=E2ncias=20 do ponto (1,a) aos pontos (0,1) e (2,0). Em outras palavras, queremos = o ponto=20 P(1,a) na reta x=3D1 que minimiza as somas das dist=E2ncias aos pontos = B(0,1) e=20 C(2,0) -- que s=E3o fixos e se encontram um de cada lado da reta! Ora, = o menor=20 caminho BPC =E9 o segmento de reta que liga B a C! Assim, o m=EDnimo = se d=E1 quando=20 B,P e C est=E3o alinhados; note que, ent=E3o, P ser=E1 o ponto m=E9dio = de BC, isto =E9,=20 a=3D1/2.
 
Legal?
 
Abra=E7o,
       =20 Ralph
 
 -----Mensagem=20 original-----
De: Eder=20 [mailto:edalbuquerque@uol.com.br]
Enviada em: sexta-feira, = 17 de=20 maio de 2002 21:34
Para: = obm-l@mat.puc-rio.br
Assunto:=20 [obm-l] valor m=EDnimo
Ol=E1,
 =C9 poss=EDvel=20 determinar para que valor de a,tem-se y=3D sqrt( 1+ (1-a)=B2) + = sqrt(1+ a=B2)=20 m=EDnimo?
 
 
------=_NextPart_000_0021_01C20030.514FA700-- ========================================================================= Instruções para entrar na lista, sair da lista e usar a lista em http://www.mat.puc-rio.br/~nicolau/olimp/obm-l.html O administrador desta lista é ========================================================================= From owner-obm-l@sucuri.mat.puc-rio.br Mon May 20 21:59:41 2002 Return-Path: Received: (from majordom@localhost) by sucuri.mat.puc-rio.br (8.9.3/8.9.3) id VAA29172 for obm-l-list; Mon, 20 May 2002 21:58:32 -0300 Received: from br.inter.net (opt-0-15.br.inter.net [200.185.56.15] (may be forged)) by sucuri.mat.puc-rio.br (8.9.3/8.9.3) with ESMTP id VAA29168 for ; Mon, 20 May 2002 21:58:30 -0300 Received: from dialup-200-184-32-225.intelignet.com.br ([200.184.32.225] helo=ig) by br.inter.net with esmtp (Exim 3.36 #1) id 179xlP-0003LE-00 for obm-l@mat.puc-rio.br; Mon, 20 May 2002 21:44:32 -0300 Message-ID: <006101c20061$d2eefaa0$e120b8c8@ig> From: "Josimar" To: References: <765A72978645D4118B1C0000E229806D05B6B393@FGVRJ23> <007901c20059$8a4a2700$b826fea9@Eder> Subject: [obm-l] =?iso-8859-1?Q?Re:_=5Bobm-l=5D_Re:_=5Bobm-l=5D_RES:_=5Bobm-l=5D_valor_m?= =?iso-8859-1?Q?=EDnimo?= Date: Mon, 20 May 2002 21:52:36 -0300 MIME-Version: 1.0 Content-Type: multipart/alternative; boundary="----=_NextPart_000_005B_01C20048.A737EA20" X-Priority: 3 X-MSMail-Priority: Normal X-Mailer: Microsoft Outlook Express 5.00.2615.200 X-MIMEOLE: Produced By Microsoft MimeOLE V5.00.2615.200 Sender: owner-obm-l@sucuri.mat.puc-rio.br Precedence: bulk Reply-To: obm-l@mat.puc-rio.br This is a multi-part message in MIME format. ------=_NextPart_000_005B_01C20048.A737EA20 Content-Type: text/plain; charset="iso-8859-1" Content-Transfer-Encoding: quoted-printable N=E3o =E9 um AR-15 pra matar uma formiga? Planificando o cubo, basta calcular a medida da diagonal de um = ret=E2ngulo de lados medindo 1 e 1+1=3D2.=20 resposta: sqrt(5). ----- Original Message -----=20 From: Eder=20 To: obm-l@mat.puc-rio.br=20 Sent: Monday, May 20, 2002 6:58 PM Subject: [obm-l] Re: [obm-l] RES: [obm-l] valor m=EDnimo Valeu Ralph, =20 =20 Essa express=E3o surgiu do seguinte problema: detrerminar o menor = caminho que uma formiguinha pode fazer por sobre a superf=EDcie de um = cubo de aresta 1,de um v=E9rtice a outro "diagonalmente oposto". =20 Eu admiti uma trajet=F3ria gen=E9rica e cheguei a esse valor para o = caminho.Eu j=E1 imaginava que a=3D1/2,mas queria provar algebricamente. ----- Original Message -----=20 From: Ralph Teixeira=20 To: 'obm-l@mat.puc-rio.br'=20 Sent: Monday, May 20, 2002 3:55 PM Subject: [obm-l] RES: [obm-l] valor m=EDnimo =C9 poss=EDvel sim. =20 1) Via c=E1lculo =20 Derive a express=E3o com rela=E7=E3o a "a", iguale a zero, d=E1 uma = equa=E7=E3o meio feia mas sai que a=3D1/2; =20 1.1) C=E1lculo incrementado Note que, se f(x)=3Dsqrt(1+(1-x)^2)+sqrt(1+x^2), ent=E3o = f(x)=3Df(1-x), isto =E9, o gr=E1fico da fun=E7=E3o =E9 sim=E9trico com = rela=E7=E3o =E0 reta x=3D1/2. Isto sugere fazer y=3Dx-1/2, e ent=E3o = f(y)=3Dsqrt(1+(y-1/2)^2)+sqrt(1+(y+1/2)^2). Os c=E1lculos aqui j=E1 = s=E3o um pouco mais simples... D=E1 at=E9 para fazer a partir daqui sem = c=E1lculo, com m=E1gica....=20 =20 2) Por geometria =20 ...mas se voc=EA quer uma maneira BEM m=E1gica de fazer, pense = assim: =20 f(a)=3Dsqrt(1+(1-a)^2)+sqrt(1+a^2) =E9 a soma das dist=E2ncias do = ponto (1,a) aos pontos (0,1) e (2,0). Em outras palavras, queremos o = ponto P(1,a) na reta x=3D1 que minimiza as somas das dist=E2ncias aos = pontos B(0,1) e C(2,0) -- que s=E3o fixos e se encontram um de cada lado = da reta! Ora, o menor caminho BPC =E9 o segmento de reta que liga B a C! = Assim, o m=EDnimo se d=E1 quando B,P e C est=E3o alinhados; note que, = ent=E3o, P ser=E1 o ponto m=E9dio de BC, isto =E9, a=3D1/2. =20 Legal? =20 Abra=E7o, Ralph =20 -----Mensagem original----- De: Eder [mailto:edalbuquerque@uol.com.br] Enviada em: sexta-feira, 17 de maio de 2002 21:34 Para: obm-l@mat.puc-rio.br Assunto: [obm-l] valor m=EDnimo Ol=E1, =C9 poss=EDvel determinar para que valor de a,tem-se y=3D sqrt( 1+ = (1-a)=B2) + sqrt(1+ a=B2) m=EDnimo? =20 =20 ------=_NextPart_000_005B_01C20048.A737EA20 Content-Type: text/html; charset="iso-8859-1" Content-Transfer-Encoding: quoted-printable
N=E3o =E9 um AR-15 pra matar uma = formiga?
Planificando o cubo, basta calcular a = medida da=20 diagonal de um ret=E2ngulo de lados medindo 1 e 1+1=3D2.
resposta: sqrt(5).
----- Original Message -----
From:=20 Eder
Sent: Monday, May 20, 2002 6:58 = PM
Subject: [obm-l] Re: [obm-l] = RES: [obm-l]=20 valor m=EDnimo

Valeu Ralph,
 
 
Essa express=E3o surgiu do seguinte = problema:=20 detrerminar o menor caminho que uma formiguinha pode fazer por sobre a = superf=EDcie de um cubo de aresta 1,de um v=E9rtice a outro = "diagonalmente=20 oposto".
 
Eu admiti uma trajet=F3ria gen=E9rica = e cheguei a=20 esse valor para o caminho.Eu j=E1 imaginava que a=3D1/2,mas queria = provar=20 algebricamente.
----- Original Message -----
From:=20 Ralph = Teixeira
To: 'obm-l@mat.puc-rio.br'
Sent: Monday, May 20, 2002 = 3:55=20 PM
Subject: [obm-l] RES: [obm-l] = valor=20 m=EDnimo

=C9=20 poss=EDvel sim.
 
1)=20 Via c=E1lculo
 
Derive a express=E3o com rela=E7=E3o a "a", iguale a zero, = d=E1 uma equa=E7=E3o=20 meio feia mas sai que a=3D1/2;
 
    1.1) C=E1lculo = incrementado
    Note que, se=20 f(x)=3Dsqrt(1+(1-x)^2)+sqrt(1+x^2), ent=E3o f(x)=3Df(1-x), isto =E9, = o gr=E1fico da=20 fun=E7=E3o =E9 sim=E9trico com rela=E7=E3o =E0 reta x=3D1/2. Isto = sugere fazer y=3Dx-1/2, e=20 ent=E3o f(y)=3Dsqrt(1+(y-1/2)^2)+sqrt(1+(y+1/2)^2).=20 Os c=E1lculos aqui j=E1 s=E3o um pouco mais simples... D=E1 = at=E9 para fazer a=20 partir daqui sem c=E1lculo, com m=E1gica....
 
2)=20 Por geometria
 
...mas se voc=EA quer uma maneira BEM m=E1gica = de fazer, pense=20 assim:
 
f(a)=3Dsqrt(1+(1-a)^2)+sqrt(1+a^2) =E9 a soma = das dist=E2ncias=20 do ponto (1,a) aos pontos (0,1) e (2,0). Em outras palavras, = queremos o=20 ponto P(1,a) na reta x=3D1 que minimiza as somas das dist=E2ncias = aos pontos=20 B(0,1) e C(2,0) -- que s=E3o fixos e se encontram um de cada lado da = reta!=20 Ora, o menor caminho BPC =E9 o segmento de reta que liga B a C! = Assim, o=20 m=EDnimo se d=E1 quando B,P e C est=E3o alinhados; note que, = ent=E3o, P ser=E1 o ponto=20 m=E9dio de BC, isto =E9, a=3D1/2.
 
Legal?
 
Abra=E7o,
        = Ralph
 
 -----Mensagem=20 original-----
De: Eder=20 [mailto:edalbuquerque@uol.com.br]
Enviada em: sexta-feira, = 17 de=20 maio de 2002 21:34
Para: = obm-l@mat.puc-rio.br
Assunto:=20 [obm-l] valor m=EDnimo
Ol=E1,
 =C9 poss=EDvel=20 determinar para que valor de a,tem-se y=3D sqrt( 1+ (1-a)=B2) + = sqrt(1+ a=B2)=20 m=EDnimo?
 
 
------=_NextPart_000_005B_01C20048.A737EA20-- ========================================================================= Instruções para entrar na lista, sair da lista e usar a lista em http://www.mat.puc-rio.br/~nicolau/olimp/obm-l.html O administrador desta lista é ========================================================================= From owner-obm-l@sucuri.mat.puc-rio.br Mon May 20 21:59:46 2002 Return-Path: Received: (from majordom@localhost) by sucuri.mat.puc-rio.br (8.9.3/8.9.3) id VAA29161 for obm-l-list; Mon, 20 May 2002 21:58:30 -0300 Received: from br.inter.net (opt-0-15.br.inter.net [200.185.56.15] (may be forged)) by sucuri.mat.puc-rio.br (8.9.3/8.9.3) with ESMTP id VAA29157 for ; Mon, 20 May 2002 21:58:28 -0300 Received: from dialup-200-184-32-225.intelignet.com.br ([200.184.32.225] helo=ig) by br.inter.net with esmtp (Exim 3.36 #1) id 179xlO-0003LE-00 for obm-l@mat.puc-rio.br; Mon, 20 May 2002 21:44:30 -0300 Message-ID: <006001c20061$d203fe60$e120b8c8@ig> From: "Josimar" To: References: <20020520230852.88886.qmail@web10104.mail.yahoo.com> Subject: Re: [obm-l] logaritmo de (-10)^2 Date: Mon, 20 May 2002 21:43:04 -0300 MIME-Version: 1.0 Content-Type: text/plain; charset="iso-8859-1" Content-Transfer-Encoding: 8bit X-Priority: 3 X-MSMail-Priority: Normal X-Mailer: Microsoft Outlook Express 5.00.2615.200 X-MIMEOLE: Produced By Microsoft MimeOLE V5.00.2615.200 Sender: owner-obm-l@sucuri.mat.puc-rio.br Precedence: bulk Reply-To: obm-l@mat.puc-rio.br log(x^n) = n*log x <==> x > 0. Assim como sqrt(x^2) = x <===> x>=0. sqrt[(-10)^2] = sqrt 100 = 10. A rigor, sqrt(x^2) = abs ( x ). []s, Josimar ----- Original Message ----- From: Rafael WC To: OBM Sent: Monday, May 20, 2002 8:08 PM Subject: [obm-l] logaritmo de (-10)^2 Oi Pessoal! Caiu uma questão num concurso só para professores de matemática ontem que me deixou intrigado: Dada a função f: f(x) = x + raiz(x^2) - log(base 10)(x^2) Calcule f(-10). A resposta foi -2. Mas depois da prova surgiu a maior discussão porque existia uma alternativa que era "f(-10) não está definida". O pessoal questinou que estando f(-10) definida, devia valer a propriedade do expoente de logaritmo e poderíamos escrever: f(x) = x + raiz(x^2) - log(base 10)(x^2) f(x) = x + raiz(x^2) - 2.log(base 10)(x) E aí vemos claramente que não podemos tirar o log de -10. Mas como o gabarito da comissão organizadora foi -2, ficamos todos na dúvida: está definida f(-10)??? Um abraço, Rafael. ===== Rafael Werneck Cinoto ICQ# 107011599 rwcinoto@yahoo.com rafael.caixa@gov.com.br matduvidas@yahoo.com.br http://www.rwcinoto.hpg.com.br/ __________________________________________________ Do You Yahoo!? LAUNCH - Your Yahoo! Music Experience http://launch.yahoo.com ========================================================================= Instruções para entrar na lista, sair da lista e usar a lista em http://www.mat.puc-rio.br/~nicolau/olimp/obm-l.html O administrador desta lista é ========================================================================= ========================================================================= Instruções para entrar na lista, sair da lista e usar a lista em http://www.mat.puc-rio.br/~nicolau/olimp/obm-l.html O administrador desta lista é ========================================================================= From owner-obm-l@sucuri.mat.puc-rio.br Mon May 20 23:00:19 2002 Return-Path: Received: (from majordom@localhost) by sucuri.mat.puc-rio.br (8.9.3/8.9.3) id WAA30492 for obm-l-list; Mon, 20 May 2002 22:59:54 -0300 Received: from traven.uol.com.br (traven.uol.com.br [200.231.206.184]) by sucuri.mat.puc-rio.br (8.9.3/8.9.3) with ESMTP id WAA30485 for ; Mon, 20 May 2002 22:59:51 -0300 Received: from Eder ([200.227.70.162]) by traven.uol.com.br (8.9.1/8.9.1) with SMTP id WAA22755 for ; Mon, 20 May 2002 22:36:45 -0300 (BRT) Message-ID: <000c01c20069$3caafc80$38b7fea9@Eder> From: "Eder" To: Subject: [obm-l] ?? Date: Mon, 20 May 2002 22:43:56 -0300 MIME-Version: 1.0 Content-Type: multipart/alternative; boundary="----=_NextPart_000_0009_01C2004F.D34499E0" X-Priority: 3 X-MSMail-Priority: Normal X-Mailer: Microsoft Outlook Express 5.00.2314.1300 X-MimeOLE: Produced By Microsoft MimeOLE V5.00.2314.1300 Sender: owner-obm-l@sucuri.mat.puc-rio.br Precedence: bulk Reply-To: obm-l@mat.puc-rio.br This is a multi-part message in MIME format. ------=_NextPart_000_0009_01C2004F.D34499E0 Content-Type: text/plain; charset="iso-8859-1" Content-Transfer-Encoding: quoted-printable Ol=E1, Ficarei muito grato a quem me ajuar com o seguinte problema: "Mostre que para todo m>0, sqrt(x)+m=3Dx tem exatamente uma raiz." =C9 do volume 1 da cole=E7=E3o Matem=E1tica para o ensino m=E9dio. ------=_NextPart_000_0009_01C2004F.D34499E0 Content-Type: text/html; charset="iso-8859-1" Content-Transfer-Encoding: quoted-printable
Ol=E1,
 
 
Ficarei muito grato a quem me = ajuar com o=20 seguinte problema:
 
"Mostre que para todo m>0, = sqrt(x)+m=3Dx tem=20 exatamente uma raiz."
 
=C9 do volume 1 da cole=E7=E3o = Matem=E1tica para o ensino=20 m=E9dio.
------=_NextPart_000_0009_01C2004F.D34499E0-- ========================================================================= Instruções para entrar na lista, sair da lista e usar a lista em http://www.mat.puc-rio.br/~nicolau/olimp/obm-l.html O administrador desta lista é ========================================================================= From owner-obm-l@sucuri.mat.puc-rio.br Tue May 21 01:39:57 2002 Return-Path: Received: (from majordom@localhost) by sucuri.mat.puc-rio.br (8.9.3/8.9.3) id BAA32000 for obm-l-list; Tue, 21 May 2002 01:39:45 -0300 Received: from imo-m07.mx.aol.com (imo-m07.mx.aol.com [64.12.136.162]) by sucuri.mat.puc-rio.br (8.9.3/8.9.3) with ESMTP id BAA31996 for ; Tue, 21 May 2002 01:39:42 -0300 From: DEOLIVEIRASOU@aol.com Received: from DEOLIVEIRASOU@aol.com by imo-m07.mx.aol.com (mail_out_v32.5.) id z.17d.89e3410 (3311) for ; Tue, 21 May 2002 00:27:37 -0400 (EDT) Message-ID: <17d.89e3410.2a1b26b8@aol.com> Date: Tue, 21 May 2002 00:27:36 EDT Subject: [obm-l] Trigo To: obm-l@mat.puc-rio.br MIME-Version: 1.0 Content-Type: multipart/alternative; boundary="part1_17d.89e3410.2a1b26b8_boundary" X-Mailer: AOL 7.0 for Windows BR sub 10501 Sender: owner-obm-l@sucuri.mat.puc-rio.br Precedence: bulk Reply-To: obm-l@mat.puc-rio.br --part1_17d.89e3410.2a1b26b8_boundary Content-Type: text/plain; charset="ISO-8859-1" Content-Transfer-Encoding: quoted-printable 1)Calcular o valor de tg20.tg40.tg80 2)Mostre que tg20+tg70=3D2sec50. Agrade=E7o quem ajudar nessas quest=F5es. --part1_17d.89e3410.2a1b26b8_boundary Content-Type: text/html; charset="ISO-8859-1" Content-Transfer-Encoding: quoted-printable 1)Calcular o valor de tg20.tg40.tg80
2)Mostre que tg20+tg70=3D2sec50.
Agrade=E7o quem ajudar nessas quest=F5es.
--part1_17d.89e3410.2a1b26b8_boundary-- ========================================================================= Instruções para entrar na lista, sair da lista e usar a lista em http://www.mat.puc-rio.br/~nicolau/olimp/obm-l.html O administrador desta lista é ========================================================================= From owner-obm-l@sucuri.mat.puc-rio.br Tue May 21 03:26:31 2002 Return-Path: Received: (from majordom@localhost) by sucuri.mat.puc-rio.br (8.9.3/8.9.3) id DAA00558 for obm-l-list; Tue, 21 May 2002 03:26:12 -0300 Received: from web21306.mail.yahoo.com (web21306.mail.yahoo.com [216.136.129.60]) by sucuri.mat.puc-rio.br (8.9.3/8.9.3) with SMTP id DAA00554 for ; Tue, 21 May 2002 03:26:09 -0300 Message-ID: <20020521061410.70914.qmail@web21306.mail.yahoo.com> Received: from [200.227.209.110] by web21306.mail.yahoo.com via HTTP; Tue, 21 May 2002 03:14:10 ART Date: Tue, 21 May 2002 03:14:10 -0300 (ART) From: =?iso-8859-1?q?Marcos=20Reynaldo?= Subject: Re: [obm-l] Trigo To: obm-l@mat.puc-rio.br In-Reply-To: <17d.89e3410.2a1b26b8@aol.com> MIME-Version: 1.0 Content-Type: text/plain; charset=iso-8859-1 Content-Transfer-Encoding: 8bit Sender: owner-obm-l@sucuri.mat.puc-rio.br Precedence: bulk Reply-To: obm-l@mat.puc-rio.br --- DEOLIVEIRASOU@aol.com escreveu: > 1)Calcular o valor de tg20.tg40.tg80 > 2)Mostre que tg20+tg70=2sec50. > Agradeço quem ajudar nessas questões. > ===================================================== 1) Vou pensar um pouco mais, na calculadora deu raiz de 3 (tg60). 2) Lembrando que tg20=cotg70, temos tg20+tg70=cotg70+tg70=(cos70/sen70)+(sen70+cos70)=1/(sen70.cos70)=2/(2sen70.cos70)=2/sen140=2/sen40=2/cos50=2sec50 que eh o que queriamos demonstrar. []'s Marcos _______________________________________________________________________ Yahoo! Encontros O lugar certo para você encontrar aquela pessoa que falta na sua vida. Cadastre-se hoje mesmo! http://br.encontros.yahoo.com/ ========================================================================= Instruções para entrar na lista, sair da lista e usar a lista em http://www.mat.puc-rio.br/~nicolau/olimp/obm-l.html O administrador desta lista é ========================================================================= From owner-obm-l@sucuri.mat.puc-rio.br Tue May 21 06:48:51 2002 Return-Path: Received: (from majordom@localhost) by sucuri.mat.puc-rio.br (8.9.3/8.9.3) id GAA02812 for obm-l-list; Tue, 21 May 2002 06:48:24 -0300 Received: from hotmail.com (f38.pav1.hotmail.com [64.4.31.38]) by sucuri.mat.puc-rio.br (8.9.3/8.9.3) with ESMTP id GAA02808 for ; Tue, 21 May 2002 06:48:21 -0300 Received: from mail pickup service by hotmail.com with Microsoft SMTPSVC; Tue, 21 May 2002 02:36:21 -0700 Received: from 200.241.125.162 by pv1fd.pav1.hotmail.msn.com with HTTP; Tue, 21 May 2002 09:36:21 GMT X-Originating-IP: [200.241.125.162] From: "Felipe Marinho" To: obm-l@mat.puc-rio.br Subject: Re: [obm-l] Trigo Date: Tue, 21 May 2002 05:36:21 -0400 Mime-Version: 1.0 Content-Type: text/plain; charset=iso-8859-1; format=flowed Message-ID: X-OriginalArrivalTime: 21 May 2002 09:36:21.0937 (UTC) FILETIME=[F7AA5210:01C200AA] Sender: owner-obm-l@sucuri.mat.puc-rio.br Precedence: bulk Reply-To: obm-l@mat.puc-rio.br Caro amigo DEOLIVEIRASOU, Vou tentar ajudá-lo apenas na resolução da 1a. questão, pois já notifiquei que outro nosso colega da lista o ajudou em demonstrar o valor da 2a. questão. Então, vamos lá: ---------------- Resolvi a questão apenas com a fórmula abaixo: tg(3x) = tg(x).tg(60-x).tg(60+x) Fazendo, então, x=20, teríamos: tg(60) = tg(20).tg(40).tg(80) Ou seja: tg(20).tg(40).tg(80) = tg(60) tg(20).tg(40).tg(80) = sqrt(3). ------- FIM ------- Bem, e como sei tambem que você gostaria da demonstração da fórmula, tentarei lhe ajudar mais uma vez. Então, vamos lá: Sabendo que: tg(a+b) = [tg(a) + tg(b)]/[1 - tg(a).tg(b)] tg(a-b) = [tg(a) - tg(b)]/[1 + tg(a).tg(b)] Vamos agora calcular quanto vale tg(3x): tg(3x) = [tg(x) + tg(2x)]/[1 - tg(x).tg(2x)] tg(3x) = (tg(x) + tg(x) + tg(x) ) (1 - tg(x).[tg(x) + tg(x)]) ( ------------- ) / ( -------------- ) ( [1 - tg²(x)] ) ( [1 - tg²(x)] ) Para nos facilitar, chamemos tg(x)=a. Então, temos: tg(3x) = [a + 2a/(1-a²)]/[1 - a(2a)/(1-a²)] tg(3x) = [(a-a³+2a)/(1-a²)]/[(1-a²-2a²)/(1-a²)] tg(3x) = [(-a³+3a)/(1-a²)]/[(1-3a²)/(1-a²) tg(3x) = [(-a³+3a)/(1-3a²)] x(-1) tg(3x) = [(a³-3a)/(3a²-1)] tg(3x) = [a(a²-3)/(3a²-1)] tg(3x) = a(a + sqrt[3])(a - sqrt[3]) / (sqrt(3)a + 1)(sqrt(3) - 1) E como a = tg(x), temos: tg(3x) = tg(x). (tg(x) + sqrt[3])(tg(x) - sqrt[3]) ----------------------------------- (sqrt[3)tg(x) + 1)(sqrt[3]tg(x) - 1) Sabemos que sqrt(3) = tg(60), não é verdade ? Multiplicando o numerador e o dividendo por (-1), temos: tg(3x) = tg(x). [tg(x)+tg(60)][tg(60) - tg(x)] ---------------------------------- [1 + tg(60)tg(x)][1 - tg(60)tg(x)] Bem, agora fica fácil ver que podemos simplificar a fração, pois sabemos que: tg(60+x)=[tg(60)+tg(x)]/[1-tg(60)tg(x)] tg(60-x)=[tg(60)-tg(x)]/[1+tg(60)tg(x)] E, com isso, temos que: tg(3x) = tg(x).tg(60+x).tg(60-x) (c.q.d) ----- FIM ------ Caro amigo, o problema em questão limita-se a fazer x=20, para termos: tg(60) = tg(20).tg(80).tg(40) ----------------------------- Bem amigo, de coração, espero ter ajudado! Desculpe qualquer coisa. Um grande abraço, Felipe Marinho. >From: DEOLIVEIRASOU@aol.com >Reply-To: obm-l@mat.puc-rio.br >To: obm-l@mat.puc-rio.br >Subject: [obm-l] Trigo >Date: Tue, 21 May 2002 00:27:36 EDT > >1)Calcular o valor de tg20.tg40.tg80 >2)Mostre que tg20+tg70=2sec50. >Agradeço quem ajudar nessas questões. _________________________________________________________________ O MSN Photos é o modo mais fácil de compartilhar e imprimir suas fotos: http://photos.msn.com/support/worldwide.aspx ========================================================================= Instruções para entrar na lista, sair da lista e usar a lista em http://www.mat.puc-rio.br/~nicolau/olimp/obm-l.html O administrador desta lista é ========================================================================= From owner-obm-l@sucuri.mat.puc-rio.br Tue May 21 08:45:45 2002 Return-Path: Received: (from majordom@localhost) by sucuri.mat.puc-rio.br (8.9.3/8.9.3) id IAA04222 for obm-l-list; Tue, 21 May 2002 08:44:02 -0300 Received: (from nicolau@localhost) by sucuri.mat.puc-rio.br (8.9.3/8.9.3) id IAA04217 for obm-l@mat.puc-rio.br; Tue, 21 May 2002 08:44:01 -0300 Date: Tue, 21 May 2002 08:44:01 -0300 From: "Nicolau C. Saldanha" To: obm-l@mat.puc-rio.br Subject: [obm-l] Re: =?iso-8859-1?Q?=5Bobm-l=5D_En:_=5Bobm-l=5D_Re:_=5Bobm-l=5D_Re:_Sob_que_c?= =?iso-8859-1?Q?ondi=E7oes_uma_deformacao_preserva_medidas?= Message-ID: <20020521084401.C4118@sucuri.mat.puc-rio.br> References: <00f801c2002f$1413cca0$2902dcc8@josefran> Mime-Version: 1.0 Content-Type: text/plain; charset=iso-8859-1 Content-Disposition: inline Content-Transfer-Encoding: 8bit User-Agent: Mutt/1.2.5i In-Reply-To: <00f801c2002f$1413cca0$2902dcc8@josefran>; from jfgcosta@unisys.com.br on Mon, May 20, 2002 at 03:48:43PM -0300 Sender: owner-obm-l@sucuri.mat.puc-rio.br Precedence: bulk Reply-To: obm-l@mat.puc-rio.br On Mon, May 20, 2002 at 03:48:43PM -0300, Jose Francisco Guimaraes Costa wrote: > Rearrumá-las sem deformá-las? Exato. > > V poderia dar uma idéia da demonstração, isto é, a linha geral seguida na demonstração? Acho difícil. > > Alguém saberia dizer se as publicações citadas podem ser encontradas em alguma biblioteca no Rio, que seja aberta a público externo? Um monte. Por exemplo, na PUC, no IMPA, ... []s, N. ... > : On Sun, 4 Feb 2001, Nicolau C. Saldanha wrote: > : > : > : > Aliás um grande problema da matemática do século XX foi o da quadratura > : > do círculo: não aquele proposto pelos gregos e cuja demostração foi > : > concluída com a prova da transcendência de pi. O problema século XX > : > da quadratura do círculo é: será possível decompor um círculo de área 1 > : > em um número finito de peças e rearrumá-las para formar um quadrado > : > de área 1? A resposta é que sim, é possível. > : > > : > []s, N. > : > > : > > : > > Isto foi provado por Miklos Laczkovich: > > M. Laczkovich, Equidecomposability and discrepancy; a solution of Tarski's circle-squaring problem, Journal für die Reine und Angewandte Mathematik, 403 (1990) 77-117 > > Veja também, > > R. J. Gardner and S. Wagon, At long last, the circle has been squared, Notices of the American Mathematical Society, 36 (1989) - 1338-1343 > ========================================================================= Instruções para entrar na lista, sair da lista e usar a lista em http://www.mat.puc-rio.br/~nicolau/olimp/obm-l.html O administrador desta lista é ========================================================================= From owner-obm-l@sucuri.mat.puc-rio.br Tue May 21 08:51:22 2002 Return-Path: Received: (from majordom@localhost) by sucuri.mat.puc-rio.br (8.9.3/8.9.3) id IAA04347 for obm-l-list; Tue, 21 May 2002 08:50:05 -0300 Received: (from nicolau@localhost) by sucuri.mat.puc-rio.br (8.9.3/8.9.3) id IAA04342 for obm-l@mat.puc-rio.br; Tue, 21 May 2002 08:50:05 -0300 Date: Tue, 21 May 2002 08:50:05 -0300 From: "Nicolau C. Saldanha" To: obm-l@mat.puc-rio.br Subject: [obm-l] IMO 2002 Message-ID: <20020521085005.D4118@sucuri.mat.puc-rio.br> Mime-Version: 1.0 Content-Type: text/plain; charset=iso-8859-1 Content-Disposition: inline Content-Transfer-Encoding: 8bit User-Agent: Mutt/1.2.5i Sender: owner-obm-l@sucuri.mat.puc-rio.br Precedence: bulk Reply-To: obm-l@mat.puc-rio.br A equipe Brasileira que participará da IMO-2002 (19 a 30 de julho de 2002, Glasgow - UK) é a seguinte: Líder da delegação: Prof. Edmilson Motta (São Paulo-SP) Vice-líder da delegação: Prof. Ralph Teixeira (Niterói-RJ) Equipe (em ordem alfabética): BRA1: Alex Correa Abreu (Niterói-RJ) BRA2: Davi Maximo Alexandrino Nogueira (Fortaleza-CE) BRA3: Guilherme Issao Camarinha Fujiwara (São Paulo-SP) BRA4: Larissa Cavalcante Queiroz de Lima (Fortaleza-CE) BRA5: Thiago da Silva Sobral (Fortaleza-CE) BRA6: Yuri Gomes Lima (Fortaleza-CE) Boa sorte para nossos representantes. []s, N. ========================================================================= Instruções para entrar na lista, sair da lista e usar a lista em http://www.mat.puc-rio.br/~nicolau/olimp/obm-l.html O administrador desta lista é ========================================================================= From owner-obm-l@sucuri.mat.puc-rio.br Tue May 21 09:01:50 2002 Return-Path: Received: (from majordom@localhost) by sucuri.mat.puc-rio.br (8.9.3/8.9.3) id JAA04695 for obm-l-list; Tue, 21 May 2002 09:01:44 -0300 Received: (from nicolau@localhost) by sucuri.mat.puc-rio.br (8.9.3/8.9.3) id JAA04690 for obm-l@mat.puc-rio.br; Tue, 21 May 2002 09:01:44 -0300 Date: Tue, 21 May 2002 09:01:44 -0300 From: "Nicolau C. Saldanha" To: obm-l@mat.puc-rio.br Subject: Re: [obm-l] logaritmo de (-10)^2 Message-ID: <20020521090144.E4118@sucuri.mat.puc-rio.br> References: <20020520230852.88886.qmail@web10104.mail.yahoo.com> <006001c20061$d203fe60$e120b8c8@ig> Mime-Version: 1.0 Content-Type: text/plain; charset=iso-8859-1 Content-Disposition: inline Content-Transfer-Encoding: 8bit User-Agent: Mutt/1.2.5i In-Reply-To: <006001c20061$d203fe60$e120b8c8@ig>; from josimat@openlink.com.br on Mon, May 20, 2002 at 09:43:04PM -0300 Sender: owner-obm-l@sucuri.mat.puc-rio.br Precedence: bulk Reply-To: obm-l@mat.puc-rio.br On Mon, May 20, 2002 at 09:43:04PM -0300, Josimar wrote: > log(x^n) = n*log x <==> x > 0. > Assim como sqrt(x^2) = x <===> x>=0. > sqrt[(-10)^2] = sqrt 100 = 10. > A rigor, sqrt(x^2) = abs ( x ). > []s, Josimar > ----- Original Message ----- > From: Rafael WC > To: OBM > Sent: Monday, May 20, 2002 8:08 PM > Subject: [obm-l] logaritmo de (-10)^2 > > > Oi Pessoal! > > Caiu uma questão num concurso só para professores de > matemática ontem que me deixou intrigado: > Dada a função f: > f(x) = x + raiz(x^2) - log(base 10)(x^2) > > Calcule f(-10). > > A resposta foi -2. Mas depois da prova surgiu a maior > discussão porque existia uma alternativa que era > "f(-10) não está definida". > > O pessoal questinou que estando f(-10) definida, devia > valer a propriedade do expoente de logaritmo e > poderíamos escrever: > f(x) = x + raiz(x^2) - log(base 10)(x^2) > f(x) = x + raiz(x^2) - 2.log(base 10)(x) > > E aí vemos claramente que não podemos tirar o log de > -10. Mas como o gabarito da comissão organizadora foi > -2, ficamos todos na dúvida: está definida f(-10)??? A resposta do Josimar (e uma outra que eu já esqueci de quem era) foi boa, e a opção correta claramente é f(-10) = -2. ...mas só para tumultuar um pouco, vou questionar a frase "claramente não podemos tirar o log de (-10)"; log(x) está definido para números negativos sim, desde que permitamos respostas complexas. Isto vem de exp(a + ib) = exp(a) (cos(b) + i sen(b)) donde por exemplo exp(Pi i) = -1 e descobrimos que (Pi i) é um logaritmo de (-1). Digo um logaritmos pq aqui temos o mesmo problema que para raízes quadradas: dado um número complexo w há mais de um número complexo z com exp(z) = w. É preciso escolher um valor favorito para que log(z) seja o nome de um número. []s, N. ========================================================================= Instruções para entrar na lista, sair da lista e usar a lista em http://www.mat.puc-rio.br/~nicolau/olimp/obm-l.html O administrador desta lista é ========================================================================= From owner-obm-l@sucuri.mat.puc-rio.br Tue May 21 09:14:03 2002 Return-Path: Received: (from majordom@localhost) by sucuri.mat.puc-rio.br (8.9.3/8.9.3) id JAA05250 for obm-l-list; Tue, 21 May 2002 09:13:59 -0300 Received: (from nicolau@localhost) by sucuri.mat.puc-rio.br (8.9.3/8.9.3) id JAA05245 for obm-l@mat.puc-rio.br; Tue, 21 May 2002 09:13:59 -0300 Date: Tue, 21 May 2002 09:13:59 -0300 From: "Nicolau C. Saldanha" To: obm-l@mat.puc-rio.br Subject: [obm-l] Re: =?iso-8859-1?Q?=5Bobm-l=5D_Re:_=5Bobm-l=5D_RES:_=5Bobm-l=5D_valor_m=EDni?= =?iso-8859-1?Q?mo?= Message-ID: <20020521091359.F4118@sucuri.mat.puc-rio.br> References: <765A72978645D4118B1C0000E229806D05B6B393@FGVRJ23> <007901c20059$8a4a2700$b826fea9@Eder> Mime-Version: 1.0 Content-Type: text/plain; charset=iso-8859-1 Content-Disposition: inline Content-Transfer-Encoding: 8bit User-Agent: Mutt/1.2.5i In-Reply-To: <007901c20059$8a4a2700$b826fea9@Eder>; from edalbuquerque@uol.com.br on Mon, May 20, 2002 at 06:58:24PM -0300 Sender: owner-obm-l@sucuri.mat.puc-rio.br Precedence: bulk Reply-To: obm-l@mat.puc-rio.br On Mon, May 20, 2002 at 06:58:24PM -0300, Eder wrote: > Valeu Ralph, > > > Essa expressão surgiu do seguinte problema: detrerminar o menor caminho que > uma formiguinha pode fazer por sobre a superfície de um cubo de aresta 1,de > um vértice a outro "diagonalmente oposto". Não acompanhei a conversa toda, posso estar repetindo o que alguém já falou, mas o problema da formiguinha pode ser resolvido usando simplesmente que a distância mais curta entre dois pontos é a linha reta, sem conta nenhuma. Imagine o cubo pendurado por um vértice (que fica em cima). Imagine que a formiga inicialmente está no vértice de cima e deseja chegar no vértice de baixo. Há três faces em cima e três em baixo e um hexágono não planar em zigue-zague de arestas separando as três faces de cima das três de baixo. Claramente que a distância mais curta de um qualquer ponto do zigue-zague até o vértice de cima é uma linha reta que só toca o zigue-zague na ponta; idem para o vértice de baixo. Claramente a formiga cruza o zigue-zague; como ela segue o caminho mais curto, ela cruza o zigue-zague em um único ponto; este ponto está sobre uma das seis arestas (talvez na ponta). Como as seis arestas são exatamente iguais (ou melhor, há isometrias do cubo preservando os vértices de cima e de baixo que levam qualquer aresta em qualquer outra) podemos escolher uma aresta e supor que a formiga passa por ali. Mas agora a formiga está resolvendo um problema essencialmente planar: há dois quadrados colados por um lado comum e desdobrar a superfície para colocar os dois quadrados no plano não muda em nada a vida da formiga. Moral: a formiga anda em linha reta (no seu ponto de vista) e passa pelo meio de uma das seis arestas do zigue-zague. []s, N. ========================================================================= Instruções para entrar na lista, sair da lista e usar a lista em http://www.mat.puc-rio.br/~nicolau/olimp/obm-l.html O administrador desta lista é ========================================================================= From owner-obm-l@sucuri.mat.puc-rio.br Tue May 21 10:25:44 2002 Return-Path: Received: (from majordom@localhost) by sucuri.mat.puc-rio.br (8.9.3/8.9.3) id KAA07101 for obm-l-list; Tue, 21 May 2002 10:24:41 -0300 Received: from hotmail.com (f193.sea1.hotmail.com [207.68.163.193]) by sucuri.mat.puc-rio.br (8.9.3/8.9.3) with ESMTP id KAA07097 for ; Tue, 21 May 2002 10:24:38 -0300 Received: from mail pickup service by hotmail.com with Microsoft SMTPSVC; Tue, 21 May 2002 06:12:17 -0700 Received: from 200.211.119.214 by sea1fd.sea1.hotmail.msn.com with HTTP; Tue, 21 May 2002 13:12:16 GMT X-Originating-IP: [200.211.119.214] From: "Frederico Reis Marques de Brito" To: obm-l@mat.puc-rio.br Subject: Re: [obm-l] ?? Date: Tue, 21 May 2002 10:12:16 -0300 Mime-Version: 1.0 Content-Type: text/plain; charset=iso-8859-1; format=flowed Message-ID: X-OriginalArrivalTime: 21 May 2002 13:12:17.0190 (UTC) FILETIME=[21990C60:01C200C9] Sender: owner-obm-l@sucuri.mat.puc-rio.br Precedence: bulk Reply-To: obm-l@mat.puc-rio.br Oi Eder. Uma das piores coisas em Matemática é quando superestimamos um problema, isto é, quando começamos imaginando que ele é muito difícil, mas na verdade não é. Isto faz com que busquemos soluções sofisticadas, usando ferramentas pesadas da Matemática, o que nos desvia de caminhos mais naturais. Com esta questão me ocorreu isto. Fiquei uns 10 minutos, usando todo o ferramentário do cálculo diferencial, provando a injetividade da função em certos intervalos, calculando limites, na tentativa de provar a existência da raiz via Teorema do Valor Intermediário. Mas, em seguida vi que estava dando " bobeira" . Note o seguinte: x - sqrt(x) = m <=> sqrt(x) . ( sqrt(x) - 1 ) = m . Faça sqrt(x) = y => y . ( y - 1 ) = m <=> y^2 - y - m = 0 . REsolvendo esta eq. do 2o grau, obtemos DElta = 1 + 4m > 0 , pois m> 0 e analisando as raízes y' e y'' , vemos que só uma delas é positiva, e já que y = sqrt(x) > 0 , segue a unicidade. É só escrever tudo com detalhes, para se obter uma solução formalzinha. Um abraço, FRed. >From: "Eder" >Reply-To: obm-l@mat.puc-rio.br >To: >Subject: [obm-l] ?? >Date: Mon, 20 May 2002 22:43:56 -0300 > >Olá, > > >Ficarei muito grato a quem me ajuar com o seguinte problema: > >"Mostre que para todo m>0, sqrt(x)+m=x tem exatamente uma raiz." > >É do volume 1 da coleção Matemática para o ensino médio. Eder. _________________________________________________________________ Converse com amigos on-line, conheça o MSN Messenger: http://messenger.msn.com ========================================================================= Instruções para entrar na lista, sair da lista e usar a lista em http://www.mat.puc-rio.br/~nicolau/olimp/obm-l.html O administrador desta lista é ========================================================================= From owner-obm-l@sucuri.mat.puc-rio.br Tue May 21 10:40:46 2002 Return-Path: Received: (from majordom@localhost) by sucuri.mat.puc-rio.br (8.9.3/8.9.3) id KAA07395 for obm-l-list; Tue, 21 May 2002 10:40:25 -0300 Received: from pina.terra.com.br (pina.terra.com.br [200.176.3.17]) by sucuri.mat.puc-rio.br (8.9.3/8.9.3) with ESMTP id KAA07392 for ; Tue, 21 May 2002 10:40:22 -0300 Received: from smtp2-bra.terra.com.br (smtp2-bra.terra.com.br [200.176.3.33]) by pina.terra.com.br (Postfix) with ESMTP id ED7F352EB9 for ; Tue, 21 May 2002 10:28:24 -0300 (EST) Received: from Lucelindo (unknown [200.177.64.96]) (authenticated user ldias1) by smtp2-bra.terra.com.br (Postfix) with ESMTP id AF1E1885D1 for ; Tue, 21 May 2002 10:28:23 -0300 (EST) Message-ID: <005101c200ca$cd169c00$6040b1c8@Lucelindo> From: "Lucelindo D. Ferreira" To: References: <000c01c20069$3caafc80$38b7fea9@Eder> Subject: Re: [obm-l] ?? Date: Tue, 21 May 2002 10:24:11 -0300 MIME-Version: 1.0 Content-Type: multipart/alternative; boundary="----=_NextPart_000_004E_01C200B1.A6274CC0" X-Priority: 3 X-MSMail-Priority: Normal X-Mailer: Microsoft Outlook Express 5.00.2615.200 X-MimeOLE: Produced By Microsoft MimeOLE V5.00.2615.200 Sender: owner-obm-l@sucuri.mat.puc-rio.br Precedence: bulk Reply-To: obm-l@mat.puc-rio.br This is a multi-part message in MIME format. ------=_NextPart_000_004E_01C200B1.A6274CC0 Content-Type: text/plain; charset="iso-8859-1" Content-Transfer-Encoding: quoted-printable Oba Eder! Td OKey? Bom, pelo m=E9todo da mudan=E7a de vari=E1vel: u =3D sqrtx, fica u + m =3D u^2, logo temos u^2 -u -m =3D 0 Suas prov=E1veis ra=EDzes em R s=E3o [1 + sqrt(4m + 1)]/2 e [1 - sqrt(4m = + 1)]/2=20 A segunda ra=EDz =F1 satisfaz a condi=E7=E3o de u >=3D 0 para todo m( = s=F3 para valores de m menores ou igual a 3/4).Logo a eq em u s=F3 tem = uma solu=E7=E3o em R e portanto a eq em x tem apenas uma solu=E7=E3o. Int=E9!! FABIO ----- Original Message -----=20 From: Eder=20 To: obm-l@mat.puc-rio.br=20 Sent: Monday, May 20, 2002 10:43 PM Subject: [obm-l] ?? Ol=E1, =20 =20 Ficarei muito grato a quem me ajuar com o seguinte problema: =20 "Mostre que para todo m>0, sqrt(x)+m=3Dx tem exatamente uma raiz." =20 =C9 do volume 1 da cole=E7=E3o Matem=E1tica para o ensino m=E9dio. ------=_NextPart_000_004E_01C200B1.A6274CC0 Content-Type: text/html; charset="iso-8859-1" Content-Transfer-Encoding: quoted-printable
Oba Eder! Td OKey? Bom, pelo = m=E9todo da mudan=E7a=20 de vari=E1vel:
u =3D sqrtx, fica
u + m =3D u^2, logo temos  u^2 -u = -m =3D=20 0
Suas prov=E1veis ra=EDzes em R = s=E3o [1 + sqrt(4m +=20 1)]/2 e [1 - sqrt(4m +=20 1)]/2 
A segunda ra=EDz =F1 satisfaz a = condi=E7=E3o de u >=3D=20 0 para todo m( s=F3 para valores de m menores ou igual a = 3/4).Logo=20 a eq em u s=F3 tem uma solu=E7=E3o em R e portanto a eq em x tem apenas = uma=20 solu=E7=E3o.
Int=E9!!
      FABIO
----- Original Message -----
From:=20 Eder
Sent: Monday, May 20, 2002 = 10:43 PM
Subject: [obm-l] ??

Ol=E1,
 
 
Ficarei muito grato a quem me = ajuar com o=20 seguinte problema:
 
"Mostre que para todo m>0, = sqrt(x)+m=3Dx tem=20 exatamente uma raiz."
 
=C9 do volume 1 da cole=E7=E3o = Matem=E1tica para o ensino=20 m=E9dio.
------=_NextPart_000_004E_01C200B1.A6274CC0-- ========================================================================= Instruções para entrar na lista, sair da lista e usar a lista em http://www.mat.puc-rio.br/~nicolau/olimp/obm-l.html O administrador desta lista é ========================================================================= From owner-obm-l@sucuri.mat.puc-rio.br Tue May 21 11:15:40 2002 Return-Path: Received: (from majordom@localhost) by sucuri.mat.puc-rio.br (8.9.3/8.9.3) id LAA08174 for obm-l-list; Tue, 21 May 2002 11:13:59 -0300 Received: from mat.puc-rio.br (IDENT:root@perere.mat.puc-rio.br [139.82.27.60]) by sucuri.mat.puc-rio.br (8.9.3/8.9.3) with ESMTP id LAA08170 for ; Tue, 21 May 2002 11:13:56 -0300 Received: from localhost (fredpalm@localhost) by mat.puc-rio.br (8.9.3/8.9.3) with ESMTP id DAA01012 for ; Tue, 21 May 2002 03:12:18 -0300 Date: Tue, 21 May 2002 03:12:17 -0300 (BRT) From: Carlos Frederico Borges Palmeira To: obm-l@mat.puc-rio.br Subject: Re: [obm-l] Re: =?iso-8859-1?Q?=5Bobm-l=5D_Re:_=5Bobm-l=5D_RES:_=5Bobm-l=5D_valor_m=EDni?= =?iso-8859-1?Q?mo?= In-Reply-To: <20020521091359.F4118@sucuri.mat.puc-rio.br> Message-ID: MIME-Version: 1.0 Content-Type: TEXT/PLAIN; charset=X-UNKNOWN Content-Transfer-Encoding: 8bit X-MIME-Autoconverted: from QUOTED-PRINTABLE to 8bit by sucuri.mat.puc-rio.br id LAA08171 Sender: owner-obm-l@sucuri.mat.puc-rio.br Precedence: bulk Reply-To: obm-l@mat.puc-rio.br On Tue, 21 May 2002, Nicolau C. Saldanha wrote: > > > > Essa expressão surgiu do seguinte problema: detrerminar o menor caminho que > > uma formiguinha pode fazer por sobre a superfície de um cubo de aresta 1,de > > um vértice a outro "diagonalmente oposto". > > Não acompanhei a conversa toda, posso estar repetindo o que alguém já falou, > mas o problema da formiguinha pode ser resolvido usando simplesmente que > a distância mais curta entre dois pontos é a linha reta, sem conta nenhuma. Uma solucao alternativa e' pensar num cubo de papel, e abri-lo, obtendo uma cruz. O que se quer e' ligar 2 vertices opostos de um retangulo formado por dois quadrados. E' facil ver que a diagonal deste retangulo passa no meio do lado comum aos dois quadrados, o que da' o caminho da formiga. Fred Palmeira > ========================================================================= > Instruções para entrar na lista, sair da lista e usar a lista em > http://www.mat.puc-rio.br/~nicolau/olimp/obm-l.html > O administrador desta lista é > ========================================================================= > ========================================================================= Instruções para entrar na lista, sair da lista e usar a lista em http://www.mat.puc-rio.br/~nicolau/olimp/obm-l.html O administrador desta lista é ========================================================================= From owner-obm-l@sucuri.mat.puc-rio.br Tue May 21 13:42:22 2002 Return-Path: Received: (from majordom@localhost) by sucuri.mat.puc-rio.br (8.9.3/8.9.3) id NAA11231 for obm-l-list; Tue, 21 May 2002 13:40:41 -0300 Received: from hotmail.com (law2-f124.hotmail.com [216.32.181.124]) by sucuri.mat.puc-rio.br (8.9.3/8.9.3) with ESMTP id NAA11227 for ; Tue, 21 May 2002 13:40:38 -0300 Received: from mail pickup service by hotmail.com with Microsoft SMTPSVC; Tue, 21 May 2002 09:28:39 -0700 Received: from 32.94.119.254 by lw2fd.hotmail.msn.com with HTTP; Tue, 21 May 2002 16:28:39 GMT X-Originating-IP: [32.94.119.254] From: "Paulo Santa Rita" To: obm-l@mat.puc-rio.br Subject: [obm-l] =?iso-8859-1?B?UmU6IFtvYm0tbF0gcGVybXV0Yef1ZXMgY2lyY3VsYXJlcyBjb20gcmVw?= =?iso-8859-1?B?ZXRp5+Nv?= Date: Tue, 21 May 2002 16:28:39 +0000 Mime-Version: 1.0 Content-Type: text/plain; charset=iso-8859-1; format=flowed Message-ID: X-OriginalArrivalTime: 21 May 2002 16:28:39.0786 (UTC) FILETIME=[909284A0:01C200E4] Sender: owner-obm-l@sucuri.mat.puc-rio.br Precedence: bulk Reply-To: obm-l@mat.puc-rio.br Ola Rafael e demais colegas desta lista, O unico TEOREMA DE MOREAU que eu conheco e aquele. Eu nunca vi aquele teorema ser aplicado para resolver problemas do tipo que voce apresentou. Todavia, o Prof Morgado explicitamente cita TEOREMA DE MOREAU. Segue que : 1) O teorema de moreau QUE EU CONHECO tem aplicacoes QUE EU NAO CONHECO. 2) Existe um teorema de moreau QUE EU NAO CONHECO que tem aplicacoes QUE EU CONHECO. Independente de tudo isso, com os modestos conhecimentos habituais de Analise Combinatoria podemos abordar o problema que voce propos ... A - O MISTERIO Se alguem lhe perguntar de quantas formas distintas N pessoas podem sentar em uma mesa redonda, incontinenti voce respondera : de (N-1)! maneiras. E a conhecidissima formula de permutacoes circulares ... COM ELEMENTOS, DOIS A DOIS, DISTINTOS ! Bom ... Por que essa formula e assim ? Dentre um montao de respostas igualmente validas, e digna de destaque aquela que diz que A CADA PERMUTACAO CIRCULAR DE N ELEMENTOS,DOIS A DOIS DISTINTOS correspondem N PERMUTACOES LINEARES. Logo, calculando o total possivel de permutacoes lineares, basta dividir esse total por N para obtermos o total de permutacoes circulares : N*PC = PL, PL=N! => PC= N!/N => PC=(N-1)! Um exemplo : A permutacao circular ABCD correspondem as permutacoes lineares ABCD, BCDA, CDAB e DABC. E isto e, evidentemente, absolutamente geral ... UM PERMUTACAO CIRCULAR E( OU PODE SER DEFINIDA COMO ), EM VERDADE, UM CONJUNTO DE PERMUTACOES LINEARES ... Mas, e preciso tomar cuidado ... Quando entram elementos repetidos, nem sempre a quantidade de permutacoes lineares que correspondem a uma dada permutacao circular e constante ! Para ver isso, considere a permutacao circular : 1) ABAB. Correspondem a ela as seguintes permutacoes lineares : ABAB e BABA. Duas portanto ! 2) AABB. Correspondem a ela as seguintes permutacoes lineares : AABB, ABBA, BBAA, BAAB. Quatro portanto ! Portanto, os mesmos objetos - A,A,B,B - distribuidos ao longo de um circulo de duas maneiras diferentes geraram quantidades distintas de permutacoes lineares. Isso so ocorre, claramente, quando nas permutacoes circulares entram objetos repetidos . Bom, e dai ? O que fazer ? Considerando que : 1) Calcular e construir permutracoes lineares com elementos repetidos e um problema facil e ja bem resolvido. 2) Cada permutacao circular corresponde a um conjunto de permutacoes lineares O que se deve fazer e estudar, observando e caracterizando com precisao todos os fenomenos que ocorrem e, so entao, passar a uma possivel explicacao. B - OS FENOMENOS 1) E facilmente observavel e pode-se confirmar com inumeras experiencias o seguinte : se uma permutacao circular pode ser expressa como multiplo de um de seus SUB-GRUPOS entao a quantidade de permutacoes lineares geradas sera igual a extensao do SUB-GRUPO. Exemplo : ABBCABBC = 2(ABBC) A permutacao acima gera apenas 4 permutacoes lineares : ABBCABBC, BBCABBCA, BCABBCAB, CABBCABB Daqui se conclui, imediatamente, que se a multiplicidade dos elementos de uma permutacao circular tem MDC igual a 1 entao, mesmo havendo elementos repetidos, a quantidade de permutacoes lineares geradas sera sempre igual a extensao da permutacao circular, isto e : N*PC = PL => PC = PL/N como PL = (M1+M2+...+Mn)!/(M1!*M2!*...*Mn!) segue que : PC = (M1+M2+...+Mn - 1)!/(M1!*M2!*...*Mn!) Onde Mi sao as multiplicidades dos objetos e M1+M2+...+Mn=N 2) Se o MDC entre as multiplidades nao e um entao a permutacao circular pode ser expressa sucessivamente como multiplo dos diversos divisores do MDC e essas serao as unicas permutacoes circulares para as quais ha a contracao no numero de permutacoes lineares geradas. Exemplo : ABABABAB = 4(AB) implica nas PL's ABABABAB, BABABABA AABBAABB = 2(AABB) implica nas PL's AABBAABB, ABBAABBA, BBAABBAA, BAABBAAB Nos caso em que o MDC das Multiplicidades e diferente de 1 somos obrigados a admitir que so os SUB-GRUPOS com extensao igual aos divisores do MDC geram contracoes nas PL's pois, de outra forma, O MDC seria diferente, o que e um absurdo ! Com os casos 1) e 2) exaurem todas as possibilidades nas quais uma PC corresponde a PL's em numero inferior a sua extensao e resolve o seu problema. SO UM DETALHE : Eu observo que muitas pessoas, diante de um problema, buscam previamente uma tecnica com a qual podem trata-lo e resolve-lo. A verdadeira tecnica de resolucao de qualquer problema e o pensamento : primeiro nos observamos as coisas, DEPOIS procuramos entende-las. A tecnica ou teoria aparece depois, naturalmente. Newton foi muito feliz quando falava sobre isso : " Pois A MELHOR E MAIS SEGURA maneira de pensar parece-me ser a seguinte : PRIMEIRO, pesquisar com afinco as propriedades das coisas; COMPROVAR estas propriedades, por intermedio de experiencias e, SO ENTAO, adiantar-se LENTAMENTE para o campo das hipoteses relativas a explicacao. As teorias devem ser criadas para explicar as propriedades verificaveis das coisas e nao para impor propriedades a elas " (NEWTON) Um abraco Paulo Santa Rita 3,1304,210502 >From: Rafael WC >Reply-To: obm-l@mat.puc-rio.br >To: obm-l@mat.puc-rio.br >Subject: Re: [obm-l] permutações circulares com repetição >Date: Mon, 20 May 2002 14:55:57 -0700 (PDT) > >Olá Pessoal! > >Obrigado Morgado e Paulo pela ajuda. > >Paulo, entrei na página que você indicou. Encontrei a >demonstração de um teorema de Moreau que sinceramente >não consegui associar nem de longe com permutações >circulares com repetição. > >Como você falou que conhece um caminho alternativo, >acho que vou abusar da sua boa vontade e perguntar >qual é. Se puder me ajudar, agradeço muito. Hoje fui à >biblioteca da faculdade e andei procurando alguns >livros, mas todos param nas permutações circulares >simples!! > >Muito obrigado, > >Rafael. _________________________________________________________________ Converse com amigos on-line, conheça o MSN Messenger: http://messenger.msn.com ========================================================================= Instruções para entrar na lista, sair da lista e usar a lista em http://www.mat.puc-rio.br/~nicolau/olimp/obm-l.html O administrador desta lista é ========================================================================= From owner-obm-l@sucuri.mat.puc-rio.br Tue May 21 14:25:38 2002 Return-Path: Received: (from majordom@localhost) by sucuri.mat.puc-rio.br (8.9.3/8.9.3) id OAA12124 for obm-l-list; Tue, 21 May 2002 14:25:02 -0300 Received: from www.zipmail.com.br (smtp.zipmail.com.br [200.187.242.10]) by sucuri.mat.puc-rio.br (8.9.3/8.9.3) with ESMTP id OAA12120 for ; Tue, 21 May 2002 14:24:59 -0300 From: peterdirichlet@zipmail.com.br Received: from [200.206.103.3] by www.zipmail.com.br with HTTP; Tue, 21 May 2002 14:11:38 -0300 Message-ID: <3CEA77EA0000019C@www.zipmail.com.br> Date: Tue, 21 May 2002 14:11:38 -0300 In-Reply-To: <17d.89e3410.2a1b26b8@aol.com> Subject: [obm-l] =?iso-8859-1?Q?Re=3A=20=5Bobm=2Dl=5D=20Trigo?= To: obm-l@mat.puc-rio.br MIME-Version: 1.0 Content-Type: text/plain; charset="iso-8859-1" Content-Transfer-Encoding: 8bit X-MIME-Autoconverted: from quoted-printable to 8bit by sucuri.mat.puc-rio.br id OAA12121 Sender: owner-obm-l@sucuri.mat.puc-rio.br Precedence: bulk Reply-To: obm-l@mat.puc-rio.br Senhor Crom,que tal voce vir nos fazer uma visitinha aqui em Sao Paulo?Na Av.Paulista,predio da Gazeta.Ass.:Edson Abe. Bem,sen20/cos20+sen70/cos70=sen20/cos20+cos20/sen20=sen20*sen20 +cos20*cos20/sen20*cos20=2/sen40=2*cosec40=2*sec50. a outra ja e bem mais longa.Mas e so prostaferizar que sai.Dica:nunca use outra coisa alem de senos e co-senos. Ate mais.Celso Pitta -- Mensagem original -- >1)Calcular o valor de tg20.tg40.tg80 >2)Mostre que tg20+tg70=2sec50. >Agradeço quem ajudar nessas questões. > TRANSIRE SVVM PECTVS MVNDOQUE POTIRE CONGREGATI EX TOTO ORBE MATHEMATICI OB SCRIPTA INSIGNIA TRIBVERE Medalha Fields(John Charles Fields) ========================================================================= Instruções para entrar na lista, sair da lista e usar a lista em http://www.mat.puc-rio.br/~nicolau/olimp/obm-l.html O administrador desta lista é ========================================================================= From owner-obm-l@sucuri.mat.puc-rio.br Tue May 21 14:30:46 2002 Return-Path: Received: (from majordom@localhost) by sucuri.mat.puc-rio.br (8.9.3/8.9.3) id OAA12288 for obm-l-list; Tue, 21 May 2002 14:30:22 -0300 Received: from www.zipmail.com.br (smtp.zipmail.com.br [200.187.242.10]) by sucuri.mat.puc-rio.br (8.9.3/8.9.3) with ESMTP id OAA12284 for ; Tue, 21 May 2002 14:30:19 -0300 From: peterdirichlet@zipmail.com.br Received: from [200.206.103.3] by www.zipmail.com.br with HTTP; Tue, 21 May 2002 14:18:19 -0300 Message-ID: <3CEA77EA000001F1@www.zipmail.com.br> Date: Tue, 21 May 2002 14:18:19 -0300 In-Reply-To: <20020520230852.88886.qmail@web10104.mail.yahoo.com> Subject: [obm-l] =?iso-8859-1?Q?Re=3A=20=5Bobm=2Dl=5D=20logaritmo=20de=20=28=2D10=29=5E2?= To: obm-l@mat.puc-rio.br MIME-Version: 1.0 Content-Type: text/plain; charset="iso-8859-1" Content-Transfer-Encoding: 8bit X-MIME-Autoconverted: from quoted-printable to 8bit by sucuri.mat.puc-rio.br id OAA12285 Sender: owner-obm-l@sucuri.mat.puc-rio.br Precedence: bulk Reply-To: obm-l@mat.puc-rio.br E ai Werneck,beleza? Bem,se a banca definisse"...a funçao f:C->C...",ai tudo bem.Eu nao me lembro da definiçao agora mas tinha algo a ver com forma polar de complexos. Por hoje e so pessoaal!!!!!Peterdirichlet -- Mensagem original -- >Oi Pessoal! > >Caiu uma questão num concurso só para professores de >matemática ontem que me deixou intrigado: >Dada a função f: >f(x) = x + raiz(x^2) - log(base 10)(x^2) > >Calcule f(-10). > >A resposta foi -2. Mas depois da prova surgiu a maior >discussão porque existia uma alternativa que era >"f(-10) não está definida". > >O pessoal questinou que estando f(-10) definida, devia >valer a propriedade do expoente de logaritmo e >poderíamos escrever: >f(x) = x + raiz(x^2) - log(base 10)(x^2) >f(x) = x + raiz(x^2) - 2.log(base 10)(x) > >E aí vemos claramente que não podemos tirar o log de >-10. Mas como o gabarito da comissão organizadora foi >-2, ficamos todos na dúvida: está definida f(-10)??? > >Um abraço, > >Rafael. > >===== >Rafael Werneck Cinoto > ICQ# 107011599 > rwcinoto@yahoo.com > rafael.caixa@gov.com.br > matduvidas@yahoo.com.br >http://www.rwcinoto.hpg.com.br/ > >__________________________________________________ >Do You Yahoo!? >LAUNCH - Your Yahoo! Music Experience >http://launch.yahoo.com >========================================================================= >Instruções para entrar na lista, sair da lista e usar a lista em >http://www.mat.puc-rio.br/~nicolau/olimp/obm-l.html >O administrador desta lista é >========================================================================= > TRANSIRE SVVM PECTVS MVNDOQUE POTIRE CONGREGATI EX TOTO ORBE MATHEMATICI OB SCRIPTA INSIGNIA TRIBVERE Medalha Fields(John Charles Fields) ------------------------------------------ Use o melhor sistema de busca da Internet Radar UOL - http://www.radaruol.com.br ========================================================================= Instruções para entrar na lista, sair da lista e usar a lista em http://www.mat.puc-rio.br/~nicolau/olimp/obm-l.html O administrador desta lista é ========================================================================= From owner-obm-l@sucuri.mat.puc-rio.br Tue May 21 15:39:39 2002 Return-Path: Received: (from majordom@localhost) by sucuri.mat.puc-rio.br (8.9.3/8.9.3) id PAA13803 for obm-l-list; Tue, 21 May 2002 15:37:32 -0300 Received: from www.zipmail.com.br (smtp.zipmail.com.br [200.187.242.10]) by sucuri.mat.puc-rio.br (8.9.3/8.9.3) with ESMTP id PAA13798 for ; Tue, 21 May 2002 15:37:28 -0300 From: peterdirichlet@zipmail.com.br Received: from [200.206.103.3] by www.zipmail.com.br with HTTP; Tue, 21 May 2002 15:25:20 -0300 Message-ID: <3CEA87DA000001BD@www.zipmail.com.br> Date: Tue, 21 May 2002 15:25:20 -0300 In-Reply-To: <3CE92016000001A0@www.zipmail.com.br> Subject: [obm-l] =?iso-8859-1?Q?Re=3A=20=5Bobm=2Dl=5D=20Corre=E7ao=3AApelo=3A=20Mais=20da=20Iberoamericana=28ques?= =?iso-8859-1?Q?tao=20pessoal=29?= To: =?iso-8859-1?Q?Lista=20de=20Discussao?= , p_ssr@hotmail.com MIME-Version: 1.0 Content-Type: text/plain; charset="iso-8859-1" Content-Transfer-Encoding: 8bit X-MIME-Autoconverted: from quoted-printable to 8bit by sucuri.mat.puc-rio.br id PAA13799 Sender: owner-obm-l@sucuri.mat.puc-rio.br Precedence: bulk Reply-To: obm-l@mat.puc-rio.br >ANSWER:Tudo bem.Antes,so uma coisa:quando eu falei do Professor Nicolau >Saldanha,nao quis dar uma ma impressao(*bem pelo contrario!!!).So fiz uma >comparaçao:ele,que e muito ocupado,ja que e um dos lideres da OBM,teve tempo >de responder-me um e-mail sobre os inteiros de Eisenstein e o problema 6 >da IMO 2001(alias ja troquei de servidor MILHAAAAARES de vezes por causa >disso.Tanto que ele so recebia mensagens minhas em HTML).E fiquei espantado >pela demora(sera que ninguem me respondia?)!!!Mas se isto teve um tom pejorativo,perdoem-me.Afinal >letras nao conseguem expressar ideias com perfeiçao.Acabei sendo antipatico,mas >sem a menor ideia do que isso causaria. >O que eu realmente achei estranho foi o fato de ninguem(entre muitos de >voces) me responder ha tanto tempo(1 mes e meio,por volta disso),a nao ser >voce.Eu devia ter lhe respondido essa pergunta mas acabei cancelando sem >querer a mensagem de resposta(junto com meu login),comentando a genialidade >da ideia,que alias tinha um furo(esta das raizes cubicas.Eu tentei algo >com Teoria dos Numeros mas nao obtive exito.Voce usou graficos de funçoes.O problema era que a s funcoes eram discretas(naturais)e nao densas(reais).). >E desculpe-me mas esse problema da "primes cube-root" nao e iberoamericano.Eu >resolvi inclui-lo deliberadamente na lista.E nao especifiquei.E,a velha e boa pressa de sempre... >E -da proxima vez que eu for usar o SHIFT,vou pensar um pouco mais. >Bem,agora eu fui avisado.E muito obri-valeu por tudo > Atenciosa e enfadonhamente,ate!!!!!! >Peterdirichlet. >-- Mensagem original -- > >>Ola Dirichlet, >>Tudo legal ? >> >>Estou lhe respondendo particularmente por acredito que a inabilidade que >> >>voce demonstra ter nao me parece fruto de um carater distorcido, mas apenas >> >>as consequencias previsiveis de determinadas conjunturas sociais e >>familiares ... >> >>1) As pessoas da lista NAO SAO OBRIGADAS a responder qualquer mensagem. >Elas >> >>respondem as que querem. De forma que ao colocarmos um problema la devemos >> >>ter isso em mente. Se ninguem nos responder, E UM DIREITO DELES que deve >>e >>precisa ser respeitado, pois os nossos direitos terminam quando comecam >os >> >>dos outros. PORTANTO, SE NINGUEM LHE RESPONDER, NAO RECLAME ! >> >>2) Na lista esta o coordenador nacional, Prof Nicolau Saldanha, e muitos >>( >>senão todos ! ) coordenadores regionais. Se voce criticar pode ser que >no >> >>futuro, quando voce precisar, nao encontre tao boa vontade quanto >>encontraria se procurasse - respeitados os principios de dignidade e >>honradez - cativar as pessoas que leem suas mensagem, tornando uteis suas >> >>publicacoes. Assim, por prudencia, EVITE TECER COMENTARIOS NEGATIVOS SOBRE >> >>QUALQUER MEMBRO ESPECIFICO DA LISTA ! >> >>3)Se voce quer ver suas duvidas serem respondidas, procure SER HUMILDE. > >>HUMILDADE NAO E MEDIOCRIDADE. Muitas pessoas da lista tem muito mais estudo >> >>que voce, de forma que aquilo que para voce parece dificil pode ser facil >> >>para algum membro. Mas, se voce for arrogante e ironico, vai antipatizar >>com >>as pessoas e ninguem te respondera. >> >>Bom, meu : NUNCA DIGA QUE NINGUEM LHE AVISOU ! >> >>Um abraco >>Paulo Santa Rita >>6,1558,170502 >> >> >> >> >> >> >> >>>From: peterdirichlet@zipmail.com.br >>>Reply-To: obm-l@mat.puc-rio.br >>>To: obm-l@mat.puc-rio.br >>>Subject: [obm-l] Correçao:Apelo: Mais da Iberoamericana >>>Date: Fri, 17 May 2002 14:04:55 -0300 >>> >>>ANSWER:Bem,apoveito e respondo o e-mail do Bruno.Bem,acho que o intuito >>>nao seria o de explora-los. Afinal,so porque NINGUEM RECEBE SALARIO nao >>>significa que nao possa responder.Fiquei espantado pela demora.Certa vez >>>o Nicolau me respondeu uma questao 1 semana depois que enviei o e-mail.E >>>agora fiquei mo cara sem resposta.Talvez nao me interpretei direito... >>>Mas se as questoes estavam mal-formuladas,por que nao me avisaram? >>>Corrigindo:1)p*q+r tambem pertence ao dito conjunto.E o r e o n sao >>>iguais(erros >>>de grafia e pressa). >>>3)Resto da DIVISAO,ta? >>>4)Eu modifiquei os enunciados(de modo imperceptivel:no lugar de lideres >>>da OIM estava representantes). Mas e isso mesmo,a ideia e IGUALZINHA >>>Meu,sera que me fiz claro? >>>Um abraço.Peterdirichlet >>> >>> >>> >>> >>> >>>-- Mensagem original -- >>> >>> >Ola Dirichlet, >>> > >>> >Ninguem respondeu, MUITO PROVAVELMENTE, porque as suas questoes, >>> > >>> >1) Estao mal formuladas. Por exemplo, voce escreveu : >>> > >>> >>5)Sabe-se que num conjunto de primos se p e q sao elementos(iguais >ou >>> >>> >> >nao)entao p*q+r,em que r e constante.Quantos elementos tem S com >>> >>n=4?>Generalize o r. >>> > >>> >p*q+r O QUE ? E ESSE n=4, O QUE E ? >>> > >>> >Essa mal formulacao EVIDENTE impossibilita uma solucao pode ter lancado >>>uma >>> > >>> >descrenca quanto a correcao do enunciado das demais questoes. >>> > >>> >2) A maioria delas ja tem solucao nos arquivos de mensagens que o Prof >>> >>> >Nicolau guarda. por exemplo, voce escreveu : >>> > >>> >>1)Ache todos os naturais n de 3,2 ou 1 digito tal que o quadrado de >n >>> >seja >>> > >>> >>o cubo da soma dos digitos. >>> > >>> >Esta questao ( ou outra semelhante ) ja foi respondida e a solucao esta >>>no >>> > >>> >arquivos de mensagens a que me referi acima. La voce vai ver uma linha >>>de >>> > >>> >raciocinio proxima de : >>> > >>> >(a+b)^3=(10a+b)^2 => a+b = [(10a+b)/a+b]^2 e portanto "a+b" e quadrado >>> >>> >perfeito e a+b divide 10a+b. Como a+b =< 18, os possiveis valores >>>quadrado >>> > >>> >de a+b serao ... >>> > >>> >>3)L>0 e tal que -L^2+1998*L+1=0.Seja a recorrencia a(0)=1 e a(n+1)>parte >>> > >>> >>inteira de L*a(n)=[L*a(n)].Calcule a(1998)mod 1998(x mod y e o >resto >>>de >>> >x >>> >>por y). >>> > >>> >RESTO DE QUE ? ADVINHANDO ... Se x e a unica solucao positiva da equacao >>> >do >>> >2 grau do enunciado de sua questao e [x] e a funcao maximo inteiro, >o >>> >problema consiste em determinar o valor de >>> > >>> >[x...[x[x[x]]]...] >>> > >>> >com 1998 colchetes. como >>> >[x] = k se k =< x < K+1 >>> >entao [x[x]] e a anlise de [kx] e assim sucessivamente. ESTA QUESTAO >JA >>>FOI >>> > >>> >RESOLVIDA NA LISTA ! >>> > >>> >>4)na mesa da banca de lideres da OIM estao lideres de P paises de modo >>> >>que se dois lideres quaisquer sao de mesmo pais entao seus vizinhos >>> >> >direitos nao sao.Quantos lideres ha no maximo ? >>> > >>> >FALTAM INFORMACOES ! Se P=3, sejam A,B e C lideres, dispostos ao longo >>>de >>> > >>> >uma mesa nesta ordem. Posso sempre inserir entre dois deles o terceiro, >>>de >>> > >>> >forma que a direita de cada um nao havera duplicacao. EVIDENTEMENTE >que >>>este >>> > >>> >processo pode ser extendido AD INFINITUM ! >>> > >>> >Meu, que coisas sao essas?Ate agora ninguem ENTENDEU !!!!!! >>> > >>> >Um abraco >>> >Paulo Santa Rita >>> >5,1827,160502 >>> > >>> >>From: peterdirichlet@zipmail.com.br >>> >>Reply-To: obm-l@mat.puc-rio.br >>> >>To: obm-l@mat.puc-rio.br >>> >>Subject: [obm-l] Apelo: Mais da Iberoamericana >>> >>Date: Thu, 16 May 2002 14:21:45 -0300 >>> >> >>> >>Meu,que coisa e essa?Ate agora ninguem me respondeu!!!!!!!! >>> >> >>> >>-- Mensagem original -- >>> >> >>> >> >Alo turma!!!!!!Tenho mais perguntas a fazer(da Iberoamericana): >>> >> >1)Ache todos os naturais n de 3,2 ou 1 digito tal que o quadrado >de >>>n >>> > >>> >>seja >>> >> >o cubo da soma dos digitos. >>> >> >2)Encontre o menor n tal que se pegarmos n dos 999 primeiros inteiros >>> > >>> >>positivos >>> >> >sempre se acham 4 numeros diferentes a,b,c,d com a+2*b+3*c-4*d=0. >>> >> >3)L>0 e tal que -L^2+1998*L+1=0.Seja a recorrencia a(0)=1 e >>>a(n+1)=parte >>> >> >inteira de L*a(n)=[L*a(n)].Calcule a(1998)mod 1998(x mod y e o resto >>> >de >>> >> >x por y). >>> >> >4)na mesa da banca de lideres da OIM estao lideres de P paises de >modo >>> >>que >>> >> >se dois lideres quaisquer sao de mesmo pais entao seus vizinhos >>>direitos >>> >> >nao sao.Quantos lideres ha no maximo? >>> >> >5)Sabe-se que num conjunto de primos se p e q sao elementos(iguais >>ou >>> > >>> >>nao)entao >>> >> >p*q+r,em que r e constante.Quantos elementos tem S com n=4?Generalize >>> >o >>> >> >r. >>> >> > >>> >> >TRANSIRE SVVM PECTVS MVNDOQUE POTIRE >>> >> >CONGREGATI EX TOTO ORBE MATHEMATICI OB SCRIPTA INSIGNIA TRIBVERE >>> >> >Medalha Fields(John Charles Fields) >>> >> > >>> >> > >>> >> >------------------------------------------ >>> >> >Use o melhor sistema de busca da Internet >>> >> >Radar UOL - http://www.radaruol.com.br >>> >> > >>> >> > >>> >> > >>> >> >>> >========================================================================= >>> >> >Instruções para entrar na lista, sair da lista e usar a lista em >>> >> >http://www.mat.puc-rio.br/~nicolau/olimp/obm-l.html >>> >> >O administrador desta lista é >>> >> >>> >========================================================================= >>> >> > >>> >> >>> >>TRANSIRE SVVM PECTVS MVNDOQUE POTIRE >>> >>CONGREGATI EX TOTO ORBE MATHEMATICI OB SCRIPTA INSIGNIA TRIBVERE >>> >>Medalha Fields(John Charles Fields) >>> >> >>> >> >>> >>------------------------------------------ >>> >>Use o melhor sistema de busca da Internet >>> >>Radar UOL - http://www.radaruol.com.br >>> >> >>> >> >>> >> >>> >>========================================================================= >>> >>Instruções para entrar na lista, sair da lista e usar a lista em >>> >>http://www.mat.puc-rio.br/~nicolau/olimp/obm-l.html >>> >>O administrador desta lista é >>> >>========================================================================= >>> > >>> > >>> > >>> > >>> >_________________________________________________________________ >>> >Chegou o novo MSN Explorer. Instale já. É gratuito: >>> >http://explorer.msn.com.br >>> > >>> >========================================================================= >>> >Instruções para entrar na lista, sair da lista e usar a lista em >>> >http://www.mat.puc-rio.br/~nicolau/olimp/obm-l.html >>> >O administrador desta lista é >>> >========================================================================= >>> > >>> >>>TRANSIRE SVVM PECTVS MVNDOQUE POTIRE >>>CONGREGATI EX TOTO ORBE MATHEMATICI OB SCRIPTA INSIGNIA TRIBVERE >>>Medalha Fields(John Charles Fields) >>> >>> >>>------------------------------------------ >>>Use o melhor sistema de busca da Internet >>>Radar UOL - http://www.radaruol.com.br >>> >>> >>> >>>========================================================================= >>>Instruções para entrar na lista, sair da lista e usar a lista em >>>http://www.mat.puc-rio.br/~nicolau/olimp/obm-l.html >>>O administrador desta lista é >>>========================================================================= >> >> >> >>_________________________________________________________________ >>Converse com amigos on-line, conheça o MSN Messenger: >>http://messenger.msn.com >> >> > >TRANSIRE SVVM PECTVS MVNDOQUE POTIRE >CONGREGATI EX TOTO ORBE MATHEMATICI OB SCRIPTA INSIGNIA TRIBVERE >Medalha Fields(John Charles Fields) > > >------------------------------------------ >Use o melhor sistema de busca da Internet >Radar UOL - http://www.radaruol.com.br > > > > TRANSIRE SVVM PECTVS MVNDOQUE POTIRE CONGREGATI EX TOTO ORBE MATHEMATICI OB SCRIPTA INSIGNIA TRIBVERE Medalha Fields(John Charles Fields) ------------------------------------------ Use o melhor sistema de busca da Internet Radar UOL - http://www.radaruol.com.br ========================================================================= Instruções para entrar na lista, sair da lista e usar a lista em http://www.mat.puc-rio.br/~nicolau/olimp/obm-l.html O administrador desta lista é ========================================================================= From owner-obm-l@sucuri.mat.puc-rio.br Tue May 21 16:52:01 2002 Return-Path: Received: (from majordom@localhost) by sucuri.mat.puc-rio.br (8.9.3/8.9.3) id QAA15434 for obm-l-list; Tue, 21 May 2002 16:50:47 -0300 Received: from hotmail.com (law2-f83.hotmail.com [216.32.181.83]) by sucuri.mat.puc-rio.br (8.9.3/8.9.3) with ESMTP id QAA15430 for ; Tue, 21 May 2002 16:50:39 -0300 Received: from mail pickup service by hotmail.com with Microsoft SMTPSVC; Tue, 21 May 2002 12:38:33 -0700 Received: from 200.229.244.100 by lw2fd.hotmail.msn.com with HTTP; Tue, 21 May 2002 19:38:32 GMT X-Originating-IP: [200.229.244.100] From: "Paulo Santa Rita" To: obm-l@mat.puc-rio.br Subject: [obm-l] =?iso-8859-1?B?UmU6IFtvYm0tbF0gUmU6IFtvYm0tbF0gQ29ycmXnYW86QXBlbG86IE1h?= =?iso-8859-1?B?aXMgZGEgSWJlcm9hbWVyaWNhbmEocXVlc3RhbyBwZXNzb2FsKQ==?= Date: Tue, 21 May 2002 19:38:32 +0000 Mime-Version: 1.0 Content-Type: text/plain; charset=iso-8859-1; format=flowed Message-ID: X-OriginalArrivalTime: 21 May 2002 19:38:33.0203 (UTC) FILETIME=[1793FC30:01C200FF] Sender: owner-obm-l@sucuri.mat.puc-rio.br Precedence: bulk Reply-To: obm-l@mat.puc-rio.br Ola Dirichlet ! Tudo legal ? Quando eu falei sobre o problema das raizes cubicas, NAO PROVEI NADA, apenas dei algumas sugestoes para uma eventual demonstracao sua. A densidade dos reais nao me pareceu um obstaculo, antes um auxilio ... De fato. Uma definicao satisfatoria de convexidade pode ser : F((X+Y)/2) > (F(x)+F(Y))/2 TRADUZINDO : A base media do trapezio retangulo de bases F(X) e F(Y) e sempre menor que o valor da ordenada da funcao convexa no ponto medio. Mas isso E UMA DEFINICAO SATISFATORIA ... significa que ela comporta implicacoes que estao ligadas a funcao que nao ficam claras na definicao acima ... para ver isso, seja F(X) crescente. Entao : X < Y => F(X) < F(Y) => P*F(X) + q*F(X) < p*F(X) + q*F(Y) F(X) < (p*F(X) + q*F(Y))/(p+q) com p,q reais positivos. Analogamente : (p*F(X) + q*F(Y))/(p+q) < F(Y) Das duas desigualdades acima segue que a media ponderada das bases sempre estara entre os valores das duas ordenadas, quaisquer que sejam os pesos reais e positivos. Segue que se associarmos uma reta passando por, digamos, raiz_N(A) e raiz_N(B), A e B naturais, estaremos, em verdade, fazendo a associacao : raiz_N(A) -> P <=> C + D*P = raiz_N(A) raiz_N(B) -> Q <=> C + D*Q = raiz_N(B) A solucao do sistema acima vai fornecer C e D, que sao os termos que irao caracterizar univocamente a PA que contem as duas raizes N-esimas. Ora, para para qualquer valor natural entre A e B, o ponto da reta que contem as raizes N-esimas A e B e uma ordenada que pode ser expressa como uma media ponderada entre estas raizes, conforme voce pode verificar diretamente. Dai segue que raiz_N(C+D*(valor entre A e B)) tambem e convexa e teremos : raiz_N(C+D*(valor entre A e B)) > media ponderada entre as raizes enesimas de A e B. Supor que uma das raizes estara na reta vai entrar em contradicao com o fato acima, pois C+D*N, dado que assume os valores raiz_N(A) em P e raiz_N(B) em que Q e continua e, em particular, vai assumir qualquer outro valor N-esimo entre estas duas raizes. Era essa a ideia da demonstracao. Talvez eu nao tenha conseguido ser claro naquele momento. E obrigado pelo elogio. Um abraco Fica com Deus Paulo Santa Rita 3,1635,210502 >From: peterdirichlet@zipmail.com.br >Reply-To: obm-l@mat.puc-rio.br >To: Lista de Discussao , p_ssr@hotmail.com >Subject: [obm-l] Re: [obm-l] Correçao:Apelo: Mais da Iberoamericana >( questao pessoal ) >Date: Tue, 21 May 2002 15:25:20 -0300 >O que eu realmente achei estranho foi o fato de ninguem(entre muitos de >voces) me responder ha tanto tempo(1 mes e meio,por volta disso),a nao >ser voce.Eu devia ter lhe respondido essa pergunta mas acabei >cancelando >sem >querer a mensagem de resposta (junto com meu login), comentando a > >genialidade da ideia,que alias tinha um furo(esta das raizes >cubicas.Eu >tentei algo com Teoria dos Numeros mas nao obtive >exito.Voce usou graficos >de funçoes. O problema era que a s funcoes >eram discretas(naturais)e nao >densas(reais).). _________________________________________________________________ O MSN Photos é o modo mais fácil de compartilhar e imprimir suas fotos: http://photos.msn.com/support/worldwide.aspx ========================================================================= Instruções para entrar na lista, sair da lista e usar a lista em http://www.mat.puc-rio.br/~nicolau/olimp/obm-l.html O administrador desta lista é ========================================================================= From owner-obm-l@sucuri.mat.puc-rio.br Tue May 21 18:36:15 2002 Return-Path: Received: (from majordom@localhost) by sucuri.mat.puc-rio.br (8.9.3/8.9.3) id SAA17562 for obm-l-list; Tue, 21 May 2002 18:35:11 -0300 Received: from sr1.terra.com.br (sr1.terra.com.br [200.176.3.16]) by sucuri.mat.puc-rio.br (8.9.3/8.9.3) with ESMTP id SAA17558 for ; Tue, 21 May 2002 18:35:09 -0300 Received: from smtp4-poa.terra.com.br (smtp4-poa.terra.com.br [200.176.3.35]) by sr1.terra.com.br (Postfix) with ESMTP id A813C6EAE5 for ; Tue, 21 May 2002 18:23:12 -0300 (EST) Received: from stabel (dl-nas1-poa-C89A0156.p001.terra.com.br [200.154.1.86]) (authenticated user dudasta) by smtp4-poa.terra.com.br (Postfix) with ESMTP id 48EE7AC63A for ; Tue, 21 May 2002 18:23:10 -0300 (EST) Message-ID: <004f01c2010d$b48e9140$19009ac8@stabel> From: "Eduardo Casagrande Stabel" To: References: Subject: [obm-l] =?iso-8859-1?Q?Re:_=5Bobm-l=5D_Re:_=5Bobm-l=5D_Re:_=5Bobm-l=5D_Corr?= =?iso-8859-1?Q?e=E7ao:Apelo:_Mais_da_Iberoamericana=28questao_pessoal=29?= Date: Tue, 21 May 2002 18:04:49 -0300 MIME-Version: 1.0 Content-Type: text/plain; charset="iso-8859-1" Content-Transfer-Encoding: 8bit X-Priority: 3 X-MSMail-Priority: Normal X-Mailer: Microsoft Outlook Express 6.00.2600.0000 X-MimeOLE: Produced By Microsoft MimeOLE V6.00.2600.0000 Sender: owner-obm-l@sucuri.mat.puc-rio.br Precedence: bulk Reply-To: obm-l@mat.puc-rio.br From: "Paulo Santa Rita" > Ola Dirichlet ! > Tudo legal ? > > Quando eu falei sobre o problema das raizes cubicas, NAO PROVEI NADA, apenas > dei algumas sugestoes para uma eventual demonstracao sua. A densidade dos > reais nao me pareceu um obstaculo, antes um auxilio ... > > De fato. Uma definicao satisfatoria de convexidade pode ser : > > F((X+Y)/2) > (F(x)+F(Y))/2 > > TRADUZINDO : A base media do trapezio retangulo de bases F(X) e F(Y) e > sempre menor que o valor da ordenada da funcao convexa no ponto medio. > > Mas isso E UMA DEFINICAO SATISFATORIA ... significa que ela comporta > implicacoes que estao ligadas a funcao que nao ficam claras na definicao > acima ... para ver isso, seja F(X) crescente. Entao : > > X < Y => F(X) < F(Y) => P*F(X) + q*F(X) < p*F(X) + q*F(Y) > F(X) < (p*F(X) + q*F(Y))/(p+q) com p,q reais positivos. > Analogamente : (p*F(X) + q*F(Y))/(p+q) < F(Y) > > Das duas desigualdades acima segue que a media ponderada das bases sempre > estara entre os valores das duas ordenadas, quaisquer que sejam os pesos > reais e positivos. Segue que se associarmos uma reta passando por, digamos, > raiz_N(A) e raiz_N(B), A e B naturais, estaremos, em verdade, fazendo a > associacao : > > raiz_N(A) -> P <=> C + D*P = raiz_N(A) > raiz_N(B) -> Q <=> C + D*Q = raiz_N(B) > > A solucao do sistema acima vai fornecer C e D, que sao os termos que irao > caracterizar univocamente a PA que contem as duas raizes N-esimas. Ora, para > para qualquer valor natural entre A e B, o ponto da reta que contem as > raizes N-esimas A e B e uma ordenada que pode ser expressa como uma media > ponderada entre estas raizes, conforme voce pode verificar diretamente. Dai > segue que raiz_N(C+D*(valor entre A e B)) tambem e convexa e teremos : > > raiz_N(C+D*(valor entre A e B)) > media ponderada entre as raizes enesimas > de A e B. > > Supor que uma das raizes estara na reta vai entrar em contradicao com o fato > acima, pois C+D*N, dado que assume os valores raiz_N(A) em P e raiz_N(B) em > que Q e continua e, em particular, vai assumir qualquer outro valor N-esimo > entre estas duas raizes. Oi Paulo! Não sei se compreendi bem esse seu e-mail. :) O problema é o seguinte: desenhe no plano os pontos (R_N(x), x) para todo x inteiro positivo. Você vai ter destacado alguns pontos da função contínua f(x) = R_N(x) para todo x real. Agora escolha uma PA de 3 termos inteiros positivos, digamos Y_1, Y_2, Y_3. Marque os três pontos no eixo ordenado (0,Y_1), (0, Y_2) e (0, Y_3), para cada um deles trace uma reta horizontal, ou seja, paralela ao eixo das abscissas. Suponhamos que essas três retas passem por três dos pontos do gráfico que você tinha destacado. Esses três pontos chame de (X_1, Y_1) também (X_2, Y_2) e finalmente (X_3, Y_3). O que nós temos, agora, é que os Y_1, Y_2 e Y_3 está em forma de PA, mas isso não precisa acontecer com os X_1, X_2 e X_3. OU SEJA, os pontos (X_n, Y_n) NÃO PRECISAM ESTAR SOBRE UMA RETA. E daí esse seu argumento não prova nada sobre o problema inicial. Em outras palavras, o que você demonstrou pela convexidade da f, a saber, que nenhuma reta corta o gráfico da f em três pontos distintos, não garante que se p, q e r foram primos distintos então f(p), f(q) e f(r) não formam uma PA. Se o seu argumento é só um passo para resolver o problema, perdoe o meu comentário acima: mas eu acredito que esse caminho não vai levar a uma solução. E só para terminar o e-mail: o Fabio Dias Moreira é que ressaltou isso que eu disse aí em cima, e pelo visto, o Paulo não chegou a ler a mensagem dele. Um abraço! Eduardo Casagrande Stabel. PS. eu não descarto a possibilidade de eu não ter compreendido bem as mensagens do Paulo e do Fábio, e eu é que precise de uma explicação. > > Era essa a ideia da demonstracao. Talvez eu nao tenha conseguido ser claro > naquele momento. E obrigado pelo elogio. > > Um abraco > Fica com Deus > Paulo Santa Rita > 3,1635,210502 > > > >From: peterdirichlet@zipmail.com.br > >Reply-To: obm-l@mat.puc-rio.br > >To: Lista de Discussao , p_ssr@hotmail.com > >Subject: [obm-l] Re: [obm-l] Correçao:Apelo: Mais da Iberoamericana > >( questao pessoal ) > >Date: Tue, 21 May 2002 15:25:20 -0300 > >O que eu realmente achei estranho foi o fato de ninguem(entre muitos de > >voces) me responder ha tanto tempo(1 mes e meio,por volta disso),a nao > >ser voce.Eu devia ter lhe respondido essa pergunta mas acabei >cancelando > >sem > >querer a mensagem de resposta (junto com meu login), comentando a > > >genialidade da ideia,que alias tinha um furo(esta das raizes >cubicas.Eu > >tentei algo com Teoria dos Numeros mas nao obtive >exito.Voce usou graficos > >de funçoes. O problema era que a s funcoes >eram discretas(naturais)e nao > >densas(reais).). > > > _________________________________________________________________ > O MSN Photos é o modo mais fácil de compartilhar e imprimir suas fotos: > http://photos.msn.com/support/worldwide.aspx > > ========================================================================= > Instruções para entrar na lista, sair da lista e usar a lista em > http://www.mat.puc-rio.br/~nicolau/olimp/obm-l.html > O administrador desta lista é > ========================================================================= > > ========================================================================= Instruções para entrar na lista, sair da lista e usar a lista em http://www.mat.puc-rio.br/~nicolau/olimp/obm-l.html O administrador desta lista é ========================================================================= From owner-obm-l@sucuri.mat.puc-rio.br Tue May 21 18:57:34 2002 Return-Path: Received: (from majordom@localhost) by sucuri.mat.puc-rio.br (8.9.3/8.9.3) id SAA17924 for obm-l-list; Tue, 21 May 2002 18:57:00 -0300 Received: from imo-r01.mx.aol.com (imo-r01.mx.aol.com [152.163.225.97]) by sucuri.mat.puc-rio.br (8.9.3/8.9.3) with ESMTP id SAA17920 for ; Tue, 21 May 2002 18:56:57 -0300 From: Korshinoi@aol.com Received: from Korshinoi@aol.com by imo-r01.mx.aol.com (mail_out_v32.5.) id z.91.1d4a4836 (4365) for ; Tue, 21 May 2002 17:44:51 -0400 (EDT) Message-ID: <91.1d4a4836.2a1c19d3@aol.com> Date: Tue, 21 May 2002 17:44:51 EDT Subject: [obm-l] (nenhum assunto) To: obm-l@mat.puc-rio.br MIME-Version: 1.0 Content-Type: multipart/alternative; boundary="part1_91.1d4a4836.2a1c19d3_boundary" X-Mailer: AOL 7.0 for Windows BR sub 10501 Sender: owner-obm-l@sucuri.mat.puc-rio.br Precedence: bulk Reply-To: obm-l@mat.puc-rio.br --part1_91.1d4a4836.2a1c19d3_boundary Content-Type: text/plain; charset="ISO-8859-1" Content-Transfer-Encoding: quoted-printable mostre que para todo n natural, o n=FAmero n(n+1)/2 est=E1 em IN e que seu=20 algarismo das unidades n=E3o pode ser 2, nem 4, nem 7, nem 9. Obrigado Korshin=F3i --part1_91.1d4a4836.2a1c19d3_boundary Content-Type: text/html; charset="ISO-8859-1" Content-Transfer-Encoding: quoted-printable mostre que para todo n natural, o n=FAmero n(n+1)/2 es= t=E1 em IN e que seu algarismo das unidades n=E3o pode ser 2, nem 4, nem 7,=20= nem 9.
        Obrigado
            &nbs= p;   Korshin=F3i
--part1_91.1d4a4836.2a1c19d3_boundary-- ========================================================================= Instruções para entrar na lista, sair da lista e usar a lista em http://www.mat.puc-rio.br/~nicolau/olimp/obm-l.html O administrador desta lista é ========================================================================= From owner-obm-l@sucuri.mat.puc-rio.br Tue May 21 20:15:18 2002 Return-Path: Received: (from majordom@localhost) by sucuri.mat.puc-rio.br (8.9.3/8.9.3) id UAA19704 for obm-l-list; Tue, 21 May 2002 20:14:27 -0300 Received: from gorgo.centroin.com.br (gorgo.centroin.com.br [200.225.63.128]) by sucuri.mat.puc-rio.br (8.9.3/8.9.3) with ESMTP id UAA19700 for ; Tue, 21 May 2002 20:14:25 -0300 Received: from centroin.com.br (du104b.rjo.centroin.com.br [200.225.57.104]) (authenticated bits=0) by gorgo.centroin.com.br (8.12.2/8.12.1) with ESMTP id g4LN2lMu004076 for ; Tue, 21 May 2002 20:02:48 -0300 (BRT) Message-ID: <3CEAD258.1040708@centroin.com.br> Date: Tue, 21 May 2002 20:03:52 -0300 From: Augusto =?ISO-8859-1?Q?C=E9sar?= Morgado User-Agent: Mozilla/5.0 (Windows; U; Win98; en-US; rv:0.9.2) Gecko/20010726 Netscape6/6.1 X-Accept-Language: en-us MIME-Version: 1.0 To: obm-l@mat.puc-rio.br Subject: Re: [obm-l] (nenhum assunto) References: <91.1d4a4836.2a1c19d3@aol.com> Content-Type: multipart/alternative; boundary="------------000703010204030905030502" Sender: owner-obm-l@sucuri.mat.puc-rio.br Precedence: bulk Reply-To: obm-l@mat.puc-rio.br --------------000703010204030905030502 Content-Type: text/plain; charset=ISO-8859-1; format=flowed Content-Transfer-Encoding: 8bit n, n+1 sao dois naturais consecutivos; logo, um deles eh par e o produto n(n+1) eh par. A tabela a seguir mostra os algarismos das unidades: n n+1 n(n+1) n(n+1)/2 0 1 0 5 ou 0 1 2 2 1 ou 6 2 3 6 3 ou 8 3 4 2 3 ou 6 4 5 0 0 ou 5 5 6 0 0 ou 5 6 7 2 1 ou 6 7 8 6 3 ou 8 8 9 2 1 ou 6 9 0 0 0 ou 5 Korshinoi@aol.com wrote: > mostre que para todo n natural, o número n(n+1)/2 está em IN e que seu > algarismo das unidades não pode ser 2, nem 4, nem 7, nem 9. > Obrigado > Korshinói --------------000703010204030905030502 Content-Type: text/html; charset=us-ascii Content-Transfer-Encoding: 7bit n, n+1 sao dois naturais consecutivos; logo, um deles eh par e o produto n(n+1) eh par.
A tabela a seguir mostra os algarismos das unidades:
n              n+1           n(n+1)                 n(n+1)/2
0               1                0                           5 ou 0
1                2                2                           1 ou 6
2              3                 6                             3 ou 8
3              4                 2                             3  ou 6
4              5                 0                            0  ou  5
5              6                 0                           0  ou   5
6              7                 2                            1 ou 6
7              8                 6                            3 ou 8
8             9                   2                          1  ou 6
9             0                  0                          0  ou  5

Korshinoi@aol.com wrote:
mostre que para todo n natural, o número n(n+1)/2 está em IN e que seu algarismo das unidades não pode ser 2, nem 4, nem 7, nem 9.
        Obrigado
                Korshinói

--------------000703010204030905030502-- ========================================================================= Instruções para entrar na lista, sair da lista e usar a lista em http://www.mat.puc-rio.br/~nicolau/olimp/obm-l.html O administrador desta lista é ========================================================================= From owner-obm-l@sucuri.mat.puc-rio.br Tue May 21 21:09:35 2002 Return-Path: Received: (from majordom@localhost) by sucuri.mat.puc-rio.br (8.9.3/8.9.3) id VAA20623 for obm-l-list; Tue, 21 May 2002 21:09:22 -0300 Received: from Euler.impa.br (euler.impa.br [147.65.1.3]) by sucuri.mat.puc-rio.br (8.9.3/8.9.3) with ESMTP id VAA20619 for ; Tue, 21 May 2002 21:09:19 -0300 Received: from [147.65.11.11] (dial11.impa.br [147.65.11.11]) by Euler.impa.br (8.11.6/8.11.6) with ESMTP id g4LNvMm21816 for ; Tue, 21 May 2002 20:57:22 -0300 (EST) Message-Id: <200205212357.g4LNvMm21816@Euler.impa.br> X-Mailer: Microsoft Outlook Express Macintosh Edition - 4.5 (0410) Date: Thu, 23 May 2002 20:55:23 -0300 Subject: Re: [obm-l] Re: [obm-l] Re: [obm-l] RES: [obm-l] valor m =?ISO-8859-1?B?7Q==?=nimo From: "Eduardo Wagner" To: obm-l@mat.puc-rio.br Mime-version: 1.0 X-Priority: 3 Content-type: text/plain; charset="ISO-8859-1" Content-Transfer-Encoding: 8bit X-MIME-Autoconverted: from quoted-printable to 8bit by sucuri.mat.puc-rio.br id VAA20620 Sender: owner-obm-l@sucuri.mat.puc-rio.br Precedence: bulk Reply-To: obm-l@mat.puc-rio.br ---------- >From: "Nicolau C. Saldanha" >To: obm-l@mat.puc-rio.br >Subject: [obm-l] Re: [obm-l] Re: [obm-l] RES: [obm-l] valor mínimo >Date: Tue, May 21, 2002, 9:13 AM > > On Mon, May 20, 2002 at 06:58:24PM -0300, Eder wrote: >> Valeu Ralph, >> >> >> Essa expressão surgiu do seguinte problema: detrerminar o menor caminho que >> uma formiguinha pode fazer por sobre a superfície de um cubo de aresta 1,de >> um vértice a outro "diagonalmente oposto". > > Não acompanhei a conversa toda, posso estar repetindo o que alguém já falou, > mas o problema da formiguinha pode ser resolvido usando simplesmente que > a distância mais curta entre dois pontos é a linha reta, sem conta nenhuma. > > Imagine o cubo pendurado por um vértice (que fica em cima). > Imagine que a formiga inicialmente está no vértice de cima e deseja > chegar no vértice de baixo. Há três faces em cima e três em baixo > e um hexágono não planar em zigue-zague de arestas separando as três > faces de cima das três de baixo. Claramente que a distância mais curta > de um qualquer ponto do zigue-zague até o vértice de cima é uma linha > reta que só toca o zigue-zague na ponta; idem para o vértice de baixo. > Claramente a formiga cruza o zigue-zague; como ela segue o caminho mais > curto, ela cruza o zigue-zague em um único ponto; este ponto está sobre > uma das seis arestas (talvez na ponta). Como as seis arestas são exatamente > iguais (ou melhor, há isometrias do cubo preservando os vértices de cima > e de baixo que levam qualquer aresta em qualquer outra) podemos escolher > uma aresta e supor que a formiga passa por ali. Mas agora a formiga está > resolvendo um problema essencialmente planar: há dois quadrados colados > por um lado comum e desdobrar a superfície para colocar os dois quadrados > no plano não muda em nada a vida da formiga. Moral: a formiga anda em linha > reta (no seu ponto de vista) e passa pelo meio de uma das seis arestas > do zigue-zague. > > []s, N. Eu tambem nao acompanhei a conversa, mas se voce conseguir um cubo de plastico ou de acrilico (as escolas costumam ter), experimente pendura-lo por um vertice e pingar no vertice superior uma gota (grande) de mel. Observe o mel escorrendo na superficie do cubo ate pingar pelo vertice inferior. Voce vera tudo o que o Nicolau falou. Abraco, Wagner. > ========================================================================= > Instruções para entrar na lista, sair da lista e usar a lista em > http://www.mat.puc-rio.br/~nicolau/olimp/obm-l.html > O administrador desta lista é > ========================================================================= ========================================================================= Instruções para entrar na lista, sair da lista e usar a lista em http://www.mat.puc-rio.br/~nicolau/olimp/obm-l.html O administrador desta lista é ========================================================================= From owner-obm-l@sucuri.mat.puc-rio.br Tue May 21 21:12:49 2002 Return-Path: Received: (from majordom@localhost) by sucuri.mat.puc-rio.br (8.9.3/8.9.3) id VAA20679 for obm-l-list; Tue, 21 May 2002 21:12:47 -0300 Received: from smtp-4.ig.com.br (smtp-4.ig.com.br [200.226.132.153] (may be forged)) by sucuri.mat.puc-rio.br (8.9.3/8.9.3) with SMTP id VAA20675 for ; Tue, 21 May 2002 21:12:45 -0300 From: ezer@ig.com.br Received: (qmail 12986 invoked from network); 22 May 2002 00:00:33 -0000 Received: from shasta057161.ig.com.br (HELO house) (200.151.57.161) by smtp-4.ig.com.br with SMTP; 22 May 2002 00:00:33 -0000 To: obm-l@mat.puc-rio.br Date: Wed, 1 Jan 1997 00:26:06 -0200 MIME-Version: 1.0 Content-type: Multipart/Mixed; boundary=Message-Boundary-13530 Subject: =?ISO-8859-1?Q?Re:_[obm-l]_tri=E2ngulo?= Message-ID: <32C9AF1E.27179.15DE27@localhost> In-reply-to: <20020515190642.95877.qmail@web10104.mail.yahoo.com> X-mailer: Pegasus Mail for Windows (v4.01) Sender: owner-obm-l@sucuri.mat.puc-rio.br Precedence: bulk Reply-To: obm-l@mat.puc-rio.br --Message-Boundary-13530 Content-type: text/plain; charset=US-ASCII Content-transfer-encoding: 7BIT Content-description: Mail message body --Message-Boundary-13530 Content-type: text/plain; charset=US-ASCII Content-disposition: inline Content-description: Attachment information. The following section of this message contains a file attachment prepared for transmission using the Internet MIME message format. If you are using Pegasus Mail, or any another MIME-compliant system, you should be able to save it or view it from within your mailer. If you cannot, please ask your system administrator for assistance. ---- File information ----------- File: figura.gif Date: 1 Jan 1997, 4:05 Size: 1494 bytes. Type: GIF-image --Message-Boundary-13530 Content-type: Image/GIF; name="figura.gif" Content-disposition: attachment; filename="figura.gif" Content-transfer-encoding: BASE64 R0lGODlhQAHIANAAAAAAAP///ywAAAAAQAHIAAAC/oyPqcvtD6OctNqLs968+w+G4kiW5omm 6sq27gvH8kzX9o3n+s73PgkA/IbEojESPCqXzF0w2YxKp6mnkIrNajFQ6PYLDhu6V7H53LRa 0ez20BuAu+d0mTxeruv31fzYzxcoOEhYaHiI6HaXyNiYsOYYGakmWZmotmipSYeZufmJ9oQn ClrKRjlqqiq2dgW5CksF6Uoaa8v0WlZ7y0v0Gnfw2zusI5y3S5xsgwwczKwM7WKs8BxtjSLc jFB93S2Sre3s6U2+AR4uXq7+cY7+N74e/9Du/i5/X8E9xkCP7z8KaFuDfv/k9QuYruAegisI Inz3UCErWjUYRkwlcQ4Z/hoM623Tl3HLRjsgBc6DF1JLRxMlTTpYmTINxRgrL0KMeWbkC5g2 7eGc+AcGTI/UWv5copPFUKJFex5FajTEUqaPoj61NJXqR5TsUF3NklXr1oZOvyqx+igf2gxc zZKkELZsQmxy3QKBMxNJp718+/pde8Gr3Ra78p78izgx3z5BBxPGE6zxYcWUK9c1d9mxOcj7 JhizvFdvWw+jNXeYWTZX36pkFr8szcWw6RO6hKRuLbipLoi5t2ZWC3u2htpiAQLkOssZ74eA hf84hty1aFfKxT0L69zJ2Mm9J2O803tq0uxQu0NILpm1nHOFGwcnvwxT7C7VFwhmj9f9b/gk /s1PDz3QL9XYthtn7/FHk38S/HVeLeAQGBmCUii4IH0WqBYQahJGQWGF1AVGSjbE7bdhfwc2 1RmISYioHIkl7tShhymqSCAg0Ln4IlmdnGZhjOlMM1eOPeyI3XecHdngiicKeYN8N7FFnxel AcgkLrnFtaJksO1YZXkDHihKXsFx2WURZOp2YZZHjulkmW+0yQ9aYWqZGZxu8kBlgFOqKWWd dt6JQyujdTQnb2meCWigrcHVUqHD+ZVoDoJe5lBzy9FSZKQgzAImPFmdiaimMPa5XztLhRqq qEo5uKRx9kWVqnWtqgolXrM+2OifullK63x+8Lqdb6/p+qqPvZIG/t6s1glb7FBhAnush79C yyxG6gGWZ7QjiKisej5dilmN2paAK45xohIrcK6Ou+mX5p67GrKYstuuJ9TqSeyjmL7LLj33 DtstiuvSq+9hLEFILqn8RntQwFU9ua3CBNdq07/1pbJwsNZOzGh0CycrG483Zqwpod1m0l69 1XKc5G04joNMwDFbnChPzZV0ncjTOlwzm5S6HGe4zbIM8KGMqhtgrUMTvTTSDap43nznMo0m 1Ccp/bTT3lINsdWvCr1gx/hyPbDXQWJdoYxjM51pYR+PKVrRRGdqD88bexYRznZv+C/dxbKl Z8ss+112VySCXDHNCPqtJMnfAq6x3P1S/ivxN8riJi29RTKj+N3DeS75sdjBbLfFcnZuGpaC qyzvlnun3ineDt+LXua9nvrz2++6bbuqzv5GOOWc9x6pzeHufuuXqxdvFO3Cu3jQ8ndaRHLb S1IfepmVJtyqpTytPb2njl+Ldsdswhu+veM3C3y88JbadJX4SSOe+7tiVjWT3K7/H3DTss+O rQnpM/GpSYgQJquM2Yp/BREQA2nksRqxinBF6ZqEVDOkdLmqcRokTeH4c0Bf2G+DIzwYBSWS rwyCJoUmfOA6jIWnFXIEdcOI1QpviMMc6nCHivmKrdjHwyAKcYhEhOE9fgjEIipxiUxkkA+h Q7ZKdNAsRiQGuhJn0yd8XFEz63EhI7bomNeZAowayY8RxFiKKZohZWbyYhT3IaUjoPGNaYnj Gf9Hx2+45yzEyaMeg+JGw80xiiPy4xcjZ0hC3CiRjGykIx8JyUhKcpKU9NIgK/kmTArikprM YCf5wMlPFiOQomwSKUs5Q1TWIZSqrMgpWwnLWMpylrSspS1victc6nKXvOylL38JzGAKc5jE LKYxj4nMZCpzmcxspjOfCc1oSnOa1KymNa+JzWxqk5YFAAA7 --Message-Boundary-13530-- ========================================================================= Instruções para entrar na lista, sair da lista e usar a lista em http://www.mat.puc-rio.br/~nicolau/olimp/obm-l.html O administrador desta lista é ========================================================================= From owner-obm-l@sucuri.mat.puc-rio.br Tue May 21 21:12:58 2002 Return-Path: Received: (from majordom@localhost) by sucuri.mat.puc-rio.br (8.9.3/8.9.3) id VAA20713 for obm-l-list; Tue, 21 May 2002 21:12:54 -0300 Received: from smtp-4.ig.com.br (smtp-4.ig.com.br [200.226.132.153] (may be forged)) by sucuri.mat.puc-rio.br (8.9.3/8.9.3) with SMTP id VAA20703 for ; Tue, 21 May 2002 21:12:51 -0300 From: ezer@ig.com.br Received: (qmail 13257 invoked from network); 22 May 2002 00:00:38 -0000 Received: from shasta057161.ig.com.br (HELO house) (200.151.57.161) by smtp-4.ig.com.br with SMTP; 22 May 2002 00:00:38 -0000 To: obm-l@mat.puc-rio.br Date: Wed, 1 Jan 1997 01:29:26 -0200 MIME-Version: 1.0 Subject: =?ISO-8859-1?Q?Re:_[obm-l]_tri=E2ngulos?= Message-ID: <32C9BDF6.25835.4FDA64@localhost> In-reply-to: <20020519014346.92568.qmail@web10104.mail.yahoo.com> X-mailer: Pegasus Mail for Windows (v4.01) Content-type: text/plain; charset=ISO-8859-1 Content-description: Mail message body Content-Transfer-Encoding: 8bit X-MIME-Autoconverted: from Quoted-printable to 8bit by sucuri.mat.puc-rio.br id VAA20710 Sender: owner-obm-l@sucuri.mat.puc-rio.br Precedence: bulk Reply-To: obm-l@mat.puc-rio.br Olah Rafael, Nao sei se entendi; vc quer uma formula geral para calcular o numero de triangulos iguais que um triangulo semelhante a estes suporta em funcao do numero de lados dos pequenos que cabem num grande? Se for isso, eu pensei assim: Podemos perceber que o numero de triangulos de uma carreira decresce a cada carreira que se conta. E podemos perceber que para cada carreira tem uma outra de numero de triangulos iguais, soh que de cabeca pra baixo, menos a carreira da base. Com base nesses dados, podemos esbocar uma formula: sendo N o numero de triangulos, e L o numero de lados; N = L + 2(L-1) + 2(L-2) + 2(L-3) + ... + 2(L-(L-1)) Como podemos perceber, temos L termos, levando em conta o L. N = L + 2((L -1)+(L-2)+(L-3)+... ) + 2(L-L+1) N = L + 2 +2((L-1)+(L-2)...) Nos temos L-2 termos dentro dos colchetes (sem levar em conta o L e o 2). Logo: N = L + 2 +2((L-2)L) -2(1+2+3+4...) Aqui temos uma PA de termo inicial 1, razao 1 e termo final L-2 Logo, 1+2+3+4... +L-2 = [(1+L-2)(L-2)]/2 = (L^2 - 3L + 2)/2 Substituindo: N = L + 2 +2((L-2)L) -2(L^2 - 3L +2)/2 N = L+2 +2L^2 - 4L -L^2 +3L - 2 Fazendo a continha, chegamos a incrivel formula: N = L^2 :c) Grande Abraco, Ezer F. da Silva On 18 May 2002 at 18:43, Rafael WC wrote: > Pessoal, ontem mandei uma dúvida sobre contar o total > de triângulos de todos os tamanhos de uma figura como > a que enviei abaixo novamente. Pensei muito sobre esse > problema e cheguei a uma fórmula não muito amigável, > mas até que não é ruim. Já dá até pra escrever um > algoritmo pra rodar no computador se quiser. > > Primeiro, eu chamei de x o número de lados de > triângulos que temos na base. Por exemplo, se tivermos > um triângulo só x = 1. > /_\ > > Se tivermos uma figura com quatro triângulos de menor > tamanho, temos: > /_\ > /_\ /_\ > > x = 2 > > Na figura que mandei, temos x = 4. > > Com isso, já que você tem triângulos de diferentes > tamanhos, você deve contar separadamente os triângulos > que têm como lado 1 traço, 2 traços, 3 traços...E > depois tem que contar os triângulos que estão de > cabeça pra baixo com esses mesmos tamanhos. > > Se você fizer isso em função dos traços da base não > fica muito ruim. Todas as linhas vou escrever a soma > de várias parcelas de x menos alguma coisa. Quando > você for calcular para algum x, você vai fazer as > subtrações até encontrar o valor zero, aí você para. > Por exemplo, na primeira linha temos: > x + (x - 1) + (x - 2) + (x - 3) + ... > > Se você tiver x = 2, você irá somar até x + (x - 1), > porque o próximo dará zero e aí você deve parar. > > Bom, no final você encontra isso: > triângulos de lado 1: > cabeça pra cima = x + (x - 1) + (x - 2) + (x - 3) + > ... > cabeça pra baixo = (x - 1) + (x - 2) + (x - 3) + ... > total = x + 2.[(x - 1) + (x - 2) + (x - 3) + ...] > É como se o triângulo maior de todos fosse dividido em > várias linhas, aí você vai contando de cada linha. > > triângulos de lado 2: > cabeça pra cima = (x - 1) + (x - 2) + (x - 3) + (x - > 4) + ... > cabeça pra baixo = (x - 3) + (x - 4) + (x - 5) + ... > total = (x - 1) + (x - 2) + 2.[(x - 3) + (x - 4) + > ...] > > Por que aqui começamos a ter de cabeça pra baixo só > com (x - 3)? Porque para termos um triângulo de cabeça > pra baixo, o triângulo maior tem que ter o dobro de > traços na base do que o tamanho do triângulo. Como > esse tem lado 2, precisamos ter x = 4, que se fizermos > (x - 3) dará 1. Enquanto x for menor que 4 esse número > será negativo ou zero e aí não vamos contar. > > triângulos de lado 3: > cabeça pra cima = (x - 2) + (x - 3) + (x - 4) + (x - > 5) + ... > cabeça pra baixo = (x - 5) + (x - 6) + (x - 7) + ... > total = (x - 2) + (x - 3) + (x - 4) + 2.[(x - 5) + > ...] > > E assim teremos sempre esse padrão. Os triângulos de > cabeça pra cima começam sempre com (x - a), onde "a" é > o número anterior ao tamanho do triângulo. E os > triângulos de cabeça pra baixo começam sempre com x - > (2a - 1). Depois os outros termos você vai tirando > sempre 1. > > No final das contas você pode somar tudo isso. Soma os > triângulo de cabeça pra cima com os de cabeça pra > baixo de todos os tamanhos. O problema é que não pode > desenvolver muita coisa, porque não pode misturar x - > 3 com x - 4, porque se você tiver x = 4, você não terá > o termo x - 4. Mas somando apenas x - 1 com x - 1 e x > - 2 com x -2, você terá: > total = x + 3.(x - 1) + 4.(x - 2) + 6.(x - 3) + 7.(x - > 4) + 9.(x - 5) + 10.(x - 6) + 12.(x - 7) + 13.(x - 8) > + ... > > No final você tem então todos os fatores x, x - 1, x - > 2, x - 3, ... e os coeficientes de cada um têm uma > ordem até boazinha: > 1, (pula o 2), 3, 4, (pula o 5), 6, 7, (pula o 8), 9, > 10, (pula o 11), 12, 13, (pula o 14), ... > > E você vai usar a fórmula até o termo em que quando > fizer a diferença de x com alguma coisa dê zero. Ou > você pode até fazer a seguinte regra: considere que > desse valor total você vai pegar apenas os x primeiros > termos. > > Por exemplo, vamos pegar o triângulo da figura que tem > 4 traços na base, ou seja x = 4. Então vamos pegar até > o quarto termo dessa fórmula e fazer x = 4: > total = x + 3.(x - 1) + 4.(x - 2) + 6.(x - 3) > total = 4 + 3.(4 - 1) + 4.(4 - 2) + 6.(4 - 3) > total = 4 + 3.3 + 4.2 + 6.1 > total = 4 + 9 + 8 + 6 > total = 27 > > E aí você pode fazer pra qualquer x. Aquele menor que > tinha x = 2, só pegamos os 2 primeiros termos: > total = x + 3.(x - 1) > total = 2 + 3.(2 - 1) > total = 2 + 3.1 > total = 2 + 3 > total = 5 > > De qualquer jeito você não precisa ficar contando um > por um e correr o risco de se perder mais facilmente. > > Mas o meu problema agora é o seguinte. Suspeito que > ainda dê para simplificar a fórmula, considerando duas > fórmulas, uma para quando x é par e outra para quando > x é ímpar. Talvez simplifique, mas aí você tem duas > fórmulas, não sei. Ainda não consegui. > > Será que alguém consegue melhorar daqui pra frente. O > pior acho que já passou. > > Um abraço, > > Rafael. > > ===== > Rafael Werneck Cinoto > ICQ# 107011599 > rwcinoto@yahoo.com > rafael.caixa@gov.com.br > matduvidas@yahoo.com.br > http://www.rwcinoto.hpg.com.br/ > > __________________________________________________ > Do You Yahoo!? > LAUNCH - Your Yahoo! Music Experience > http://launch.yahoo.com > ========================================================================= Instruções para entrar na lista, sair da lista e usar a lista em http://www.mat.puc-rio.br/~nicolau/olimp/obm-l.html O administrador desta lista é ========================================================================= From owner-obm-l@sucuri.mat.puc-rio.br Tue May 21 21:30:29 2002 Return-Path: Received: (from majordom@localhost) by sucuri.mat.puc-rio.br (8.9.3/8.9.3) id VAA21514 for obm-l-list; Tue, 21 May 2002 21:29:59 -0300 Received: from smtp-5.ig.com.br (smtp-5.ig.com.br [200.226.132.154] (may be forged)) by sucuri.mat.puc-rio.br (8.9.3/8.9.3) with SMTP id VAA21510 for ; Tue, 21 May 2002 21:29:57 -0300 From: ezer@ig.com.br Received: (qmail 21211 invoked from network); 22 May 2002 00:17:44 -0000 Received: from shasta057161.ig.com.br (HELO house) (200.151.57.161) by smtp-5.ig.com.br with SMTP; 22 May 2002 00:17:44 -0000 To: obm-l@mat.puc-rio.br Date: Wed, 1 Jan 1997 01:46:36 -0200 MIME-Version: 1.0 Subject: [obm-l] Sequencia Sinistra! Message-ID: <32C9C1FC.18697.5F9261@localhost> X-mailer: Pegasus Mail for Windows (v4.01) Content-type: text/plain; charset=US-ASCII Content-transfer-encoding: 7BIT Content-description: Mail message body Sender: owner-obm-l@sucuri.mat.puc-rio.br Precedence: bulk Reply-To: obm-l@mat.puc-rio.br Olah Nicolau e todos da lista, Nicolau, eu estava fucando os arquivos, e achei um email seu sobre a sequencia numerica 1^1 + 2^2 + 3^3 +... N^N, e sobre achar uma forma fechada para ela. Vc poderia mostrar a forma fechada desta "aberracao", e qual foi o raciocinio usado para chegar a ela? Desde jah agradeco, Ezer F. da Silva ========================================================================= Instruções para entrar na lista, sair da lista e usar a lista em http://www.mat.puc-rio.br/~nicolau/olimp/obm-l.html O administrador desta lista é ========================================================================= From owner-obm-l@sucuri.mat.puc-rio.br Tue May 21 23:45:43 2002 Return-Path: Received: (from majordom@localhost) by sucuri.mat.puc-rio.br (8.9.3/8.9.3) id XAA23447 for obm-l-list; Tue, 21 May 2002 23:45:24 -0300 Received: from smtp-6.ig.com.br (smtp-6.ig.com.br [200.226.132.155] (may be forged)) by sucuri.mat.puc-rio.br (8.9.3/8.9.3) with SMTP id XAA23443 for ; Tue, 21 May 2002 23:45:22 -0300 Received: (qmail 16152 invoked from network); 22 May 2002 02:33:06 -0000 Received: from shasta008228.ig.com.br (HELO Maredu) (200.151.8.228) by smtp-6.ig.com.br with SMTP; 22 May 2002 02:33:06 -0000 Message-ID: <000d01c20137$9449f8a0$e40897c8@Maredu> From: "Eduardo Quintas" To: References: <200205201505.g4KF5FKC003807@trex.centroin.com.br> Subject: [obm-l] Problema Date: Tue, 21 May 2002 23:22:29 -0300 MIME-Version: 1.0 Content-Type: text/plain; charset="iso-8859-1" Content-Transfer-Encoding: 8bit X-Priority: 3 X-MSMail-Priority: Normal X-Mailer: Microsoft Outlook Express 5.50.4133.2400 X-MIMEOLE: Produced By Microsoft MimeOLE V5.50.4133.2400 Sender: owner-obm-l@sucuri.mat.puc-rio.br Precedence: bulk Reply-To: obm-l@mat.puc-rio.br resolva a equação : x^(sqrt x) = 1/2 PS.: x elevado a raiz quadrada de x = 1/2 ========================================================================= Instruções para entrar na lista, sair da lista e usar a lista em http://www.mat.puc-rio.br/~nicolau/olimp/obm-l.html O administrador desta lista é ========================================================================= From owner-obm-l@sucuri.mat.puc-rio.br Wed May 22 09:59:10 2002 Return-Path: Received: (from majordom@localhost) by sucuri.mat.puc-rio.br (8.9.3/8.9.3) id JAA27505 for obm-l-list; Wed, 22 May 2002 09:57:01 -0300 Received: (from nicolau@localhost) by sucuri.mat.puc-rio.br (8.9.3/8.9.3) id JAA27500 for obm-l@mat.puc-rio.br; Wed, 22 May 2002 09:57:01 -0300 Date: Wed, 22 May 2002 09:57:01 -0300 From: "Nicolau C. Saldanha" To: obm-l@mat.puc-rio.br Subject: Re: [obm-l] Sequencia Sinistra! Message-ID: <20020522095701.D27368@sucuri.mat.puc-rio.br> References: <32C9C1FC.18697.5F9261@localhost> Mime-Version: 1.0 Content-Type: text/plain; charset=iso-8859-1 Content-Disposition: inline Content-Transfer-Encoding: 8bit User-Agent: Mutt/1.2.5i In-Reply-To: <32C9C1FC.18697.5F9261@localhost>; from ezer@ig.com.br on Wed, Jan 01, 1997 at 01:46:36AM -0200 Sender: owner-obm-l@sucuri.mat.puc-rio.br Precedence: bulk Reply-To: obm-l@mat.puc-rio.br On Wed, Jan 01, 1997 at 01:46:36AM -0200, ezer@ig.com.br wrote: > > Olah Nicolau e todos da lista, > > Nicolau, eu estava fucando os arquivos, > e achei um email seu sobre a sequencia numerica > 1^1 + 2^2 + 3^3 +... N^N, e sobre achar uma forma fechada > para ela. Vc poderia mostrar a forma fechada desta > "aberracao", e qual foi o raciocinio usado para chegar a ela? Não me lembro muito bem dessa história. Eu acabo de procurar a seqüência em http://www.research.att.com/~njas/sequences/ que é uma enciclopédia on-line de seqüências de inteiros. Lá há uma referência: Problem 4155, Amer. Math. Monthly, 53 (1946), 471. mas duvido que exista uma fórmula fechada. []s, N. ========================================================================= Instruções para entrar na lista, sair da lista e usar a lista em http://www.mat.puc-rio.br/~nicolau/olimp/obm-l.html O administrador desta lista é ========================================================================= From owner-obm-l@sucuri.mat.puc-rio.br Wed May 22 13:05:22 2002 Return-Path: Received: (from majordom@localhost) by sucuri.mat.puc-rio.br (8.9.3/8.9.3) id NAA29962 for obm-l-list; Wed, 22 May 2002 13:03:30 -0300 Received: from hotmail.com (law2-f82.hotmail.com [216.32.181.82]) by sucuri.mat.puc-rio.br (8.9.3/8.9.3) with ESMTP id NAA29958 for ; Wed, 22 May 2002 13:03:26 -0300 Received: from mail pickup service by hotmail.com with Microsoft SMTPSVC; Wed, 22 May 2002 08:51:31 -0700 Received: from 32.94.119.254 by lw2fd.hotmail.msn.com with HTTP; Wed, 22 May 2002 15:51:31 GMT X-Originating-IP: [32.94.119.254] From: "Paulo Santa Rita" To: obm-l@mat.puc-rio.br Subject: [obm-l] =?iso-8859-1?B?UmU6IFtvYm0tbF0gUmU6IFtvYm0tbF0gUmU6IFtvYm0tbF0gUmU6IFtv?= =?iso-8859-1?B?Ym0tbF0gQ29ycmXnYW86QXBlbG86IE1haXMgZGEgSWJlcm9hbWVyaWNh?= =?iso-8859-1?B?bmEocXVlc3RhbyBwZXNzb2FsKQ==?= Date: Wed, 22 May 2002 15:51:31 +0000 Mime-Version: 1.0 Content-Type: text/plain; charset=iso-8859-1; format=flowed Message-ID: X-OriginalArrivalTime: 22 May 2002 15:51:31.0523 (UTC) FILETIME=[8AD65130:01C201A8] Sender: owner-obm-l@sucuri.mat.puc-rio.br Precedence: bulk Reply-To: obm-l@mat.puc-rio.br Grande Duda ! E ai maluco, tudo blz ? Realmente, eu nao recebi a mensagem do Fabio a que voce se refere. Aqui onde estou ha uma preocupacao muito grande com seguranca, mas o sub-sistema que cuida disso ta meio doido e as vezes passa sistematicamente a bloquear certas mensagens, que ele identifica por letras contidas no nome da pessoa que envia. Ele vai ser trocado, mas, ate la, sou obrigado a conviver com isso. O problema surgiu com o nosso colega Dirichlet, que perguntou : E POSSIVEL QUE AS RAIZES CUBICAS DE TRES NUMEROS PRIMOS, DOIS A DOIS DISTINTOS, SEJAM TERMOS DE UMA MESMA PROGRESSAO ARITMETICA ? Eu conjecturei algo mais amplo, a saber : SE "A", "B" e "C" SAO NATURAIS, DOIS A DOIS DISTINTOS, NENHUM DELES POTENCIA N-ESIMA DE OUTRO NATURAL, ENTAO ELES NAO PODEM SER TERMOS DE UMA MESMA PROGRESSAO ARITMETICA. Claramente que a prova do fato acima responde a pergunta do Dirichlet. PRIMEIRO PASSO DA IDEIA Sem perda de generalidade podemos supor A < B < C. Evidentemente : RAIZ_N(A) < RAIZ_N(B) < RAIZ_N(C). Dizer que essas raizes sao termos de uma mesma PA significa dizer que existem naturais "R", "S" e "T" tais que : X + YR = RAIZ_N(A) X + YS = RAIZ_N(B) X + YT = RAIZ_N(C) Para algum par (X,Y) de numeros reais ( que serao, respectivamente : X=primeiro termo da PA, Y=razao da PA ) Veja que eu nao estou impondo que "R", "S" e "T" estejam em PA. Nao estou impondo tambem uma ordem qualquer sobre eles, isto e, nao estou impondo que, por exemplo, R < S < T. O certo e que haverao os ponto (R,RAIZ_N(A)) e (T,RAIZ_N(C)). Como a funcao X + Y*N - X e Y reais fixos e N percorrendo os naturais - e linear, se Y > o ela sera crescente e, obrigatoriamente, R < S < T. Se Y < 0 ela sera decrescente e R > S > T. Nos dois casos, a RAIZ_N(B) sera a ordenada de um ponto interior ao intervalo de extremos R e T. Vamos supor doravante, sem perda de generalidade, que R < T. Queremos, pois, saber se pode existir um natural Z do conjunto R+1, R+2, ..., T-2,T-1 tal que X + YZ = RAIZ_N(B). SEGUNDO PASSO DA IDEIA. Imagine que voce esta no ponto (R,RAIZ_N(A)). Qual sera a ordenada do ponto que esta sobre a reta que liga (R,RAIZ_N(A)) a (T,RAIZ_N(C)) e que tem abscissa R+1 ? sera : RAIZ_N(A) + (RAIZ_N(C)-RAIZ_N(A))/(T-R) = [RAIZ_N(A)*(T-R-1) + RAIZ_N(C)]/(T-R) Se fosse no ponto de abscissa R+2, seria : [RAIZ_N(A)*(T-R-2) + 2RAIZ_N(C)]/(T-R) Os pesos sao sempre da forma : "T-R-i" e "i", isto e, nos estamos diante de uma media ponderada da forma : (p*RAIZ_N(A)+ q*RAIZ_N(C))/(p+q) com p e q naturais e p+q=T-R. Essa e a forma das ordenadas dos pontos sobre a reta que liga (R,RAIZ_N(A)) a (T,RAIZ_N(C)). Ja vimos que a RAIZ_N(B) tem que estar entre estes dois pontos. Logo, devem existir p e q atendendo as condicoes que especificamos acima e tais que : RAIZ_N(B) = (p*RAIZ_N(A) + q*RAIZ_N(C))/(p+q) TERCEIRO PASSO DA IDEIA : A funcao Y=RAIZ_N(X) e CONTINUA, CRESCENTE e CONVEXA. Isto e, para quaisquer naturais A e C vale : RAIZ_N((A+C)/2) > (RAIZ_N(A) + RAIZ_N(C))/2 O que me pareceu e que a contradicao vai surgir aqui, pois a expressao de convexidade acima pode ser trabalhada para incluir uma media ponderada tal como a que vimos no segundo passo. Mas, em verdade, EU NAO FIZ UMA DEMONSTRACAO, vale dizer, NAO PROVEI NADA, apenas dei uma sugestao de um caminho que me pareceu viavel. ALERTEI QUE AS RAIZES N-ESIMAS DE PONTENCIAS N-ESIMAS E UMA PA, EVIDENTEMENTE ! Um abraco Paulo Santa Rita 4,1247,220502 >Oi Paulo! > >Não sei se compreendi bem esse seu e-mail. :) > >O problema é o seguinte: desenhe no plano os pontos (R_N(x), x) para todo x >inteiro positivo. Você vai ter destacado alguns pontos da função contínua >f(x) = R_N(x) para todo x real. Agora escolha uma PA de 3 termos inteiros >positivos, digamos Y_1, Y_2, Y_3. Marque os três pontos no eixo ordenado >(0,Y_1), (0, Y_2) e (0, Y_3), para cada um deles trace uma reta horizontal, >ou seja, paralela ao eixo das abscissas. Suponhamos que essas três retas >passem por três dos pontos do gráfico que você tinha destacado. Esses três >pontos chame de (X_1, Y_1) também (X_2, Y_2) e finalmente (X_3, Y_3). O que >nós temos, agora, é que os Y_1, Y_2 e Y_3 está em forma de PA, mas isso não >precisa acontecer com os X_1, X_2 e X_3. OU SEJA, os pontos (X_n, Y_n) NÃO >PRECISAM ESTAR SOBRE UMA RETA. E daí esse seu argumento não prova nada >sobre >o problema inicial. Em outras palavras, o que você demonstrou pela >convexidade da f, a saber, que nenhuma reta corta o gráfico da f em três >pontos distintos, não garante que se p, q e r foram primos distintos então >f(p), f(q) e f(r) não formam uma PA. > >Se o seu argumento é só um passo para resolver o problema, perdoe o meu >comentário acima: mas eu acredito que esse caminho não vai levar a uma >solução. > >E só para terminar o e-mail: o Fabio Dias Moreira é que ressaltou isso que >eu disse aí em cima, e pelo visto, o Paulo não chegou a ler a mensagem >dele. > >Um abraço! > >Eduardo Casagrande Stabel. > >PS. eu não descarto a possibilidade de eu não ter compreendido bem as >mensagens do Paulo e do Fábio, e eu é que precise de uma explicação. > > > > > > Era essa a ideia da demonstracao. Talvez eu nao tenha conseguido ser >claro > > naquele momento. E obrigado pelo elogio. > > > > Um abraco > > Fica com Deus > > Paulo Santa Rita > > 3,1635,210502 > > > > > > >From: peterdirichlet@zipmail.com.br > > >Reply-To: obm-l@mat.puc-rio.br > > >To: Lista de Discussao , p_ssr@hotmail.com > > >Subject: [obm-l] Re: [obm-l] Correçao:Apelo: Mais da Iberoamericana > > >( questao pessoal ) > > >Date: Tue, 21 May 2002 15:25:20 -0300 > > >O que eu realmente achei estranho foi o fato de ninguem(entre muitos de > > >voces) me responder ha tanto tempo(1 mes e meio,por volta disso),a nao > > >ser voce.Eu devia ter lhe respondido essa pergunta mas acabei > >cancelando > > >sem > > >querer a mensagem de resposta (junto com meu login), comentando a > > > >genialidade da ideia,que alias tinha um furo(esta das raizes > >cubicas.Eu > > >tentei algo com Teoria dos Numeros mas nao obtive >exito.Voce usou >graficos > > >de funçoes. O problema era que a s funcoes >eram discretas(naturais)e >nao > > >densas(reais).). > > > > > > _________________________________________________________________ > > O MSN Photos é o modo mais fácil de compartilhar e imprimir suas fotos: > > http://photos.msn.com/support/worldwide.aspx > > > > >========================================================================= > > Instruções para entrar na lista, sair da lista e usar a lista em > > http://www.mat.puc-rio.br/~nicolau/olimp/obm-l.html > > O administrador desta lista é > > >========================================================================= > > > > > >========================================================================= >Instruções para entrar na lista, sair da lista e usar a lista em >http://www.mat.puc-rio.br/~nicolau/olimp/obm-l.html >O administrador desta lista é >========================================================================= _________________________________________________________________ Chegou o novo MSN Explorer. Instale já. É gratuito: http://explorer.msn.com.br ========================================================================= Instruções para entrar na lista, sair da lista e usar a lista em http://www.mat.puc-rio.br/~nicolau/olimp/obm-l.html O administrador desta lista é ========================================================================= From owner-obm-l@sucuri.mat.puc-rio.br Wed May 22 14:28:47 2002 Return-Path: Received: (from majordom@localhost) by sucuri.mat.puc-rio.br (8.9.3/8.9.3) id OAA31464 for obm-l-list; Wed, 22 May 2002 14:28:08 -0300 Received: from www.zipmail.com.br (smtp.zipmail.com.br [200.187.242.10]) by sucuri.mat.puc-rio.br (8.9.3/8.9.3) with ESMTP id OAA31460 for ; Wed, 22 May 2002 14:28:05 -0300 From: peterdirichlet@zipmail.com.br Received: from [200.206.103.3] by www.zipmail.com.br with HTTP; Wed, 22 May 2002 14:15:24 -0300 Message-ID: <3CEBC9E20000019E@www.zipmail.com.br> Date: Wed, 22 May 2002 14:15:24 -0300 In-Reply-To: <91.1d4a4836.2a1c19d3@aol.com> Subject: [obm-l] =?iso-8859-1?Q?Re=3A=20=5Bobm=2Dl=5D=20=28nenhum=20assunto=29?= To: obm-l@mat.puc-rio.br MIME-Version: 1.0 Content-Type: text/plain; charset="iso-8859-1" Content-Transfer-Encoding: 8bit X-MIME-Autoconverted: from quoted-printable to 8bit by sucuri.mat.puc-rio.br id OAA31461 Sender: owner-obm-l@sucuri.mat.puc-rio.br Precedence: bulk Reply-To: obm-l@mat.puc-rio.br ANSWER:Bem,a parte 1 sai por paridades.E so ver que n e n+1 nao sao ambos impares. A segunda parte e bem mecanica.Teste n(n+1)mod 10 na porrada ate achar um ciclo e prove que o digito final deste n(n+1) nao pode ser 4 ou 8. Ate mais!!!!!Peterdirichlet. -- Mensagem original -- >mostre que para todo n natural,1) o número n(n+1)/2 está em IN e que 2)seu >algarismo das unidades não pode ser 2, nem 4, nem 7, nem 9. > Obrigado > Korshinói > TRANSIRE SVVM PECTVS MVNDOQUE POTIRE CONGREGATI EX TOTO ORBE MATHEMATICI OB SCRIPTA INSIGNIA TRIBVERE Medalha Fields(John Charles Fields) ------------------------------------------ Use o melhor sistema de busca da Internet Radar UOL - http://www.radaruol.com.br ========================================================================= Instruções para entrar na lista, sair da lista e usar a lista em http://www.mat.puc-rio.br/~nicolau/olimp/obm-l.html O administrador desta lista é ========================================================================= From owner-obm-l@sucuri.mat.puc-rio.br Wed May 22 18:41:14 2002 Return-Path: Received: (from majordom@localhost) by sucuri.mat.puc-rio.br (8.9.3/8.9.3) id SAA03558 for obm-l-list; Wed, 22 May 2002 18:40:48 -0300 Received: from fgvrj23.fgv.br (fgvrj23.fgv.br [200.20.164.23]) by sucuri.mat.puc-rio.br (8.9.3/8.9.3) with ESMTP id SAA03553 for ; Wed, 22 May 2002 18:40:46 -0300 Received: by FGVRJ23 with Internet Mail Service (5.5.2653.19) id <20VA58JB>; Wed, 22 May 2002 18:25:56 -0300 Message-ID: <765A72978645D4118B1C0000E229806D05B6B3A0@FGVRJ23> From: Ralph Teixeira To: "'obm-l@mat.puc-rio.br'" Subject: RES: [obm-l] Problema Date: Wed, 22 May 2002 18:25:55 -0300 MIME-Version: 1.0 X-Mailer: Internet Mail Service (5.5.2653.19) Content-Type: text/plain; charset="iso-8859-1" Content-Transfer-Encoding: 8bit X-MIME-Autoconverted: from quoted-printable to 8bit by sucuri.mat.puc-rio.br id SAA03555 Sender: owner-obm-l@sucuri.mat.puc-rio.br Precedence: bulk Reply-To: obm-l@mat.puc-rio.br resolva a equação : x^(sqrt x) = 1/2 Deixa eu ver... Note que temos de ter x>0. Então, vou fazer y=1/sqrt(x), isto é, x=1/y^2 para começar. Note que y>0 também. (1/y^2)^(1/y)=1/2 y^2^(-1/y)=1/2 y^2=2^y Ah-ha! Esse problema eu já vi por aqui.... Se eu me lembro bem, a gente tem três soluções: y=2, y=4 e uma outra solução negativa que usava o "Lambertiano" (eu já escrevi isso aqui antes). A solução negativa neste caso não presta, então ficamos só com x=1/4 ou x=1/16. Abraço, Ralph ========================================================================= Instruções para entrar na lista, sair da lista e usar a lista em http://www.mat.puc-rio.br/~nicolau/olimp/obm-l.html O administrador desta lista é ========================================================================= From owner-obm-l@sucuri.mat.puc-rio.br Wed May 22 18:59:20 2002 Return-Path: Received: (from majordom@localhost) by sucuri.mat.puc-rio.br (8.9.3/8.9.3) id SAA04008 for obm-l-list; Wed, 22 May 2002 18:59:12 -0300 Received: from sr1.terra.com.br (sr1.terra.com.br [200.176.3.16]) by sucuri.mat.puc-rio.br (8.9.3/8.9.3) with ESMTP id SAA04004 for ; Wed, 22 May 2002 18:59:09 -0300 Received: from mucuri.terra.com.br (mucuri.terra.com.br [200.176.3.39]) by sr1.terra.com.br (Postfix) with ESMTP id D18F86EBCB for ; Wed, 22 May 2002 18:47:16 -0300 (EST) Received: from nt (RJ231244.user.veloxzone.com.br [200.165.231.244]) (authenticated user ensr) by mucuri.terra.com.br (Postfix) with ESMTP id 4E1C292F37 for ; Wed, 22 May 2002 18:47:16 -0300 (EST) Message-ID: <001e01c201d9$ff2c0080$5400a8c0@ensrbr> From: "Luis Lopes" To: References: <32C9C1FC.18697.5F9261@localhost> <20020522095701.D27368@sucuri.mat.puc-rio.br> Subject: Re: [obm-l] Sequencia Sinistra! Date: Wed, 22 May 2002 18:45:29 -0300 MIME-Version: 1.0 Content-Type: text/plain; charset="iso-8859-1" Content-Transfer-Encoding: 8bit X-Priority: 3 X-MSMail-Priority: Normal X-Mailer: Microsoft Outlook Express 5.00.2615.200 X-MimeOLE: Produced By Microsoft MimeOLE V5.00.2615.200 Sender: owner-obm-l@sucuri.mat.puc-rio.br Precedence: bulk Reply-To: obm-l@mat.puc-rio.br Sauda,c~oes, Quando esse assunto estava fresco eu fui olhar a referência que o N. indicara. Não há fórmula fechada. Há somente um resultado A <= S_n <= B. Mas nem A nem B são interessantes. []'s Luis -----Mensagem Original----- De: Nicolau C. Saldanha Para: Enviada em: quarta-feira, 22 de maio de 2002 09:57 Assunto: Re: [obm-l] Sequencia Sinistra! > On Wed, Jan 01, 1997 at 01:46:36AM -0200, ezer@ig.com.br wrote: > > > > Olah Nicolau e todos da lista, > > > > Nicolau, eu estava fucando os arquivos, > > e achei um email seu sobre a sequencia numerica > > 1^1 + 2^2 + 3^3 +... N^N, e sobre achar uma forma fechada > > para ela. Vc poderia mostrar a forma fechada desta > > "aberracao", e qual foi o raciocinio usado para chegar a ela? > > Não me lembro muito bem dessa história. > Eu acabo de procurar a seqüência em > > http://www.research.att.com/~njas/sequences/ > > que é uma enciclopédia on-line de seqüências de inteiros. > Lá há uma referência: > > Problem 4155, Amer. Math. Monthly, 53 (1946), 471. > > mas duvido que exista uma fórmula fechada. []s, N. > > ========================================================================= > Instruções para entrar na lista, sair da lista e usar a lista em > http://www.mat.puc-rio.br/~nicolau/olimp/obm-l.html > O administrador desta lista é > ========================================================================= > ========================================================================= Instruções para entrar na lista, sair da lista e usar a lista em http://www.mat.puc-rio.br/~nicolau/olimp/obm-l.html O administrador desta lista é ========================================================================= From owner-obm-l@sucuri.mat.puc-rio.br Wed May 22 21:53:42 2002 Return-Path: Received: (from majordom@localhost) by sucuri.mat.puc-rio.br (8.9.3/8.9.3) id VAA06067 for obm-l-list; Wed, 22 May 2002 21:53:10 -0300 Received: from smtp-5.ig.com.br (smtp-5.ig.com.br [200.226.132.154] (may be forged)) by sucuri.mat.puc-rio.br (8.9.3/8.9.3) with SMTP id VAA06063 for ; Wed, 22 May 2002 21:53:08 -0300 From: ezer@ig.com.br Received: (qmail 5331 invoked from network); 23 May 2002 00:40:59 -0000 Received: from shasta037102.ig.com.br (HELO house) (200.151.37.102) by smtp-5.ig.com.br with SMTP; 23 May 2002 00:40:59 -0000 To: obm-l@mat.puc-rio.br Date: Wed, 1 Jan 1997 01:31:15 -0200 MIME-Version: 1.0 Subject: Re: [obm-l] Problema Message-ID: <32C9BE63.5058.193B59@localhost> In-reply-to: <000d01c20137$9449f8a0$e40897c8@Maredu> X-mailer: Pegasus Mail for Windows (v4.01) Content-type: text/plain; charset=ISO-8859-1 Content-description: Mail message body Content-Transfer-Encoding: 8bit X-MIME-Autoconverted: from Quoted-printable to 8bit by sucuri.mat.puc-rio.br id VAA06064 Sender: owner-obm-l@sucuri.mat.puc-rio.br Precedence: bulk Reply-To: obm-l@mat.puc-rio.br Dica: O que eh exatamente sqtr(x)? A que deducao este fato pode nos levar? Pense mais um pouco.. Esse problema eh semelhante a um que jah foi discutido aqui.., soh que este eh verificavel :c) Ha uma resolucao do problema no final do email, mas tente antes vc mesmo ;c) Abracos, Ezer F. da Silva On 21 May 2002 at 23:22, Eduardo Quintas wrote: > resolva a equação : > > x^(sqrt x) = 1/2 > > PS.: x elevado a raiz quadrada de x = 1/2 > > ========================================================================= > Instruções para entrar na lista, sair da lista e usar a lista em > http://www.mat.puc-rio.br/~nicolau/olimp/obm-l.html > O administrador desta lista é > ========================================================================= x^sqrt(x) = 1/2 = x^x^(1/2) = 1/2 Como 1/2 eh igual a x^x^(1/2), podemos substituir na equacao: x^x^x^x^(1/2) = 1/2. E podemos substituir infinitamente. x^x^x^x^x^... = 1/2. Como 1/2 = x^x^x^x^x^..., entao podemos substituir e ficaremos com x^(1/2) = 1/2 (isso soh eh possivel pq infinito - 1 = infinito, ou seja, um conjunto infinito nao aumenta nem diminui quando lhe tiramos ou acrescentamos um numero finito de termos) x^(1/2) = 1/2 x = 1/4 Substituindo para verificar: x^sqtr(x) = 1/2 | (1/4)^sqtr(1/4) = 1/2 | sqtr(1/4) = 1/2 | 1/2 = 1/2 ========================================================================= Instruções para entrar na lista, sair da lista e usar a lista em http://www.mat.puc-rio.br/~nicolau/olimp/obm-l.html O administrador desta lista é ========================================================================= From owner-obm-l@sucuri.mat.puc-rio.br Thu May 23 08:03:53 2002 Return-Path: Received: (from majordom@localhost) by sucuri.mat.puc-rio.br (8.9.3/8.9.3) id IAA10606 for obm-l-list; Thu, 23 May 2002 08:03:21 -0300 Received: from calhau.terra.com.br (calhau.terra.com.br [200.176.3.20]) by sucuri.mat.puc-rio.br (8.9.3/8.9.3) with ESMTP id IAA10602 for ; Thu, 23 May 2002 08:03:19 -0300 Received: from mucuri.terra.com.br (mucuri.terra.com.br [200.176.3.39]) by calhau.terra.com.br (Postfix) with ESMTP id 83080474C5 for ; Thu, 23 May 2002 10:51:28 +0000 (GMT) Received: from stabel (dl-nas3-poa-C89A0650.p001.terra.com.br [200.154.6.80]) (authenticated user dudasta) by mucuri.terra.com.br (Postfix) with ESMTP id 8EB3392F55 for ; Thu, 23 May 2002 07:51:26 -0300 (EST) Message-ID: <003d01c20247$c7e01440$b7009ac8@stabel> From: "Eduardo Casagrande Stabel" To: References: Subject: [obm-l] =?iso-8859-1?Q?Re:_=5Bobm-l=5D_Re:_=5Bobm-l=5D_Re:_=5Bobm-l=5D_Re:_=5Bobm?= =?iso-8859-1?Q?-l=5D_Re:_=5Bobm-l=5D_Corre=E7ao:Apelo:_Mais_da_Iberoameri?= =?iso-8859-1?Q?cana=28questao_pessoal=29?= Date: Thu, 23 May 2002 07:51:21 -0300 MIME-Version: 1.0 Content-Type: text/plain; charset="iso-8859-1" Content-Transfer-Encoding: 8bit X-Priority: 3 X-MSMail-Priority: Normal X-Mailer: Microsoft Outlook Express 6.00.2600.0000 X-MimeOLE: Produced By Microsoft MimeOLE V6.00.2600.0000 Sender: owner-obm-l@sucuri.mat.puc-rio.br Precedence: bulk Reply-To: obm-l@mat.puc-rio.br From: "Paulo Santa Rita" > Grande Duda ! > E ai maluco, tudo blz ? > > Realmente, eu nao recebi a mensagem do Fabio a que voce se refere. Aqui onde > estou ha uma preocupacao muito grande com seguranca, mas o sub-sistema que > cuida disso ta meio doido e as vezes passa sistematicamente a bloquear > certas mensagens, que ele identifica por letras contidas no nome da pessoa > que envia. Ele vai ser trocado, mas, ate la, sou obrigado a conviver com > isso. > > O problema surgiu com o nosso colega Dirichlet, que perguntou : > > E POSSIVEL QUE AS RAIZES CUBICAS DE TRES NUMEROS PRIMOS, DOIS A DOIS > DISTINTOS, SEJAM TERMOS DE UMA MESMA PROGRESSAO ARITMETICA ? > > Eu conjecturei algo mais amplo, a saber : > > SE "A", "B" e "C" SAO NATURAIS, DOIS A DOIS DISTINTOS, NENHUM DELES POTENCIA > N-ESIMA DE OUTRO NATURAL, ENTAO ELES NAO PODEM SER TERMOS DE UMA MESMA > PROGRESSAO ARITMETICA. > > Claramente que a prova do fato acima responde a pergunta do Dirichlet. > > > PRIMEIRO PASSO DA IDEIA > > Sem perda de generalidade podemos supor A < B < C. Evidentemente : RAIZ_N(A) > < RAIZ_N(B) < RAIZ_N(C). Dizer que essas raizes sao termos de uma mesma PA > significa dizer que existem naturais "R", "S" e "T" tais que : > > X + YR = RAIZ_N(A) > X + YS = RAIZ_N(B) > X + YT = RAIZ_N(C) > > Para algum par (X,Y) de numeros reais ( que serao, respectivamente : > X=primeiro termo da PA, Y=razao da PA ) > > Veja que eu nao estou impondo que "R", "S" e "T" estejam em PA. Nao estou > impondo tambem uma ordem qualquer sobre eles, isto e, nao estou impondo que, > por exemplo, R < S < T. > > O certo e que haverao os ponto (R,RAIZ_N(A)) e (T,RAIZ_N(C)). Como a funcao > X + Y*N - X e Y reais fixos e N percorrendo os naturais - e linear, se Y > > o ela sera crescente e, obrigatoriamente, R < S < T. Se Y < 0 ela sera > decrescente e R > S > T. Nos dois casos, a RAIZ_N(B) sera a ordenada de um > ponto interior ao intervalo de extremos R e T. > > Vamos supor doravante, sem perda de generalidade, que R < T. Queremos, pois, > saber se pode existir um natural Z do conjunto R+1, R+2, ..., T-2,T-1 tal > que X + YZ = RAIZ_N(B). > > > > SEGUNDO PASSO DA IDEIA. > > Imagine que voce esta no ponto (R,RAIZ_N(A)). Qual sera a ordenada do ponto > que esta sobre a reta que liga (R,RAIZ_N(A)) a (T,RAIZ_N(C)) e que tem > abscissa R+1 ? sera : > > RAIZ_N(A) + (RAIZ_N(C)-RAIZ_N(A))/(T-R) = > [RAIZ_N(A)*(T-R-1) + RAIZ_N(C)]/(T-R) > > Se fosse no ponto de abscissa R+2, seria : > [RAIZ_N(A)*(T-R-2) + 2RAIZ_N(C)]/(T-R) > > Os pesos sao sempre da forma : "T-R-i" e "i", isto e, nos estamos diante de > uma media ponderada da forma : > > (p*RAIZ_N(A)+ q*RAIZ_N(C))/(p+q) com p e q naturais e p+q=T-R. > > Essa e a forma das ordenadas dos pontos sobre a reta que liga (R,RAIZ_N(A)) > a (T,RAIZ_N(C)). Ja vimos que a RAIZ_N(B) tem que estar entre estes dois > pontos. Logo, devem existir p e q atendendo as condicoes que especificamos > acima e tais que : > > RAIZ_N(B) = (p*RAIZ_N(A) + q*RAIZ_N(C))/(p+q) > > Caro amigo Paulo, usando a sua notação. Suponhamos que existe três pontos a < b < c naturais (não potências n-ésimas) de forma que R_n(a), R_n(b) e R_n(c) pertencem a uma progressão aritmética. Essa progressão possui primeiro termo X e razão Y. E existem dois inteiros R e T tais que: X + Y*R = R_n(a) X + Y*T = R_n(c) Você mostrou que caso seja verdade que R_n(b) pertenca a essa mesma progressão aritmética então vai existir um natural S tal que: X + Y*S = R_n(b) E mais o S divide o segmento [R, T] na mesma proporção que o R_n(b) divide o segmento [R_n(a), R_n(c)]. Em outras palavras o ponto (S, R_n(b)) pertence à reta que liga os dois pontos (R, R_n(a)) e (T, R_n(c)). Mas repare que nenhum desses tres pontos pertence ao gráfico da função contínua f(x) = R_n(x). Os pontos que pertencem ao gráfico são os seguintes: (a, R_n(a)), (b, R_n(b)) e (c, R_n(c)) Ou ainda (a, X + Y*R), (b, X + Y*S) e (c, X + Y*T) E esses três pontos não precisam estar sobre uma reta. Por que os a, b e c não pertencem a uma mesma progressão aritmética, pelo menos isso você não demonstrou. E desse modo a convexidade da função f não contradisse a existência dos R_n(a), R_n(b) e R_n(c) dentro de uma progressão aritmética. Outro detalhe: onde na sua prova, você usa o fato de que R_n(a), R_n(b) e R_n(c) não são inteiros? Um abração! Eduardo Casagrande Stabel. Porto Alegre, RS. > > TERCEIRO PASSO DA IDEIA : > > A funcao Y=RAIZ_N(X) e CONTINUA, CRESCENTE e CONVEXA. Isto e, para quaisquer > naturais A e C vale : > > RAIZ_N((A+C)/2) > (RAIZ_N(A) + RAIZ_N(C))/2 > > O que me pareceu e que a contradicao vai surgir aqui, pois a expressao de > convexidade acima pode ser trabalhada para incluir uma media ponderada tal > como a que vimos no segundo passo. Mas, em verdade, EU NAO FIZ UMA > DEMONSTRACAO, vale dizer, NAO PROVEI NADA, apenas dei uma sugestao de um > caminho que me pareceu viavel. ALERTEI QUE AS RAIZES N-ESIMAS DE PONTENCIAS > N-ESIMAS E UMA PA, EVIDENTEMENTE ! > > Um abraco > Paulo Santa Rita > 4,1247,220502 > > > > ========================================================================= Instruções para entrar na lista, sair da lista e usar a lista em http://www.mat.puc-rio.br/~nicolau/olimp/obm-l.html O administrador desta lista é ========================================================================= From owner-obm-l@sucuri.mat.puc-rio.br Thu May 23 10:07:20 2002 Return-Path: Received: (from majordom@localhost) by sucuri.mat.puc-rio.br (8.9.3/8.9.3) id KAA12134 for obm-l-list; Thu, 23 May 2002 10:06:00 -0300 Received: from smtp-4.ig.com.br (smtp-4.ig.com.br [200.226.132.153] (may be forged)) by sucuri.mat.puc-rio.br (8.9.3/8.9.3) with SMTP id KAA12128 for ; Thu, 23 May 2002 10:05:53 -0300 From: ezer@ig.com.br Received: (qmail 19542 invoked from network); 23 May 2002 12:53:45 -0000 Received: from shasta024223.ig.com.br (HELO house) (200.151.24.223) by smtp-4.ig.com.br with SMTP; 23 May 2002 12:53:45 -0000 To: obm-l@mat.puc-rio.br Date: Wed, 1 Jan 1997 00:58:29 -0200 MIME-Version: 1.0 Subject: [obm-l] Re: Aumento de corda Message-ID: <32C9B6B5.14696.341499@localhost> In-reply-to: X-mailer: Pegasus Mail for Windows (v4.01) Content-type: text/plain; charset=ISO-8859-1 Content-description: Mail message body Content-Transfer-Encoding: 8bit X-MIME-Autoconverted: from Quoted-printable to 8bit by sucuri.mat.puc-rio.br id KAA12131 Sender: owner-obm-l@sucuri.mat.puc-rio.br Precedence: bulk Reply-To: obm-l@mat.puc-rio.br On 19 May 2002 at 11:51, Adherbal Rocha Filho wrote: > > mais um probleminha: > suponha q a Terra eh uma esfera e que uma corda está amarrada ao redor da > linha do equador.Agora suponha que esta corda eh aumentada em um metro > ,formando uma circunferencia maior,qual será a distancia entre a superficie > da Terra e a corda? E se eu fizesse o mesmo pra uma bola de futebol,qual > seria a distancia? > > valeu! > []´s > Adherbal > > Aloha manos! O primeiro comprimento da corda eh C = 2*P i*R O segundo eh C+1 = 2*Pi*R' O que ele quer saber eh a diferenca entre o primeiro raio e o novo raio, ou seja, a distancia da corda a superficie da Terra. Entao: Dist = R' - R | Dist = (C+1)/2*Pi - C/2*Pi Dist = 1/2Pi Para a bola a distancia seria a mesma, jah que a resposta nao depende de nenhuma variavel. Agora, eu gostaria de questionar uma coisa: como a distancia da corda a bola e da corda a Terra podem ser iguais? Supoe-se que a corda que na qual a Terra estah envolta seja monstruosamente grande, e que a corda que envolve a bola seja bem pequena. Como um acrescimo igual as duas pode resultar numa distancia igual? Para a corda da bola o aumento foi de talvez 100%, jah para a corda da Terra eh algo tipo 0,000.. trocentos zeros... 0001%... ========================================================================= Instruções para entrar na lista, sair da lista e usar a lista em http://www.mat.puc-rio.br/~nicolau/olimp/obm-l.html O administrador desta lista é ========================================================================= From owner-obm-l@sucuri.mat.puc-rio.br Thu May 23 12:12:10 2002 Return-Path: Received: (from majordom@localhost) by sucuri.mat.puc-rio.br (8.9.3/8.9.3) id MAA14476 for obm-l-list; Thu, 23 May 2002 12:11:04 -0300 Received: from hotmail.com (law2-f71.hotmail.com [216.32.181.71]) by sucuri.mat.puc-rio.br (8.9.3/8.9.3) with ESMTP id MAA14472 for ; Thu, 23 May 2002 12:11:01 -0300 Received: from mail pickup service by hotmail.com with Microsoft SMTPSVC; Thu, 23 May 2002 07:59:08 -0700 Received: from 200.229.244.100 by lw2fd.hotmail.msn.com with HTTP; Thu, 23 May 2002 14:59:07 GMT X-Originating-IP: [200.229.244.100] From: "Paulo Santa Rita" To: obm-l@mat.puc-rio.br Subject: [obm-l] =?iso-8859-1?B?UmU6IFtvYm0tbF0gUmU6IFtvYm0tbF0gUmU6IFtvYm0tbF0gUmU6IFtv?= =?iso-8859-1?B?Ym0tbF0gUmU6IFtvYm0tbF0gUmU6IFtvYm0tbF0gQ29ycmXnYW86QXBl?= =?iso-8859-1?B?bG86IE1haXMgZGEgSWJlcm9hbWVyaWNhbmEocXVlc3RhbyBwZXNzb2Fs?= =?iso-8859-1?B?KQ==?= Date: Thu, 23 May 2002 14:59:07 +0000 Mime-Version: 1.0 Content-Type: text/plain; charset=iso-8859-1; format=flowed Message-ID: X-OriginalArrivalTime: 23 May 2002 14:59:08.0374 (UTC) FILETIME=[63C99760:01C2026A] Sender: owner-obm-l@sucuri.mat.puc-rio.br Precedence: bulk Reply-To: obm-l@mat.puc-rio.br Grande Duda, Parece que agora, realmente, estamos fazendo jus ao titulo da lista : lista de DISCUSSAO de problemas de Matematica. Varias inteligencias pensando produzem muito mais que uma. A linguagem e um problema e fica dificil voce exprimir com precisao as coisas. Voce pensa que passou bem uma ideia e ai alguem, com toda justica, entende outra. O ideal seria que as pessoas se comunicassem pelo pensamento. Bom, vamos la. O que voce diz ai embaixo e precisamente o que eu falei ... Quando eu coloquei : RAIZ_N[ (A+B)/2 ] > (RAIZ_N(A)+RAIZ_N(B))/2 DISSE ( ou penso que disse ! ) que o lado direito da desigualdade pode ser trabalhado para aparecer a media ponderada que eu havia falado no passo anterior e que dai e que vai surgir a contradicao, pois o fato das tres raizes estarem em PA implica que a raiz do meio deve, obrigatoriamente, ser uma media ponderada das raizes dos extremos. Sejam A e B dois numeros naturais com B > A+1. tomando um natural qualquer C tal que A < C < B e sempre possivel exprimir C como uma media poderada entre A e B. Seja C=A+i [A*(B-A-i)+ B*i]/(B-A)=[(A+i)(B-A)]/(B-A)=A+i=C A primeira razao e uma media ponderada entre A e B. POR ESSA RAZAO EU DISSE QUE ERA POSSIVEL TRABALHAR A DEFINICAO HABITUAL DE CONVEXIDADE DE FORMA A APARECER A MEDIA PONDERADA, ou seja, EU FALEI EXATAMENTE O QUE VOCE OBSERVOU. A ideia imediata de quem olha uma primeira vez e trabalhar com : Existe p e q tais que p+q=T-R e RAIZ_N(B) = (p*RAIZ_N(A) + q*RAIZ_N(C))/(p+q) ? Mas e isso justamente o que estamos pressupondo que acontece e que queremos mostrar que conduz a uma contradicao, respeitadas as condicoes do problema. Entao, vamos admitir isso e trabalhar com as propriedadeS de Y=RAIZ_N(X). Essa foi a proposta de trabalho. Eu vou pensar um pouco mais sobre a questao e depois escrevo. Um abracao Paulo Santa Rita 5,1156,230502 >From: "Eduardo Casagrande Stabel" >Reply-To: obm-l@mat.puc-rio.br >To: >Subject: [obm-l] Re: [obm-l] Re: [obm-l] Re: [obm-l] Re: [obm-l] Re: >[obm-l] Correçao:Apelo: Mais da Iberoamericana(questao pessoal) >Date: Thu, 23 May 2002 07:51:21 -0300 >Caro amigo Paulo, >Mas repare que nenhum desses tres pontos pertence ao gráfico da função >contínua f(x) = R_n(x). Os pontos que pertencem ao gráfico são os >seguintes: >(a, R_n(a)), (b, R_n(b)) e (c, R_n(c)) >Ou ainda >(a, X + Y*R), (b, X + Y*S) e (c, X + Y*T) >E esses três pontos não precisam estar sobre uma reta. Por que os a, b e c >não pertencem a uma mesma progressão aritmética, pelo menos isso você não >demonstrou. _________________________________________________________________ O MSN Photos é o modo mais fácil de compartilhar e imprimir suas fotos: http://photos.msn.com/support/worldwide.aspx ========================================================================= Instruções para entrar na lista, sair da lista e usar a lista em http://www.mat.puc-rio.br/~nicolau/olimp/obm-l.html O administrador desta lista é ========================================================================= From owner-obm-l@sucuri.mat.puc-rio.br Thu May 23 13:32:34 2002 Return-Path: Received: (from majordom@localhost) by sucuri.mat.puc-rio.br (8.9.3/8.9.3) id NAA15716 for obm-l-list; Thu, 23 May 2002 13:29:41 -0300 Received: from matinhos.terra.com.br (matinhos.terra.com.br [200.176.3.21]) by sucuri.mat.puc-rio.br (8.9.3/8.9.3) with ESMTP id NAA15711 for ; Thu, 23 May 2002 13:29:38 -0300 Received: from pacuiba.terra.com.br (pacuiba.terra.com.br [200.176.3.40]) by matinhos.terra.com.br (Postfix) with ESMTP id B565A47043 for ; Thu, 23 May 2002 16:17:47 +0000 (GMT) Received: from stabel (dl-nas3-poa-C89A070E.p001.terra.com.br [200.154.7.14]) (authenticated user dudasta) by pacuiba.terra.com.br (Postfix) with ESMTP id 0B6018035 for ; Thu, 23 May 2002 13:17:46 -0300 (EST) Message-ID: <002a01c20275$5eae0fd0$0e079ac8@stabel> From: "Eduardo Casagrande Stabel" To: References: Subject: [obm-l] =?iso-8859-1?Q?Re:_=5Bobm-l=5D_Re:_=5Bobm-l=5D_Re:_=5Bobm-l=5D_Re:_=5Bobm?= =?iso-8859-1?Q?-l=5D_Re:_=5Bobm-l=5D_Re:_=5Bobm-l=5D_Re:_=5Bobm-l=5D_?= =?iso-8859-1?Q?Corre=E7ao:Apelo:_Mais_da_Iberoamericana=28questao_pesso?= =?iso-8859-1?Q?al=29?= Date: Thu, 23 May 2002 13:16:33 -0300 MIME-Version: 1.0 Content-Type: text/plain; charset="iso-8859-1" Content-Transfer-Encoding: 8bit X-Priority: 3 X-MSMail-Priority: Normal X-Mailer: Microsoft Outlook Express 6.00.2600.0000 X-MimeOLE: Produced By Microsoft MimeOLE V6.00.2600.0000 Sender: owner-obm-l@sucuri.mat.puc-rio.br Precedence: bulk Reply-To: obm-l@mat.puc-rio.br From: "Paulo Santa Rita" > Mas e isso justamente o que estamos pressupondo que acontece e que queremos > mostrar que conduz a uma contradicao, respeitadas as condicoes do problema. > Entao, vamos admitir isso e trabalhar com as propriedadeS de Y=RAIZ_N(X). > Essa foi a proposta de trabalho. > > Eu vou pensar um pouco mais sobre a questao e depois escrevo. > > Um abracao > Paulo Santa Rita > 5,1156,230502 Caro amigo Paulo, eu nao acordo todas as manhãs, torcendo o meu bigode, e maquinando para tentar destruir as tuas demonstrações. Eu apenas tinha achado, precipitadamente, que voce tinha achado que ja tinha apresentado uma solução completa para a questão. Erro meu. Você estava apresentando uma idéia que poderia levar a uma solução. Mas sei que você deve entender perfeitamente o meu mal entendido. Um abraço! Eduardo Casagrande Stabel. Porto Alegre, RS. PS. eu não possuo bigode. PS2. acho que essa mensagem bate o recorde de Re's concecutivos da lista :) Só para não ficar completamente sem matemática, vai aí uma questão: como funciona a intuição matemática? Por que a mente de muitas pessoas conseguem enunciar conjecturas complicadas sem saber demonstrá-las? De onde vem essa matemática fantasma? ========================================================================= Instruções para entrar na lista, sair da lista e usar a lista em http://www.mat.puc-rio.br/~nicolau/olimp/obm-l.html O administrador desta lista é ========================================================================= From owner-obm-l@sucuri.mat.puc-rio.br Thu May 23 14:43:36 2002 Return-Path: Received: (from majordom@localhost) by sucuri.mat.puc-rio.br (8.9.3/8.9.3) id OAA17203 for obm-l-list; Thu, 23 May 2002 14:41:19 -0300 Received: from hotmail.com (law2-f4.hotmail.com [216.32.181.4]) by sucuri.mat.puc-rio.br (8.9.3/8.9.3) with ESMTP id OAA17199 for ; Thu, 23 May 2002 14:41:13 -0300 Received: from mail pickup service by hotmail.com with Microsoft SMTPSVC; Thu, 23 May 2002 10:29:20 -0700 Received: from 200.229.244.100 by lw2fd.hotmail.msn.com with HTTP; Thu, 23 May 2002 17:29:19 GMT X-Originating-IP: [200.229.244.100] From: "Paulo Santa Rita" To: obm-l@mat.puc-rio.br Subject: [obm-l] =?iso-8859-1?B?UmU6IFtvYm0tbF0gUmU6IFtvYm0tbF0gUmU6IFtvYm0tbF0gUmU6IFtv?= =?iso-8859-1?B?Ym0tbF0gUmU6IFtvYm0tbF0gUmU6IFtvYm0tbF0gUmU6IFtvYm0tbF0g?= =?iso-8859-1?B?UmU6IFtvYm0tbF0gQ29ycmXnYW86QXBlbG86IE1haXMgZGEgSWJlcm9h?= =?iso-8859-1?B?bWVyaWNhbmEocXVlc3RhbyBwZXNzb2FsKQ==?= Date: Thu, 23 May 2002 17:29:19 +0000 Mime-Version: 1.0 Content-Type: text/plain; charset=iso-8859-1; format=flowed Message-ID: X-OriginalArrivalTime: 23 May 2002 17:29:20.0304 (UTC) FILETIME=[5F50FB00:01C2027F] Sender: owner-obm-l@sucuri.mat.puc-rio.br Precedence: bulk Reply-To: obm-l@mat.puc-rio.br Querido Duda ... Querido nao, que isso e coisa de boiola ! Caro Duda, Mas de forma alguma eu ficaria chateado com voce ou com qualquer outra pessoa que porventura mostrasse uma falha ou varias falhas em meus raciocinios, pois se ate os Grandes Prof's desta lista falham e comentem erros, quanto mais eu, um simples estudante ainda lutando para aprender alguma coisa... Em verdade, so nao cometem erros e nao tem duvidas DEUS E OS IMBECIS. Como nao sou uma coisa e nem outra, eu cometi, cometo e cometerei muitos erros : e gostam de mim aqueles que me alertam para que eu me corrija ! Francamente, nao sou castelinho de areia ou estrelinha de papel que a qualquer toque (critica) se desmancha e fica emburradinho no canto guardando rancor. Sem essas viadagens e frescuras vou procurando ser util ao ideal olimpico. Para que essa mensagem nao fique fora de escopo, apresento um problema bacaninho que vi em um cartaz : NUM TRIANGULO ABC, AB=5 e BC=6. QUAL A AREA DO TRIANGULO ABC SE O ANGULO C E MAXIMO ? OBS : O problema e de nivel medio. Portanto, nao vale usar derivadas ou qualquer outro teorema ou raciocinio do CALCULO. Um Grande abraco a Todos ! Paulo Santa Rita 5,1425,230502 PS : Po, Duda. Essa de acordar de manha, caminhando pra la e pra ca retorcendo o bigode foi genial. To rindo ate agora ! >From: "Eduardo Casagrande Stabel" >Reply-To: obm-l@mat.puc-rio.br >To: >Subject: [obm-l] Re: [obm-l] Re: [obm-l] Re: [obm-l] Re: [obm-l] Re: >[obm-l] Re: [obm-l] Re: [obm-l] Correçao:Apelo: Mais da >Iberoamericana(questao pessoal) >Date: Thu, 23 May 2002 13:16:33 -0300 > >From: "Paulo Santa Rita" > > Mas e isso justamente o que estamos pressupondo que acontece e que >queremos > > mostrar que conduz a uma contradicao, respeitadas as condicoes do >problema. > > Entao, vamos admitir isso e trabalhar com as propriedadeS de >Y=RAIZ_N(X). > > Essa foi a proposta de trabalho. > > > > Eu vou pensar um pouco mais sobre a questao e depois escrevo. > > > > Um abracao > > Paulo Santa Rita > > 5,1156,230502 > >Caro amigo Paulo, > >eu nao acordo todas as manhãs, torcendo o meu bigode, e maquinando para >tentar destruir as tuas demonstrações. Eu apenas tinha achado, >precipitadamente, que voce tinha achado que ja tinha apresentado uma >solução >completa para a questão. Erro meu. Você estava apresentando uma idéia que >poderia levar a uma solução. Mas sei que você deve entender perfeitamente o >meu mal entendido. > >Um abraço! > >Eduardo Casagrande Stabel. Porto Alegre, RS. > >PS. eu não possuo bigode. >PS2. acho que essa mensagem bate o recorde de Re's concecutivos da lista :) > >Só para não ficar completamente sem matemática, vai aí uma questão: como >funciona a intuição matemática? Por que a mente de muitas pessoas conseguem >enunciar conjecturas complicadas sem saber demonstrá-las? De onde vem essa >matemática fantasma? > > >========================================================================= >Instruções para entrar na lista, sair da lista e usar a lista em >http://www.mat.puc-rio.br/~nicolau/olimp/obm-l.html >O administrador desta lista é >========================================================================= _________________________________________________________________ Una-se ao maior serviço de email do mundo: o MSN Hotmail. http://www.hotmail.com ========================================================================= Instruções para entrar na lista, sair da lista e usar a lista em http://www.mat.puc-rio.br/~nicolau/olimp/obm-l.html O administrador desta lista é ========================================================================= From owner-obm-l@sucuri.mat.puc-rio.br Thu May 23 14:54:09 2002 Return-Path: Received: (from majordom@localhost) by sucuri.mat.puc-rio.br (8.9.3/8.9.3) id OAA17399 for obm-l-list; Thu, 23 May 2002 14:52:44 -0300 Received: from www.zipmail.com.br (smtp.zipmail.com.br [200.187.242.10]) by sucuri.mat.puc-rio.br (8.9.3/8.9.3) with ESMTP id OAA17395 for ; Thu, 23 May 2002 14:52:41 -0300 From: peterdirichlet@zipmail.com.br Received: from [200.206.103.3] by www.zipmail.com.br with HTTP; Thu, 23 May 2002 14:40:43 -0300 Message-ID: <3CED26DE00000036@www.zipmail.com.br> Date: Thu, 23 May 2002 14:40:43 -0300 Subject: [obm-l] =?iso-8859-1?Q?Re=3A=5Bobm=2Dl=5DRe=3A=20=5Bobm=2Dl=5D=20Re=3A=20=5Bobm=2Dl=5D=20Re=3A=20=5Bobm=2Dl=5D=20Re=3A=20=5Bobm=2D?= =?iso-8859-1?Q?l=5D=20Re=3A=20=5Bobm=2Dl=5D=20Re=3A=20=5Bobm=2Dl=5D=20Re=3A=20=5Bobm=2Dl=5D=20Re=3A=20=5Bobm=2Dl=5D=20Corr?= =?iso-8859-1?Q?e=E7ao=3AApelo=3A=20Mais=20da=20Iberoamericana=28questao=20pessoal=29?= To: obm-l@mat.puc-rio.br MIME-Version: 1.0 Content-Type: text/plain; charset="iso-8859-1" Content-Transfer-Encoding: 8bit X-MIME-Autoconverted: from quoted-printable to 8bit by sucuri.mat.puc-rio.br id OAA17396 Sender: owner-obm-l@sucuri.mat.puc-rio.br Precedence: bulk Reply-To: obm-l@mat.puc-rio.br ANSWER:quem comecou essa discussao esta aqui agora.E dizer que tudo isso foi um simples esfrega no moral de certas pessoas que nao pensam no que escrevem...Enfim,este trocitchoz comecou numa aula do Etapa com os professores Shine e Tengan(GRAAAANDES Shine e Tengan!!!!).Eu naquela duvida com isso de uma aula de Divisibilidade,resolvi me inscrever nessa lista.So que demporei que so vendo...E ja tentei enviar essa mensagem pelo hotmail mas so dava html!!!!!E resolvi usar o Zipmail.Dai tive sucesso na de Eisenstein,enviei essa.Mas ai o Paulo resolveu de um jeito estranhissimo.O furo dessa e que como os reais sao densos nao da pra definir a divisao com perfeiçao(restos e etc.) Outra hora vou empentelhar-lhes a paciencia.... ATEEEEEEEE MAAAAAAAAAAAIS!!!!Ploft!!!Peterdirichlet -- Mensagem original -- >From: "Paulo Santa Rita" >> Mas e isso justamente o que estamos pressupondo que acontece e que >queremos >> mostrar que conduz a uma contradicao, respeitadas as condicoes do >problema. >> Entao, vamos admitir isso e trabalhar com as propriedadeS de Y=RAIZ_N(X). >> Essa foi a proposta de trabalho. >> >> Eu vou pensar um pouco mais sobre a questao e depois escrevo. >> >> Um abracao >> Paulo Santa Rita >> 5,1156,230502 > >Caro amigo Paulo, > >eu nao acordo todas as manhãs, torcendo o meu bigode, e maquinando para >tentar destruir as tuas demonstrações. Eu apenas tinha achado, >precipitadamente, que voce tinha achado que ja tinha apresentado uma solução >completa para a questão. Erro meu. Você estava apresentando uma idéia que >poderia levar a uma solução. Mas sei que você deve entender perfeitamente >o >meu mal entendido. > >Um abraço! > >Eduardo Casagrande Stabel. Porto Alegre, RS. > >PS. eu não possuo bigode. >PS2. acho que essa mensagem bate o recorde de Re's concecutivos da lista >:) > >Só para não ficar completamente sem matemática, vai aí uma questão: como >funciona a intuição matemática? Por que a mente de muitas pessoas conseguem >enunciar conjecturas complicadas sem saber demonstrá-las? De onde vem essa >matemática fantasma? > > >========================================================================= >Instruções para entrar na lista, sair da lista e usar a lista em >http://www.mat.puc-rio.br/~nicolau/olimp/obm-l.html >O administrador desta lista é >========================================================================= > TRANSIRE SVVM PECTVS MVNDOQUE POTIRE CONGREGATI EX TOTO ORBE MATHEMATICI OB SCRIPTA INSIGNIA TRIBVERE Medalha Fields(John Charles Fields) ------------------------------------------ Use o melhor sistema de busca da Internet Radar UOL - http://www.radaruol.com.br ========================================================================= Instruções para entrar na lista, sair da lista e usar a lista em http://www.mat.puc-rio.br/~nicolau/olimp/obm-l.html O administrador desta lista é ========================================================================= From owner-obm-l@sucuri.mat.puc-rio.br Thu May 23 15:00:24 2002 Return-Path: Received: (from majordom@localhost) by sucuri.mat.puc-rio.br (8.9.3/8.9.3) id OAA17577 for obm-l-list; Thu, 23 May 2002 14:58:44 -0300 Received: from www.zipmail.com.br (smtp.zipmail.com.br [200.187.242.10]) by sucuri.mat.puc-rio.br (8.9.3/8.9.3) with ESMTP id OAA16884; Thu, 23 May 2002 14:26:31 -0300 From: peterdirichlet@zipmail.com.br Received: from [200.206.103.3] by www.zipmail.com.br with HTTP; Thu, 23 May 2002 14:14:31 -0300 Message-ID: <3CECD09E00000951@www.zipmail.com.br> Date: Thu, 23 May 2002 14:14:31 -0300 Subject: [obm-l] =?iso-8859-1?Q?Uma=20solu=E7ao=20bonitinha=20da=20IMO=20da=20India=20=20=20=20=20=20=20=20=20=20=20=20=20=20=20=20=20=20?= =?iso-8859-1?Q?=20?= To: =?iso-8859-1?Q?Antonio=20Neto?= , =?iso-8859-1?Q?Lista=20de=20Discussao?= , Lista.de.Discussao" "@sucuri.mat.puc-rio.br MIME-Version: 1.0 Content-Type: text/plain; charset="iso-8859-1" Content-Transfer-Encoding: 8bit X-MIME-Autoconverted: from quoted-printable to 8bit by sucuri.mat.puc-rio.br id OAA16885 Sender: owner-obm-l@sucuri.mat.puc-rio.br Precedence: bulk Reply-To: obm-l@mat.puc-rio.br Esse problema foi considerado "O Imortal"(o menos respondido de toda a historia da IMO).apenas 2 romenos e 4 armenios resolveram-no.TODA A EQUIPE CHINESA ZEROU ESSE.IMO 1996 Problem 5 Let ABCDEF be a convex hexagon such that AB is parallel to DE, BC is parallel to EF, and CD is parallel to FA. Let RA, RC, RE denote the circumradii of triangles FAB, BCD, DEF respectively, and let p denote the perimeter of the hexagon. Prove that: RA + RC + RE >= p/2. Solution The starting point is the formula for the circumradius R of a triangle ABC: 2R = a/sin A = b/sin B = c/sin C. [Proof: the side a subtends an angle 2A at the center, so a = 2R sin A.] This gives that 2RA = BF/sin A, 2RC = BD/sin C, 2RE = FD/sin E. It is clearly not true in general that BF/sin A > BA + AF, although it is true if angle FAB >= 120, so we need some argument that involves the hexagon as a whole. Extend sides BC and FE and take lines perpendicular to them through A and D, thus forming a rectangle. Then BF is greater than or equal to the side through A and the side through D. We may find the length of the side through A by taking the projections of BA and AF giving AB sin B + AF sin F. Similarly the side through D is CD sin C + DE sin E. Hence: 2BF >= AB sin B + AF sin F + CD sin C + DE sin E. Similarly: 2BD >= BC sin B + CD sin D + AF sin A + EF sin E, and 2FD >= AB sin A + BC sin C + DE sin D + EF sin F. Hence 2BF/sin A + 2BD/sin C + 2FD/sin E >= AB(sin A/sin E + sin B/sin A) + BC(sin B/sin C + sin C/sin E) + CD(sin C/sin A + sin D/sin C) + DE(sin E/sin A + sin D/sin E) + EF(sin E/sin C + sin F/sin E) + AF(sin F/sin A + sin A/sin C). We now use the fact that opposite sides are parallel, which implies that opposite angles are equal: A = E, B = E, C = F. Each of the factors multiplying the sides in the last expression now has the form x + 1/x which has minimum value 2 when x = 1. Hence 2(BF/sin A + BD/sin C + FD/sin E) >= 2p and the result is proved. Essa soluçao e a oficial.A mais bonita e a de Ciprian Manolescu,o unico Perfect Score da prova.Ele usou a famosa Desigualdade de Erdös-Mordell.Depois eu envio a resposta dele.ATEEEEEEEEE.Peterdirichlet TRANSIRE SVVM PECTVS MVNDOQUE POTIRE CONGREGATI EX TOTO ORBE MATHEMATICI OB SCRIPTA INSIGNIA TRIBVERE Medalha Fields(John Charles Fields) ------------------------------------------ Use o melhor sistema de busca da Internet Radar UOL - http://www.radaruol.com.br ========================================================================= Instruções para entrar na lista, sair da lista e usar a lista em http://www.mat.puc-rio.br/~nicolau/olimp/obm-l.html O administrador desta lista é ========================================================================= From owner-obm-l@sucuri.mat.puc-rio.br Thu May 23 16:28:58 2002 Return-Path: Received: (from majordom@localhost) by sucuri.mat.puc-rio.br (8.9.3/8.9.3) id QAA20030 for obm-l-list; Thu, 23 May 2002 16:27:41 -0300 Received: from hotmail.com (law2-f36.hotmail.com [216.32.181.36]) by sucuri.mat.puc-rio.br (8.9.3/8.9.3) with ESMTP id QAA20023 for ; Thu, 23 May 2002 16:27:35 -0300 Received: from mail pickup service by hotmail.com with Microsoft SMTPSVC; Thu, 23 May 2002 12:15:39 -0700 Received: from 32.94.119.254 by lw2fd.hotmail.msn.com with HTTP; Thu, 23 May 2002 19:15:39 GMT X-Originating-IP: [32.94.119.254] From: "Paulo Santa Rita" To: obm-l@mat.puc-rio.br Subject: [obm-l] A Intuicao Matematica Date: Thu, 23 May 2002 19:15:39 +0000 Mime-Version: 1.0 Content-Type: text/plain; charset=iso-8859-1; format=flowed Message-ID: X-OriginalArrivalTime: 23 May 2002 19:15:39.0421 (UTC) FILETIME=[3990FCD0:01C2028E] Sender: owner-obm-l@sucuri.mat.puc-rio.br Precedence: bulk Reply-To: obm-l@mat.puc-rio.br Ola Duda e demais colegas desta lista, Eu nao sei responder a pergunta que voce fez abaixo, mas acredito que a intuicao matematica esta para a mente de um matematico assim como a intuicao sensivel esta para a mente humana ... Quando voce olha para um objeto voce IMEDIATAMENTE deduz varias propriedades SEM USAR NENHUM RACIOCINIO MEDIADOR. Por exemplo, voce tem uma IDEIA INSTANTANEA ( INTUICAO SENSIVEL ) do tamanho, da cor, da forma, da proximidade dele para com outros objetos, etc. Todos esses conhecimentos sao validos e foram obtidos por INTUICAO SENSIVEL 9PERCEPCAO INSTANTANEA ), isto e, sem que fosse necessario usar um RACIOCINIO LOGICO OU DISCURSSIVO para se chegar a eles. Me parece que a intuicao matematica se aproxima disso. Voce "olha" ( com o olhar da mente matematica ) os objetos do mundo matematico E SENTE que eles devem ter ou manter determinadas relacoes, sem que voce consiga, de imediato, forjar uma demonstracao para estas relacoes ou propriedades. Veja como Gauss fala em dois momentos : 1) "Encontrei um maravilhoso teorema, mas, infelizmente, ainda nao consigo demonstrar" Ele fala sobre a lei da reciprocidade quadratica. Veja bem. Ele estava convencido da correcao do teorema ANTES DE DEMONSTRA-LO. E comum voce aprender em algumas escolas o seguinte : Voce so pode ter certeza de um teorema depois de demonstra-lo ! 2) "Durante este outono, preocupei-me largamente COM AS CONSIDERACOES GERAIS sobre as superficies curvas, o que conduz a um campo ilimitado ... Essas pesquisas se ligam fortemente com a metafisica da geometria e nao e sem ingentes esforcos que consigo me arracar das consequencias que dai advem. Qual seria o verdadeiro sentido da raiz quadrada de -1 ? Nestes momentos, sinto vibrar vivamente em mim o verdadeiro significado destas coisas mas creio que sera terrivelmente dificil exprimir este significado em palavras" Ve-se aqui que Gauss PRIMEIRO, SENTE A REALIDADE DO MUNDO MATEMATICO. E SO DEPOIS ele vai exprimir o que sente atraves das teorias e formulas matematicas. Esses exemplos reforcam o que eu suspeito. A verdadeira inteligencia esta na SENSIBILIDADE ou CAPACIDADE DE VIVENCIAR INTERNAMENTE E EMOTIVAMENTE os objetos matematicos. Logica, demonstracao e teorias sao coisas que vem depois. Primeiro voce pensa, vale dizer, vivenciar internamente e emotivamente, atraves da imaginacao, os objetos e fatos matematicos, depois, quanto mais viva for essa vivencia, mais profundas serao suas conclusoes e mais facil ficara a posterior e necessaria demonstacao. Ainda a esse respeito e interessante destacar que, conforme relata Voltaire, uma vez perguntaram a Newton como ele havia descoberto a lei de gravitacao. Newton respondeu : pensando continuamente sobre ela ! Quer dizer, imaginando e vivenciando os objetos voce desvela os misterios que os encombrem ... Os exemplos acima mostram que e fundamental que o lado emocional esteja presente : se o cara nao conseguir apreciar a beleza destas coisas, nao tiver, por alguma razao, entusiasmo por elas, dificilmente ele vai trocar os prazeres imediatos e efemeros da vida por elas ... Como ultimo exemplo para reforcar minha tese, cito um exemplo contemporaneo. Ha poucos meses atras eu assisti uma exposicao de um Matematico da area de equacoes diferenciais. Em verdade, de um Grande Matematico, com varios premios internacionais e conhecido no mundo inteiro. Ele explicava a ideia de DECOMPOSICAO FOCAL. Seja dada uma equacao diferencial F(X, DX, ...)=0. A cada ponto (X,Y) do plano ele associava a quantidade de solucoes desta equacao que passam por aquele ponto. Isso implicava em dividir o plano em regioes : O conjunto de pontos onde nao passam solucoes e a regiao R0, o conjunto de pontos onde ha uma unica solucao e a regiao R1, o conjunto de pontos onde ha duas solucoes e a Regiao R2 e dai sucessivamente. Dai o termo DECOMPOSICAO : O R^2 fica "decomposto" na uniao disjunta R0 U R1 U R2 U R3 .... O termo "Focal" e porque ele partia, para facilitar, sempre do valor inicial Dx(0)=0. Este era o "foco". Equacoes simples tem decomposicao focal simples (poucas regioes), equacoes mais complexas tem uma decomposicao mais complexa. Este matematico foi a Princeton falar com o Smale. Smale disse que esta ideia se encaixava como uma luva com o problema de Feynam das trajetorias. Em certo ponto apareceram algumas perguntas complicadas ... Mas o Ilustre Matematico respondeu que nao se poderia se perder em detalhes ... A Beleza e aplicacoes daquelas ideias eram o maior sustentaculo para se continuar a pesquisa ! Era incrivel a emocao e o entusiasmo que o Ilustre Prof Demonstrava enquanto expunha as suas ideias, mesmo sendo uma pessoa ja de provecta idade( possivelmente > 70 anos ). Um Sr verdadeiramente Genial ! Eu conclui duas coisas : 1) Que a decomposicao focal tambem pode ser usada para classificar as equacoes diferencias ! 2) Pesquisa e EMOCAO PURA ! E ENTUSIASMO ! E INTUICAO ! E MAGICA ! e isso nao depende de idade. E esse emocional que gera a intuicao ! Um abraco Paulo Santa Rita 5,1608,230502 EM TEMPO : Ia esquecendo. O nome do Genial Matematico : Mauricio Matos Peixoto. Pesquisador Emerito do IMPA. >From: "Eduardo Casagrande Stabel" >Reply-To: obm-l@mat.puc-rio.br >To: >Subject: [obm-l] Re: [obm-l] Re: [obm-l] Re: [obm-l] Re: [obm-l] Re: >[obm-l] Re: [obm-l] Re: [obm-l] Correçao:Apelo: Mais da >Iberoamericana(questao pessoal) >Date: Thu, 23 May 2002 13:16:33 -0300 >Só para não ficar completamente sem matemática, vai aí uma questão: >como >funciona a intuição matemática? Por que a mente de muitas pessoas > >conseguem enunciar conjecturas complicadas sem saber demonstrá-las? De > >onde vem essa matemática fantasma ? _________________________________________________________________ O MSN Photos é o modo mais fácil de compartilhar e imprimir suas fotos: http://photos.msn.com/support/worldwide.aspx ========================================================================= Instruções para entrar na lista, sair da lista e usar a lista em http://www.mat.puc-rio.br/~nicolau/olimp/obm-l.html O administrador desta lista é ========================================================================= From owner-obm-l@sucuri.mat.puc-rio.br Thu May 23 18:29:02 2002 Return-Path: Received: (from majordom@localhost) by sucuri.mat.puc-rio.br (8.9.3/8.9.3) id SAA22393 for obm-l-list; Thu, 23 May 2002 18:28:24 -0300 Received: from fgvrj23.fgv.br (fgvrj23.fgv.br [200.20.164.23]) by sucuri.mat.puc-rio.br (8.9.3/8.9.3) with ESMTP id SAA22389 for ; Thu, 23 May 2002 18:28:22 -0300 Received: by FGVRJ23 with Internet Mail Service (5.5.2653.19) id <20VA7B86>; Thu, 23 May 2002 18:21:13 -0300 Message-ID: <765A72978645D4118B1C0000E229806D05B6B3A6@FGVRJ23> From: Ralph Teixeira To: "'obm-l@mat.puc-rio.br'" Subject: [obm-l] Raizes cubicas em P.A. Date: Thu, 23 May 2002 18:21:12 -0300 MIME-Version: 1.0 X-Mailer: Internet Mail Service (5.5.2653.19) Content-Type: text/plain; charset="iso-8859-1" Content-Transfer-Encoding: 8bit X-MIME-Autoconverted: from quoted-printable to 8bit by sucuri.mat.puc-rio.br id SAA22390 Sender: owner-obm-l@sucuri.mat.puc-rio.br Precedence: bulk Reply-To: obm-l@mat.puc-rio.br Problema: Mostre que, se a, b e c são primos entre si (não todos cubos perfeitos), então suas raízes cúbicas não estão em P.A. Solução: Suponha que, de fato, que as raizes cubicas (vou chama-las de x, y e z repectivamente) estao em P.A.: 2y=x+z Entao: 8y^3=x^3+z^3+3xz(x+z)=x^3+z^3+3xz(2y) (8b-a-c)/6 = xyz [(8b-a-c)/6]^3=abc Como o lado esquerdo é um "racional ao cubo" e o lado direito é um inteiro, concluímos que ambos são um cubo perfeito. Como a,b e c são primos entre si e abc é cubo perfeito, cada um deles (a,b e c) tem de ser cubo perfeito, contradizendo o enunciado. Abraço, Ralph Que tal? ========================================================================= Instruções para entrar na lista, sair da lista e usar a lista em http://www.mat.puc-rio.br/~nicolau/olimp/obm-l.html O administrador desta lista é ========================================================================= From owner-obm-l@sucuri.mat.puc-rio.br Thu May 23 19:34:48 2002 Return-Path: Received: (from majordom@localhost) by sucuri.mat.puc-rio.br (8.9.3/8.9.3) id TAA23400 for obm-l-list; Thu, 23 May 2002 19:34:21 -0300 Received: from hotmail.com (law2-f147.hotmail.com [216.32.181.147]) by sucuri.mat.puc-rio.br (8.9.3/8.9.3) with ESMTP id TAA23396 for ; Thu, 23 May 2002 19:34:18 -0300 Received: from mail pickup service by hotmail.com with Microsoft SMTPSVC; Thu, 23 May 2002 15:22:27 -0700 Received: from 200.229.244.100 by lw2fd.hotmail.msn.com with HTTP; Thu, 23 May 2002 22:22:27 GMT X-Originating-IP: [200.229.244.100] From: "Paulo Santa Rita" To: obm-l@mat.puc-rio.br Subject: Re: [obm-l] Raizes cubicas em P.A. Date: Thu, 23 May 2002 22:22:27 +0000 Mime-Version: 1.0 Content-Type: text/plain; charset=iso-8859-1; format=flowed Message-ID: X-OriginalArrivalTime: 23 May 2002 22:22:27.0931 (UTC) FILETIME=[525BE2B0:01C202A8] Sender: owner-obm-l@sucuri.mat.puc-rio.br Precedence: bulk Reply-To: obm-l@mat.puc-rio.br Ola Prof Ralph, > Que tal? Em minha opiniao, muito bom ! As restricoes em relacao ao problema original que nos estavamos discutindo sao : 1) No problema original as raizes sao N-esimas. O Sr restringiu a raizes cubicas. 2) No problemas original as raizes N-esimas devem ser termos de uma mesma PA. O Sr restringiu a TERMOS CONSECUTIVOS de uma mesma PA. Bom, mas e evidentemente melhor dar pequenos passos viaveis, um por vez, que querer dar um grande passo inviavel de uma vez ... Que tal considerar agora o problema : > Problema: Mostre que, se a, b e c são primos entre si (não todos >cubos perfeitos), então suas raízes cúbicas não estão em UMA P.A. Um Abracao Paulo Santa Rita 5,1916,230502 >From: Ralph Teixeira >Reply-To: obm-l@mat.puc-rio.br >To: "'obm-l@mat.puc-rio.br'" >Subject: [obm-l] Raizes cubicas em P.A. >Date: Thu, 23 May 2002 18:21:12 -0300 > > > Problema: Mostre que, se a, b e c são primos entre si (não todos >cubos perfeitos), então suas raízes cúbicas não estão em P.A. > > Solução: Suponha que, de fato, que as raizes cubicas (vou chama-las >de x, y e z repectivamente) estao em P.A.: > >2y=x+z > > Entao: > >8y^3=x^3+z^3+3xz(x+z)=x^3+z^3+3xz(2y) >(8b-a-c)/6 = xyz >[(8b-a-c)/6]^3=abc > > Como o lado esquerdo é um "racional ao cubo" e o lado direito é um >inteiro, concluímos que ambos são um cubo perfeito. Como a,b e c são primos >entre si e abc é cubo perfeito, cada um deles (a,b e c) tem de ser cubo >perfeito, contradizendo o enunciado. > > Abraço, > Ralph > > Que tal? > >========================================================================= >Instruções para entrar na lista, sair da lista e usar a lista em >http://www.mat.puc-rio.br/~nicolau/olimp/obm-l.html >O administrador desta lista é >========================================================================= _________________________________________________________________ Chegou o novo MSN Explorer. Instale já. É gratuito: http://explorer.msn.com.br ========================================================================= Instruções para entrar na lista, sair da lista e usar a lista em http://www.mat.puc-rio.br/~nicolau/olimp/obm-l.html O administrador desta lista é ========================================================================= From owner-obm-l@sucuri.mat.puc-rio.br Thu May 23 23:46:59 2002 Return-Path: Received: (from majordom@localhost) by sucuri.mat.puc-rio.br (8.9.3/8.9.3) id XAA25632 for obm-l-list; Thu, 23 May 2002 23:46:04 -0300 Received: from web13303.mail.yahoo.com (web13303.mail.yahoo.com [216.136.175.39]) by sucuri.mat.puc-rio.br (8.9.3/8.9.3) with SMTP id XAA25627 for ; Thu, 23 May 2002 23:46:01 -0300 Message-ID: <20020524023410.65027.qmail@web13303.mail.yahoo.com> Received: from [200.226.38.54] by web13303.mail.yahoo.com via HTTP; Thu, 23 May 2002 19:34:10 PDT Date: Thu, 23 May 2002 19:34:10 -0700 (PDT) From: Carlos Yuzo Shine Subject: Re: [obm-l] Raizes cubicas (e n-ésimas) em P.A. To: obm-l@mat.puc-rio.br In-Reply-To: <765A72978645D4118B1C0000E229806D05B6B3A6@FGVRJ23> MIME-Version: 1.0 Content-Type: text/plain; charset=us-ascii Sender: owner-obm-l@sucuri.mat.puc-rio.br Precedence: bulk Reply-To: obm-l@mat.puc-rio.br A solução do Ralph pode ser perfeitamente adaptada para o problema de haver três raízes cúbicas em uma PA (não necessariamente três termos consecutivos). Vou, inclusive, aproveitar o email dele: --- Ralph Teixeira wrote: > Problema: Mostre que, se a, b e c são primos entre si (não todos cubos perfeitos), então suas raízes cúbicas não estão em P.A. > Solução: Suponha que, de fato, que as raizes cubicas (vou chama-las de x, y e z repectivamente) estao em P.A.: > 2y=x+z ---> Aqui, em vez de 2y = x + z, considere y = qx + rz, onde q e r são racionais (se x, y e z estão em PA, então existem esses racionais). Agora, elevando ao cubo: y^3=q^3x^3+r^3z^3+3qxrz(ax+bz)=q^3x^3+r^3z^3+3qrxzy <=> b = q^3*a + r^3*c + 3qrxyz <=> [(b - q^3*a - r^3*c)/(3qr)]^3=abc Repetindo o argumento do Ralph: > Como o lado esquerdo é um "racional ao cubo" e o lado direito é um inteiro, concluímos que ambos são um cubo perfeito. Como a,b e c são primos entre si e abc é cubo perfeito, cada um deles (a,b e c) tem de ser cubo perfeito, contradizendo o enunciado. ---> Agora, vou tentar (não sei se vai dar certo) para o caso das raízes n-ésimas: vou usar a mesma notação acima, sendo x^n = a, y^n = b e z^n = c, a, b e c primos dois a dois, nenhum deles n-ésima potência perfeita. Suponha, por absurdo, que existem racionais q e r tais que y = qx + rz <=> 1 = q(x/y) + r(z/y) (*) Sejam S = q(x/y) + r(z/y) e P = q(x/y)*r(z/y) = qr(xz/y^2). Veja que x/y e z/y são raízes n-ésimas de racionais e P também, pois x, y e z são primos dois a dois. Elevando a n, obtemos um polinômio em S e P, com coeficientes racionais. Como sei disso? Sejam t = q(x/y) e u = r(z/y). Podemos calcular S(k) = t^k + u^k, k natural, da seguinte forma: |t^2 = St - P => |t^(m+2) = St^(m+1) - Pt^m |u^2 = Su - P |u^(m+2) = Su^(m+1) - Pu^m => (t^(m+2)+u^(m+2)) = S(t^(m+1)+u^(m+1)) - P(t^m+u^m) <=> S(m+2) = S*S(m+1) - P*S(m) Como S(0) = 2 e S(1) = S, indutivamente temos que S(n) é um polinômio em S e P, com coeficientes inteiros. Mas S(n) = [q(x/y)]^n + [r(z/y)]^n = q^n(a/b) + r^n(c/b) é racional. Logo ao elevarmos os dois lados de (*) a n obtemos uma equação em S e P. Substituindo S = 1, temos uma equação polinomial em P, de grau no máximo n/2 (Veja que se k > n/2, P^k = (tu)^k seria o produto de k t's e k u's, totalizando 2k > n fatores, o que não é possível pois há no máximo n fatores). Mas P = qr(xz/y^2) é um número algébrico de grau n (a, b e c são primos dois a dois), contradição. Não sei se o que fiz está certo... Talvez eu esteja viajando forte, agora são 20 para a meia-noite e estou bem cansado... []'s Shine __________________________________________________ Do You Yahoo!? LAUNCH - Your Yahoo! Music Experience http://launch.yahoo.com ========================================================================= Instruções para entrar na lista, sair da lista e usar a lista em http://www.mat.puc-rio.br/~nicolau/olimp/obm-l.html O administrador desta lista é ========================================================================= From owner-obm-l@sucuri.mat.puc-rio.br Fri May 24 12:01:33 2002 Return-Path: Received: (from majordom@localhost) by sucuri.mat.puc-rio.br (8.9.3/8.9.3) id MAA32087 for obm-l-list; Fri, 24 May 2002 12:00:10 -0300 Received: from web21108.mail.yahoo.com (web21108.mail.yahoo.com [216.136.227.110]) by sucuri.mat.puc-rio.br (8.9.3/8.9.3) with SMTP id MAA32083 for ; Fri, 24 May 2002 12:00:08 -0300 Message-ID: <20020524144818.94921.qmail@web21108.mail.yahoo.com> Received: from [150.161.2.4] by web21108.mail.yahoo.com via HTTP; Fri, 24 May 2002 11:48:18 ART Date: Fri, 24 May 2002 11:48:18 -0300 (ART) From: =?iso-8859-1?q?Carlos=20Ma=E7aranduba?= Subject: Re: [obm-l] A Intuicao Matematica To: obm-l@mat.puc-rio.br In-Reply-To: MIME-Version: 1.0 Content-Type: text/plain; charset=iso-8859-1 Content-Transfer-Encoding: 8bit Sender: owner-obm-l@sucuri.mat.puc-rio.br Precedence: bulk Reply-To: obm-l@mat.puc-rio.br Segundo leibnitz(que desenvolveu em paralelo a newtow o calculo integral e diferencial) existem verdades que são inatas ao espírito(o individuo já nasce com elas, ver "Novos Ensaios sobre o Entendimento Humano" de sua autoria ) o que explicaria porque a lógica "é lógica" e porque individuos soubessem da verdade de certas conjecturas sem prova-lás.Concordo com Paulo quando fala do emocional , ele é uam das peças chaves do sentir e vibrar com certas inferencias e conclusoes(aliás em lógica existe uma técnica interessante chamada dedução natural que tenta imitar as regras de inferencia do homem).Essas questões são interessantes também no âmbito da inteligencia artificial.Criariamos máquinas com emoções e intuições???Qual lógica trataria de tal coisa???Uma das limitações da lógica é imposta pelo Teorema da incompletude de Godel , máquinas nao seriam capazes de lidar com isso mas nós humanos compreendemos e até brincamos com estas loucuras(a lembrar o paradoxo de Russel do barbeiro) que me leva a conjecturar(que lógica é essa?) que somos muito mais que meras máquinas computacionais . Eu não posso ser provado. --- Paulo Santa Rita escreveu: > Ola Duda e demais > colegas desta lista, > > Eu nao sei responder a pergunta que voce fez abaixo, > mas acredito que a > intuicao matematica esta para a mente de um > matematico assim como a intuicao > sensivel esta para a mente humana ... > > Quando voce olha para um objeto voce IMEDIATAMENTE > deduz varias propriedades > SEM USAR NENHUM RACIOCINIO MEDIADOR. Por exemplo, > voce tem uma IDEIA > INSTANTANEA ( INTUICAO SENSIVEL ) do tamanho, da > cor, da forma, da > proximidade dele para com outros objetos, etc. Todos > esses conhecimentos sao > validos e foram obtidos por INTUICAO SENSIVEL > 9PERCEPCAO INSTANTANEA ), isto > e, sem que fosse necessario usar um RACIOCINIO > LOGICO OU DISCURSSIVO para se > chegar a eles. > > Me parece que a intuicao matematica se aproxima > disso. Voce "olha" ( com o > olhar da mente matematica ) os objetos do mundo > matematico E SENTE que eles > devem ter ou manter determinadas relacoes, sem que > voce consiga, de > imediato, forjar uma demonstracao para estas > relacoes ou propriedades. > > Veja como Gauss fala em dois momentos : > > 1) "Encontrei um maravilhoso teorema, mas, > infelizmente, ainda nao consigo > demonstrar" > > Ele fala sobre a lei da reciprocidade quadratica. > Veja bem. Ele estava > convencido da correcao do teorema ANTES DE > DEMONSTRA-LO. E comum voce > aprender em algumas escolas o seguinte : Voce so > pode ter certeza de um > teorema depois de demonstra-lo ! > > 2) "Durante este outono, preocupei-me largamente COM > AS CONSIDERACOES GERAIS > sobre as superficies curvas, o que conduz a um campo > ilimitado ... Essas > pesquisas se ligam fortemente com a metafisica da > geometria e nao e sem > ingentes esforcos que consigo me arracar das > consequencias que dai advem. > Qual seria o verdadeiro sentido da raiz quadrada de > -1 ? Nestes momentos, > sinto vibrar vivamente em mim o verdadeiro > significado destas coisas mas > creio que sera terrivelmente dificil exprimir este > significado em palavras" > > Ve-se aqui que Gauss PRIMEIRO, SENTE A REALIDADE DO > MUNDO MATEMATICO. E SO > DEPOIS ele vai exprimir o que sente atraves das > teorias e formulas > matematicas. > > Esses exemplos reforcam o que eu suspeito. A > verdadeira inteligencia esta na > SENSIBILIDADE ou CAPACIDADE DE VIVENCIAR > INTERNAMENTE E EMOTIVAMENTE os > objetos matematicos. Logica, demonstracao e teorias > sao coisas que vem > depois. Primeiro voce pensa, vale dizer, vivenciar > internamente e > emotivamente, atraves da imaginacao, os objetos e > fatos matematicos, depois, > quanto mais viva for essa vivencia, mais profundas > serao suas conclusoes e > mais facil ficara a posterior e necessaria > demonstacao. > > Ainda a esse respeito e interessante destacar que, > conforme relata Voltaire, > uma vez perguntaram a Newton como ele havia > descoberto a lei de gravitacao. > Newton respondeu : pensando continuamente sobre ela > ! > > Quer dizer, imaginando e vivenciando os objetos voce > desvela os misterios > que os encombrem ... Os exemplos acima mostram que e > fundamental que o lado > emocional esteja presente : se o cara nao conseguir > apreciar a beleza destas > coisas, nao tiver, por alguma razao, entusiasmo por > elas, dificilmente ele > vai trocar os prazeres imediatos e efemeros da vida > por elas ... > > Como ultimo exemplo para reforcar minha tese, cito > um exemplo contemporaneo. > > Ha poucos meses atras eu assisti uma exposicao de um > Matematico > da area de equacoes diferenciais. Em verdade, de um > Grande Matematico, com > varios premios internacionais e conhecido no mundo > inteiro. > > Ele explicava a ideia de DECOMPOSICAO FOCAL. Seja > dada uma equacao > diferencial F(X, DX, ...)=0. A cada ponto (X,Y) do > plano ele associava a > quantidade de solucoes desta equacao que passam por > aquele ponto. Isso > implicava em dividir o plano em regioes : O conjunto > de pontos onde nao > passam solucoes e a regiao R0, o conjunto de pontos > onde ha uma unica > solucao e a regiao R1, o conjunto de pontos onde ha > duas solucoes e a Regiao > R2 e dai sucessivamente. Dai o termo DECOMPOSICAO : > O R^2 fica "decomposto" > na uniao disjunta R0 U R1 U R2 U R3 .... O termo > "Focal" e porque ele > partia, para facilitar, sempre do valor inicial > Dx(0)=0. Este era o "foco". > > Equacoes simples tem decomposicao focal simples > (poucas regioes), equacoes > mais complexas tem uma decomposicao mais complexa. > > Este matematico foi a Princeton falar com o Smale. > Smale disse que esta > ideia se encaixava como uma luva com o problema de > Feynam das trajetorias. > Em certo ponto apareceram algumas perguntas > complicadas ... Mas o Ilustre > Matematico respondeu que nao se poderia se perder em > detalhes ... A Beleza e > aplicacoes daquelas ideias eram o maior sustentaculo > para se continuar a > pesquisa ! > > Era incrivel a emocao e o entusiasmo que o Ilustre > Prof Demonstrava enquanto > expunha as suas ideias, mesmo sendo uma pessoa ja de > provecta idade( > possivelmente > 70 anos ). Um Sr verdadeiramente > Genial ! Eu conclui duas > coisas : > > 1) Que a decomposicao focal tambem pode ser usada > para classificar as > equacoes diferencias ! > 2) Pesquisa e EMOCAO PURA ! E ENTUSIASMO ! E > INTUICAO ! E MAGICA ! e isso > nao depende de idade. > > E esse emocional que gera a intuicao ! > > Um abraco > Paulo Santa Rita > 5,1608,230502 > > EM TEMPO : Ia esquecendo. O nome do Genial > Matematico : Mauricio Matos > Peixoto. Pesquisador Emerito do IMPA. > > > > > > > > >From: "Eduardo Casagrande Stabel" > > >Reply-To: obm-l@mat.puc-rio.br > >To: > >Subject: [obm-l] Re: [obm-l] Re: [obm-l] Re: > [obm-l] Re: [obm-l] Re: > >[obm-l] Re: [obm-l] Re: [obm-l] Correçao:Apelo: > Mais da > >Iberoamericana(questao pessoal) > >Date: Thu, 23 May 2002 13:16:33 -0300 > > >Só para não ficar completamente sem matemática, vai > aí uma questão: >como > >funciona a intuição matemática? Por que a mente de > muitas pessoas > > >conseguem enunciar conjecturas complicadas sem > saber demonstrá-las? De > > >onde vem essa matemática fantasma ? > > > > _________________________________________________________________ > O MSN Photos é o modo mais fácil de compartilhar e > imprimir suas fotos: > http://photos.msn.com/support/worldwide.aspx > === message truncated === _______________________________________________________________________ Yahoo! Encontros O lugar certo para você encontrar aquela pessoa que falta na sua vida. Cadastre-se hoje mesmo! http://br.encontros.yahoo.com/ ========================================================================= Instruções para entrar na lista, sair da lista e usar a lista em http://www.mat.puc-rio.br/~nicolau/olimp/obm-l.html O administrador desta lista é ========================================================================= From owner-obm-l@sucuri.mat.puc-rio.br Fri May 24 12:07:55 2002 Return-Path: Received: (from majordom@localhost) by sucuri.mat.puc-rio.br (8.9.3/8.9.3) id MAA32219 for obm-l-list; Fri, 24 May 2002 12:07:47 -0300 Received: from smtp018.mail.yahoo.com (smtp018.mail.yahoo.com [216.136.174.115]) by sucuri.mat.puc-rio.br (8.9.3/8.9.3) with SMTP id MAA32215 for ; Fri, 24 May 2002 12:07:44 -0300 Received: from shasta213250.ig.com.br (HELO delta) (maurelio1234@200.151.213.250 with login) by smtp.mail.vip.sc5.yahoo.com with SMTP; 24 May 2002 14:55:53 -0000 Message-ID: <135901c20332$d6778380$fad597c8@delta> From: =?iso-8859-1?Q?Marcos_Aur=E9lio_Almeida_da_Silva?= To: References: Subject: [obm-l] Re: Date: Fri, 24 May 2002 11:38:00 -0300 MIME-Version: 1.0 Content-Type: text/plain; charset="iso-8859-1" Content-Transfer-Encoding: 8bit X-Priority: 3 X-MSMail-Priority: Normal X-Mailer: Microsoft Outlook Express 5.50.4133.2400 X-MimeOLE: Produced By Microsoft MimeOLE V5.50.4133.2400 Sender: owner-obm-l@sucuri.mat.puc-rio.br Precedence: bulk Reply-To: obm-l@mat.puc-rio.br Supondo R o raio da Terra o comprimento inicial da corda seria: C0 =2 pi R Como o comprimento aumentou 1 m: C = 2 pi R + 1 o novo raio (R') da corda seria 2 pi R' = 2 pi R + 1 R' = R + 1/2 * 1/pi ou seja: R' = R + 0,16m a distância seria de 16cm. Você deve ter estranhado o fato de que esse valor independe do diâmetro do objeto "laçado". ----- Original Message ----- From: "Adherbal Rocha Filho" To: Sent: Sunday, May 19, 2002 8:51 AM > > mais um probleminha: > suponha q a Terra eh uma esfera e que uma corda está amarrada ao redor da > linha do equador.Agora suponha que esta corda eh aumentada em um metro > ,formando uma circunferencia maior,qual será a distancia entre a superficie > da Terra e a corda? E se eu fizesse o mesmo pra uma bola de futebol,qual > seria a distancia? > > valeu! > []´s > Adherbal > > > _________________________________________________________________ > Envie e receba emails com o Hotmail no seu dispositivo móvel: > http://mobile.msn.com > > ========================================================================= > Instruções para entrar na lista, sair da lista e usar a lista em > http://www.mat.puc-rio.br/~nicolau/olimp/obm-l.html > O administrador desta lista é > ========================================================================= _________________________________________________________ Do You Yahoo!? Get your free @yahoo.com address at http://mail.yahoo.com ========================================================================= Instruções para entrar na lista, sair da lista e usar a lista em http://www.mat.puc-rio.br/~nicolau/olimp/obm-l.html O administrador desta lista é ========================================================================= From owner-obm-l@sucuri.mat.puc-rio.br Fri May 24 12:27:34 2002 Return-Path: Received: (from majordom@localhost) by sucuri.mat.puc-rio.br (8.9.3/8.9.3) id MAA00438 for obm-l-list; Fri, 24 May 2002 12:27:11 -0300 Received: from smtp-5.ig.com.br (smtp-5.ig.com.br [200.226.132.154]) by sucuri.mat.puc-rio.br (8.9.3/8.9.3) with SMTP id MAA00434 for ; Fri, 24 May 2002 12:27:07 -0300 Received: (qmail 25969 invoked from network); 24 May 2002 15:15:03 -0000 Received: from 249.151.225.200.in-addr.arpa.ig.com.br (HELO oemcomputer) (200.225.151.249) by smtp-5.ig.com.br with SMTP; 24 May 2002 15:15:03 -0000 Message-ID: <003401c20338$13e19d00$f997e1c8@oemcomputer> From: "Daniel" To: References: <3CECD09E00000951@www.zipmail.com.br> Subject: [obm-l] =?iso-8859-1?Q?Re:_=5Bobm-l=5DUma_solu=E7ao_bonitinha_da_IMO_da_India____?= =?iso-8859-1?Q?_______________?= Date: Fri, 24 May 2002 12:31:28 -0300 MIME-Version: 1.0 Content-Type: text/plain; charset="iso-8859-1" Content-Transfer-Encoding: 7bit X-Priority: 3 X-MSMail-Priority: Normal X-Mailer: Microsoft Outlook Express 5.00.2615.200 X-MIMEOLE: Produced By Microsoft MimeOLE V5.00.2615.200 Sender: owner-obm-l@sucuri.mat.puc-rio.br Precedence: bulk Reply-To: obm-l@mat.puc-rio.br TRANSIRE SVVM PECTVS MVNDOQUE POTIRE > CONGREGATI EX TOTO ORBE MATHEMATICI OB SCRIPTA INSIGNIA TRIBVERE > Medalha Fields(John Charles Fields) O que significa essa frase? Daniel ========================================================================= Instruções para entrar na lista, sair da lista e usar a lista em http://www.mat.puc-rio.br/~nicolau/olimp/obm-l.html O administrador desta lista é ========================================================================= From owner-obm-l@sucuri.mat.puc-rio.br Fri May 24 13:02:07 2002 Return-Path: Received: (from majordom@localhost) by sucuri.mat.puc-rio.br (8.9.3/8.9.3) id NAA01493 for obm-l-list; Fri, 24 May 2002 13:01:42 -0300 Received: from hotmail.com (f31.law9.hotmail.com [64.4.9.31]) by sucuri.mat.puc-rio.br (8.9.3/8.9.3) with ESMTP id NAA01487 for ; Fri, 24 May 2002 13:01:39 -0300 Received: from mail pickup service by hotmail.com with Microsoft SMTPSVC; Fri, 24 May 2002 08:49:50 -0700 Received: from 200.190.10.32 by lw9fd.law9.hotmail.msn.com with HTTP; Fri, 24 May 2002 15:49:50 GMT X-Originating-IP: [200.190.10.32] From: "Rogerio Fajardo" To: obm-l@mat.puc-rio.br Subject: [obm-l] Re: Date: Fri, 24 May 2002 15:49:50 +0000 Mime-Version: 1.0 Content-Type: text/plain; format=flowed Message-ID: X-OriginalArrivalTime: 24 May 2002 15:49:50.0645 (UTC) FILETIME=[A3890E50:01C2033A] Sender: owner-obm-l@sucuri.mat.puc-rio.br Precedence: bulk Reply-To: obm-l@mat.puc-rio.br Sendo r o raio da Terra (ou da bola), o comprimento da terra é de 2pi*r. Aumentando em 1, o comprimento passa a ser 2pi*r+1. O raio dessa corda passa a ser (2pi*r+1)/2pi. Logo, a diferença dos raios é de r+1/2pi-r=1/2pi, independente se r é o raio da Terra ou da bola! >From: "Adherbal Rocha Filho" >Reply-To: obm-l@mat.puc-rio.br >To: obm-l@mat.puc-rio.br >Date: Sun, 19 May 2002 11:51:23 +0000 > > > mais um probleminha: >suponha q a Terra eh uma esfera e que uma corda está amarrada ao redor da >linha do equador.Agora suponha que esta corda eh aumentada em um metro >,formando uma circunferencia maior,qual será a distancia entre a superficie >da Terra e a corda? E se eu fizesse o mesmo pra uma bola de futebol,qual >seria a distancia? > >valeu! >[]´s >Adherbal > > >_________________________________________________________________ >Envie e receba emails com o Hotmail no seu dispositivo móvel: >http://mobile.msn.com > >========================================================================= >Instruções para entrar na lista, sair da lista e usar a lista em >http://www.mat.puc-rio.br/~nicolau/olimp/obm-l.html >O administrador desta lista é >========================================================================= _________________________________________________________________ Chat with friends online, try MSN Messenger: http://messenger.msn.com ========================================================================= Instruções para entrar na lista, sair da lista e usar a lista em http://www.mat.puc-rio.br/~nicolau/olimp/obm-l.html O administrador desta lista é ========================================================================= From owner-obm-l@sucuri.mat.puc-rio.br Fri May 24 15:15:24 2002 Return-Path: Received: (from majordom@localhost) by sucuri.mat.puc-rio.br (8.9.3/8.9.3) id PAA03892 for obm-l-list; Fri, 24 May 2002 15:13:46 -0300 Received: from www.zipmail.com.br (smtp.zipmail.com.br [200.187.242.10]) by sucuri.mat.puc-rio.br (8.9.3/8.9.3) with ESMTP id PAA03888 for ; Fri, 24 May 2002 15:13:42 -0300 From: peterdirichlet@zipmail.com.br Received: from [200.206.103.3] by www.zipmail.com.br with HTTP; Fri, 24 May 2002 15:01:53 -0300 Message-ID: <3CEE6B3F0000031A@www.zipmail.com.br> Date: Fri, 24 May 2002 15:01:53 -0300 In-Reply-To: <003401c20338$13e19d00$f997e1c8@oemcomputer> Subject: [obm-l] =?iso-8859-1?Q?Re=3A=20=5Bobm=2Dl=5D=20Re=3A=20=5Bobm=2Dl=5DUma=20solu=E7ao=20bonitinha=20da=20IMO=20da=20?= =?iso-8859-1?Q?India=20=20=20=20=20=20=20=20=20=20=20=20=20=20=20=20=20=20=20?= To: obm-l@mat.puc-rio.br MIME-Version: 1.0 Content-Type: text/plain; charset="iso-8859-1" Content-Transfer-Encoding: 8bit X-MIME-Autoconverted: from quoted-printable to 8bit by sucuri.mat.puc-rio.br id PAA03889 Sender: owner-obm-l@sucuri.mat.puc-rio.br Precedence: bulk Reply-To: obm-l@mat.puc-rio.br "Superar as proprias limitaçoes e dominar o universo Matematicos de todo o mundo reunidos prestam homenagem por obras notaveis".E uma inscriçao da Medalha Fields(de ouro maciço,acho).Na frente tem uma efigie de Arquimedes,seu nome em grego,e a primeira frase; atras tem uma esfera inscrita num cilindro,um ramo de uma certa planta(acho que e louro),e a segunda frase.E ai,gostou? TRANSIRE SVVM PECTVS MVNDOQUE POTIRE CONGREGATI EX TOTO ORBE MATHEMATICI OB SCRIPTA INSIGNIA TRIBVERE Medalha Fields(John Charles Fields) ------------------------------------------ Use o melhor sistema de busca da Internet Radar UOL - http://www.radaruol.com.br ========================================================================= Instruções para entrar na lista, sair da lista e usar a lista em http://www.mat.puc-rio.br/~nicolau/olimp/obm-l.html O administrador desta lista é ========================================================================= From owner-obm-l@sucuri.mat.puc-rio.br Fri May 24 16:25:01 2002 Return-Path: Received: (from majordom@localhost) by sucuri.mat.puc-rio.br (8.9.3/8.9.3) id QAA05030 for obm-l-list; Fri, 24 May 2002 16:23:49 -0300 Received: from www.zipmail.com.br (smtp.zipmail.com.br [200.187.242.10]) by sucuri.mat.puc-rio.br (8.9.3/8.9.3) with ESMTP id QAA05026 for ; Fri, 24 May 2002 16:23:46 -0300 From: peterdirichlet@zipmail.com.br Received: from [200.206.103.3] by www.zipmail.com.br with HTTP; Fri, 24 May 2002 16:11:57 -0300 Message-ID: <3CEE6B3F000005AA@www.zipmail.com.br> Date: Fri, 24 May 2002 16:11:57 -0300 In-Reply-To: <3CECD09E00000951@www.zipmail.com.br> Subject: [obm-l] =?iso-8859-1?Q?Re=3A=20=5Bobm=2Dl=5D=20Uma=20solu=E7ao=20bonitinha=20da=20IMO=20da=20India=20=20=20=20=20=20?= =?iso-8859-1?Q?=20=20=20=20=20=20=20=20=20=20=20=20=20?= To: obm-l@mat.puc-rio.br MIME-Version: 1.0 Content-Type: text/plain; charset="iso-8859-1" Content-Transfer-Encoding: 8bit X-MIME-Autoconverted: from quoted-printable to 8bit by sucuri.mat.puc-rio.br id QAA05027 Sender: owner-obm-l@sucuri.mat.puc-rio.br Precedence: bulk Reply-To: obm-l@mat.puc-rio.br PRIMEIRA PARTE:Para aplicar Erdös-Mordell neste problema,e bom que ponhamos tudo para dentro!Traçando os paralelogramos DEFM,NFAB,PBCD,e o triangulo XYZ tal que ZYe perpendicular a BP,XZ a DM e XY a FN,da para reescrever a desigualdade.Prove que XM+YN+ZP>=BN+BP+DP+DM+FM+FN. Essa e a proxima missao.ATEEEEEEEEEEE!!!!!Ploft!Peterdirichlet -- Mensagem original -- >Esse problema foi considerado "O Imortal"(o menos respondido de toda a historia >da IMO).apenas 2 romenos e 4 armenios resolveram-no.TODA A EQUIPE CHINESA >ZEROU ESSE.IMO 1996 > > > >Problem 5 > >Let ABCDEF be a convex hexagon such that AB is parallel to DE, BC is parallel >to EF, and CD is parallel to FA. Let RA, RC, RE denote the circumradii of >triangles FAB, BCD, DEF respectively, and let p denote the perimeter of >the hexagon. Prove that: > > RA + RC + RE >= p/2. > > > >Solution > > >The starting point is the formula for the circumradius R of a triangle ABC: >2R = a/sin A = b/sin B = c/sin C. [Proof: the side a subtends an angle 2A >at the center, so a = 2R sin A.] This gives that 2RA = BF/sin A, 2RC = BD/sin >C, 2RE = FD/sin E. It is clearly not true in general that BF/sin A > BA >+ AF, although it is true if angle FAB >= 120, so we need some argument >that involves the hexagon as a whole. > >Extend sides BC and FE and take lines perpendicular to them through A and >D, thus forming a rectangle. Then BF is greater than or equal to the side >through A and the side through D. We may find the length of the side through >A by taking the projections of BA and AF giving AB sin B + AF sin F. Similarly >the side through D is CD sin C + DE sin E. Hence: > > 2BF >= AB sin B + AF sin F + CD sin C + DE sin E. Similarly: > > 2BD >= BC sin B + CD sin D + AF sin A + EF sin E, and > > 2FD >= AB sin A + BC sin C + DE sin D + EF sin F. > >Hence 2BF/sin A + 2BD/sin C + 2FD/sin E >= AB(sin A/sin E + sin B/sin A) >+ BC(sin B/sin C + sin C/sin E) + CD(sin C/sin A + sin D/sin C) + DE(sin >E/sin A + sin D/sin E) + EF(sin E/sin C + sin F/sin E) + AF(sin F/sin A >+ sin A/sin C). > >We now use the fact that opposite sides are parallel, which implies that >opposite angles are equal: A = E, B = E, C = F. Each of the factors multiplying >the sides in the last expression now has the form x + 1/x which has minimum >value 2 when x = 1. Hence 2(BF/sin A + BD/sin C + FD/sin E) >= 2p and the >result is proved. > >Essa soluçao e a oficial.A mais bonita e a de Ciprian Manolescu,o unico > Perfect Score da prova.Ele usou a famosa Desigualdade de Erdös-Mordell.Depois >eu envio a resposta dele.ATEEEEEEEEE.Peterdirichlet > > > > > > >TRANSIRE SVVM PECTVS MVNDOQUE POTIRE >CONGREGATI EX TOTO ORBE MATHEMATICI OB SCRIPTA INSIGNIA TRIBVERE >Medalha Fields(John Charles Fields) > > >------------------------------------------ >Use o melhor sistema de busca da Internet >Radar UOL - http://www.radaruol.com.br > > > >========================================================================= >Instruções para entrar na lista, sair da lista e usar a lista em >http://www.mat.puc-rio.br/~nicolau/olimp/obm-l.html >O administrador desta lista é >========================================================================= > TRANSIRE SVVM PECTVS MVNDOQUE POTIRE CONGREGATI EX TOTO ORBE MATHEMATICI OB SCRIPTA INSIGNIA TRIBVERE Medalha Fields(John Charles Fields) ------------------------------------------ Use o melhor sistema de busca da Internet Radar UOL - http://www.radaruol.com.br ========================================================================= Instruções para entrar na lista, sair da lista e usar a lista em http://www.mat.puc-rio.br/~nicolau/olimp/obm-l.html O administrador desta lista é ========================================================================= From owner-obm-l@sucuri.mat.puc-rio.br Fri May 24 16:47:33 2002 Return-Path: Received: (from majordom@localhost) by sucuri.mat.puc-rio.br (8.9.3/8.9.3) id QAA05516 for obm-l-list; Fri, 24 May 2002 16:47:28 -0300 Received: from www.zipmail.com.br (smtp.zipmail.com.br [200.187.242.10]) by sucuri.mat.puc-rio.br (8.9.3/8.9.3) with ESMTP id QAA05512 for ; Fri, 24 May 2002 16:47:26 -0300 From: peterdirichlet@zipmail.com.br Received: from [200.206.103.3] by www.zipmail.com.br with HTTP; Fri, 24 May 2002 16:35:37 -0300 Message-ID: <3CEE6B3F0000067F@www.zipmail.com.br> Date: Fri, 24 May 2002 16:35:37 -0300 In-Reply-To: Subject: [obm-l] =?iso-8859-1?Q?Re=3A=20=5Bobm=2Dl=5D=20Re=3A=20=5Bobm=2Dl=5D=20Re=3A=20=5Bobm=2Dl=5D=20Re=3A=20=5Bobm=2Dl=5D=20Re=3A=20=5Bob?= =?iso-8859-1?Q?m=2Dl=5D=20Re=3A=20=5Bobm=2Dl=5D=20Re=3A=20=5Bobm=2Dl=5D=20Re=3A=20=5Bobm=2Dl=5D=20Re=3A=20=5Bobm=2Dl=5D=20Co?= =?iso-8859-1?Q?rre=E7ao=3AApelo=3A=20Mais=20da=20Iberoamericana=28questao=20pessoal=29?= To: obm-l@mat.puc-rio.br MIME-Version: 1.0 Content-Type: text/plain; charset="iso-8859-1" Content-Transfer-Encoding: 8bit X-MIME-Autoconverted: from quoted-printable to 8bit by sucuri.mat.puc-rio.br id QAA05513 Sender: owner-obm-l@sucuri.mat.puc-rio.br Precedence: bulk Reply-To: obm-l@mat.puc-rio.br NOOOOSSAAAAA!!!Ta irritado hoje!?!?!!!!?!?!?!?!?!?!?!?!?!?!?!? De onde voce tirou tanta raiva acumulada? Resposta do trekitchoz:o angulo maximo deve ser 90 graus.Ai o treco ja fica legal. -- Mensagem original -- >Querido Duda ... >Querido nao, que isso e coisa de boiola ! > >Caro Duda, > >Mas de forma alguma eu ficaria chateado com voce ou com qualquer outra >pessoa que porventura mostrasse uma falha ou varias falhas em meus >raciocinios, pois se ate os Grandes Prof's desta lista falham e comentem > >erros, quanto mais eu, um simples estudante ainda lutando para aprender >alguma coisa... > >Em verdade, so nao cometem erros e nao tem duvidas DEUS E OS IMBECIS. Como > >nao sou uma coisa e nem outra, eu cometi, cometo e cometerei muitos erros >: >e gostam de mim aqueles que me alertam para que eu me corrija ! Francamente, > >nao sou castelinho de areia ou estrelinha de papel que a qualquer toque >(critica) se desmancha e fica emburradinho no canto guardando rancor. Sem > >essas viadagens e frescuras vou procurando ser util ao ideal olimpico. > >Para que essa mensagem nao fique fora de escopo, apresento um problema >bacaninho que vi em um cartaz : > >NUM TRIANGULO ABC, AB=5 e BC=6. QUAL A AREA DO TRIANGULO ABC SE O ANGULO >C E >MAXIMO ? > >OBS : O problema e de nivel medio. Portanto, nao vale usar derivadas ou >qualquer outro teorema ou raciocinio do CALCULO. > >Um Grande abraco a Todos ! >Paulo Santa Rita >5,1425,230502 > >PS : Po, Duda. Essa de acordar de manha, caminhando pra la e pra ca >retorcendo o bigode foi genial. To rindo ate agora ! > > >>From: "Eduardo Casagrande Stabel" >>Reply-To: obm-l@mat.puc-rio.br >>To: >>Subject: [obm-l] Re: [obm-l] Re: [obm-l] Re: [obm-l] Re: [obm-l] Re: >>[obm-l] Re: [obm-l] Re: [obm-l] Correçao:Apelo: Mais da >>Iberoamericana(questao pessoal) >>Date: Thu, 23 May 2002 13:16:33 -0300 >> >>From: "Paulo Santa Rita" >> > Mas e isso justamente o que estamos pressupondo que acontece e que >>queremos >> > mostrar que conduz a uma contradicao, respeitadas as condicoes do >>problema. >> > Entao, vamos admitir isso e trabalhar com as propriedadeS de >>Y=RAIZ_N(X). >> > Essa foi a proposta de trabalho. >> > >> > Eu vou pensar um pouco mais sobre a questao e depois escrevo. >> > >> > Um abracao >> > Paulo Santa Rita >> > 5,1156,230502 >> >>Caro amigo Paulo, >> >>eu nao acordo todas as manhãs, torcendo o meu bigode, e maquinando para >>tentar destruir as tuas demonstrações. Eu apenas tinha achado, >>precipitadamente, que voce tinha achado que ja tinha apresentado uma >>solução >>completa para a questão. Erro meu. Você estava apresentando uma idéia que >>poderia levar a uma solução. Mas sei que você deve entender perfeitamente >o >>meu mal entendido. >> >>Um abraço! >> >>Eduardo Casagrande Stabel. Porto Alegre, RS. >> >>PS. eu não possuo bigode. >>PS2. acho que essa mensagem bate o recorde de Re's concecutivos da lista >:) >> >>Só para não ficar completamente sem matemática, vai aí uma questão: como >>funciona a intuição matemática? Por que a mente de muitas pessoas conseguem >>enunciar conjecturas complicadas sem saber demonstrá-las? De onde vem essa >>matemática fantasma? >> >> >>========================================================================= >>Instruções para entrar na lista, sair da lista e usar a lista em >>http://www.mat.puc-rio.br/~nicolau/olimp/obm-l.html >>O administrador desta lista é >>========================================================================= > > > > >_________________________________________________________________ >Una-se ao maior serviço de email do mundo: o MSN Hotmail. >http://www.hotmail.com > >========================================================================= >Instruções para entrar na lista, sair da lista e usar a lista em >http://www.mat.puc-rio.br/~nicolau/olimp/obm-l.html >O administrador desta lista é >========================================================================= > TRANSIRE SVVM PECTVS MVNDOQUE POTIRE CONGREGATI EX TOTO ORBE MATHEMATICI OB SCRIPTA INSIGNIA TRIBVERE Medalha Fields(John Charles Fields) ------------------------------------------ Use o melhor sistema de busca da Internet Radar UOL - http://www.radaruol.com.br ========================================================================= Instruções para entrar na lista, sair da lista e usar a lista em http://www.mat.puc-rio.br/~nicolau/olimp/obm-l.html O administrador desta lista é ========================================================================= From owner-obm-l@sucuri.mat.puc-rio.br Sat May 25 12:44:14 2002 Return-Path: Received: (from majordom@localhost) by sucuri.mat.puc-rio.br (8.9.3/8.9.3) id MAA13023 for obm-l-list; Sat, 25 May 2002 12:42:16 -0300 Received: from www.zipmail.com.br (smtp.zipmail.com.br [200.187.242.10]) by sucuri.mat.puc-rio.br (8.9.3/8.9.3) with ESMTP id MAA13019 for ; Sat, 25 May 2002 12:42:14 -0300 From: peterdirichlet@zipmail.com.br Received: from [200.206.103.3] by www.zipmail.com.br with HTTP; Sat, 25 May 2002 12:30:28 -0300 Message-ID: <3CEE9B8600001323@www.zipmail.com.br> Date: Sat, 25 May 2002 12:30:28 -0300 In-Reply-To: <3CEE6B3F000005AA@www.zipmail.com.br> Subject: [obm-l] =?iso-8859-1?Q?=20Uma=20solu=E7ao=20bonitinha=20da=20IMO=20da=20India=28de=20novo=29=20=20=20=20=20=20=20=20?= =?iso-8859-1?Q?=20=20=20=20=20=20=20=20=20=20=20?= To: obm-l@mat.puc-rio.br MIME-Version: 1.0 Content-Type: text/plain; charset="iso-8859-1" Content-Transfer-Encoding: 8bit X-MIME-Autoconverted: from quoted-printable to 8bit by sucuri.mat.puc-rio.br id MAA13020 Sender: owner-obm-l@sucuri.mat.puc-rio.br Precedence: bulk Reply-To: obm-l@mat.puc-rio.br Prova do lema:veja que os triangulos DEF e DMF sao semelhantes,logo tem o mesmo circunraio.Mas XM e o diametro do triangulo DMF.Logo XM=2*RA.E reformule a desigualdade. Vamos quebrar isso em dois casos: 1)M,N e P coincidem.E ai e so usar Erdös-Mordell em XYZ. 2)MNP e um triangulo.Depois eu continuo. ATEEEEEEE!!!!!!loft1Peterdirichlet. -- Mensagem original -- >PRIMEIRA PARTE:Para aplicar Erdös-Mordell neste problema,e bom que ponhamos >tudo para dentro!Traçando os paralelogramos DEFM,NFAB,PBCD,e o triangulo >XYZ tal que ZYe perpendicular a BP,XZ a DM e XY a FN,da para reescrever >a desigualdade.Prove que >XM+YN+ZP>=BN+BP+DP+DM+FM+FN. >Essa e a proxima missao.ATEEEEEEEEEEE!!!!!Ploft!Peterdirichlet > >-- Mensagem original -- > >>Esse problema foi considerado "O Imortal"(o menos respondido de toda a >historia >>da IMO).apenas 2 romenos e 4 armenios resolveram-no.TODA A EQUIPE CHINESA >>ZEROU ESSE.IMO 1996 >> >> >> >>Problem 5 >> >>Let ABCDEF be a convex hexagon such that AB is parallel to DE, BC is parallel >>to EF, and CD is parallel to FA. Let RA, RC, RE denote the circumradii >of >>triangles FAB, BCD, DEF respectively, and let p denote the perimeter of >>the hexagon. Prove that: >> >> RA + RC + RE >= p/2. >> >> >> >>Solution >> >> >>The starting point is the formula for the circumradius R of a triangle >ABC: >>2R = a/sin A = b/sin B = c/sin C. [Proof: the side a subtends an angle >2A >>at the center, so a = 2R sin A.] This gives that 2RA = BF/sin A, 2RC = >BD/sin >>C, 2RE = FD/sin E. It is clearly not true in general that BF/sin A > BA >>+ AF, although it is true if angle FAB >= 120, so we need some argument >>that involves the hexagon as a whole. >> >>Extend sides BC and FE and take lines perpendicular to them through A and >>D, thus forming a rectangle. Then BF is greater than or equal to the side >>through A and the side through D. We may find the length of the side through >>A by taking the projections of BA and AF giving AB sin B + AF sin F. Similarly >>the side through D is CD sin C + DE sin E. Hence: >> >> 2BF >= AB sin B + AF sin F + CD sin C + DE sin E. Similarly: >> >> 2BD >= BC sin B + CD sin D + AF sin A + EF sin E, and >> >> 2FD >= AB sin A + BC sin C + DE sin D + EF sin F. >> >>Hence 2BF/sin A + 2BD/sin C + 2FD/sin E >= AB(sin A/sin E + sin B/sin A) >>+ BC(sin B/sin C + sin C/sin E) + CD(sin C/sin A + sin D/sin C) + DE(sin >>E/sin A + sin D/sin E) + EF(sin E/sin C + sin F/sin E) + AF(sin F/sin A >>+ sin A/sin C). >> >>We now use the fact that opposite sides are parallel, which implies that >>opposite angles are equal: A = E, B = E, C = F. Each of the factors multiplying >>the sides in the last expression now has the form x + 1/x which has minimum >>value 2 when x = 1. Hence 2(BF/sin A + BD/sin C + FD/sin E) >= 2p and the >>result is proved. >> >>Essa soluçao e a oficial.A mais bonita e a de Ciprian Manolescu,o unico >> Perfect Score da prova.Ele usou a famosa Desigualdade de Erdös-Mordell.Depois >>eu envio a resposta dele.ATEEEEEEEEE.Peterdirichlet >> TRANSIRE SVVM PECTVS MVNDOQUE POTIRE CONGREGATI EX TOTO ORBE MATHEMATICI OB SCRIPTA INSIGNIA TRIBVERE Medalha Fields(John Charles Fields) ------------------------------------------ Use o melhor sistema de busca da Internet Radar UOL - http://www.radaruol.com.br ========================================================================= Instruções para entrar na lista, sair da lista e usar a lista em http://www.mat.puc-rio.br/~nicolau/olimp/obm-l.html O administrador desta lista é ========================================================================= From owner-obm-l@sucuri.mat.puc-rio.br Sat May 25 18:51:03 2002 Return-Path: Received: (from majordom@localhost) by sucuri.mat.puc-rio.br (8.9.3/8.9.3) id SAA15470 for obm-l-list; Sat, 25 May 2002 18:49:10 -0300 Received: from web13707.mail.yahoo.com (web13707.mail.yahoo.com [216.136.175.140]) by sucuri.mat.puc-rio.br (8.9.3/8.9.3) with SMTP id SAA15466 for ; Sat, 25 May 2002 18:49:08 -0300 Message-ID: <20020525213723.70767.qmail@web13707.mail.yahoo.com> Received: from [200.213.88.151] by web13707.mail.yahoo.com via HTTP; Sat, 25 May 2002 18:37:23 ART Date: Sat, 25 May 2002 18:37:23 -0300 (ART) From: =?iso-8859-1?q?pichurin?= Subject: [obm-l] Óptica(offtopic) To: obm-l@mat.puc-rio.br MIME-Version: 1.0 Content-Type: text/plain; charset=iso-8859-1 Content-Transfer-Encoding: 8bit Sender: owner-obm-l@sucuri.mat.puc-rio.br Precedence: bulk Reply-To: obm-l@mat.puc-rio.br Uma lente convergente e uma outra, divergente, ambas com 250mm de distância focal, estão à distância de 75mm, com os eixos óptocos coincidentes. Determine a natureza e a posição de um objeto colocado 357mm à frente da lente convergente. _______________________________________________________________________ Yahoo! Encontros O lugar certo para você encontrar aquela pessoa que falta na sua vida. Cadastre-se hoje mesmo! http://br.encontros.yahoo.com/ ========================================================================= Instruções para entrar na lista, sair da lista e usar a lista em http://www.mat.puc-rio.br/~nicolau/olimp/obm-l.html O administrador desta lista é ========================================================================= From owner-obm-l@sucuri.mat.puc-rio.br Sun May 26 12:48:47 2002 Return-Path: Received: (from majordom@localhost) by sucuri.mat.puc-rio.br (8.9.3/8.9.3) id MAA23002 for obm-l-list; Sun, 26 May 2002 12:46:28 -0300 Received: from www.zipmail.com.br (smtp.zipmail.com.br [200.187.242.10]) by sucuri.mat.puc-rio.br (8.9.3/8.9.3) with ESMTP id MAA22998 for ; Sun, 26 May 2002 12:46:26 -0300 From: bmat@zipmail.com.br Received: from [200.220.5.22] by www.zipmail.com.br with HTTP; Sun, 26 May 2002 12:34:44 -0300 Message-ID: <3CF0407700000BE2@www.zipmail.com.br> Date: Sun, 26 May 2002 12:34:44 -0300 In-Reply-To: <20020525213723.70767.qmail@web13707.mail.yahoo.com> Subject: [obm-l] =?iso-8859-1?Q?Derivadas?= To: obm-l@mat.puc-rio.br MIME-Version: 1.0 Content-Type: text/plain; charset="iso-8859-1" Content-Transfer-Encoding: 8bit X-MIME-Autoconverted: from quoted-printable to 8bit by sucuri.mat.puc-rio.br id MAA22999 Sender: owner-obm-l@sucuri.mat.puc-rio.br Precedence: bulk Reply-To: obm-l@mat.puc-rio.br Bom dia. Eu estava pensando sobre derivadas de funções e cheguei a uma dúvida: será que d(x*|x|)/dx = 2|x|? Como se prova isso? ( E se for, a integral de |x|dx será 1/2x*|x|? ) Existe algum método para calcular derivadas de funções mais complicadas envolvendo módulo, tais como d(ln(x + |x+1|)/dx? Agradeço qualquer esclarecimento. ------------------------------------------ Use o melhor sistema de busca da Internet Radar UOL - http://www.radaruol.com.br ========================================================================= Instruções para entrar na lista, sair da lista e usar a lista em http://www.mat.puc-rio.br/~nicolau/olimp/obm-l.html O administrador desta lista é ========================================================================= From owner-obm-l@sucuri.mat.puc-rio.br Sun May 26 16:47:48 2002 Return-Path: Received: (from majordom@localhost) by sucuri.mat.puc-rio.br (8.9.3/8.9.3) id QAA24622 for obm-l-list; Sun, 26 May 2002 16:46:25 -0300 Received: from hotmail.com (f144.pav1.hotmail.com [64.4.31.144]) by sucuri.mat.puc-rio.br (8.9.3/8.9.3) with ESMTP id QAA24614 for ; Sun, 26 May 2002 16:46:23 -0300 Received: from mail pickup service by hotmail.com with Microsoft SMTPSVC; Sun, 26 May 2002 12:34:40 -0700 Received: from 200.151.59.1 by pv1fd.pav1.hotmail.msn.com with HTTP; Sun, 26 May 2002 19:34:40 GMT X-Originating-IP: [200.151.59.1] From: "Adriano Almeida Faustino" To: obm-l@mat.puc-rio.br Subject: [obm-l] Analise Combinatoria Date: Sun, 26 May 2002 19:34:40 +0000 Mime-Version: 1.0 Content-Type: text/plain; charset=iso-8859-1; format=flowed Message-ID: X-OriginalArrivalTime: 26 May 2002 19:34:40.0705 (UTC) FILETIME=[61105B10:01C204EC] Sender: owner-obm-l@sucuri.mat.puc-rio.br Precedence: bulk Reply-To: obm-l@mat.puc-rio.br Considere uma turma com n alunos ,numerados de 1 a n. Deseja-se organizar uma comissao de 3 alunos.De quantas maneiras pode ser formada esta comissao,de modo que nao facam parte da mesma dois ou tres alunosdesignados por numeros consecutivos ? [] s Adriano. _________________________________________________________________ Una-se ao maior serviço de email do mundo: o MSN Hotmail. http://www.hotmail.com ========================================================================= Instruções para entrar na lista, sair da lista e usar a lista em http://www.mat.puc-rio.br/~nicolau/olimp/obm-l.html O administrador desta lista é ========================================================================= From owner-obm-l@sucuri.mat.puc-rio.br Sun May 26 20:03:54 2002 Return-Path: Received: (from majordom@localhost) by sucuri.mat.puc-rio.br (8.9.3/8.9.3) id UAA25958 for obm-l-list; Sun, 26 May 2002 20:01:45 -0300 Received: from home.iis.com.br (mail.iis.com.br [200.202.96.2]) by sucuri.mat.puc-rio.br (8.9.3/8.9.3) with ESMTP id UAA25954 for ; Sun, 26 May 2002 20:01:40 -0300 From: mcohen@iis.com.br Received: from localhost (services2.iis.com.br [200.202.96.26]) by home.iis.com.br (8.11.6/8.11.6/1.1.1.16) with SMTP id g4QMnw209601 for ; Sun, 26 May 2002 19:49:58 -0300 Date: Sun, 26 May 2002 19:49:58 -0300 Message-Id: <200205262249.g4QMnw209601@home.iis.com.br> To: Subject: [obm-l] Conicas e Latus Rectum X-Originating-Ip: [200.20.121.235] X-Mailer: NOCC v0.9.3pl1 Content-Type: text/plain; Content-Transfer-Encoding: 8bit X-AntiVirus: Antivirus for sendmail by Petr Rehor Sender: owner-obm-l@sucuri.mat.puc-rio.br Precedence: bulk Reply-To: obm-l@mat.puc-rio.br Como mostrar que a menor corda focal de uma elipse eh sempre perpendicular ao eixo maior? Esse problema me persegue ha bastante tempo ... qdo eu era aluno no 2o grau, li em algum lugar essa propriedade... q dentre todas as cordas da elipse passando por um determinado foco, aquela q fosse perpendicular ao eixo maior tinha comprimento minimo.. parece q existem resultados analogos para as outras conicas.. Eu nao lembro aonde li isso, mas nunca encontrei a demonstracao. e agora eu precisei demonstrar isso e nao consegui. se alguem puder ajudar, agradeco!! (tentei inicialmente colocar o comprimento da corda em funcao do angulo x com a horizontal, e depois analisar a funcao f(tan(x)).. a expressao era um pouco grande (4 fracoes racionais de grau <= 2, duas delas ao quadrado) mas nao oobtive sucesso... Obrigado, Marcio ========================================================================= Instruções para entrar na lista, sair da lista e usar a lista em http://www.mat.puc-rio.br/~nicolau/olimp/obm-l.html O administrador desta lista é ========================================================================= From owner-obm-l@sucuri.mat.puc-rio.br Sun May 26 21:56:10 2002 Return-Path: Received: (from majordom@localhost) by sucuri.mat.puc-rio.br (8.9.3/8.9.3) id VAA27050 for obm-l-list; Sun, 26 May 2002 21:54:38 -0300 Received: from smtp-3.ig.com.br (smtp-3.ig.com.br [200.226.132.152]) by sucuri.mat.puc-rio.br (8.9.3/8.9.3) with SMTP id VAA27046 for ; Sun, 26 May 2002 21:54:36 -0300 Received: (qmail 13556 invoked from network); 27 May 2002 00:42:39 -0000 Received: from shasta041037.ig.com.br (HELO araujoimeig) (200.151.41.37) by smtp-3.ig.com.br with SMTP; 27 May 2002 00:42:39 -0000 Message-ID: <002001c20517$49351f00$252997c8@com.br> From: =?iso-8859-1?B?QW5kcuk=?= To: "OBM" References: <200205262249.g4QMnw209601@home.iis.com.br> Subject: Re: [obm-l] Conicas e Latus Rectum Date: Sun, 26 May 2002 21:41:13 -0300 MIME-Version: 1.0 Content-Type: text/plain; charset="iso-8859-1" Content-Transfer-Encoding: 8bit X-Priority: 3 X-MSMail-Priority: Normal X-Mailer: Microsoft Outlook Express 5.00.2615.200 X-MimeOLE: Produced By Microsoft MimeOLE V5.00.2615.200 Sender: owner-obm-l@sucuri.mat.puc-rio.br Precedence: bulk Reply-To: obm-l@mat.puc-rio.br E ai Marcio, blz? Primeiro vamos lembrar de algumas definicoes. 1) Na elipse, o lugar dos meios das cordas paralelas a uma direcao dada delta eh um segmento de reta D passando pelo centro. Diz-se que D eh o diametro relativo a direcao delta. 2) Na elipse, se D for o diametro relativo a direcao delta, o diametro D* relativo a direcao D sera paralelo a direcao delta. Diz-se que D e D* sao dois diametros conjugados. Lema1: Numa elipse, qq diametro eh a media geometrica entre a corda focal que lhe eh paralela e o eixo maior. Ou seja: (D1D2)^2 = (M1M2) x (2*a), onde D1D2 eh diametro qq e M1M2 corda focal paralela a D1D2. Prova: Exercicio para vc. Lema2: Numa elipse, a soma dos quadrados de dois semidiametros conjugados eh constante e igual a soma dos quadrados dos semi eixos. Ou seja: [(D1D2)/2]^2 + [(E1E2)/2]^2 = a^2 + b^2, onde D1D2 e E1E2 sao diametros conjugados. Prova: Exercicio para vc. Teorema: Numa elipse, a soma de duas cordas focais, paralelas a dois diametros conjugados, eh constante. Prova: Sejam dois diametros conjugados D1D2 e E1E2 e as cordas focais M1M2, P1P2 q lhe sao paralelas. Do lema 1, temos que: (D1D2)^2 = (M1M2) x (2*a) (E1E2)^2 = (P1P2) x (2*a) Somando as expressoes: (M1M2 + P1P2) x (2*a) = (D1D2)^2 + (E1E2)^2, do lema 2, temos que: M1M2 + P1P2 = 2*( a + (b^2)/a) = constante. Em particular tome P1P2 igual ao eixo maior, ou seja P1P2 = 2*a (maximo) e M1M2 = 2*(b^2)/a (minimo, perpendicular a P1P2). Isto eh tudo... Andre Araujo. ----- Original Message ----- From: To: Sent: Sunday, May 26, 2002 7:49 PM Subject: [obm-l] Conicas e Latus Rectum > Como mostrar que a menor corda focal de uma elipse eh sempre perpendicular ao eixo maior? > > Esse problema me persegue ha bastante tempo ... qdo eu era aluno no 2o grau, li em algum lugar essa propriedade... q dentre todas as cordas da elipse passando por um determinado foco, aquela q fosse perpendicular ao eixo maior tinha comprimento minimo.. parece q existem resultados analogos para as outras conicas.. Eu nao lembro aonde li isso, mas nunca encontrei a demonstracao. e agora eu precisei demonstrar isso e nao consegui. se alguem puder ajudar, agradeco!! (tentei inicialmente colocar o comprimento da corda em funcao do angulo x com a horizontal, e depois analisar a funcao > f(tan(x)).. a expressao era um pouco grande (4 fracoes racionais de grau <= 2, duas delas ao quadrado) mas nao oobtive sucesso... > > Obrigado, > Marcio > > > > > ========================================================================= > Instruções para entrar na lista, sair da lista e usar a lista em > http://www.mat.puc-rio.br/~nicolau/olimp/obm-l.html > O administrador desta lista é > ========================================================================= > ========================================================================= Instruções para entrar na lista, sair da lista e usar a lista em http://www.mat.puc-rio.br/~nicolau/olimp/obm-l.html O administrador desta lista é ========================================================================= From owner-obm-l@sucuri.mat.puc-rio.br Sun May 26 22:31:08 2002 Return-Path: Received: (from majordom@localhost) by sucuri.mat.puc-rio.br (8.9.3/8.9.3) id WAA27602 for obm-l-list; Sun, 26 May 2002 22:29:49 -0300 Received: from imo-r07.mx.aol.com (imo-r07.mx.aol.com [152.163.225.103]) by sucuri.mat.puc-rio.br (8.9.3/8.9.3) with ESMTP id WAA27598 for ; Sun, 26 May 2002 22:29:47 -0300 From: SSayajinGoten@aol.com Received: from SSayajinGoten@aol.com by imo-r07.mx.aol.com (mail_out_v32.5.) id z.96.26ee802c (17528) for ; Sun, 26 May 2002 21:18:02 -0400 (EDT) Message-ID: <96.26ee802c.2a22e349@aol.com> Date: Sun, 26 May 2002 21:18:01 EDT Subject: Re: [obm-l] Analise Combinatoria To: obm-l@mat.puc-rio.br MIME-Version: 1.0 Content-Type: text/plain; charset="ISO-8859-1" X-Mailer: AOL 4.0 for Windows sub 108 Content-Transfer-Encoding: 8bit X-MIME-Autoconverted: from quoted-printable to 8bit by sucuri.mat.puc-rio.br id WAA27599 Sender: owner-obm-l@sucuri.mat.puc-rio.br Precedence: bulk Reply-To: obm-l@mat.puc-rio.br Putz, Adriano hehe, tu deve ter abrido esse mail lokinho lokinho crente crente que alguem teria resolvido seu problema, né?, pois é..eu não vim para te ajudar, e sim para te complicar! hehe , ja que tu gosta desse tipo de problemas toma mais 4 ai pro c!: Com os algarismos 1, 2, 3, 4, 5 e 6 formam-se números naturais de 6 algarismos distintos. Sabendo-se que neles não aparecem juntos, dois algarismos pares nem dois algarismos ímpares, calcular o número total de naturais assim formados? Quantos números pares, de cinco algarismos, podem ser formados sem repetir algarismos? Seis pessoas, -A, B, C, D e F - ficam em pé uma ao lado da outra para uma fotografia. Se A e B se recusam a ficar lado a lado e C e D insistem em aparecer uma ao lado da outra, determinar o número de possibilidades distintas para as seis pessoas se disporem? Para um campeonato de volêi foram convocados 10 jogadores: 4 mineiros e 6 cariocas, sendo que 3 dos mineiros são juvenis. Lembrando que um time em quadra tem 6 jogadores, determinar quantos são os times possíveis contendo, pelo menos, 1 carioca e 1 mineiro, sabendo que no time não pode haver mais do que 1 juvenil? Junior-Rio=) ========================================================================= Instruções para entrar na lista, sair da lista e usar a lista em http://www.mat.puc-rio.br/~nicolau/olimp/obm-l.html O administrador desta lista é ========================================================================= From owner-obm-l@sucuri.mat.puc-rio.br Sun May 26 23:57:16 2002 Return-Path: Received: (from majordom@localhost) by sucuri.mat.puc-rio.br (8.9.3/8.9.3) id XAA28452 for obm-l-list; Sun, 26 May 2002 23:55:34 -0300 Received: from shannon.bol.com.br (shannon.bol.com.br [200.221.24.13]) by sucuri.mat.puc-rio.br (8.9.3/8.9.3) with ESMTP id XAA28448 for ; Sun, 26 May 2002 23:55:32 -0300 Received: from bol.com.br (200.221.24.77) by shannon.bol.com.br (5.1.071) id 3CE1A947002EEAF6 for obm-l@mat.puc-rio.br; Sun, 26 May 2002 23:43:08 -0300 Date: Sun, 26 May 2002 23:43:07 -0300 Message-Id: Subject: Re:[obm-l] Analise Combinatoria MIME-Version: 1.0 Content-Type: text/plain;charset="iso-8859-1" From: "rafaelc.l" To: obm-l@mat.puc-rio.br X-XaM3-API-Version: 2.4.3.4.4 X-SenderIP: 172.187.111.254 Content-Transfer-Encoding: 8bit X-MIME-Autoconverted: from quoted-printable to 8bit by sucuri.mat.puc-rio.br id XAA28449 Sender: owner-obm-l@sucuri.mat.puc-rio.br Precedence: bulk Reply-To: obm-l@mat.puc-rio.br A resposta não seria: (n-1)/6*(n^2-8n+6)? Considere uma turma com n alunos ,numerados de 1 a n. > Deseja-se organizar uma comissao de 3 alunos.De quantas maneiras pode ser > formada esta comissao,de modo que nao facam parte da mesma dois ou tres > alunosdesignados por numeros consecutivos ? > > _____________________________________________ ____________________ > Una-se ao maior serviço de email do mundo: o MSN Hotmail. > http://www.hotmail.com > > ============================================= ============================ > Instruções para entrar na lista, sair da lista e usar a lista em > http://www.mat.puc- rio.br/~nicolau/olimp/obm-l.html > O administrador desta lista é > ============================================= ============================ > __________________________________________________________________________ Quer ter seu próprio endereço na Internet? Garanta já o seu e ainda ganhe cinco e-mails personalizados. DomíniosBOL - http://dominios.bol.com.br ========================================================================= Instruções para entrar na lista, sair da lista e usar a lista em http://www.mat.puc-rio.br/~nicolau/olimp/obm-l.html O administrador desta lista é ========================================================================= From owner-obm-l@sucuri.mat.puc-rio.br Mon May 27 00:12:13 2002 Return-Path: Received: (from majordom@localhost) by sucuri.mat.puc-rio.br (8.9.3/8.9.3) id AAA28820 for obm-l-list; Mon, 27 May 2002 00:10:55 -0300 Received: from shannon.bol.com.br (shannon.bol.com.br [200.221.24.13]) by sucuri.mat.puc-rio.br (8.9.3/8.9.3) with ESMTP id AAA28816 for ; Mon, 27 May 2002 00:10:54 -0300 Received: from bol.com.br (200.221.24.77) by shannon.bol.com.br (5.1.071) id 3CE1A947002EF34B for obm-l@mat.puc-rio.br; Sun, 26 May 2002 23:58:29 -0300 Date: Sun, 26 May 2002 23:58:29 -0300 Message-Id: Subject: Re: [obm-l] Analise Combinatoria MIME-Version: 1.0 Content-Type: text/plain;charset="iso-8859-1" From: "rafaelc.l" To: obm-l@mat.puc-rio.br X-XaM3-API-Version: 2.4.3.4.4 X-SenderIP: 172.187.82.73 Content-Transfer-Encoding: 8bit X-MIME-Autoconverted: from quoted-printable to 8bit by sucuri.mat.puc-rio.br id AAA28817 Sender: owner-obm-l@sucuri.mat.puc-rio.br Precedence: bulk Reply-To: obm-l@mat.puc-rio.br mande as respostas deles pra mim resolver please... __________________________________________________________________________ Quer ter seu próprio endereço na Internet? Garanta já o seu e ainda ganhe cinco e-mails personalizados. DomíniosBOL - http://dominios.bol.com.br ========================================================================= Instruções para entrar na lista, sair da lista e usar a lista em http://www.mat.puc-rio.br/~nicolau/olimp/obm-l.html O administrador desta lista é ========================================================================= From owner-obm-l@sucuri.mat.puc-rio.br Mon May 27 01:16:06 2002 Return-Path: Received: (from majordom@localhost) by sucuri.mat.puc-rio.br (8.9.3/8.9.3) id BAA29757 for obm-l-list; Mon, 27 May 2002 01:11:51 -0300 Received: from toole.uol.com.br (toole.uol.com.br [200.231.206.186]) by sucuri.mat.puc-rio.br (8.9.3/8.9.3) with ESMTP id BAA29753 for ; Mon, 27 May 2002 01:11:49 -0300 Received: from oemcomputer ([200.227.69.82]) by toole.uol.com.br (8.9.1/8.9.1) with SMTP id AAA22618 for ; Mon, 27 May 2002 00:55:37 -0300 (BRT) Message-ID: <007301c20532$f8f4ed60$5245e3c8@oemcomputer> From: "Paulo Rodrigues" To: References: Subject: Re: [obm-l] Analise Combinatoria Date: Mon, 27 May 2002 00:59:54 -0300 MIME-Version: 1.0 Content-Type: text/plain; charset="iso-8859-1" Content-Transfer-Encoding: 8bit X-Priority: 3 X-MSMail-Priority: Normal X-Mailer: Microsoft Outlook Express 5.00.2615.200 X-MimeOLE: Produced By Microsoft MimeOLE V5.00.2615.200 Sender: owner-obm-l@sucuri.mat.puc-rio.br Precedence: bulk Reply-To: obm-l@mat.puc-rio.br : Considere uma turma com n alunos ,numerados de 1 a n. : Deseja-se organizar uma comissao de 3 alunos.De quantas maneiras pode ser : formada esta comissao,de modo que nao facam parte da mesma dois ou tres : alunosdesignados por numeros consecutivos ? Seja C={x, y, z} uma comissão satisfazendo às condições do problema, com x y +1 > x+2. C1 é necessariamente um subconjunto de [n-2]={1,2,...,n-2} e prova-se facilmente que essa função que leva C em C1 é uma bijeção do conjunto considerado no conjunto dos 3-subconjuntos de [n-2]. Portanto, o número de subconjuntos C é igual ao número de subconjuntos C1, igual a binomial(n-2,3) = (n-2)(n-3)(n-4)/6. --- esta mensagem não contém vírus! Checked by AVG anti-virus system (http://www.grisoft.com). Version: 6.0.363 / Virus Database: 201 - Release Date: 21/05/2002 ========================================================================= Instruções para entrar na lista, sair da lista e usar a lista em http://www.mat.puc-rio.br/~nicolau/olimp/obm-l.html O administrador desta lista é ========================================================================= From owner-obm-l@sucuri.mat.puc-rio.br Mon May 27 11:22:04 2002 Return-Path: Received: (from majordom@localhost) by sucuri.mat.puc-rio.br (8.9.3/8.9.3) id LAA01869 for obm-l-list; Mon, 27 May 2002 11:21:44 -0300 Received: from trex.centroin.com.br (trex.centroin.com.br [200.225.63.134]) by sucuri.mat.puc-rio.br (8.9.3/8.9.3) with ESMTP id LAA01856 for ; Mon, 27 May 2002 11:21:42 -0300 Received: from trex.centroin.com.br (localhost [127.0.0.1]) by trex.centroin.com.br (8.12.1/8.12.1) with ESMTP id g4REAFb6007916 for ; Mon, 27 May 2002 11:10:16 -0300 (EST) Received: by trex.centroin.com.br (8.12.1/8.12.1/Submit) id g4READkj007896; Mon, 27 May 2002 11:10:13 -0300 (EST) Message-Id: <200205271410.g4READkj007896@trex.centroin.com.br> Received: from 200.165.213.58 by trex.centroin.com.br (CIPWM versao 1.4C1) with HTTPS for ; Mon, 27 May 2002 11:10:13 -0300 (EST) Date: Mon, 27 May 2002 11:10:13 -0300 (EST) From: Augusto Cesar de Oliveira Morgado To: obm-l@mat.puc-rio.br Subject: Re: [obm-l] Analise Combinatoria MIME-Version: 1.0 X-Mailer: CentroIn Internet Provider WebMail v. 1.4C1 (http://www.centroin.com.br/) Content-Type: text/plain; charset="iso-8859-1" Content-Transfer-Encoding: 8bit X-MIME-Autoconverted: from quoted-printable to 8bit by sucuri.mat.puc-rio.br id LAA01860 Sender: owner-obm-l@sucuri.mat.puc-rio.br Precedence: bulk Reply-To: obm-l@mat.puc-rio.br Uma solucao mais elementar seria imaginar os alunos 1 2 ... n e marcar com o sinal de + os escolhidos e com o sinal - os não escolhidos. Formaremos uma fila com 3 sinais + e n-3 sinais -, nao podendo haver dois sinais + consecutivos. Para isso, ponha os n-3 sinais - em fila e vejamos de quantos modos podemos enfiar entre eles (ou antes do primeiro ou depois do ultimo) os sinais +. Sao n-2 espaços dos quais devemos escolher 3 e a resposta eh C(n-2,2). Em Mon, 27 May 2002 00:59:54 -0300, Paulo Rodrigues disse: > : Considere uma turma com n alunos ,numerados de 1 a n. > : Deseja-se organizar uma comissao de 3 alunos.De quantas maneiras pode ser > : formada esta comissao,de modo que nao facam parte da mesma dois ou tres > : alunosdesignados por numeros consecutivos ? > > Seja C={x, y, z} uma comissão satisfazendo às condições do problema, com > x z > y +1 > x+2. C1 é necessariamente um subconjunto de [n-2]={1,2,...,n-2} > e prova-se facilmente que essa função que leva C em C1 é uma bijeção do > conjunto considerado no conjunto dos 3-subconjuntos de [n-2]. Portanto, o > número de subconjuntos C é igual ao número de subconjuntos C1, igual a > binomial(n-2,3) = (n-2)(n-3)(n-4)/6. > > > --- > esta mensagem não contém vírus! > Checked by AVG anti-virus system (http://www.grisoft.com). > Version: 6.0.363 / Virus Database: 201 - Release Date: 21/05/2002 > > ========================================================================= > Instruções para entrar na lista, sair da lista e usar a lista em > http://www.mat.puc-rio.br/~nicolau/olimp/obm-l.html > O administrador desta lista é > ========================================================================= > > ========================================================================= Instruções para entrar na lista, sair da lista e usar a lista em http://www.mat.puc-rio.br/~nicolau/olimp/obm-l.html O administrador desta lista é ========================================================================= From owner-obm-l@sucuri.mat.puc-rio.br Mon May 27 11:22:11 2002 Return-Path: Received: (from majordom@localhost) by sucuri.mat.puc-rio.br (8.9.3/8.9.3) id LAA01859 for obm-l-list; Mon, 27 May 2002 11:21:43 -0300 Received: from trex.centroin.com.br (trex.centroin.com.br [200.225.63.134]) by sucuri.mat.puc-rio.br (8.9.3/8.9.3) with ESMTP id LAA01854 for ; Mon, 27 May 2002 11:21:39 -0300 Received: from trex.centroin.com.br (localhost [127.0.0.1]) by trex.centroin.com.br (8.12.1/8.12.1) with ESMTP id g4REAEb6007914 for ; Mon, 27 May 2002 11:10:15 -0300 (EST) Received: by trex.centroin.com.br (8.12.1/8.12.1/Submit) id g4REACo4007879; Mon, 27 May 2002 11:10:12 -0300 (EST) Message-Id: <200205271410.g4REACo4007879@trex.centroin.com.br> Received: from 200.165.213.58 by trex.centroin.com.br (CIPWM versao 1.4C1) with HTTPS for ; Mon, 27 May 2002 11:10:11 -0300 (EST) Date: Mon, 27 May 2002 11:10:11 -0300 (EST) From: Augusto Cesar de Oliveira Morgado To: obm-l@mat.puc-rio.br Subject: Re: [obm-l] Analise Combinatoria MIME-Version: 1.0 X-Mailer: CentroIn Internet Provider WebMail v. 1.4C1 (http://www.centroin.com.br/) Content-Type: text/plain; charset="iso-8859-1" Content-Transfer-Encoding: 8bit X-MIME-Autoconverted: from quoted-printable to 8bit by sucuri.mat.puc-rio.br id LAA01855 Sender: owner-obm-l@sucuri.mat.puc-rio.br Precedence: bulk Reply-To: obm-l@mat.puc-rio.br Uma solucao mais elementar seria imaginar os alunos 1 2 ... n e marcar com o sinal de + os escolhidos e com o sinal - os não escolhidos. Formaremos uma fila com 3 sinais + e n-3 sinais -, nao podendo haver dois sinais + consecutivos. Para isso, ponha os n-3 sinais - em fila e vejamos de quantos modos podemos enfiar entre eles (ou antes do primeiro ou depois do ultimo) os sinais +. Sao n-2 espaços dos quais devemos escolher 3 e a resposta eh C(n-2,2). Em Mon, 27 May 2002 00:59:54 -0300, Paulo Rodrigues disse: > : Considere uma turma com n alunos ,numerados de 1 a n. > : Deseja-se organizar uma comissao de 3 alunos.De quantas maneiras pode ser > : formada esta comissao,de modo que nao facam parte da mesma dois ou tres > : alunosdesignados por numeros consecutivos ? > > Seja C={x, y, z} uma comissão satisfazendo às condições do problema, com > x z > y +1 > x+2. C1 é necessariamente um subconjunto de [n-2]={1,2,...,n-2} > e prova-se facilmente que essa função que leva C em C1 é uma bijeção do > conjunto considerado no conjunto dos 3-subconjuntos de [n-2]. Portanto, o > número de subconjuntos C é igual ao número de subconjuntos C1, igual a > binomial(n-2,3) = (n-2)(n-3)(n-4)/6. > > > --- > esta mensagem não contém vírus! > Checked by AVG anti-virus system (http://www.grisoft.com). > Version: 6.0.363 / Virus Database: 201 - Release Date: 21/05/2002 > > ========================================================================= > Instruções para entrar na lista, sair da lista e usar a lista em > http://www.mat.puc-rio.br/~nicolau/olimp/obm-l.html > O administrador desta lista é > ========================================================================= > > ========================================================================= Instruções para entrar na lista, sair da lista e usar a lista em http://www.mat.puc-rio.br/~nicolau/olimp/obm-l.html O administrador desta lista é ========================================================================= From owner-obm-l@sucuri.mat.puc-rio.br Mon May 27 14:32:37 2002 Return-Path: Received: (from majordom@localhost) by sucuri.mat.puc-rio.br (8.9.3/8.9.3) id OAA05023 for obm-l-list; Mon, 27 May 2002 14:30:03 -0300 Received: from www.zipmail.com.br (smtp.zipmail.com.br [200.187.242.10]) by sucuri.mat.puc-rio.br (8.9.3/8.9.3) with ESMTP id OAA05019 for ; Mon, 27 May 2002 14:30:01 -0300 From: PETERDIRICHLET@zipmail.com.br Received: from [200.206.103.3] by www.zipmail.com.br with HTTP; Mon, 27 May 2002 14:17:36 -0300 Message-ID: <3CF17E8200000A02@www.zipmail.com.br> Date: Mon, 27 May 2002 14:17:36 -0300 In-Reply-To: <3CEE9B8600001323@www.zipmail.com.br> Subject: [obm-l] =?iso-8859-1?Q?=20=5Bobm=2Dl=5D=20=20Uma=20solu=E7ao=20bonitinha=20da=20IMO=20da=20India=28reformu?= =?iso-8859-1?Q?lados=29=20=20=20=20=20=20=20=20=20=20=20=20=20=20=20=20=20=20=20?= To: obm-l@mat.puc-rio.br MIME-Version: 1.0 Content-Type: text/plain; charset="iso-8859-1" Content-Transfer-Encoding: 8bit X-MIME-Autoconverted: from quoted-printable to 8bit by sucuri.mat.puc-rio.br id OAA05020 Sender: owner-obm-l@sucuri.mat.puc-rio.br Precedence: bulk Reply-To: obm-l@mat.puc-rio.br Para aplicar Erdös-Mordell neste problema,e bom que ponhamos tudo para dentro!Traçando os paralelogramos DEFM,NFAB,PBCD,e o triangulo XYZ tal que ZY e perpendicular a BP,XZ a DM e XY a FN,da para reescrever a desigualdade.Veja que os triangulos DEF e DMF sao semelhantes,logo tem o mesmo circunraio.Mas XM e o diametro do triangulo DMF.Logo XM=2*RA.E portanto vamos provar que XM+YN+ZP>=BN+BP+DP+DM+FM+FN. Vamos quebrar isso em dois casos: 1)M,N e P coincidem.E ai e so usar Erdös-Mordell em XYZ direto 2)MNP e um triangulo.Ai fica mais complicado...Mas a ideia e a mesma. Vamos colocar um espelho na bissetriz do angulo ZXY e pegar as imagens de X e de Z(X' e Z' nesta ordem).Considere os pes das perpendiculares de X e M(H e G nesta ordem).Sejam tambem x=YZ,y=ZX e z =XY as medidas dos lados de XYZ. Entao [XZ'M]+[Z'Y'M]+[Y'XM]=[XY'Z'],ou x*XH=x*MG+y*FM+z*MD.Agora vamos usar a desigualdade triangular em XMG e a desigualdade hipotenusa>cateto em XHG(o angulo em H e reto):XM+MG>=XG>=XH,logo XM>=(z/x)*DM+(y/x)*FM. E nao e dificil concluir que XM+YN+ZP>=z/x*DM+y/x*FM+x/y*FN+z/y*BN+y/z*BP+x/z*DP.Daqui sai o fim do problema. Depois eu continuo. ATEEEEEEE!!!!!!Ploft!Peterdirichlet. >>-- Mensagem original -- >> >>>Esse problema foi considerado "O Imortal"(o menos respondido de toda a >>historia >>>da IMO):apenas 2 romenos e 4 armenios resolveram-no.TODA A EQUIPE CHINESA >>>ZEROU ESSE.IMO 1996 >>> >>> >>> >>>Problem 5 >>> >>>Let ABCDEF be a convex hexagon such that AB is parallel to DE, BC is parallel >>>to EF, and CD is parallel to FA. Let RA, RC, RE denote the circumradii >>of >>>triangles FAB, BCD, DEF respectively, and let p denote the perimeter of >>>the hexagon. Prove that: >>> >>> RA + RC + RE >= p/2. >>> >>> >>> >>>Solution >>> >>> >>>The starting point is the formula for the circumradius R of a triangle >>ABC: >>>2R = a/sin A = b/sin B = c/sin C. [Proof: the side a subtends an angle >>2A >>>at the center, so a = 2R sin A.] This gives that 2RA = BF/sin A, 2RC = >>BD/sin >>>C, 2RE = FD/sin E. It is clearly not true in general that BF/sin A > BA >>>+ AF, although it is true if angle FAB >= 120, so we need some argument >>>that involves the hexagon as a whole. >>> >>>Extend sides BC and FE and take lines perpendicular to them through A >and >>>D, thus forming a rectangle. Then BF is greater than or equal to the side >>>through A and the side through D. We may find the length of the side through >>>A by taking the projections of BA and AF giving AB sin B + AF sin F. Similarly >>>the side through D is CD sin C + DE sin E. Hence: >>> >>> 2BF >= AB sin B + AF sin F + CD sin C + DE sin E. Similarly: >>> >>> 2BD >= BC sin B + CD sin D + AF sin A + EF sin E, and >>> >>> 2FD >= AB sin A + BC sin C + DE sin D + EF sin F. >>> >>>Hence 2BF/sin A + 2BD/sin C + 2FD/sin E >= AB(sin A/sin E + sin B/sin >A) >>>+ BC(sin B/sin C + sin C/sin E) + CD(sin C/sin A + sin D/sin C) + DE(sin >>>E/sin A + sin D/sin E) + EF(sin E/sin C + sin F/sin E) + AF(sin F/sin >A >>>+ sin A/sin C). >>> >>>We now use the fact that opposite sides are parallel, which implies that >>>opposite angles are equal: A = E, B = E, C = F. Each of the factors multiplying >>>the sides in the last expression now has the form x + 1/x which has minimum >>>value 2 when x = 1. Hence 2(BF/sin A + BD/sin C + FD/sin E) >= 2p and >the >>>result is proved. >>> >>>Essa soluçao e a oficial.A mais bonita e a de Ciprian Manolescu,o unico >>> Perfect Score da prova.Ele usou a famosa Desigualdade de Erdös-Mordell.Depois eu envio a resposta dele. ATEEEEEEEEE.Peterdirichlet TRANSIRE SVVM PECTVS MVNDOQUE POTIRE CONGREGATI EX TOTO ORBE MATHEMATICI OB SCRIPTA INSIGNIA TRIBVERE Medalha Fields(John Charles Fields) ------------------------------------------ Use o melhor sistema de busca da Internet Radar UOL - http://www.radaruol.com.br ========================================================================= Instruções para entrar na lista, sair da lista e usar a lista em http://www.mat.puc-rio.br/~nicolau/olimp/obm-l.html O administrador desta lista é ========================================================================= From owner-obm-l@sucuri.mat.puc-rio.br Mon May 27 14:45:26 2002 Return-Path: Received: (from majordom@localhost) by sucuri.mat.puc-rio.br (8.9.3/8.9.3) id OAA05199 for obm-l-list; Mon, 27 May 2002 14:45:10 -0300 Received: from hotmail.com (law2-f117.hotmail.com [216.32.181.117]) by sucuri.mat.puc-rio.br (8.9.3/8.9.3) with ESMTP id OAA05195 for ; Mon, 27 May 2002 14:45:05 -0300 Received: from mail pickup service by hotmail.com with Microsoft SMTPSVC; Mon, 27 May 2002 10:33:26 -0700 Received: from 32.94.119.254 by lw2fd.hotmail.msn.com with HTTP; Mon, 27 May 2002 17:33:25 GMT X-Originating-IP: [32.94.119.254] From: "Paulo Santa Rita" To: obm-l@mat.puc-rio.br Subject: Re: [obm-l] Conicas e Latus Rectum Date: Mon, 27 May 2002 17:33:25 +0000 Mime-Version: 1.0 Content-Type: text/plain; charset=iso-8859-1; format=flowed Message-ID: X-OriginalArrivalTime: 27 May 2002 17:33:26.0158 (UTC) FILETIME=[9B8252E0:01C205A4] Sender: owner-obm-l@sucuri.mat.puc-rio.br Precedence: bulk Reply-To: obm-l@mat.puc-rio.br Ola Marcio e demais colegas desta lista, E verdade. Isso vale para todas conicas e o valor minimo e o Latus Rectum. No Livro : Geometria Analitica Nikolai Efimov Editora mir Voce vai encontra todas estas coisas e muito mais. Nao e um livro facil de achar, mas vale a pena estuda-lo com atencao. Inclusive as quadricas sao abordadas de maneira simples e, a meu ver, completa. Ai no IME ou muito provavelmente na Biblioteca do IMPA ou da UFRJ deve ter. Se eu fosse voce nao usaria as equacoes retangulares. Usada a equacao polar das conicas e procedia conforme a sua intuicao esta indicando : equacao (pola ) da reta, inteseccao com as conicas, distancia entre as interseccoes e passagem aos pontos criticos (minimo). No livro acima voce vai saber nao so sobre o latus rectum, mas muitas outras coisas que poucos sabem. Desde ja te advirto para uma coisa : o escritor russo nao faz muito floreio e nem se preocupa muito com fundamentacao, mas e sempre genial em seus problemas e conclusoes. Em geral se gasta um certo tempo ate a gente se acostumar com o jeitao deles. Mas vale a pena. Um abraco Paulo Santa Rita 2,1423,270502 >From: mcohen@iis.com.br >Reply-To: obm-l@mat.puc-rio.br >To: >Subject: [obm-l] Conicas e Latus Rectum >Date: Sun, 26 May 2002 19:49:58 -0300 > >Como mostrar que a menor corda focal de uma elipse eh sempre perpendicular >ao eixo maior? > >Esse problema me persegue ha bastante tempo ... qdo eu era aluno no 2o >grau, li em algum lugar essa propriedade... q dentre todas as cordas da >elipse passando por um determinado foco, aquela q fosse perpendicular ao >eixo maior tinha comprimento minimo.. parece q existem resultados analogos >para as outras conicas.. Eu nao lembro aonde li isso, mas nunca encontrei a >demonstracao. e agora eu precisei demonstrar isso e nao consegui. se alguem >puder ajudar, agradeco!! (tentei inicialmente colocar o comprimento da >corda em funcao do angulo x com a horizontal, e depois analisar a funcao >f(tan(x)).. a expressao era um pouco grande (4 fracoes racionais de grau <= >2, duas delas ao quadrado) mas nao oobtive sucesso... > >Obrigado, >Marcio > > > > >========================================================================= >Instruções para entrar na lista, sair da lista e usar a lista em >http://www.mat.puc-rio.br/~nicolau/olimp/obm-l.html >O administrador desta lista é >========================================================================= _________________________________________________________________ Envie e receba emails com o Hotmail no seu dispositivo móvel: http://mobile.msn.com ========================================================================= Instruções para entrar na lista, sair da lista e usar a lista em http://www.mat.puc-rio.br/~nicolau/olimp/obm-l.html O administrador desta lista é ========================================================================= From owner-obm-l@sucuri.mat.puc-rio.br Mon May 27 15:39:24 2002 Return-Path: Received: (from majordom@localhost) by sucuri.mat.puc-rio.br (8.9.3/8.9.3) id PAA06655 for obm-l-list; Mon, 27 May 2002 15:38:19 -0300 Received: from hotmail.com (law2-f137.hotmail.com [216.32.181.137]) by sucuri.mat.puc-rio.br (8.9.3/8.9.3) with ESMTP id PAA06651 for ; Mon, 27 May 2002 15:38:17 -0300 Received: from mail pickup service by hotmail.com with Microsoft SMTPSVC; Mon, 27 May 2002 11:26:37 -0700 Received: from 32.94.119.254 by lw2fd.hotmail.msn.com with HTTP; Mon, 27 May 2002 18:26:37 GMT X-Originating-IP: [32.94.119.254] From: "Paulo Santa Rita" To: obm-l@mat.puc-rio.br Subject: Re: [obm-l] Analise Combinatoria Date: Mon, 27 May 2002 18:26:37 +0000 Mime-Version: 1.0 Content-Type: text/plain; charset=iso-8859-1; format=flowed Message-ID: X-OriginalArrivalTime: 27 May 2002 18:26:37.0983 (UTC) FILETIME=[09FC2AF0:01C205AC] Sender: owner-obm-l@sucuri.mat.puc-rio.br Precedence: bulk Reply-To: obm-l@mat.puc-rio.br Ola Pessoal, Em muitos problemas de Analise Combinatoria, como no caso abaixo, o enunciado faz algumas restricoes. Um caminho natural e que, quase sempre, conduz a uma solucao satisfatoria e considerar as restricoes e conta-las separadamente ... O total de comissoes com 3 alunos e : BINOM(N,3). Desse total temos que retirar as restricoes. Vamos conta-las : 1) tres numeros consecutivos : {1,2,3},{2,3,4}, ..., {N-2,N-1,N} Evidentemente, N-2 casos SUBTOTAL1 : (N-2) casos 2) dois numeros consecutivos : {1,2) + Um escolhido entre {4,5,...,N} = N-3 casos {N-1,N} + Um escolhido entre {1,2,...,N-3) = N-3 casos SUBTOTAL2 : 2(N-3) casos Pares : {2,3},{3,4},...,{N-2,N-1} -> (N-3) pares Para cada um desses pares podemos escolher um terceiro numero de N-4 maneiras. Logo : (N-3)(N-4) pares SUBTOTAL3 : (N-3)(N-4) CALCULO FINAL : T=BINOM(N,3) - [ SUBTOTAL1 + SUBTOTAL2 + SUBTOTAL3 ] T=BINOM(N,3) - [(N-2)^2] T=[N(N-1)(N-2)/6] - [(N-2)^2]=((N-2)(N-3)(N-4))/6 UMA OUTRA FORMA DE FAZER : Seja a equacao : X+Y+Z=N. Sem duvida que voce sabe calcular o numero de solucoes inteiras nao negativas desta equacao, pois este problema basico e abordado em todo livro de Analise Combinatoria. Para obter as "solucoes positivas", faca : X=X'+1 ; Y=Y'+1 e Z=Z'+1 E resolva : X'+Y'+Z'=N-3. Se voce considerar que uma solucao de uma tal equacao e apenas uma forma de separa N-3 esferas por duas barras, a cada colucao inteira positiva corresponde uma forma de escolher duas pessoas nao consecutivas. Exemplo ( vou considerar |=Barra, A=unidade e N-3=5 ) A solucao (1,2,2) correspode a A | AA | AA isto significaria escolher a segundo e o 5 aluno; AA|AA|A=(2,2,1) equivale a escolher o 3 e o 6 aluno; A|AAA|A=(1,3,1) equivale a escolher o 2 e o 6 aluno e assim sucessivamente. Ha, portanto, uma funcao entre as solucoes inteiras positivas da equacao e as escolhas que voce pode fazer. E so pensar com calma que a solucao sai por aqui facil. Os detalhes voce completa UMA OUTRA SOLUCAO ? : Procure se informar sobre os lemas de Kaplanski. No livro de Analise Combinatoria do Prof Morgado ha tudo isso e muito mais. Este livro, em minha opiniao, e o que ha de melhor no Brasil nesta area e vai te dar uma base para voce ser bem sucedido em qualquer vestibular que exija tal tema. Um abraco Paulo Santa Rita 2,1521,270502 >Considere uma turma com n alunos ,numerados de 1 a n. >Deseja-se organizar uma comissao de 3 alunos.De quantas maneiras pode ser >formada esta comissao,de modo que nao facam parte da mesma dois ou tres >alunosdesignados por numeros consecutivos ? _________________________________________________________________ Envie e receba emails com o Hotmail no seu dispositivo móvel: http://mobile.msn.com ========================================================================= Instruções para entrar na lista, sair da lista e usar a lista em http://www.mat.puc-rio.br/~nicolau/olimp/obm-l.html O administrador desta lista é ========================================================================= From owner-obm-l@sucuri.mat.puc-rio.br Mon May 27 15:59:30 2002 Return-Path: Received: (from majordom@localhost) by sucuri.mat.puc-rio.br (8.9.3/8.9.3) id PAA07083 for obm-l-list; Mon, 27 May 2002 15:59:14 -0300 Received: from hotmail.com (law2-f38.hotmail.com [216.32.181.38]) by sucuri.mat.puc-rio.br (8.9.3/8.9.3) with ESMTP id PAA07077 for ; Mon, 27 May 2002 15:59:11 -0300 Received: from mail pickup service by hotmail.com with Microsoft SMTPSVC; Mon, 27 May 2002 11:47:32 -0700 Received: from 200.229.244.100 by lw2fd.hotmail.msn.com with HTTP; Mon, 27 May 2002 18:47:29 GMT X-Originating-IP: [200.229.244.100] From: "Paulo Santa Rita" To: obm-l@mat.puc-rio.br Subject: Re: [obm-l] Analise Combinatoria (Ops ... Cochilo) Date: Mon, 27 May 2002 18:47:29 +0000 Mime-Version: 1.0 Content-Type: text/plain; charset=iso-8859-1; format=flowed Message-ID: X-OriginalArrivalTime: 27 May 2002 18:47:32.0657 (UTC) FILETIME=[F5D43A10:01C205AE] Sender: owner-obm-l@sucuri.mat.puc-rio.br Precedence: bulk Reply-To: obm-l@mat.puc-rio.br Ola Pessoal, Corrigindo um cochilo. Na segunda solucao e necessario considerar uma equacao a quatro variaveis e nao a duas : X+Y+Z+W=N-3 Em geral, pensa-se em N-3 como o numero de unidades que devem ser separadas por tres barras. Exemplo : X+Y+Z+W=12 UUU|UUU|UUUU|UU -> solucao : (3,3,4,2) U|UUUUU|U|UUUUU -> solucao : (1,5,1,5) E assim sucessivamente ... O que eu estou propondo e que se olhe as barras como alunos escolhidos : UUU|UUU|UUUU|UU -> alunos escolhidos : {4,8,13} U|UUUUU|U|UUUUU -> alunos escolhidos : {2,8,10} Olhando as coisas assim ( eu sei que e um olhar estranho e nao convencional, mas entenda que funciona ) a toda solucao da equacao "sem zero" corresponde uma escolha valida dos alunos. Basta agora considerar separadamente as solucoes da forma : IUU..., UUU..U| e IUU...UU| Um abraco Paulo Santa Rita 2,1544,270502 >From: "Paulo Santa Rita" >Reply-To: obm-l@mat.puc-rio.br >To: obm-l@mat.puc-rio.br >Subject: Re: [obm-l] Analise Combinatoria >Date: Mon, 27 May 2002 18:26:37 +0000 > >Ola Pessoal, > >Em muitos problemas de Analise Combinatoria, como no caso abaixo, o >enunciado faz algumas restricoes. Um caminho natural e que, quase sempre, >conduz a uma solucao satisfatoria e considerar as restricoes >e conta-las separadamente ... > >O total de comissoes com 3 alunos e : BINOM(N,3). > >Desse total temos que retirar as restricoes. Vamos conta-las : > >1) tres numeros consecutivos : >{1,2,3},{2,3,4}, ..., {N-2,N-1,N} >Evidentemente, N-2 casos >SUBTOTAL1 : (N-2) casos > >2) dois numeros consecutivos : >{1,2) + Um escolhido entre {4,5,...,N} = N-3 casos >{N-1,N} + Um escolhido entre {1,2,...,N-3) = N-3 casos >SUBTOTAL2 : 2(N-3) casos > >Pares : {2,3},{3,4},...,{N-2,N-1} -> (N-3) pares >Para cada um desses pares podemos escolher um terceiro numero de N-4 >maneiras. Logo : (N-3)(N-4) pares >SUBTOTAL3 : (N-3)(N-4) > >CALCULO FINAL : > >T=BINOM(N,3) - [ SUBTOTAL1 + SUBTOTAL2 + SUBTOTAL3 ] >T=BINOM(N,3) - [(N-2)^2] >T=[N(N-1)(N-2)/6] - [(N-2)^2]=((N-2)(N-3)(N-4))/6 > > >UMA OUTRA FORMA DE FAZER : > >Seja a equacao : X+Y+Z=N. Sem duvida que voce sabe calcular o numero de >solucoes inteiras nao negativas desta equacao, pois este problema basico e >abordado em todo livro de Analise Combinatoria. > >Para obter as "solucoes positivas", faca : > >X=X'+1 ; Y=Y'+1 e Z=Z'+1 > >E resolva : X'+Y'+Z'=N-3. Se voce considerar que uma solucao de uma tal >equacao e apenas uma forma de separa N-3 esferas por duas barras, a cada >colucao inteira positiva corresponde uma forma de escolher duas pessoas nao >consecutivas. > >Exemplo ( vou considerar |=Barra, A=unidade e N-3=5 ) > >A solucao (1,2,2) correspode a A | AA | AA isto significaria escolher a >segundo e o 5 aluno; AA|AA|A=(2,2,1) equivale a escolher o 3 e o 6 aluno; >A|AAA|A=(1,3,1) equivale a escolher o 2 e o 6 aluno e assim sucessivamente. >Ha, portanto, uma funcao entre as solucoes inteiras positivas da equacao e >as escolhas que voce pode fazer. E so pensar com calma que a solucao sai >por >aqui facil. Os detalhes voce completa > > >UMA OUTRA SOLUCAO ? : > >Procure se informar sobre os lemas de Kaplanski. No livro de Analise >Combinatoria do Prof Morgado ha tudo isso e muito mais. Este livro, em >minha >opiniao, e o que ha de melhor no Brasil nesta area e vai te dar uma base >para voce ser bem sucedido em qualquer vestibular que exija tal tema. > >Um abraco >Paulo Santa Rita >2,1521,270502 > >>Considere uma turma com n alunos ,numerados de 1 a n. >>Deseja-se organizar uma comissao de 3 alunos.De quantas maneiras pode ser >>formada esta comissao,de modo que nao facam parte da mesma dois ou tres >>alunosdesignados por numeros consecutivos ? > > >_________________________________________________________________ >Envie e receba emails com o Hotmail no seu dispositivo móvel: >http://mobile.msn.com > >========================================================================= >Instruções para entrar na lista, sair da lista e usar a lista em >http://www.mat.puc-rio.br/~nicolau/olimp/obm-l.html >O administrador desta lista é >========================================================================= _________________________________________________________________ Converse com amigos on-line, conheça o MSN Messenger: http://messenger.msn.com ========================================================================= Instruções para entrar na lista, sair da lista e usar a lista em http://www.mat.puc-rio.br/~nicolau/olimp/obm-l.html O administrador desta lista é ========================================================================= From owner-obm-l@sucuri.mat.puc-rio.br Mon May 27 22:13:40 2002 Return-Path: Received: (from majordom@localhost) by sucuri.mat.puc-rio.br (8.9.3/8.9.3) id WAA11176 for obm-l-list; Mon, 27 May 2002 22:12:33 -0300 Received: from Euler.impa.br (euler.impa.br [147.65.1.3]) by sucuri.mat.puc-rio.br (8.9.3/8.9.3) with ESMTP id WAA11172 for ; Mon, 27 May 2002 22:12:31 -0300 Received: from [147.65.11.7] (dial07.impa.br [147.65.11.7]) by Euler.impa.br (8.11.6/8.11.6) with ESMTP id g4S10r715865 for ; Mon, 27 May 2002 22:00:53 -0300 (EST) Message-Id: <200205280100.g4S10r715865@Euler.impa.br> X-Mailer: Microsoft Outlook Express Macintosh Edition - 4.5 (0410) Date: Wed, 29 May 2002 21:58:39 -0300 Subject: Re: [obm-l] Conicas e Latus Rectum From: "Eduardo Wagner" To: obm-l@mat.puc-rio.br Mime-version: 1.0 X-Priority: 3 Content-type: text/plain; charset="ISO-8859-1" Content-Transfer-Encoding: 8bit X-MIME-Autoconverted: from quoted-printable to 8bit by sucuri.mat.puc-rio.br id WAA11173 Sender: owner-obm-l@sucuri.mat.puc-rio.br Precedence: bulk Reply-To: obm-l@mat.puc-rio.br > > Como mostrar que a menor corda focal de uma elipse eh sempre perpendicular > ao eixo maior? > Vamos la. Ponha a elipse nos eixos na posicao canonica: centrada na origem e com o eixo maior sobre o eixo X. Sejam: a 0 semi eixo maior, b o menor F = (c, 0) o foco da direita e, e = c/a, a excentricidade. Seja P = (x, y) um ponto qualquer da elipse. Usando a definição de elipse, PF + PF' = 2a, mostre que PF = a - ex. Seja PF = r e seja ainda t o angulo XFP. Tem-se entao a partir da relacao anterior: r = a - e(c + rcost), o que da r = (b^2)/(a (1 - e.cost)). Logo, r = PF sera minimo quando t = 90 graus. Abraco, Wagner. > Esse problema me persegue ha bastante tempo ... qdo eu era aluno no 2o > grau, li em algum lugar essa propriedade... q dentre todas as cordas da > elipse passando por um determinado foco, aquela q fosse perpendicular ao > eixo maior tinha comprimento minimo.. parece q existem resultados analogos > para as outras conicas.. Eu nao lembro aonde li isso, mas nunca encontrei a > demonstracao. e agora eu precisei demonstrar isso e nao consegui. se alguem > puder ajudar, agradeco!! (tentei inicialmente colocar o comprimento da > corda em funcao do angulo x com a horizontal, e depois analisar a funcao > f(tan(x)).. a expressao era um pouco grande (4 fracoes racionais de grau <= > 2, duas delas ao quadrado) mas nao oobtive sucesso... > > Obrigado, > Marcio > > > > > ========================================================================= > Instruções para entrar na lista, sair da lista e usar a lista em > http://www.mat.puc-rio.br/~nicolau/olimp/obm-l.html > O administrador desta lista é > ========================================================================= ========================================================================= Instruções para entrar na lista, sair da lista e usar a lista em http://www.mat.puc-rio.br/~nicolau/olimp/obm-l.html O administrador desta lista é ========================================================================= From owner-obm-l@sucuri.mat.puc-rio.br Tue May 28 00:57:33 2002 Return-Path: Received: (from majordom@localhost) by sucuri.mat.puc-rio.br (8.9.3/8.9.3) id AAA12805 for obm-l-list; Tue, 28 May 2002 00:55:59 -0300 Received: from hotmail.com (f156.pav2.hotmail.com [64.4.37.156]) by sucuri.mat.puc-rio.br (8.9.3/8.9.3) with ESMTP id AAA12801 for ; Tue, 28 May 2002 00:55:57 -0300 Received: from mail pickup service by hotmail.com with Microsoft SMTPSVC; Mon, 27 May 2002 20:44:15 -0700 Received: from 200.199.189.31 by pv2fd.pav2.hotmail.msn.com with HTTP; Tue, 28 May 2002 03:44:15 GMT X-Originating-IP: [200.199.189.31] From: "Fernanda Medeiros" To: obm-l@mat.puc-rio.br Subject: [obm-l] desigualdades e cone sul Date: Tue, 28 May 2002 03:44:15 +0000 Mime-Version: 1.0 Content-Type: text/plain; charset=iso-8859-1; format=flowed Message-ID: X-OriginalArrivalTime: 28 May 2002 03:44:15.0705 (UTC) FILETIME=[F0576490:01C205F9] Sender: owner-obm-l@sucuri.mat.puc-rio.br Precedence: bulk Reply-To: obm-l@mat.puc-rio.br Olá pessoal,gostaria de um help nessas questões: 1.Seja n um nº natural ,n>3. Demonstrar que entre os multiplos de 9 menores q 10^n há mais nºs com a soma de seus digitos igual a 9(n-2) que nºs com a soma de seus digitos igual a 9(n-1) 2.Sejam a,b e c os comprimentos dos lados de um triangulo.Mostre que a função f(x)=b^2x^2 +(b^2 +c^2 -a^2)x +c^2 é positiva ,pra todo real x. (ps. essa eu fiz assim,pra f(x)ser >0 devemos ter delta<0 dae fica [(b^2+c^2-a^2)^2 - (2bc)^2] fatorando agumas vezes chegamos a [(b+c-a)(b+c+a)][(b-(c+a))(b-c+a)] daí por desigualdade triangular,vemos q esse produto é <0 ... tá certo?) 3.Sejam x,y reais positivos satisfazendo x^2+xy+y^2>3 .Prove q pelo menos um dos nºs x^2 +xy e y^2 +xy é maior que 2. Obrigada!! []´s Fê _________________________________________________________________ O MSN Photos é o modo mais fácil de compartilhar e imprimir suas fotos: http://photos.msn.com/support/worldwide.aspx ========================================================================= Instruções para entrar na lista, sair da lista e usar a lista em http://www.mat.puc-rio.br/~nicolau/olimp/obm-l.html O administrador desta lista é ========================================================================= From owner-obm-l@sucuri.mat.puc-rio.br Tue May 28 13:13:19 2002 Return-Path: Received: (from majordom@localhost) by sucuri.mat.puc-rio.br (8.9.3/8.9.3) id NAA20063 for obm-l-list; Tue, 28 May 2002 13:11:26 -0300 Received: from hotmail.com (law2-f109.hotmail.com [216.32.181.109]) by sucuri.mat.puc-rio.br (8.9.3/8.9.3) with ESMTP id NAA20059 for ; Tue, 28 May 2002 13:11:23 -0300 Received: from mail pickup service by hotmail.com with Microsoft SMTPSVC; Tue, 28 May 2002 08:59:40 -0700 Received: from 32.94.119.254 by lw2fd.hotmail.msn.com with HTTP; Tue, 28 May 2002 15:59:40 GMT X-Originating-IP: [32.94.119.254] From: "Paulo Santa Rita" To: obm-l@mat.puc-rio.br Subject: [obm-l] Mais Informacoes sobre o Teorema de Riemann Date: Tue, 28 May 2002 15:59:40 +0000 Mime-Version: 1.0 Content-Type: text/plain; charset=iso-8859-1; format=flowed Message-ID: X-OriginalArrivalTime: 28 May 2002 15:59:40.0600 (UTC) FILETIME=[ACD40B80:01C20660] Sender: owner-obm-l@sucuri.mat.puc-rio.br Precedence: bulk Reply-To: obm-l@mat.puc-rio.br Ola Pessoal, Eu estou precisando de mais informacoes sobre o TEOREMA DE RIEMANN acerca das series condicionalmente convergentes. O que todos os livros de Analise que eu conheco ( e sao muitos ! ) falam e insuficiente para os meus propositos : eles tao somente enunciam o teorema e mostram aplicacoes e exemplos elementares e/ou classicos, deixando em aberto uma enormidade de perguntas consistentes que podem ser feitas. Sera que alguem conhece algum estudo mais aprofundado sobre este Teorema, tal como um artigo ou uma Tese de Mestrado ou de Doutorado ? O Teorema de Riemann ao qual me refiro e aquele que diz que uma serie condicionalmente convergente pode divergir ou assumir um valor qualquer atraves de um rearranjo dos seus termos. Desde ja agradeco qualquer ajuda Um Grande abraco a todos Paulo Santa Rita 3,1255,280502 _________________________________________________________________ Chegou o novo MSN Explorer. Instale já. É gratuito: http://explorer.msn.com.br ========================================================================= Instruções para entrar na lista, sair da lista e usar a lista em http://www.mat.puc-rio.br/~nicolau/olimp/obm-l.html O administrador desta lista é ========================================================================= From owner-obm-l@sucuri.mat.puc-rio.br Tue May 28 15:43:22 2002 Return-Path: Received: (from majordom@localhost) by sucuri.mat.puc-rio.br (8.9.3/8.9.3) id PAA23027 for obm-l-list; Tue, 28 May 2002 15:41:45 -0300 Received: from hotmail.com (oe65.law7.hotmail.com [216.33.236.204]) by sucuri.mat.puc-rio.br (8.9.3/8.9.3) with ESMTP id PAA23023 for ; Tue, 28 May 2002 15:41:41 -0300 Received: from mail pickup service by hotmail.com with Microsoft SMTPSVC; Tue, 28 May 2002 11:29:56 -0700 X-Originating-IP: [200.241.240.154] From: "Marcelo Rufino de Oliveira" To: References: Subject: Re: [obm-l] desigualdades e cone sul Date: Tue, 28 May 2002 15:35:55 -0300 MIME-Version: 1.0 Content-Type: text/plain; charset="iso-8859-1" Content-Transfer-Encoding: 8bit X-Priority: 3 X-MSMail-Priority: Normal X-Mailer: Microsoft Outlook Express 5.00.2314.1300 X-MimeOLE: Produced By Microsoft MimeOLE V5.00.2314.1300 Message-ID: X-OriginalArrivalTime: 28 May 2002 18:29:56.0420 (UTC) FILETIME=[AAAD0040:01C20675] Sender: owner-obm-l@sucuri.mat.puc-rio.br Precedence: bulk Reply-To: obm-l@mat.puc-rio.br Acho que a primeira sai da seguinte forma: > 1.Seja n um nº natural ,n>3. > Demonstrar que entre os multiplos de 9 menores q 10^n há mais nºs com a soma > de seus digitos igual a 9(n-2) que nºs com a soma de seus digitos igual a > 9(n-1) Seja A = {ak} o conjunto de todos os inteiros menores que 10^n e cuja soma dos dígitos é 9(n - 2) e seja B = {bk}o conjunto de todos os inteiros menores que 10^n e cuja soma dos dígitos é 9(n - 1). Associe a cada número de A o elemento xk = 999...99 - ak e a cada número de B o elemento yk = 999...99 - bk, onde cada número 999...99 possui n 9's. Desde que a soma dos dígitos de ak é 9(n - 2) então a soma dos dígitos de xk é 9n - 9(n - 2) = 18. Analogamente a soma dos dígitos de cada yk é 9n - 9(n - 1) = 9. Como para cada ak existe um único xk, então a quantidade de elementos de A é igual ao número de inteiros menores que 10^n e cuja soma dos dígitos vale 18. Da mesma forma, a quantidade de elementos de B é igual ao número de inteiros menores que 10^n e cuja soma dos dígitos vale 9. Agora fica fácil demonstrar que n(A) > n(B). > Obrigada!! > []´s > Fê > Até mais, Marcelo Rufino de Oliveira ========================================================================= Instruções para entrar na lista, sair da lista e usar a lista em http://www.mat.puc-rio.br/~nicolau/olimp/obm-l.html O administrador desta lista é ========================================================================= From owner-obm-l@sucuri.mat.puc-rio.br Tue May 28 17:43:48 2002 Return-Path: Received: (from majordom@localhost) by sucuri.mat.puc-rio.br (8.9.3/8.9.3) id RAA25438 for obm-l-list; Tue, 28 May 2002 17:43:13 -0300 Received: from www.zipmail.com.br (smtp.zipmail.com.br [200.187.242.10]) by sucuri.mat.puc-rio.br (8.9.3/8.9.3) with ESMTP id RAA25434 for ; Tue, 28 May 2002 17:43:11 -0300 From: peterdirichlet@zipmail.com.br Received: from [200.206.103.3] by www.zipmail.com.br with HTTP; Tue, 28 May 2002 14:24:41 -0300 Message-ID: <3CF2718E0000146E@www.zipmail.com.br> Date: Tue, 28 May 2002 14:24:41 -0300 In-Reply-To: <3CF17E8200000A02@www.zipmail.com.br> Subject: [obm-l] =?iso-8859-1?Q?=20Uma=20solu=E7ao=20bonitinha=20do=20problema=205=20da=20IMO=20da=20India=28Er?= =?iso-8859-1?Q?d=F6s=2DMordell=20na=20veia=21=21=21=21=21=29=20=20=20=20=20=20=20?= To: obm-l@mat.puc-rio.br MIME-Version: 1.0 Content-Type: text/plain; charset="iso-8859-1" Content-Transfer-Encoding: 8bit X-MIME-Autoconverted: from quoted-printable to 8bit by sucuri.mat.puc-rio.br id RAA25435 Sender: owner-obm-l@sucuri.mat.puc-rio.br Precedence: bulk Reply-To: obm-l@mat.puc-rio.br Problema 5-IMO 1996(Bombaim,India) Seja ABCDEF um hexagono convexo tal que AB e paralelo a DE, BC e paralelo a EF,e CD e paralelo a FA. Sejam R_A, R_C, R_E os circunraios dos triangulos FAB, BCD, DEF respectivamente,e seja p o perimetro do hexagono. Prove que: R_A + R_C + R_E >= p/2. Esse problema foi considerado "O Imortal"(o menos respondido de toda a historia da IMO):apenas 2 romenos e 4 armenios resolveram-no completamente.TODA A EQUIPE CHINESA ZEROU ESTE PROBLEMA!!!!!!!A soluçao mais bonita e a do estudante romeno Ciprian Manolescu,o unico Perfect Score(pontuaçao maxima:42 pontos)da prova.Ele baseou-se na famosa Desigualdade de Erdös-Mordell.Aqui,mostrarei esta soluçao. Para aplicar Erdös-Mordell neste problema(esta ideia e bem razoavel,ja que a expressao sugere isto),e bom que ponhamos tudo para dentro!Traçando os paralelogramos MDEF,NFAB,PBCD,e o triangulo XYZ tal que ZY e perpendicular a BP,XZ a DM e XY a FN,e tal que o hexagono fique inscrito ao triangulo XYZ,pode-se reescrever a desigualdade.Veja que os triangulos DEF e DMF sao semelhantes,logo tem o mesmo circunraio.Mas XM e o diametro do circuncirculo do triangulo DMF(ou do quadrilatero DMFX se preferir).Logo XM=2*RA.E portanto vamos provar que XM+YN+ZP>=BN+BP+DP+DM+FM+FN,o que equivale ao problema inicial. Vamos quebrar isso em dois casos: 1)M,N e P coincidem.E o problema se transforma em Erdös-Mordell. 2)MNP e um triangulo.Ai fica mais complicado...Mas a ideia e a mesma. Vamos colocar um espelho na bissetriz do angulo ZXY e considerar as imagens de X e de Z(X' e Z' nesta ordem).Considere os pes das perpendiculares de X e M(H e G,nesta ordem).Sejam tambem x=YZ,y=ZX e z =XY as medidas dos lados de XYZ.E [ABCD...Z] indica a area do poligono ABCD...Z. Entao [XZ'M]+[Z'Y'M]+[Y'XM]=[XY'Z'],ou x*XH=x*MG+y*FM+z*MD. Agora vamos usar a desigualdade triangular em XMG e a desigualdade hipotenusa>cateto em XHG(o angulo em H e reto):XM+MG>=XG>=XH,logo XM>=(z/x)*DM+(y/x)*FM. E nao e dificil concluir que XM+YN+ZP>=z/x*DM+y/x*FM+x/y*FN+z/y*BN+y/z*BP+x/z*DP. (*) Daqui sai o fim do problema.Agora,veja que 2*(y/z*BP+z/y*BN)=(y/z+z/y)*(BN+BP)+(y/z-z/y)(BN-BP) Como os triangulos XYZ e DEF sao semelhantes,podemos definir K=(FM-FN)/XY=(BN-BP)/YZ=(DP-DM)/ZX. Agora,usando Medias,temos que y/z*BP+z/y*BN>=(BP+BN)+K*(y*x/z-z*x/y) Analogamente obteriamos resultados analogos a este ultimo para os pares de lados (x,z) e (x,y).Agora basta substituir todos eles em (*) e acabou! Esta segunda soluçao e de autoria do lider armenio(o mesmo que o propos).Voce pode encontra-la no site http://www.cie.uva.es/algebra/fdelgado/seccion/principal_sen.htm Let ABCDEF be a convex hexagon such that AB is parallel to DE, BC is parallel to EF, and CD is parallel to FA. Let RA, RC, RE denote the circumradii of triangles FAB, BCD, DEF respectively, and let p denote the perimeter of the hexagon. Prove that: R_A + R_C + R_E >= p/2. Solution(Bank) The starting point is the formula for the circumradius R of a triangle ABC(Sines Theorem):2R = a/sin A = b/sin B = c/sin C. [Proof: the side a subtends an angle 2A at the center, so a = 2R sin A.] This gives that 2RA = BF/sin A, 2RC>=BD/sin C, 2R_E = FD/sin E. It is clearly not true in general that BF/sin A > BA+AF, although it is true if angle FAB >= 120, so we need some argumentthat involves the hexagon as a whole. Extend sides BC and FE and take lines perpendicular to them through A and D, thus forming a rectangle. Then BF is greater than or equal to the sidethrough A and the side through D. We may find the length of the side through >>>>A by taking the projections of BA and AF giving AB sin B + AF sin F. >Similarly the side through D is CD sin C + DE sin E. Hence: >>>> >>>> 2BF >= AB sin B + AF sin F + CD sin C + DE sin E. Similarly: >>>> >>>> 2BD >= BC sin B + CD sin D + AF sin A + EF sin E, and >>>> >>>> 2FD >= AB sin A + BC sin C + DE sin D + EF sin F. >>>> >>>>Hence 2BF/sin A + 2BD/sin C + 2FD/sin E >= AB(sin A/sin E + sin B/sin >>A) >>>>+ BC(sin B/sin C + sin C/sin E) + CD(sin C/sin A + sin D/sin C) + DE(sin >>>>E/sin A + sin D/sin E) + EF(sin E/sin C + sin F/sin E) + AF(sin F/sin >>A >>>>+ sin A/sin C). >>>> >>>>We now use the fact that opposite sides are parallel, which implies that >>>>opposite angles are equal: A = E, B = E, C = F. Each of the factors multiplying >>>>the sides in the last expression now has the form x + 1/x which has minimum >>>>value 2 when x = 1. Hence 2(BF/sin A + BD/sin C + FD/sin E) >= 2p and >>the >>>>result is proved. >>>> >>>>Essa soluçao e a oficial.A mais bonita e a de Ciprian Manolescu,o unico >>>> Perfect Score da prova.Ele usou a famosa Desigualdade de Erdös-Mordell.Depois >eu envio a resposta dele. >ATEEEEEEEEE.Peterdirichlet > >TRANSIRE SVVM PECTVS MVNDOQUE POTIRE >CONGREGATI EX TOTO ORBE MATHEMATICI OB SCRIPTA INSIGNIA TRIBVERE >Medalha Fields(John Charles Fields) > > >------------------------------------------ >Use o melhor sistema de busca da Internet >Radar UOL - http://www.radaruol.com.br > > > >========================================================================= >Instruções para entrar na lista, sair da lista e usar a lista em >http://www.mat.puc-rio.br/~nicolau/olimp/obm-l.html >O administrador desta lista é >========================================================================= > TRANSIRE SVVM PECTVS MVNDOQUE POTIRE CONGREGATI EX TOTO ORBE MATHEMATICI OB SCRIPTA INSIGNIA TRIBVERE Medalha Fields(John Charles Fields) ------------------------------------------ Use o melhor sistema de busca da Internet Radar UOL - http://www.radaruol.com.br ========================================================================= Instruções para entrar na lista, sair da lista e usar a lista em http://www.mat.puc-rio.br/~nicolau/olimp/obm-l.html O administrador desta lista é ========================================================================= From owner-obm-l@sucuri.mat.puc-rio.br Tue May 28 17:43:48 2002 Return-Path: Received: (from majordom@localhost) by sucuri.mat.puc-rio.br (8.9.3/8.9.3) id RAA25448 for obm-l-list; Tue, 28 May 2002 17:43:21 -0300 Received: from www.zipmail.com.br (smtp.zipmail.com.br [200.187.242.10]) by sucuri.mat.puc-rio.br (8.9.3/8.9.3) with ESMTP id RAA25444 for ; Tue, 28 May 2002 17:43:18 -0300 From: peterdirichlet@zipmail.com.br Received: from [200.206.103.3] by www.zipmail.com.br with HTTP; Tue, 28 May 2002 14:24:29 -0300 Message-ID: <3CF2718E0000146D@www.zipmail.com.br> Date: Tue, 28 May 2002 14:24:29 -0300 In-Reply-To: <3CF17E8200000A02@www.zipmail.com.br> Subject: [obm-l] =?iso-8859-1?Q?=20Uma=20solu=E7ao=20bonitinha=20do=20problema=205=20da=20IMO=20da=20India=28Er?= =?iso-8859-1?Q?d=F6s=2DMordell=20na=20veia=21=21=21=21=21=29=20=20=20=20=20=20=20?= To: obm-l@mat.puc-rio.br MIME-Version: 1.0 Content-Type: text/plain; charset="iso-8859-1" Content-Transfer-Encoding: 8bit X-MIME-Autoconverted: from quoted-printable to 8bit by sucuri.mat.puc-rio.br id RAA25445 Sender: owner-obm-l@sucuri.mat.puc-rio.br Precedence: bulk Reply-To: obm-l@mat.puc-rio.br Problema 5-IMO 1996(Bombaim,India) Seja ABCDEF um hexagono convexo tal que AB e paralelo a DE, BC e paralelo a EF,e CD e paralelo a FA. Sejam R_A, R_C, R_E os circunraios dos triangulos FAB, BCD, DEF respectivamente,e seja p o perimetro do hexagono. Prove que: R_A + R_C + R_E >= p/2. Esse problema foi considerado "O Imortal"(o menos respondido de toda a historia da IMO):apenas 2 romenos e 4 armenios resolveram-no completamente.TODA A EQUIPE CHINESA ZEROU ESTE PROBLEMA!!!!!!!A soluçao mais bonita e a do estudante romeno Ciprian Manolescu,o unico Perfect Score(pontuaçao maxima:42 pontos)da prova.Ele baseou-se na famosa Desigualdade de Erdös-Mordell.Aqui,mostrarei esta soluçao. Para aplicar Erdös-Mordell neste problema(esta ideia e bem razoavel,ja que a expressao sugere isto),e bom que ponhamos tudo para dentro!Traçando os paralelogramos MDEF,NFAB,PBCD,e o triangulo XYZ tal que ZY e perpendicular a BP,XZ a DM e XY a FN,e tal que o hexagono fique inscrito ao triangulo XYZ,pode-se reescrever a desigualdade.Veja que os triangulos DEF e DMF sao semelhantes,logo tem o mesmo circunraio.Mas XM e o diametro do circuncirculo do triangulo DMF(ou do quadrilatero DMFX se preferir).Logo XM=2*RA.E portanto vamos provar que XM+YN+ZP>=BN+BP+DP+DM+FM+FN,o que equivale ao problema inicial. Vamos quebrar isso em dois casos: 1)M,N e P coincidem.E o problema se transforma em Erdös-Mordell. 2)MNP e um triangulo.Ai fica mais complicado...Mas a ideia e a mesma. Vamos colocar um espelho na bissetriz do angulo ZXY e considerar as imagens de X e de Z(X' e Z' nesta ordem).Considere os pes das perpendiculares de X e M(H e G,nesta ordem).Sejam tambem x=YZ,y=ZX e z =XY as medidas dos lados de XYZ.E [ABCD...Z] indica a area do poligono ABCD...Z. Entao [XZ'M]+[Z'Y'M]+[Y'XM]=[XY'Z'],ou x*XH=x*MG+y*FM+z*MD. Agora vamos usar a desigualdade triangular em XMG e a desigualdade hipotenusa>cateto em XHG(o angulo em H e reto):XM+MG>=XG>=XH,logo XM>=(z/x)*DM+(y/x)*FM. E nao e dificil concluir que XM+YN+ZP>=z/x*DM+y/x*FM+x/y*FN+z/y*BN+y/z*BP+x/z*DP. (*) Daqui sai o fim do problema.Agora,veja que 2*(y/z*BP+z/y*BN)=(y/z+z/y)*(BN+BP)+(y/z-z/y)(BN-BP) Como os triangulos XYZ e DEF sao semelhantes,podemos definir K=(FM-FN)/XY=(BN-BP)/YZ=(DP-DM)/ZX. Agora,usando Medias,temos que y/z*BP+z/y*BN>=(BP+BN)+K*(y*x/z-z*x/y) Analogamente obteriamos resultados analogos a este ultimo para os pares de lados (x,z) e (x,y).Agora basta substituir todos eles em (*) e acabou! Esta segunda soluçao e de autoria do lider armenio(o mesmo que o propos).Voce pode encontra-la no site http://www.cie.uva.es/algebra/fdelgado/seccion/principal_sen.htm Let ABCDEF be a convex hexagon such that AB is parallel to DE, BC is parallel to EF, and CD is parallel to FA. Let RA, RC, RE denote the circumradii of triangles FAB, BCD, DEF respectively, and let p denote the perimeter of the hexagon. Prove that: R_A + R_C + R_E >= p/2. Solution(Bank) The starting point is the formula for the circumradius R of a triangle ABC(Sines Theorem):2R = a/sin A = b/sin B = c/sin C. [Proof: the side a subtends an angle 2A at the center, so a = 2R sin A.] This gives that 2RA = BF/sin A, 2RC>=BD/sin C, 2R_E = FD/sin E. It is clearly not true in general that BF/sin A > BA+AF, although it is true if angle FAB >= 120, so we need some argumentthat involves the hexagon as a whole. Extend sides BC and FE and take lines perpendicular to them through A and D, thus forming a rectangle. Then BF is greater than or equal to the sidethrough A and the side through D. We may find the length of the side through >>>>A by taking the projections of BA and AF giving AB sin B + AF sin F. >Similarly the side through D is CD sin C + DE sin E. Hence: >>>> >>>> 2BF >= AB sin B + AF sin F + CD sin C + DE sin E. Similarly: >>>> >>>> 2BD >= BC sin B + CD sin D + AF sin A + EF sin E, and >>>> >>>> 2FD >= AB sin A + BC sin C + DE sin D + EF sin F. >>>> >>>>Hence 2BF/sin A + 2BD/sin C + 2FD/sin E >= AB(sin A/sin E + sin B/sin >>A) >>>>+ BC(sin B/sin C + sin C/sin E) + CD(sin C/sin A + sin D/sin C) + DE(sin >>>>E/sin A + sin D/sin E) + EF(sin E/sin C + sin F/sin E) + AF(sin F/sin >>A >>>>+ sin A/sin C). >>>> >>>>We now use the fact that opposite sides are parallel, which implies that >>>>opposite angles are equal: A = E, B = E, C = F. Each of the factors multiplying >>>>the sides in the last expression now has the form x + 1/x which has minimum >>>>value 2 when x = 1. Hence 2(BF/sin A + BD/sin C + FD/sin E) >= 2p and >>the >>>>result is proved. >>>> >>>>Essa soluçao e a oficial.A mais bonita e a de Ciprian Manolescu,o unico >>>> Perfect Score da prova.Ele usou a famosa Desigualdade de Erdös-Mordell.Depois >eu envio a resposta dele. >ATEEEEEEEEE.Peterdirichlet > >TRANSIRE SVVM PECTVS MVNDOQUE POTIRE >CONGREGATI EX TOTO ORBE MATHEMATICI OB SCRIPTA INSIGNIA TRIBVERE >Medalha Fields(John Charles Fields) > > >------------------------------------------ >Use o melhor sistema de busca da Internet >Radar UOL - http://www.radaruol.com.br > > > >========================================================================= >Instruções para entrar na lista, sair da lista e usar a lista em >http://www.mat.puc-rio.br/~nicolau/olimp/obm-l.html >O administrador desta lista é >========================================================================= > TRANSIRE SVVM PECTVS MVNDOQUE POTIRE CONGREGATI EX TOTO ORBE MATHEMATICI OB SCRIPTA INSIGNIA TRIBVERE Medalha Fields(John Charles Fields) ------------------------------------------ Use o melhor sistema de busca da Internet Radar UOL - http://www.radaruol.com.br ========================================================================= Instruções para entrar na lista, sair da lista e usar a lista em http://www.mat.puc-rio.br/~nicolau/olimp/obm-l.html O administrador desta lista é ========================================================================= From owner-obm-l@sucuri.mat.puc-rio.br Tue May 28 17:52:58 2002 Return-Path: Received: (from majordom@localhost) by sucuri.mat.puc-rio.br (8.9.3/8.9.3) id RAA25624 for obm-l-list; Tue, 28 May 2002 17:52:45 -0300 Received: from www.zipmail.com.br (smtp.zipmail.com.br [200.187.242.10]) by sucuri.mat.puc-rio.br (8.9.3/8.9.3) with ESMTP id RAA25620 for ; Tue, 28 May 2002 17:52:40 -0300 From: ghaeser@zipmail.com.br Received: from [143.106.39.221] by www.zipmail.com.br with HTTP; Tue, 28 May 2002 17:40:57 -0300 Message-ID: <3CF3E8CB00000073@www.zipmail.com.br> Date: Tue, 28 May 2002 17:40:57 -0300 Subject: [obm-l] =?iso-8859-1?Q?unicidade=20de=20polinomios?= To: obm-l@mat.puc-rio.br MIME-Version: 1.0 Content-Type: text/plain; charset="iso-8859-1" Content-Transfer-Encoding: 8bit X-MIME-Autoconverted: from quoted-printable to 8bit by sucuri.mat.puc-rio.br id RAA25621 Sender: owner-obm-l@sucuri.mat.puc-rio.br Precedence: bulk Reply-To: obm-l@mat.puc-rio.br Olá pessoal da lista, me surgiu uma dúvida durante uma aula de análise que a professora nao conseguiu tirar.. Seja f(x)=a0+a1x+..+anx^n, g(x)=b0+b1x+..+bnx^n polinômios de grau n, onde f(x)=g(x) qualquer que seja x, prove que ai=bi para i=0,..,n .. para todo n natural. demonstração da professora: seja h(x)=f(x)-g(x)=0 para todo x real, por hipotese logo como h(0)=0 entao a0=b0 por inducao, suponha que a(n-1)=b(n-1), logo, como h(1)=0 temos: a0+..+a(n-1)+an=b0+..+b(n-1)+bn, como a0+..+a(n-1)=b0+..+b(n-1), temos que an=bn logo, por indução temos que vale para todo n minha dúvida é: seja um polinomio h(x) de grau n, onde h(0)=0 e h(1)=0, prove que a0=a1=..=an=0 para todo n. obviamente isto é falso, mas eu consigo demonstrar utilizando a prova dela.. por isso acho q tem alguma coisa errada com a hipotese de inducao .. talvez deva haver uma inclusao da hipotese de haver n+1 zeros para o grau n .. tentei explicar isto para ela, mas ela nao concordou .. será que alguém pode me ajudar ?? muito obrigado !! Gabriel Haeser www.gabas.cjb.net "Mathematicus nascitur, non fit" Matemáticos não são feitos, eles nascem --------------------------------------- Gabriel Haeser www.gabas.cjb.net ------------------------------------------ Use o melhor sistema de busca da Internet Radar UOL - http://www.radaruol.com.br ========================================================================= Instruções para entrar na lista, sair da lista e usar a lista em http://www.mat.puc-rio.br/~nicolau/olimp/obm-l.html O administrador desta lista é ========================================================================= From owner-obm-l@sucuri.mat.puc-rio.br Tue May 28 20:39:43 2002 Return-Path: Received: (from majordom@localhost) by sucuri.mat.puc-rio.br (8.9.3/8.9.3) id UAA28949 for obm-l-list; Tue, 28 May 2002 20:38:30 -0300 Received: from web13301.mail.yahoo.com (web13301.mail.yahoo.com [216.136.175.37]) by sucuri.mat.puc-rio.br (8.9.3/8.9.3) with SMTP id UAA28945 for ; Tue, 28 May 2002 20:38:28 -0300 Message-ID: <20020528232652.50585.qmail@web13301.mail.yahoo.com> Received: from [200.207.158.20] by web13301.mail.yahoo.com via HTTP; Tue, 28 May 2002 16:26:52 PDT Date: Tue, 28 May 2002 16:26:52 -0700 (PDT) From: Carlos Yuzo Shine Subject: [obm-l] Novidades no site da OPM To: obm-l@mat.puc-rio.br, teoremalista@yahoogrupos.com.br, ocmlista@yahoogrupos.com.br In-Reply-To: MIME-Version: 1.0 Content-Type: text/plain; charset=us-ascii Sender: owner-obm-l@sucuri.mat.puc-rio.br Precedence: bulk Reply-To: obm-l@mat.puc-rio.br Oi gente! O site da OPM está com novidades! Uma delas é que estamos oferecendo mais um curso do nosso Programa de Aperfeiçoamento em Matemática. Alunos e professores estão convidados! Cheque em http://www.opm.mat.br/ []'s Shine __________________________________________________ Do You Yahoo!? Yahoo! - Official partner of 2002 FIFA World Cup http://fifaworldcup.yahoo.com ========================================================================= Instruções para entrar na lista, sair da lista e usar a lista em http://www.mat.puc-rio.br/~nicolau/olimp/obm-l.html O administrador desta lista é ========================================================================= From owner-obm-l@sucuri.mat.puc-rio.br Wed May 29 10:14:55 2002 Return-Path: Received: (from majordom@localhost) by sucuri.mat.puc-rio.br (8.9.3/8.9.3) id KAA02514 for obm-l-list; Wed, 29 May 2002 10:10:35 -0300 Received: from smtp.ieg.com.br (194.125.226.200.in-addr.arpa.ig.com.br [200.226.125.194] (may be forged)) by sucuri.mat.puc-rio.br (8.9.3/8.9.3) with ESMTP id KAA02510 for ; Wed, 29 May 2002 10:10:32 -0300 Received: from ieg.com.br (hendrix.protocoloweb.com.br [200.185.63.19]) by smtp.ieg.com.br (IeG relay/8.9.3) with SMTP id g4TCtO54002247; Wed, 29 May 2002 09:55:24 -0300 (BRT) From: "Arnaldo" To: ghaeser@zipmail.com.br, obm-l@mat.puc-rio.br Date: Wed, 29 May 2002 12:51:17 GMT Subject: Re: [obm-l] unicidade de polinomios X-Mailer: DMailWeb Web to Mail Gateway 2.7v, http://netwinsite.com/top_mail.htm Message-id: <3cf4cec5.3c89.0@ieg.com.br> X-User-Info: 146.164.38.60 MIME-Version: 1.0 Content-Type: text/plain; charset="iso-8859-1" Content-Transfer-Encoding: 8bit X-MIME-Autoconverted: from quoted-printable to 8bit by sucuri.mat.puc-rio.br id KAA02511 Sender: owner-obm-l@sucuri.mat.puc-rio.br Precedence: bulk Reply-To: obm-l@mat.puc-rio.br >Olá pessoal da lista, >me surgiu uma dúvida durante uma aula de análise que a professora nao conseguiu >tirar.. > >Seja f(x)=a0+a1x+..+anx^n, g(x)=b0+b1x+..+bnx^n polinômios de grau n, onde >f(x)=g(x) qualquer que seja x, prove que ai=bi para i=0,..,n .. para todo >n natural. > >demonstração da professora: > >seja h(x)=f(x)-g(x)=0 para todo x real, por hipotese >logo como h(0)=0 entao a0=b0 >por inducao, suponha que a(n-1)=b(n-1), logo, como h(1)=0 temos: >a0+..+a(n-1)+an=b0+..+b(n-1)+bn, como >a0+..+a(n-1)=b0+..+b(n-1), temos que an=bn > >logo, por indução temos que vale para todo n > >minha dúvida é: > >seja um polinomio h(x) de grau n, onde h(0)=0 e h(1)=0, prove que a0=a1=..=an=0 >para todo n. > >obviamente isto é falso, mas eu consigo demonstrar utilizando a prova dela.. >por isso acho q tem alguma coisa errada com a hipotese de inducao .. talvez >deva haver uma inclusao da hipotese de haver n+1 zeros para o grau n .. >tentei explicar isto para ela, mas ela nao concordou .. será que alguém >pode me ajudar ?? > >muito obrigado !! > >Gabriel Haeser >www.gabas.cjb.net > >Bem ! Da maneira que foi feito a prova também acho que tem problemas, pois dados os dois polinomios do problema ( de grau n ) tente provar que a1=b1 usando que a0=b0 (a prova dela fura). Acho que uma maneira mais convincente seria usando o teorema fundamental da algebra para o polinomio h(x). >"Mathematicus nascitur, non fit" >Matemáticos não são feitos, eles nascem >--------------------------------------- >Gabriel Haeser >www.gabas.cjb.net > > >------------------------------------------ >Use o melhor sistema de busca da Internet >Radar UOL - http://www.radaruol.com.br > > > >========================================================================= >Instruções para entrar na lista, sair da lista e usar a lista em >http://www.mat.puc-rio.br/~nicolau/olimp/obm-l.html >O administrador desta lista é >========================================================================= > http://www.ieg.com.br ========================================================================= Instruções para entrar na lista, sair da lista e usar a lista em http://www.mat.puc-rio.br/~nicolau/olimp/obm-l.html O administrador desta lista é ========================================================================= From owner-obm-l@sucuri.mat.puc-rio.br Wed May 29 11:05:10 2002 Return-Path: Received: (from majordom@localhost) by sucuri.mat.puc-rio.br (8.9.3/8.9.3) id LAA03483 for obm-l-list; Wed, 29 May 2002 11:03:05 -0300 Received: from mat.puc-rio.br (IDENT:root@perere.mat.puc-rio.br [139.82.27.60]) by sucuri.mat.puc-rio.br (8.9.3/8.9.3) with ESMTP id LAA03479 for ; Wed, 29 May 2002 11:03:04 -0300 Received: from localhost (fredpalm@localhost) by mat.puc-rio.br (8.9.3/8.9.3) with ESMTP id DAA06536 for ; Wed, 29 May 2002 03:03:17 -0300 Date: Wed, 29 May 2002 03:03:17 -0300 (BRT) From: Carlos Frederico Borges Palmeira To: obm-l@mat.puc-rio.br Subject: Re: [obm-l] =?iso-8859-1?Q?unicidade=20de=20polinomios?= In-Reply-To: <3CF3E8CB00000073@www.zipmail.com.br> Message-ID: MIME-Version: 1.0 Content-Type: TEXT/PLAIN; charset=X-UNKNOWN Content-Transfer-Encoding: 8bit X-MIME-Autoconverted: from QUOTED-PRINTABLE to 8bit by sucuri.mat.puc-rio.br id LAA03480 Sender: owner-obm-l@sucuri.mat.puc-rio.br Precedence: bulk Reply-To: obm-l@mat.puc-rio.br oi gabriel, o sua professora de fato escorregou na utilizacao do principio da inducao. A forma usual e' provar que vale para n=0, e provar que se vale para n entao vale para n+1. Outra forma e' provar que vale para n=0 e provar que se vale para todo k de 0 a n , entao vale para n+1. Apaarentemente ela disse que ia usar a 1a e de fato usou a 2a. Como voce observou a 1a forma e' insuficiente para esta demonstracao. na passagem destacada abaixo, e' de fato necessario a 2a forma. []s Fred Palmeira On Tue, 28 May 2002 ghaeser@zipmail.com.br wrote: > Olá pessoal da lista, > me surgiu uma dúvida durante uma aula de análise que a professora nao conseguiu > tirar.. > > Seja f(x)=a0+a1x+..+anx^n, g(x)=b0+b1x+..+bnx^n polinômios de grau n, onde > f(x)=g(x) qualquer que seja x, prove que ai=bi para i=0,..,n .. para todo > n natural. > > demonstração da professora: > > seja h(x)=f(x)-g(x)=0 para todo x real, por hipotese > logo como h(0)=0 entao a0=b0 > por inducao, suponha que a(n-1)=b(n-1), logo, como h(1)=0 temos: > a0+..+a(n-1)+an=b0+..+b(n-1)+bn, para passar para a proxima linha precisa supor verdade para todo k de 0 a n. ************************************ como > a0+..+a(n-1)=b0+..+b(n-1), temos que an=bn **************************************** > > logo, por indução temos que vale para todo n > > minha dúvida é: > > seja um polinomio h(x) de grau n, onde h(0)=0 e h(1)=0, prove que a0=a1=..=an=0 > para todo n. > > obviamente isto é falso, mas eu consigo demonstrar utilizando a prova dela.. > por isso acho q tem alguma coisa errada com a hipotese de inducao .. talvez > deva haver uma inclusao da hipotese de haver n+1 zeros para o grau n .. > tentei explicar isto para ela, mas ela nao concordou .. será que alguém > pode me ajudar ?? > > muito obrigado !! > > Gabriel Haeser > www.gabas.cjb.net > > > "Mathematicus nascitur, non fit" > Matemáticos não são feitos, eles nascem > --------------------------------------- > Gabriel Haeser > www.gabas.cjb.net > > > ------------------------------------------ > Use o melhor sistema de busca da Internet > Radar UOL - http://www.radaruol.com.br > > > > ========================================================================= > Instruções para entrar na lista, sair da lista e usar a lista em > http://www.mat.puc-rio.br/~nicolau/olimp/obm-l.html > O administrador desta lista é > ========================================================================= > ========================================================================= Instruções para entrar na lista, sair da lista e usar a lista em http://www.mat.puc-rio.br/~nicolau/olimp/obm-l.html O administrador desta lista é ========================================================================= From owner-obm-l@sucuri.mat.puc-rio.br Wed May 29 13:35:18 2002 Return-Path: Received: (from majordom@localhost) by sucuri.mat.puc-rio.br (8.9.3/8.9.3) id NAA06014 for obm-l-list; Wed, 29 May 2002 13:32:58 -0300 Received: from mat.puc-rio.br (IDENT:root@perere.mat.puc-rio.br [139.82.27.60]) by sucuri.mat.puc-rio.br (8.9.3/8.9.3) with ESMTP id NAA06010 for ; Wed, 29 May 2002 13:32:56 -0300 Received: from localhost (fredpalm@localhost) by mat.puc-rio.br (8.9.3/8.9.3) with ESMTP id FAA06729 for ; Wed, 29 May 2002 05:33:10 -0300 Date: Wed, 29 May 2002 05:33:09 -0300 (BRT) From: Carlos Frederico Borges Palmeira To: obm-l@mat.puc-rio.br Subject: Re: [obm-l] =?iso-8859-1?Q?unicidade=20de=20polinomios?= In-Reply-To: Message-ID: MIME-Version: 1.0 Content-Type: TEXT/PLAIN; charset=X-UNKNOWN Content-Transfer-Encoding: 8bit X-MIME-Autoconverted: from QUOTED-PRINTABLE to 8bit by sucuri.mat.puc-rio.br id NAA06011 Sender: owner-obm-l@sucuri.mat.puc-rio.br Precedence: bulk Reply-To: obm-l@mat.puc-rio.br olhando com mais cuidado, vejo que escorreguei tambem. De fato se for verdade para todo k de 0 a n-1, e' verdade para n. isto e' o que foi demonstrado. so que n esta' fixo. como foi observado pelo arnaldo, nao da nem para passar de 0 para 1. Uma alternativa para o passo de inducao e' supondo verdade a igualdade dos coeficientes ate' k ,obtemos a igualdade a(k+1)x^k+1 + a(k+2)x^k+2+...= b(k+1)x^k+1+... Dividindo por x^k+1 e fazendo x=0, obtem-se a(k+1)=b(k+1). Fred Palmeira On Wed, 29 May 2002, Carlos Frederico Borges Palmeira wrote: > oi gabriel, > o sua professora de fato escorregou na utilizacao do principio da > inducao. A forma usual e' provar que vale para n=0, e provar que se vale > para n entao vale para n+1. Outra forma e' provar que vale para n=0 e > provar que se vale para todo k de 0 a n , entao vale para n+1. > Apaarentemente ela disse que ia usar a 1a e de fato usou a 2a. Como voce > observou a 1a forma e' insuficiente para esta demonstracao. na passagem > destacada abaixo, e' de fato necessario a 2a forma. > []s > Fred Palmeira > > > > > > On Tue, 28 May 2002 ghaeser@zipmail.com.br wrote: > > > Olá pessoal da lista, > > me surgiu uma dúvida durante uma aula de análise que a professora nao conseguiu > > tirar.. > > > > Seja f(x)=a0+a1x+..+anx^n, g(x)=b0+b1x+..+bnx^n polinômios de grau n, onde > > f(x)=g(x) qualquer que seja x, prove que ai=bi para i=0,..,n .. para todo > > n natural. > > > > demonstração da professora: > > > > seja h(x)=f(x)-g(x)=0 para todo x real, por hipotese > > logo como h(0)=0 entao a0=b0 > > por inducao, suponha que a(n-1)=b(n-1), logo, como h(1)=0 temos: > > a0+..+a(n-1)+an=b0+..+b(n-1)+bn, > para passar para a proxima linha precisa supor verdade para todo k de 0 a > n. > ************************************ > como > > a0+..+a(n-1)=b0+..+b(n-1), temos que an=bn > **************************************** > > > > > logo, por indução temos que vale para todo n > > > > minha dúvida é: > > > > seja um polinomio h(x) de grau n, onde h(0)=0 e h(1)=0, prove que a0=a1=..=an=0 > > para todo n. > > > > obviamente isto é falso, mas eu consigo demonstrar utilizando a prova dela.. > > por isso acho q tem alguma coisa errada com a hipotese de inducao .. talvez > > deva haver uma inclusao da hipotese de haver n+1 zeros para o grau n .. > > tentei explicar isto para ela, mas ela nao concordou .. será que alguém > > pode me ajudar ?? > > > > muito obrigado !! > > > > Gabriel Haeser > > www.gabas.cjb.net > > > > > > "Mathematicus nascitur, non fit" > > Matemáticos não são feitos, eles nascem > > --------------------------------------- > > Gabriel Haeser > > www.gabas.cjb.net > > > > > > ------------------------------------------ > > Use o melhor sistema de busca da Internet > > Radar UOL - http://www.radaruol.com.br > > > > > > > > ========================================================================= > > Instruções para entrar na lista, sair da lista e usar a lista em > > http://www.mat.puc-rio.br/~nicolau/olimp/obm-l.html > > O administrador desta lista é > > ========================================================================= > > > > ========================================================================= > Instruções para entrar na lista, sair da lista e usar a lista em > http://www.mat.puc-rio.br/~nicolau/olimp/obm-l.html > O administrador desta lista é > ========================================================================= > ========================================================================= Instruções para entrar na lista, sair da lista e usar a lista em http://www.mat.puc-rio.br/~nicolau/olimp/obm-l.html O administrador desta lista é ========================================================================= From owner-obm-l@sucuri.mat.puc-rio.br Wed May 29 14:07:38 2002 Return-Path: Received: (from majordom@localhost) by sucuri.mat.puc-rio.br (8.9.3/8.9.3) id OAA06692 for obm-l-list; Wed, 29 May 2002 14:06:02 -0300 Received: from www.zipmail.com.br (smtp.zipmail.com.br [200.187.242.10]) by sucuri.mat.puc-rio.br (8.9.3/8.9.3) with ESMTP id OAA06684 for ; Wed, 29 May 2002 14:05:52 -0300 From: peterdirichlet@zipmail.com.br Received: from [200.206.103.3] by www.zipmail.com.br with HTTP; Wed, 29 May 2002 13:54:18 -0300 Message-ID: <3CF3C4B400001462@www.zipmail.com.br> Date: Wed, 29 May 2002 13:54:18 -0300 In-Reply-To: <3CF2718E0000146D@www.zipmail.com.br> Subject: [obm-l] =?iso-8859-1?Q?Re=3A=20=5Bobm=2Dl=5D=20=20Uma=20solu=E7ao=20bonitinha=20do=20problema=205=20da=20IMO?= =?iso-8859-1?Q?=20da=20India=28Erd=F6s=2DMordell=20na=20veia=21=21=21=21=21=29=20=20=20=20=20=20=20?= To: obm-l@mat.puc-rio.br MIME-Version: 1.0 Content-Type: text/plain; charset="iso-8859-1" Content-Transfer-Encoding: 8bit X-MIME-Autoconverted: from quoted-printable to 8bit by sucuri.mat.puc-rio.br id OAA06689 Sender: owner-obm-l@sucuri.mat.puc-rio.br Precedence: bulk Reply-To: obm-l@mat.puc-rio.br -- Mensagem original -- >Problema 5-IMO 1996(Bombaim,India) Seja ABCDEF um hexagono convexo tal que >AB e paralelo a DE, BC e paralelo a EF,e CD e paralelo a FA. Sejam R_A, >R_C, R_E os circunraios dos triangulos FAB, BCD, DEF respectivamente,e seja > p o perimetro do hexagono. Prove que: > > R_A + R_C + R_E >= p/2. > >Esse problema foi considerado "O Imortal"(o menos respondido de toda a historia >da IMO):apenas 2 romenos e 4 armenios resolveram-no completamente.TODA A >EQUIPE CHINESA ZEROU ESTE PROBLEMA!!!!!!!A soluçao mais bonita e a do estudante >romeno Ciprian Manolescu,o unico Perfect Score(pontuaçao maxima:42 pontos)da >prova.Ele baseou-se na famosa Desigualdade de Erdös-Mordell.Aqui,mostrarei >esta soluçao. > >Para aplicar Erdös-Mordell neste problema(esta ideia e bem razoavel,ja que >a expressao sugere isto),e bom que ponhamos tudo para dentro!Traçando os >paralelogramos MDEF,NFAB,PBCD,e o triangulo XYZ tal que ZY e perpendicular >a BP,XZ a DM e XY a FN,e tal que o hexagono fique inscrito ao triangulo >XYZ,pode-se reescrever a desigualdade.Veja que os triangulos DEF e DMF sao >semelhantes,logo tem o mesmo circunraio.Mas XM e o diametro do circuncirculo >do triangulo DMF(ou do quadrilatero DMFX se preferir).Logo XM=2*RA.E portanto >vamos provar que XM+YN+ZP>=BN+BP+DP+DM+FM+FN,o que equivale ao problema >inicial. > >Vamos quebrar isso em dois casos: >1)M,N e P coincidem.E o problema se transforma em Erdös-Mordell. >2)MNP e um triangulo.Ai fica mais complicado...Mas a ideia e a mesma. >Vamos colocar um espelho na bissetriz do angulo ZXY e considerar as imagens >de X e de Z(X' e Z' nesta ordem).Considere os pes das perpendiculares de >X e M(H e G,nesta ordem).Sejam tambem x=YZ,y=ZX e z =XY as medidas dos lados >de XYZ.E [ABCD...Z] indica a area do poligono ABCD...Z. >Entao [XZ'M]+[Z'Y'M]+[Y'XM]=[XY'Z'],ou > > x*XH=x*MG+y*FM+z*MD. > >Agora vamos usar a desigualdade triangular em XMG e a desigualdade hipotenusa>cateto >em XHG(o >angulo em H e reto):XM+MG>=XG>=XH,logo > XM>=(z/x)*DM+(y/x)*FM. >E nao e dificil concluir que > > XM+YN+ZP>=z/x*DM+y/x*FM+x/y*FN+z/y*BN+y/z*BP+x/z*DP. (*) > >Daqui sai o fim do problema.Agora,veja que >2*(y/z*BP+z/y*BN)=(y/z+z/y)*(BN+BP)+(y/z-z/y)(BN-BP) > >Como os triangulos XYZ e DEF sao semelhantes,podemos definir > K=(FM-FN)/XY=(BN-BP)/YZ=(DP-DM)/ZX. >Agora,usando Medias,temos que > > y/z*BP+z/y*BN>=(BP+BN)+K*(y*x/z-z*x/y) > >Analogamente obteriamos resultados analogos a este ultimo para os pares >de lados (x,z) e (x,y).Agora basta substituir todos eles em (*) e acabou! > > >Esta segunda soluçao e de autoria do lider armenio(o mesmo que o propos).Voce >pode encontra-la no site http://www.cie.uva.es/algebra/fdelgado/seccion/principal_sem.htm > > >Let ABCDEF be a convex hexagon such that AB is parallel to DE, BC is parallel >to EF, and CD is parallel to FA. Let RA, RC, RE denote the circumradii of >triangles FAB, BCD, DEF respectively, and let p denote the perimeter of >the hexagon. Prove that: > > R_A + R_C + R_E >= p/2. >Solution(Bank) > >The starting point is the formula for the circumradius R of a triangle ABC(Sines >Theorem):2R = a/sin A = b/sin B = c/sin C. [Proof: the side a subtends an >angle 2A at the center, so a = 2R sin A.] This gives that 2RA = BF/sin A, >2RC>=BD/sin C, 2R_E = FD/sin E. It is clearly not true in general that BF/sin >A > >BA+AF, although it is true if angle FAB >= 120, so we need some argumentthat >involves the hexagon as a whole. >Extend sides BC and FE and take lines perpendicular to them through A >and D, thus forming a rectangle. Then BF is greater than or equal to the >sidethrough A and the side through D. We may find the length of the side >through >>>>>A by taking the projections of BA and AF giving AB sin B + AF sin F. >>Similarly the side through D is CD sin C + DE sin E. Hence: >>>>> >>>>> 2BF >= AB sin B + AF sin F + CD sin C + DE sin E. Similarly: >>>>> >>>>> 2BD >= BC sin B + CD sin D + AF sin A + EF sin E, and >>>>> >>>>> 2FD >= AB sin A + BC sin C + DE sin D + EF sin F. >>>>> >>>>>Hence 2BF/sin A + 2BD/sin C + 2FD/sin E >= AB(sin A/sin E + sin B/sin >>>A) >>>>>+ BC(sin B/sin C + sin C/sin E) + CD(sin C/sin A + sin D/sin C) + DE(sin >>>>>E/sin A + sin D/sin E) + EF(sin E/sin C + sin F/sin E) + AF(sin F/sin >>>A >>>>>+ sin A/sin C). >>>>> >>>>>We now use the fact that opposite sides are parallel, which implies >that >>>>>opposite angles are equal: A = E, B = E, C = F. Each of the factors >multiplying >>>>>the sides in the last expression now has the form x + 1/x which has >minimum >>>>>value 2 when x = 1. Hence 2(BF/sin A + BD/sin C + FD/sin E) >= 2p and >>>the >>>>>result is proved. >>>>> >>>>>Essa soluçao e a oficial.A mais bonita e a de Ciprian Manolescu,o unico >>>>> Perfect Score da prova.Ele usou a famosa Desigualdade de Erdös-Mordell.Depois >>eu envio a resposta dele. >>ATEEEEEEEEE.Peterdirichlet >> >>TRANSIRE SVVM PECTVS MVNDOQUE POTIRE >>CONGREGATI EX TOTO ORBE MATHEMATICI OB SCRIPTA INSIGNIA TRIBVERE >>Medalha Fields(John Charles Fields) >> >> >>------------------------------------------ >>Use o melhor sistema de busca da Internet >>Radar UOL - http://www.radaruol.com.br >> >> >> >>========================================================================= >>Instruções para entrar na lista, sair da lista e usar a lista em >>http://www.mat.puc-rio.br/~nicolau/olimp/obm-l.html >>O administrador desta lista é >>========================================================================= >> > >TRANSIRE SVVM PECTVS MVNDOQUE POTIRE >CONGREGATI EX TOTO ORBE MATHEMATICI OB SCRIPTA INSIGNIA TRIBVERE >Medalha Fields(John Charles Fields) > > >------------------------------------------ >Use o melhor sistema de busca da Internet >Radar UOL - http://www.radaruol.com.br > > > >========================================================================= >Instruções para entrar na lista, sair da lista e usar a lista em >http://www.mat.puc-rio.br/~nicolau/olimp/obm-l.html >O administrador desta lista é >========================================================================= > TRANSIRE SVVM PECTVS MVNDOQUE POTIRE CONGREGATI EX TOTO ORBE MATHEMATICI OB SCRIPTA INSIGNIA TRIBVERE Medalha Fields(John Charles Fields) ------------------------------------------ Use o melhor sistema de busca da Internet Radar UOL - http://www.radaruol.com.br ========================================================================= Instruções para entrar na lista, sair da lista e usar a lista em http://www.mat.puc-rio.br/~nicolau/olimp/obm-l.html O administrador desta lista é ========================================================================= From owner-obm-l@sucuri.mat.puc-rio.br Wed May 29 14:17:29 2002 Return-Path: Received: (from majordom@localhost) by sucuri.mat.puc-rio.br (8.9.3/8.9.3) id OAA06941 for obm-l-list; Wed, 29 May 2002 14:15:48 -0300 Received: from www.zipmail.com.br (smtp.zipmail.com.br [200.187.242.10]) by sucuri.mat.puc-rio.br (8.9.3/8.9.3) with ESMTP id OAA06937 for ; Wed, 29 May 2002 14:15:44 -0300 From: peterdirichlet@zipmail.com.br Received: from [200.206.103.3] by www.zipmail.com.br with HTTP; Wed, 29 May 2002 14:04:08 -0300 Message-ID: <3CF3C4B4000014C0@www.zipmail.com.br> Date: Wed, 29 May 2002 14:04:08 -0300 In-Reply-To: <20020528232652.50585.qmail@web13301.mail.yahoo.com> Subject: [obm-l] =?iso-8859-1?Q?Re=3A=20=5Bobm=2Dl=5DD=FAvida=3A=20Novidades=20no=20site=20da=20OPM?= To: obm-l@mat.puc-rio.br, =?iso-8859-1?Q?Carlos=20Shine?= MIME-Version: 1.0 Content-Type: text/plain; charset="iso-8859-1" Content-Transfer-Encoding: 8bit X-MIME-Autoconverted: from quoted-printable to 8bit by sucuri.mat.puc-rio.br id OAA06938 Sender: owner-obm-l@sucuri.mat.puc-rio.br Precedence: bulk Reply-To: obm-l@mat.puc-rio.br Shine,estou com uma duvida:como uma pessoa cujo colegio nao a cadastrou pode fazer a prova da OPM?E que um colega meu vai participar este ano. ATE MAIS!!!Anderson -- Mensagem original -- >Oi gente! > >O site da OPM está com novidades! > >Uma delas é que estamos oferecendo mais um curso do >nosso Programa de Aperfeiçoamento em Matemática. >Alunos e professores estão convidados! > >Cheque em > http://www.opm.mat.br/ > >[]'s >Shine > >__________________________________________________ >Do You Yahoo!? >Yahoo! - Official partner of 2002 FIFA World Cup >http://fifaworldcup.yahoo.com >========================================================================= >Instruções para entrar na lista, sair da lista e usar a lista em >http://www.mat.puc-rio.br/~nicolau/olimp/obm-l.html >O administrador desta lista é >========================================================================= > TRANSIRE SVVM PECTVS MVNDOQUE POTIRE CONGREGATI EX TOTO ORBE MATHEMATICI OB SCRIPTA INSIGNIA TRIBVERE Medalha Fields(John Charles Fields) ------------------------------------------ Use o melhor sistema de busca da Internet Radar UOL - http://www.radaruol.com.br ========================================================================= Instruções para entrar na lista, sair da lista e usar a lista em http://www.mat.puc-rio.br/~nicolau/olimp/obm-l.html O administrador desta lista é ========================================================================= From owner-obm-l@sucuri.mat.puc-rio.br Wed May 29 15:15:05 2002 Return-Path: Received: (from majordom@localhost) by sucuri.mat.puc-rio.br (8.9.3/8.9.3) id PAA08493 for obm-l-list; Wed, 29 May 2002 15:12:54 -0300 Received: from calhau.terra.com.br (calhau.terra.com.br [200.176.3.20]) by sucuri.mat.puc-rio.br (8.9.3/8.9.3) with ESMTP id PAA08489 for ; Wed, 29 May 2002 15:12:49 -0300 Received: from smtp4-poa.terra.com.br (smtp4-poa.terra.com.br [200.176.3.35]) by calhau.terra.com.br (Postfix) with ESMTP id 622604778F for ; Wed, 29 May 2002 18:01:17 +0000 (GMT) Received: from nt (RJ231080.user.veloxzone.com.br [200.165.231.80]) (authenticated user ensr) by smtp4-poa.terra.com.br (Postfix) with ESMTP id A2E7AAC632 for ; Wed, 29 May 2002 15:01:16 -0300 (EST) Message-ID: <002b01c2073a$927f2960$5400a8c0@ensrbr> From: "Luis Lopes" To: References: Subject: Re: [obm-l] unicidade de polinomios Date: Wed, 29 May 2002 14:59:25 -0300 MIME-Version: 1.0 Content-Type: text/plain; charset="iso-8859-1" Content-Transfer-Encoding: 8bit X-Priority: 3 X-MSMail-Priority: Normal X-Mailer: Microsoft Outlook Express 5.00.2615.200 X-MimeOLE: Produced By Microsoft MimeOLE V5.00.2615.200 Sender: owner-obm-l@sucuri.mat.puc-rio.br Precedence: bulk Reply-To: obm-l@mat.puc-rio.br Sauda,c~oes, A demonstração que conheço e que está no livro de Indução que escrevi (problema 95) é meio delicada. Começa assim: Demonstre o seguinte teorema: um polinômio p(x) é identicamente nulo se, e somente se, todos os seus coeficientes são nulos. Observação: diz-se que um polinômio p(x) é identicamente nulo se, e somente se, p(x)=0 para todo x. Tendo demonstrado o teorema, cito e demonstro dois corolários. 1) dois polinômios p(x) e q(x) são idênticos se, e somente se, eles têm o mesmo grau e os coeficientes das potências de mesmo grau em x são iguais. 2) seja p(x) o polinômio a_nx^n + .... + a_1x + a_0. Então, podemos escrever p(x) como p(x)=a_n(x-r_1)....(x-r_n), onde os r_i são as raízes de p(x). Este resultado depende do resultado do problema 92. O uso do teorema do problema 95 é concluir que a_n tem que ser a_n (por ser o coeficiente de x^n nos dois polinômios). []'s Luis -----Mensagem Original----- De: Carlos Frederico Borges Palmeira Para: Enviada em: quarta-feira, 29 de maio de 2002 05:33 Assunto: Re: [obm-l] unicidade de polinomios > olhando com mais cuidado, vejo que escorreguei tambem. De fato se for > verdade para todo k de 0 a n-1, e' verdade para n. isto e' o que foi > demonstrado. so que n esta' fixo. como foi observado pelo arnaldo, nao da > nem para passar de 0 para 1. Uma alternativa para o passo de inducao e' > supondo verdade a igualdade dos coeficientes ate' k ,obtemos a igualdade > a(k+1)x^k+1 + a(k+2)x^k+2+...= b(k+1)x^k+1+... Dividindo por x^k+1 e > fazendo x=0, obtem-se a(k+1)=b(k+1). > Fred Palmeira > > On Wed, 29 May 2002, Carlos Frederico Borges Palmeira wrote: > > > oi gabriel, > > o sua professora de fato escorregou na utilizacao do principio da > > inducao. A forma usual e' provar que vale para n=0, e provar que se vale > > para n entao vale para n+1. Outra forma e' provar que vale para n=0 e > > provar que se vale para todo k de 0 a n , entao vale para n+1. > > Apaarentemente ela disse que ia usar a 1a e de fato usou a 2a. Como voce > > observou a 1a forma e' insuficiente para esta demonstracao. na passagem > > destacada abaixo, e' de fato necessario a 2a forma. > > []s > > Fred Palmeira > > > > > > > > > > > > On Tue, 28 May 2002 ghaeser@zipmail.com.br wrote: > > > > > Olá pessoal da lista, > > > me surgiu uma dúvida durante uma aula de análise que a professora nao conseguiu > > > tirar.. > > > > > > Seja f(x)=a0+a1x+..+anx^n, g(x)=b0+b1x+..+bnx^n polinômios de grau n, onde > > > f(x)=g(x) qualquer que seja x, prove que ai=bi para i=0,..,n .. para todo > > > n natural. > > > > > > demonstração da professora: > > > > > > seja h(x)=f(x)-g(x)=0 para todo x real, por hipotese > > > logo como h(0)=0 entao a0=b0 > > > por inducao, suponha que a(n-1)=b(n-1), logo, como h(1)=0 temos: > > > a0+..+a(n-1)+an=b0+..+b(n-1)+bn, > > para passar para a proxima linha precisa supor verdade para todo k de 0 a > > n. > > ************************************ > > como > > > a0+..+a(n-1)=b0+..+b(n-1), temos que an=bn > > **************************************** > > > > > > > > logo, por indução temos que vale para todo n > > > > > > minha dúvida é: > > > > > > seja um polinomio h(x) de grau n, onde h(0)=0 e h(1)=0, prove que a0=a1=..=an=0 > > > para todo n. > > > > > > obviamente isto é falso, mas eu consigo demonstrar utilizando a prova dela.. > > > por isso acho q tem alguma coisa errada com a hipotese de inducao .. talvez > > > deva haver uma inclusao da hipotese de haver n+1 zeros para o grau n .. > > > tentei explicar isto para ela, mas ela nao concordou .. será que alguém > > > pode me ajudar ?? > > > > > > muito obrigado !! > > > > > > Gabriel Haeser > > > www.gabas.cjb.net > > > > > > > > > "Mathematicus nascitur, non fit" > > > Matemáticos não são feitos, eles nascem > > > --------------------------------------- > > > Gabriel Haeser > > > www.gabas.cjb.net > > > > > > > > > ------------------------------------------ > > > Use o melhor sistema de busca da Internet > > > Radar UOL - http://www.radaruol.com.br > > > > > > > > > > > > ========================================================================= > > > Instruções para entrar na lista, sair da lista e usar a lista em > > > http://www.mat.puc-rio.br/~nicolau/olimp/obm-l.html > > > O administrador desta lista é > > > ========================================================================= > > > > > > > ========================================================================= > > Instruções para entrar na lista, sair da lista e usar a lista em > > http://www.mat.puc-rio.br/~nicolau/olimp/obm-l.html > > O administrador desta lista é > > ========================================================================= > > > > > ========================================================================= > Instruções para entrar na lista, sair da lista e usar a lista em > http://www.mat.puc-rio.br/~nicolau/olimp/obm-l.html > O administrador desta lista é > ========================================================================= > > ========================================================================= Instruções para entrar na lista, sair da lista e usar a lista em http://www.mat.puc-rio.br/~nicolau/olimp/obm-l.html O administrador desta lista é ========================================================================= From owner-obm-l@sucuri.mat.puc-rio.br Wed May 29 15:31:58 2002 Return-Path: Received: (from majordom@localhost) by sucuri.mat.puc-rio.br (8.9.3/8.9.3) id PAA08755 for obm-l-list; Wed, 29 May 2002 15:30:20 -0300 Received: from murici.ufu.br (murici.ufu.br [200.19.146.7]) by sucuri.mat.puc-rio.br (8.9.3/8.9.3) with ESMTP id PAA08751 for ; Wed, 29 May 2002 15:30:17 -0300 Received: from ufu.br ([200.131.206.62]) by murici.ufu.br (Post.Office MTA v3.5.4 release 224 ID# 124-61694U2500L250S0V35) with ESMTP id br for ; Wed, 29 May 2002 17:16:36 -0200 Message-ID: <3CF522EC.86EE06CF@ufu.br> Date: Wed, 29 May 2002 15:50:20 -0300 From: salomao@ufu.br (Luiz Alberto Duran Salomao) X-Mailer: Mozilla 4.08 [en] (Win98; I) MIME-Version: 1.0 To: obm-l@mat.puc-rio.br Subject: Re: [obm-l] Mais Informacoes sobre o Teorema de Riemann References: Content-Type: text/plain; charset=iso-8859-1 Content-Transfer-Encoding: 8bit Sender: owner-obm-l@sucuri.mat.puc-rio.br Precedence: bulk Reply-To: obm-l@mat.puc-rio.br Caro Paulo Santa Rita: No ultimo numero da revista Matematica Universitaria - SBM, saiu um artigo intitulado "Series Incondicionalmente Convergentes: de Dirichlet a Dvoretzki-Rogers" que talvez lhe interesse. Voce jah o conhece ? Um abraco a todos, Luiz Alberto Paulo Santa Rita wrote: > Ola Pessoal, > > Eu estou precisando de mais informacoes sobre o TEOREMA DE RIEMANN acerca > das series condicionalmente convergentes. O que todos os livros de Analise > que eu conheco ( e sao muitos ! ) falam e insuficiente para os meus > propositos : eles tao somente enunciam o teorema e mostram aplicacoes e > exemplos elementares e/ou classicos, deixando em aberto uma enormidade de > perguntas consistentes que podem ser feitas. Sera que alguem conhece algum > estudo mais aprofundado sobre este Teorema, tal como um artigo ou uma Tese > de Mestrado ou de Doutorado ? > > O Teorema de Riemann ao qual me refiro e aquele que diz que uma serie > condicionalmente convergente pode divergir ou assumir um valor qualquer > atraves de um rearranjo dos seus termos. > > Desde ja agradeco qualquer ajuda > Um Grande abraco a todos > Paulo Santa Rita > 3,1255,280502 > > _________________________________________________________________ > Chegou o novo MSN Explorer. Instale já. É gratuito: > http://explorer.msn.com.br > > ========================================================================= > Instruções para entrar na lista, sair da lista e usar a lista em > http://www.mat.puc-rio.br/~nicolau/olimp/obm-l.html > O administrador desta lista é > ========================================================================= ========================================================================= Instruções para entrar na lista, sair da lista e usar a lista em http://www.mat.puc-rio.br/~nicolau/olimp/obm-l.html O administrador desta lista é ========================================================================= From owner-obm-l@sucuri.mat.puc-rio.br Wed May 29 15:47:01 2002 Return-Path: Received: (from majordom@localhost) by sucuri.mat.puc-rio.br (8.9.3/8.9.3) id PAA09403 for obm-l-list; Wed, 29 May 2002 15:46:17 -0300 Received: from www.zipmail.com.br (smtp.zipmail.com.br [200.187.242.10]) by sucuri.mat.puc-rio.br (8.9.3/8.9.3) with ESMTP id PAA09399 for ; Wed, 29 May 2002 15:46:15 -0300 From: peterdirichlet@zipmail.com.br Received: from [200.206.103.3] by www.zipmail.com.br with HTTP; Wed, 29 May 2002 15:34:24 -0300 Message-ID: <3CF3C4B4000017D3@www.zipmail.com.br> Date: Wed, 29 May 2002 16:34:24 -0200 Subject: [obm-l] =?iso-8859-1?Q?Duvidas=20diversas=20sobre=20diversos?= To: =?iso-8859-1?Q?Lista=20de=20Discussao?= , =?iso-8859-1?Q?Olimpiada=20Brasileira=20de=20Matem=2E?= , =?iso-8859-1?Q?Paulo=20Rodrigues?= MIME-Version: 1.0 Content-Type: text/plain; charset="iso-8859-1" Content-Transfer-Encoding: 8bit X-MIME-Autoconverted: from quoted-printable to 8bit by sucuri.mat.puc-rio.br id PAA09400 Sender: owner-obm-l@sucuri.mat.puc-rio.br Precedence: bulk Reply-To: obm-l@mat.puc-rio.br Ola turma!!!Faz um bom tempo que eu nao escrevo para a lista da OBM.E vou chegar metendo bala: 01)Sejam a,b,c,d reais nao negativos tais que ab+bc+cd+da=1.Prove que (a^3/b+c+d)+(b^3/a+c+d)+(c^3/a+b+d)+(d^3/a+b+c)>=1/3 e determine a igualdade. 02)Considere uma sequencia de inteiros positivos a_1,a_2,a_3... tal que a_(n+1) nao possa ser escrito como combinaçao linear dos termos anteriores.E possivel que ela seja infinita? 03)Num triangulo ABC a bissetriz do angulo A corta BC em A' e corta o circuncirculo em A".Seja v(A)=AA'/(BA"+CA").Definimos analogamente v(b) e v(c).Prove que 4*(v(a)+v(b)+v(c))>=3 04)Seja f_n o ultimo algarismo diferente de zero de n!.Sera que a_1,a_2,...,a_n fica periodica a partir de certo ponto? 05)Encontre todos os naturais n tais que para qualquer par de divisores a e b(primos entre si),n seja multiplo de a+b-1 06)Seja n um inteiro maior que 1.Prove que se P(x)=x^2+x+n e primo para todos os inteiros x entre 0 e (n/3)^1/2,entao P sera primo com x de 0 ate n-2. prove que o conjunto dos reais x tais que 1/(x-1)+2/(x-2)+3/(x-3)+...+70/(x-70)>=5/4 e uma soma de intervalos disjuntos de soma 1988.Generalize. 07)N,P e Q sao inteiros tais que N>P+Q.Construa uma sequencia de N termos tal que o primeiro e o ultimo termo sao nulos e a diferença entre um termo e seu antecessor seja P ou -Q.Prove que existe um par de termos dessa sequencia(que nao o par de extremos)cujos valores sao iguais. Quem tiver paciencia para me dizer o que fazer nestes problemas,agradeço.Peterdirichlet TRANSIRE SVVM PECTVS MVNDOQUE POTIRE CONGREGATI EX TOTO ORBE MATHEMATICI OB SCRIPTA INSIGNIA TRIBVERE Medalha Fields(John Charles Fields) ------------------------------------------ Use o melhor sistema de busca da Internet Radar UOL - http://www.radaruol.com.br ========================================================================= Instruções para entrar na lista, sair da lista e usar a lista em http://www.mat.puc-rio.br/~nicolau/olimp/obm-l.html O administrador desta lista é ========================================================================= From owner-obm-l@sucuri.mat.puc-rio.br Wed May 29 16:31:31 2002 Return-Path: Received: (from majordom@localhost) by sucuri.mat.puc-rio.br (8.9.3/8.9.3) id QAA10835 for obm-l-list; Wed, 29 May 2002 16:30:42 -0300 Received: from hotmail.com (law2-f127.hotmail.com [216.32.181.127]) by sucuri.mat.puc-rio.br (8.9.3/8.9.3) with ESMTP id QAA10830 for ; Wed, 29 May 2002 16:30:39 -0300 Received: from mail pickup service by hotmail.com with Microsoft SMTPSVC; Wed, 29 May 2002 12:19:07 -0700 Received: from 200.229.244.100 by lw2fd.hotmail.msn.com with HTTP; Wed, 29 May 2002 19:19:06 GMT X-Originating-IP: [200.229.244.100] From: "Paulo Santa Rita" To: obm-l@mat.puc-rio.br Subject: Re: [obm-l] Mais Informacoes sobre o Teorema de Riemann Date: Wed, 29 May 2002 19:19:06 +0000 Mime-Version: 1.0 Content-Type: text/plain; charset=iso-8859-1; format=flowed Message-ID: X-OriginalArrivalTime: 29 May 2002 19:19:07.0103 (UTC) FILETIME=[B3D53EF0:01C20745] Sender: owner-obm-l@sucuri.mat.puc-rio.br Precedence: bulk Reply-To: obm-l@mat.puc-rio.br Carissimo Salomao, Nao, nao conheco o artigo. Mas com certeza vou correndo procura-lo e passar o feriado estudando-o em detalhes. E crucial que eu compile tudo sobre esse tema pois percebi algo maravilhoso que nao consigo mostrar por causa de minhas deficiencias ou porque o que eu preciso ainda nao existe. Muitissimo obrigado ! Que esta luz, Deus lhe retriua em dobro ! Com os melhores votos de paz profunda, sou Paulo Santa Rita 4,1616,290502 >From: salomao@ufu.br (Luiz Alberto Duran Salomao) >Reply-To: obm-l@mat.puc-rio.br >To: obm-l@mat.puc-rio.br >Subject: Re: [obm-l] Mais Informacoes sobre o Teorema de Riemann >Date: Wed, 29 May 2002 15:50:20 -0300 > >Caro Paulo Santa Rita: >No ultimo numero da revista Matematica Universitaria - SBM, saiu um artigo >intitulado "Series Incondicionalmente Convergentes: de Dirichlet a >Dvoretzki-Rogers" >que talvez lhe interesse. Voce jah o conhece ? >Um abraco a todos, >Luiz Alberto > >Paulo Santa Rita wrote: > > > Ola Pessoal, > > > > Eu estou precisando de mais informacoes sobre o TEOREMA DE RIEMANN >acerca > > das series condicionalmente convergentes. O que todos os livros de >Analise > > que eu conheco ( e sao muitos ! ) falam e insuficiente para os meus > > propositos : eles tao somente enunciam o teorema e mostram aplicacoes e > > exemplos elementares e/ou classicos, deixando em aberto uma enormidade >de > > perguntas consistentes que podem ser feitas. Sera que alguem conhece >algum > > estudo mais aprofundado sobre este Teorema, tal como um artigo ou uma >Tese > > de Mestrado ou de Doutorado ? > > > > O Teorema de Riemann ao qual me refiro e aquele que diz que uma serie > > condicionalmente convergente pode divergir ou assumir um valor qualquer > > atraves de um rearranjo dos seus termos. > > > > Desde ja agradeco qualquer ajuda > > Um Grande abraco a todos > > Paulo Santa Rita > > 3,1255,280502 > > > > _________________________________________________________________ > > Chegou o novo MSN Explorer. Instale já. É gratuito: > > http://explorer.msn.com.br > > > > >========================================================================= > > Instruções para entrar na lista, sair da lista e usar a lista em > > http://www.mat.puc-rio.br/~nicolau/olimp/obm-l.html > > O administrador desta lista é > > >========================================================================= > >========================================================================= >Instruções para entrar na lista, sair da lista e usar a lista em >http://www.mat.puc-rio.br/~nicolau/olimp/obm-l.html >O administrador desta lista é >========================================================================= _________________________________________________________________ Envie e receba emails com o Hotmail no seu dispositivo móvel: http://mobile.msn.com ========================================================================= Instruções para entrar na lista, sair da lista e usar a lista em http://www.mat.puc-rio.br/~nicolau/olimp/obm-l.html O administrador desta lista é ========================================================================= From owner-obm-l@sucuri.mat.puc-rio.br Wed May 29 16:47:16 2002 Return-Path: Received: (from majordom@localhost) by sucuri.mat.puc-rio.br (8.9.3/8.9.3) id QAA11140 for obm-l-list; Wed, 29 May 2002 16:46:48 -0300 Received: from hotmail.com (oe43.law7.hotmail.com [216.33.236.79]) by sucuri.mat.puc-rio.br (8.9.3/8.9.3) with ESMTP id QAA11136 for ; Wed, 29 May 2002 16:46:45 -0300 Received: from mail pickup service by hotmail.com with Microsoft SMTPSVC; Wed, 29 May 2002 12:35:13 -0700 X-Originating-IP: [200.241.240.154] From: "Marcelo Rufino de Oliveira" To: References: <3CF3C4B4000017D3@www.zipmail.com.br> Subject: Re: [obm-l] Duvidas diversas sobre diversos Date: Wed, 29 May 2002 16:41:17 -0300 MIME-Version: 1.0 Content-Type: text/plain; charset="iso-8859-1" Content-Transfer-Encoding: 8bit X-Priority: 3 X-MSMail-Priority: Normal X-Mailer: Microsoft Outlook Express 5.00.2314.1300 X-MimeOLE: Produced By Microsoft MimeOLE V5.00.2314.1300 Message-ID: X-OriginalArrivalTime: 29 May 2002 19:35:13.0128 (UTC) FILETIME=[F3A0F680:01C20747] Sender: owner-obm-l@sucuri.mat.puc-rio.br Precedence: bulk Reply-To: obm-l@mat.puc-rio.br > Ola turma!!!Faz um bom tempo que eu nao escrevo para a lista da OBM.E vou > chegar metendo bala: > 01)Sejam a,b,c,d reais nao negativos tais que ab+bc+cd+da=1.Prove > que (a^3/b+c+d)+(b^3/a+c+d)+(c^3/a+b+d)+(d^3/a+b+c)>=1/3 e determine a igualdade. Seja A = b + c + d, B = a + c + d, C = a + b + d e D = a + b + c. Como (a - b)^2 + (b - c)^2 + (c - d)^2 + (d - a)^2 >= 0 => a^2 + b^2 + c^2 + d^2 >= ab + bc + cd + da = 1 (1) Suponha que a >= b >= c >= d => a^3 >= b^3 >= c^3 >= d^3 e 1/A >= 1/B >= 1/C >= 1/D. Assim, pela desigualdade de Chebyshev: a^3/A + b^3/B + c^3/C + d^3/D >= (a^3 + b^3 + c^3 + d^3)(1/A + 1/B + 1/C + 1/D)/4 (2) Também pela desigualdade de Chebyshev: a^3 + b^3 + c^3 + d^3 >= (a^2 + b^2 + c^2 + d^2)(a + b + c + d)/4 (3) Note também que 3(a + b + c + d) = A + B + C + D (4) Pela desigualdade entre as médias aritmética e geométrica temos que (A + B + C + D)(1/A + 1/B + 1/C + 1/D) >= 16 (5) (1), (2), (3), (4) e (5) => a^3/A + b^3/B + c^3/C + d^3/D >= (A + B + C + D)(1/A + 1/B + 1/C + 1/D)/48 >= 16/48 = 3 A igualdade ocorre quando A = B = C = D => a = b = c = d. Até mais, Marcelo Rufino de Oliveira ========================================================================= Instruções para entrar na lista, sair da lista e usar a lista em http://www.mat.puc-rio.br/~nicolau/olimp/obm-l.html O administrador desta lista é ========================================================================= From owner-obm-l@sucuri.mat.puc-rio.br Thu May 30 00:55:07 2002 Return-Path: Received: (from majordom@localhost) by sucuri.mat.puc-rio.br (8.9.3/8.9.3) id AAA16105 for obm-l-list; Thu, 30 May 2002 00:54:51 -0300 Received: from smtp-5.ig.com.br (smtp-5.ig.com.br [200.226.132.154]) by sucuri.mat.puc-rio.br (8.9.3/8.9.3) with SMTP id AAA16101 for ; Thu, 30 May 2002 00:54:49 -0300 From: ezer@ig.com.br Received: (qmail 12571 invoked from network); 30 May 2002 03:43:02 -0000 Received: from shasta026173.ig.com.br (HELO house) (200.151.26.173) by smtp-5.ig.com.br with SMTP; 30 May 2002 03:43:02 -0000 To: obm-l@mat.puc-rio.br Date: Wed, 1 Jan 1997 00:36:24 -0200 MIME-Version: 1.0 Subject: [obm-l] =?ISO-8859-1?Q?Progress=E1lise_Combitm=E9tica?= Message-ID: <32C9B188.7346.13EDC9@localhost> X-mailer: Pegasus Mail for Windows (v4.01) Content-type: text/plain; charset=ISO-8859-1 Content-description: Mail message body Content-Transfer-Encoding: 8bit X-MIME-Autoconverted: from Quoted-printable to 8bit by sucuri.mat.puc-rio.br id AAA16102 Sender: owner-obm-l@sucuri.mat.puc-rio.br Precedence: bulk Reply-To: obm-l@mat.puc-rio.br Olá pessoal, Eu estava pensando num problema que eu me formulei de Analise Combinatoria, e de Progressao Aritmetica: Quantos sao os numeros de quatro algarismos que estao em Progressao Aritmetica com o seu vizinho (p. ex., 2468 serve, 4286, nao) Desde jah agradeco, =) Ezer ========================================================================= Instruções para entrar na lista, sair da lista e usar a lista em http://www.mat.puc-rio.br/~nicolau/olimp/obm-l.html O administrador desta lista é ========================================================================= From owner-obm-l@sucuri.mat.puc-rio.br Thu May 30 11:52:32 2002 Return-Path: Received: (from majordom@localhost) by sucuri.mat.puc-rio.br (8.9.3/8.9.3) id LAA20767 for obm-l-list; Thu, 30 May 2002 11:49:58 -0300 Received: from hotmail.com (f207.pav1.hotmail.com [64.4.31.207]) by sucuri.mat.puc-rio.br (8.9.3/8.9.3) with ESMTP id LAA20740 for ; Thu, 30 May 2002 11:48:47 -0300 Received: from mail pickup service by hotmail.com with Microsoft SMTPSVC; Thu, 30 May 2002 07:37:11 -0700 Received: from 200.151.11.94 by pv1fd.pav1.hotmail.msn.com with HTTP; Thu, 30 May 2002 14:37:02 GMT X-Originating-IP: [200.151.11.94] From: "Adriano Almeida Faustino" To: obm-l@mat.puc-rio.br Subject: Re: [obm-l] Analise Combinatoria Date: Thu, 30 May 2002 14:37:02 +0000 Mime-Version: 1.0 Content-Type: text/plain; charset=iso-8859-1; format=flowed Message-ID: X-OriginalArrivalTime: 30 May 2002 14:37:11.0561 (UTC) FILETIME=[7BCC1F90:01C207E7] Sender: owner-obm-l@sucuri.mat.puc-rio.br Precedence: bulk Reply-To: obm-l@mat.puc-rio.br O que fez praticamente fez foi o 1ºlema de Kaplansky ( C(n-p+1,p) ),para p=3 ?E o que adiantou ele falar ``dois` ou tres alunos` ?,o que esse `dois` esta influindo? []`s Adriano. >From: Augusto Cesar de Oliveira Morgado >Reply-To: obm-l@mat.puc-rio.br >To: obm-l@mat.puc-rio.br >Subject: Re: [obm-l] Analise Combinatoria >Date: Mon, 27 May 2002 11:10:13 -0300 (EST) > > >Uma solucao mais elementar seria imaginar os alunos 1 2 ... n e marcar com >o sinal de + os escolhidos e com o sinal - os não escolhidos. Formaremos >uma fila com 3 sinais + e n-3 sinais -, nao podendo haver dois sinais + >consecutivos. Para isso, ponha os n-3 sinais - em fila e vejamos de quantos >modos podemos enfiar entre eles (ou antes do primeiro ou depois do ultimo) >os sinais +. >Sao n-2 espaços dos quais devemos escolher 3 e a resposta eh C(n-2,2). > >Em Mon, 27 May 2002 00:59:54 -0300, Paulo Rodrigues >disse: > > > : Considere uma turma com n alunos ,numerados de 1 a n. > > : Deseja-se organizar uma comissao de 3 alunos.De quantas maneiras pode >ser > > : formada esta comissao,de modo que nao facam parte da mesma dois ou >tres > > : alunosdesignados por numeros consecutivos ? > > > > Seja C={x, y, z} uma comissão satisfazendo às condições do problema, com > > xpois > > z > y +1 > x+2. C1 é necessariamente um subconjunto de >[n-2]={1,2,...,n-2} > > e prova-se facilmente que essa função que leva C em C1 é uma bijeção do > > conjunto considerado no conjunto dos 3-subconjuntos de [n-2]. Portanto, >o > > número de subconjuntos C é igual ao número de subconjuntos C1, igual a > > binomial(n-2,3) = (n-2)(n-3)(n-4)/6. > > > > > > --- > > esta mensagem não contém vírus! > > Checked by AVG anti-virus system (http://www.grisoft.com). > > Version: 6.0.363 / Virus Database: 201 - Release Date: 21/05/2002 > > > > >========================================================================= > > Instruções para entrar na lista, sair da lista e usar a lista em > > http://www.mat.puc-rio.br/~nicolau/olimp/obm-l.html > > O administrador desta lista é > > >========================================================================= > > > > > >========================================================================= >Instruções para entrar na lista, sair da lista e usar a lista em >http://www.mat.puc-rio.br/~nicolau/olimp/obm-l.html >O administrador desta lista é >========================================================================= _________________________________________________________________ Chegou o novo MSN Explorer. Instale já. É gratuito: http://explorer.msn.com.br ========================================================================= Instruções para entrar na lista, sair da lista e usar a lista em http://www.mat.puc-rio.br/~nicolau/olimp/obm-l.html O administrador desta lista é ========================================================================= From owner-obm-l@sucuri.mat.puc-rio.br Thu May 30 15:50:30 2002 Return-Path: Received: (from majordom@localhost) by sucuri.mat.puc-rio.br (8.9.3/8.9.3) id PAA23142 for obm-l-list; Thu, 30 May 2002 15:47:29 -0300 Received: from web14802.mail.yahoo.com (web14802.mail.yahoo.com [216.136.224.218]) by sucuri.mat.puc-rio.br (8.9.3/8.9.3) with SMTP id PAA23138 for ; Thu, 30 May 2002 15:47:26 -0300 Message-ID: <20020530183557.88051.qmail@web14802.mail.yahoo.com> Received: from [200.243.206.94] by web14802.mail.yahoo.com via HTTP; Thu, 30 May 2002 15:35:57 ART Date: Thu, 30 May 2002 15:35:57 -0300 (ART) From: =?iso-8859-1?q?Ricardo=20Miranda?= Subject: Re: [obm-l] Progressálise_Combitmética + Duvidas sobre Logica Matematica To: obm-l@mat.puc-rio.br In-Reply-To: <32C9B188.7346.13EDC9@localhost> MIME-Version: 1.0 Content-Type: text/plain; charset=iso-8859-1 Content-Transfer-Encoding: 8bit Sender: owner-obm-l@sucuri.mat.puc-rio.br Precedence: bulk Reply-To: obm-l@mat.puc-rio.br --- ezer@ig.com.br escreveu: > Olá pessoal, > > Eu estava pensando num problema que eu me formulei > de Analise Combinatoria, e de Progressao Aritmetica: > > Quantos sao os numeros de quatro algarismos que > estao em Progressao Aritmetica com o seu vizinho > (p. ex., 2468 serve, 4286, nao) > > Desde jah agradeco, =) > > Ezer As razoes podem ser 1 e 2. Razao 3 nao dá, "1 4 7 0" r=1 1234 2345 3456 4567 5678 6789 r=2 1357 2468 3579 Total, 9. Nao sei se está certo, e nao consegui pensar em uma solucao algébrica. Gostariam que me ajudassem nessas questoes: 1) Determinar o valor lógico da proposicao: "A expressao n^2-n+41 só produz numeros primos". Como eu posso provar que para TODOS os valores de n o resultado será um numero primo? 2) Simplificar a seguinte proposicao e indicar em cima de cada simbolo de equivalencia a propriedade logica utilizada. ~((~P -> ~Q) OU ((Q E P) <-> ~P)) ===== []s Ricardo Miranda ricardomirandabr@yahoo.com.br http://rm2.hpg.ig.com.br/ _______________________________________________________________________ Yahoo! Encontros O lugar certo para você encontrar aquela pessoa que falta na sua vida. Cadastre-se hoje mesmo! http://br.encontros.yahoo.com/ ========================================================================= Instruções para entrar na lista, sair da lista e usar a lista em http://www.mat.puc-rio.br/~nicolau/olimp/obm-l.html O administrador desta lista é ========================================================================= From owner-obm-l@sucuri.mat.puc-rio.br Thu May 30 15:52:33 2002 Return-Path: Received: (from majordom@localhost) by sucuri.mat.puc-rio.br (8.9.3/8.9.3) id PAA23179 for obm-l-list; Thu, 30 May 2002 15:49:57 -0300 Received: from gorgo.centroin.com.br (gorgo.centroin.com.br [200.225.63.128]) by sucuri.mat.puc-rio.br (8.9.3/8.9.3) with ESMTP id PAA23175 for ; Thu, 30 May 2002 15:49:56 -0300 Received: from centroin.com.br (du118b.rjo.centroin.com.br [200.225.57.118]) (authenticated bits=0) by gorgo.centroin.com.br (8.12.2/8.12.1) with ESMTP id g4UIcsOA024547 for ; Thu, 30 May 2002 15:38:54 -0300 (BRT) Message-ID: <3CF67203.8020700@centroin.com.br> Date: Thu, 30 May 2002 15:40:03 -0300 From: Augusto =?ISO-8859-1?Q?C=E9sar?= Morgado User-Agent: Mozilla/5.0 (Windows; U; Win98; en-US; rv:0.9.4.1) Gecko/20020508 Netscape6/6.2.3 X-Accept-Language: en-us MIME-Version: 1.0 To: obm-l@mat.puc-rio.br Subject: Re: [obm-l] Analise Combinatoria References: Content-Type: text/plain; charset=ISO-8859-1; format=flowed Content-Transfer-Encoding: 8bit Sender: owner-obm-l@sucuri.mat.puc-rio.br Precedence: bulk Reply-To: obm-l@mat.puc-rio.br Claro, o que eu fiz foi deduzir localmente o lema de Kaplansky. Mas nao entendi o final do seu comentario. O que seria dispensavel no enunciado da questao seria o "tres" e nao o dois. Morgado Adriano Almeida Faustino wrote: > O que fez praticamente fez foi o 1ºlema de Kaplansky ( C(n-p+1,p) > ),para p=3 ?E o que adiantou ele falar ``dois` ou tres alunos` ?,o que > esse `dois` esta influindo? > []`s > Adriano. > > >> From: Augusto Cesar de Oliveira Morgado >> Reply-To: obm-l@mat.puc-rio.br >> To: obm-l@mat.puc-rio.br >> Subject: Re: [obm-l] Analise Combinatoria >> Date: Mon, 27 May 2002 11:10:13 -0300 (EST) >> >> >> Uma solucao mais elementar seria imaginar os alunos 1 2 ... n e >> marcar com o sinal de + os escolhidos e com o sinal - os não >> escolhidos. Formaremos uma fila com 3 sinais + e n-3 sinais -, nao >> podendo haver dois sinais + consecutivos. Para isso, ponha os n-3 >> sinais - em fila e vejamos de quantos modos podemos enfiar entre eles >> (ou antes do primeiro ou depois do ultimo) os sinais +. >> Sao n-2 espaços dos quais devemos escolher 3 e a resposta eh C(n-2,2). >> >> Em Mon, 27 May 2002 00:59:54 -0300, Paulo Rodrigues >> disse: >> >> > : Considere uma turma com n alunos ,numerados de 1 a n. >> > : Deseja-se organizar uma comissao de 3 alunos.De quantas maneiras >> pode ser >> > : formada esta comissao,de modo que nao facam parte da mesma dois >> ou tres >> > : alunosdesignados por numeros consecutivos ? >> > >> > Seja C={x, y, z} uma comissão satisfazendo às condições do >> problema, com >> > x> elementos pois >> > z > y +1 > x+2. C1 é necessariamente um subconjunto de >> [n-2]={1,2,...,n-2} >> > e prova-se facilmente que essa função que leva C em C1 é uma >> bijeção do >> > conjunto considerado no conjunto dos 3-subconjuntos de [n-2]. >> Portanto, o >> > número de subconjuntos C é igual ao número de subconjuntos C1, igual a >> > binomial(n-2,3) = (n-2)(n-3)(n-4)/6. >> > >> > >> > --- >> > esta mensagem não contém vírus! >> > Checked by AVG anti-virus system (http://www.grisoft.com). >> > Version: 6.0.363 / Virus Database: 201 - Release Date: 21/05/2002 >> > >> > >> ========================================================================= >> >> > Instruções para entrar na lista, sair da lista e usar a lista em >> > http://www.mat.puc-rio.br/~nicolau/olimp/obm-l.html >> > O administrador desta lista é >> > >> ========================================================================= >> >> > >> > >> >> ========================================================================= >> >> Instruções para entrar na lista, sair da lista e usar a lista em >> http://www.mat.puc-rio.br/~nicolau/olimp/obm-l.html >> O administrador desta lista é >> ========================================================================= >> > > > > _________________________________________________________________ > Chegou o novo MSN Explorer. Instale já. É gratuito: > http://explorer.msn.com.br > > ========================================================================= > Instruções para entrar na lista, sair da lista e usar a lista em > http://www.mat.puc-rio.br/~nicolau/olimp/obm-l.html > O administrador desta lista é > ========================================================================= > > ========================================================================= Instruções para entrar na lista, sair da lista e usar a lista em http://www.mat.puc-rio.br/~nicolau/olimp/obm-l.html O administrador desta lista é ========================================================================= From owner-obm-l@sucuri.mat.puc-rio.br Thu May 30 18:37:18 2002 Return-Path: Received: (from majordom@localhost) by sucuri.mat.puc-rio.br (8.9.3/8.9.3) id SAA25098 for obm-l-list; Thu, 30 May 2002 18:34:39 -0300 Received: from ginsberg.uol.com.br (ginsberg.uol.com.br [200.231.206.26]) by sucuri.mat.puc-rio.br (8.9.3/8.9.3) with ESMTP id SAA25094 for ; Thu, 30 May 2002 18:34:37 -0300 Received: from computador ([200.213.211.193]) by ginsberg.uol.com.br (8.9.1/8.9.1) with SMTP id SAA21364 for ; Thu, 30 May 2002 18:22:01 -0300 (BRT) From: =?iso-8859-1?Q?Moacyr_Rodrigues_J=FAnior?= To: Subject: [obm-l] =?iso-8859-1?Q?RES:_=5Bobm-l=5D_Progress=E1lise=5FCombitm=E9tica_+_Du?= =?iso-8859-1?Q?vidas_sobre_Logica_Matematica?= Date: Thu, 30 May 2002 18:22:55 -0300 Message-ID: MIME-Version: 1.0 Content-Type: text/plain; charset="iso-8859-1" Content-Transfer-Encoding: 8bit X-Priority: 3 (Normal) X-MSMail-Priority: Normal X-Mailer: Microsoft Outlook IMO, Build 9.0.2416 (9.0.2910.0) In-Reply-To: <20020530183557.88051.qmail@web14802.mail.yahoo.com> X-MimeOLE: Produced By Microsoft MimeOLE V6.00.2600.0000 Importance: Normal Sender: owner-obm-l@sucuri.mat.puc-rio.br Precedence: bulk Reply-To: obm-l@mat.puc-rio.br 1) Determinar o valor lógico da proposicao: "A expressao n^2-n+41 só produz numeros primos". Como eu posso provar que para TODOS os valores de n o resultado será um numero primo? Você não pode provar! Pois para n=41 a expressão tem valor numérico 41^2 que não é primo. ========================================================================= Instruções para entrar na lista, sair da lista e usar a lista em http://www.mat.puc-rio.br/~nicolau/olimp/obm-l.html O administrador desta lista é ========================================================================= From owner-obm-l@sucuri.mat.puc-rio.br Thu May 30 20:44:03 2002 Return-Path: Received: (from majordom@localhost) by sucuri.mat.puc-rio.br (8.9.3/8.9.3) id UAA26281 for obm-l-list; Thu, 30 May 2002 20:42:48 -0300 Received: from wool.vetor.com.br (200.160.244.7.metrored.net.br [200.160.244.7] (may be forged)) by sucuri.mat.puc-rio.br (8.9.3/8.9.3) with ESMTP id UAA26277 for ; Thu, 30 May 2002 20:42:46 -0300 Received: (from root@localhost) by wool.vetor.com.br (8.11.4/8.11.4) id g4UNUNl14392 for obm-l@mat.puc-rio.br; Thu, 30 May 2002 20:30:23 -0300 Received: from rodrigo (dl-ct-ip182.wb.com.br [200.160.244.182] (may be forged)) by wool.vetor.com.br (8.11.4/8.11.4) with SMTP id g4UNULg14384 for ; Thu, 30 May 2002 20:30:22 -0300 Message-ID: <003d01c20832$00c08840$b6f4a0c8@rodrigo> From: "Rodrigo Villard Milet" To: "Obm" Subject: [obm-l] Caminhos no quadriculado Date: Thu, 30 May 2002 20:30:35 -0300 MIME-Version: 1.0 Content-Type: multipart/alternative; boundary="----=_NextPart_000_003A_01C20818.DA66C240" X-Priority: 3 X-MSMail-Priority: Normal X-Mailer: Microsoft Outlook Express 4.72.3110.5 X-MimeOLE: Produced By Microsoft MimeOLE V4.72.3110.3 X-Virus-Scanned: by AMaViS perl-11 Sender: owner-obm-l@sucuri.mat.puc-rio.br Precedence: bulk Reply-To: obm-l@mat.puc-rio.br This is a multi-part message in MIME format. ------=_NextPart_000_003A_01C20818.DA66C240 Content-Type: text/plain; charset="iso-8859-1" Content-Transfer-Encoding: quoted-printable Gostaria de saber se h=E1 uma f=F3rmula fechada para a resposta desse = problema em fun=E7=E3o de n : Dado um quadriculado n x n, quantos s=E3o os caminhos que saem do canto = inferior esquerdo e chegam ao canto superior direito, de forma que o = caminho n=E3o passe duas vezes pelo mesmo lugar. O caminho s=F3 pode ser = feito sobre os lados dos quadradinhos 1x1 do quadriculado. Abra=E7os, =20 Villard ------=_NextPart_000_003A_01C20818.DA66C240 Content-Type: text/html; charset="iso-8859-1" Content-Transfer-Encoding: quoted-printable
Gostaria de saber se há uma = fórmula=20 fechada para a resposta desse problema em função de n=20 :
Dado um quadriculado n x n, quantos = são os=20 caminhos que saem do canto inferior esquerdo e chegam ao canto superior = direito,=20 de forma que o caminho não passe duas vezes pelo mesmo lugar. O = caminho=20 só pode ser feito sobre os lados dos quadradinhos 1x1 do=20 quadriculado.
 
Abraços,  
    =20 Villard
------=_NextPart_000_003A_01C20818.DA66C240-- ========================================================================= Instruções para entrar na lista, sair da lista e usar a lista em http://www.mat.puc-rio.br/~nicolau/olimp/obm-l.html O administrador desta lista é ========================================================================= From owner-obm-l@sucuri.mat.puc-rio.br Thu May 30 21:55:08 2002 Return-Path: Received: (from majordom@localhost) by sucuri.mat.puc-rio.br (8.9.3/8.9.3) id VAA27021 for obm-l-list; Thu, 30 May 2002 21:54:46 -0300 Received: from sec.secrel.com.br (sec.secrel.com.br [200.194.96.34]) by sucuri.mat.puc-rio.br (8.9.3/8.9.3) with SMTP id VAA27017 for ; Thu, 30 May 2002 21:54:44 -0300 Received: (qmail 7930 invoked from network); 31 May 2002 00:43:16 -0000 Received: from unknown (HELO rwv) (200.150.132.180) by sec.secrel.com.br with SMTP; 31 May 2002 00:43:16 -0000 Message-ID: <000001c2083c$1d51df40$b48496c8@rwv> From: "ReNNeR" To: "[OBM]" Subject: [obm-l] =?iso-8859-1?Q?algu=E9m_explica_isso=3F?= Date: Thu, 30 May 2002 11:42:55 -0300 MIME-Version: 1.0 Content-Type: multipart/alternative; boundary="----=_NextPart_000_0015_01C207CF.234ABB00" X-Priority: 3 X-MSMail-Priority: Normal X-Mailer: Microsoft Outlook Express 6.00.2600.0000 X-MimeOLE: Produced By Microsoft MimeOLE V6.00.2600.0000 Sender: owner-obm-l@sucuri.mat.puc-rio.br Precedence: bulk Reply-To: obm-l@mat.puc-rio.br This is a multi-part message in MIME format. ------=_NextPart_000_0015_01C207CF.234ABB00 Content-Type: text/plain; charset="iso-8859-1" Content-Transfer-Encoding: quoted-printable 1 + 1 =3D x (1 + 1)=B2 =3D x=B2 1 + 2 + 1 =3D x=B2 (1 + 1) + 2 =3D x=B2 x + 2 =3D x=B2 x=B2 - x - 2 =3D 0 delta =3D b=B2 - 4ac=20 delta =3D (-1)=B2-4.1.(-2)=20 delta =3D 1+8=20 delta =3D 9=20 x =3D [1 +?9]:2 ou x =3D [1-?9]:2=20 x =3D [1 + 3]:2 ou x =3D [1-3]:2=20 x =3D 2 ou x =3D -1.=20 1+1 =3D 2 ou; 1+1 =3D -1 ?? ------=_NextPart_000_0015_01C207CF.234ABB00 Content-Type: text/html; charset="iso-8859-1" Content-Transfer-Encoding: quoted-printable
1 + 1 =3D x
(1 + 1)=B2 =3D x=B2
1 + 2 + 1 =3D x=B2
(1 + 1) + 2 =3D x=B2
x + 2 =3D x=B2
x=B2 - x - 2 =3D 0
 
delta =3D b=B2 -=20 4ac
delta =3D (-1)=B2-4.1.(-2) =
delta =3D 1+8
delta =3D 9
x =3D [1 +√9]:2 ou x =3D [1-√9]:2 =
x = =3D [1 + 3]:2=20 ou x =3D [1-3]:2
x =3D 2=20 ou x =3D -1.
1+1 =3D=20 2 ou;
1+1 =3D=20 -1 ??
------=_NextPart_000_0015_01C207CF.234ABB00-- ========================================================================= Instruções para entrar na lista, sair da lista e usar a lista em http://www.mat.puc-rio.br/~nicolau/olimp/obm-l.html O administrador desta lista é ========================================================================= From owner-obm-l@sucuri.mat.puc-rio.br Thu May 30 22:28:32 2002 Return-Path: Received: (from majordom@localhost) by sucuri.mat.puc-rio.br (8.9.3/8.9.3) id WAA27493 for obm-l-list; Thu, 30 May 2002 22:25:56 -0300 Received: from hotmail.com (oe60.law8.hotmail.com [216.33.240.195]) by sucuri.mat.puc-rio.br (8.9.3/8.9.3) with ESMTP id WAA27489 for ; Thu, 30 May 2002 22:25:54 -0300 Received: from mail pickup service by hotmail.com with Microsoft SMTPSVC; Thu, 30 May 2002 18:14:26 -0700 X-Originating-IP: [200.217.201.208] From: "Adriana Valeska Daniel" To: References: <000001c2083c$1d51df40$b48496c8@rwv> Subject: [obm-l] =?iso-8859-1?Q?Exclus=E3o?= Date: Thu, 30 May 2002 22:19:26 -0300 MIME-Version: 1.0 Content-Type: multipart/alternative; boundary="----=_NextPart_000_0014_01C20828.0EDEED40" X-Priority: 3 X-MSMail-Priority: Normal X-Mailer: Microsoft Outlook Express 6.00.2600.0000 X-MimeOLE: Produced By Microsoft MimeOLE V6.00.2600.0000 Message-ID: X-OriginalArrivalTime: 31 May 2002 01:14:26.0020 (UTC) FILETIME=[814F9240:01C20840] Sender: owner-obm-l@sucuri.mat.puc-rio.br Precedence: bulk Reply-To: obm-l@mat.puc-rio.br This is a multi-part message in MIME format. ------=_NextPart_000_0014_01C20828.0EDEED40 Content-Type: text/plain; charset="iso-8859-1" Content-Transfer-Encoding: quoted-printable Gostaria de saber, por favor, como fa=E7o para retirar o meu = email da lista. = Agrade=E7o antecipadamente! ------=_NextPart_000_0014_01C20828.0EDEED40 Content-Type: text/html; charset="iso-8859-1" Content-Transfer-Encoding: quoted-printable
 
           = Gostaria de=20 saber, por favor, como fa=E7o para retirar o meu email da = lista.
 
          &nbs= p;            = ;            =             &= nbsp;           &n= bsp;           =20 Agrade=E7o antecipadamente!
------=_NextPart_000_0014_01C20828.0EDEED40-- ========================================================================= Instruções para entrar na lista, sair da lista e usar a lista em http://www.mat.puc-rio.br/~nicolau/olimp/obm-l.html O administrador desta lista é ========================================================================= From owner-obm-l@sucuri.mat.puc-rio.br Thu May 30 22:53:49 2002 Return-Path: Received: (from majordom@localhost) by sucuri.mat.puc-rio.br (8.9.3/8.9.3) id WAA28000 for obm-l-list; Thu, 30 May 2002 22:51:15 -0300 Received: from gorgo.centroin.com.br (gorgo.centroin.com.br [200.225.63.128]) by sucuri.mat.puc-rio.br (8.9.3/8.9.3) with ESMTP id WAA27996 for ; Thu, 30 May 2002 22:51:13 -0300 Received: from centroin.com.br (du107b.rjo.centroin.com.br [200.225.57.107]) (authenticated bits=0) by gorgo.centroin.com.br (8.12.2/8.12.1) with ESMTP id g4V1e3OA008080 for ; Thu, 30 May 2002 22:40:04 -0300 (BRT) Message-ID: <3CF6D4BA.705@centroin.com.br> Date: Thu, 30 May 2002 22:41:14 -0300 From: Augusto =?ISO-8859-1?Q?C=E9sar?= Morgado User-Agent: Mozilla/5.0 (Windows; U; Win98; en-US; rv:0.9.4.1) Gecko/20020508 Netscape6/6.2.3 X-Accept-Language: en-us MIME-Version: 1.0 To: obm-l@mat.puc-rio.br Subject: Re: [obm-l] =?ISO-8859-1?Q?algu=E9m?= explica isso? References: <000001c2083c$1d51df40$b48496c8@rwv> Content-Type: multipart/alternative; boundary="------------080908010807020505050605" Sender: owner-obm-l@sucuri.mat.puc-rio.br Precedence: bulk Reply-To: obm-l@mat.puc-rio.br --------------080908010807020505050605 Content-Type: text/plain; charset=ISO-8859-1; format=flowed Content-Transfer-Encoding: 8bit Nao ha o que explicar. Voce provou que se x=2 (ou x=1+1, como voce escreveu) entao x=2 ou x= -1. OU! Realmente, x=2. ReNNeR wrote: > 1 + 1 = x > > (1 + 1)² = x² > > 1 + 2 + 1 = x² > > (1 + 1) + 2 = x² > > x + 2 = x² > > x² - x - 2 = 0 > > > > delta = b² - 4ac > > delta = (-1)²-4.1.(-2) > > delta = 1+8 > > delta = 9 > > x = [1 + ?9]:2 ou x = [1-?9]:2 > > x = [1 + 3]:2 ou x = [1-3]:2 > > x = 2 ou x = -1. > > 1+1 = 2ou; > > 1+1 = -1 ?? > --------------080908010807020505050605 Content-Type: text/html; charset=us-ascii Content-Transfer-Encoding: 7bit Nao ha o que explicar. Voce provou que se x=2 (ou x=1+1, como voce escreveu) entao x=2 ou x= -1.
OU!
Realmente, x=2.


ReNNeR wrote:
1 + 1 = x
(1 + 1)² = x²
1 + 2 + 1 = x²
(1 + 1) + 2 = x²
x + 2 = x²
x² - x - 2 = 0
 
delta = b² - 4ac
delta = (-1)²-4.1.(-2)
delta = 1+8
delta = 9
x = [1 + √9]:2 ou x = [1-√9]:2
x = [1 + 3]:2 ou x = [1-3]:2
x = 2 ou x = -1.
1+1 = 2ou;
1+1 = -1 ??

--------------080908010807020505050605-- ========================================================================= Instruções para entrar na lista, sair da lista e usar a lista em http://www.mat.puc-rio.br/~nicolau/olimp/obm-l.html O administrador desta lista é ========================================================================= From owner-obm-l@sucuri.mat.puc-rio.br Thu May 30 23:08:20 2002 Return-Path: Received: (from majordom@localhost) by sucuri.mat.puc-rio.br (8.9.3/8.9.3) id XAA28376 for obm-l-list; Thu, 30 May 2002 23:08:10 -0300 Received: from sec.secrel.com.br (sec.secrel.com.br [200.194.96.34]) by sucuri.mat.puc-rio.br (8.9.3/8.9.3) with SMTP id XAA28370 for ; Thu, 30 May 2002 23:08:08 -0300 Received: (qmail 10297 invoked from network); 31 May 2002 01:56:40 -0000 Received: from unknown (HELO rwv) (200.150.132.180) by sec.secrel.com.br with SMTP; 31 May 2002 01:56:40 -0000 Message-ID: <005601c20846$66cf4db0$b48496c8@rwv> From: "ReNNeR" To: "[OBM]" Subject: [obm-l] =?iso-8859-1?Q?n=E3o_entend=ED...?= Date: Thu, 30 May 2002 22:56:37 -0300 MIME-Version: 1.0 Content-Type: multipart/alternative; boundary="----=_NextPart_000_0053_01C2082D.40EF72A0" X-Priority: 3 X-MSMail-Priority: Normal X-Mailer: Microsoft Outlook Express 6.00.2600.0000 X-MimeOLE: Produced By Microsoft MimeOLE V6.00.2600.0000 Sender: owner-obm-l@sucuri.mat.puc-rio.br Precedence: bulk Reply-To: obm-l@mat.puc-rio.br This is a multi-part message in MIME format. ------=_NextPart_000_0053_01C2082D.40EF72A0 Content-Type: text/plain; charset="iso-8859-1" Content-Transfer-Encoding: quoted-printable ent=E3o, como x=3D 1+1 e pode ser -1, 1 + 1 =3D -1 ??? ------=_NextPart_000_0053_01C2082D.40EF72A0 Content-Type: text/html; charset="iso-8859-1" Content-Transfer-Encoding: quoted-printable
ent=E3o, como x=3D 1+1 e pode ser -1, 1 = + 1 =3D -1=20 ???
------=_NextPart_000_0053_01C2082D.40EF72A0-- ========================================================================= Instruções para entrar na lista, sair da lista e usar a lista em http://www.mat.puc-rio.br/~nicolau/olimp/obm-l.html O administrador desta lista é ========================================================================= From owner-obm-l@sucuri.mat.puc-rio.br Thu May 30 23:16:24 2002 Return-Path: Received: (from majordom@localhost) by sucuri.mat.puc-rio.br (8.9.3/8.9.3) id XAA28594 for obm-l-list; Thu, 30 May 2002 23:16:16 -0300 Received: from web20405.mail.yahoo.com (web20405.mail.yahoo.com [216.136.226.124]) by sucuri.mat.puc-rio.br (8.9.3/8.9.3) with SMTP id XAA28590 for ; Thu, 30 May 2002 23:16:13 -0300 Message-ID: <20020531020445.78439.qmail@web20405.mail.yahoo.com> Received: from [200.182.207.235] by web20405.mail.yahoo.com via HTTP; Thu, 30 May 2002 23:04:45 ART Date: Thu, 30 May 2002 23:04:45 -0300 (ART) From: =?iso-8859-1?q?Helder=20Suzuki?= Subject: Re: [obm-l] alguém_explica_isso? To: obm-l@mat.puc-rio.br In-Reply-To: <000001c2083c$1d51df40$b48496c8@rwv> MIME-Version: 1.0 Content-Type: text/plain; charset=iso-8859-1 Content-Transfer-Encoding: 8bit Sender: owner-obm-l@sucuri.mat.puc-rio.br Precedence: bulk Reply-To: obm-l@mat.puc-rio.br --- ReNNeR escreveu: > 1 + 1 = x > (1 + 1)² = x² > 1 + 2 + 1 = x² > (1 + 1) + 2 = x² > x + 2 = x² > x² - x - 2 = 0 > > delta = b² - 4ac > delta = (-1)²-4.1.(-2) > delta = 1+8 > delta = 9 > x = [1 +?9]:2 ou x = [1-?9]:2 > x = [1 + 3]:2 ou x = [1-3]:2 > x = 2 ou x = -1. > 1+1 = 2 ou; > 1+1 = -1 ?? > ao elevar ao quadrado você está criando uma nova solução parcial. então você deverá testar todas as soluções parciais(raizes) com a expressão inicial e ver qual delas é a verdadeira. nesse caso: i) x = 2 1 + 1 = 2 (VERDADE) ii) x = -1 1 + 1 = -1 (FALSO) então, x só pode ser 2 ;) []'s, Hélder Suzuki _______________________________________________________________________ Yahoo! Encontros O lugar certo para você encontrar aquela pessoa que falta na sua vida. Cadastre-se hoje mesmo! http://br.encontros.yahoo.com/ ========================================================================= Instruções para entrar na lista, sair da lista e usar a lista em http://www.mat.puc-rio.br/~nicolau/olimp/obm-l.html O administrador desta lista é ========================================================================= From owner-obm-l@sucuri.mat.puc-rio.br Thu May 30 23:49:20 2002 Return-Path: Received: (from majordom@localhost) by sucuri.mat.puc-rio.br (8.9.3/8.9.3) id XAA29649 for obm-l-list; Thu, 30 May 2002 23:49:15 -0300 Received: from sec.secrel.com.br (sec.secrel.com.br [200.194.96.34]) by sucuri.mat.puc-rio.br (8.9.3/8.9.3) with SMTP id XAA29645 for ; Thu, 30 May 2002 23:49:12 -0300 Received: (qmail 32405 invoked from network); 31 May 2002 02:37:41 -0000 Received: from unknown (HELO rwv) (200.150.132.180) by sec.secrel.com.br with SMTP; 31 May 2002 02:37:41 -0000 Message-ID: <007301c2084c$210779f0$b48496c8@rwv> From: "ReNNeR" To: "[OBM]" Subject: [obm-l] valew! Date: Thu, 30 May 2002 23:37:38 -0300 MIME-Version: 1.0 Content-Type: multipart/alternative; boundary="----=_NextPart_000_0070_01C20832.FB8639A0" X-Priority: 3 X-MSMail-Priority: Normal X-Mailer: Microsoft Outlook Express 6.00.2600.0000 X-MimeOLE: Produced By Microsoft MimeOLE V6.00.2600.0000 Sender: owner-obm-l@sucuri.mat.puc-rio.br Precedence: bulk Reply-To: obm-l@mat.puc-rio.br This is a multi-part message in MIME format. ------=_NextPart_000_0070_01C20832.FB8639A0 Content-Type: text/plain; charset="iso-8859-1" Content-Transfer-Encoding: quoted-printable obrigado a=ED pessoal.. =3Dp ------=_NextPart_000_0070_01C20832.FB8639A0 Content-Type: text/html; charset="iso-8859-1" Content-Transfer-Encoding: quoted-printable
obrigado a=ED pessoal.. = =3Dp
------=_NextPart_000_0070_01C20832.FB8639A0-- ========================================================================= Instruções para entrar na lista, sair da lista e usar a lista em http://www.mat.puc-rio.br/~nicolau/olimp/obm-l.html O administrador desta lista é ========================================================================= From owner-obm-l@sucuri.mat.puc-rio.br Thu May 30 23:56:51 2002 Return-Path: Received: (from majordom@localhost) by sucuri.mat.puc-rio.br (8.9.3/8.9.3) id XAA29727 for obm-l-list; Thu, 30 May 2002 23:56:48 -0300 Received: from smtp-3.ig.com.br (smtp-3.ig.com.br [200.226.132.152]) by sucuri.mat.puc-rio.br (8.9.3/8.9.3) with SMTP id XAA29723 for ; Thu, 30 May 2002 23:56:47 -0300 From: ezer@ig.com.br Received: (qmail 28838 invoked from network); 31 May 2002 02:45:02 -0000 Received: from shasta084005.ig.com.br (HELO house) (200.151.84.5) by smtp-3.ig.com.br with SMTP; 31 May 2002 02:45:02 -0000 To: obm-l@mat.puc-rio.br Date: Wed, 1 Jan 1997 00:17:35 -0200 MIME-Version: 1.0 Subject: =?ISO-8859-1?Q?Re:_[obm-l]_Progress=E1lise=5FCombitm=E9tica_+_Duvidas_sobre_Logica_Matematica?= Message-ID: <32C9AD1F.29816.E893E@localhost> References: <32C9B188.7346.13EDC9@localhost> In-reply-to: <20020530183557.88051.qmail@web14802.mail.yahoo.com> X-mailer: Pegasus Mail for Windows (v4.01) Content-type: text/plain; charset=US-ASCII Content-transfer-encoding: 7BIT Content-description: Mail message body Sender: owner-obm-l@sucuri.mat.puc-rio.br Precedence: bulk Reply-To: obm-l@mat.puc-rio.br Entao no caso serao 18, levado em conta os numeros de algarismos com razao r = -1 e r = -2, ou seja numeros com algarismos em ordem inversa dos que apresentaste Se alguem pudesse me apresentar uma solucao por analise combinatoria, seria assaz enriquecedor =) Ezer ========================================================================= Instruções para entrar na lista, sair da lista e usar a lista em http://www.mat.puc-rio.br/~nicolau/olimp/obm-l.html O administrador desta lista é ========================================================================= From owner-obm-l@sucuri.mat.puc-rio.br Fri May 31 00:34:47 2002 Return-Path: Received: (from majordom@localhost) by sucuri.mat.puc-rio.br (8.9.3/8.9.3) id AAA30929 for obm-l-list; Fri, 31 May 2002 00:34:17 -0300 Received: from gorgo.centroin.com.br (gorgo.centroin.com.br [200.225.63.128]) by sucuri.mat.puc-rio.br (8.9.3/8.9.3) with ESMTP id AAA30925 for ; Fri, 31 May 2002 00:34:16 -0300 Received: from centroin.com.br (du111b.rjo.centroin.com.br [200.225.57.111]) (authenticated bits=0) by gorgo.centroin.com.br (8.12.2/8.12.1) with ESMTP id g4V3NFOA017773 for ; Fri, 31 May 2002 00:23:16 -0300 (BRT) Message-ID: <3CF6ECEA.2000905@centroin.com.br> Date: Fri, 31 May 2002 00:24:26 -0300 From: Augusto =?ISO-8859-1?Q?C=E9sar?= Morgado User-Agent: Mozilla/5.0 (Windows; U; Win98; en-US; rv:0.9.4.1) Gecko/20020508 Netscape6/6.2.3 X-Accept-Language: en-us MIME-Version: 1.0 To: obm-l@mat.puc-rio.br Subject: Re: [obm-l] =?ISO-8859-1?Q?n=E3o=20entend=ED=2E=2E=2E?= References: <005601c20846$66cf4db0$b48496c8@rwv> Content-Type: multipart/alternative; boundary="------------070906010803010706080200" Sender: owner-obm-l@sucuri.mat.puc-rio.br Precedence: bulk Reply-To: obm-l@mat.puc-rio.br --------------070906010803010706080200 Content-Type: text/plain; charset=ISO-8859-1; format=flowed Content-Transfer-Encoding: 8bit Não pode ser -1. Pode ser 2 OU -1. No caso, eh 2. ReNNeR wrote: > então, como x= 1+1 e pode ser -1, 1 + 1 = -1 ??? > --------------070906010803010706080200 Content-Type: text/html; charset=us-ascii Content-Transfer-Encoding: 7bit Não pode ser -1. Pode ser 2 OU -1. No caso, eh 2.

ReNNeR wrote:
então, como x= 1+1 e pode ser -1, 1 + 1 = -1 ???

--------------070906010803010706080200-- ========================================================================= Instruções para entrar na lista, sair da lista e usar a lista em http://www.mat.puc-rio.br/~nicolau/olimp/obm-l.html O administrador desta lista é ========================================================================= From owner-obm-l@sucuri.mat.puc-rio.br Fri May 31 11:43:32 2002 Return-Path: Received: (from majordom@localhost) by sucuri.mat.puc-rio.br (8.9.3/8.9.3) id LAA02690 for obm-l-list; Fri, 31 May 2002 11:41:28 -0300 Received: from candeias.terra.com.br (candeias.terra.com.br [200.176.3.18]) by sucuri.mat.puc-rio.br (8.9.3/8.9.3) with ESMTP id LAA02686 for ; Fri, 31 May 2002 11:41:26 -0300 Received: from bica.terra.com.br (bica.terra.com.br [200.176.3.32]) by candeias.terra.com.br (Postfix) with ESMTP id 7E8AE43E1E for ; Fri, 31 May 2002 11:30:01 -0300 (EST) Received: from Lucelindo (unknown [200.177.64.129]) (authenticated user ldias1) by bica.terra.com.br (Postfix) with ESMTP id 82516D544D for ; Fri, 31 May 2002 11:30:00 -0300 (EST) Message-ID: <006101c208af$02d04da0$8140b1c8@Lucelindo> From: "Lucelindo D. Ferreira" To: References: Subject: Re: [obm-l] desigualdades e cone sul Date: Fri, 31 May 2002 11:25:26 -0300 MIME-Version: 1.0 Content-Type: text/plain; charset="iso-8859-1" Content-Transfer-Encoding: 8bit X-Priority: 3 X-MSMail-Priority: Normal X-Mailer: Microsoft Outlook Express 5.00.2615.200 X-MimeOLE: Produced By Microsoft MimeOLE V5.00.2615.200 Sender: owner-obm-l@sucuri.mat.puc-rio.br Precedence: bulk Reply-To: obm-l@mat.puc-rio.br Olá Fê! Td legal! Eu fiz mas acho q ñ concebi muito bem a solução. Eu fiz + - a terceira: Seja (x^2 + xy) + (y^2 + xy) = S Agora considere o conjunto dos máximos dos pares q satisfazem a eq acima.O valor mínimo desse conjunto deverá satisfazer x^2 + xy = y^2 + xy .: x = y Da desigualdade dada: x^2 + xx + x^2 > 3.: x > 1. Se x > 1 x^2 + xy > 2 e y^2 + xy > 2.: Todos os outros pares tem pelo menos um elemento maior q 2(máx). É mais ou menos isso aí. Ficou claro pra vc? Um abraço! ----- Original Message ----- From: Fernanda Medeiros To: Sent: Tuesday, May 28, 2002 12:44 AM Subject: [obm-l] desigualdades e cone sul > > Olá pessoal,gostaria de um help nessas questões: > 1.Seja n um nº natural ,n>3. > Demonstrar que entre os multiplos de 9 menores q 10^n há mais nºs com a soma > de seus digitos igual a 9(n-2) que nºs com a soma de seus digitos igual a > 9(n-1) > > 2.Sejam a,b e c os comprimentos dos lados de um triangulo.Mostre que a > função f(x)=b^2x^2 +(b^2 +c^2 -a^2)x +c^2 é positiva ,pra todo real x. > (ps. essa eu fiz assim,pra f(x)ser >0 devemos ter delta<0 dae fica > [(b^2+c^2-a^2)^2 - (2bc)^2] fatorando agumas vezes chegamos a > [(b+c-a)(b+c+a)][(b-(c+a))(b-c+a)] daí por desigualdade triangular,vemos q > esse produto é <0 ... tá certo?) > 3.Sejam x,y reais positivos satisfazendo x^2+xy+y^2>3 .Prove q pelo menos um > dos nºs x^2 +xy e y^2 +xy é maior que 2. > > Obrigada!! > []´s > Fê > > > > _________________________________________________________________ > O MSN Photos é o modo mais fácil de compartilhar e imprimir suas fotos: > http://photos.msn.com/support/worldwide.aspx > > ========================================================================= > Instruções para entrar na lista, sair da lista e usar a lista em > http://www.mat.puc-rio.br/~nicolau/olimp/obm-l.html > O administrador desta lista é > ========================================================================= > ========================================================================= Instruções para entrar na lista, sair da lista e usar a lista em http://www.mat.puc-rio.br/~nicolau/olimp/obm-l.html O administrador desta lista é ========================================================================= From owner-obm-l@sucuri.mat.puc-rio.br Fri May 31 12:18:03 2002 Return-Path: Received: (from majordom@localhost) by sucuri.mat.puc-rio.br (8.9.3/8.9.3) id MAA03487 for obm-l-list; Fri, 31 May 2002 12:17:11 -0300 Received: from bidu.ime.usp.br (bidu.ime.usp.br [143.107.45.12]) by sucuri.mat.puc-rio.br (8.9.3/8.9.3) with SMTP id MAA03482 for ; Fri, 31 May 2002 12:17:09 -0300 Received: (qmail 8168 invoked from network); 31 May 2002 15:05:38 -0000 Received: from fradim.ime.usp.br (143.107.45.37) by bidu.ime.usp.br with SMTP; 31 May 2002 15:05:38 -0000 Received: (qmail 4152 invoked by uid 1604); 31 May 2002 15:03:06 -0000 Date: Fri, 31 May 2002 12:03:06 -0300 (EST) From: Salvador Addas Zanata X-Sender: sazanata@fradim To: obm-l@mat.puc-rio.br Subject: Re: [obm-l] desigualdades e cone sul In-Reply-To: <006101c208af$02d04da0$8140b1c8@Lucelindo> Message-ID: MIME-Version: 1.0 Content-Type: TEXT/PLAIN; charset=ISO-8859-1 Content-Transfer-Encoding: 8bit X-MIME-Autoconverted: from QUOTED-PRINTABLE to 8bit by sucuri.mat.puc-rio.br id MAA03483 Sender: owner-obm-l@sucuri.mat.puc-rio.br Precedence: bulk Reply-To: obm-l@mat.puc-rio.br Acho que um outro jeito e: x^2+(x^2+y^2)/2+y^2 >= x^2+xy+y^2>3, pela desigualdade das medias. Ai da: x^2+y^2>2. Agora e so observar que x>=y ou y>=x. No primeiro caso, x^2+xy>=x^2+y^2>2, o outro caso e igual. Abraco, Salvador On Fri, 31 May 2002, Lucelindo D. Ferreira wrote: > Olá Fê! Td legal! Eu fiz mas acho q ñ concebi muito bem a solução. > Eu fiz + - a terceira: > Seja (x^2 + xy) + (y^2 + xy) = S > Agora considere o conjunto dos máximos dos pares q satisfazem a eq acima.O > valor mínimo desse conjunto deverá satisfazer > x^2 + xy = y^2 + xy .: x = y > Da desigualdade dada: > x^2 + xx + x^2 > 3.: x > 1. > Se x > 1 > x^2 + xy > 2 e y^2 + xy > 2.: Todos os outros pares tem pelo menos um > elemento maior q 2(máx). > > É mais ou menos isso aí. Ficou claro pra vc? > Um abraço! > > ----- Original Message ----- > From: Fernanda Medeiros > To: > Sent: Tuesday, May 28, 2002 12:44 AM > Subject: [obm-l] desigualdades e cone sul > > > > > > Olá pessoal,gostaria de um help nessas questões: > > 1.Seja n um nº natural ,n>3. > > Demonstrar que entre os multiplos de 9 menores q 10^n há mais nºs com a > soma > > de seus digitos igual a 9(n-2) que nºs com a soma de seus digitos igual a > > 9(n-1) > > > > 2.Sejam a,b e c os comprimentos dos lados de um triangulo.Mostre que a > > função f(x)=b^2x^2 +(b^2 +c^2 -a^2)x +c^2 é positiva ,pra todo real x. > > (ps. essa eu fiz assim,pra f(x)ser >0 devemos ter delta<0 dae fica > > [(b^2+c^2-a^2)^2 - (2bc)^2] fatorando agumas vezes chegamos a > > [(b+c-a)(b+c+a)][(b-(c+a))(b-c+a)] daí por desigualdade triangular,vemos q > > esse produto é <0 ... tá certo?) > > 3.Sejam x,y reais positivos satisfazendo x^2+xy+y^2>3 .Prove q pelo menos > um > > dos nºs x^2 +xy e y^2 +xy é maior que 2. > > > > Obrigada!! > > []´s > > Fê > > > > > > > > _________________________________________________________________ > > O MSN Photos é o modo mais fácil de compartilhar e imprimir suas fotos: > > http://photos.msn.com/support/worldwide.aspx > > > > ========================================================================= > > Instruções para entrar na lista, sair da lista e usar a lista em > > http://www.mat.puc-rio.br/~nicolau/olimp/obm-l.html > > O administrador desta lista é > > ========================================================================= > > > > ========================================================================= > Instruções para entrar na lista, sair da lista e usar a lista em > http://www.mat.puc-rio.br/~nicolau/olimp/obm-l.html > O administrador desta lista é > ========================================================================= > ========================================================================= Instruções para entrar na lista, sair da lista e usar a lista em http://www.mat.puc-rio.br/~nicolau/olimp/obm-l.html O administrador desta lista é ========================================================================= From owner-obm-l@sucuri.mat.puc-rio.br Fri May 31 14:24:40 2002 Return-Path: Received: (from majordom@localhost) by sucuri.mat.puc-rio.br (8.9.3/8.9.3) id OAA05190 for obm-l-list; Fri, 31 May 2002 14:24:21 -0300 Received: from web21110.mail.yahoo.com (web21110.mail.yahoo.com [216.136.227.112]) by sucuri.mat.puc-rio.br (8.9.3/8.9.3) with SMTP id OAA05186 for ; Fri, 31 May 2002 14:24:18 -0300 Message-ID: <20020531171252.26377.qmail@web21110.mail.yahoo.com> Received: from [150.161.2.4] by web21110.mail.yahoo.com via HTTP; Fri, 31 May 2002 14:12:52 ART Date: Fri, 31 May 2002 14:12:52 -0300 (ART) From: =?iso-8859-1?q?Carlos=20Ma=E7aranduba?= Subject: Re: [obm-l] Progressálise_Combitmética + Duvidas sobre Logica Matematica To: obm-l@mat.puc-rio.br In-Reply-To: <20020530183557.88051.qmail@web14802.mail.yahoo.com> MIME-Version: 1.0 Content-Type: text/plain; charset=iso-8859-1 Content-Transfer-Encoding: 8bit Sender: owner-obm-l@sucuri.mat.puc-rio.br Precedence: bulk Reply-To: obm-l@mat.puc-rio.br > > 2) Simplificar a seguinte proposicao e indicar em > cima de cada simbolo de > equivalencia a propriedade logica utilizada. > > ~((~P -> ~Q) OU ((Q E P) <-> ~P)) ~((P OU ~Q) OU ( ( (Q E P)-> ~P ) E (~P->(Q E P) ) ) ~((P OU ~Q) OU ( ( ~(Q E P) OU ~P ) E ( P OU (Q E P) ) ) ) ~((P OU ~Q) OU ( ( ~Q OU ~P OU ~P ) E ( P OU ( Q E P))) ~((P OU ~Q) OU ( ~Q OU ~P E ( P OU ( Q E P) ) ) ) ~((P OU ~Q) OU ( ~Q OU FALSO OU ~P E Q E P ) ) ~((P OU ~Q) OU ( ~Q E Q E FALSO ) ) ~((P OU ~Q) OU FALSO ) ~( P OU ~Q ) ~P E Q > > ===== > []s > Ricardo Miranda > ricardomirandabr@yahoo.com.br > http://rm2.hpg.ig.com.br/ > > _______________________________________________________________________ > Yahoo! Encontros > O lugar certo para você encontrar aquela pessoa que > falta na sua vida. Cadastre-se hoje mesmo! > http://br.encontros.yahoo.com/ > ========================================================================= > Instruções para entrar na lista, sair da lista e > usar a lista em > http://www.mat.puc-rio.br/~nicolau/olimp/obm-l.html > O administrador desta lista é > > ========================================================================= _______________________________________________________________________ Yahoo! Encontros O lugar certo para você encontrar aquela pessoa que falta na sua vida. Cadastre-se hoje mesmo! http://br.encontros.yahoo.com/ ========================================================================= Instruções para entrar na lista, sair da lista e usar a lista em http://www.mat.puc-rio.br/~nicolau/olimp/obm-l.html O administrador desta lista é ========================================================================= From owner-obm-l@sucuri.mat.puc-rio.br Fri May 31 16:40:46 2002 Return-Path: Received: (from majordom@localhost) by sucuri.mat.puc-rio.br (8.9.3/8.9.3) id QAA06844 for obm-l-list; Fri, 31 May 2002 16:40:12 -0300 Received: from imo-r05.mx.aol.com (imo-r05.mx.aol.com [152.163.225.101]) by sucuri.mat.puc-rio.br (8.9.3/8.9.3) with ESMTP id QAA06840 for ; Fri, 31 May 2002 16:40:09 -0300 From: Korshinoi@aol.com Received: from Korshinoi@aol.com by imo-r05.mx.aol.com (mail_out_v32.5.) id z.11f.1165aa74 (4364) for ; Fri, 31 May 2002 15:28:34 -0400 (EDT) Message-ID: <11f.1165aa74.2a2928e1@aol.com> Date: Fri, 31 May 2002 15:28:33 EDT Subject: [obm-l] (nenhum assunto) To: obm-l@mat.puc-rio.br MIME-Version: 1.0 Content-Type: multipart/alternative; boundary="part1_11f.1165aa74.2a2928e1_boundary" X-Mailer: AOL 7.0 for Windows BR sub 10501 Sender: owner-obm-l@sucuri.mat.puc-rio.br Precedence: bulk Reply-To: obm-l@mat.puc-rio.br --part1_11f.1165aa74.2a2928e1_boundary Content-Type: text/plain; charset="ISO-8859-1" Content-Transfer-Encoding: quoted-printable E a=ED rapaziada!! Tenho duvidas em alguns problemas de contagem. No primeir= o,=20 meu resultado deu 35o fazendo uso de combina=E7=F5es...Gostaria de saber se=20= o=20 resultado =E9 esse mesmo e se pode ser feito s=F3 com o principio fundamenta= l da=20 contagem...ai v=E3o eles: 1)(ita) Uma urna cont=E9m 12 bolas, das quais 7 s=E3o pretas e 5 brancas. De= =20 quantos modos podemos tirar 6 bolas da urna, das quais 2 s=E3o brancas?? 2) De quantos modos se pode pintar um cubo, usando seis cores diferentes,=20 sendo cada face de uma cor? ps- Esse problema parece ser simples, mas tenho=20 duvidas se tenho que usar permuta=E7=F5es circulares nas faces laterais do=20 cubo...ser=E1 que estou viajando na maionese? 3)O n=FAmero 3 pode ser expresso como uma soma ordenada de um ou mais inteir= os=20 positivos de quatro modos, como : 3, 1+2, 2+1, 1+1+1. ..Mostre que um inteir= o=20 positivo n pode ser expresso de 2^(n-1) modos. --part1_11f.1165aa74.2a2928e1_boundary Content-Type: text/html; charset="ISO-8859-1" Content-Transfer-Encoding: quoted-printable E a=ED rapaziada!! Tenho duvidas em alguns problemas d= e contagem. No primeiro, meu resultado deu 35o fazendo uso de combina=E7=F5e= s...Gostaria de saber se o resultado =E9 esse mesmo e se pode ser feito s= =F3 com o principio fundamental da contagem...ai v=E3o eles:
1)(ita) Uma urna cont=E9m 12 bolas, das quais 7 s=E3o pretas e 5 brancas. De= quantos modos podemos tirar 6 bolas da urna, das quais 2 s=E3o brancas?? 2) De quantos modos se pode pintar um cubo, usando seis cores diferentes, se= ndo cada face de uma cor? ps- Esse problema parece ser simples, mas tenho du= vidas se tenho que usar permuta=E7=F5es circulares nas faces laterais do cub= o...ser=E1 que estou viajando na maionese?
3)O n=FAmero 3 pode ser expresso como uma soma ordenada de um ou mais inteir= os positivos de quatro modos, como : 3, 1+2, 2+1, 1+1+1. ..Mostre que um int= eiro positivo n pode ser expresso de 2^(n-1) modos.
--part1_11f.1165aa74.2a2928e1_boundary-- ========================================================================= Instruções para entrar na lista, sair da lista e usar a lista em http://www.mat.puc-rio.br/~nicolau/olimp/obm-l.html O administrador desta lista é ========================================================================= From owner-obm-l@sucuri.mat.puc-rio.br Fri May 31 16:51:35 2002 Return-Path: Received: (from majordom@localhost) by sucuri.mat.puc-rio.br (8.9.3/8.9.3) id QAA07025 for obm-l-list; Fri, 31 May 2002 16:51:24 -0300 Received: from imo-r07.mx.aol.com (imo-r07.mx.aol.com [152.163.225.103]) by sucuri.mat.puc-rio.br (8.9.3/8.9.3) with ESMTP id QAA07021 for ; Fri, 31 May 2002 16:51:21 -0300 From: Korshinoi@aol.com Received: from Korshinoi@aol.com by imo-r07.mx.aol.com (mail_out_v32.5.) id z.172.91a4591 (4364) for ; Fri, 31 May 2002 15:39:48 -0400 (EDT) Message-ID: <172.91a4591.2a292b84@aol.com> Date: Fri, 31 May 2002 15:39:48 EDT Subject: [obm-l] OBM-2001 To: obm-l@mat.puc-rio.br MIME-Version: 1.0 Content-Type: multipart/alternative; boundary="part1_172.91a4591.2a292b84_boundary" X-Mailer: AOL 7.0 for Windows BR sub 10501 Sender: owner-obm-l@sucuri.mat.puc-rio.br Precedence: bulk Reply-To: obm-l@mat.puc-rio.br --part1_172.91a4591.2a292b84_boundary Content-Type: text/plain; charset="ISO-8859-1" Content-Transfer-Encoding: quoted-printable Esse =E9 muito importante pra mim....se alguem conhecer o problema e me=20 passar a resolu=E7=E3o , eu ficarei muito agradecido. Resumidamente.... Uma bota tem n pares de furos pelos quais o cadar=E7o deve passar. Para n= =E3o se=20 aborrecer, o dono da bota gosta de diversificar as maneiras de passar o=20 cadar=E7o pelos furos, obedecendo sempre as seguintes regras. a) O cadar=E7o deve formar um padr=E3o sim=E9trico em rela=E7=E3o ao eixo ve= rtical; b) O cadar=E7o deve passar exatamente uma =FAnica vez por cada furo, sendo=20 indiferente se ele o faz por cima ou por baixo; c) O cadar=E7o deve come=E7ar e terminar nos dois furos superiores e deve li= gar=20 diretamente( isto =E9, sem passar por outros furos) os dois furos inferiores= . Determine em fun=E7=E3o de n>=3D2, o n=FAmero total de maneiras de passar o=20= cadar=E7o=20 pelos furos obedecendo as regras acima.... Desde j=E1 agrade=E7o quem puder me orientar nesse problema... Korshin=F3i --part1_172.91a4591.2a292b84_boundary Content-Type: text/html; charset="ISO-8859-1" Content-Transfer-Encoding: quoted-printable Esse =E9  muito importante pra mim....se alguem=20= conhecer o problema e me passar a resolu=E7=E3o , eu ficarei muito agradecid= o.
Resumidamente....
Uma bota tem n pares de furos pelos quais o cadar=E7o deve passar. Para n= =E3o se aborrecer, o dono da bota gosta de diversificar as maneiras de passa= r o cadar=E7o pelos furos, obedecendo sempre as seguintes regras.
a) O cadar=E7o deve formar um padr=E3o sim=E9trico em rela=E7=E3o ao eixo ve= rtical;
b) O cadar=E7o deve passar exatamente uma =FAnica vez por cada furo, sendo i= ndiferente se ele o faz por cima ou por baixo;
c) O cadar=E7o deve come=E7ar e terminar nos dois furos superiores e deve li= gar diretamente( isto =E9, sem passar por outros furos) os dois furos inferi= ores.
Determine em fun=E7=E3o de n>=3D2, o n=FAmero total de maneiras de passar= o cadar=E7o pelos furos obedecendo as regras acima....
Desde j=E1 agrade=E7o quem puder me orientar nesse problema...
            &nbs= p;            &n= bsp;         Korshin=F3i
--part1_172.91a4591.2a292b84_boundary-- ========================================================================= Instruções para entrar na lista, sair da lista e usar a lista em http://www.mat.puc-rio.br/~nicolau/olimp/obm-l.html O administrador desta lista é ========================================================================= From owner-obm-l@sucuri.mat.puc-rio.br Fri May 31 17:31:07 2002 Return-Path: Received: (from majordom@localhost) by sucuri.mat.puc-rio.br (8.9.3/8.9.3) id RAA07993 for obm-l-list; Fri, 31 May 2002 17:30:59 -0300 Received: from web13709.mail.yahoo.com (web13709.mail.yahoo.com [216.136.175.251]) by sucuri.mat.puc-rio.br (8.9.3/8.9.3) with SMTP id RAA07989 for ; Fri, 31 May 2002 17:30:56 -0300 Message-ID: <20020531201930.29680.qmail@web13709.mail.yahoo.com> Received: from [200.154.0.193] by web13709.mail.yahoo.com via HTTP; Fri, 31 May 2002 17:19:30 ART Date: Fri, 31 May 2002 17:19:30 -0300 (ART) From: =?iso-8859-1?q?Andre=20S?= Subject: Re: [obm-l] OBM-2001 To: obm-l@mat.puc-rio.br In-Reply-To: <172.91a4591.2a292b84@aol.com> MIME-Version: 1.0 Content-Type: text/plain; charset=iso-8859-1 Content-Transfer-Encoding: 8bit Sender: owner-obm-l@sucuri.mat.puc-rio.br Precedence: bulk Reply-To: obm-l@mat.puc-rio.br Acho que o problema pode ser resolvido dessa forma: Como começa e termina nos furos superiores, e a simetria é necessária, para escolher o primeiro furo há 2(n-2) possibilidades, já que devem ser excluídos o furo com que inicia, o com que termina e os dois inferiores, que devem ser ligados diretamente, ou seja, no meio do cadarço. Para escolher o segundo furo, há 2(n-3) possibilidades, e assim sucessivamente até sobrar a escolha entre os dois furos inferiores, com 2 possibilidades. Por PFC, há 2(n-2).2(n-3).2(n-4)....2(n-(n-2)).2(n-(n-1)).2 = P logo, P = 2^(n-1) . (n-2)! Desculpe-me pela falta de clareza, mas não sou muito bom com palavras, por isso gosto de matemática (risadas). Qualquer dúvida posterior, há a prova resolvida na página da obm, arquivo de provas. www.obm.org.br/frameset-provas.htm []'s, A.S. --- Korshinoi@aol.com escreveu: > Esse é muito importante pra mim....se alguem > conhecer o problema e me > passar a resolução , eu ficarei muito agradecido. > Resumidamente.... > Uma bota tem n pares de furos pelos quais o cadarço > deve passar. Para não se > aborrecer, o dono da bota gosta de diversificar as > maneiras de passar o > cadarço pelos furos, obedecendo sempre as seguintes > regras. > a) O cadarço deve formar um padrão simétrico em > relação ao eixo vertical; > b) O cadarço deve passar exatamente uma única vez > por cada furo, sendo > indiferente se ele o faz por cima ou por baixo; > c) O cadarço deve começar e terminar nos dois furos > superiores e deve ligar > diretamente( isto é, sem passar por outros furos) os > dois furos inferiores. > Determine em função de n>=2, o número total de > maneiras de passar o cadarço > pelos furos obedecendo as regras acima.... > Desde já agradeço quem puder me orientar nesse > problema... > Korshinói > _______________________________________________________________________ Yahoo! Encontros O lugar certo para você encontrar aquela pessoa que falta na sua vida. Cadastre-se hoje mesmo! http://br.encontros.yahoo.com/ ========================================================================= Instruções para entrar na lista, sair da lista e usar a lista em http://www.mat.puc-rio.br/~nicolau/olimp/obm-l.html O administrador desta lista é ========================================================================= From owner-obm-l@sucuri.mat.puc-rio.br Fri May 31 18:35:14 2002 Return-Path: Received: (from majordom@localhost) by sucuri.mat.puc-rio.br (8.9.3/8.9.3) id SAA09028 for obm-l-list; Fri, 31 May 2002 18:34:35 -0300 Received: from candeias.terra.com.br (candeias.terra.com.br [200.176.3.18]) by sucuri.mat.puc-rio.br (8.9.3/8.9.3) with ESMTP id SAA09024 for ; Fri, 31 May 2002 18:34:33 -0300 Received: from srv11-sao.terra.com.br (srv11-sao.terra.com.br [200.176.3.38]) by candeias.terra.com.br (Postfix) with ESMTP id E079143DE8 for ; Fri, 31 May 2002 18:23:08 -0300 (EST) Received: from terra.com.br (200-206-243-51.dsl.telesp.net.br [200.206.243.51]) (authenticated user lponce) by srv11-sao.terra.com.br (Postfix) with ESMTP id 9A188C94C7 for ; Fri, 31 May 2002 18:23:07 -0300 (EST) Message-ID: <3CF7E99F.F6F9ADFB@terra.com.br> Date: Fri, 31 May 2002 18:22:39 -0300 From: Luiz Antonio Ponce Alonso X-Mailer: Mozilla 4.79 [en] (Windows NT 5.0; U) X-Accept-Language: en MIME-Version: 1.0 To: obm-l@mat.puc-rio.br Subject: Re: [obm-l] Conicas e Latus Rectum (erro na =?iso-8859-1?Q?demonstra=E7=E3o?= do Eduardo Wagner) Content-Type: text/plain; charset=iso-8859-1 Content-Transfer-Encoding: 8bit Sender: owner-obm-l@sucuri.mat.puc-rio.br Precedence: bulk Reply-To: obm-l@mat.puc-rio.br Aos amigos da lista: Com relação a pergunta feita pelo amigo Marcio (mcohen) Como mostrar que a menor corda focal de uma elipse é sempre perpendicular ao eixo maior? Na demonstração feita pelo famoso Eduardo Wagner dada abaixo, acredito ter ocorrido dois pequenos erros decorrente da rapidez com que respondemos estes emails. Primeiro: Um pequeno erro ao isolar r, obtendo r = (b^2)/(a (1 - e.cost)). ao invés de r = (b^2)/(a (1 + e.cost)), donde conclue-se que r é mínimo se.e somente se, t = 0.graus e não no caso t = 90 graus como mencionado pelo Eduardo Wagner. Assim, da demonstração do Eduardo podemos concluir que r = PF sera minimo se, e somente se, t = 0 graus. Segundo: O minimo de r , r = PF, não implica na corda focal mínima. Assim, não tem sentido procurar o mínimo de PF, com o intuito de conseguir com isso determinar em que condições ocorre a corda focal mínima. Este erro ocorreu por que o Eduardo provavelmente estava preocupado apenas com o comprimento mínimo de PF e tinha esquecido que o objetivo do problema era outro. A demonstração deste teorema e de outros, mencionado pelo amigo Andre Araujo em outro email, sobre elipse você pode encontrar no livro do Caronet sobre Cônicas e e em menor quantidade no livro Geometria Analitica de Nikolai Efimov Editora mir, como mencionado pelo Paulo Santa Rita. Eu tenho também um arquivo com vários destes teoremas e suas respectivas demonstrações em meu computador que posso passar por email para qualquer um interessado. Verificarei com o Nicolau a possibilidade de deixar uma cópia em sua homepage, para um futuro interesse. Uma demonstração possivel que a menor corda focal é perpendicular ao eixo maior Vejamos abaixo uma possível demonstração da proposição do Marcio dada pela pergunta acima. Inicialmente aproveitemos as considerações dadas pelo Eduardo Wagner.. Ponha a elipse nos eixos na posicao canonica: centrada na origem e com o eixo maior sobre o eixo X, ( o que não perde em generalidade). Sejam: a o semi eixo maior, b o menor, F = (c, 0) o foco da direita e, e = c/a, a sua excentricidade. Seja P = (x, y) um ponto qualquer da elipse. Usando a definição de elipse, PF + PF' = 2a, mostre que PF = a - ex. Seja PF = r e seja ainda t o ângulo XFP, assim a abscissa do ponto P é dada por: x = c + rcost Tem-se então a partir da relacâo anterior: r = a - e(c + rcost), o que dá PF = (a - ec) / ( 1 + ecost ) ... (1) Sendo PP’ a corda focal relativa ao foco F, em que t é a medida do ângulo XFP. tem-se ainda de (1) que P´F = (a - ec) / ( 1 + ecos(180+ t ) = (a - ec) / ( 1 - ecost ) .. (2) Nestas condições, uma corda focal PP´ qualquer,tem o seu comprimento dado pela PF + PF´ . Portanto, em função de t , decorre de (1) e (2) que PP´= 2(a-ec)/ ( 1 - (ecost)^2) consequentemente PP´ sera mínimo quando t = 90 graus, ou seja, quando a corda focal PP´ da elipse for perpendicular ao seu eixo maior. com isto chegamos ao fim da demonstração desejada pelo Marcio. Desculpe-me por qualquer erro Um abraço de seu amigo PONCE Eduardo Wagner wrote: > > Como mostrar que a menor corda focal de uma elipse eh sempre perpendicular > ao eixo maior? > Vamos la. Ponha a elipse nos eixos na posicao canonica: centrada na origem e com o eixo maior sobre o eixo X. Sejam: a 0 semi eixo maior, b o menor F = (c, 0) o foco da direita e, e = c/a, a excentricidade. Seja P = (x, y) um ponto qualquer da elipse. Usando a definição de elipse, PF + PF' = 2a, mostre que PF = a - ex. Seja PF = r e seja ainda t o angulo XFP. Tem-se entao a partir da relacao anterior: r = a - e(c + rcost), o que da r = (b^2)/(a (1 - e.cost)). Logo, r = PF sera minimo quando t = 90 graus. Abraco, Wagner. > Esse problema me persegue ha bastante tempo ... qdo eu era aluno no 2o > grau, li em algum lugar essa propriedade... q dentre todas as cordas da > elipse passando por um determinado foco, aquela q fosse perpendicular ao > eixo maior tinha comprimento minimo.. parece q existem resultados analogos > para as outras conicas.. Eu nao lembro aonde li isso, mas nunca encontrei a > demonstracao. e agora eu precisei demonstrar isso e nao consegui. se alguem > puder ajudar, agradeco!! (tentei inicialmente colocar o comprimento da > corda em funcao do angulo x com a horizontal, e depois analisar a funcao > f(tan(x)).. a expressao era um pouco grande (4 fracoes racionais de grau <= > 2, duas delas ao quadrado) mas nao oobtive sucesso... > > Obrigado, > Marcio > > > > > ========================================================================= > Instruções para entrar na lista, sair da lista e usar a lista em > http://www.mat.puc-rio.br/~nicolau/olimp/obm-l.html > O administrador desta lista é > ========================================================================= ========================================================================= Instruções para entrar na lista, sair da lista e usar a lista em http://www.mat.puc-rio.br/~nicolau/olimp/obm-l.html O administrador desta lista é ===================================== ========================================================================= Instruções para entrar na lista, sair da lista e usar a lista em http://www.mat.puc-rio.br/~nicolau/olimp/obm-l.html O administrador desta lista é ========================================================================= From owner-obm-l@sucuri.mat.puc-rio.br Fri May 31 18:43:28 2002 Return-Path: Received: (from majordom@localhost) by sucuri.mat.puc-rio.br (8.9.3/8.9.3) id SAA09115 for obm-l-list; Fri, 31 May 2002 18:43:15 -0300 Received: from smtp-6.ig.com.br (smtp-6.ig.com.br [200.226.132.155]) by sucuri.mat.puc-rio.br (8.9.3/8.9.3) with SMTP id SAA09111 for ; Fri, 31 May 2002 18:43:13 -0300 From: ezer@ig.com.br Received: (qmail 1620 invoked from network); 31 May 2002 21:31:26 -0000 Received: from shasta039163.ig.com.br (HELO house) (200.151.39.163) by smtp-6.ig.com.br with SMTP; 31 May 2002 21:31:26 -0000 To: obm-l@mat.puc-rio.br Date: Wed, 1 Jan 1997 04:02:25 -0200 MIME-Version: 1.0 Subject: =?ISO-8859-1?Q?Re:_Re:_[obm-l]_Progress=E1lise=5FCombitm=E9tica?= Message-ID: <32C9E1D1.24790.76C79B@localhost> References: <32C9B188.7346.13EDC9@localhost> In-reply-to: <20020530183557.88051.qmail@web14802.mail.yahoo.com> X-mailer: Pegasus Mail for Windows (v4.01) Content-type: text/plain; charset=ISO-8859-1 Content-description: Mail message body Content-Transfer-Encoding: 8bit X-MIME-Autoconverted: from Quoted-printable to 8bit by sucuri.mat.puc-rio.br id SAA09112 Sender: owner-obm-l@sucuri.mat.puc-rio.br Precedence: bulk Reply-To: obm-l@mat.puc-rio.br Eu estava pensando no problema, e acabei resolvendo-o. Eis a resolucao: 0,1,2,3,4,5,6,7,8,9 Podemos perceber que, se seus termos estao em P.A., quando selecionamos dois algarismos, nao ha mais decisoes a tomar (pois os outros termos jah estarao determinados, pois a razao jah estarah determinada). Entao vamos fazer caso a caso, em funcao do primeiro algarismo da esquerda: Se escolhemos 4 ou 5: __ __ __ __ Razoes possiveis p/ o 4 e 5: r = 1, r = 0 ou r = -1 2 * 3 * 1* 1 = 6 Se escolhemos 3 ou 6: __ __ __ __ Razoes possiveis p/ o 3: r =2, p/ o 6: r = -2 e p/ os dois: r = (0,1, -1) 2 * 4 * 1* 1 = 8 Se escolhemos 1,2,7 ou 8: __ __ __ __ Razoes possiveis: r = (0,|1|,|2|) 4 * 3 * 1* 1 = 12 Se escolhemos 9: __ __ __ __ Razoes Possiveis: r = (0,-1,-2,-3) 1 * 4 *1 * 1 = 4 Nao podemos escolher o zero, pq o numero tem 4 algarismos: Entao, temos: 6+8+12+4 = 30 numeros Para ter certeza, criei um algoritmo que deu como resposta 30 numeros. Para quem quiser conferir, os colei no final do e-mail De qualquer forma, obrigado pela ajuda =) Ezer > --- ezer@ig.com.br escreveu: > > Olá pessoal, > > > > Eu estava pensando num problema que eu me formulei > > de Analise Combinatoria, e de Progressao Aritmetica: > > > > Quantos sao os numeros de quatro algarismos que > > estao em Progressao Aritmetica com o seu vizinho > > (p. ex., 2468 serve, 4286, nao) > > > > Desde jah agradeco, =) > > > > Ezer > > As razoes podem ser 1 e 2. > Razao 3 nao dá, "1 4 7 0" > > r=1 > 1234 > 2345 > 3456 > 4567 > 5678 > 6789 > > r=2 > 1357 > 2468 > 3579 > > Total, 9. > > Nao sei se está certo, e nao consegui pensar em uma solucao algébrica. 1 1 1 1 1 2 3 4 1 3 5 7 2 2 2 2 2 3 4 5 2 4 6 8 3 2 1 0 3 3 3 3 3 4 5 6 3 5 7 9 4 3 2 1 4 4 4 4 4 5 6 7 5 4 3 2 5 5 5 5 5 6 7 8 6 4 2 0 6 5 4 3 6 6 6 6 6 7 8 9 7 5 3 1 7 6 5 4 7 7 7 7 8 6 4 2 8 7 6 5 8 8 8 8 9 6 3 0 9 7 5 3 9 8 7 6 9 9 9 9 ========================================================================= Instruções para entrar na lista, sair da lista e usar a lista em http://www.mat.puc-rio.br/~nicolau/olimp/obm-l.html O administrador desta lista é ========================================================================= From owner-obm-l@sucuri.mat.puc-rio.br Fri May 31 20:10:56 2002 Return-Path: Received: (from majordom@localhost) by sucuri.mat.puc-rio.br (8.9.3/8.9.3) id UAA10678 for obm-l-list; Fri, 31 May 2002 20:10:46 -0300 Received: from home.iis.com.br (mail.iis.com.br [200.202.96.2]) by sucuri.mat.puc-rio.br (8.9.3/8.9.3) with ESMTP id UAA10674 for ; Fri, 31 May 2002 20:10:44 -0300 Received: from Marcio (rio-tc0-tty464.iis.com.br [200.202.99.210]) by home.iis.com.br (8.11.6/8.11.6/1.1.1.16) with SMTP id g4VMxJ228367 for ; Fri, 31 May 2002 19:59:19 -0300 Message-ID: <001f01c208f6$ee301220$d263cac8@epq.ime.eb.br> From: "Marcio" To: References: <3CF7E99F.F6F9ADFB@terra.com.br> Subject: [obm-l] =?iso-8859-1?Q?Re:_=5Bobm-l=5D_Conicas_e_Latus_Rectum_=28erro_na_demons?= =?iso-8859-1?Q?tra=E7=E3o_do_Eduardo_Wagner=29?= Date: Fri, 31 May 2002 20:00:15 -0300 MIME-Version: 1.0 Content-Type: text/plain; charset="iso-8859-1" Content-Transfer-Encoding: 8bit X-Priority: 3 X-MSMail-Priority: Normal X-Mailer: Microsoft Outlook Express 5.50.4133.2400 X-MimeOLE: Produced By Microsoft MimeOLE V5.50.4133.2400 X-AntiVirus: Antivirus for sendmail by Petr Rehor Sender: owner-obm-l@sucuri.mat.puc-rio.br Precedence: bulk Reply-To: obm-l@mat.puc-rio.br Oi Ponce!! Obrigado a vc e ao resto do pessoal que me ajudou!! Na verdade o principal da solucao do Eduardo Wagner (e tmb do Paulo Santa Rita) foi a ideia de usar coordenadas polares, e eu ja tinha conseguido completar a ideia exatamente do jeito que vc (Ponce) acabou de expor aqui! Na solucao do Andre, eu a principio nao tinha conseguido provar os "exercicios para voce", mas eles ficaram simples usando coordenadas polares (ou melhor, usando a propriedade PF/d(p,diretriz) = e).. Valeu pela ajuda de todo mundo!! Silenciou uma duvida minha q ja durava bastante :)) Abracos, Marcio ----- Original Message ----- From: "Luiz Antonio Ponce Alonso" To: Sent: Friday, May 31, 2002 6:22 PM Subject: Re: [obm-l] Conicas e Latus Rectum (erro na demonstração do Eduardo Wagner) > > Aos amigos da lista: > Com relação a pergunta feita pelo amigo Marcio (mcohen) > Como mostrar que a menor corda focal de uma elipse é sempre > perpendicular ao eixo maior? > > Na demonstração feita pelo famoso Eduardo Wagner dada abaixo, acredito > ter ocorrido dois pequenos > erros decorrente da rapidez com que respondemos estes emails. > > Primeiro: Um pequeno erro ao isolar r, obtendo r = (b^2)/(a (1 - > e.cost)). > ao invés de r = (b^2)/(a (1 + e.cost)), donde conclue-se que r é mínimo > se.e somente se, t = 0.graus > e não no caso t = 90 graus como mencionado pelo Eduardo Wagner. > Assim, da demonstração do Eduardo podemos concluir que r = PF sera > minimo se, e somente se, > t = 0 graus. > Segundo: O minimo de r , r = PF, não implica na corda focal mínima. > Assim, não tem sentido procurar > o mínimo de PF, com o intuito de conseguir com isso determinar em que > condições ocorre a corda > focal mínima. > Este erro ocorreu por que o Eduardo provavelmente estava preocupado > apenas com o comprimento > mínimo de PF e tinha esquecido que o objetivo do problema era outro. > > A demonstração deste teorema e de outros, mencionado pelo amigo Andre > Araujo em outro email, > sobre elipse você pode encontrar no livro do Caronet sobre Cônicas e e > em menor quantidade no > livro Geometria Analitica de Nikolai Efimov Editora mir, como mencionado > pelo Paulo Santa Rita. > Eu tenho também um arquivo com vários destes teoremas e suas respectivas > demonstrações em > meu computador que posso passar por email para qualquer um interessado. > Verificarei com o Nicolau a possibilidade de deixar uma cópia em sua > homepage, para um futuro > interesse. > ========================================================================= Instruções para entrar na lista, sair da lista e usar a lista em http://www.mat.puc-rio.br/~nicolau/olimp/obm-l.html O administrador desta lista é ========================================================================= From owner-obm-l@sucuri.mat.puc-rio.br Fri May 31 21:36:00 2002 Return-Path: Received: (from majordom@localhost) by sucuri.mat.puc-rio.br (8.9.3/8.9.3) id VAA11482 for obm-l-list; Fri, 31 May 2002 21:35:28 -0300 Received: from gorgo.centroin.com.br (gorgo.centroin.com.br [200.225.63.128]) by sucuri.mat.puc-rio.br (8.9.3/8.9.3) with ESMTP id VAA11478 for ; Fri, 31 May 2002 21:35:26 -0300 Received: from centroin.com.br (du92b.rjo.centroin.com.br [200.225.57.92]) (authenticated bits=0) by gorgo.centroin.com.br (8.12.2/8.12.1) with ESMTP id g510OSOA008936 for ; Fri, 31 May 2002 21:24:28 -0300 (BRT) Message-ID: <3CF81481.1010905@centroin.com.br> Date: Fri, 31 May 2002 21:25:37 -0300 From: Augusto =?ISO-8859-1?Q?C=E9sar?= Morgado User-Agent: Mozilla/5.0 (Windows; U; Win98; en-US; rv:0.9.4.1) Gecko/20020508 Netscape6/6.2.3 X-Accept-Language: en-us MIME-Version: 1.0 To: obm-l@mat.puc-rio.br Subject: Re: [obm-l] (nenhum assunto) References: <11f.1165aa74.2a2928e1@aol.com> Content-Type: multipart/alternative; boundary="------------030609020705000500080203" Sender: owner-obm-l@sucuri.mat.puc-rio.br Precedence: bulk Reply-To: obm-l@mat.puc-rio.br --------------030609020705000500080203 Content-Type: text/plain; charset=ISO-8859-1; format=flowed Content-Transfer-Encoding: 8bit 3) Bonito problema. O numero de soluçoes inteiras e positivas de x 1+x2+...+x p = n eh C(n-1, n-p). O numero total de decomposiçoes eh a soma dos numeros de decomposiçoes em 1, 2,..., n parcelas, isto eh, C(n-1, n-1) + C(n-1, n-2)+...+C(n-1, 0) = 2^(n-1). 2) Ha 6 modos de pintar a face de cima, 5 de pintar a face de baixo,... A resposta eh, aparentemente, 6x5x4x3x2x1=720. Pensando melhor, vemos que contamos cada pintura varias vezes ( branco em cima e preto em baixo eh o mesmo cubo pintado que preto em cima e branco em baixo; eh este de cabeça para baixo). Devemos corrigir a "resposta" dividindo-a pelo numero de vezes que contamos cada cubo pintado. Ora, cada cubo pintado foi contado uma vez em cada posiçao que ele pode ser colocado. Esse numero de posiçoes eh 6(numero de modos de escolher a face que ficarah em baixo)x4(numero de modos de escolher nesta face a aresta que fica de frente)=24 e a resposta eh 720/24=30. 1) Interpretando como exatamente duas sao brancas, que a retirada eh simultanea, a resposta eh C(5,2)xC(7,4) = 10x35=350 Korshinoi@aol.com wrote: > E aí rapaziada!! Tenho duvidas em alguns problemas de contagem. No > primeiro, meu resultado deu 35o fazendo uso de combinações...Gostaria > de saber se o resultado é esse mesmo e se pode ser feito só com o > principio fundamental da contagem...ai vão eles: > 1)(ita) Uma urna contém 12 bolas, das quais 7 são pretas e 5 brancas. > De quantos modos podemos tirar 6 bolas da urna, das quais 2 são brancas?? > 2) De quantos modos se pode pintar um cubo, usando seis cores > diferentes, sendo cada face de uma cor? ps- Esse problema parece ser > simples, mas tenho duvidas se tenho que usar permutações circulares > nas faces laterais do cubo...será que estou viajando na maionese? > 3)O número 3 pode ser expresso como uma soma ordenada de um ou mais > inteiros positivos de quatro modos, como : 3, 1+2, 2+1, 1+1+1. > ..Mostre que um inteiro positivo n pode ser expresso de 2^(n-1) modos. --------------030609020705000500080203 Content-Type: text/html; charset=us-ascii Content-Transfer-Encoding: 7bit 3) Bonito problema.
 O numero de soluçoes inteiras e positivas de x 1+x2+...+x p = n eh C(n-1, n-p).
O numero total de decomposiçoes eh a soma dos numeros de decomposiçoes em 1, 2,..., n parcelas, isto eh, C(n-1, n-1) + C(n-1, n-2)+...+C(n-1, 0) = 2^(n-1).

2) Ha 6 modos de pintar a face de cima, 5 de pintar a face de baixo,...
A resposta eh, aparentemente, 6x5x4x3x2x1=720.
Pensando melhor, vemos que contamos cada pintura varias vezes ( branco em cima e preto em baixo eh o mesmo cubo pintado que preto em cima e branco em baixo; eh este de cabeça para baixo). Devemos corrigir a "resposta" dividindo-a pelo numero de vezes que contamos cada cubo pintado. Ora, cada cubo pintado foi contado uma vez em cada posiçao que ele pode ser colocado. Esse numero de posiçoes eh 6(numero de modos de escolher a face que ficarah em baixo)x4(numero de modos de escolher nesta face a aresta que fica de frente)=24 e a resposta eh 720/24=30.
1) Interpretando como exatamente duas sao brancas, que a retirada eh simultanea, a resposta eh
C(5,2)xC(7,4) = 10x35=350

Korshinoi@aol.com wrote:
E aí rapaziada!! Tenho duvidas em alguns problemas de contagem. No primeiro, meu resultado deu 35o fazendo uso de combinações...Gostaria de saber se o resultado é esse mesmo e se pode ser feito só com o principio fundamental da contagem...ai vão eles:
1)(ita) Uma urna contém 12 bolas, das quais 7 são pretas e 5 brancas. De quantos modos podemos tirar 6 bolas da urna, das quais 2 são brancas??
2) De quantos modos se pode pintar um cubo, usando seis cores diferentes, sendo cada face de uma cor? ps- Esse problema parece ser simples, mas tenho duvidas se tenho que usar permutações circulares nas faces laterais do cubo...será que estou viajando na maionese?
3)O número 3 pode ser expresso como uma soma ordenada de um ou mais inteiros positivos de quatro modos, como : 3, 1+2, 2+1, 1+1+1. ..Mostre que um inteiro positivo n pode ser expresso de 2^(n-1) modos.

--------------030609020705000500080203-- ========================================================================= Instruções para entrar na lista, sair da lista e usar a lista em http://www.mat.puc-rio.br/~nicolau/olimp/obm-l.html O administrador desta lista é ========================================================================= From owner-obm-l@sucuri.mat.puc-rio.br Fri May 31 23:11:24 2002 Return-Path: Received: (from majordom@localhost) by sucuri.mat.puc-rio.br (8.9.3/8.9.3) id XAA12389 for obm-l-list; Fri, 31 May 2002 23:11:08 -0300 Received: from smtp.ieg.com.br (193.125.226.200.in-addr.arpa.ig.com.br [200.226.125.193] (may be forged)) by sucuri.mat.puc-rio.br (8.9.3/8.9.3) with ESMTP id XAA12385 for ; Fri, 31 May 2002 23:11:06 -0300 Received: from igor ([200.222.145.181]) by smtp.ieg.com.br (IeG relay/8.9.3) with SMTP id g5121XZl002779 for ; Fri, 31 May 2002 23:01:39 -0300 (BRT) Message-ID: <000d01c2090f$ee0a3140$b591dec8@igor> From: "Igor Castro" To: Subject: [obm-l] Limites?!?! Date: Fri, 31 May 2002 22:59:12 -0300 MIME-Version: 1.0 Content-Type: multipart/alternative; boundary="----=_NextPart_000_000A_01C208F6.C7F38EC0" X-Priority: 3 X-MSMail-Priority: Normal X-Mailer: Microsoft Outlook Express 5.00.2615.200 X-MimeOLE: Produced By Microsoft MimeOLE V5.00.2615.200 Sender: owner-obm-l@sucuri.mat.puc-rio.br Precedence: bulk Reply-To: obm-l@mat.puc-rio.br This is a multi-part message in MIME format. ------=_NextPart_000_000A_01C208F6.C7F38EC0 Content-Type: text/plain; charset="iso-8859-1" Content-Transfer-Encoding: quoted-printable Ol=E1 colegas da lista,=20 estou iniciando ainda neste assunto mas algu=E9m poderia dar uma ajuda = neste limite?=20 LIM [sqrt(x+2) + sqrt(x)] / x x-> -1 n=E3o consigo fugir da indetermina=E7=E3o ou de uma resposta com "i"(=E9 = valido para respostas de limite?) ou talvez o limite nem exista... deixo a analise para vcs.. : ) agrade=E7o desde j=E1... []'s ------=_NextPart_000_000A_01C208F6.C7F38EC0 Content-Type: text/html; charset="iso-8859-1" Content-Transfer-Encoding: quoted-printable
Ol=E1 colegas da lista,
estou iniciando ainda neste assunto mas = algu=E9m=20 poderia dar uma ajuda neste limite?
 
LIM        =20 [sqrt(x+2) + sqrt(x)] / x
 x-> -1
 
n=E3o consigo fugir da = indetermina=E7=E3o ou de uma=20 resposta com "i"(=E9 valido para respostas de limite?)
ou talvez o limite nem exista... deixo = a analise=20 para vcs.. : )
agrade=E7o desde j=E1...
[]'s
------=_NextPart_000_000A_01C208F6.C7F38EC0-- ========================================================================= Instruções para entrar na lista, sair da lista e usar a lista em http://www.mat.puc-rio.br/~nicolau/olimp/obm-l.html O administrador desta lista é =========================================================================